You are on page 1of 604

• Lecture Notes For Ty-2024

FEATURES
• 80+Numerical with Updated Solution
• Chapter Wise Sorted Past Papers with
Examiner comments and marking
Scheme
• Numerical Sheets and Summary
• Additional Questions for
Advance Practice

By
FRK

SPILLING
Scan this to contact me

THE Tax TEA


Content
S. No Chapter Name Page No Page | 1
Section – A [Income Tax]
1 Ch# 1 Basics of Income Tax & Scope of Income 7
2 Ch# 2 Income from Salary 29
3 Ch# 3 Income from Property 84
4 Ch# 4 Income from Capital Gains 104
5 Ch# 5 Income from Other Source 133
6 Ch# 6 Final Tax Regime 148
7 Ch# 7 Deductible allowances & rebates 158
8 Ch# 8 Income Tax Numerical 173
9 Ch# 9 Income from Business Basic 240
10 Ch # 10 Income from business In-admissible Expenses 248
11 Ch # 11 Income from business Depreciation 255
12 Ch # 12 Income from Business Other Areas 279
13 Ch # 13 Association of Persons 304
14 Ch # 14 Losses & Rebates 327
15 Ch # 15 Income Tax Numerical (Business) 342
16 Ch # 16 Income Tax Area not covered Earlier 389
17 Ch # 17 Returns and Wealth Statement 403
18 Ch # 18 Assessments 411
19 Ch # 19 Appeals 421
20 Ch # 20 Practice Questions Returns, Assessment & Appeals 427
Section – B [Sales Tax]
21 Ch # 21 Sales Tax Deifinitions 453
22 Ch # 22 Sales Tax Provisions 464
23 Ch # 23 Sales Tax Registration and De-registration 480
24 Ch # 24 Sales Tax Records 486
25 Ch # 25 Sales Tax Theoretical Questions 492
26 Ch # 26 Sales Tax Numerical 512
Section – C [Basics, Ethics and Constitution]
27 Ch # 27 System of Taxation in Pakistan 563
28 Ch # 28 Constitutional Provisions 574
29 Ch # 29 Ethics 585
30 Ch # 30 Code of Ethics 598

Scan this code to contact me on WhatsApp

1|Page
Scan the following codes to access my online courses

CAF-01 Financial Accounting and Reporting -1 at Nearpeer


Page | 2

CAF-02 Tax Practice at Nearpeer

CAF-02 Fast track at Nearpeer

CAF-01 Financial Accounting and Reporting -1 SBI

CAF-02 Tax Practice at SBI

CAF-02 Fast Track at SBI

YouTube Channel for Free Videos and Revisions

Classes at CA MADE EASY

2|Page
PREFACE

This book will be useful to the all students who are fresh as well as to the
repeaters.
Page | 3
This book is mainly for the ICAP students, and can be served as class notes, yet
students of other professional bodies may also get benefit from its quality
material. Although the study text of ICAP is available and it covers all aspects of
the syllabus in detailed form, but this book covers the whole syllabus in smart
way with developing link between topics, and providing the tools for memorizing
the provisions easily.

This book consists of smart notes, flow charts, summary sheets, practice
material and even advance practical material, just to make it sure you are well
prepared for the exam.
Well this book is not the alternative to study text, but this can be used along
with Study text.

Although reasonable efforts have been put to make the book free from majority of
the errors, but chances of human error always exist. In case of any error or any
issue in the book please reach me out, via email amjadfarrukh@hotmail.com or
WhatsApp only at 0334-0405073

Hope this book would INSHA ALLAH be serving you with its easy to do approach

May ALLAH bless all of you with success in every exam of both lives. Please also
remember us in your prayers

Important

For Chapter Wise Notes, or for any Updates in taxation Please scan the QR Code
and keep it touch with the updated book always, in case of any rectification, the
soft form will be immediately available so scan this QR Code and get yourself
Updated Always

Farrukh Amjad

3|Page
SECTION- A INCOME TAX
Syllabus Weightage

Syllabus Teaching
Grid Weightage
Ref. Hours Page | 4
B Income Tax Laws 70-80 60-75

Syllabus Proficiency Testing


Learning Outcomes
Ref. levels levels
Income Tax Laws
B

A Central Concepts

Describe the central concepts and scope of income P2 T2

b Chargeability and Computation of Income and Tax

1 Compute income, taxable income and tax thereon under various P2 T2


heads of income for non-corporates i.e. salary, income from property,
income from business, capital gains and income from other sources

2 Apply the provisions relating to carry forward, deductible P2 T2


allowances, set-off of losses, tax credit and tax
exemptions/concessions

Procedural Aspects: Returns, Assessments, Appeals and Records


c
Explain tax compliance requirements and related submissions
1 P2 T2
Identify persons required to furnish a return of income and wealth
2 statement. P2 T2

3 Explain the provisions of law relating to the method of filing,


revision, due dates for filing and extension in the date for filing of
return/statement.

Prepare return of income and wealth statement along with its


4 reconciliation. P2 T2

Communicate with Inland Revenue Authorities (filing applications,


5 representations and extensions). P2 T2

Explain the provisions of law relating to various types of assessment.


6 P2 T2
Discuss the provisions of law relating to maintenance of records and
7 audit of income tax affairs of a person P2 T2

Describe procedure involved in appeals before various appellate


8 authorities i.e. Commissioner (Appeals), Appellate Tribunal, High P2 T2
Court and Supreme Court.

State provisions of law relating to alternative dispute resolution.


9 P2 T2

4|Page
Tax Rates Used for Tax Year 2024
Page | 5

Tax rates for salaried individual (Salary Exceeds 75% of Taxable Income)

Sr.no Taxable Income Rate of Tax


1 Where the taxable income does not exceed Rs. 600,000 0%
2 Where taxable income exceeds Rs. 600,000 but does not 2.5% of the amount exceeding Rs.
exceed Rs. 1,200,000 600,000
3 Where taxable income exceeds Rs. 1,200,000 but does not Rs. 15,000 plus 12.5% of the amount
exceed Rs. 2,400,000 exceeding Rs. 1,200,000
4 Where taxable income exceeds Rs. 2,400,000 but does not Rs. 165,000 plus 22.5% of the amount
exceed Rs. 3,600,000 exceeding Rs. 2,400,000
5 Where taxable income exceeds Rs. 3,600,000 but does not Rs. 435,000 plus 27.5% of the amount
exceed Rs. 6,000,000 exceeding Rs. 3,600,000
6 Where taxable income exceeds Rs. 6,000,000 Rs. 1,095,000 plus 35% of the amount
exceeding Rs. 6,000,000

Tax rates for non-salaried individuals and Association of Persons

Sr.no` Taxable Income Rate


1 Where the taxable income does not exceed Rs.600,000 0%
2 Where the taxable income exceeds Rs.600,000 but does 7.5% of the amount exceeding
not exceed Rs. 800,000 Rs.600,000
3 Where taxable income exceeds Rs. 800,000 but does not Rs. 15,000 plus 15% of the amount
exceed Rs. 1,200,000 exceeding Rs. 800,000
4 Where taxable income exceeds Rs. 1,200,000 but does not Rs. 75,000 plus 20% of the amount
exceed Rs. 2,400,000 exceeding Rs, 1,200,000
5 Where taxable income exceeds Rs. 2,400,000 but does not Rs. 315,000 plus 25% of the amount
exceed Rs. 3,000,000 exceeding Rs. 2,400,000
6 Where taxable income exceeds Rs. 3,000,000 but does not Rs. 465,000 plus 30% of the amount
exceed Rs. 4,000,000 exceeding Rs. 3,000,000
7 Where taxable income exceeds Rs. 4,000,000 Rs. 765,000 plus 35% of the amount
exceeding Rs. 4,000,000

Capital gain on immovable property


Constructed
Sr.no Holding Period Open Plot Flats
Property
1. Where holding period does not exceed one year 15% 15% 15%
2. Exceeds one year but does not exceed 2 years 12.5% 10% 7.5%
3. Exceeds 2 years but does not exceed 3 years 10% 7.5% 0%
4. Exceeds 3 years but does not exceed 4 years 7.5% 5% -
5. Exceeds 4 years but does not exceed 5 years 5% 0% -
6. Exceeds 5 years but does not exceed 6 years 2.5% - -
7. Exceeds 6 years 0% -

5|Page
Page | 6

Tax year 2024 - Where securities acquired on or after 01 July 2022

Holding Period Rate of Tax


Less than one year 15%
More than one year but less than two years 12.5%
More than two years but less than three years 10%
More than three years but less than four years 7.5%
More than four years but less than five years 5%
More than five year but less than six years 2.5%
More than 6 years 0%

Tax year 2024 - Where securities acquired between 01 July 2013 to 30 June 2022

Holding Period Rate of Tax


Irrespective of the holding period 12.5%
Tax year 2024 - Where securities acquired before 01 July 2013

Holding Period Rate of Tax


Irrespective of the holding period 0%

Depreciation Table
Sr.no Type of Assets Rate
I. Building (all types). 10%
Furniture (including fittings) and machinery and plant (not otherwise
II. specified), Motor vehicles (all types), ships, technical or professional 15%
books.
Computer hardware including printer, monitor and allied items,
III. Machinery & Equipment used in manufacture of IT Products, aircrafts 30%
and aero engines.
In case of mineral oil concerns the income of which is liable to be
IV. computed in accordance with the rules in Part-I of the Fifth Schedule. 20%
q Offshore platform and production Installations.
A ramp built to provide access to persons with disabilities not
V. exceeding 100%
Rs. 250,000 each.

6|Page
Chapter: 1
Basics of Income Tax & Scope of Income
Definition: Page | 7
Income includes;
• Any amount chargeable to tax under the Income Tax Ordinance, 2001 (e.g. income from salary)
• Any amount subject to collection or deduction of tax under final tax regime (e.g. on exports);
• Any amount treated as income under any provision of the Ordinance (e.g. dividends, royalty, profit on
debt etc.), and
• Any loss of income
Heads of income
For the purposes of the imposition of tax and the computation of total income, all incomes shall be classified
under the following heads, namely:
• 1. Salary;
• 2. Income from property;
• 3. Income from business;
• 4. Capital gains; and
• 5. Income from other sources
Sum of amounts chargeable to tax under any particular head xxx
Less:
Deductions (expenses) allowed in relevant head of income (xx)

Income under a particular head of income xxx

Tax Year
1. Tax Year-Section 74
There are three kinds of tax years as stipulated in Section 74 of the Income Tax Ordinance, 2001:
1. Normal tax year,
2. Special tax year
3. Transitional tax year.
1.1. Normal Tax Year:
A period of 12 months from 1 July to 30 June denoted by the calendar year in which the normal tax year ends.
For the year ending 30.6.2022 the tax year shall be 2022. The tax year shall be a period of 12 months ending on
June 30th (hereinafter referred to as 'normal tax year') and shall be denoted by the calendar year in which 30th
June falls.
Calendar year is a year which ends on December 31.
1.2. Special Tax Year:
Any income year ending other than 30 June is special tax year and denoted by the calendar year relevant to the
normal tax year in which the closing date of the special tax year falls. Where a person is allowed, to use a 12
months' period different from normal tax year, such period shall be that person's tax year (hereinafter referred
to as 'special tax year') and shall, be denoted by the calendar year relevant to normal tax year in which the
closing date of the special tax year falls.
Persons Accounting year
Honda Atlas Cars April 1, 2009 - March 31, 2010
Mr. Ahsan July 1, 2010- June 30, 2011
GEF Firm January 1, 2010-December 31, 2010

7|Page
ABC Company October 1, 2009-September 30, 2010
Example:
Special year 1.1.2022 to 31.12.2022 - this year end falls in the normal tax year 1.7.2022 to 30.6.2023 and
therefore tax year relevant to the normal tax year i.e. TAX YEAR 2023 shall be the Tax year for this special
year as well.
Special tax year 1.10.2022 to 30.9.2023-tax year shall be 2024.
Page | 8
The FBR has authority to prescribe any special tax year in respect of any particular class of taxpayers. Few
examples of specified special tax year are as under:
Specified Special Year End
Sugar Mills 30th September
Insurance companies 31st December
Banks 31st December

The Board,-
• In the case of a class of persons having a special tax year may permit, through official Gazette, to use a
normal tax year; and
• In the case of a class of persons having a normal tax year may permit, through official Gazette, to use a
special tax year.
Class of person Special tax year
Companies manufacturing sugar 1st October to 30th September
All insurance and banking companies 1st January to 31st December

1.3. Transitional Tax Year:


If a normal tax year or special tax year changes then the period from the day next following the last full
tax year to the date of commencement of new tax year shall be treated as transitional tax year.
Example:
Normal tax year 1.7.2021 to 30.6.2022 i.e. Tax year 2022 changes to special year 1.1.2023 to 31.12.2023 i.e.
Tax year 2024.
In this case, period from 1.7.2022 to 31.12.2022 shall be treated as a transitional tax year i.e. Transitional tax
year 2023.

Where the tax year of person changes, the period between:


• the end of the last tax year prior to change and
• the date on which the changed tax year commences
Shall be treated as a separate tax year, to be known as the "transitional tax year"
Change in the Tax Year:
a) A person using normal tax year may apply to the Commissioner to allow him to use any special tax
year.
b) A person using special tax year may apply to the Commissioner to allow him to change tax year
c) The Commissioner shall grant permission subject to conditions, if any, as the Commission special tax
year or normal tax year, may impose only if the person has shown a compelling need for the change.
d) If the Commissioner wants to reject the application, he shall provide an opportunity of being heard to
the person and shall record in the order the reasons for such rejection. In this person may file a review
application to the FBR and the decision of the FBR shall be final. case the
e) Similarly, if the Commissioner wants to withdraw his permission earlier allowed, he shall provide an
opportunity of being heard to the person and shall record in the order the reasons for such withdrawal
of permission. In this case the person may file a review application to the FBR and the decision of the
FBR shall be final.

8|Page
Exercise:
Determine the tax year in respect of each accounting periods mentioned below:
a) 1.09.2022 to 31.08.2023
b) 1.01.2023 to 31.12.2023
c) 1.04.2023 to 31.03.2024
d) 1.05.2023 to 30.04.2024
e) 1.07.2023 to 30.06.2024 Page | 9

Answer
For all the five cases mentioned above, relevant tax year will be 2024 i.e. calendar year relevant to normal tax year
[1.07.2023 to 30.06.2024) in which the closing date (31.08.2023, 31.12.2023, 31.03.2024, 30.04.2024, 30.06.2024) of
special year falls.

Exercise:
Rs
Mr. Junaid has following incomes from different heads of income:

Salary 1,075,000
Income from Property 335,000
Income from Business 1,025,000
Capital Gains on Immovable Property 237,900
Income from other sources 90,000
Compute his taxable income.
Answer
Taxable income under normal tax regime of Mr. Junaid is the sum of all above sources of income which is computed at
Rs. 2,762,900.
Basic Definitions:
Definition: Person
• an individual;
• a company or association of persons incorporated, formed, organized or established in Pakistan or
elsewhere;
• the Federal Government, a foreign government, a political subdivision of a foreign government,
• or public international organization
Definition: Association of Persons
“Association of persons” includes a firm, a Hindu undivided family, any artificial juridical person and anybody
of persons formed under a foreign law, but does not include a company;

“Firm” means the relation between persons who have agreed to share the profits of a business carried on by all
or any of them acting for all.

Definitions: Board
• the Central Board of Revenue established under the Central Board of Revenue Act, 1924 (IV of 1924), and
• on the commencement of Federal Board of Revenue Act, 2007, the Federal Board of Revenue established
under section 3 thereof;

Definitions: Company
“Company” means:
1. a company as defined in the Companies Act, 2017
2. a Small company as defined in section 2 of the Income Tax Ordinance,2001
3. a body corporate formed by or under any law in force in Pakistan;
4. a modaraba;

9|Page
5. a body incorporated by or under the law of a country outside Pakistan relating to incorporation of
companies;
6. a foreign association, whether incorporated or not, which the Board has, by general or special order,
declared to be a company for the purposes of this Ordinance; a Provincial Government; or
7. a Local Government in Pakistan; or
8. a co-operative society, a finance society or any other society
9. a non-profit organization Page | 10
10. a trust, an equity or a body of persons established or constituted by or under any law for the time being
in force
Definition: Trust
“trust” means an obligation annexed to the ownership of property and arising out of the confidence reposed in
and accepted by the owner, or declared and accepted by the owner for the benefit of another, or of another and
the owner, and includes a unit trust;
Definition: Public Company
“Public company” means —
(a) a company in which not less than 50% of the shares are held by the Federal Government or Provincial
Government;
(b) a company in which not less than 50% of the shares are held by a foreign Government, or a foreign company
owned by a foreign Government;
(c) a company whose shares were traded on a registered stock exchange in Pakistan at any time in the tax year
and which remained listed on that exchange at the end of that year; or
(d) a unit trust whose units are widely available to the public and any other trust as defined in the Trusts Act,

There are two types of Companies

Private Public
Company Company

A company other- Public Company Includes:


than Public company
- 50% ≥ Federal Government
- 50% ≥ Provisional Government
- 50% ≥ Foreign Government
- 50% ≥ Foreign Government
[Owned 100% by Foreign Govt]

Definition: Unit trust


“Unit trust” means any trust under which beneficial interests are divided into units such that the entitlements of
the beneficiaries to income or capital are determined by the number of units held
Definition: Private company
“Private company” means a company that is not a public company
Definition: Non-profit organization
“Non-profit organization” means any person other than an individual which is
(i) established for religious, educational, charitable, welfare purpose for general public or
development purposes, or for the promotion of an amateur sport;
(ii) formed and registered under any law as a non- profit organization;

10 | P a g e
(iii) approved by the Commissioner for specified period, on an application made by such person in the
prescribed form and manner, accompanied by the prescribed documents and, on requisition, such
other documents as may be required by the Commissioner;
and none of the assets of such person are available for private benefit to any other person.
Definition: Charitable purpose
“Charitable purpose” includes;
Page | 11
• relief of the poor, education, medical relief and
• the advancement of any other object of general public utility

Exercise
1) Briefly state, with reasons, whether or not you consider the below mentioned companies to be a
public company for tax purpose.
• PPL is a company incorporated under the Companies Act, 2017 and is not listed on any
stock exchange in Pakistan. 59 per cent of the shares in PPL are held by BBC Ltd, a
company incorporated in United Kingdom. United Kingdom holds 97% of the shares in
BBC Ltd.
• XYZ Limited is a public company incorporated under the Companies Act, 2017 whose
shares were traded on the Pakistan Stock Exchange from 01 August 2023 until 29 June
2024 on which date the company was delisted on the exchange.
• The Provincial Government of NWFP holds 50% of the shares in ABC Ltd, a public
company under the Companies Act, 2017. ABC Ltd is not listed on any stock exchange in
Pakistan.
• BRR is a public company under the Companies Act, 2017. 41% of the shares are held by
the Federal Government, 50% by the Government of Saudi Arabia and 9% by the
individuals and group companies. BRR is not listed on any stock exchange in Pakistan.
2) Anderson Inc, a public company incorporated under the law of the United Kingdom relating to
the incorporation of companies, has been operating in Pakistan for over 50 years. The control and
management of the Pakistan branch for the accounting year ended 31 December 2023 was
situated wholly outside Pakistan.
Required: Briefly state, with reasons whether Anderson Inc. will be assessed as a company for
Pakistan tax purposes for the relevant tax year.

Answer (a)
(iv) A public company for Pakistan tax purposes, inter alia includes a company in which not less
than 50% of the shares are held by a foreign government or a foreign company owned by a
foreign government. 59% of the shares in PPL are owned by BBC Ltd, which is a foreign
company but BBC Ltd is not wholly owned by the United Kingdom (foreign government).
Therefore, PPL is not a public company for Pakistan tax purpose.
(v) A company whose shares are traded on a registered stock exchange in Pakistan at any time in
the tax year and which remained listed on that exchange at the end of that year is a public
company for tax purpose. Although the shares of XYZ Limited were traded on the Pakistan
stock exchange during the tax year 2024, XYZ Ltd did not meet the test of being a public
company for tax purpose since its shares were not listed on the Lahore stock exchange on 30
June 2024. XYZ Ltd is therefore not a public company for tax purpose.
(vi) A company in which not less than 50% of the shares are held by the Federal Government or
Provincial Government is a public company for tax purpose. Since the Provincial Government
of NWFP holds 50% of the shares in ABC Ltd, ABC Ltd is a public company for tax purpose

11 | P a g e
(vii) A public company for Pakistan tax purposes, inter alia means a company in which not less than
50% of the shares are held by a foreign government, therefore, BRR is a public company as
50% of the shares are held by Government of Saudi Arabia.
(b) As per section 80, a company mean a body incorporated by or under the law of a country outside
Pakistan relating to incorporation of companies. Therefore, Anderson Inc. will be treated as company for
Pakistan tax purpose. Page | 12
2. Residential Status of an Individual
Residential status for tax purpose has no direct relationship with nationality or domicile. Residential status of
an individual is generally based on number of days he is physically present in Pakistan during a tax year.
Therefore, a foreigner can also be a resident person for Pakistan tax purpose. On the other hand, a Pakistan
national may become non-resident for tax purpose. A person may be a resident person in one tax year and may
be a non-resident person in other tax year.
Residential Criteria for an Individual:
An individual would be a resident person for Pakistan tax purposes if the individual:
a) Is present in Pakistan for a period of, or periods amounting in aggregate to, 183 days or more in the tax
year; or
b) Is an employee of the Federal Government or a Provincial Government posted abroad in the tax year;
or
c) Being a citizen of Pakistan, is not present in any other country for more than 182 days during the tax
year or who is not a resident taxpayer of any other country.

Rule 14 of the Income Tax Rules 2002:


Rule to count days an individual present in Pakistan becomes vital when a person has frequent visits to or from
Pakistan. Rule 14 of the Income Tax Rules prescribes the procedure for counting of days as under:
a) Part of a day that an individual is present in Pakistan counts as a whole day including:
b) A day of arrival in Pakistan
c) A day of departure from Pakistan
d) A public holiday
e) A day of leave
f) A day that the individual's activity in Pakistan is interrupted because of a strike, lock- out or delay in
receipt of supplies
g) A holiday spent in Pakistan before, during or after any activity in Pakistan A day or part of a day where
an individual is in Pakistan solely by reason of being in transit between two different places outside
Pakistan does not count as a day present in Pakistan.

Residential Status of a company


A company incorporated in Pakistan, provincial government and local government are resident without any
condition.
Other company (i.e. a company incorporated outside Pakistan) is resident if control and management of the
affairs is situated wholly in Pakistan in the year.
Residential Status of Association of Persons
AOP shall be considered as resident if control and management of the affairs is situated wholly or partly in
Pakistan in the year.

12 | P a g e
INDIVIDUAL

Citizen Non-Citizen

Page | 13

If NOT Applied,
183 Days Criteria Then Resident in any other 183 Days Criteria Only
country in Tax Year

YES
YES NO

Resident
Resident of Spent more than 182 days
that Country in any other country.

Day in transit NO Will be Pakistani Resident


count 183
FSI/PSI 183

Exercise:
Explain the residential status of the following persons for the tax year 2024:
(viii) Mr. Raza is working as Director Operations in the Ministry of Tourism. On 15 July 2023 he was
posted to Pakistan Embassy in Italy for two years.
(ix) Anderson LLC was incorporated as limited liability Company in UK. The control and
management of its affairs was situated wholly in Pakistan. However, with effect from 01
November 2023, the entire management and control was shifted to UK.
(x) On 01 February 2024, Mr.Sameel a citizen of Pakistan was sent to Pakistan by his UK based
company to work on a special project. He left Pakistan on 23 August 2024.
(xi) BBL is a non-listed public company incorporated under the Companies Act, 2017. All the
shareholders of the company are individuals. The control and management of affairs of the
company during the year was outside Pakistan.
(xii) Mr. Salman a property dealer in USA came to Pakistan on 01 February 2023. During his stay
upto 02 August 2023 in Pakistan, he remained in Peshawar upto 30 June 2023 and thereafter till
his departure from Pakistan, in Quetta. Assume that Commissioner has granted him permission
to use calendar year as special tax year.

Answer
(i) Being an employee of Federal Government, Mr. Raza would be treated as resident irrespective
of number of days he stays in Pakistan.
(ii) A company shall be resident if control and management of the affairs of the company is
situated wholly in Pakistan at any time in the year. Therefore, company is resident irrespective
of the fact that it was incorporated in UK.
(iii) The stay of Mr. Sameel for the purpose of tax year 2024 is 150 days (28+31+30+31+30).
Since his stay in Pakistan is less than 183 days in tax year 2024, he is non-resident for tax

13 | P a g e
purposes. However, if he is not present in any other country for more than 182 days during the
tax year or he is not a resident taxpayer of any other country then he will be treated as resident
of Pakistan.
(iv) If a company is incorporated or formed by or under any law in force in Pakistan, it is treated as
a resident company. Such company cannot be treated as non-resident merely on the basis that
the control and management of the affairs of the company were situated abroad. Therefore,
BBL is a resident company. Page | 14
(v) It is immaterial where he stayed in Pakistan. Number of days shall be counted from the day of
his arrival in Pakistan to the day of his departure in the following manner:
Accounting period 01 January 2023 to 31 December 2023 (Tax year 2024)

Month No. of days


February 2023 29
March 2023 31
April 2023 30
May 2023 31
June 2023 30
July 2023 31
August 2023 2
Total 184
Since he was present in Pakistan for 184 days, therefore, he is resident individual.

3. Income with reference to Resident and Non-Resident (i.e. Scope of Total / Taxable Income)

The income of a resident person is computed by taking into account amounts that are his Pakistan-source income
and amounts that are his foreign-source income.
On the other hand, income of a non-resident person is computed by taking into account only the amounts that
are his Pakistan-source income.

14 | P a g e
Page | 15

Tax Treaty
Tax treaty shall apply in case of any contradiction between local laws and tax treaty e.g. if a tax treaty provides
exemption to a particular income say dividend income, then the local laws regarding taxability of dividend
would have no effect.
Foreign source income of a short-term resident
An individual shall be exempt in respect of his foreign-source income which is not brought / received in Pakistan
if he is resident only by reason of his employment and he is present in Pakistan for not exceeding 3 years.
This section does not apply on business established in Pakistan by an individual foreigner.
Foreign source income of a returning expatriate
If an individual citizen of Pakistan (returning expatriate) is resident in the current tax year but was non-resident
in the 4 preceding tax years, his foreign-source Income shall be exempt in the current tax year and in the
following tax year.
Foreign source salary of resident individual
Foreign source salary by a resident individual is exempt in Pakistan if he has paid foreign income tax on foreign
source salary or his employer has deducted tax at source from salary and paid to the revenue authority of that
foreign country.
Salary earned outside Pakistan shall be exempt if a citizen of Pakistan leaves Pakistan during a tax year and
remains abroad during that tax year.

15 | P a g e
PAST PAPER QUESTIONS
Question # 1 [Q.6 (1) November 1994, Q.1 (iii) November 1995, Q.5 (1) May 1996, Q.1
(1) September 1998, Q.1 March 1999, Q.1(c) March 2003]

Define the term "Tax Year under the Income Tax Ordinance. Page | 16

Question # 2 [Q.6 (ii) November 1994, Q.1 (a) November 1996, Q.1 () October 1997, Q.1
(1) March 1999
Define and explain the term 'Company' with reference to the Income Tax Ordinance

Question # 3 [Q.1(1) November 1995, Q.1(c) November 1996, Q.1(iii) October 1997,
Q.1(1) September 2001, Q.7 September 2004, Q.5 September 2005, Q.5(a) March 2007]
Describe the circumstances under which the following shall be treated as resident for the purposes of charge
of tax:
(a) An individual
(b) A company
(c) An association of person

Question # 4 [Q.1 (1) October 1997, Q.1 (a) March 2003]


What does the term person mean under the Income Tax Ordinance???

Question 5 [Q.2 March 2000, Q.6 September 2000]


State the basis of taxation regarding residents and non-resident (i.e. scope of taxable income).

Question 6 [Q.1 (1) September 2001]


Define and explain the term 'Special Tax Year' with reference to the Income Tax Ordinance.

Question 7 [Q.1 (b) March 2003]


Define the term 'Resident Company'

Question 8 [Q.6 (a) September 2003, Q.6 (a) March 2014]


Under what circumstances a resident individual is entitled to claim exemption from tax on his foreign source
salary, and when is the foreign tax treated as having been paid?

Question 9 [Q.4 (b) March 2006]


Mr. A, a Pakistani Citizen, returned to Pakistan in November 20X7 after completing his employment contract
in United Arab Emirates (UAE). He worked till October 20X7 in UAE where there was no tax on salaries. Mr.
A is in Pakistan since then and has been employed by a local company.
Explain the tax implication on Mr. A's income, earned in UAE and Pakistan, for the tax year 20X8.

Question 10 [Q.2 (a) September 2006]


Explain the different types of tax years as enumerated in the Income Tax Ordinance, 2001.

Question 11 [Q.2 (b) September 2007, Q.4 (a) March 2013]


One of your clients is a subsidiary of a foreign company wants to change its accounting year from June 30 to
December 31 as the income year of its parent company ends on December 31.
Advise the client about the requirements of the Income Tax Ordinance, 2001 regarding change in tax year from
normal to special.

Question 12 [Q.2 (b) March 2009]


Briefly discuss the residential status of the following persons under the Income Tax Ordinance, 2001 for the tax
year 20X8:

16 | P a g e
• Asif is an employee of Baluchistan Government, who has been sent to United Kingdom for an official
assignment on 1.12.20X6 for two years.
• Messrs. Akhtar & Co. is a partnership firm, doing business of financial consultancy in Pakistan as well
as United Arab Emirates (UAE). The management and control of its affairs is situated partly in UAE
and partly in Pakistan.

Question 13 [Q.5 (a) March 2010] Page | 17


State the provisions of the Income Tax Ordinance, 2001 regarding the residential status of companies and
association of persons.

Question 14 [Q.4 (a) September 2011]


Discuss the residential status of the following persons for the tax year 20X8 under the Income Tax Ordinance,
2001:
• Mr. Shah has been working as an Information Analyst in the Ministry of Foreign Affairs. On 1.11.2017,
he was posted to Pakistan Embassy in Canada for three years.
• AS Learning Center is a partnership concern, providing IT training to professionals in Pakistan, UAE
and Saudi Arabia. Up to 31.7.20x7, the management and control of its affairs was situated partly in
Pakistan. However, with effect from 1.8.20X7, the entire management and control of the affairs of the
partnership was shifted to Dubai.
• Mr. LT was sent to Pakistan on a special assignment by his UK-based company on 1.3.20X8. He left
Pakistan on 9.9.20X8.
• F Trading LLC was incorporated as a limited liability company in UAE. The management and control
of its affairs are situated wholly in Pakistan.

Question 15 [Q.3 (a) September 2012]


State the provisions of the Income Tax Ordinance, 2001 with regards to the residential status of individuals and
companies.

Question 16 [Q.3 (b) September 2012]


Margaret, a German national was employed as a Technical Manager of F Chemicals Ltd, a resident company,
on 1.10.20X6 for a term of two years. Under the terms of employment, she was allowed to deliver lectures at
various professional organizations. During tax year 20X8, she conducted three workshop sessions, the details
of which are as follows:
Workshop Session in Lahore:
A fee of US$ 15,000 in equivalent Pak Rupees was received from a local event manager. The fee was credited
to her bank account maintained in Karachi.
Workshop Session in Munich:
A fee of US$ 25,000 was received in Germany in her Munich bank account.
Workshop Session in Dubai:
A fee of US$ 20,000 was remitted to her bank account in Karachi.
Required:
Discuss the taxability of the amounts received by Margaret for conducting the workshop sessions during tax
year 20X8.

Question 17 [Q.3 (a) March 2013)]


State the provisions of the Income Tax Ordinance, 2001 for determining the residential status of an
Association of Persons.

Question 18 [Q.4 (b) September 2013]


In view of the provisions of Income Tax Ordinance, 2001 and Rules made thereunder, determine the residential
status of the following persons for the tax year 2003:
• Mr. R proceeded to Saudi Arabia on 24.12.20X2 to assume responsibilities on his new job.
• He visited Karachi from 20.6.2013 to 24.6.2003 for presenting a paper in a seminar but due to
unavoidable circumstances, the seminar was cancelled.

17 | P a g e
• Mr. K, an officer working at Ministry of Foreign Affairs, since last three years, was posted to the
Pakistan's mission in Geneva from 1.8.2002 to 30.6.20x3.
• AA Associates is a partnership firm and provides consultancy services in Pakistan as well as United
Kingdom (UK). The management and control of its affairs is situated partly in UK and partly in
Pakistan.
• Mr. S, a Nigerian football coach, came to Pakistan for the first time on 28.2.2003. He left the country
on 31.8.20X3 Page | 18

Question 19 [Q.3 (a) and (b) March 2016]


Under the provisions of the Income Tax Ordinance, 2001 explain Special tax year, Transitional tax year.

Question 20 [Q.2 (a) September 2017]


Under the provisions of Income Tax Ordinance, 2001 and rules made thereunder, discuss the residential status
for tax year 20X7 in each of the following situations:
• On 21.9.2006 Asif proceeded to Dubai to join his new job. Due to certain professional issues with his
employer in Dubai, he resigned on 1.5.20X7 and came back to Pakistan. On 16.5.2007 he got a new job
in Pakistan which he continued till 30.6.2007.
• S Associates is an association of persons and provides accounting services in Dubai. On 2.1.20X7, the
entire management and control of its affairs was shifted from Karachi to Dubai.

Question 21 [Q.2 (b) September 2017]


Explain the treatment of foreign source income for tax year 2007 under each of the following independent
situations:
• Joseph, a South African, is working in Pakistan under a two years' employment contract since 20x6.
During the tax year 20X7, he earned foreign source income from his business established in South
Africa and brought 25% of the income to Pakistan.
• On 15.1.20X6 Farhan returned to Pakistan from London after 10 years and has been living in Pakistan
since then. During the tax year 2007, he received GBP 5,000 as return.

Question # 22 Spring 2018 Q. 3(c)


On 1 December 20X7 Bruce Lee was appointed by a Chinese company as a Technical Director for Pakistan.
He has provided you the following details

Arrival in Pakistan 15 December 20X7


Joined office in Pakistan 20 December 20X7
Visit to Dubai on an official trip 21-30 March 20X8
Visit to South Korea for vacations 12-21 April 20X8
Visit to northern areas of Pakistan for personal nip 4-9 June 20X8
In view of the provisions of the Income tax Ordinance, 2001 and related Rules thereunder, comment on the
residential status of Bruce Lee for the tax year 20X8. (03)

Question # 23 Autumn 2018 Q. 2


Kaleem Limited (KL) is a listed company and its accounting year ends on 30 June. KL is now considering to
change its accounting year from 30 June to 30 September.

Under the provisions of the Income Tax Ordinance, 2001:


(a) briefly describe normal , special and transitional tax year. (06)
(b) state the requirements regarding change in tax year from normal to special. (02)
(c) state the tax year corresponding to the income year ended 30 September 20X8 and the due date for filing
the return of income. (02)

Question # 24 Spring 2019 Q. 2

18 | P a g e
Mohsin has been working at the head office of Lewis Consulting, Inc. (LCI) situated in New York, USA. On 1
January 20X8, LCI had established its branch office in Pakistan and had sent Mohsin for two years as Country
Manager for looking after the Pakistan operations.
During the tax year 20X9, apart from salary income, Mohsin earned/received the following amounts:
On 15 December 20X8, he conducted a seminar in USA for a fee of USD 18,000. On his request, the event
manager transferred the amount (net of tax) directly to his personal bank account in Islamabad on 10 January
20X9. Page | 19
On 31 May 20X9, he earned income from his business established in USA and brought 40% of the income to
Pakistan.
Required:
Under the Income Tax Ordinance, 2001:
(i) state the residential status of Mohsin for the tax year 20X8. (01)
(ii) discuss the taxability of his foreign source incomes for the tax year 20X9. (04)

Question # 25 Autumn 2019 Q. 3(a)


Respond to the following independent scenarios, under the provisions of the Income Tax Ordinance, 2001:
Jean Francois, a French designer, often visits to Pakistan for promotion of his products. During his last visit he
stayed in Pakistan from 10 July 20X8 to 25 February 20X9. Determine the residential status of Jean Francois
for tax year 20X9, assuming that the Commissioner has granted him permission to use calendar year as special
tax year. (02)

Question # 26 Spring 2021 Q. 1(b)


Dr. Jamal is planning to establish a hospital as a non-profit organization.
Required:
Discuss the conditions that should be complied with by Dr. Jamal, under the Income Tax Ordinance, 2001.
(03)

Question # 27 Autumn 2021 Q. 3(a)

(a) State the provisions of the Income Tax Ordinance, 2001 relating to each of the following:
(i) Change of tax year from special to normal (02)
(ii) Change in the method of accounting for income chargeable to tax under the head ‘income from business’
(03)

19 | P a g e
ANSWERS
Question # 1 [Q.6 (1) November 1994, Q.1 (iii) November 1995, Q.5 (1) May 1996, Q.1
(1) September 1998, Q.1 March 1999, Q.1(c) March 2003]
Tax Year:
Tax year shall be a period of 12 months ending on 30th day of June i.e. ' shall be denoted by the calendar year Page | 20
in which the said date falls and includes a special year or a transitional year.

Question # 2 [Q.6 (ii) November 1994, Q.1 (a) November 1996, Q.1 () October 1997, Q.1
(1) March 1999

A company means;
i. A company as defined in the Companies Act 2017 including a small company
ii. A body corporate formed by or under any law in force in Pakistan;
iii. A modaraba;
iv. A body incorporated under the law of a country outside Pakistan relating to incorporation of companies;
v. A co-operative society, a finance society or any other society;
vi. A non-profit organization;
vii. A trust, an entity or a body of persons established or constituted by or under any law for the time being
in force;
viii. A foreign association, whether incorporated or not, which the FBR has, by general or special order,
declared to be a company for tax purposes; or
ix. A Provincial Government or a Local Government in Pakistan.

Question # 3 [Q.1(1) November 1995, Q.1(c) November 1996, Q.1(iii) October 1997,
Q.1(1) September 2001, Q.7 September 2004, Q.5 September 2005, Q.5(a) March 2007]
Taxpayer may be an individual, a company or an association of persons (AOP). Residential status of taxpayer
is summarized as under:
Residential Status of an Individual:
Residential status of an individual is generally based on number of days he is physically present in Pakistan
during a tax year. An individual would be a resident person for Pakistan tax purposes if the individual:
i. Is present in Pakistan for a period of, or periods amounting in aggregate to, 183 days or more in the tax
year; or
ii. Is an employee of the Federal Government or a Provincial Government posted abroad in the tax year;
or
iii. Being a citizen of Pakistan, is not present in any other country for more than 182 days during the tax
year or who is not a resident taxpayer of any other country.
Residential Status of a Company:
A company incorporated in Pakistan, provincial government and local government are resident without any
condition. Other company (i.e. a company incorporated outside Pakistan) is resident if control and management
of the affairs is situated wholly in Pakistan in the year.
Residential status of an AOP:
An AOP shall be considered as resident if control and management of the affairs is situated wholly or partly in
Pakistan in the year

Question # 4 [Q.1 (1) October 1997, Q.1 (a) March 2003]


"Person" the following shall be treated as persons for tax purposes, namely:-
i. An individual;
ii. A company or AOP incorporated, formed, organized or established in Pakistan or elsewhere;

20 | P a g e
iii. The Federal Government, a foreign government, a political sub-division of a foreign government, or
public international organization.

Question 5 [Q.2 March 2000, Q.6 September 2000]

Scope of total / taxable income:


Page | 21
A resident person is taxable for his world over income subject to agreement for the avoidance of double
taxation (Tax Treaty).
A non-resident person is taxable only for his Pakistan-source income subject to Tax Treaty. However, a
resident person shall not be taxable for his foreign source income in the following cases:
Foreign source income of a short-term resident:
An individual shall be exempt in respect of his foreign-source income which is not brought / received in
Pakistan if he is resident only by reason of his employment and he is present in Pakistan for not exceeding 3
years.
This concept does not apply on business established in Pakistan by an individual foreigner.
Foreign source income of a returning expatriate:
If an individual citizen of Pakistan (returning expatriate) is resident in the current tax year but was non-
resident in the 4 preceding tax years, his foreign-source income shall be exempt in the current tax year and in
the following tax year.
Foreign source salary of resident individual:
Foreign source salary by a resident individual is exempt in Pakistan if he has paid foreign income tax on
foreign source salary or his employer has deducted tax at source from salary and paid to the revenue authority
of that foreign country. Salary earned outside Pakistan shall also be exempt if a citizen of Pakistan leaves
Pakistan during a tax year and remains abroad during that tax year.

Question 6 [Q.1 (1) September 2001]

Special Tax Year:


Any income year ending other than 30 June is special tax year and denoted by the calendar year relevant to the
normal tax year in which the closing date of the special tax year falls.
Example:
Special year 1.1.2018 to 31.12 * 0.2018 - this year end falls in the normal tax year 1.7.2018 to 30.6.2019 and
therefore tax year relevant to the normal tax year i.e. TAX YEAR 2019 shall be the tax year for this special
year as well.

Question 7 [Q.1 (b) March 2003]

Resident Company:
A company incorporated in Pakistan, provincial government and local government are resident without any
condition and other company (i.e. a company incorporated outside Pakistan) is resident if control and
management of the affairs is situated wholly in Pakistan in the year.

Question 8 [Q.6 (a) September 2003, Q.6 (a) March 2014]

Foreign source salary by a resident individual is exempt in Pakistan if he has paid foreign income tax on
foreign source salary or his employer has deducted tax at source from salary and paid to the revenue authority
of that foreign country.
Salary earned outside Pakistan shall also be exempt if a citizen of Pakistan leaves Pakistan during a tax year
and remains abroad during that tax year.

Question 9 [Q.4 (b) March 2006]

21 | P a g e
Mr. A is a resident person during the year for tax purpose as he was in Pakistan during November to 30 ^ m
June i.e. for more than 182 days.
His Pakistan source salary is taxable in Pakistan.
Foreign source salary income of a resident person is exempt in Pakistan inter alia if a resident person is a
returning expatriate. If an individual citizen of Pakistan is resident in the current tax year but was non-resident
Page | 22
in at least 4 preceding tax years, his foreign-source income including foreign source salary shall be exempt in
the current tax year and in the following tax year.

Question 10 [Q.2 (a) September 2006]

Tax year is of 3 types; Normal tax year, Special tax year and Transitional tax year.
Normal Tax Year:
A period of 12 months from 1 July to 30 June denoted by the calendar year in which the normal tax year ends
e.g. for the year ending 30.6.2022 the tax year shall be 2022.
Special Tax Year:
Any income year ending other than 30th June is special tax year and denoted by the calendar year relevant to
the normal tax year in which the closing date of the special tax year falls.
Example: Special year 1.1.2018 to 31.12.2018 - this year end falls in the normal tax year 1.7.2018 to
30.6.2019 and therefore tax year relevant to the normal tax year i.e. TAX YEAR 2019 shall be the tax year for
this special year as well.
Transitional Tax Year:
If a normal tax year or special tax year changes then the period from the day next following the last full tax
year to the date of commencement of new tax year shall be treated as transitional tax year.

Question 11 [Q.2 (b) September 2007, Q.4 (a) March 2013]

(a) A person using normal tax year may apply to the Commissioner to allow him to use any special tax year.
(b) The Commissioner shall grant permission subject to conditions, if any, as the Commissioner may impose
only if the person has shown a compelling need for the change.
(c) If the Commissioner wants to reject the application, he shall provide an opportunity of being heard to the
person and shall record in the order the reasons for such rejection. In this case the person may file a review
application to the FBR and the decision of the FBR shall be final.

Question 12 [Q.2 (b) March 2009]


i. A government employee posted abroad in the tax year is resident irrespective of his physical stay in
Pakistan and therefore Mr. Asif is a resident person for tax purpose.
ii. An Association of Persons (AOP) shall be considered as resident if control and management of the
affairs is situated wholly or partly in Pakistan in the year. Therefore, Akhter & Co. being an AOP
having management and control of its affairs partly in Pakistan is a resident person.

Question 13 [Q.5 (a) March 2010]

Residential status of a company:


A company incorporated in Pakistan, provincial government and local government are resident without any
condition.
Other company (i.e. a company incorporated outside Pakistan) is resident if control and management of the
affairs is situated wholly in Pakistan in the year.
Residential status of an AOP:
An Association of Persons shall be considered as resident if control and management of the affairs is situated
wholly or partly in Pakistan in the year.

22 | P a g e
Question 14 [Q.4 (a) September 2011]

1. Being an employee of Federal Government Mr. Shah would be treated as a resident person
irrespective of number of days he stays in Pakistan.
2. AS Learning Centre (ALC), being a partnership firm AOP shall be a resident person for a tax year if
the control and management of its affairs, is an AOP, situated wholly or partly in Pakistan at any time Page | 23
in the year. Since ALC's management and control of affairs were present partly in Pakistan during the
tax year, it shall be considered as a resident person.
3. The stay of Mr. LT during the relevant tax year is 122 days i.e. from 1.3.20X8 to 30.6.20x8 (31+30
+31+30). Since his stay in Pakistan is less than 183 days, he is a non-resident for tax purposes.
4. Since the management and control of affairs of F Trading LLC was wholly situated in Pakistan during
the relevant tax year, it is a resident company irrespective of the fact that it was incorporated in UAE.

Question 15 [Q.3 (a) September 2012]

Residential status of an individual is generally based on number of days he is physically present in Pakistan
during a tax year.
An individual would be a resident person for Pakistan tax purposes if the individual:
i. Is Present in Pakistan for a period of, or periods amounting in aggregate to, 183 day or more in the tax
year; or
ii. Is an employee of the Federal Government or a Provincial Government posted abroad in the tax year;
or
iii. Being a citizen of Pakistan, is not present in any other country for more than 182 days during the tax
year or who is not a resident taxpayer of any other country.
Residential Status of a company
A company incorporated in Pakistan, provincial government and local government are resident without any
condition.
Other company (i.e. a company incorporated outside Pakistan) is resident if control and management of the
affairs is situated wholly in Pakistan in the year.

Question 16 [Q.3 (b) September 2012]


The foreign-source income of a short-term resident individual is exempt if he/she is:
(I) a resident individual solely by reason of the individual's employment; and
(ii) Present in Pakistan for not exceeding three years
Ms. Margaret is a short-term resident person having fulfilled the above conditions.
However, exemption is not available to any income derived from a business of the person established in
Pakistan; or any foreign-source income brought into or received in Pakistan.
Keeping in view the above provisions, taxability of her activities is as under:
(1) Receipt of US$ 15,000 in equivalent Pak Rupees for conducting the workshop in Lahore is Pakistan source
income and taxable wherever received.
(2) Receipt of US$ 25,000 for conducting the workshop in Munich is foreign source income, and shall not be
taxable as it has neither been received in nor brought into Pakistan.
(3) Receipt of US$ 20,000 for conducting the workshop in Dubai is foreign source income and shall be
taxable as it has been brought into Pakistan.

Question 17 [Q.3 (a) March 2013)]

An Association of Persons shall be considered as resident if control and management of the affairs is situated
wholly or partly in Pakistan in the year.

23 | P a g e
Question 18 [Q.4 (b) September 2013]

(1) Total number of days Mr. R is present in Pakistan during the tax year 20X3 is as under:
1.7.20X2 to 24.12.20X2 177
20.6.20X3 to 24.6.20x3 5 Page | 24
Days physically present in Pakistan 182
Total stay in tax year 20X3 is less than 183 days and therefore Mr. R is a non-resident person.
(2) A Government Employee posted abroad is considered as a resident person irrespective of his physical stay
in Pakistan.
Therefore, Mr. K is a resident person for the tax year 2013.
(3) AOP shall be considered as resident if control and management of the affairs is situated wholly or partly in
Pakistan in the year.
Therefore, AA Associates, being an AOP, is resident person as its control and management partly exists in
Pakistan.
(4) Total number of days Mr. S is present in Pakistan during the tax year 20X3 is as under:
28.2.20X2 to 30.6.20X3 123
Total stay in tax year 2013 is less than 183 days and therefore Mr. S is a non-resident person.
Physical stay in July and August 20X3 will be considered in the tax year 20X4.

Question 19 [Q.3 (a) and (b) March 2016]

Special Tax Year:


Any income year ending other than 30th June is special tax year and denoted by the calendar year relevant to
the normal tax year in which the closing date of the special tax year falls.
Transitional Tax Year:
If a normal tax year or special tax year changes then the period from the day next following the last full tax
year to the date of commencement of new tax year shall be treated as transitional tax year.

Question 20 [Q.2 (a) September 2017]

(1) An AOP is treated as a resident person if the management and control of its affairs is situated partly or
wholly in Pakistan during the year.
Therefore, S Associates would be considered a resident person irrespective of the fact that its entire
management and control of affairs was subsequently shifted from Karachi to Dubai.

Question 21 [Q.2 (b) September 2017]

(1) Joseph is considered as a short-term resident in Pakistan for income tax purpose as:
His stay in Pakistan is solely by reason of his employment; and he is present in Pakistan for not exceeding 3
years
Foreign source income of a short-term resident is exempt in Pakistan if it is not brought / received in Pakistan.
Therefore, 25% of his foreign source income which he brought into Pakistan is taxable whereas 75% of his
foreign source income which he kept outside Pakistan is exempt.

24 | P a g e
(ii) If an individual citizen of Pakistan (returning expatriate) is resident in the current tax year but was non-
resident in the 4 preceding tax years, his foreign-source income shall be exempt in the current tax year and in
the following tax year.
Therefore, foreign source income of Mr. Farhan in the tax year 20X7 is exempt.

Question # 22 Spring 2018 Q. 3(c)


Page | 25
Mr. Bruce Lee is not a resident for the tax year 2017 as he was present for less than 183 days in Pakistan i-e
178 days as follows:
Months Days
December 2016 (15-12-16 is also included) 17
January 2017 31
February 2017 28
March 2017 31
April 2017 30
May 2017 31
June 2017 30
198
Less: Number of days visited to Dubai - March (8)
Less: Number of days visited to South Korea - April (8)
182
For not considering the days spent on personal trip within Pakistan.

Examiner Comments:
This part required the candidates to comment on the residential status of a Chinese citizen
(Bruce Lee) who was appointed as Technical Director for Pakistan by a Chinese company.
Though the performance was reasonable, the following errors were commonly observed:

▪ While calculating the number of days Bruce Lee stayed in Pakistan:


- 15 December 20X7 i.e. the date of his arrival in Pakistan was not counted.
- His visit to Dubai on an official trip 21-30 March 20X8 was counted as nine days.
- His visit to South Korea for vacations from 12-21 April 20X8 was counted as nine
days.
- Days spent on his visit to northern areas of Pakistan for personal trip were excluded
for the purpose of calculating the stay in Pakistan.
- Many students did not calculate the number of days Bruce Lee was in Pakistan and
simply mentioned that Bruce Lee was non-resident.
▪ Some students stated that Bruce Lee was appointed by a Chinese company or was the
employee of a Chinese company and he is in Pakistan for the sake of his employment as
technical director, so he is a non-resident.

Marking Scheme:
▪ Determination of sale proceeds 1.5
▪ Computation of gain/loss under correct head of account 1.0

Question # 23 Autumn 2018 Q. 2


(a) Normal tax year
Normal tax year is a period of twelve months ending on the 30th day of June and is denoted by the calendar
year in which the said date falls.

Special tax year

25 | P a g e
Where a person’s income year is different from the normal tax year, or where, by an order, a person has
been allowed by the Commissioner to use a twelve months’ period different from normal tax year, such
income year or such period shall be that person’s special tax year and shall be denoted by the calendar year
relevant to the normal tax year in which the closing date of the special tax year falls.

Transitional tax year


Where the tax year of a person changes as a result of an order by the Commissioner of Income tax either Page | 26
from the normal tax year to special tax year or vice versa, the period between the end of the last tax year
prior to change and the date on which the changed tax year commences shall be treated as transitional tax
year. (Also called separate tax year)

(b) Change of tax year from normal to special


▪ A person shall apply in writing to the Commissioner for change in tax year from normal to special.
▪ The Commissioner should grant permission only if he is satisfied that the company has a compelling
need to use special tax year.
▪ While giving the permission, the Commissioner may impose such conditions as he may deem fit.

(c)
▪ Tax year is 20X9.
▪ Due date for filing of return is 30 September 20X9.

Examiner Comments:
This question consisted of three parts. The overall performance was reasonable as 52%
candidates secured passing marks. Part wise comments are given below.

Marking Scheme:
(a) Brief explanation of the provisions related to:
▪ normal tax year 1.5
▪ special tax year 2.5
▪ transitional tax year 2.0
(b) Explanation of the provisions related to the change of tax year from normal to special 2.0
(c) Identification of:
▪ the tax year corresponding to the income year ended 30 September 20X8 1.0
▪ the due date for filling the return of income 1.0

Question # 24 Spring 2019 Q. 2

(b)
(i) Mohsin is not a resident individual as he spent less than 183 days in Pakistan during the tax year 20X8.
Days Count
Month Days
January 31
Febrary 28
March 31
April 30
May 31
June 30
Total 181

(ii) Under the ITO-2001 the foreign-source income of a short term resident individual shall be exempt from
tax if he/she is:
▪ a resident individual solely by reason of the individual’s employment; and

26 | P a g e
▪ present in Pakistan for a period or periods not exceeding three years.

However, the above rule is not applicable to any foreign-source income brought into or received in
Pakistan.

Mohsin is a short term resident individual as he is in Pakistan for employment and his stay is less than
three years. Based on the above rule: Page | 27
▪ Receipt of US$ 18,000 in equivalent Pak Rupees for conduction the seminar session in USA shall be
taxable as it has been received in Pakistan. However, he can claim the foreign tax credit of the amount
paid as income tax in USA.
▪ 40% of his foreign source income which he brought into Pakistan is taxable whereas 60% of his foreign
source income which he kept outside Pakistan is exempt from tax.

Examiner Comments:
(b)
(i) The amount recovered will be taxable under the head ‘income from other sources’. In
fact, it should be charged under ‘income from business’.
(ii) Students failed to mention that the amount would only be offered for tax if the write off
has been claimed / allowed in the previous year(s).

Marking Scheme:
(b) (i) Comment on the residential status of Mohsin 1.0
(ii)
▪ Discussion on the relevant provision of the law 2.0
▪ Application of the law on the given scenario 2.0

Question # 25 Autumn 2019 Q. 3(a)

In view of the permission granted by Commissioner-IR to Jean Francois to use special tax year, the number of
days he spent in Pakistan beyond 31 December 20X8 would fall under tax year 20Y0. As a result, John is a non-
resident person because his total stay in tax year 20X9 is 175 days (i.e. from 10 July 20X8 to 31 December
20X8) which is less than 183 days.
Days Count.
Month Days
July 22
August 31
September 30
Octobeer 31
November 30
December 31
Total 175

Examiner Comments:
Majority of the examinees ignored the fact that Jean Francois was allowed to use special
tax year by Commissioner-IR and wrongly determined his residential status based on
normal tax year as resident individual.

Marking Scheme:
▪ Discussion on provision relating to special tax year 1.5
▪ Conclusion 0.5

27 | P a g e
Question # 26 Spring 2021 Q. 1(b)

Under the ITO-2001, an individual cannot form a non-profit organization. However, Dr. Jamal can run his clinic
as a non-profit organization if:
1. it is established for charitable purposes only;
2. he gets his clinic registered as a company or association of persons under any law;
3. he gets approval of the Commission for specified period; and Page | 28
4. none of the assets of his clinic are available for private benefit to any other person.

Examiner Comments:
▪ Examinees did not know that Jamal as an individual cannot form a non-profit
organization and he has to get his clinic registered as a company or an association of
persons.
▪ Examinees correctly stated that approval of the Commission is required but did not
mention that it should be for a specified period.
▪ Examinees wasted time in mentioning the requirement relating to Companies Act, 2017.

Marking Scheme:
Up to 0.75 mark for identification of each condition required to be complied with 3.0

Question # 27 Autumn 2021 Q. 3(a)

(i) A person may apply in writing to the Commissioner for change in tax year from special to normal. The
Commissioner grants permission only if he is satisfied that it has a compelling need to use normal tax year.
While giving the permission, the Commissioner may impose conditions as he may deem fit. An order of
the Commissioner for change of tax year shall take effect from such date, being the first day of the normal
tax year as may be specified in the order.

(ii) A person may apply in writing for a change in the person’s method of accounting to the Commissioner.
The Commissioner may, by notice in writing, approve such an application but only if satisfied that the
change is necessary to clearly reflect the person’s income chargeable to tax under the head “Income from
Business”. If a person’s method of accounting has changed, the person shall make adjustments to items of
income, deduction, or credit, or to any other items affected by the change so that no item is omitted, and
no item is taken into account more than once.

Examiner Comments:
▪ In point no. (i), examinees failed to mention that while giving the permission, the
Commissioner may impose conditions as he may deem fit.
▪ In point no. (ii), examinees failed to mention that items affected by the change shall be
adjusted in a manner that no item is omitted and no item is taken into account more than
once.

Marking Scheme:
0.5 mark for each provision related to change of tax year from special to normal 2.0

28 | P a g e
Chapter 2 INCOME FROM SALARY
Mr. Name
Individual ; Resident Page | 29

Tax year ; 2024


Computation of Taxable Income for the year
1. Salary;
2. Income from property;
3. Income from business;
4. Capital gains; and
5. Income from other sources
Salary
Model: Benefits received from Employer, No Deductions are allowed
Definitions (Sec. 21
"Employee" means any individual engaged in employment;
"Employer" means any person who engages and remunerates an employee;
"Employment" includes-
I. A directorship or other office involved in the management of a company (Any remuneration paid to the
directors for the purposes of rendering management services to the company will fall under the definition of
employment.)
II. A position which gives the employee a fixed or ascertainable remuneration (It includes a person which
is remunerated on the basis of sales made by him)
III. The holding or acting in any public office (Public offices refers to MNAs and MPAs)
Note:
It's important that master servant relationship exists between the person who is using the service and the person who is
rendering the service. If it does not exist, it will not be treated as an employment, rather it will be considered as
independent service rendered to the person who has hired him.
Example 1
The relationship between SBI College of accountancy and permanent faculty member is an employment relationship,
while the relation between SBI College of accountancy and visiting faculty member is not an employment
relationship.
The remuneration paid to the permanent faculty member is fixed irrespective of number of lectures taught by him,
while the remuneration paid to the visiting faculty member varies with the number of lectures taught by him.
Example 2
ABC Company is in the business of repairing computers. The company has engaged Mr. Aslam on a monthly salary of
Rs. 30,000 and his job is to repair the computers of the company. One of the computers was damaged badly due to
some mishandling. The company called Mr. Babar from Hafeez center who charged the company Rs. 200/ hour for
three hours worked by him.
The relationship between ABC Company and Mr. Aslam is an employment relationship, while the relationship
between ABC Company and Mr. Babar is not an employment relationship because he is providing services in an
independent capacity.

29 | P a g e
Basis of Taxation
Salary is taxable on receipt basis i.e. any salary received by an employee in a tax year shall be chargeable to
tax.
Receipt [M.A.P]
A person shall be treated as having received an amount, benefit, or perquisite if it is - Page | 30
a) Actually received by the person;
b) Paid
1. on behalf of the person,
2. at instruction of the person or
3. under any law; or
c) Made available to the person. [Section 69

Explanation
a) Mr. A has received the salary in cash

b) Applied on behalf
Employer deducts Rs. 2,000 per month from employee's salary and deposits the same amount in a provident fund
maintained on behalf of the employee.
At the instruction of person
Mr. Shah is working as an employee in a company. He has instructed to pay Rs. 10,000 as club membership fee
Under any law
Mr. Ajmal kanju is working as an employee in company on a monthly salary of Rs. 400,000. He has made default
in the payment of property tax amounting to Rs. 25,000.The Court passed an order to the company to deduct Rs
25,000 from his salary and deposit the same amount in Government treasury.
When the employer will follow the instructions, the employee will be treated to have received the salary.
c) Made available to the person
Employer has handed over the cheque to the employee. Employer has provided to employee the benefit in kind
instead of cash.
Laptop given to an employee instead of salary (benefit in kind).

Example
Mr. Daniyal is working as Front Desk Officer in SBI. He joined the SBI on August 1, 2022. In TY 2023 salary
per month was Rs. 30,000 and in TY 2024 his salary per month was Rs. 40,000. As per the policy company pays
salary to the employee on the 2nd day after the end of each month. Calculate his salary income TY 2022 and
2023 on the basis of available information.
Answer
Salary Income for TY 2022
(10 months x Rs 30,000/month) = Rs. 300,000
(First receipt is in the month of September)

Salary Income for TY 2023


(1 months x Rs 30,000/month + 11 month x Rs 40,000/ month) = Rs. 470,000
(First receipt is in the month of July (TY 2012) which represents June 2011 salary)

Exception to the Receipt Basis

30 | P a g e
• In case of receipt of amount under salary which is paid in arrears and is expected to be charged at
rate higher than the rate which would have been charged if the amount was received in its relevant
tax year, the employee may by a notice to the Commissioner elect for tax rate applicable in the tax
year in which such salary was earned.
• The above option shall be exercised by the due date for furnishing employee’s return of income for
the tax year in which the amount was received or such later date as may be extended by the Page | 31
Commissioner.
Question
Mr. Rana has disclosed the following information
TỶ 2020 TỶ 2021
Salary/ Month 30,000 50,000
Due to some disturbance in city the salary of last 4 months of TY 2020 was given in TY 2021 and all other
payments were made on time.
You should assume the following slab rates:
200,001-400,000 0%
400,001-650,000 1%
650,001-900,000 5%

Answer
Option 1(On received basis)
TY 2020
Taxable Income (30,000x8) 240,000
Tax Liability Nil
TY 2021
Taxable Income (30,000x4+50,000x12) 720,000
Tax Liability (720,000x5%) 10,800
Option 2(On accrual basis)
TY 2020
Taxable Income (30,000x12) 360,000
Tax liability Nil
TY 2021
Taxable Income (50,000x12) 600,000
Tax liability (600,000x1%) 6,000
Tax Payable:
Option I 10,800
Option 2 6,000
Therefore option 2 is feasible
Definition of salary
Salary received by an employee from any employment, whether of a revenue or capital nature, including -
Capital receipts an example of capital receipt is golden hand shake receipt at the time of retirement.
(1) Any pay, wages or remuneration provided to an employee, including leave pay, payment in lieu of
leave overtime, bonus, commission, fees, gratuity or work condition supplements (such as for
unpleasant dangerous working conditions):
Gratuity: [It will be discussed later in chapter]
Work condition supplements [Mr. Ameer is working in a company which produces synthetic acid.
During the process of manufacturing some liquid evaporates which is dangerous for the health of
employees. In compensation for working in such dangerous conditions company pays Rs 2,000 per
month to employee. It will be Salary income of Mr. Ameer]
(2) Any perquisite [It will Be discussed Later in chapter]

31 | P a g e
(3) Any allowance provided by an employer including a cost of living, subsistence, rent, utilities,
education, entertainment, or travel allowance, excluding allowances solely expended for performing
official duties;
Explanation: Any allowance solely expended in the performance of employee's duty does not include:
❖ Allowance which is paid in monthly salary on fixed basis or percentage of salary or
❖ Allowance which is not wholly, exclusively, necessarily or actually spent on behalf of
Page | 32
employer;
(4) An expense of an employee that is paid or reimbursed by the employer, other than reimbursements
performing official duties;
(5) The amount of any profits in lieu of, or in addition to, salary including any amount received -
❖ By a person for agreeing to enter into an employment relationship;
❖ By a person for agreeing to any conditions of employment or any changes to those
conditions;
❖ On termination of employment, including compensation for redundancy or loss of
employment a golden handshake payment;
❖ From a provident or other fund, if it is not a repayment of contribution by the employee [it
will be discussed at the end of this document]; and
❖ By a person for agreeing to a restrictive covenant for any past, present or prospective
employment.
(6) any pension or annuity or any supplement to a pension or annuity received / receivable from
employer;
(7) any amount of gain earned under “Employee Share Scheme”;
(8) amount of tax chargeable on employee’s salary and borne by employer.
Example:
Mr. Arif Habib is a CFO of HBL Ltd., which is a leading commercial bank. A new company, Tesla Ltd. (TL) has recently
established with the strategy of acquiring top executives from the market. TL has approached Mr. Arif to work as a CFO.
TL offers him a handsome salary along with following benefits:
Rs. 2,000,000 for agreeing to enter into contract of employment with PJL and leaving MBL
[Example of (5i)]
Rs. 4,200,000 for condition that Mr. Arif will not enter into contract with any other Company for a period of 5 years.
[Example of (5v)]
Other terms:
During the term of employment, Mr., Arif will not be posted abroad. If Mr. Arif is posted abroad, he will be paid
compensation. [Example of (5ii)]
If due to recession, PJL intends to layoff the employees, each employee will have an option to continue employment or
opt voluntary retirement in consideration of some termination benefit. [Example of (5iii)]
All such amounts shall be taxable under the head "Income from Salary"

➢ An employee’s salary income, wherever received is taxed in Pakistan to the extent it relates to
employment exercised in Pakistan. However, salary received by Pakistan Government employee is
taxable in Pakistan whether employment is exercised in Pakistan or abroad.

32 | P a g e
➢ A salaried taxpayer means a taxpayer whose salary income constitutes more than 75% of his taxable
income.

Page | 33

Example
Mr. Bilal is working as an executive in a beverage company. He has provided you with the following details in
order to calculate his taxable income.
Rs.
Basic salary per month 100,000
House rent allowance per month 30,000
Allowance for official duty is 5% of basic salary
He received an allowance of Rs. 600,000 for attending a business meeting in foreign country. He spent Rs.
450,000.

Solution
Basic salary (100,000 x 12) 1,200,000
House rent allowance (30,000 x 12) 360,000
Allowance for official duty (1,200,000 x 5%) 60,000
Allowance for meeting -----
Taxable Income 1,620,000

Amount or Perquisite when treated received


An amount or perquisite shall be treated as received by an employee from any employment regardless of
whether the amount or perquisite is paid or provided:
➢ By the employee’s employer, an associate of the employer, or by a third party under an arrangement
with the employer or an associate of the employer;
➢ By a past employer or a prospective employer; or
➢ To the employee or to an associate of the employee or to a third party under an agreement with the
employee or an associate of the employee

33 | P a g e
Allowances
Any benefit provided by employer to the employee in fixed amount and in cash
Example
Cost of Living Allowance
Page | 34
House Rent Allowance

Medical Allowance

Utilities' Allowance

Educational Allowance

Conveyance Allowance

Traveling Allowance

Dearness Allowance

Introduction to valuation of perquisites


The term ‘perquisites’ may be defined as the casual emolument, fee or profit attached to an office or position
in addition to salary or wages. Perquisite may be cash (such as utility) or in kind (such as accommodation or
motor vehicle provided by the employer to an employee).
Conveyance Provided

Following will be added in the income if employer has provided employee a motor vehicle in a tax year.
Use Rule
Partly for personal and 5% of
partly for official use (a) the cost to the employer For Owned Vehicle; or
(b) The fair market value of the motor vehicle at the commencement of
its lease, if it is taken on lease.
For personal Use 10% of
(a) the cost to the employer For Owned Vehicle
(b) The fair market value of the motor vehicle at the commencement of
its lease, if it is taken on lease
For official use 0%

Running and maintenance benefit for car will be separately taxable.


For above rule, "employee" includes a director of a company.
If in the question it is mentioned that employee is paying anything to employer in respect of service availed
then this will not be deducted because as per law only 10% or 5% is to be added as the case may be.

Question-1 Car acquired through purchase (by the employer)


Mr. Atif is a director of SBI. The SBI has provided him a car as a part of his salary package. The cost of the
car purchased by company is Rs. 8,000,000. Compute the amount to be added in the salary income if the
car has been provided for personal use only.
Answer-1
Amount to be added in salary income (10% x Rs. 8,000,000) 800,000

Question-2 Car acquired on lease (by the employer)


Mr. JZ is a campus director of SBI. The SBI has provided him a car as a part of his salary package. The fair
market value of the car is Rs. 1,800,000 at the inception of lease. Gross lease rentals payable by the

34 | P a g e
employer amounts to Rs. 32,000 per month for five years. Compute the amounts to be added in the salary
income of Mr. JZ if:
(i) The car has been provided for both official and personal use
(ii) The car has been provided for personal use only.
Answer-2
(a) Amount to be added in salary income: (5% x Rs. 1,800,000) 90,000
(b) Amount to be added in salary income: (10% x Rs. 1,800,000) 180,000 Page | 35

Note: Answer-2
Lease rentals is irrelevant i.e. Rs. 1,320,000 (22,000 per month x 12 months x 5 years) is irrelevant. We will
use fair market value at the inception of lease
➢ Where, the services of a housekeeper, driver, gardener or other domestic assistant are provided by an
employer to an employee, the following shall be added in the salary income: The salary paid by
employer to person providing service less amount paid by employee to employer.
➢ "Services" includes the provision of any facility;
➢ If utilities are provided by an employer to an employee, the "Salary" shall include the fair market
value the utilities less amount paid by the employee to employer. "Utilities" includes electricity, gas,
water and telephone.
Question:
Mr. Imran.is working for Mr Hafiz. As per the terms of employment the employer pays directly for the
utility bills of the accommodation to the service providers of utilities against which employee has to pay a
nominal amount of Rs. 2,000 per month to the employer. The actual cost incurred by employer in whole of
the tax year for utilities for Imran’s accommodation is as follows:
Water 100,000
Electricity 200,000
Gas 30,000
Basic salary per month is Rs. 50,000. Calculate taxable income.
Answer
Basic salary (50,000 x 12) 600,000
Cost incurred by employer (100,000+ 200,000+30,000) 330,000
Less: Paid by employee to employer (2,000 x12) (24,000) 306,000
Taxable income 906,000

Interest free/concessional loan


Where a loan is provided by an employer to an employee the salary shall include-
Scenario Amount to be added in salary income

Where no profit (interest) on loan is Interest on loan calculated at bench mark rate
paid by an employee
Where profit (interest) on loan paid by The difference between the profit on loan paid by
employee is less than benchmark rate the employee and profit on loan computed at the
benchmark rate
However, nothing will be added in the salary income if:
- The loan is up to Rs. 1,000,000.
- The interest benefit is arising because employee has waived the interest to employer when
employee had provided the loan to the employer. (For example, as in case of provident fund
sometimes)
It means that if the employee has given an interest free loan to the employer and in compensation employer
also gives to the employee an interest free loan than nothing will be added in salary income of employee
irrespective of the fact that he is earning a benefit.

35 | P a g e
"BENCHMARK RATE" MEANS-
(i) For the tax year 2003, a rate of 5% per annum; and
(ii) The rate for each coming year will be increased by 1%, but a cap will be placed at 10%
So rate for current and coming years is fix at 10%.

Question Page | 36
Mr. Aleem is an employee of CPC Company. Mr. Aleem has availed a loan of Rs. 2,000,000 in tax year
2023
What amount shall be added into his income from salary on following alternate assumptions:
(a). No profit on loan is payable to company by Mr. Aleem
(b). Profit is payable to company at 6% per annum
(c). Profit is payable to company at 18% per annum
Answer
(a) Amount to be added in salary income on account of profit on loan.
(Rs. 2,000,000 x 10%) 200,000
(b) Amount to be added in salary income on account of profit on loan.
[Rs. 2,000,000 x (10% -6%)] 80,000
(c) Nothing will be added in his income.

Where employee uses a loan obtained from employer for acquisition of any asset or property producing
income under any head of income, it will be assumed that employee has paid profit (interest) equal to the
benchmark rate or actual [Higher of]

Question
Mr. Faizan has obtained a loan of Rs. 1,600,000 from employer on February 1, 2023 for the purpose of
purchasing a shop for his Tandoor. The interest charged by employer on this loan is 4% per annum. He has
provided you following details regarding TY 2023.
Details regarding business income
Revenue earned from business 1,200,000
Expenses incurred 300,000
Details regarding salary income
Basic salary per month 50,000
Hose rent allowance per month 30,000
Required: Calculate his taxable income?

Answer
Income from salary
Basic Salary (50,000 x 12) 600,000
House rent allowance (30,000 x 12) 360,000
Interest benefit on loan (1,600,000 x (10%-4%) x 5/12) 40,000 1,000,000

Income from business


Revenue earned from business 1,200,000
Less: Expenses incurred (300,000)
Less: Interest on loan calculated at
Benchmark rate (1,600,000 x 10% x 5/12) (66,667) 833,333
1,833,333
Note: If in above case interest charged by employer is higher than bench mark rate, than actual interest paid
will be allowed as expense.

36 | P a g e
Where an obligation of an employee towards employer is waived by the employer, it will be added in salary.
Obligation of employees may the following:
- Loan received by an employee
- Goods or services purchased on credit by employee
Question
Mr. Ali was entitled to the following during the tax year 2023 from his employer
Basic Salary for the tax year: 520,000 Page | 37
Education Allowance; 100,000
Mr. Ali obtained loan from his employer in tax year 2015 amounting to Rs. 1,400,000 but was unable to pay
till now. His employer has decided to waive the loan.
Required:
Compute the taxable income from salary of Mr. Ali?

Answer
Basic Salary for the tax year 520,000
Education Allowance 100,000
Loan waived by the employer 1,400,000
Taxable income from salary 2,020,000

➢ Where an obligation of an employee towards a third party is paid by the employer, it will be added in
salary.

Question
Suppose in above example, Mr. Ali also obtained loan from a Bank in tax year 2017 amounting to Rs.
380,000 but was unable to pay till present tax year. As a benefit, his employer has decided to pay the loan as
well. Compute the taxable income from salary of Mr. Ali for the tax year.
Answer:
Salary for the tax year 520,000
Education Allowance 100,000
Loan waived by the employer 1,400,000
Loan paid to the bank by the employer on Mr. Anjum's behalf 380,000
Taxable income from salary 2,400,000
Where a property is transferred or services are provided by an employer to an employee, salary shall include:
➢ The Fair market value of service at the time it is provided less amount paid by employee to the
employer.
➢ Fair market value of property at the time of transfer less amount paid by employee to the employer.

Question-1
Employer of Mr. Moye Moye agrees to provide pick and drop service from school to his home for his
children. The monthly expenditure for the employer in this regard amounts to Rs. 25,000.
Answer:
Rs. 300,000 (25,000 x 12 months) shall be added in the salary income of the Mr. Moye Moye
Question-2
Suppose Mr. Moye Moye pays Rs. 7,000 per month on account of pick and drop service provided by the
employer.
Answer:
Amount to be added in income from salary shall be computed as follows:
Amount paid by employer: 300,000
Less: Amount paid by Mr. Moye Moye: (7,000 x 12 months) (84,000)
Amount to be added in income from salary 216,000

37 | P a g e
Valuation of Accommodation [Rule 4 of Income Tax Rules, 2002]
The value of accommodation provided by an employer to an employee shall be higher of:
a) 45% of the minimum of the time scale of the basic salary or the basic salary or
b) Amount that would have been paid by employer had no accommodation been provided
Employee includes a director of a company.
Page | 38
Note for students: If "Amount that would have been paid by employer had no accommodation been
provided" is not given in exam than we will compare 45% of basic salary with fair market rental of property
(if it is given).
Minimum of time scale
It is the amount from where the salary scale of an employee starts e.g. (5,000-1,000-7,000) means salary will
starts with Rs. 5,000 with increment of Rs. 1,000 per annum up to maximum of Rs. 7,000.
Where house rent allowance is admissible @30% than in above provision 45% will be replaced with 30%.
(Note for student: It is not general rule. It is applicable for certain Government employees if question specifically refers
to it.)
Example-1
Basic salary of employee is Rs. 100,000/month. Employer has provided employee with a home. Calculate
taxable income?
Solution:
Basic salary (100,000 x 12) 1,200,000
Accommodation provided (45% of 1,200,000) 540,000
Taxable income 1,740,000

Note: As a general rule higher of 45% of Basic Salary or the amount that would have been paid had no
accommodation been provided is added in salary. The later one is not given. Hence, I am only adding 45%
of basic salary.

Example-2
An employees is paid a basic salary of Rs. 90,000 per month. Employer has provided the employee with a
furnished house. The house is not owned by the employer. Fair market rent of the house is Rs. 32,000 per
month.
Solution:
Basic Salary (90,000 x 12) 1,080,000
Accommodation provided: (Higher of)
45% of basic salary (1,080,000 x 45%) 486,000
Or-Fair market rent (32,000 x 12-384,000) 384,000 486,000
1,566,000

Example-3
Mr. Maheesh is an employee of ABC Company. He has provided you with the following details for TY
2011. He joined on August 1, 2010.
His basic salary per month is Rs. 40,000 and education allowance and entertainment allowance of
Rs. 5000/m and Rs.6000/m respectively.
Employer has offered him two options either to take home or take a monthly cash allowance of Rs. 20,000.
Mr. Maheesh has opted for home.
He is allowed to use the electricity, telephone, and water facility in home on nominal amount of Rs. 1,000
per month charged by his employer against all of these facilities. However he pays gas bill himself. The
actual amount of bills during TY 2011.
Electricity 20,000
Telephone 30,000
Gas 10,000
Water 6,000
Calculate his taxable income?

38 | P a g e
Answer-4
Basic salary (40,000 x11) 440,000
Education allowance (5,000 x11) 55,000
Entertainment allowance (6,000 x11) 66,000
Accommodation facility
Higher of: -45% of Basic salary (440,000x45%) 198,000
Or-Amount that would have been paid had no accommodation provided Page | 39
(20,000x11) 220,000 220,000
Utilities excluding gas
Electricity 20,000
Telephone 30,000
Water 6,000
Less: Paid by employee to employer (1,000 x 11) (11,000) 45,000
Taxable income 826,000

Self-hiring of property
If employee or his spouse owns a house which is given on rent to the employer and the employer has provided
the same house to employee as rent-free accommodation, then it will have two-fold effect:
1 Rent actually received by employee from employer will be income from property. (Fair market rent
will be ignored) (Chapter 9)
2 It will be a perquisite in the form of accommodation 45% of basic salary will be added under the head
salary.
Where an employer has provided a perquisite which is not covered above, the "Salary income" shall include
the fair market value of the perquisite at the time it is provided, less amount paid by the employee.
Description Asset is Provided (only for a specified Transferred/sold/ Gifted
period/ means it is returnable to employer)
Car 5% or 10% of cost for purchased vehicle FMV – Consideration Paid
5% or 10% of FMV of vehicle obtained on =Benefit in Salary
lease
House Higher of FMV – Consideration Paid
45% basic salary or =Benefit in Salary
Amount that would have been paid.

Laptop/oven/ Fair market rent of asset for the year less FMV – Consideration Paid
washing machine/ amount paid by employee to employer =Benefit in Salary
Fridge etc. OR
Depreciation charged by employer in his
books less amount paid by employee to
employer
Medical facility/reimbursement or medical allowance

S. Types Tax treatment


No
1 Medical facility/Reimbursement of medical expenses is Exempt (Provided that National
provided as per terms of employment Tax Number of the hospital or
clinic, as the case may be, is
given and the employer also
certifies and attests the medical
or hospital bills)
2. Medical facility/Reimbursement of medical expenses not as Fully taxable.
per terms of employment
3. Only Medical allowance Exempt up to 10% of basic
salary.

39 | P a g e
4. Medical allowance + Medical facility/ Reimbursement of Allowance is Fully taxable
medical expenses in accordance with terms of Facility/Reimbursement will be
Facility/Reimbursement will be exempt. Employment exempt.

5. Medical allowance + Medical facility/ Reimbursement of Allowance is exempt up to 10%


medical expenses not in accordance with terms of of basic salary.
employment Facility/Reimbursement will be Page | 40
fully taxable.

Exam note:
1. The term medical facility (hospitalization) comprises of the following scenarios:
a) medical insurance for hospitalization is borne by employer
b) Reimbursement of medical expense on account of hospitalization is made by employer;
c) Hospitalization is provided by insurance policy;
d) Personal medical expenses of employee are reimbursed by employer; and
e) Free medical treatment is provided by employer to employee.
2. If in income from salary question, both medical allowance and medical facility are given and medical
facility is as per terms, medical allowance will be fully taxable and facility will be exempt.

Question-1
Basic salary of Mr. Z is Rs. 40,000/month. Dearness allowance is Rs. 4,000/ month and house rent
allowance is Rs. 6,000/month. As per the terms of employment he is entitled for free hospitalization
for which employer incurred cost of Rs. 30,000.
Calculate his taxable income?
Answer-1
Basic salary (40,000 x12) 480,000
Dearness allowance (4,000 x 12) 48,000
House rent allowance (6,000 x 12) 72,000
Hospitalization (Exempt being as per terms of employment)
Taxable Income 600,000

Question-2
Basic salary of Mr. AB is Rs. 40,000/month. Dearness allowance is Rs. 4,000/ month and house
rent allowance is Rs. 6,000/month. Medical allowance is 15% of basic salary.
Calculate his taxable income?
Answer-2
Basic salary (40,000 x 12) 480,000
Dearness allowance (4,000 x 12) 48,000
House rent allowance (6,000 x 12) 72,000
Medical allowance (480,000 x 15%) 72,000
Less: Exempt up to 10% of basic salary (480,000 x 10%) (48,000) 24,000
Taxable Income 624,000

Question-3
Basic salary of Mr. IK is Rs. 40,000/month. Dearness allowance is Rs. 4,000/ month and house rent
allowance is Rs. 6,000/month. As per the terms of employment he is entitled for free hospitalization
for which employer incurred cost of Rs. 30,000 which is in addition to medical allowance of
Rs. 3,000/month. Calculate his taxable income?
Answer-3
Basic salary (40,000x12) 480,000
Dearness allowance (4,000x12) 48,000
House rent allowance (6,000x12) 72,000

40 | P a g e
Medical allowance (Fully taxable) (3,000x12) 36,000
Hospitalization (Exempt being as per terms) --
Taxable Income 636,000

Question-4
Basic salary of Miss Inza is Rs.40,000/month. Dearness allowance is Rs. 4,000/ month and house
rent allowance is Rs. 6,000/month. Medical allowance is Rs. 5,000/month. Free hospitalization Page | 41
provided cost Rs. 50,000 which is not as per terms of employment.
Calculate his taxable income?
Answer-4
Basic salary (40,000x12)
480,000
Dearness allowance (4,000×12)
48,000
House rent allowance (6,000x12)
72,000
Hospitalization (Chargeable being not as per terms)
50,000
Medical allowance (5,000×12) 60,000
Less: Exempt up to 10% of basic salary (480,000 x10%) (48,000)
12,000
Taxable Income
662,000

DETERMINATION/COMPUTATION OF OTHER COMPONENTS OF SALARY


Termination of employment / Golden Hand-Shake
An employee who has received an amount on termination of employment, including compensation for
redundancy or loss of employment and golden handshake.by notice to Commissioner, elect for the amount to
be taxed at following rate
A/B%
A is the tax paid by the employee on the employee's taxable income for 3 preceding tax years and
B is the employee's taxable income for 3 preceding tax years.
The above option can be exercised by due date of furnishing return of income. Commissioner may allow a
longer period.
Note: If there is a loss, say in one of the 3 preceding tax years then we will take the average of remaining
years.
Question
Mr. Babar Azam is an employee of PCB Company. He has provided you with the following details for TY
2021. His basic salary is Rs. 40,000/month. Dearness allowance is Rs. 2,000/ month and house rent allowance
is Rs. 20,000/month. He retired on April 30, 2021. Golden Handshake payment received on retirement
amounted to Rs. 950,000
Details of preceding four tax years taxable income and tax liability is as follows. Calculate his taxable income
and tax liability,
Tax Year Taxable income Tax liability
2007 600,000 10,000
2008 1,000,000 48,000
2009 800,0000 24,000
2010 900,000 36,000

41 | P a g e
Treating Golden hand shake receipt in normal way
Income from salary (W-2) 1,570,000
Taxable Income 1,570,000
Tax liability (Table 2) (15,000+12.5% x 370,000) 61,250

Treating Golden hand shake receipt as a separate block


Page | 42
Income from salary 1,570,000
Total income 1,570,000
Less: Golden handshake separate block (950,000)
Taxable income-taxable under NTR 620,000_

Tax Liability
Tax liability on salary income other than golden hand shake (20,000 x 2.5%) 500
Add: Tax on Golden hand shake (950,000 x (W-1) 4%) 38,000
38,500
(Working-1)
Tax Year Taxable Income Tax Liability
2008 1,000,000 48,000
2009 800,000 24,000
2010 900,000 36,000
2,700,000 108,000

Average rate (108,000/2,700,000*100) 4%


(Working-2)
Income from salary
Basic salary (40,000x10) 400,000
Dearness allowance (2,000x10) 20,000
House rent allowance (20,000x10) 200,000
Golden handshake 950,000
1,570,000
Gratuity
Definition: Section 2(3), “approved gratuity fund” means a gratuity fund approved by the
Commissioner in accordance with Part III of the Sixth Schedule;
Gratuity received from approved gratuity fund is fully exempt.
Gratuity received from approved scheme and unapproved fund or scheme is exempt upto the following limits:
Govt. Employees Fully Exempt

Approved Gratuity Fund Fully Exempt

Approved Scheme Exempt up to Rs. 300,000

Un-Approved Gratuity* Amount Received XXX


Less: Lower of
Rs. 75,000
or
50% of amount received (XX)
Add in Salary XXX

42 | P a g e
Exemption in respect of unapproved gratuity shall not apply in the following cases:
(i) Any payment not received in Pakistan
(ii) Any payment received by a director of a company who is not a regular employee of such company
(iii) Any payment received by a non-resident
Page | 43
(iv) Any gratuity received by an employee who has already received any gratuity from the same or other
employer.
PensionDefinition: Section 2(3C), “Approved Pension Fund” means Pension Fund approved by Securities and
Exchange Commission of Pakistan (SECP) under Voluntary Pension System Rules, 2005, and managed by a Pension
Fund Manager registered with the SECP under Voluntary Pension System Rules, 2005;

• Pension received by the citizen of Pakistan from the former employer shall be exempt from tax except
where the person continues to work for the same employer or an associate of the employer. Where a
person receives more than one pension, the exemption shall apply to higher of such pensions.
• For a person over 60 years of age, all such pensions are exempt irrespective of the above mentioned
conditions
• Pension received in respect of services rendered by a member of Armed Forces of Pakistan or Federal
Government or a Provincial Government is exempt from tax.
Provident Fund (PF)
Provident fund is categorized into the following three categories:
(i) Government provident fund
(ii) Recognized provident fund
(iii) Unrecognized provident fund
Provisions regarding taxability in respect of employer/employee contribution, interest credited and
accumulated balance thereon is as follows

Particulars Treatment (Recognized) Unrecognized

Employee’s No Treatment No Treatment


Contribution
Employer’s Employer’s contribution XX No Treatment
Contribution Less: Lower of
Rs. 150,000 (XX)
Or 10% of (B. S+D.A) XXX [Add in Salary]
Interest Credited Interest Credited XXX No Treatment
Less: Higher of
16% of Acc. Balance (XX)
Or 1/3rd of (B. S+D.A) XXX [Add in Salary]
On Withdrawal No Treatment only the employer’s
contribution and
interest on
accumulated balance
is taxable in the year
of receipt

• Salary for the purpose of provident fund includes basic salary + dearness allowance. All other
allowances are excluded.
• There is no treatment of employee contribution as the amount is paid from salary and the same is
already included in his salary.

43 | P a g e
Benevolent fund:
Any benevolent grant paid from a Benevolent Fund to employees or members of their families is exempt.
Other benefits
Certain Perquisites without by virtue of employment
The following perquisites received by an employee shall be exempt: Page | 44
1 Free or subsidized food provided by hotels and restaurants to its employees during duty hours;
2 Free or subsidized education provided by an educational institution to the children of employees;
3 Free or subsidized medical treatment provided by a hospital/clinic to its employees;
And other benefit for which the employer do not bear any marginal cost, as notified by Board
Question
Mr. Waseem is teaching in SBI College. His basic salary is Rs. 40,000/month. Dearness allowance is Rs.
2,000/ month and house rent allowance is Rs. 6,000/month. His child is also getting education in same school.
College charges a concessional fee of Rs. 40,000 per annum against the market charge of Rs. 65,000.
Calculate income.
Answer-1
Basic salary (40,000x12) 480,000
Dearness allowance (2,000x12) 24,000
House rent allowance (6,000x12) 72,000
Taxable Income 576,000
Subsidized education provided by the college is exempt under clause 53A of Part-1 Second Schedule.
Leave Encashment
Any amount received on encashment of leave preparatory to retirement is exempt, if received by a
Government employee& member of Armed Forces of Pakistan

Worker's Profit Participation Fund


It is fully exempt.
Foreign source salary of resident individuals [Already discussed in Chapter 1]
(1) Any foreign-source salary received by a resident individual shall be exempt if the individual has paid
foreign income tax on salary.
(2) It will be assumed that resident individual has paid foreign income tax if tax is withheld from salary by the
employer and paid to the revenue authority of the foreign country.
Where a citizen of Pakistan leaves Pakistan during a tax year and remains abroad during a tax year than salary
income earned outside Pakistan during that year shall be exempt from tax.
Note: In such case even if no tax is paid the foreign source salary income will remain exempt.

Example
Miss Hania Amir a citizen of Pakistan is employed in XYZ Company. She worked in XYZ Company from
July 1, 2010 to January 31, 2011. She got a job in Saudi Arabia where she joined on March 1, 2011. She
departed from Pakistan on February 20, 2011. her monthly salary in Pakistan is Rs. 50,000 per month. Her
monthly salary in Saudi Arabia is Rs. 80,000 in Pak rupees. As on June 30th she was still in Saudi Arabia. she
did not paid any tax on his salary in Saudi Arabia. Calculate his taxable income?
Answer-1
Salary income (PSI) (50,000x7) 350,000
Salary income (FSI) Exempt
Taxable Income 350,000
Because hania remained outside Pakistan at the end of tax year, therefore her foreign source salary income is
exempt from tax irrespective of whether she has paid foreign income tax or not.

44 | P a g e
Example
Mr. A a citizen of Pakistan is employed in XYZ Company. He worked in XYZ Company from July 1, 2010 to
January 31, 2011. He got a job in Saudi Arabia where he joined on March 1, 2011. He departed from Pakistan
on February 20, 2011. His monthly salary in Pakistan is Rs. 50,000 per month. His monthly salary in Saudi
Arabia is Rs. 80,000 in Pak rupees. On May 31, 2011 he decided to resign from the Saudi job and came back
in Pakistan on the same date. Since then he has yet not got any job. He did not paid any tax on his salary in
Page | 45
Saudi Arabia as there is no tax on salaries there.
Calculate his taxable income for TY 2011?
Answer-2
Salary income (PSI) (50,000x7) 350,000
Salary income (FSI) (80,000x3) 240,000
Taxable Income 590,000

Because A did not remain outside Pakistan at the end of tax year, therefore his foreign source salary income is
chargeable to tax

Exercise
Being a tax consultant, you are required to explain the tax implications/taxable income under the
appropriate head in respect of each of the following independent situations:
(i) As part of remuneration package, a company provides for reimbursement of telephone costs on actual
basis to its employees.
(ii) Actual expenditure incurred by an employee in relation to travelling and daily allowances is less than
the amount of allowances paid by the employer.
(iii) Mr. Hamid, a citizen of Pakistan was working with Zee (Pvt.) Ltd for last 15 years when he opted for
early retirement on 31 October 2023. He was due Rs. 5 million as a gratuity under the gratuity scheme of
Zee (Pvt.) Limited. The scheme was not approved by the FBR. Due to cash constraints, the gratuity though
due to Hamid on 31 October 2023 was not paid to Hamid. 0n 30 April 2024 at the request of Zee (Pvt.)
Limited, Kee (Pvt.) Ltd- an associated company of Zee (Pvt.) Ltd transferred the equivalent of Rs. 5 million
in US Dollars into Hamid's US dollar account in UAE in lieu of gratuity due from Zee (Pvt.) Limited.
(iv) A company has taken health insurance cover for its employees. The insurance company reimburses
employees for actual cost of medical services for themselves and their dependents.
(v) ABC Ltd has provided scholarship to one of his employees for higher studies abroad.
(vi) Mr. A has leased a car and pays for its lease rentals from his own sources. He uses the car for business
purpose. What will be the treatment of lease rentals paid and expenditure incurred on vehicle running
and maintenance?
(vii) A partner in a firm is entitled to a fixed remuneration each month. Would this constitute his salary
income?
(viii) Mr. Azhar is 65 years old and his taxable salary for the tax year is Rs. 943,000. Mr. Azhar has obtained
a housing loan from a local bank. How the tax reduction for senior citizenship and deductible allowance
for mark-up paid on loan will be calculated.
(ix) Mr. Aslam is 67 years old and employed as research scholar in a recognized non-profit institution. His
taxable salary for the tax year is Rs. 654,000. Azhar is of the view that he is entitled to both reductions i.e.
in respect of senior citizen allowance as well as for full time teacher allowance.
(x) Mr. Sarmad has purchased a generator amounting to Rs. 1,000,000 from an interest free loan taken
from his employer. He rented the generator at an annual rental value of Rs. 250,000. Total expense of Rs.
25,000 was expanded on repair, transport and maintenance of the generator

Answer
(i) Reimbursements of telephone expenses by the company will be treated as taxable benefits of
employees in case the facility is used for private purposes. There will be no tax
consequences to the extent the facility is used for official purpose.

45 | P a g e
(ii) Travelling and daily allowance spent to the extent for performance of duty will be exempt provided it is
not paid with monthly salary or on fixed basis. Any amount given in excess of actual spending will be fully
taxable.
(iii) Since gratuity scheme is not approved, amount exempt from tax should be 50% of the amount
received or Rs. 75,000, whichever is less. However, since the payment is received outside Pakistan, the
said exemption is not available. The whole amount is chargeable to tax.
Page | 46
(iv) Reimbursement of actual medical expenditure by an employer is tax exempt. On a similar basis, there
will be no tax implications on reimbursement by the insurance company on behalf of the employer, or any
tax consequences for the employees on payment of health insurance premium by the employer.
(v) Scholarship granted to the employee will be exempt from tax provided the employer and the
employee are not associates
(vi) Expenditure incurred by an employee to earn salary income including travelling expenses and lease
rental payments is not tax deductible.
(vii) The remuneration paid by a firm to a partner is considered his share in the firm’s profit as partner is
not an employee of the firm.
(viii) Mr. Azhar is entitled to deductible allowance relating to housing loan has already allowed against his
total income. Further the tax reduction for senior citizen is no more available.

Tax free salary to employee


Where an employer agrees to pay tax on employee's salary, the employee's salary income shall be grossed up
by the tax payable by employer.

For Alternative Step Scan the QR Code and learn another Method

Question-1
Mr. A provided following:
Basic Salary 340,000/m
House Rent allowance 130,000/m
Tax on salary is to be borne by employer.

Answer:
Income from Salary
Basic Salary (340,000x12) 4,080,000
House Rent allowance (130,000x12) 1,560,000
5,640,000
Add: Tax Benefit (W) 1,430,160
Salary Income 7,070,160

Tax liability (Table 2) (1,095,000+35% of 1,070,160) 1,469,556


Less: Tax already paid by the employer (1,430,160)
Tax payable with return *39,396__
*(Ideally it should be Rs. 0 but we have stopped calculation at Step 3 so it is not zero.)
Working:
Step 1 Salary Income 5,640,000
Tax on above (Table 2) (435,000+27.5% of 2,040,000) 996,000
Step 2 Salary Income (5,640,000+996,000) 6,636,000
Tax on above (Table 2) (1,095,000 +35% of 636,000) 1,317,600
Step 3 Salary Income (5,640,000+1,317,600) 6,957,000

46 | P a g e
Tax on above (Table 2) (1,095,000 + 35% of 957,600) 1,430,160

Question-2
Mr. A provided following:
Income from salary
Basic Salary 340,000/m
Page | 47
House Rent allowance 130,000/m
Capital Gains 600,000
Answer:
Income from Salary
Basic Salary (340,000x12) 4,080,000
House Rent allowance (130,000x12) 1,560,000
5,640,000
Add: Tax Benefit (W) 1,430,160
Salary Income 7,070,160
Income from Capital Gains __600,000
Taxable Income 7,670,160
Tax liability (Table 2) (1,095,000+35% of 1,670,160) 1,679,556
Less: Tax already paid by the employer (1,430,160)
Tax payable with return 249,396__

Working:
Step 1 Salary Income 5,640,000
Tax on above (Table 2) (435,000+27.5% of 2,040,000) 996,000
Step 2 Salary Income (5,640,000+996,000) 6,636,000
Tax on above (Table 2) (1,095,000 +35% of 636,000) 1,317,600
Step 3 Salary Income (5,640,000+1,317,600) 6,957,000
Tax on above (Table 2) (1,095,000 + 35% of 957,600) 1,430,160

Question-3
Mr. B provided following:
Income from salary
Basic Salary 40,000/m
House Rent allowance 30,000/m
Tax of Rs. 500 will be borne by employer.
Answer:
Income from Salary
Basic Salary (40,000x12) 480,000
House Rent allowance (30,000x12) 360,000
840,000
Add: Tax Benefit (W) 500_
Taxable Income 840,500

Tax liability (Table 2) (2.5% of 240,500) 6,013


Less: Tax already paid by the employer (500)
Tax payable with return 5,513

Question-4
Mr. A provided following:
Income from salary
Basic Salary 90,000/m

47 | P a g e
House Rent allowance 60,000/m
Bonus Paid 200,000
Employer will himself pay tax on employer’s Salary.
Answer:
Income from Salary
Basic Salary (90,000x12) 1,080,000
Page | 48
House Rent allowance (60,000x12) 720,000
Bonus 200,000
2,000,000
Add: Tax Benefit (W) __131,172
Taxable Income 2,131,172

Tax liability (Table 2) (15,000 + 12.5% x 931,172) 131,397


Less: Tax already paid by the employer (131,172)
Tax payable with return 225
Working:
Step 1 Salary Income 2,000,000
Tax on above (Table 2) (15,000 + 12.5% x 800,000) 115,000
Step 2 Salary Income (2,000,000+115,000) 2,115,000
Tax on above (Table 2) (15,000 + 12.5% x 915,000) 129,375
Step 3 Salary Income (2,000,000 + 129,375) 2,129,375
Tax on above (Table 2) (15,000 + 12.5% x 929,375) 131,172

No deduction shall be allowed for an expense paid by an employee under the head "Salary"

Employee Share Schemes


"Employee share scheme" means any agreement under which a company may issue shares in the company to:
(a) An employee of the company or an employee of an associated company; or
(b) The trustee of a trust and the trustee may transfer the shares to an employee of the company or an
employee of an associated company.

Basic understanding of operation of employee share scheme

Terminologies
I. Right/Option Simply, A letter stating that employee has a right to acquire shares at
a particular rate.
II. Exercise of right Person with the right letter purchases the shares at the rate
mentioned in the right letter. He then becomes the owner of shares.
III. Face value of a share The denomination of the share. Generally, it is Rs. 10 per share.

IV. Market value of a share Price at which the share is traded in the stock market. This may be less
than, equal to or more than face value.

(b) Following events take place in employee shares scheme


Announcement of scheme Issuance of Right letter Exercise of right or Issuance of shares by the
company.
Value of right/option (Taxability at time of grant of right or option)
The value of a right or option to acquire shares under an employee share scheme is not chargeable to tax.
Example
Mr. R bought a right for Rs. 3,000 on 01-April-2010. On 30-06-2010 fair value of the right is

48 | P a g e
Rs. 4,200. On 25-07-2010 he sold it for Rs. 4,400.
Answer
Tax year 2010 = No gain or loss will be recorded because value of a right is not chargeable to tax.
Tax year 2011 = Income from salary = Gain=A-B=4,400-3,000 1,400 [S. 14 (5)]

Disposal of right or option/ Sale of right or option


Page | 49
Where in a tax year, an employee disposes off a right or options to acquire shares under an employee share
scheme, the amount chargeable under the head "Salary" shall include the amount of any gain as calculated
below:-
A-B
Where -
A is the consideration received for the disposal of the right or option; and

B is the employee's cost in respect of the right or option.

Formula for disposal of right


Consideration received for the disposal of the right or option XXX
Less: Cost of right or option (xxx)
XXX
Exercise of right/Acquisition of shares/Issue of shares by company
Where shares are issued to an employee under an employee share scheme (including as a result of the
Exercise of an option), following amount is chargeable:
1 The fair market value of the shares at the date of issue less consideration given by the employee for
the shares and for rights or options.

Formula for exercise of right


Fair market value of the shares at the date of issue XXX
Less: Cost of share (xxx)
Less: Cost of right (xxx)
XXX
Shares issued by company with restriction on transfer
Where shares are issued to an employee under an employee share scheme and there is a restriction on the
transfer nothing will be added in the income until the employee has a free right to transfer the shares. On
getting the free right to transfer, the salary shall include the fair market value of the shares at the time the
employee has a free right to transfer the shares less consideration given by the employee for the shares and for
rights or options.
However if the employee disposes off the shares before getting the free right to transfer the shares, the salary
shall include the fair market value of the shares at the time the employee disposes of the shares less
consideration given by the employee for the shares and for rights or options.

Calculation of cost of shares for computing capital gain at the time of disposal
The cost of the shares shall be sum of-
(a) The consideration given by the employee for the shares;
(b) The consideration given by the employee for the grant of right or option; and
(c) The amount chargeable to tax under the head "Salary".
[For details refer examples of Section 14(4) in Chapter 10]

Note
1. In case of disposal of shares of private company, gain will be chargeable under the head capital gain.
2. In case of disposal of shares of public company, gain will be taxed as separate block because shares of
public company fall under the definition of securities

49 | P a g e
Salary Summary Sheet
Sr. Taxable
No Particulars Rs. Rs
Basic Salary (Salary per month × Number of months employed during the tax
1
year) X
2 Bonus X Page | 50
3 Commission X
4 Fees and Rewards X
5 Overtime Payment received from employer X
6 Amount received from employer for meeting any condition X
7 Amount received from employer as a condition for joining his organization X
8 Director's fee X
9 Fee's for attending BOD meetings X
10 Any allowances provided by employer I cash ( other than Conveyance, House rent and
medical allowance)
• Dearness Allowance
• Milk allowance

Totally Taxable
• Cost of living allowance
• Lunch allowance
• Education allowance
• Utilities allowance
• Inducement allowance
• Entertainment allowance
Amount received as work condition supplements due to unpleasant or dangerous
11
working conditions X
12 Special merit award X
13 Leave fare assistant provided by employer (OR) Air ticket reimbursed X
14 Reimbursement of personal expenses of employee provided by employer
• Utility bills of employee reimbursed by employer
• Internet usage of employee reimbursed by employer

Taxable
Totally
• Children education expenses and school fee reimbursement by employer
• Funeral expenses of employee’s parents paid by employer
• Petrol provided by employer for residential generator of employee
15 Reimbursement of expenses (Allowance received) to meet official duties
• TA/DA ( Special allowances ) received from employer
Exempt
Totally

• Conveyance allowance received to meet official duties


• Free pick and drop facilities from home to office and vice versa
• Training expense of employee reimbursed by employer
16 Obligation or liability of employee towards third party paid by employer
• Life Insurance premium of employee paid by employer
• Annual subscription of club joined by employee paid by the employer
Taxable
Totally

• Vehicle obtained by employee on rent a car basis but rent of vehicle is paid by
the employer
• School fee paid by employer
17 Obligation or liability of employee towards employer is waived by the employer
• Loan waived by employer X
18 Services of domestic servants (Housekeeper, gardener, driver) provided by employer
Salary of domestic servants borne and paid by employer X

50 | P a g e
Less: Amount given by employee to employer for domestic servants (if any) (X) X
Assets (TV, Car, House, Fridge, Furniture, Washing machine) owned by employer but
19
transferred/sold to employee
FMV of asset at the date of transfer or sale to employee X
Less: Amount paid by employee for purchase of asset from employer (X) X
Assets (TV, Fridge, Furniture, Washing machine) owned by employer but given to Page | 51
20
employee for use only
Depreciation of the assets charged by the employer X
Accommodation
21 House rent allowance X
22 Accommodation provided by the employer (Higher of )
a) Annual letting Value
b) 45%( or 30%) of basic salary or MTS X X
Conveyance
23 Conveyance or travelling allowance X
24 Motor Vehicle Provided by employer only for official use
25 Motor Vehicle Provided by employer only for personal use
• Owned vehicle given by the employer : 10% of the cost of the vehicle to the
employer
(Or)
• Leases vehicle given by the employer : 10% of the FMV of the vehicle at the
inception of the lease X
26 Motor Vehicle Provided by employer for official and private use
• Owned vehicle given by the employer : 5% of the cost of the vehicle to the
employer
(Or)
• Leases vehicle given by the employer : 5% of the FMV of the vehicle at the
inception of the lease X
Guidelines for conveyance allowance:
• Taxable benefits of conveyance will be time apportioned if the vehicle isn’t
provided for the complete year
• If any expenditure paid by the employee connected with the conveyance shall
be deducted from the allowance provided

Medical Allowance and Medical facility


27 Medical Allowance only
Amount received X
Less: 10% of the basic salary (X) X
Medical facility ( free hospitalization or medical expenses reimbursement) provided
28
by the employer only
• As per terms of contract provided with the NTN number of the practitioner
(Fully Exempt)
• Not as per the terms of employment ( Totally Taxable) X
29 Medical allowance and medical facility both are provided by the employer
• Medical facility per terms of contract provided with the NTN number of the
practitioner (Fully Exempt)
• Medical allowance (Fully taxable )
(Or)

51 | P a g e
• Medical facility not as per the terms of employment ( Fully Taxable) X
• Medical allowance received X
Less : 10% of basic Salary (X) X
Concessional Loan from Employer
30 Interest on Loan @ 10% Benchmark Rate X
Less Actual amount of interest paid to the employer (X) X Page | 52
Condition for loan:
• The loan must be obtained after the 1st July 2002
• The amount of loan must be more than R. 1,000,000
• The Rate at which the interest is obtained must be less than the benchmark
rate
• Where the loan benefit is extended due to waiver of interest by such employee
then the loan will be exempt
Note:
• If the loan obtained from employer is used in the purchase of asset or property
or business for which the income is chargeable under the head of income then
the actual amount of interest or interest @ 10% whichever is higher shall be
allowed as deduction
• If the loan obtained from the employer is used in the purchase of property or
construction of new house then the actual amount of interest paid to the
employer shall be allowed as deductible allowance under section 64 A.
Provident Fund
31 Government Provident fund ---
32 Unrecognized provident fund
• Employee’s contribution ( No treatment) X ---
• Employer’s contribution ( Fully Exempt ) X ---
• Interest Credited ( Fully Exempt ) X ---
• Receipt of accumulated balance ( Only employer’s contribution and interest
credited shall be taxable ) X X
33 Recognized provident fund
• Employee’s contribution ( No treatment) X ---
• Employer’s Contribution X

Less : Exempt up to lower of


i. Rs. 150,000 (or)
ii. 1/10 of ( Basic salary + Dearness allowance ) (X) X
• Interest on recognized provident fund credited X
Less : Exempt up to higher of
i. 1/3rd of ( Basic salary + Dearness allowance ) or
ii. Interest credited @ 16% (X) X
• Receipt of accumulated amount under the recognized provident fund ( Fully
Exempt ) X ---
Gratuity or commutation of pension
34 Gratuity received by Govt. employees (Fully Exempt) X ---
Gratuity received by non-government employees from gratuity fund approved by the
35
commissioner ( Fully Exempt ) X ---
Gratuity received by non-government employees from gratuity fund approved by the X
36 FBR
Less : Exempt up to Rs 300,000 (300,000) X
Gratuity received by non-government employees from unapproved gratuity fund X
37
Less Exempt up to lower of X

52 | P a g e
1. Rs. 75,000 or
2. 50% of the amount received (X)

Important guideline :
Unapproved gratuity or unapproved commutation of pension shall not be exempt in
the following cases:
a. Amount is not received in Pakistan Page | 53
b. Amount received by the director of the company who is not regular employee
of such company
c. Amount received by the non-resident person
d. Amount received by an employee who has already received an gratuity from
the same or any other employer.
Pension
38 Pension received by the citizen of Pakistan from the former employee (Fully Exempt) X ---
Important guideline:
Pension received shall be taxable if :
a. A person receive a pension works for same employer or his associate after
retirement.
b. A person receives more than one pension then only one higher amount will be
exempt and remaining number of pensions will be taxable
The both conditions didn’t apply if the person is more than 60 years old.
Employee Share Scheme
39 When option ( right to acquire shares ) is obtained – Not a taxable event ---

40 When option ( right to acquire shares ) is sold / disposed off


Consideration received on the disposal of option X
Less : Incidental expenses incurred to acquire the option (X) X

41 When shares are acquired (without any restrictions )


FMV at the date of issue of the shares X
Less: Cost paid to acquire the shares (X)
Less : Cost paid to acquire the option (X) X

When shares are acquired (with restriction on transfer) and employee retains
42
the shares till the uplifting of the restriction
FMV at the date when employee have the free right to transfer X
Less: Cost paid to acquire the shares (X)
Less : Cost paid to acquire the option (X) X

When shares are acquired (with restriction on transfer) and employee disposes
43
the shares before the uplifting of the restriction
FMV at the date when employee actually disposes of the shares X
Less: Cost paid to acquire the shares (X)
Less : Cost paid to acquire the option (X) X

44 When shares are disposed after getting the free right of disposal (C. Gain)
Consideration received on the disposal of shares X
Less: Cost paid to acquire the shares (X)
Less : Cost paid to acquire the option (X)

53 | P a g e
Less : Incidental expenses paid to disposes of the shares (X)
Less : Amount already taxed under the salary as a benefit received under the ESS (X)
Total Taxable Salary XXX

PAST PAPER QUESTIONS Page | 54


Question # 1 Spring 2015 Q. 3

Munir resigned from his employment with Ali Industries Limited (AIL) with effect from 31 December 2014. He
received following amounts in final settlement:
▪ Rs. 150,000 as Leave Encashment.
▪ Rs. 4,000,000 under a Golden Handshake Scheme.

Munir had received a salary of Rs. 350,000 per month for a period of six months upto December 2014. His
taxable income and tax liability during the preceding five tax years were as under:

Tax year 2010 2011 2012 2013 2014


Total taxable income (Rs) 2,000,000 2,450,000 2,700,000 3,100,000 3,650,000
Total tax paid (Rs) 300,000 392,000 472,500 542,500 650,000

Required:
As a tax consultant, advise Munir about the amount of income tax payable by him for the tax year 2015, under
the Income Tax Ordinance, 2001. (Tax rates are given on the last page) (06)

Question # 2 Autumn 2017 Q. 3(a)

Under the provisions of the Income Tax Ordinance, 2001 compute taxable income or loss, under the correct
head of income for tax year 2017, in each of the following cases:
(a) Under an employee share scheme, 30,000 shares of Dawood Limited were issued to Qamar, on 1 August
2013 for Rs. 30 each. According to the scheme, he was not allowed to sell/transfer the shares before
completion of three years from the date of issue. The face value of each share is Rs. 10 per share. Fair
market value of each share on different dates was as follows:

1 August 2013 30 June 2016 31 July 2016


Rs. 40 Rs. 30 Rs. 50

He sold 10,000 shares on 31 May 2017 for Rs. 65 per share. (04)

Question # 3 Spring 2018 Q. 2(c)

Hasrat has been working as Director HR in Shakir Limited (SL) for many years. During the tax year 20X8 he
received basic salary of Rs. 6 million. SL also contributed Rs. 50,000 per month towards a recognized provident
fund. An equal amount was contributed by Hasrat. Interest income of Rs. 3,391,000 at the rate of 20% of
accumulated balance of the fund was credited to Hasrat’s account. (04)

Question # 4 Spring 2020 Q. 5(b)

Sajid retired from Sun Chemicals Limited (SCL) as a marketing manager with effect from 31 December 2019.
He received the following amounts in final settlement from SCL:
(i) Leave encashment of Rs. 600,000.

54 | P a g e
(ii) Rs. 4,000,000 from unapproved provident fund. 50% of this amount was contributed by Sajid.
(iii) Un-approved gratuity of Rs. 2,500,000.

He also acquired the vehicle, provided to him by SCL, at accounting written down value of Rs. 500,000. The
market value of the vehicle at the time of retirement was Rs. 2,000,000.
Page | 55
Required:
Under the Income Tax Ordinance, 2001 and Rules made thereunder, discuss the tax treatment of the above
benefits received by Sajid on retirement. (04)

Question # 5 Spring 2021 Q. 1(a)

On 31 December 20X1, Dr. Jamal resigned from his employment with General Hospital Limited. In January
20X2, he received following amounts in final settlement:
▪ Rs. 600,000 as leave encashment.
▪ Rs. 8,510,000 from recognised provident fund.
▪ Rs. 1,300,000 and Rs. 1,700,000 as salary arrears relating to tax year 20W9 and 20X0 respectively.

Dr. Jamal had received a monthly salary of Rs. 500,000 from July 20X1 to December 20X1. His taxable income
and tax liability during the preceding four tax years were as under:

Tax year 20W8 20W9 20X0 20X1


Total taxable income (Rs.) 2,800,000 3,200,000 3,800,000 4,800,000
Total tax paid 359,500 404,500 300,000 630,000

Required:
As a tax consultant, advise Dr. Jamal about the amount of income tax payable by him for the tax year 20X2,
under the Income Tax Ordinance, 2001. (07)

Question # 6 Spring 2021 Q. 4(b)(i)

Briefly explain the provisions of the Income Tax Ordinance, 2001 and Rules made thereunder relating to:
(i) interest free loan provided by an employer to its employee for marriage of his/her daughter. (02)

55 | P a g e
ANSWERS

Answer # 1 Spring 2015 Q. 3

Option-1 Treating GHS in NTR


Page | 56
Basic Salary (350,000 x 6) 2,100,000
Leave encashment 150,000
GHS 4,000,000
6,250,000
Tax liability (1,005,000 + 32.5 % x 250,000) 1,086,250

Option-2 Treating GHS in as Separate Block

Basic Salary (350,000 x 6) 2,100,000


Leave encashment 150,000
Termination benefit 4,000,000
Total Income 6,250,000
Termination Benefit (4,000,000)
Taxable Income 2,250,000
Tax (15,000 + 12.5% x 1,050,000) 146,250
Add: tax on GHS (4,000,000 x 6.22%w-1 248,800
Total Tax Liability 395,050

W-1 (185,000 + 130,000 + 273,000) / (3,650,000 + 3,100,000 + 2,700,000) = 6.22%

Examiner Comments:
According to the scenario given in this question, an individual had received three types of
payments from the employer i.e. normal monthly salary, leave encashment and amount of
golden hand shake. In such situations, the tax payer has the option to either treat all the
payments as income in the year of receipt or treat salary and leave encashment as
current year's income and have the amount of golden handshake taxed at the average
rate of tax of the preceding three years. The candidates were supposed to evaluate both
options and advise the taxpayer about the better option.
The performance in this question was quite good. However, the major issue was that a
significant number of candidates restricted their answers to the computation of tax
liability under the latter option and did not give any advice. Following mistakes were also
observed in the computations:
▪ Rate of tax for non-salaried individuals were applied.
▪ Average rate of tax was calculated on the basis of last five tax years instead of last three
tax years.
▪ Leave encashment was considered as exempt from tax.
▪ Like Golden Handshake payment, leave encashment was also taxed by applying the
preceding three years average rate.
▪ Some students were so confused that they added the tax liability calculated at the
average rate, to the other salary income.

Answer # 2 Autumn 2017 Q. 3(a)

Taxable Income

56 | P a g e
Share of DL - Employees share scheme Rupees
Fail' market value of shares on 31 July 2016 (30,000x 50) 1,500,000
Purchase price (30000x30) 900,000
Income from salary 600,000

Share of DL - Employees share scheme Rupees


Page | 57
Sale proceeds of shares on 31 May 2017 (10,000x65) 650,000
Purchase price (10,000x50) 500,000
Capital Gain 150,000

Examiner Comments:
This question carrying 11 marks contained four parts. Each part contained an independent
situation and the candidates were required to determine the taxable income or loss under
correct head of income. The overall performance remained good as about 51% of the
candidates secured passing marks. Comments on each part are given below:

This part was based on the requirements of section 14(3)(b) of the Income Tax Ordinance,
2001, and candidates were required to calculate gain on the sale of shares and income on
shares allotted under an employee share scheme with a restriction on sale/transfer of
shares before completion of three years from date of issue. Some of the common mistakes
were:
▪ Majority of the candidates were not aware that on completion of the period of
restriction, capital gain would be recorded based on the fair market value on the date
the restriction period was completed and the amount paid to purchase the shares. Many
candidates who computed the gain correctly booked it as capital gain instead of salary
income.
▪ The amount of gain on sale of 10,000 shares was reduced by 25%.
▪ Cost of sale of 10,000 shares was calculated on the basis of Rs. 30 per share instead of
fair market value of shares on completion of the period of restriction i.e. Rs. 50 per share.

Marking Scheme:
▪ Determination of the amount of gain under employee share scheme that should be taken
to ‘Income from salary’ 2.0
▪ Determination of amount of gain on disposal of shares that should be taken to ‘Income
from capital gain’ 2.0

Answer # 3 Spring 2018 Q. 2(c)

Head of income: Income from salary


-----------Rupees-----------
Salary 6,000,000
Provident fund contribution by SL (50,000x 12) 600,000
Tax exemption up to 150,000 or 10% of salary (150,000) 450,000

Interest credited to provident fund


Amount credited 3,391,000
Exempt higher of:
- 2,712,800 (3,391,000x16%-20%); or
- l/3rd of salary i.e. 2,000,000 (6,000,000-?3) 2,712,800 678,200
Taxable income of Hasrat 7,128,000
Tax Liability (1,005,000 + (1,128,200 x 32.5%) 1,371,665

57 | P a g e
Examiner Comments:
The overall performance was below average as only 29% of the candidates secured passing
marks. Part-wise performance is discussed below:

This part of the question tested the taxability of provident fund contribution and interest
Page | 58
earned thereon. Most of the students knew about the taxability of the amount of
contribution but did not know about the taxability of the interest on the provident fund
balance. However, some of the students were of the view that the entire amount of
contribution was exempt whereas many students considered the exemption limit as Rs.
100,000 instead of Rs. 150,000. Many students wrote correctly that 16% of the balance or
1/3rd of the salary, whichever is higher, would be considered as exempt but were unable to
correctly calculate the amount representing 16% of the balance. Further, many students
considered it as taxable under the head ‘income from other sources’ instead of salary
income.

Marking Scheme:
▪ Identification of correct head of income 1.0
▪ Computation of taxable income or loss 3.0

Answer # 4 Spring 2020 Q. 5(b)

The benefit received by Sajid on his retirement would be treated as follows:


(i) Leave encashment comes under the definition of salary and therefore it would be fully taxable.
(ii) Since the amount was received from unapproved PF, the employer’s contribution and interest on
accumulated balance would be taxable in the year of receipt.
(iii) In the case of unapproved gratuity, exemption is available up to Rs. 75,000or 50% of the amount
receivable whichever is lower. Therefore, the amount to be included in Sajid’s taxable income would be
Rs. 2,425,000 (2,500,000‒75,000).
(iv) Since the market value of the vehicle was more than cost of acquisition the difference i.e. 1,500,000
would be included in his taxable income.

Examiner Comments:
▪ Examinees failed to appreciate that leave encashment was chargeable to tax under the
head ‘Salary’.
▪ The exemption limit of unapproved gratuity was wrongly considered to be Rs 150,000 .
▪ Examinees considered fair market value of the vehicle as fully taxable.

Marking Scheme:
01 mark for explaining the tax treatment of each benefit received by Sajid 4.0

Answer # 5 Spring 2021 Q. 1(a)

Following options are available to Jamal (Salaried individual):


Option 1: By applying applicable tax rate to total taxable income
Rupees
Salary (500,000x6) 3,000,000
Leave encashment 600,000

58 | P a g e
Arrear for 20W9 & 20X0 3,000,000
Total taxable income 6,600,000

Tax computations:
On Rs. 6,000,000 1,005,000
On balance [(6,600,000 - 6,000,000) x 32.5%] 195,000 Page | 59
Tax liability under option 1 1,200,000

Option 2
Rupees
Salary (500,000x6) 3,000,000
Leave encashment 600,000
3,600,000
Tax computation - Salary
On (Rs. 3,600,000) 405,000

405,000
Tax on amount received in arrear for the tax year 20W9
Received in 20W9 3,200,000
Received in 20X2 1,300,000
4,500,000

Tax on Rs. 3,600,000 405,000


On balance Rs. 900,000 @ 25% 225,000
630,000
Already paid (404,500)
225,500
Tax on amount received in arrear for the tax year 20X0
Received in 20X0 3,800,000
Received in 20X2 1,700,000
5,500,000

Tax on Rs. 3,600,000 405,000

On balance Rs. 1,900,000 @ 25% 475,000


880,000
Already paid (300,000)
580,000

If use accrual basis (405,000 + 880,000 + 630,000) 1,915,000


If use receipt basis (1,200,000 + 300,000 + 404,500) 1,904,500
Tax saving (1,915,000-1,904,500) 10,500

Note: Since Provident fund is recognized, it is fully exempt.


Conclusion: Jamal should select option 1 as it would result in tax saving of Rs. 8,000 (1,038,000 – 1,030,000).

Examiner Comments:
▪ Examines did not know that Jamal has two options for computation of tax on salary
received in arrears.
▪ Examinees calculated tax liability under option 1 only i.e. apply the current year tax rate

59 | P a g e
on the entire amount of salary received during the year. However, they did not consider
option 2 i.e. apply the rate that would have been applicable if the amount had been paid
to the employees in the tax year in which the services were rendered.
▪ Leave encashment was also charged to tax at average tax rate for preceding three tax
years.
▪ Provident fund was claimed to be exempted at lower of 1/10 of basic salary or 150,000
Page | 60
Marking Scheme:
▪ Computation of tax payable for salary (current year) and arrears if:
- applicable tax rate applies to total income 2.5
- tax rate applies that would have applicable in case tax was paid in the year of services
3.5
▪ Conclusion 1.0

Answer # 6 Spring 2021 Q. 4(b)(i)

• Where a loan is given to an employee then the amount will be included in salary income of the employee
in the following manner:
- If no interest is payable by the employee - the amount of interest computed at the benchmark rate
(i.e. 10%).
- If interest is payable at less than benchmark rate - the interest amount computed at the benchmark
rate less the actual amount of interest paid by the employee.
• The interest for loan amount would be chargeable to tax only for amount exceeding Rs. 1,000,000.
• If interest free loan is extended by the employer due to waiver of interest by such employee on his
accounts maintained with the employer (e.g. Provident Fund), no amount of interest would be charged.

Examiner Comments:
▪ Examinees failed to appreciate that interest free loan would be chargeable to tax only
when the loan amount exceeds Rs 1,000,000.
▪ Examinees failed to mention that no interest would be charged where interest free loan
is extended by the employer due to waiver of interest by the employee on his account
maintained with the employer.

Marking Scheme:
Discussion on provisions relating to a loan provided to an employee 2.0

60 | P a g e
SALARY EXTRACTED NUMERICAL FOR PRACTICE
Question # 1

Mr. Zulfiqar, a senior executive of Mirza Petroleum Ltd (MPL), retired on 31" March after completion of
19 years of dedicated service. Page | 61

The details of Mr. Zulfiqar's income for the year ended 30.6.20X8 are given below:
Income from MPL
Rupees
Basic salary 280,000
Medical allowance 45,000
Utilities allowance 45,000
Cost of living allowance 25,000
Total monthly salary 395,000

Market value of rent free accommodation provided 120,000

(ii) As per terms of employment, tax liability of Mr. Zulfiqar to the extent of Rs. 200,000 is to be borne by
MPL.
(iii) On his retirement, he received gratuity of Rs. 2,660,000 from an unapproved gratuity fund
maintained by MPL.
(iv) He is receiving pension amounting to Rs.50,000 per month from the date of his retirement.
Required:
Compute taxable income and tax liability of Mr. Zulfiqar.

Question # 2

Mr. Zameer Ansari is working as a Chief Executive Officer in Wimpy (Private) Limited (WPL). Following
are the details of his income / receipts during the tax year 2023:

(a) His monthly cash remuneration in WPL is as follows:


Rupees
Basic salary 200,000
Medical allowance 30,000
Utilities allowance 10,000

(b) In addition to the above, he was also provided the following benefits in accordance with his terms of
employment:
(i) Medical insurance for hospitalization and surgery, limited to Rs. 1,500,000 per annum.
(ii) Payment of his children’s school fees of Rs. 15,000 per month. The fee is deposited directly into
the school’s bank account.
(iii) Rent free furnished accommodation on 1000 square yards. The accommodation is located
within the municipal limits of Karachi.
(iv) Two company-maintained cars. One of the cars was purchased by WPL for Rs. 3,000,000 and is
exclusively for his business use. The second car was obtained on lease on February 1, 2017 and is
used partly for official and partly for personal purposes. The fair market value of the leased vehicle
at the time of lease was Rs. 1,800,000.
(v) Leave encashment amounting to Rs. 100,000 was paid to Mr. Zameer on July 5, 2023.

61 | P a g e
(c) Mr. Zameer had received 15,000 shares of WPL on December 1, 2020 under an employee share scheme.
He had the option to transfer the shares on or after January 1, 2022. However, he sold all the shares on
April 1, 2010.
Fair value of the shares were as follows:
▪ Rs. 35 per share on December 1, 2020 Page | 62
▪ Rs. 42 per share on January 1, 2022
▪ Rs. 48 per share on April 1, 2023
Tax deducted at source from his salary, amounted to Rs. 650,000.

Required:
Compute the taxable income, tax liability and tax payable by Mr. Zameer Ansari for the tax year 2023.

Question # 3

Mateen was employed with Melody Ltd (ML) as an event organizer. On 30.6.2022 he resigned from his
employment without completion of notice period. On 1.7.2022 he joined another company Rock Star Ltd
(RSL) as a senior event organizer.

Following information is available relating to his assessment for the tax year 2023:
(a) On 1.7.2022 RSL paid Rs.280,000 to ML as compensation in lieu of un-served notice period by
Mateen.
(b) On 15.7.2022 Mateen received a gratuity of Rs.350,000 from an unrecognized gratuity fund
maintained by ML. He also received Rs.150,000 as leave encashment.
(c) In accordance with the terms of his employment with RSL, Mateen was provided with the following
emoluments / benefits during the tax year 2023:
(i) Basic salary of Rs.245,000 per month and utility allowance of Rs 21,000 per month.
(ii) A reimbursement of personal medical expenses up to 15% of the annual basic salary and
Rs.250,000 on account of hospitalization charges of his daughter were made after procuring
hospital bills showing the national tax number of the hospital. These bills were also attested and
certified by RSL.
(iii) For the first two months of his employment, a pick and drop facility was provided to Mateen at a
monthly rent of Rs.25,000. On 1.9.2022 RSL provided a company maintained 1300 cc car which
was partly used for private purposes. The cost of the car was Rs.2,500,000.
(iv) Monthly salary of Rs.6,000 was paid to Mateen's house keeper. Mateen however, reimbursed
20% of the house keeper's salary to RSL.
(v) A special allowance of Rs.50,000 was paid to meet expenses necessarily to be incurred in the
performance of his official duties. Actual expenditure was Rs.40,000.
(vi) On 1.1.2023, he was provided an interest free loan of Rs.1,500,000. The prescribed benchmark
rate is 10%per annum.
(vii) A commission of Rs.500,000 for introducing new clients to the company. Withholding tax was
deducted by RSL @ 12% from such payments.
(viii) The tax deducted at source from his salary by RSL for the tax year 2023 amounted to Rs.550,000
(other than tax deducted on commission).
Required:
Compute the taxable income, mix liability and tax payable / refundable, if any, by Mateen for the tax year
2023.

62 | P a g e
Question # 4

Mr. Khursheed, Pakistani national, was employed as the chief financial officer in Zulfiqar Gas Company
(ZGC), since 2012. Following information pertains to his income for the tax year 2023. Assume that he has
provided the following information on 25.9.2023 to his tax consultant for the purpose of computation of Page | 63
taxable income and tax liability:
(I) Income from ZGC
Khursheed was employed with ZGC up to 31.12.2022. During this period he received the following
emoluments:
▪ Basic salary of Rs.400,000 per month, medical allowance of Rs.75,000 per month and utility
allowance equivalent to 10% of basic salary.
▪ A company-maintained car for official and private use. The car was purchased two years ago at a
cost of Rs.5 million. According to the company's policy, ZGC deducted Rs.10,000 per month from
his salary, for private use of the car.

Khursheed had undergone a major surgery during the year and incurred an expenditure of ks,1,500,000.
ZGC reimbursed the entire amount as a special case as it was not covered under the terms of
employment.

Due to Poor health, Khursheed opted for early retirement on 31.12.2022 under the company's
voluntary retirement scheme.
He received the following benefits on his retirement:
▪ Rs.7,500,000 as a golden handshake under the voluntary retirement scheme.
▪ Rs.9,100,000 from an unapproved gratuity fund maintained by ZGC.
▪ Transfer of company's car for Rs.2,600,000. The amount was deducted from his final settlement.
The fair market value of the car as of 31.12.2022 was Rs.2,800,000.

The tax deducted at source from the salary amounted to Rs.2,500,000.


Required:
Compute the amount of taxable income, tax liability and tax payable / (refundable), if any, for the tax
year 2023.

Question # 5

Mr. Creative is working as Director Human Resources at Artistic Technologies Ltd (ATL). Following are the
details of his income/receipts during the latest tax year:

(a) Monthly cash remuneration from ATL:

Basic salary Rs. 300,000


Utilities allowance 15% of basic salary
Medical allowance 12% of basic salary

(b) In addition to above, he was also provided the following benefits in accordance with his terms of
employment:
(i) Rent-free furnished accommodation in a bungalow situated on a 500 square yard plot of land.
Rent for a comparable accommodation facility in the vicinity is Rs.150,000 per month.

63 | P a g e
(ii)
An 1800cc company-maintained car. The car was purchased two years ago at a cost of
Rs.1,600,000 and is used both for official and personal purposes.
Tax deducted at source from his salary was Rs.400,000.

Required:
Compute the taxable income, tax liability and tax payable for the latest tax year. Page | 64
Question # 6

Mrs. Aslam was employed with Sahal Ltd (SL) as a Marketing Manager. On 30.6.2022 she resigned from
her employment with SL On 1.7.2022, she joined H Pakistan Ltd (HPL), a quoted company, as a Marketing
Director.
She has provided you the following information in respect of the tax year 2023:

(i) In July 2022, she received following amounts from SL in final settlement:
▪ Leave encashment amounting to Rs.95,000.
▪ Gratuity of Rs.500,000 from an unrecognized gratuity fund maintained by SL.
▪ Reimbursement of Rs.100,000 against a health insurance policy. The insurance claim was lodged
by SL on behalf of Mrs. Aslam in January 2022.
(ii) In accordance with the terms of her employment, income tax related to her salary and benefits is to be
borne by HPL. Her emoluments/benefits during the tax year were as follows:
▪ Basic salary of Rs.200,000 per month.
▪ Medical allowance of Rs.60,000 per month.
▪ Rent free accommodation with annual letting value of Rs.480,000.
▪ Traveling allowance of Rs.50,000 per month. 60% of the amount was spent in the performance of
official duties.
▪ Provident fund @ 10% of basic salary. An equal amount was contributed by HPL.
(iii) Under an employee share scheme, Mrs. Aslam was awarded 5,000 shares in HPL on 1.1.2023. Under
the scheme she was not allowed to sell the shares up to 31.3.2023. She sold all the shares in HR. on
1.5.2023.
Fair value of the shares on the above dates was as follows:
▪ Rs.20 per share on 1.1.2023
▪ Rs.28 per share on 31.3.2023
▪ Its.32 per share on 1.5.2023
(iv) On 31-12-2022, she received a loan of Rs.400,000 from HPL. The loan carries a mark-up of MO per
arum The prescribed benchmark rate is 10%.
(v) She won the best executive employee award of HPL and received a laptop having a fair market value
of Rs.100,000.
Required:
Compute the taxable income, tax liability and tax payable for the tax year 2023.

64 | P a g e
Question # 7

Sultan is working as electronic engineer with Ansari Electrical Company Limited (AECL).
He has provided you with the following information for the tax year ended 30 June 2023:

(a) His monthly cash remuneration in AECL is as follows: Page | 65


Rupees
Basic salary 480,000
Medical allowance 48,000
Utilities allowance 55,000

Market value of rent free accommodation 75,000

(b) He was also provided the following benefits in accordance with the terms of his employment: (i)
Leave encashment amounting to Rs. 300,000.
(ii) Hospitalization cost is covered by an insurance policy upto the amount of Rs. 1.5 million. The
insurance premium relating to this benefit amounted to Rs. 55,000.
(iii) He is allowed to use his personal car for office use. Reimbursement of car running and maintenance
expenses amounted to Rs. 550,000. 15% of these expenses pertain to personal use.
(c) Rs. 200,000 were received from a private limited company for attending board meetings.
Required:
Under the provisions of the Income Tax Ordinance, 2001 compute the taxable income and tax thereon
for the tax year 2023. Tax rates are given on the last page.

Question # 8

Mukarram is working as a Commercial Manager in Airmen Engineering Limited (AEL), an unlisted public
company, for the past many years. He derived following emoluments during the tax year ended 30 June
2023:

Rupees
Basic salary (per month) 250,000
Medical allowance (per month) 37,500
Housing allowance (per month) 25,000
Travel allowance (per month) 11,500

In addition to above, Mukarram was also provided the following:


(i) A used company maintained car for both business and personal use. This car was provided to him on
1 July 2022 in replacement of his previous car. This car was purchased three years ago at a price of
Rs. 1,000,000. However, the fair market value of the car on 1 July 2022 was Rs. 800,000. On 1
September 2022, in accordance with the terms of his employment, AEL transferred the previous car to
Mukarram free of cost. The market value of the car at the time of transfer was Rs. 400,000 whereas its
book value was Rs. 200,000. On 1 June 2023, Mukarram sold this car to his neighbor at a price of Rs.
350,000.
(ii) Performance related bonus of Rs. 500,000. The bonus was however, paid to him on 5 July 2023.
(iii) Two free buffet dinner coupons per month, one each for Mukarram and his wife in a five star hotel.

65 | P a g e
The coupons were provided in line with AEL’s policy for its management employees. The dinner costs
AEL Rs. 2,000 per person.
(iv) Reimbursement of Rs. 20,000 in respect of telephone and internet charges. 20% of this amount was
spent by Mukarram in performance of his official duties.
(v) Two air-conditioners and a washing machine for use at home. The combined book value of these
appliances was Rs. 300,000. The appliances are returnable to AEL after three years’ time. AEL Page | 66
charged 10% depreciation on these appliances.
(vi) An option to purchase 20,000 shares in AEL on 1 May 2023 at Rs. 25 per share. The break-up value of
AEL on that date was Rs. 85 per share.
Required:
Under the provisions of the Income Tax Ordinance, 2001 and Rules made thereunder, compute the taxable
income of Mukarram for tax year 2023.

Question # 9

Wajahat, aged 48 years, is a marketing manager in Nayaab (Pvt.) Limited (NPL), a company engaged in the
manufacture and supply of tissue papers. The details of his monthly emoluments during the year ended 30
June 2023 are as under:
Rupees
Basic salary 70,000
Dearness allowance 10,000
Conveyance allowance 8,000

In addition to the above, Wajahat was also provided the following:


(i) Provident fund (PF) contribution of Rs. 8,400 per month. An equal amount per month was
contributed by Wajahat to the fund. Interest income of Rs. 391,000 at the rate of 20% of accumulated
balance of PF was credited to his PF account.
(ii) Reimbursement of electricity bills during the year amounting to Rs. 60,000.

Required:
Under the provisions of the Income Tax Ordinance, 2001 and Rules made thereunder, compute the total
income, taxable income and net tax payable by/refundable to Wajahat during the tax year 2023.

Question # 10

Bader is working as General Manager Finance with HiFi Limited (HFL) for the past two years. The details
of his monthly emoluments during the year ended 30 June 2023 are as under:

Rupees
Basic salary 250,000
Medical allowance 28,000
House rent allowance 120,000

In addition to above, Bader was also provided the following:


(i) Rs. 900,000 for signing a bond with HFL. According to the bond Bader would not resign from his
employment before the expiry of 30 June 2025.
(ii) Company maintained car for both official and private use. The car was purchased on 1 August 2022

66 | P a g e
at a fair market value of Rs. 1,500,000.
(iii) On 1 January 2023 HFL sold an item of inventory to Bader for Rs. 12,000. The net realizable value of
the item of inventory at the end of 31 December 2022 and 30 June 2023 was Rs. 22,000 and Rs.
24,000 respectively. HFL had acquired it in July 2014 at a cost of Rs. 35,000.
(iv) An option was granted to Bader in August 2021 to acquire 2,500 shares in HFL’s parent company,
Mamoo plc. (MP), listed on Hong Kong stock exchange. However, the option was exercisable after Page | 67
completion of one year of service with HFL. Bader paid an amount equivalent to PKR 200,000 to acquire
the option when the fair market value of the option was PKR 250,000.
On 1 September 2015 he paid an amount equivalent to PKR 300,000 to acquire the shares in MP. The
shares were issued to him on 15 September 2022 when the market value of each share was equivalent
to PKR 375.
On 15 June 2023 Bader sold 2,000 shares in MP and received net proceeds equivalent to PKR 875,000
in his bank account in Pakistan. This amount was received after deduction of bank charges of PKR
5,000 and brokerage commission equivalent to PKR 10,000.

Other information relevant to tax year 2023 is as under:


(i) On 1 July 2015 Bader received following payments from his previous employer Sultan Hospital
Limited:
▪ Rs. 600,000 in respect of termination benefits under an agreement.
▪ Rs. 485,000 against gratuity under an unapproved scheme.
(ii) On 1 November 2022 Bader fell ill and was admitted to Sultan Hospital Limited. The hospital incurred
Rs. 65,000 on his treatment but did not charge anything to Bader.
(iii) Withholding tax deducted from Bader’s salary during tax year 2023 amounted to Rs. 1,105,000.
(iv) His total assessed taxable income and total taxes paid thereon during the three preceding tax years
amounted to Rs. 10,500,000 and Rs. 1,260,000 respectively.
Required:
Under the provisions of the Income Tax Ordinance, 2001 and Rules made thereunder, compute the taxable
income and net tax payable by or refundable to Bader for tax year 2023.

Question # 11

Taqi Ahmed is working as Director Marketing with Zee Textiles Limited (ZTL) for the last twenty five
years. Details of his monthly emoluments during the year ended 30 June 2023 are as under:

Rupees
Basic salary 440,000
Conveyance allowance 44,000
Medical allowance 44,000

In addition to the above, Taqi Ahmed has provided the following information:
(i) He and his family members are covered under the health insurance policy in accordance with the
terms of employment. The amount of annual premium paid by ZTL was Rs. 200,000.
(ii) During the year, daily allowance of Rs. 400,000 was received to meet the expenses for working on
assignments at ZTL’s factories located in Lahore and Multan.
(iii) On 31 July 2023, the HR Committee approved a performance bonus for all employees for the year
ended 30 June 2023. Taqi received Rs. 1,200,000 as performance bonus on 15 August 2023.
(iv) On 31 March 2023, in recognition of completion of twenty five years of his service with ZTL, the
board of directors approved to waive the outstanding amount of loan taken by Taqi Ahmed. This

67 | P a g e
interest free loan of Rs. 2,500,000 was taken on 1 January 2021 and was repayable in fifty equal monthly
instalments commencing from May 20X5. The prescribed benchmark rate is 10% per annum.
(v) During the year, he received Rs. 100,000 for attending board meetings of ZTL. No tax was withheld
from this amount.
(vi) Amount of tax withheld by ZTL from his salary amounted to Rs. 2,000,000.
Page | 68
Other information relevant to tax year 2023 is as under:
(i) Salary is transferred to the bank account on 10th of the following month.
(ii) 10% annual increase was given to him effective 1st July in each of the last three years.
Required:
Under the provisions of the Income Tax Ordinance, 2001 and Rules made thereunder, compute under
correct head of income, the total income, the taxable income and net tax payable by or refundable to Taqi
Ahmed for the year ended 30 June 2023

Question # 12

Ahmer Ghazi has been working as director production in Delta Pakistan Limited (DPL) for last three years.
He received following monthly emoluments from DPL during the year ended 30 June 2023:

Rupees
Basic salary 650,000
House rent allowance 95,000
Medical allowance 70,000

In addition to the above, the employer also provided following to Ahmer Ghazi:
(i) Health insurance for him and his family members. The amount of annual premium paid by DPL was
Rs. 50,000.
(ii) Return air ticket for Dubai worth Rs. 180,000 for him and his family as a reward for achieving the
production target.
(iii) Loan of Rs. 5 million was given to him on 1 August 20X7 at 6% per annum.
(iv) Withholding tax of Rs. 1,500,000 deducted from his salary was reimbursed to him.

Other information relevant to the tax year 2023 is as under:


(i) Under an employee share scheme 10,000 shares of DPL were allotted to Ahmer Ghazi on 1 January
2021. According to the scheme, he was not allowed to sell/transfer the shares up to 31 December
2021. On 1 April 2023, he sold 6,000 shares of DPL for Rs. 33 per share. The face value of each share
is Rs. 10. Fair market values of each share on different dates were as follows:
▪ Rs. 20 per share on 1 January 2021
▪ Rs. 23 per share on 1 January 2022
▪ Rs. 29 per share on 30 June 2023
Required:
Under the provisions of the Income Tax Ordinance, 2001 and Rules made thereunder, compute the
following for the year ended 30 June 2023:
(a) Total income
(b) Taxable income
(c) Net tax payable or refundable

68 | P a g e
Question # 13

Saeed, a citizen of Pakistan, was working on a foreign vessel belonging to Delta Shipping Company (DSL)
based in Spain for the past three years. His monthly salary was USD 15,000 which was remitted to his
Pakistani bank account through normal banking channel. The amount received during the tax year 2023 was
converted to Pak Rupees at an average exchange rate of USD 1 = PKR 131. Page | 69

On 1 October 2022, he resigned from DSL and joined Haris Pharma Limited (HPL) in Pakistan as a General
Manager. He was offered following monthly salary and allowance in HPL:

Rupees
Basic salary 600,000
Medical allowance 66,000

In addition to the above, he was also provided the following:


(i) Bonus equal to two monthly basic salaries. However, bonus amount was adjusted in proportion to the
duration of his stay in the company. The bonus amount was paid to him on 5 July 2023.
(ii) Two company maintained cars. Both cars were purchased on 1 October 2022. The car costing Rs.
3,500,000 was used for official purposes whereas the car costing Rs. 1,900,000 was used for personal
purposes.
(iii) Free lunch from the restaurant owned by one of HPL’s directors. The fair market value of food
provided to him during the year was Rs. 125,000.
(iv) A special allowance of Rs. 20,000 per month to meet expenses wholly and necessarily incurred in the
performance of his official duties. Actual expenses incurred by him during the year were Rs.
150,000.
(v) Provident fund contribution of Rs. 60,000 per month. An equal amount per month was also
contributed by Saeed to the fund.
Withholding tax deducted by HPL from Saeed’s salary during the tax year 2023 amounted to Rs.
1,300,000.

Required:
Under the provisions of the Income Tax Ordinance, 2001 and Rules made thereunder, compute under the
appropriate head of income, the total income, taxable income and net tax payable by or refundable to Saeed
for the tax year 2023

Question # 14

Sageer, a resident individual, is working as a full time professor at Knowledge Institute (KI) which is a non-
profit education and research institution and is duly recognized by Higher Education Commission. KI is
entirely owned and funded by Zinger Limited (ZL), a company listed on the Pakistan Stock Exchange.

Details of his monthly remuneration during the year ended 30 June 2023 are given below:

Rupees
Basic salary 200,000
Medical allowance 20,000

69 | P a g e
Fair market rent of accommodation 100,000

In addition to the above, he was also provided the following:


▪ Health insurance for Sageer and his dependents as per the terms of employment. For this purpose, KI is
paying annual insurance premium of Rs. 40,000.
▪ Provident fund contribution of Rs. 15,000 per month to a recognized provident fund. Page | 70
▪ An equal amount was also contributed by Sageer to the fund.

Additional information
(i) On 1 July 2022, Sageer was granted an option to acquire 10,000 shares in ZL at a price of Rs. 105 per
share under an employee share scheme. Sageer bought the option on the same date by paying Rs.
175,000 to KI when the fair market value of the option was Rs. 200,000. He exercised the option on
30 September 2022 when the fair market value was Rs. 130 per share.

As per the scheme, he was not allowed to sell or transfer the shares before 31 December 2022. On
31 December 2022, the fair market value of ZL’s shares was Rs. 142. On 30 May 2023, he sold 5,000
of these shares at Rs. 135 per share.

(ii) On 1 July 2022, Sageer obtained an interest free loan of Rs. 1,500,000 from KI in exchange for which
he agreed to waive the interest receivable on his provident fund balance maintained with KI. Interest
provided on provident fund balance for the year was 8%. The prescribed benchmark rate is 10%.

(iii) On 31 August 2022, he received leave encashment of Rs. 100,000 relating to previous year.

(iv) During the year, tax of Rs. 160,000 was deducted at source by KI.

Required:
Under the provisions of the Income Tax Ordinance, 2001 and Rules made thereunder, compute the
taxable income and net tax payable by or refundable to Sageer for the year ended 30 June 2023. Show all
relevant exemptions, exclusions and disallowances.

Question # 15

Nauman has been working as manager finance in Dua Limited (DL), a public listed company, for many
years. He received following monthly emoluments from DL during the year ended 30 June 2023:

Rupees
Basic salary 120,000
Medical allowance 20,000
House rent allowance 60,000

In addition to the above, the employer also provided him the following benefits:
(i) Company maintained car for both official and personal use. The car was purchased on 1 July 2018 at
the cost of Rs. 1,400,000. As per company policy, Nauman purchased this car at its book value of Rs.
450,000 on completion of five years i.e. 30 June 2023. Fair market value of this car on the date of sale
to Nauman was Rs. 1,000,000.
(ii) Provident fund contribution of Rs. 18,000 per month to a recognized provident fund. An equal amount
was also contributed by Nauman to the fund. Interest income of Rs. 540,000 at the rate of

70 | P a g e
18% of accumulated balance of the fund was credited to Nauman’s account.
(iii) On 1 July 2022, he was transferred to Lahore and was paid relocation allowance of Rs. 300,000.
(iv) HR Committee approved a performance bonus for the year ended 30 June 2023 for all employees.
Nauman received Rs. 400,000 as performance bonus on 15 July 2023.
(v) On 1 April 2023, Nauman obtained a loan of Rs. 5,000,000 @ 6% per annum from DL to purchase a
new house for his own use. First instalment of the loan was paid on 30 June 2023. He incurred legal Page | 71
expenses of Rs. 20,000 for obtaining the loan.
Required:
Under the provisions of the Income Tax Ordinance, 2001 and Rules made thereunder, compute total
income and taxable income of Nauman for the tax year 2023. Show all relevant exemptions, exclusions
and disallowances

Question # 16

For the purpose of this question, assume that the date today is 31 August 2023. Basit, a senior manager at
Master Limited (ML), resigned on 31 January 2023 after completion of three and a half year of service.
During the tax year 2023, he received the following emoluments from ML:
(i) Salary of Rs. 610,000 per month.
(ii) Allowance of Rs. 60,000 per month for services of domestic servant. Out of which, he paid Rs. 36,000
per month in respect of these services.
(iii) Allowance equal to 5% of salary solely expended in the performance of his duties of employment.

Additional information:
(i) On 1 July 2022, he leased a car having fair market value of Rs. 4,800,000 at a monthly rental of Rs.
120,000. He pays lease rentals from his own sources but has used this vehicle for both official and
personal purposes.
(ii) On 1 July 2022, 13000 shares of ML were allotted to Basit under an employee share scheme, against the
payment of Rs. 30 per share. According to the scheme, he was not allowed to sell transfer the shares
upto 31 December 2022. On 31 May 2023, he sold 5000 shares of ML at its fair market value (FMV).
FMV of each share on different dates are as follows:

1 July 2021 31 December 2021 31 May 2022


Rs. 50 Rs. 90 Rs. 80

(iii) On 15 February 2023, he received the following payments from ML as final settlement:
▪ Rs. 320,000 on account of leave encashment.
▪ Rs. 2,200,000 under gratuity scheme approved by the board.
▪ Rs. 700,000 salary arrears related to tax year 2022.
(iv) Withholding tax deducted by ML from Basit’s salary during the tax year 2023 amounted to
Rs.1,400,000.
Required:
Under the provisions of the Income Tax Ordinance, 2001 and Rules made thereunder:
compute the total income, taxable income and net tax payable by or refundable to Basit for the tax year
2023

Question # 17

Nasir has been working as head of finance in Asaaish (Private) Limited (APL). He received following

71 | P a g e
monthly emoluments from APL during the year ended 30 June 2023:

Rupees
Basic salary 800,000
Medical allowance 100,000
Cost of living allowance 200,000 Page | 72
In addition to the above, APL also provided him the following benefits:
(i) Residential house owned by APL for no rent. The fair market value of the rent was Rs. 300,000 per
month.
(ii) Company maintained car. The car was acquired on lease by APL on 1 July 2021 at an annual rental of
Rs. 1,100,000. The fair market value of the car as on 1 July 2021 and 30 June 2023 were Rs.
4,000,000 and Rs. 6,000,000 respectively. 70% of the car is used for office purpose while 30% is used
for personal purposes.
(iii) 250 liter of fuel every month. The average petrol price during the year was Rs. 180 per liter.
(iv) Reimbursement of car maintenance expenses upto Rs. 20,000 per month. During the year, APL
reimbursed Rs. 150,000 to him in this respect.
(v) Health insurance for Nasir and his dependents as per the terms of employment. For this purpose, APL is
paying annual insurance premium of Rs. 100,000. The insurance company incurred expenses of Rs.
500,000 on hospitalization of his dependents.
(vi) Ad-hoc relief allowance equal to one month’s basic salary keeping in view the increase in inflation.

Nasir incurred a monthly expenditure of Rs. 20,000 from July 2022 to November 2022 while working from
home under the COVID guidelines issued by APL’s management.

Withholding tax deducted by APL from his salary during the tax year 2023 amounted to Rs. 4,500,000.
Required:
Under the provisions of the Income Tax Ordinance, 2001 and Rules made thereunder:
compute the total income, taxable income and net tax payable by or refundable to Nasir for the tax year
2023.

72 | P a g e
Question 18

For the purpose of this question, assume that the date today is 30 September 2023.
Cheng, a Chinese citizen, has been in Pakistan since the year 2015. For career growth, he left his
employment with Hope Limited (HL), an unlisted FMCG, on 30 September 2022 and joined a leading
chain of hotels namely Desire Hotels (DH) on 1 October 2022. The details of his emoluments during the
year ended 30 June 2023 are a follows:
HL DH
Particulars
--------- Rupees ---------
Basic salary per month 350,000 450,000
Medical allowance per month 35,000 60,000
Utilities allowance per month 20,000 30,000
Lunch provided by the employer at subsidized
rate – per month cost to employer 10,000 25,000
Company maintained car (for both official as well
as personal purposes):
▪ cost 3,000,000 5,000,000
▪ fair market value as on 30 June 2023 2,500,000 6,000,000
Annual bonus related to the tax year 2022 350,000 -
Gratuity under an unapproved scheme 1,225,000 -
Additional information (other benefits provided by DH):
(i) Two return air tickets to China to the extent of Rs. 600,000 for Cheng and his spouse. During the year, he
incurred Rs. 550,000 on account of his traveling to China.
(ii) Rs. 750,000 received for signing a bond with DH, according to which Cheng cannot leave the
organization before 30 June 2024.
(iii) Rs. 400,000 received from DH as commission for securing a large contract.
(iv) Payment of the outstanding loan of Rs. 3,800,000 by DH as per the terms of the employment contract.
Cheng had obtained this interest free loan from HL, for the construction of a house. On 1 July
2019, the house was given on rent under a 5-year rental agreement at an annual rental of Rs. 800,000.
Required:
Under the provisions of the Income Tax Ordinance, 2001 and Rules made thereunder, compute total
income, taxable income and tax payable by or refundable to Cheng for the tax year 2023. (Show all
relevant exemptions, exclusions and disallowances)

73 | P a g e
Answer 1

Mr. Zulfiqar
Tax year 20X8
Computation of Taxable Income and Tax payable

Rs.
Salary
Basic salary 280,000 × 9 2,520,000
Medical allowance 45,000 × 9 405,000
Less: Exempt up to 10% of basic salary 252,000 153,000
Utility allowance 45,000 x 9 405,000
Cost of living allowance 25,000 x 9 225,000
Rent free accommodation 45% of basic salary 1,134,000
Tax liability borne by the employer 200,000
Gratuity from unrecognized fund 2,660,000
Less: Exempt up to Rs.75,000 or 50% of amount (75,000) 2,585,000
Pension
From Mirza Petroleum (50,000 x 3) 150,000 Exempt
From multinational Company (12,000 x 12) 144,000 144,000
Taxable salary 7,366,000

Tax Liability
1,095,000+ 35% of 1,366,000. 1,573,100

Answer 2

Mr Zameer Asari
Computation of taxable income and tax thereon
For the tax year 2023
Rupees
Income from salary (A)
Basic salary (Rs. 200,000 x 12) 2,400,000
Medical allowance (Rs. 30,000 x 12) [Note 2] 360,000
Utilities allowance (Rs. 10,000 x 12) 120,000
School fees (Rs. 15,000 x 12) 180,000
Rent free furnished accommodation (Rs. 2,400,000 x 45%) 1,080,000
Car used for business purposes only (exempted) -
Car used for personal as well as business purposes (Rs.1,800,000x5%) 90,000
Payment to an approved pension fund (exempted) -
Employee share option - charged to salary in tax year 2020 and Leave encashment (paid -
after tax year 2023)
Payment to approved pension fund-exempt -
Income from Salary 4,230,000

Tax Liability
435,000+27.5% of 630,000. 608,250

. Answer 3

Mr. Mateen
Tax Year 2023
Computation of taxable income and tax liability

74 | P a g e
Rs.
SALARY
Compensation in lieu of unserved notice 280,000
Gratuity received from unapproved fund maintained by ML 350,000
Less: Exempt up to lower of 50% of gratuity or Rs.75,000 (75,000) 275,000
Leave encashment 150,000
Basic salary (245,000 × 12) 2,940,000
Utility allowance (21,000 × 12) 252,000
Company maintained car (5% of Rs.1,500,000 × 10/12) (b) 62,500
Salary of house keeper (6,000 x 12 × 80%) 57,600
Special allowance to meet office expenditures 50,000 -

Interest free loan (1,500,000 × 10% × 6/12) (c) 75,000


Commission from ML (d) 500,000
Taxable salary 4,592,100

Tax Liability
435,000+ 27.5% of 992,100 707,827

Answer 4

Mr. Khursheed
Tax Year 2022
Computation of taxable income and tax liability
Rs.
SALARY
Basic salary 400,000 × 6 2,400,000
Utility allowance 2,400,000 x 10% 240,000
Medical allowance 75,000 x 6 450,000
Less: 10% of basic salary (240,000) 210,000
Medical reimbursement not in accordance with terms 1,500,000
Company maintained car (5% of Rs.5,000,000 × 6/12) 125,000

Golden handshake payment 7,500,000


Gratuity from unapproved fund 9,100,00
Less: Rs.75,000 or 50% of gratuity whichever is lower (75,000) 9,025,000
Vehicle purchased from employer 2,800,000 - 2,600,000 200,000
Salary Income 21,200,000

Tax Liability
1,095,000+ 35% of 15,200,00 6,415,000

Answer 5

Mr. Creative
Tax Year
Computation of taxable income and tax liability
Rs.
SALARY

75 | P a g e
Basic salary 300,000 × 12 3,600,000
Utility allowance 15% of basic salary 540,000
Medical allowance 12% of basic salary 432,000
Less: 10% of basic salary (360,000) 72,000
Accommodation 45% of basic salary 1,620,000
Company maintained car 5% of Rs. 1,600,000 80,000
Taxable salary 5,912,000

Tax Liability
435,000+ 27.5% of 2,312,000 1,070,800

Answer 6

Mrs. Aslam
Tax Year 2023
Computation of taxable income and tax liability
Salary from HPL
Basic salary 2,400,000
Medical allowance 720,000
Less: exempt up to 10% of basic salary (240,000) 480,000
Rent free accommodation: 45% of 2,400,000 1,080,000
Travelling allowance Rs.20,000 × 12 240,000
Contribution to recognized provident fund 240,000
Less: exempt up to lower of Rs.150,000 or 10% of basic + DA (150,000) 90,000
Employee share scheme 5,000 shares × Rs.28 140,000
Laptop from employer 150,000
Taxable salary from HPL excluding tax borne by HPL 4,580,000
Tax borne by HPL as per working note 853,125
Taxable salary from HPL including tax borne by HPL (A) 5,433,125

Salary from SL
Leave encashment 95,000
Gratuity from unapproved fund 500,000
Less: Exempt up to lower of Rs. 75,000 or 50% of gratuity amount (75,000) 425,000
Reimbursement against health insurance policy Exempt
Total (B) 520,000

Total Salary (A+B) 5,953,125

Tax Liability
435,000+ 27.5% of 2,353,125 1,082,109

Answer 7

Rs.
Salary
Basic Salary 480,000 x 12 5,760,000
Medical allowance 48,000 x 12 576,000
Utilities allowance 55,000 x 12 660,000
Accommodation 45% of basic salary 2,592,000

76 | P a g e
Leave encashment 300,000
Reimbursement of running expenses of personal car 550,000 x 15% 82,500
Fee for attending BOD meeting 200,000
Taxable salary 10,170,500

Tax Liability
1,095,000+ 35% of 4,170,500 2,554,675

Answer 8

Mr. Mukarram
Tax Year 2023
Computation of Taxable Income
Rs. Rs.
Salary
Basic salary Rs. 250,000 x 12 3,000,000
Medical allowance Rs. 37,500 x 12 450,000
Less: Exempt 10% of basic salary (300,000) 150,000
Housing allowance Rs. 25,000 x 12 300,000
Travel allowance Rs. 11,500 x 12 138,000
Company maintained car 5% of the Rs. 1,000,000 50,000
FMV of the car transferred by the employer 400,000
Dinner coupons 2 x 12 x Rs. 2,000 48,000
Reimbursement of telephone and internet Rs. 20,000 x 80% 16,000
Depreciation of the appliances provided by the employer 10% of Rs. 300,000
30,000
Employee share scheme: taxable in the next year --
Taxable salary 4,132,000

Tax Liability
435,000+ 27.5% of 5,132,000 1,846,300
Answer 9

Mr. Wajahat
Tax Year 2023
Computation of Taxable Income and Tax Liability

Salary Rupees Rupees Rupees


Basic Salary 70,000 x 12 840,000
Dearness allowance (DA) 10,000 x 12 120,000
Conveyance allowance 8,000 x 12 96,000
Recognized Provident Fund (PF)
Company's contribution 100,800
Less: Rs. 150,000 or 10% of basic salary + DA
Whichever is lower 96,000 4,800

Interest credited to RPF @ 20% 391,000


Less: Interest @ 16% 312,800
1/3rd of basic + DA 320,000
Whichever is Higher 320,000 71,000
Reimbursement of electricity bills 60,000

77 | P a g e
Taxable 1,191,800

Tax Liability
2.5% of 5,191,800 129,795

Answer 10

Mr. Badar
Tax Year 2023
Computation of Taxable Income and Tax Liability
Rs.
Salary from HFL
Basic Salary 3,000,000
Medical allowance 28,000 x 12 336,000
Less: exempt up to 10% of basic 300,000 36,000
House rent allowance 1,440,000
Bond signing amount 900,000
Company maintained car Rs. 1,500,000 x 5% x 11/12 68,750
Inventory item Rs. 22,000 - 12,000 10,000
Employee share scheme
- Fair market value 2,500 shares x Rs. 375 937,500
Cost paid Rs. 300,000 + 200,000 500,000 437,500
Taxable salary from HFL 5,892,250

Salary from Sultan Hospital


Termination benefits 600,000
Gratuity from unapproved scheme 485,000
Less: Exempt up to lower of Rs. 75,000 or 50% of gratuity amount 75,000 410,000
Medical treatment exempt under clause 53A Part I 2nd Schedule --
Taxable Salary 6,902,250

Tax Liability
1,095,000+ 35% of 902,250 1,410,787

Answer 11

Mr. TA
Tax Year 2023
Computation of Taxable Income and Tax Liability
Rs.
Salary
Basic salary 400,000 + (440,000 x 11) 5,240,000
Conveyance allowance 40,000 + (44,000 x 11) 524,000
Medical allowance 40,000 + (44,000 x 11 524,000
Travel and daily allowance to meet office expenses 400,000
Deemed income on interest on loan (Working Note) 120,000
Performance bonus received on 15.8.20X7 taxable in the year 20X8 1,200,000 --
Waiver of outstanding loan 2,500,000 - (22 x 50,000) 1,400,000
Fee for attending BOD meetings 100,000
8,308,000

Tax Liability

78 | P a g e
1,095,000+ 35% of 2,308,000 1,902,800

Answer 12

Mr. Ahmer Ghazi


Tax Year 2023
Computation of Taxable Income and Tax Liability
Rs.
Salary 7,800,000
Basic salary 650,000 x 12 1,140,000
House rent allowance 95,000 x 12 840,000
Health insurance --
Award for meeting sales target 180,000
Concessional loan benefits Rs. 5 million x 4% for 11 months 183,333
Tax paid by the employer 1,500,000
Taxable salary 11,643,333

Tax Liability
1095000+ 35% of 5,643,333 3,070,166

. Answer 13

Mr. Saeed
Tax Year 20X9
Computation of Taxable Income and Tax Liability
Rs.
Salary from DSL:
Foreign sourse salary income exempt being returning expatriate --
Salary from HPL
Basic salary 600,000 x 9 5,400,000
Medical allowance 66,000 x 9 595,000
Less: Exempt up to 10% of basic salary (540,000) 54,000
Bonus: not received in the tax year 20X9 --
Company maintained car for personal use 1,900,000 x 10% x 9/12 142,500
Free lunch provided through an arrangement by employer 125,000
--
Special allowance to meet office expenditure Exempt 180,000 --
Contribution to PF (assumed to be recognized)
Actual contribution by HPL 60,000 x 9 540,000
Less: 10% of basic salary or Rs. 150,000
whichever is lower 150,000 390,000
Taxable Salary 6,111,500

Tax Liability
1,095,000+ 35% of 111,500 1,134,025

79 | P a g e
Answer 14

Mr. Sageer
Tax Year 2023
Computation of Taxable Income and Tax Liability
Rs.
SALARY
Basic salary 200,000 x 12 2,400,000
Medical allowance 20,000 x 12 240,000
[Medical allowance is fully taxable if it is in addition to medical facility in
accordance with terms of employment

Rent free accommodation


Fair market rent of accommodation 100,000 x 12 1,200,000
45% of basic salary 1,080,000
Whichever is higher is taxable 1,200,000

Health insurance premium 40,000 --


Health insurance premium paid by the employer is not income of the
employee. Amount received by the employee from insurance company is
income but the same is exempt subject to the specified conditions

Contribution to recognized provident fund (PF) 15,000 x 12 180,000


Less: Exempt up to Rs. 150,000 or 10% of [basic salary + dearness
allowance], whichever is lower (150,000) 30,000

Employee Share Scheme


FMV on 31st December 10,000 shares x Rs. 142 1,420,000
Cost of shares paid 10,000 shares x Rs. 105 (1,050,000)
Cost of option paid (175,000) 195,000

[FMV of option is irrelevant]

Benefit of interest free loan Exempt

[Benefit of interest free loan is exempt in the following cases:


Where the amount of loan does not exceed Rs. 1 million; or

Where such benefit is extended by the employer due to the waiver of


interest by such employee on his account (e.g. provident fund etc,)
maintained with the employer.]

Leave encashment 100,000

[Salary is taxable on receipt basis]

Taxable salary 4,165,000

Tax Liability

80 | P a g e
435,000+ 27.5% of 565,000 590,375
Answer 15

Mr. Nauman
Computation of total income, taxable income and net tax payable/refundable
For tax year 2023
Rupees
Income from salary
Basic salary [120,000×12] 1,440,000
Medical allowance [240,000(20,000×12) –144,000(1,440,000×10%)] 96,000
House rent allowance (60,000×12) 720,000
Company maintained car for both official and personal use (1,400,000×5%) 70,000
Purchase of car on book value (1,000,000 – 450,000) 550,000
Employer’s contribution to provident fund
[18,000×12=216,000–144,000(1,440,000×10%) (Allowed limit is 1/10 of the basic
72,000
salary OR 150,000 whichever is lower)
Interest on provident fund [540,000–480,000{higher of: interest @ 16% i.e 480,000
60,000
(540,000÷18%)×16 OR 480,000(1/3rd of basic salary i.e. (1,440,000÷3)}]
Relocation allowance -
Bonus – [not taxable in TY2023 as it is received in July 2023) -
Loan obtained on concession rate [5,000,000×4%(10%-6%)×(3÷12) 50,000
Legal expenses – Not deductible being no deduction shall be allowed for expenses
-
incurred in earning salary income
Total income from salary 3,058,000

Tax Liability
165,000+ 22.5% of 6,058,000 1,528,050

Answer 16

Mr. Basit
Computation of total income, taxable income and net tax payable/refundable
For tax year 2023
Salary Rupees
Pakistan source income:
Salary [610,000×7] 4,270,000
Allowance for services of domestic servant [60,000×7] 420,000
Allowance @ 5% of salary solely expended in the performance of his duties
213,500
of employment (4,270,000×5%)
Acquired car on lease -
Shares acquired under employee share scheme [1,170,000(13,000×90)
780,000
390,000(13,000×30)]
Leave encashment 320,000
Gratuity (2,200,000–300,000) 1,900,000
Salary arrears of tax year 2022 700,000
Foreign source income:
Salary (3,200×250×3) 2,400,000
Total income from salary 11,003,500

Tax Liability
1095000+ 35% of 5,003,500 2,846,225

81 | P a g e
Answer 17

Mr. Nasir
Computation of total income, taxable income and net tax payable/refundable
For tax year 2022
Rupees
Salary
Income from salary:
Basic salary (800,000×12) 9,600,000
Medical allowance (100,000×12) 1,200,000
Cost of living allowance (200,000×12) 2,400,000
Housing [360,000{300,000 OR 360,000(45%×800,000) whichever is higher}×12] 4,320,000
Company maintained car (4,000,000×5%) 200,000
Fuel [(250×180) ×12×30%] 162,000
Maintenance of car -
Hospitalization born by insurance company -
Ad-hoc relief allowance 800,000
Expenses incurred for work from home -
18,682,000

Tax Liability
1,095,000+ 35% of 12,682,000 5,533,700

Answer 18

Mr. Cheng
Tax Year 2024
Computation of taxable income and tax liability
SALARY from HL:
Basic salary 350,000 x 3 1,050,000
Medical allowance 35,000 x 3 105,000
Less: Exempt up to 10% of basic salary 105,000 --
Utilities allowance 20,000 x 3 60,000
Lunch provided by the employer 30,000 --
Company maintained car 3,000,000 x 5% x 3/12 37,500
Bonus of previous year received in the current year 350,000
Unapproved gratuity scheme 1,225,000
Less: Exempt up to Rs.75,000 or 50% of amount receivable
whichever is lower 75,000 1,150,000
Benefit of interest free loan 3,800,000 x 10% x 3/12 95,000

SALARY from DH
Basic salary 450,000 x 9 4,050,000
Medical allowance 60,000 x 9 540,000
Less: Exempt up to 10% of basic salary 405,000
Utilities allowance 30,000 x 9 135,000
Lunch provided by the employer 270,000
Company maintained car 5,000,000 x 5% x 9/12 225,000 --
Air tickets for trip to China 187,500
Consideration for signing a bond 550,000

82 | P a g e
Commission 750,000
Repayment of loan by the employer 400,000
3,800,000
Taxable salary 12,885,000

Tax Liability
1095000+ 35% of 6,885,000 35,04,750

83 | P a g e
CHAPTER: 03
INCOME FROM PROPERTY
Income from Property

1) Movable
2) immovable

Dispose/Sell Rent Out


Capital Gain income from property

Taxation model
1) Accrual Basis
2) Revenue Model i.e. Rent -Expenditure= Net Income
3) Part of Normal tax Regime (Expense Allowed)

Income Calculation
Income – Expenditure = Net Income
Property (S.U.R.F)
1) S= Signing Amount XXX
2) U= Non-adjustable amount XXX
3) R= Rent Received XXX
4) F= Forfeited Deposit XXX
R.C.T/C.R XXX
Income Calculation
1) Signing Amount: Paid by the tenant to the Landlord, to enter him into the agreement of tenancy [Totally
Taxable]
2) Forfeited Deposit: Deposit forfeit by the owner in respect of contract for sale of property[Totally Taxable]
3) Rent: [Totally Taxable]
❖ Consideration received/ receivable
❖ By an owner of a land for a tax year
❖ For the use or occupation of land or building
❖ Also includes Forfeited Deposit
[Always takes higher of a) Rent Received / Receivable or b) Fair Market Value (A.L.V)

Example
Miss. Katrina is a relative of Miss. Depika. Miss Depika lets out a house to Miss. Katrina at a subsidized
rent per month of Rs. 12,000. However, rent prevailing in the market for the house of similar size is
Rs. 20,000 per month. For the purpose of rent income under income from property, Rs, 20,000 shall
be taken as it is higher than actual rent received

84 | P a g e
Explanation of Rent:
LAND OR BUILDING includes
• Open plot of land rented out
• Land rented along with building

Example of forfeited deposit


Miss Sonam Bajwa contacted Property dealer to sell her house, she entered into the contract and Rs.
5,000,000 and remaining payment will be received by three months, later on buyer refused to buy the
house, Sonam Bajwa confiscate the amount she received due to default of contract this is known as
forfeited deposit

Income Related to Property Taxed in Various Heads

Exempt Capital Gain Other Source Business

Agriculture Income Disposal of Personal 1) Sub-letting of 1) sale purchase


Property Property of property
2) Income Under 2) Dealer
Amenities Utilities, 3) Builders
services etc. 4) Business
3) Property Rented out Property
with Machinery
4) Vacating the
property [1/10]

Un-adjustable amounts received in relation to buildings [Sec. 16]


(1) Where the owner of a building receives from a tenant an amount which is not adjustable against the rent, it
shall be chargeable under the head "Income from Property" in the tax year in which it was received and the
following 9 tax years in equal proportion.
Amount Received × 1/10 = Add in Property
(2) Where an amount which is not adjustable against the rent is refunded by the owner to the tenant on
termination of the tenancy before the expiry of 10 years, no portion of the amount shall be allocated to the tax
year in which it is refunded or to any subsequent tax year.

(3) Where the owner has refunded non-adjustable amount to the tenant, on termination of tenancy, and the owner
lets out the building to succeeding tenant than new advance less such portion of the earlier amount that was
charged to tax, shall be chargeable under the head "Income from Property" in the tax year in which it was
received and the following 9 tax years in equal proportion.
Note: Non-adjustable advance in case of open plot of land (given on rent) is not taxable under the law.

Example
Mr. M Amir rented out his house to Qasim on 1-9-12 at monthly rent of Rs. 80,000. The fair market rent per
month is Rs. 90,000. In addition to monthly rent, Qasim also agreed to pay Rs. 600,000 as non-adjustable
advance. Qasim vacated the house on 31-1-15. The house was given to new tenant Umar on 1-02-15 at monthly

85 | P a g e
rent of Rs. 120,000. The new tenant gave Rs. 900,000 as non-adjustable amount which was partly used to repay
Rs. 600,000 to old tenant.
Required:
Calculate Gross income from property for tax year 2013, 2014 and 2015.
Solution
TY 2013
Rental income (90,000 x 10 months) 900,000
Add: Nonadjustable (600,000/10) 60,000
Gross rent 960,000

TY 2014
Rental income (90,000 x 12) 1,080,000
Add: Nonadjustable (600,000/10) 60,000
Gross rent 1,140,000

TY 2015
Rental income (90,000 x 7+120,000 x 5) 1,230,000
Add: Non-adjustable amount
Received from new tenant 900,000
Less: Already taxed In TY 2014 (600,000/10) (60,000)
In TY 2013 (600,000/10) (60,000)
780,000 /10 78,000
Gross rent 1,308,000

Example
Mr. Imran has rented out his house to Mr. Nawaz on 1 September, 2009 for a monthly rental of Rs. 35,000 due
to his friendship. Fair market rental of house is Rs. 40,000/month. He also received Rs. 300,000 as non-
adjustable amount on 1 September, 2009.
Further the house was given on rent by Mr. Imran to Mr. Shehbaz on 1 June, 2012 at a monthly rental of Rs.
45,000/month. Mr. Shehbaz gave Rs. 500,000 as non-adjustable amount which was partly used to make payment
of Rs. 300,000 to Mr. Nawaz who vacated house on 31 May, 2012.
Required: Calculate Income from Property for TY 2010, 2011 and 2012?
Solution
Tax Year 2010
Rental income (40,000 x 10) 400,000
Non-adjustable amount (300,000/10) 30,000
Gross rent 430,000

Tax Year 2011


Rental income (40,000 x 12) 480,000
Non-adjustable amount (300,000/10) 30,000
Gross rent 510,000

Tax Year 2012


Rental income (40,000 x 11) + (45,000 x 1) 485,000
Non-adjustable amount {(500,000-30,000-30,000)/10} 44,000
Gross rent 529,000

Treatment of advance adjustable against rent


If the advance is adjustable against rent it will be ignored in the question.
[Expense will be Allowed and Income from Property treated under N.T.R]

Tax Will be calculated at Normal Rates

86 | P a g e
RS
Signing Amount X
Un-adjustable Amount(1/10) X
Rent Received X
Forfeited Deposit X
Chargeable Rent XXX

Less Admissible Deductions*


Deduction in computing income chargeable under the head income from property
(1) Following deductions shall be allowed under the head "Income from Property"
a. In respect of repairs to a building, an allowance equal to one-fifth of the rent chargeable to tax:
Note: Repair allowance will not be allowed in case where only open plot is given on rent.
Note: Repair allowance will not be allowed in case where forfeited deposit is received in case
of open plot/land
Note: Repair allowance will be allowed in case where forfeited deposit is received in case of
contract for sale of house/building.
b. Insurance premium paid (or payable) in the year to insure the building;
c. local rate, tax, charge or cess paid (or payable) on property or the rent from the property to any
local authority or government in the year (excluding income tax);
d. ground rent paid (or payable) for the property;

Miss. JZ has a piece of land. FA wants to construct a house on that land and finally he
constructed his own house on JZs land. Now FA will pay JZ the ground rent of Rs.300,000 per
month. FA later on rented out his house to QZ at monthly be allowed as deduction to FA
e. any profit paid (or payable) on any money borrowed to acquire, construct, renovate, extend or
reconstruct the property;
f. if the property is acquired, constructed, renovated, extended, or reconstructed with capital
contributed by the House Building Finance Corporation or a scheduled bank, the share in rent and
share towards appreciation in the value of property;
g. where the property is mortgaged (or other capital charge), the profit or interest paid;
h. any expenditure paid (or payable) wholly and exclusively for deriving rent including
administration and collection charges up to lower of;
a. actual expense or
b. 4% of rent chargeable to tax
i. Legal charges paid (or payable) to defend the title of the property or any suit connected with the
property in court; and
j. where there are reasonable grounds to believe that unpaid rent is irrecoverable, an allowance
equal to the unpaid rent if:
i. Tenancy was bona fide, the defaulting tenant has vacated the property or steps have been
taken to compel the tenant to vacate the property and the defaulting tenant is not in
occupation of any other property of the person;
ii. Owner has initiated legal proceedings or he believe that legal proceedings would be
useless; and
iii. Unpaid rent has previously been included under the head "Income from Property" and tax
has been paid on it.
iv. Where any unpaid rent allowed as a deduction is wholly or partly recovered, the amount
recovered shall be chargeable in the tax year of receipt.

❖ Where a deduction is allowed to a person for any expenditure in deriving "Income from Property" and
the person has not paid the related liability within 3 years of the end of the tax year in which the

87 | P a g e
deduction was allowed, the unpaid liability shall be chargeable under the head "Income from Property"
in the first tax year following the end of the three years
❖ Where an unpaid liability is taken to income as above and the person subsequently pays the liability,
the person shall be allowed a deduction for the amount paid in the year of payment.
❖ Any expenditure allowed as a deduction under this head shall not be allowed under any other head.
❖ The deductions which are not allowed under the head income from business will also not
be allowed under the head income from property.
Example
Mr. Asim rented out his own house to Mr. Imran against a rent of Rs. 55,000/month. As per the terms Rs.
5,000/m is charged against rendering of utility & sweeper services. This Rs. 5,000 is included in Rs. 55,000.
The actual expenditures incurred by Mr. Asim are
Repair of house 7,000/m
Property tax of house 2,100/m
Utility bills 1,800/m
Sweeper wages 1,000/m
His Income from business in current year is Rs. 900,000.
Calculate his Total Income
Solution
-Income from business 900,000
- Income from other source (5,000-1,8000-1,000) x 12 26,400
- Income from property (W-1) 454,800
Taxable income-taxable under NTR 1,381,200

(W-1) Income from property


Gross Rent (50,000 x12) 600,000
Less: Admissible deductions
Repair allowance (600,000 x 1/5) (120.000)
Property tax of house (2,100 x 12) (25,000)
Rent chargeable to tax 454,800

Income of joint owners / Co-Ownership


(1) Where any property is owned by two or more persons and their respective shares are definite and
ascertainable:
a) The persons shall not be assessed as an AOP in respect of the property; and
b) The share of each person in the income from the property for a tax year shall be taxable in their own
hands respectively and not as AOP.
(2) This section shall not apply in computing income chargeable under the head "Income from Business". Note:
Where any property chargeable under section 15 is owned by two or more persons and their respective shares
in that property are not definite and ascertainable, the property will be considered as being jointly owned by an
association of persons (AOP) and taxable income and tax payable thereon will be computed as per the principles
of taxation for AOP.

Exam Note: If AOP is earning both income from business and income from property, then income from
property will be taxable in the hands of AOP and not the members(For Details Visit AOP)

Example
Mr. A and B are joint owners of a house and they have provided the following data:
Profit sharing ratio amongst A and B 60:40
Rental income earned from house for TY 2013 3,000,000
Calculate tax liability for A and B?
Solution-1
Mr. A
Income from property (2,400,000 x 60%) 1,440,000
Tax liability – A 75,000+ (240,000 x 20%) 123,000

88 | P a g e
Mr. B
Income from property (2,400,000 x 40%) 960,000
Tax liability 15,000+ (160,000 x 15%) 39,000

(W-1) Income from property


Gross receipt 3,000,000
Less: Admissible deductions
Repair allowance (3,000,000 x 1/5) (600,000)
2,400,000
Example
Qasim and Talha jointly own a house in Lahore. Qasim has 75% share in the house. On 1 July 2015, the house
was let out at an annual rental value of Rs. 10 million. This amount includes Rs. 100,000 per month for
utilities, cleaning and security.
During the tax year 2016, the owners incurred the following expenditures in relation to the house:
Rupees
Utility bills paid to utility companies, security charges paid to security guard and cleaning 500,000
expenses paid to sweeper
Repair and maintenance 450,000
Insurance premium 340,000
Collection charges 25,000
Qasim and Talha have no other source of income. All the above expenses were incurred by them jointly.
Required:
Calculate tax liability of Qasim and Talha for the tax year 2016.

Solution-2
Income of Qasim
Income from Other Source ((W-2) 700,000 x 75%) 525,000
Income from property (W-1) (B) 5,006,250
Taxable income: 5,531,250
Tax Payable to Govt. (765,000+35% x 1,531,250) 1,300,938

Income of Talha
Income from Other Source ((W-2) 700,000 x 25%) 175,000
Income from property (W-1) (C) 1,668,750
Taxable income 1,843,750
Tax Payable to Govt. (75,000+20% x 643,750) 203,750

(W-1) Income from property


Gross Rent (10,000,000 100,000 x 12) 8,800,000
Less Admissible deductions
Repair allowance (8,800,000 x 1/5) (1,760,000)
Insurance premium (340,000)
Collection charges lower of:
Actual charges 25,000
4% of chargeable rent (4% x 8,800,000) 352,000 (25,000)
Rent chargeable under NTR (A) 6,675,000

Share of Qasim (6,675,000 (A) x 75%) (B) 5,006,250


Share of Talha (6,675,000 (A) x 25%) (C) 1,668,750

(W-2) Income from other source


Rent charged for utilities and security (100,000 x 12) 1,200,000
Less: Utilities and security expense (500,000)
700,000

89 | P a g e
Agriculture Income
1. Agriculture income means an income, which satisfies the following conditions:
▪ The income is derived from land. The income may be in the form of rent or revenue;
▪ The land must be used for agriculture purpose and be situated in Pakistan;
2. Agricultural Income derived by a person during the tax year shall be exempt from tax.
3. Various sources of agriculture income are:
a. Income of the cultivator or receiver of rent-in-kind from the sale of agriculture produce without
performing any further process.
b. Income from the performance of any process by the cultivator or receiver of rent-in-kind,
generally employed to render the produce fit to be taken to market.
c. Income from building shall also be agriculture income, subject to the following conditions
▪ Building is owned and occupied by the cultivator or receiver of rent-in-kind;
▪ Building is situated in the immediate vicinity of the land used for agricultural purposes;
▪ Building is required as a dwelling house, a store house or other out building by the
cultivator or the receiver of rent-in-kind due to his connection with the agriculture land.

Self-Hiring of Property
Where an employee or his spouse is the owner of any such building that is given on rent to the employer and
the employer has provided the same building to the employee against his entitlement for a rent free
accommodation, then it will have following effect
• Receipt of rent of building is chargeable to tax under the head income from property.
• Any rent received by the employee or his spouse shall be property income of the recipient and be treated
accordingly.
• The building is provided by the employer to his employee as a rent free accommodation. It will be a
perquisite and added in the salary income of the employee

Where rent received or receivable is less than fair market rent for the property, the person shall be treated as
having received the fair market rent for the period the property is let on rent in the tax year. However, this shall
not apply in the case of self-hiring where fair market rent is already included in the income of the lessee,
chargeable to tax under the head “Salary”.

90 | P a g e
Income from Property (Summary Sheet)
Sr. Taxable
No Particulars Rs. Rs
Rent Chargeable to Tax (RCT)
1 Signing amount X
2 Non-Adjustable amount
• If received from 1st tenant (Amount × 1/10)
(Or)
• If received from succeeding tenant
Amount Received X
Less : Amount Already charged to tax from previous tenant (X) X
(Always take 1/10 of non-adjustable amount for current year)
3 Rent Received (Higher of )
• Rent received (or)
• FMV/ ALV X X
4 Forfeited advance against sale of property X
5 Owner’s Burden ( i.e tax etc ) paid by the tenant X
6 Irrecoverable rent received X
7 Adjustable Advance X
8 Non-Payment of expense ( previously allowed as deduction ) X
Total Rent Chargeable to Tax ( R.C.T ) XXX
Less: Allowable deductions
9 Repairs allowance ( 1/5 of RCT ) X
Note:
It is the mandatory deductions, actual repair charges will be ignored for property
(Except Open Plot)
For open plot no Repairs will be taken, only actual expenses will deducted if they
had incurred
Any tax in connection with Property
• Property Tax
10 • Corporation/Municipal tax
• Local Tax
• Any other Tax X
11 Ground rent (if property is on leased land) X
12 Interest on moneys borrowed for property X
13 Interest on loan against mortgage of property X
14 Share of Rent paid to H.B.F.C X
15 Rent collection charges ( Up to 4% of RCT ) X
16 Legal Charges in connection with the property X
17 Irrecoverable Rent ( Subject to the conditions ) X
18 Payment of expenses already treated as income due to its non-payment X (XXX)

Total Income of property XXX

91 | P a g e
PAST PAPER QUESTIONS
Question # 1 (Q.3 (a) September 2002
What is chargeable to tax under the head 'income from property"?

Question # 2 Q.4 September 2002


ABC Associates owns a building which is on rent. The following information is available:
Rent received from tenants 2,300,000
Depreciation on building under the Third Schedule to the Ordinance 400,000
Property Tax 100,000
Municipal/local government taxes (agreements with tenants provide that tenant
should pay the municipal taxes) 100,000
Rent received includes Rs. 600,000 for three years commencing from July 01 of the current year. ABC
Associates follow accrual basis of accounting and its income year is July-June 20X8.
Required: Compute the income from property of ABC Associates.

Question # 3 Q.5 March 2003


Mr. Amir-ud-din has recently constructed an office complex for the purposes of letting out. As per terms and
conditions, Mr. Amir-ud-din is also entitled to signing amount, which is non-refundable.
For the tax year 20X8 following information has been provided to you for the computation of his income from
property:
Rent for the year already received 1,150,000
Rent for the year though due but irrecoverable 50,000
Signing amount (non-adjustable, non-refundable) 100,000
Fire tax paid to the local authority 20,000
Lawyer's fee for suit to recover rent 50,000
Salary of the caretaker who collects monthly rent 46,000
Insurance premium being 1% of market value of the property 200,000
Repair maintenance expenditure 50,000
Calculate his income from property for TY 20X8.

Question # 4 Q.1 (a) September 2006

Discuss the provisions of the Income Tax Ordinance, 2001 regarding non-adjustable amount received from a
tenant by the owner of a building.

Question # 5 Spring 2012 Q. 4(b)

Yaqoot and Loha are joint owners of a bungalow which has been rented out for Rs. 70,000 per month.

Required:
Discuss the taxability of Yaqoot and Loha in respect of above income, in the light of Income Tax Ordinance,
2001. (03)

Question # 6 Spring 2014 Q. 4

Bashir and Jameel jointly own a house in Karachi. Bashir has 75% share in the house. On 1 September 20X3,
the house was let out at an annual rental value of Rs. 6,500,000. This amount includes Rs. 186,000 per month
for utilities, cleaning and security.

During the tax year 20X4, the owners incurred the following expenditures in relation to the house:

Rupees

92 | P a g e
Utilities, cleaning and security 650,000
Repair and maintenance 810,000
Insurance premium 240,000
Collection charges 25,400
Mark-up on amount borrowed for extension of the house 840,000

Bashir and Jameel have no other source of income. All the above expenses were incurred by them jointly.

Required:
Calculate taxable income of Bashir and Jameel under appropriate heads of income for the tax year 20X4. (10)

Question # 7 Spring 2015 Q. 4

(a) (i) Explain the term ‘Rent’ in context of ‘Income from property’. (02)
(ii) Specify the head of income under which the following amounts would be chargeable to tax:
▪ rent from sub lease of a building.
▪ amount included in rent for the provision of amenities, utilities and any other service connected
with renting of the building. (02)

(b) On 1 July 2014, Fahim agreed to rent out a house to Mirza at a monthly rent of Rs. 180,000 with effect
from 1 August 2014 and received one year’s rent in advance. He also received Rs. 800,000 as a security
deposit which was partly used to repay the security deposit amounting to Rs. 400,000 received from the
previous tenant in July 2010 and partly used for renovation of the house.

Fahim also incurred the following expenses in respect of the above house:
(i) property tax of Rs. 15,000.
(ii) payment of interest amounting to Rs. 200,000 to his friend against amount borrowed for renovation
of the house.
(iii) insurance premium of Rs. 110,000.
(iv) Rs. 5,000 per month to Wasif for collection of rent.

Required:
Under the provisions of the Income Tax Ordinance, 2001 compute the taxable income of Fahim for tax
year 2015 assuming he has no other income. (07)

Question # 8 Autumn 2016 Q. 4

On 1 July 2015 Farrukh borrowed Rs. 8,000,000 from Star Bank Limited and acquired a plot of land in Hub
Industrial Zone for Rs. 6,500,000. He invested the rest of the loan in a business venture with his friend. The
above loan carries mark-up at a rate of 12% per annum and is repayable in eight equal quarterly instalments
starting from 1 July 2016. On 1 August 2015 Farrukh decided to sell the plot of land to Zulfiqar Motors for Rs.
10,000,000 and received a deposit of Rs. 500,000 from them. On 15 August 2015 Farrukh forfeited the deposit
on refusal of Zulfiqar Motors to purchase the plot of land.

On 1 September 2015 Farrukh let out the plot of land to his friend Atif at a monthly rent of Rs. 150,000. He
received an un-adjustable deposit of Rs. 200,000 from Atif and paid Rs. 80,000 for levelling the ground, Rs.
50,000 as ground rent, Rs. 12,000 as insurance premium against the risk of damage or destruction by water
logging and Rs. 140,000 against rent collection charges. Farrukh had paid Rs. 25,000 to a firm of professional
valuers which determined the annual rental value of the plot of land at Rs. 2,160,000.

Required:
Under the provisions of the Income Tax Ordinance, 2001 and Rules made thereunder, compute under the
relevant head of income, taxable income of Farrukh for tax year 2016. (12)

93 | P a g e
Question # 9 Spring 2019 Q. 3(c)
On 1 July 20X8, Zahid rented out his properties as follows:
(i) An apartment was rented to Abdul Qadir at a monthly rent of Rs. 40,000. Zahid received a non-adjustable
security deposit of Rs. 300,000 which was partly used to repay the non-adjustable security deposit
amounting to Rs. 175,000 received from the previous tenant in July 20X3. He also spent Rs. 20,000 on
repairs of the apartment in February 20X9.
(ii) A bungalow was rented to a bank. Zahid and his younger brother are joint owners of the bungalow in the
ratio of 60:40 respectively. The annual rent agreed with the bank was Rs. 6,000,000 which is inclusive of
Rs. 100,000 per month for utilities, cleaning and security. Zahid paid Rs. 35,000 per month for providing
these services.

Required:
Under the provisions of Income Tax Ordinance, 2001 compute total and taxable income of Zahid for the tax
year 20X9 under appropriate heads of income. (07)

Question # 10 Autumn 2019 Q. 2


(a) Explain the term ‘Rent’ with relation to ‘Income from property’. (02)

(b) During the tax year 20X9, Amjad carried out the following transactions in respect of his properties:
(i) On 1 July 20X8, Amjad purchased a factory building in Sukkur along with the installed machinery
at the price of Rs. 9 million and Rs. 3 million respectively. To manage the shortage of funds of Rs.
2,000,000, he borrowed the same on 1 July 20X8 from his friend Shamshad through a crossed cheque.
The loan carries interest at the rate of 18% per annum.
On 1 January 20X9, he let out this building along with the machinery to Basit at a monthly rent of
Rs. 500,000 payable in advance.
(ii) On 1 July 20X8, Amjad let out his residential property situated in DHA Karachi to Mirza Limited at
a monthly rent of Rs. 300,000. Rent for the two years was received in advance on 1 August 20X8.
(iii) On 1 July 20X8, Amjad also entered into an agreement with Zeeshan for the sale of his plot situated
in Quetta for Rs. 50 million. The plot had been purchased for Rs. 40 million in 20X4. Under the
terms of sale agreement, he received Rs. 5 million at the time of signing the agreement and the
balance was to be received on 30 September 20X8. However, due to financial difficulties, Zeeshan
failed to pay the balance amount on the due date and consequently, Amjad forfeited the advance in
accordance with the terms of the agreement.
On 10 April 20X9, he finally sold the plot to Jamshed for Rs. 65 million.

(iv) Following expenditures were incurred by Amjad in respect of his properties in Sukkur and Karachi:

Property situated in
Details of expenditures
Sukkur Karachi
Repair & maintenance - building 270,000 70,000
- machinery 50,000 -
Ground rent 50,000 10,000
Insurance - building 150,000 20,000
Total 520,000 100,000

Required:
In view of the provisions of the Income Tax Ordinance, 2001 compute under appropriate head of income,
taxable income of Amjad for the tax year 20X9. (10)

Question # 11 Autumn 2019 Q. 3(e)

Farhan and Imran jointly own a building in Quetta. The building has been rented out to a company. Discuss
the tax treatment of income from such property. (02)

94 | P a g e
Question # 12 Autumn 2019 Q. 3(e)

Kashif is a resident filer who owns a single-storey bungalow in Karachi, including a basement.
He solely uses the basement portion of the bungalow which constitutes 20% of the total
bungalow area, for storing his personal belongings.

On 1 October 2019, he rented his bungalow, excluding the basement portion, to Ahmed under a
three-year rental agreement. Other details of the rental agreement are given below:

Rupees
Monthly rent 300,000
Non-adjustable security deposit 3,500,000
Monthly security charges 40,000

In addition to the above, Kashif also provides Ahmed with backup electricity from a generator
during load shedding at a fixed monthly charge of Rs. 50,000. The electricity connection of the
basement is separate from the rest of the bungalow.
On 30 September 2022, the rental agreement concluded, and Kashif agreed to sell the entire
bungalow to Ahmed. The non-adjustable security deposit was retained as a down payment for
the purchase.

On 25 October 2022, Ahmed backed out of the deal and declined to purchase the bungalow. As
per the agreement, Kashif forfeited the non-adjustable security deposit.

On 1 November 2022, Kashif rented the bungalow to a new tenant, Rashid, under a rental
agreement with the same terms as above.

During the year, Kashif paid salary of Rs. 360,000 to the security guard of the bungalow and
incurred Rs. 450,000 for running the electricity generator.

Required:
Under the provisions of the Income Tax Ordinance, 2001 and Rules made thereunder,
compute the total income of Kashif under appropriate heads of income for the tax year 2023. (08)

95 | P a g e
ANSWERS
Question # 1 (Q.3 (a) September 2002

Followings are chargeable to tax under the head income from property:
1. Any amount received or receivable by the owner of land or a building as consideration for the use or
occupation of, or the right to use or occupy, the land or building,
2. Any forfeited deposit paid under a contract for the sale of land or a building.
3. Where the owner of a building receives from a tenant an amount which is not adjustable against the rent,
the earned amount shall be treated as rent chargeable to tax under the head "Income from Property" in the tax
year in which it was received and the following nine tax years in equal proportion.

Question # 2 Q.4 September 2002

Income from Property (W-1) 1,400,000


Taxable income 1,400,000

(W-1) Income from property


Rent Received 2,300,000
Add: Municipal taxes paid by tenant 100,000
2,400,000
Less: Amount relating to next years (600,000/3 x 2) (400,000)
Gross rent chargeable to tax 2,000,000
Less: Admissible deductions
Repair allowance (2,000,000 x 1/5) (400,000)
Property tax (100,000)
Municipal/local government taxes (100,000)
1,400,000

Question # 3 Q.5 March 2003

Income from Property (W-1) 602,000


Taxable income 602,000

(W-1) Income from property


Annual rent received Rs. 1,150,000
Irrecoverable rent for the year 50,000
Non-adjustable amount (100,000/10) 10,000
1,210,000
Less: Admissible deductions
Repair allowance (1/5 x 1,210,000) (242,000)
Fire tax paid to local authority (20,000)
Lawyer's fee for suit to recover rent (50,000)
Collection charges and other admin (Lower of :)
Actual 46,000
4% of 1,210,000 48,400 (46,000)
Insurance premium (200,000)
Bad debt (50,000)
602,000

Question # 4 Q.1 (a) September 2006


Read Section 16 From the Chapter

Question # 5 Spring 2012 Q. 4(b)

96 | P a g e
Where any property is owned by two or more persons and their respective shares are definite and ascertainable-
(a) the persons shall not be assessed as an AOP in respect of the property; and

(b) The share of each person in the income from the property for a tax year shall be taxable in their own hands
respectively and not as AOP.
Therefore Yaqoot and Loha will pay tax separately on the income earned by them

Question # 6 Spring 2014 Q. 4

Income of Bashir
Income from other source [w-2 1,210,000 x 75%] 907,500
Income from property w-1 1,934,950
Taxable Income under NTR 2,842,450
Tax Payable to Govt. (315,000 + 25% x 442,450) 425,616

Income of Jameel
Income from other source [w-2 1,210,000 x 75%] 302,500
Income from Property 644,984
Taxable Income under NTR 947,484
Tax Payable to Govt. (15,000 + 15% x 147,484) 37,123

W-1 Income from Property


Gross receipt 6,500,000
Less: Income charged under other Source (186,000 x 12) (2,232,000)
4,268,000
Less: Receipt relating to the next year (4,268,000 x 2/12) (711,333)
Rent chargeable to tax 3,556,667
Less: Admissible deductions
Repair allowance (3,556,667 x 20%) (711,333)
Insurance Premium (240,000)
Collection Charges (25,400 or (4% of 3,556,667) (25,400)
Net Income from Property 2,579,934

Share of Bashir (2,579,934 x 75%) 1,934,950


Share of Jameel (2,579,934 x 25%) 644,984

W-2 Income from Other Source


Rent charged for utilities etc. (186,000 x 10) 1,860,000
Less : Expenses (650,000)
1,210,000

Question # 7 Spring 2015 Q. 4

(a)
i) “Rent” means any amount received or receivable by the owner of land or a building as consideration for
the use or occupation of, or the right to use or occupy, the land or building, and includes any forfeited
deposit paid under a contract for the sale of land or a building.
ii) Rent from sub lease of building will be charged at Income from other source
Amount from Amenities, utilities and any other services connected with the building be a part of Income
from other source

97 | P a g e
(b) Computation of Income from Property
Rent received (180,000 x 11) 1,980,000
Un-adjustable advance rent [800,000 – (400,000 /10 x 4)] / 10 64,000
Total income from property 2,044,000
Less: Allowable Deductions
Repair and maintenance (2,044,000 x 1/5) (408,800)
Property tax (15,000)
Payment of interest (200,000)
Insurance premium (110,000)
Collection charges (55,000)
Lower of (2,044,000 x 4% = 81,760) or (5,000 x 11 = 55,000)
(788,800)
Taxable Income 1,255,200

Tax Liability
[75,000 + (55,200 x 20%)] 86,040

Examiner Comments:
a(i)This part was based on Section 15(2) and 16(1) of Income Tax Ordinance, 2001 and
required the candidates to explain the term 'Rent' in the context of 'Income from
property'. The performance was satisfactory as generally the candidates were able to
reproduce the relevant provisions of the Income Tax Ordinance, 2001. A number of
candidates, however, did not mention that the amount received by the owner of building
from the tenant which is non-adjustable against the rent payable is also treated as rent.
(ii) This part was also well attempted as majority of the candidates knew that rent from sub-
lease of a building and the amount included in rent for provision of amenities, utilities
and any other service connected with renting of the building will be chargeable to tax
under the head "Income from Other Sources".

(b) This part required computation of income from property derived by an individual
having no other income. This was a simple computational question and many candidates
performed excellently and secured high marks. However, a number of candidates failed
to seize this opportunity and made elementary mistakes. Few of the commonly observed
mistakes are as under:
▪ Rent for the entire year i.e. 12 months was considered instead of 11 months only.
▪ Value of repair allowance has to be calculated as 1/5th of the total rent chargeable
to tax where total rent chargeable to tax also includes that part of the security deposit
which is included in taxable income. Many candidates ignored the security deposit
and computed repairs allowance as 1/5th of the amount of monthly rent only.
▪ A number of candidates did not know that rent collection charges are admissible
deduction at 4% of total income from property or actual, whichever is lower. They
considered that actual charges are deductible in all cases.
▪ Interest on loan for renovation of the house was treated as inadmissible.
▪ Property tax of Rs. 15,000 paid was adjusted against total tax liability.
▪ Tax liability was also calculated whereas only the computation of taxable income was
required.

98 | P a g e
Question # 8 Autumn 2016 Q. 4
Farrukh
Computation of taxable income
For the tax year 2016
Rupees
Income from property:
Forfeiture of deposit 500,000
Rent of plot of land [higher of (2,160,000 12 x 10) or (150,000 x 10)] 1,800,000
Gross rent 2,300,000
Amount not adjustable against the rent -
(Nothing is to be included in the chargeable income as this provision of law is attracted
where the owner of building and not land receives such amount.)
Less:
Repairs 1/5th of rent (leveling of ground) -
(Admissible only against the rent of the building)
Interest on loan [6,500,000 x 12%] x [10 - 12] (650,000)
(interest on only that portion of the loan which is utilized for the acquisition of land is
admissible)
Ground rent (50,000)

Insurance premium [not available on land] -


Rent collection charges (2,300,000 x 4%) (92,000)
(Lower of actual expenditure or 4% of rent is admissible)
Fee paid to professional valuer- inadmissible -
(792,000)
1,508,000

Examiner Comments:
This computational question offered 12 marks and was aimed at testing the concepts of the
students relating to taxation of income under the head “Income from Property”. The
overall performance was average. The commonly observed mistakes were as under:
▪ Some candidates classified the whole stream of income as “Income from Other Sources”
instead of “Income from Property”.
▪ Forfeiture of deposit was considered taxable over a period of ten years.
▪ Higher of fair market rent or actual rent received is to be treated as income. Many
candidates considered actual rent received as taxable. Moreover, in many cases, rent for
the entire year i.e. 12 months was calculated instead of 10 months.
▪ Many candidates included the un-adjustable deposit received on renting of plot of land
in computing income from property. They failed to realize that nothing was to be included
in the chargeable income as this provision of law is attracted where the owner of a
building receives such amount and this rule is not applicable in case of land.
▪ Repairs allowance at 1/5 of gross rent was considered admissible. Candidates did not
appreciate that such allowance was admissible only against the rent of a building not
land.
▪ Allowable interest expense was wrongly calculated in many of the scripts. Candidates
calculated interest on entire amount of loan, ignoring that interest is admissible only on
that portion of the loan which is utilized for the acquisition of land. Many students also
failed to apportion the interest over 10 months.
▪ Insurance premium is not an admissible deduction in case of land; most of the candidates
were not aware of this fact.
▪ A number of candidates did not know that rent collection charges are admissible at 4%
of gross rent or actual, whichever is lower. They considered that actual charges are
deductible in all cases. While few candidates inflated the actual collection charges by the

99 | P a g e
amount of other expenses, such as ground rent, insurance premium, etc., for comparison
purposes.
▪ Fee paid to professional valuer was wrongly considered as admissible deduction.

Marking Scheme:
▪ Computation of gross rental income 3.0
▪ Determination of the amount not adjustable against the rent 1.5
▪ Treatment of various expenses related to the property income. 7.5

Question # 9 Spring 2019 Q. 3(c)

Income from property Rupees


Rental income (40,000 × 12) 480,000
Rental income from joint property (4,800,000 (W-1) × 60%) 2,880,000
Less: Repair charges -
Add: Un-adjustable security deposits (Rs. 212,500 (W-2) × 1/10) 21,250
3,381,250
Less: Allowable Deductions
Repair Expense (3,381,250 x 20%) (676,250)
Income from Property (Net) 2,705,000

Income from other sources

Income from utilities, cleaning and security


[(1,200,000 - 420,000)×60%] 468,000
Taxable income 3,173,000
Tax Liability (465,000 + 30% x173,000) 516,900

W-1: Determination of Income from joint property (bungalow) Rupees


Total rental income 6,000,000
Less: Amount relating to utilities, cleaning and security (1,200,000)
Income from property 4,800,000

W-2: Computation of un adjustable security deposit Rupees


Received from new tenant 300,000
Less: Amount charged to tax in July 2013 to June 2018 (175,000 × 5/10) (87,500)
212,500

Examiner Comments:
(i) Amount relating to utilities, cleaning and security, being income from other sources,
was not excluded from the rental income from joint property.
(ii) Incorrect amount of the un adjustable security deposit was worked out for inclusion in
the head of income from property.
(iii) 60% of the proportionate share of income from utilities, cleaning and security was
taxable as income from other sources. However, income from providing such amenities
was to be reduced by the sum paid by the individual in providing such services before
applying the proportionate share percentage. This point is generally missed in number
of scripts.
(iv) Income from property was not deducted from total income to arrive at taxable
income.

100 | P a g e
Marking Scheme:
Computation of:
▪ income from property 5.0
▪ income from other sources 2.0

Question # 10 Autumn 2019 Q. 2

(a) ‘Rent’ means any amount received or receivable by the owner of land or a building as consideration for
the use or occupation of, or the right to use or occupy, the land or building, and includes any forfeited
deposit paid under a contract for the sale of land or a building.

Where the owner of a building receives from a tenant an amount which is not adjustable against the rent
payable by the tenant, the amount shall be treated as rent.

(b) Mr. Amjad


Computation of taxable income
For tax year 20X9
Rupees
Income from property
(i) Residential property at DHA – Karachi (W-1) 2,850,000
(ii) Amount forfeited from Zeeshan 5,000,000

Income from other sources


(i) Factory building at Sukkur – Basit (W-1) 950,000

Income from capital gain


(i) Sale of plot in Quetta – Jamshed -
Since the plot was bought in 2015, therefore no tax is payable under the law
disposal after 6 years
Total Taxable income 8,800,000
Tax Liability (765,000 + 2,800,000 x 35%) 2,445,000

W-1: Income from Income other


property sources
DHA - Karachi Sukkur - Factory
Rental Income (300,000×12), (500,000×6) 3,600,000 3,000,000
Less: Admissible expenses
Repair to building (allowed upto 1/5 of the rental
amount) (720,000) 270,000
Repair to machinery (10,000) 50,000
Ground rent - 50,000
Insurance – Building (20,000) 150,000
Depreciation: Building – Normal [Rs. 9m @ 10%] - 900,000
Plant – Normal [Rs. 3m @ 15%] - 450,000
Interest on loan from Shamshad
Rs. 2 million @ 18% for 6 months - 180,000
- 2,050,000

101 | P a g e
Net income 2,850,000 950,000

Examiner Comments:
▪ Many examinees offered 11 months’ rent to tax instead of 12 months.
▪ Expenses relating to residential property were considered to be admissible in case of
individuals.
▪ Rent from factory building together with plant and machinery was wrongly considered
under the head ‘Income from property’.
▪ Many examinees computed initial depreciation on both building and plant and
machinery. Only few examinees correctly computed depreciation for the period of 6
months instead of the whole year.
▪ Gain arising on sale of plot in Quetta was wrongly treated as taxable gain.
▪ Some examinees considered the gain to be taxable under the head ‘Income from
property’.
▪ Some examinees offered the rent received in advance for two years to be taxed under the
head Income from property.

Marking Scheme:
(a) Explanation of the term ‘Rent’ 2.0
(b) Computation of income from:
▪ property 4.0
▪ other sources 4.5
▪ capital gain 1.5

Question # 11 Autumn 2019 Q. 3(e)

Where any property is owned by two or more persons and their respective shares are definite and
ascertainable-
(a) the persons shall not be assessed as an AOP in respect of the property; and
(b) The share of each person in the income from the property for a tax year shall be taxable in their own
hands respectively and not as AOP.
However, if the shares of Imran and Farhan are not definite and ascertainable, then they will be assessed
as an AOP in respect of income form building in Quetta

Examiner Comments:
▪ Majority of the examinees could not identify the conditions under which Farhan and
Imran would have been assessed as an association of persons. They considered that both
of them would share the rent equally.
▪ Some of the examinees thought that since Farhan and Imran jointly own a building in
Quetta, rent received from the company will be chargeable to tax under the head income
from property and no deduction would be allowed to them against such income.

Marking Scheme:
▪ Discussion on provision relating to a property owned by joint owners 1.5
▪ Conclusion 0.5

102 | P a g e
Question # 12 Autumn 2023 Q. 2

Kashif
Computation of income for tax year 2023
Income from property Rupees
Monthly rent from Ahmed (300,000×3) 900,000
Forfeited deposit 3,500,000
Monthly rent from Rashid (300,000×8) 2,400,000
Non-adjustable security deposit from Rashid (2,450,000(W-1)÷10) 245,000
7,045,000
Less: Repair allowance @ 20% (1,409,000)
Total income from property 5,636,000

Income from other sources


Security charges (40,000×11) 440,000
Electricity generation (50,000×11) 550,000
990,000
Less: Security guard's salary - 20% is personal expense (360,000×80%) (288,000)
Less: Cost of running electricity generator (450,000)
252,000
Total income 5,888,000

W-1:
Non-adjustable security deposit from Rashid 3,500,000
Less: Previous advance from Ahmed charged to tax [(3,500,000÷10)×3] (1,050,000)
2,450,000

103 | P a g e
CHAPTER 04
INCOME FROM CAPITAL GAIN
Definition: Gain arise due to Disposal of Capital assets is known as capital gain
Gain:
Gain = Consideration received - cost of assets

Actual Selling Price or Fair Market


Whichever is higher
Purchase Price of Asset XXX
Incidental expense for acquiring XXX
Asset
Expenses incurred in disposal of XXX
Asset
Total XXX

Mr. Agha Majid purchased Painting costing Rs.400 and also paid commission on purchase of Rs.50.
After few days, Painting sold for Rs.1,120. Commission paid on sale amounted to Rs20. Calculate
gain/loss on disposal?
Consideration =A = 1,120
Cost =B (400 + 50 +20 = 470)
Gain = A -B
Gain =1,120 -470=650

Income from Capital Gain will be taxed on Accrual Basis


The gain is taxable in the year in which it is earned and not the year in which it is received. It means if we sell
the shares in TY 2016 and receive cash in TY 2017, income will be taxed in 2016
Disposal [D.E.S.E.R.T S.T.A. R L.C.D]
Disposal includes

1 Sold 2 Exchanged

3 Transferred 4 Distributed

5 Cancelled 6 Redeemed

7 Relinquished 8 Destroyed

9 Lost 10 Expired

104 | P a g e
11 Surrendered. 12 The transmission of an asset by succession or
under a will

13 The application of a business asset to


personal use or vice versa

Capital Assets [S.O.D.A]


“Capital asset" means property of any kind held by a person, whether or not connected with a business, but
does not include
• S = Stock-in-trade, consumable stores or raw materials held for the purpose of business;
• O = Other movable Property held for personal use by the person or his family dependent.
• D= Depreciable Assets for which depreciation is charged
• A= Amortization deduction is allowed under the head income from Business
Example of capital assets:

• Shares of a private company. Private Company is a company that does not fall under the
definition of Public company.
• Share of member in partnership firm
• A painting, sculpture, drawing or other work of art, jewelry, a rare manuscript, folio or book, a
postage
• Stamp or first day cover, a coin or medallion; or an antique held for personal use.

Income From Capital Gains

A) Normal Tax Regime B) Separate Block

Gain = A-B

105 | P a g e
Gain = Consideration received - cost of assets

Actual Selling Price or Fair Market


Purchase Price of Asset +
Whichever is higher.
Incidental expense for acquiring Asset +

Expenses incurred in disposal of Asset.


U/S 37 & 38
➢ If dispose = Gain = Totally Taxable
➢ Loss can be incurred and adjusted in U/S 37 [ discuss later]
➢ No loss can be incurred in U/S 38 [ If incurred then no adjustment will be
made]

Example of capital assets:


Category 1:

• Shares of a private company. Private Company is a company that does not fall under the
definition of Public company.
• Share of member in partnership firm
Category 2:
A painting, sculpture, drawing or other work of art, jewelry, a rare manuscript, folio or book, a postage
Stamp or first day cover, a coin or medallion; or an antique held for personal use.
Note: These categories are only prepared for better understanding of students. They should not use the
"Category" word in paper.
Tax treatment of above mentioned categories

Gain loss

Category 1 Gain is income Loss is allowed as deduction

Category 2 Gain is income Loss is not allowed as


deduction

Exempt asset Gain is not income Loss is not allowed as


deduction

Brain Teaser (Solve the Question and feel the Loss

Mr. Abdullah purchases a Necklace for his wife on their 1st month anniversary. He paid Rs. 70,000 for
gold and paid 3,000 extra to the goldsmith to engrave the name of his wife on the necklace. After 6 month
of his marriage his wife left him due to his ever lasting engagement with CA-Final.
Later he got broke and decided to sell that necklace to his cousin for Rs. 75,000.
Required: Calculate his Gain/ Loss under relevant head of income.

106 | P a g e
Immovable Property
• Disposed/ Sold
• Separate Block of Income
• Personal Property
• Depreciable Property Builders, Developer, will be a part of business

Gain = A-B

A gain arising on the disposal of immoveable property (open plot or constructed property or flat) situated in
Pakistan by a person in a tax year shall be chargeable to tax as separate block on the basis of holding period
as under:

107 | P a g e
(These rates will be given in exam don’t need to memorize them)
S.NO Holding period Open plots Constructed flats
property
(1) (2) (3) (4) (5)

1. Where the holding period does not 15% 15% 15%


exceed one year

2. Where the holding period exceeds 12.5% 10% 7.5%


one year but does

not exceed two years

3. Where the holding period exceeds 10% 7.5% 0


two year but does

not exceed three years

4. Where the holding period exceeds 7.5% 5% -


three years but does

not exceed four years

5. Where the holding period exceeds 5% 0 -


four years but does

not exceed five years

6. Where the holding period exceeds 2.5% - -


five years but does

not exceed six years

7. Where the holding period exceeds 0% - -


six years.

Purchase of immoveable property in cash [Sec 75A]


If immovable property having fair market value (FBR value or DC rate whichever is applicable) greater than
Rs. 5 million is purchased in cash, then it will have the following implications:
i. Such amount shall not be treated as cost for computation of any gain on disposal
ii. Such person shall pay a penalty of 5% of the fair market value (paid in cash).

108 | P a g e
Example
Mr. B purchased an open plot on 22.09.2023 which cost him Rs.2,000,000. The plot was sold on 25.03.2024 at
Rs.8,000,000. Another constructed property was acquired on 05.09.2023 at Rs.6,000,000 and sold at
Rs.12,000,000 on 23.05.2024. Assuming that both properties were acquired and sold as per value notified by
the Board, the capital gain and tax thereon will be calculated as under
Answer
As holding period of both plot and constructed property is upto one year, therefore 100% gain will be taxable
@ 15%
Total capital gain Rs.6m+6m = 12,000,000
Tax liability @ 15% = 1,800,000
As total capital gain is more than 10 million but less than 15 million, it will be taxed at 10% and tax payable
will be Rs.120,000

Exercise
Mr. Y purchased an open plot on 22.05.2022 at a cost of Rs.4,000,000. The plot is sold on 25.06.2024 at
Rs.7,000,000. Another constructed property is acquired on 08.09.2022 at Rs.9,000,000 and sold at Rs.
14,000,000 on 25.06.2024. Assuming that both properties were acquired and sold as per value notified by the
Board, the capital gain and tax thereon is calculated as under:-
Answer
Gain on sale of plot = 7,000,000 - 4,000,000 = Rs.3,000,000
As the holding period of plot is more than two but less than 3 years, it will be taxable @ 10%. Gain on sale of
constructed property = 14,000,000 - 9,000,000 = Rs.5,000,000
As the holding period of the constructed property is more than one but less than 2 years, it will be taxable @
10%.
Total capital gain = Rs.3,000,000 + Rs.5,000,000 = Rs.8,000,000
Tax liability @ 10% = 800,000

Example
On 15 January 2021, Mr. A sold a house situated in Karachi for Rs. 15,000,000: He had purchased this house
in July 2020 for Rs. 19,000,000 out of which Rs. 6,000,000 was paid in cash.
Answer
Cash exceeding Rs.5 million will not be considered as cost.
Therefore, gain will be calculated as follows:
Consideration =A = 15,000,000
Cost =B = 13,000,000
Gain =A-B = 2,000,000

Further Mr. A will also have to pay penalty of Rs. 6,000,000 x 5%= Rs. 300,000.

The following table sets out the application of this section:


Questions Solutions

Property Details Total Cost of Amount Paid in Cost as per ITO, Penalty @5% of
property bought Cash 2001. cash payment
Rs. Rs. Rs. Rs.

Property 1 4,000,000 1,000,000 4,000,000 0

Property 2 3,500,000 2,000,000 3,500,000 0

109 | P a g e
Property 3 95,000,000 3,000,000 92,000,000 150,000

Property 4 103,000,000 12,000,000 91,000,000 600,000


Exemption on Capital gain

Capital gain is exempt where person is dependent of Shaheed belonging to Pakistan Armed Forces (PAFs) or
a person who dies during service of PAFs or Federal or Provincial Governments.

Taxation at Reduced Rate


Capital Gain Tax rates have been reduced by 50% in case of the first sale of immovable property acquired or
allotted to ex-servicemen and serving personnel of Armed Forces or ex-employees or serving personnel of
Federal and Provincial Governments, being original allottees of the immovable property, duly certified by the
allotment authority.
Moreover, in case of any capital gain arising after completion of 3 years from date of acquisition of
immoveable property the amount of tax payable for above persons shall be reduced by 75%.

Example
Mr. Daniyal has disposed of an immovable property (Open Plot) after holding it for a period of 2.5 years.
It was purchased for Rs.100,000 on 1 September 2016 and is sold for Rs. 150,000.
Calculate tax payable?
Solution
Gain (150,000- 100,000) = Rs.50, 000
Tax liability 50,000 x 10% = Rs.5, 000

Example
Mr Shoaib disposed the following asset during the tax year

Assets Date of Date of sale cost Sale proceeds


purchase

Shares of AB private company 1.1.18 31.5.2022 40,000 70,000


Shares of DE private company 13.3.11 15.6.2022 20,000 5,000
Shares of D Public Unlisted 1.1.08 30.4.2022 30,000 20,000
Company
Shares of N Public Unlisted 1.3.11 26.2.2022 60,000 200,000
Company
Jewelry of his wife 13.7.07 15.8.2022 10,000 80,000
Painting 13.4.07 14.4.2022 90,000 20,000
Exempt asset 1 13.4.07 14.4.2022 20,000 30,000
Exempt asset 2 13.4.07 14.4.2022 25,000 5,000
Coin 13.4.07 14.4.2022 200,000 600,000
Car of son 13.4.07 14.4.2022 300,000 600,000

110 | P a g e
Home furniture 15.5.07 14.5.2022 3,000 1,500
He has also informed you that he has earned income from business of Rs. 900,000 during the year.
Calculate his tax liability?

Solution
Mr.Shoaib
Income and tax thereon
TY 2022
Income from capital gain (W-1) 615,000
Income from business 900,000
Total/taxable income 1,515,000

Tax liability (table1) (75,000+20% *315,000) 138,000


(W-1) income from capital gain
AB private company (70,000-40,000) 30,0000
DE Private company (5000-20,000) (15,000)
D Public unlisted company (20,000-30,000) (10,000)
N Public Unlisted company (200,000-60,000) 140,000
Jewelry of his wife (80,000-10,000) 70,000
Painting (loss is not allowed as deduction) -
Exempt asset 1 -
Exempt asset 2 -
Disposal of coin (600,000-200,000) 400,000
Disposal of car of son (not a capital asset) -
Disposal of home furniture (not a capital asset) -
615,000

Tax on deemed income [Section 7E]

A resident person owning capital assets in Pakistan will be taxed on deemed income arising from capital
assets (for tax year 2022 and onwards).
A definition of Capital asset' has been provided, which effectively means that such tax is leviable only in
respect of immovable property' (e.g. house, any building, manufacturing plant etc.) situated in Pakistan owned
by resident persons.

111 | P a g e
Deemed income shall be computed as 5% of the fair market value (as determined by the FBR under section
68 i.e. FBR Value or DC Rate) of capital assets (house, building etc.) situated in Pakistan held on the last day
of the tax year. The rate of tax will be 20%. [in other way you can say 1% of the FMV will be the tax]

The following definition of capital asset already discussed


“Capital asset" means property of any kind held by a person, whether or not connected with a business, but
does not include [S.O.D.A]
• S = Stock-in-trade, consumable stores or raw materials held for the purpose of business;
• O = Other movable Property held for personal use by the person or his family dependent.
• D= Depreciable Assets for which depreciation is charged
• A= Amortization deduction is allowed under the head income from Business

The following properties are also excluded, namely: -


a) One capital asset (immoveable property) owned by the resident person,
b) Self-owned business premises from where the business is carried out by the person and is appearing
on the active taxpayers' list at any time during the year;
c) Self-owned agriculture land where agriculture activity is carried out by person excluding farmhouse
and land annexed thereto;
"Farmhouse" means a house constructed on
• A total minimum area of 2,000 square yards
• With a minimum covered area of 5,000 square feet
• Used as a single dwelling unit.
d) capital asset (immoveable property) allotted to-
i. A Shaheed or dependents of a shaheed belonging to Pakistan Armed Forces;
ii. Following persons of Pakistan Armed Forces or Federal or Provincial Government:
• A person or dependents of the person who dies while in service
• War wounded person while in service
• An ex-serviceman/ ex-employee or serving personnel, (original allottees of the capital asset);
e) Any property from which income is chargeable to tax (under the Ordinance) and tax has been paid on
it;
f) Capital asset in the first tax year of acquisition where tax (under section 236K) has been paid;
g) Where the fair market value of the capital assets in aggregate (excluding the capital assets mentioned
in clauses (a)-(f)) does not exceed Rs. 25,000,000;
h) Capital assets owned by a provincial government or a local government;
i) Capital assets owned by a local authority, a development authority, builders and developers for land
development and construction.
Provided that the exclusions mentioned at clauses (a), (e), (D), and (g) of this sub-section shall not apply in
case of a person not appearing in the active taxpayers' list, other than persons covered in rule 2 of the Tenth
Schedule.
If the tax liability under section 7E is not discharged, then the registrar (or the person registering the transfer)
is required not to register the transfer of the subject property.

112 | P a g e
Example:
Miss Sonam Bajwa is doing a wholesale business and registered for income tax and sales tax purposes. Her
name is appearing on the active taxpayers' list.
Sonam has the following immovable properties in her own name on 30.6.20X8:

S# Assets FMV Rs.


1 House in DHA, Phase 8 Lahore at 500 square yards purchased in 20X2 being used 35,000,000
by Sonam Bajwa as her main residence
2 House in Bahria Town at 600 square yards purchased in 20X4 being used by 18,000,000
Sonam Bajwa occasionally with her sister katrina Kaif
3 Farmhouse in Muridkee at 5,000 square yards purchased last year being used by 15,000,000
Sonam Bajwa as her recreational activities
4 Godown in kasur, Karachi being used to store stock-in-trade 21,000,000
5 Imported machinery to be used for manufacturing of gloves as Sonam Bajwa is 45,000,000
also planning to start manufacturing activities as well
6 Open plot of 500 square yards in DHA, Islamabad purchased last year. Sonam 12,000,000
Bajwa intends to gift this plot to her Husband on his birthday
7 Flat of 2,500 square feet in Bahria Town Phase 8 Rawalpindi 2,000,000
This flat is given on rent by Sonam Bajwa at a monthly rent of Rs. 125,000 and
declared rental income after allowable deductions in her return of income
8 Flat in Gulistan e Jauhar, Karachi of 1,800 square feet purchased on 3,600,000
30.5.20X7. Sonam Bajwa intends to let out this flat but still vacant till 30.6.20X8
9 Open plot of 1,000 square yards in DHA, Lahore purchased on 25.6.20X8 and 27,000,000
paid withholding tax at the time of purchase
Sonam Bajwa intends to transfer this plot as gift to her husband Farrukh on the
occasion of his marriage on 5.7.20X8
Required:
Calculate the amount of tax to be paid on deemed income on capital assets under section 7E of The
Income Tax Ordinance, 2001.

Answer:
The Properties listed at serial number 1,4,5,7 and 9 do not fall within the ambit of deemed income
under section 7E:

FMV of capital assets for the purpose of deemed income and tax thereon:

S# Assets FMV
2 House in Bahria Town 18,000,000
3 Farmhouse in Muridkee 15,000,000
6 Open plot of 500 square yards in DHA, Islamabad 12,000,000
8 Flat in Gulistan e Jauhar, Karachi 3,600,000
Total FMV 48,600,000

113 | P a g e
Deemed income @ 5% of Rs.48,600,000 2,430,000
Income Tax on Deemed Income @ 20% 486,000
Or Alternatively
[FMV x 1% (48,600,000 x 1% = 486.000)]

Securities U/S 37-A (USIMDVD)


Includes:
• S= Shares of Public company
• I= Instrument of redeemable capital
• M= Modaraba certificates
• D= Debt securities
• V= Voucher of PTCL
• D= Derivative products
• U= Units of exchange traded fund
Note: Units of mutual fund also fall under the definition of securities
Bonus shares
Includes bonus units in a unit trust.

Derivative Products
"Derivative products" means a financial product which derives its value from underlying security, May be
traded on stock exchange and includes and includes
• deliverable future contracts,
• cash settled future contracts,
• contracts of rights
• Options
• Future commodity contracts entered into by the members of Pakistan Mercantile Exchange

Debt securities means


Corporate Debt Securities such as Term Finance Certificates (TFCs), Sukuk Certificates (Sharia Compliant
Bonds), Registered Bonds, Commercial Papers, Participation Term Certificates (PTCS) and all kinds of debt
instruments issued by any Pakistani or foreign company registered in Pakistan; and

Government Debt Securities such as Treasury Bills (T-Bills), Federal Investment Bonds (PIBS), Foreign
Currency Bonds, Government Papers, Municipal Bonds, Infrastructure Bonds and all kinds of debt
instruments issued by Federal Government, Provincial Governments.

Capital gain on disposal of securities [37A]


The capital gain from disposal of securities (other than gain that is exempt from tax) shall be chargeable to tax
at prescribed rates.
Provisions of section 37A shall not apply to the following persons:
• a banking company,
• an insurance company
Gain = A-B

Gain = Consideration received - cost of assets

Actual Selling Price or Fair Market Purchase Price of Securities


Whichever is higher Incidental expense for acquiring Securities

Expenses incurred in disposal of Securities 114 | P a g e


Rates of tax for securities (2024 and onwards)
Holding Period Securities acquired on Securities
or before 30.06.2022
acquired on or
(Between 1.7.13
after 01.07.2022
30.6.22)

1. Where the holding period does not exceed 15%


one year

2. Where the holding period exceeds one year 12.5%


but does exceed two years

3. Where the holding period exceeds two years 12.5% 10%


but does not exceed three years
[Irrespective of the
holding period]
4. Where the holding period exceeds three years 7.5%
but does not exceed four years

5. Where the holding period exceeds four years 5%


but does not exceed five years

6. Where the holding period exceeds five years 2.50%


but does not exceed six years

7. Where the holding period exceeds six years 0%

8. Future commodity contracts entered into by 5%


members of Pakistan Mercantile Exchange

The rate of 0% tax shall be charged on capital gain arising on disposal where the securities are acquired before
the first day of July, 2013.

✓ For the purpose of this section, shares of public company" shall be considered as security if such
company is a public company at the time of disposal of such shares.
✓ Gain on securities shall be treated as a separate block of income.
✓ The holding period of a security shall be counted from the date of acquisition to the date of disposal.
✓ Loss on disposal of securities in current year shall be setoff only against the gain from any other
securities. Loss that has not been set off in current year can be carried forward to the subsequent 3 tax
years from the tax year in which loss was first computed. However, loss can be adjusted only against
the gain from disposal of securities.

Market-Based Transaction and Exception to U’S 37-A


Normal tax liability as applicable under section 37 shall apply on disposal of following:
i. Shares of a listed company made otherwise than through registered stock exchange and which are not
furnished to NCCPL for computation of capital gains and tax thereon under section 37A.

115 | P a g e
ii. Shares through initial public offer during listing process except where the detail of such disposal is
settled through NCCPL; for computation of capital gains and tax thereon under section 37A.
In case of a Market-Based Transaction involving any security, a notional expense equal to 0.5% of the sale
proceeds and 0.5% of the security's cost will be applied instead of actual charges like brokerage, commission,
levy, and other related incidental expenses. This notional deduction, however, does not apply to open-ended
mutual funds' units or future contracts initiated by PMEX members.
For clarification, a "Market-Based Transaction" refers to a transaction conducted at a registered stock
exchange in Pakistan or on the platform of the National Clearing Company of Pakistan Ltd (NCCPL).
Question
Mr. Imtiaz has disposed off the following assets in TY 2025:

Assets Date of purchase Date of sale Cost Sale proceeds

Shares of AB Private Company 1.1.2020 1.1.2025 300,000 900,000

Shares of GE Unlisted 1.1.2021 31.3.2025 30,000 10,000

*Shares of R listed company 18.7.2022 15.10.2024 60,000 75,000

*Shares of P listed Company 20.5.2024 25.10.2024 35,000 40,000

*The disposal is made through registered stock exchange and is settled through NCCPL.

He has also informed you that he has earned income from business of Rs. 1,000,000 during the year.
Calculate his tax liability?

Answer
Income from business 1,000,000
Income from capital gain (W-1) 580,000
Income from capital gain - Securities (separate block) (W-2) 18,950
Total income 1,598,950

Less: Income from capital gain - Securities (separate block) 1,580,000


Taxable income-taxable under NTR (18,950)

Income from capital gain


AB Private Company (900,000-300,000) 600,000
GE unlisted Company (10,000-30,000) (20,000)
580,000
(W-2)
Income from capital gain - Separate block
R Co. (75,000-60,000)-0.5% of 135,000 14,325
P Co. (40,000-35,000) - 0.5% of 75,000 4,625

116 | P a g e
FIFO or weighted average
Gain on security shall be computed on the basis of First in First out (FIFO) method.
If shares are sold on same day, average method will apply.

Capital loss adjustment disallowed (Rule 13 F)


Capital loss adjustment shall not be admissible in the following cases, namely
1. Wash Sale
If an investor sustains loss on disposal of a security and in one month's period he or his related party purchases
the same security, thus maintaining his portfolio.
2. Cross Trade
Where coordinated reshuffle of securities between two related accounts of the same investor (or between two
related brokerage houses) is undertaken to artificially realize capital losses without selling the securities to an
outsider.
3. Tax Swap Sale
In this case investor who realized loss does not repurchase the same security. In this case he chooses another
similar security in same sector. Through this he has eliminated tax liability and has also maintained the
portfolio at the same risk return profile.

Payment of advance tax on capital gain


Every investor other than individual shall e-file statement of advance tax on capital gain within 7 days after
the end of each quarter and will pay the tax.
Note
If the sole business of a company is to manage its investment in different subsidiaries then income of
company would be:
• dividend income and
• Capital gains.
Such income of the company would be chargeable to tax under the head income from business.

Non-Recognition Rule
In the following modes of transfer of a capital asset no capital gain or loss shall arise where the recipient is a
resident in Pakistan in the relevant tax year
I. Transfer of assets between spouses under an agreement to live apart
II. Under a gift from relative, bequest or will
III. By succession, inheritance or devolution
IV. A distribution of assets on dissolution of an AOP or on liquidation of a company

“Relative” in addition to an individual means: -


An ancestor, a descendant of any of the grandparents, or an adopted child, of the individual, or of a spouse of
the individual; or
A spouse of the individual or of any person specified above

Deemed cost for the recipient


The cost of capital asset in the hands of transferor shall be treated the cost for the recipient
Taxability in the hand of recipient
✓ Transfer of assets under and agreement to live apart is exempt in the hands of recipient
✓ Transfer of assets under inheritance is a capital receipt and therefore not taxable in the hands of recipient

Employee share schemes


Where shares are issued to an employee as a result of employee share scheme, the cost of the shares to the
employee shall be the sum of-

117 | P a g e
(a) The consideration, if any, given by the employee for the shares;
(b) The consideration, if any, given by the employee for the grant of any right or option to acquire shares; and
(c) The amount chargeable to tax under the head "Salary" under those sub-sections.

Income from Capital Gains (Summary)


Sr. Taxable
No Particulars Rs. Rs
A Capital Gain under NTR
[(FMV/ Sale Price/ Amount Received) – (Cost of Asset + Incidental Expenses +
Further Charges)]= Gain / Loss On Disposal
Gain U/S 37
• Disposal of Pvt. Company Shares X
• Disposal of Membership card X
• Disposal of share in partnership firm X
• Disposal of Mining Rights X

Gain U/S 38
• Disposal of painting, sculpture, drawing, or other art work X
• Disposal of Jewellery X
• Disposal of A postage stamp X
• Disposal of a coin or medallion X
• Disposal of Antique X
Note:
No loss will be recognized on disposal of these assets U/S 38

B Capital Gain Under SBOI


[(FMV/ Sale Price/ Amount Received) – (Cost of Asset + Incidental Expenses +
Further Charges)]= Gain / Loss On Disposal

Gain U/S 37-A X


• Disposal of shares of Public Company X
• Disposal of Vouchers of PTCL X
• Disposal of Modarba Certificates X
• Disposal of Redeemable capital instruments X
• Disposal of Debt Securities X
• Disposal of Derivative Product X
Disposal of Immovable Property

FMV/ Sale proceeds ( Higher of ) X


Less : Cost of the Property ( Land + Development Expense) (X) X
Note:
The Gain on Disposal of Property shall be taxed under SBOI
• The Rate will be decided on the basis of type of property

Total Income From Capital Gain ( Including SBOI ) XXX

118 | P a g e
PAST PAPER QUESTIONS
Question # 1 Spring 2010 Q. 6

(a) Explain the term “Capital Assets” as referred to in the Income Tax Ordinance, 2001. (05)
(b) Mr. Shahbaz, a resident individual earned Rs. 700,000 from the sale of assets as shown below:

Purchase Sale Gain/loss (Rs.)


Price Price
Date Rupees Date Rupees

Shares of a listed company 10/12/21 350,000 30/06/23 200,000 (150,000)


Shares of an unlisted 15/07/21 500,000 30/11/22 900,000 400,000
company
Jewellery 15/05/21 750,000 20/12/22 1,400,000 650,000
Sculpture 01/07/21 400,000 31/01/23 300,000 (100,000)
Shares of a private limited 01/01/23 1,300,000 15/02/23 1,200,000 (100,000)
company

Required:
Discuss the treatment and the implications of each of the above transactions under the Income Tax
Ordinance, 2001. Give brief reasons to support your conclusion. (05)

Question # 2 Autumn 2011 Q. 5

Mr. Feroz has been the CEO of Aziz Foods Pakistan Limited (AFPL) for several years. He was given 2000
shares on 1 June 2009 by Aziz AG, Germany (the parent company of AFPL) at a price of €2.5 per share. The
market price on that date was €8.2 per share. The shares were transferable on completion of one year of service,
from the date of issue of shares.

The market price of the shares as on 1 June 2010 was €12.5 per share. On 10 April 2011, Mr. Feroz sold all
shares at €13 per share. He paid a commission of €50 to the brokerage house.
The relevant exchange rates are as follows:
1 June 2009 €1 = Rs. 118.10
1 June 2010 €1 = Rs. 121.40
10 April 2011 €1 = Rs. 123.90

Required:
Compute the amount to be included in the taxable income of Mr. Feroz for tax years 2009, 2010 and 2011 and
specify the head of income under which the income would be classified. (07)

Question # 3 Autumn 2012 Q. 5(a)

In May 2023, Ms. Hameeda sold certain personal assets at the following prices:

Rupees

Plot in DHA Karachi 10,000,000

Paintings 2,000,000

Jewellery 5,000,000

119 | P a g e
Additional information:
Plot in DHA Karachi was inherited by her from her father in May 2012. It was purchased by her father for Rs.
4,000,000 and market value at the time of inheritance was Rs. 5,000,000.

Paintings were inherited from her mother in July 2017. These paintings were purchased by her mother for Rs.
2,350,000 and market value at the time of inheritance was Rs.4,000,000.

Jewellery costing Rs. 3,000,000 was purchased and gifted to her by her husband in March 2015.

Required:
Discuss the taxability of Ms. Hameeda in respect of the above gains/ losses on sale of assets in the context of
Income Tax Ordinance, 2001. (06)

Question # 4 Autumn 2013 Q. 3

(a) Explain the term ‘Capital asset’ as referred to in the Income Tax Ordinance, 2001. (04)
(b) Gulzar is a Pakistani resident and operates various businesses. He disposed of the following assets during
the tax year 2023:
(i) An immovable property was sold for Rs. 50 million. The cost of the immovable property was Rs. 25
million. Tax depreciation of Rs. 4 million had been allowed on the immovable property up to the tax
year 2022. (02)
(ii) A car was disposed of for Rs.1.2 million. The car was acquired on 1 July 2021. The tax written down
value of the car at the beginning of tax year 2023 was Rs. 0.9 million. The car was being used partly
(70%) for business purposes. (02)
(iii) An antique sculpture was purchased for Rs. 350,000 on 30 August 2010. It was sold for Rs. 1,500,000
on 28 February 2023 through auction. The auctioneer was paid a commission of Rs. 150,000. Tax
was deducted and paid by Gulzar from the amount of commission within due date. (02)
(iv) Listed securities were sold as follows:

Securities Date of Purchase Purchase cost (Rs.) Date of sale Sale proceeds (Rs.)

A 20 November 2022 500,000 17 March 2023 400,000


B 05 August 2022 320,000 08 June 2023 600,000
C 01 June 2022 650,000 17 June 2023 700,000
(03)
Required:
Compute the amount of capital gain/loss arising on the above transactions under the provisions of the
Income Tax Ordinance, 2001.

Question # 5 Autumn 2014 Q. 3

Zaman is working as the Chief Executive Officer in Yasir Limited (YL). Following are the details of sale and
purchase relating to his capital assets during the tax year 2023.
1. Under an employee share scheme, 25,000 shares of YL were allotted to Zaman, on 1 December 2020 for
Rs. 25 each. According to the scheme, he was not allowed to sell/transfer the shares before completion of
two years from the date of transfer. The face value of each share is Rs. 10 per share. Fair market value of
the shares was as follows:
▪ Rs. 40 per share on 1 December 2020
▪ Rs. 48 per share on 30 June 2021
▪ Rs. 55 per share on 30 November 2022
▪ Rs. 61 per share on 30 June 2023
2. He sold 24,000 shares of HQ (Pvt.) Limited on 30 June 2023 for Rs. 200 per share. He had acquired these
shares as follows:

120 | P a g e
▪ 18,000 shares were purchased at Rs. 55 per share on 25 June 2022.
▪ 6,000 shares were allotted as bonus shares on 28 February 2023
3. A gain of Rs. 300,000 was realized on the sale of shares of Zeeshan Industries Limited (ZIL), a public
listed company, in June 2023. The shares were acquired on 31 May 2022.
4. Zaman sold a painting to his brother on 23 March 2023 for Rs. 1,800,000. Zaman had purchased this
painting for his residence, in an auction for Rs. 2,000,000 on 10 July 2020
5. He sold his old furniture to Furqan for Rs. 285,000 on 25 June 2023. The furniture was purchased in 2021
for Rs. 250,000.

Required:
Compute the amount to be included in the taxable income of Zaman for the tax year 2023 and specify the head
of income under which the income would be classified. (10)

Question # 6 Autumn 2015 Q. 4

(a) What do you understand by the terms ‘Security’ and ‘Derivative products’ as provided in the Income Tax
Ordinance, 2001 and Rules made thereunder? (03)

(b) Under the provisions of the Income Tax Ordinance, 2001 compute taxable gain or loss, under the correct
head of income, in each of the following cases. Also identify, giving reasons, whether the company is a
public or private company for tax purposes:
(i) Ashiq has 5,000 shares in Rumi (Pvt.) Limited (RPL). 52% of the shares of RPL are held by Delta
Plc. which is owned by the British Government. Ashiq inherited these shares from his father on 1
January 2014. His father had purchased these shares on 31 May 2012 at a price of Rs. 250 per share.
The market value of these shares at the time of inheritance was Rs. 300 per share.

On 30 June 2015 Ashiq sold 2,500 shares in RPL at a price of Rs. 325 per share when the break-up
value of RPL was Rs. 350 per share. (04)

(ii) What would be your answer in (i) above, if 40% of the shares of RPL were held by the Provincial
Government, 48% by the British Government and 12% by individual investors. (03)

Question # 7 Spring 2017 Q. 2(b)

Saleha is a resident person. She disposed of the following assets during the tax year 20X7.
(i) A painting which she inherited from her father was sold for Rs. 1,250,000. The market value of the
painting at the time of inheritance was Rs. 1,550,000. The painting was purchased by her father for Rs.
1,000,000. (02)
(ii) She sold jewellery for Rs. 2,300,000 which was purchased by her husband in March 20X5 for Rs.
1,300,000 and gifted to her on the same date. (02)
(iii) She disposed of her car for Rs. 1,800,000. The car was being used for the purposes of her business. The
tax written down value of the car at the beginning of tax year 20X7 was Rs. 1,600,000. The rate of
depreciation for tax purposes is 20%. (02)
(iv) On 20 October 20X6 she sold a dining table to Faheem for Rs. 18,000 which she had purchased on 15
May 20X5 for Rs. 15,000 for her personal use. (02)

Required:
Under the provisions of the Income Tax Ordinance, 2001, discuss the taxability of each of the above
transactions in the context of capital gain/loss.

Question # 8 Spring 2017 Q. 3(b)

Najam had purchased a house in 20X2 for Rs. 20 million.

121 | P a g e
On 1 July 20X6, Najam entered into an agreement with Zameer for sale of the house for Rs. 25 million. As per
the terms of the agreement, Najam received Rs. 5 million on the day the contract was signed and balance amount
was to be paid on 30 September 20X6. However, due to financial difficulties, Zameer failed to pay the balance
amount on the due date and consequently, Najam forfeited the advance in accordance with the terms of the
agreement.

On 15 February 20X7 Najam sold the house to Farid for Rs. 30 million.

Required:
Advise Najam about the taxability of the above transaction under the Income Tax Ordinance, 2001.
(04)

Question # 9 Autumn 2017 Q. 3(b)

Under the provisions of the Income Tax Ordinance, 2001 compute taxable income or loss, under the correct
head of income for tax year 2017, in each of the following cases:
(b) Zaheer sold a painting to his brother on 10 April 2017 for Rs. 2,000,000. Zaheer had purchased this
painting for his residence, in an auction on 14 August 2013 for Rs. 1,800,000. (02)

Question # 10 Autumn 2019 Q. 3(b)

Respond to the following independent scenarios, under the provisions of the Income Tax Ordinance, 2001:
(b) Haris sold two of his personal vehicles during the current year and earned profit of Rs. 550,000. Discuss
the taxability of profit earned by Haris in the context of capital gain/loss. (02)

Question # 11 Spring 2020 Q. 4(b)

Respond to the following independent situations, under the provisions of the Income Tax Ordinance, 2001:
(b) On 1 July 2014, Ahmed purchased two sculptures for Rs. 410,000 and Rs. 475,000 respectively. On 30
November 2019, during the shifting of his house, he lost both the sculptures. On 15 January 2020, he
received insurance claim of Rs. 940,000 in a single transaction against the loss of two sculptures. The fair
market value of both the sculptures at the time of loss was estimated at Rs. 360,000 and Rs. 540,000
respectively. Compute Ahmed’s taxable income or loss for the above transaction. (04)

Question # 12 Autumn 2022 Q. 3

(a) Nargis is a resident filer. During the tax year 2022, she disposed of various assets. Relevant details of
these assets are as follows

Disposal Purchase
Cash Fair market Date of Cost of Date of
consideration value disposal purchase purchase
---- Rs. in million ---- Rs. in million
Investment in shares
of a public unlisted 2.8 3.0 01-Apr-22 2.0 01-Jun-20
company
Investment in shares of 3.5 3.5 01-Jul-21 2.1 30-Jun-13
a listed company
Personal car 5.0 6.0 31-Dec-21 3.8 01-Jan-19
Painting 1.2 1.2 16-Sep-21 1.7 16-Feb-17
Jewelry 8.0 7.6 30-Jun-22 (see note 1) 01-May-16

Note 1: She received the jewelry as a gift from her mother in law at the time of her marriage when its fair
market value was Rs. 4.8 million.

122 | P a g e
Required:
Under the provisions of the Income Tax Ordinance, 2001 and Rules made thereunder:
(i) compute the amount to be chargeable to tax under the head of capital gain. Also state the reason for
ignoring gain / loss, if any. (06)
(ii) compute the tax liability of Nargis in respect of the capital gain computed in
(i) above assuming that she has no other source of income. (02)

(b) Shahid is a resident filer and has provided following information pertaining to tax year 2022
(i) On 16 June 2018, he inherited a bungalow having a fair market value of Rs. 50 million from his
father on his death. On 1 January 2022, he decided to sell the bungalow to Zamin for Rs. 60 million
and received a deposit of Rs. 6 million. On 14 February 2022, he forfeited the deposit on refusal of
Zamin to purchase the bungalow in accordance with the terms of the contract.
On 31 March 2022, he sold and transferred the bungalow to Kazim for Rs. 54 million.
(ii) He owns a factory building at Faisalabad. On 1 July 2021, he let out this factory building along with
the plant and machinery at a monthly rent of Rs. 1 million. During the year, he incurred expenses of
Rs. 3.5 million on the repair and maintenance of the factory.
(iii) He owns an agricultural land in Punjab. On 1 January 2022, he rented out the agriculture land at an
annual rent of Rs. 4 million. The fair market value of the annual rent was Rs. 5 million.

Required:
Under the provisions of the Income Tax Ordinance, 2001 and Rules made thereunder, compute the total
and taxable income of Shahid under appropriate heads of income for the tax year 2022. Also compute his
tax liability for the tax year 2022. (07)
Note: Show all relevant exemptions, exclusions and disallowances.

123 | P a g e
ANSWERS
Answer # 1 Spring 2010 Q. 6

(a)
Capital asset means property of every kind held by a person, whether or not connected with a business,
but does not include:
▪ any stock-in-trade, consumable stores or raw materials held for the purpose of business;
▪ any property with respect to which the person is entitled to a depreciation deduction under section 22
or amortization deduction under section 24; or
▪ any movable property held for personal use by the person or any member of person’s family
dependent on the person excluding capital assets mentioned under section 38(5) i.e. painting, sculpture,
drawing or other work of art, jewellery, rare manuscript, folio, book, postage stamp, first day cover,
coin, medallion or an antique.
(b)
▪ This is a loss on sale of shares of a listed company sustained in tax year 2023. This can be set off
against the gain from any securities chargeable to tax in the tax year 2023. This can be carried forward
in the next three tax years.
▪ It is a taxable gain. Full amount will be taxable.
▪ It is a taxable gain. Full amount will be taxable.
▪ Loss from sales of sculpture is not allowed to be recognized.
▪ It is a capital loss and it can be set off against capital gains only. It can also be carried forward for
adjustment against capital gains under section 37 during the succeeding six tax years.

Answer # 2 Autumn 2011 Q. 5

Tax year 2009


Were shares issued to an employee under an employee share scheme are subject to a restriction on the transfer
of the share no amount shall be chargeable to tax to the employee under the head "salary "until employee has a
free right to transfer the shares. So, nothing is chargeable to tax In TY 2009

Tax Year 2010


Where share issued to an employee under an employee share scheme are subject to a restriction on the transfer
of the share the amount chargeable to tax to employee shall be the FMV of the shares at the time the employee
has a free right to transfer the shares or as reduced by any consideration given by employee for the shares.
Amount chargeable to tax is

FMV of shares when the employee have free right to (12.5 x 2000 x 303,5000
transfer 121.40)
(2.5 x 2000 x 118.10) (590,500)

Amount taxable under salary 2,444,500

Tax year 2011


The gain arising on disposal is chargeable under the head capital gain.
For consideration, if any, given by employee for the shares;
The amount charges able to tax under the head "salary

Consideration received (13*2,000*123.90) 3,221,400


Less: cost of shares (590500 +2444500) (3,035,000)
Less: commission to brokerage house (50 x 123.9) (6,195)
Amount chargeable under capital gain 180,205

124 | P a g e
Answer # 3 Autumn 2012 Q. 5(a)

Tax implications in respect of sale of assets by Ms. Hameeda are as under:


▪ Capital gain on disposal of immoveable properties where holding period exceeds 6 years is taxable @ 0%;
therefore, disposal of plot in DHA Karachi would not be taxable.
▪ Painting
The cost of the painting for Ms. Hameeda would be Rs. 2,350,000 i.e. the cost of her mother. However, no
loss can be recognized on such assets.
▪ Jewelry
The cost of jewelry for Ms. Hameeda would be Rs. 3,000,000 i.e. cost of her husband. The gain of Rs.
2,000,000 should be recognized.

Answer # 4 Autumn 2013 Q. 3

(a) Capital asset means property of every kind held by a person, whether or not connected with a business, but
does not include:
▪ any stock-in-trade, consumable stores or raw materials held for the purpose of business;
▪ any property with respect to which the person is entitled to a depreciation deduction under section 22
or amortization deduction under section 24; or
▪ any movable property held for personal use by the person or any member of person’s family
dependent on the person excluding capital assets mentioned under section 38(5) i.e. painting, sculpture,
drawing or other work of art, jewellery, rare manuscript, folio, book, postage stamp, first day cover,
coin, medallion or an antique.
(b)
(i) As depreciable asset is specifically excluded from definition of capital asset, so no capital gain loss
will arise.
(ii) As depreciable asset is specifically excluded from definition of capital asset, so no capital gain loss
will arise

(iii)
Consideration 1,500,000
Less: Cost
Purchase Price 350,000
Commission paid 150,000
(500,000)
Gain chargeable to tax 1,000,000
(iv)

Security A Rs

Sales Proceed 400,000


Less: Cost (500,000)

Loss on Disposal (100,000)

Security B

Sales Proceed 600,000

Less: Cost (320,000)

Gain/ (Loss) 280,000

125 | P a g e
Security C
Sale Proceed 700,000

Less : Cost (650,000)

Gain / (Loss) 50,000

Answer # 5 Autumn 2014 Q. 3

1) Income from Salary


Shares when restriction is released
FMV (55 * 25000) 1,375,000
Less : cost of shares (625,000) 750,000

2) Income from Capital Gain


Gain on 18,000 shares (18000 x (200 – 55) 2,610,000
Gain on 6000 shares (6000 x (200 – (990000/24000) 952,500 3,562,500
Total Taxable income 4,312,500

3) Gain arising on disposal of shares of ZIL public listed company is not included in taxable income because
for securities separate rates are provided under the law.
4) Loss arising on sale of painting is not allowed as deduction, therefore it is not included in taxable income
5) Personal furniture is not a capital asset as per the definition of capital asset, therefore it is ignored taxable

Examiner Comments:
This question consisted of five parts all of which pertained to transactions involving sale
and purchase of assets. The candidates were required to compute the amount to be included
in taxable income and to identify the head of income under which each gain would be
classified. The performance in each part is discussed below:
(a) Gain on disposal of shares acquired under employee share scheme was tested. Majority
of the students gave correct answer. However, some students classified the gain under
the head ‘Income from Capital Gains’ instead of ‘Income from Salary’.
(b) According to the scenario, 24000 shares had been sold. Out of these, 18000 were
acquired in cash whereas 6000 shares had been received as bonus shares. This part was
much easier and most of the students performed well. Some of the candidates assumed
the cost of bonus shares as the face value i.e. Rs. 10 per share.
(c) This was the easiest of the five situations and most of the candidates secured full marks
by specifying that gain on disposal of listed company’s shares after one year holding
period is exempt and hence the gain would not be included in taxable income.
(d) According to the given situation, a painting had been sold for a loss. A significant
number of candidates erred in concluding that the loss on disposal of painting can be
adjusted against gain on disposal of other capital assets whereas under Section 38(5) of
the Income Tax Ordinance, 2001 such loss is not adjustable under any head of income.
(e) Most of the candidates did well in this part as they knew that furniture being a moveable
asset and held for personal use does not fall under the definition of capital asset and

126 | P a g e
therefore no capital gain arises on its disposal.

Answer # 6 Autumn 2015 Q. 4

(a) Security
Security means share of a public company, voucher of Pakistan Telecommunication Corporation,
Modaraba Certificate, an instrument of redeemable capital, unit of exchange traded fund, debt securities
and derivative products.
Derivative Products
“Derivative products” means a financial product which derives its value from the underlying security or
other asset, may be traded on stock exchange of Pakistan and includes deliverable future contracts, cash
settled future contracts, contracts of rights and options.
(b)(i)
Computation of taxable gain or loss

Separate block of income Rupees

Consideration received (2500 x 350 ) 875,000

Less : Cost of securities (2500 x 300 ) (750,000)

Taxable gain on securities 125,000

In this case RPL will be treated as public company because 50% of the shares are held by foreign company
who is wholly owned by the foreign govt.
(ii)
Computation of taxable gain or loss

Capital Gains Rupees


Consideration received (2500 x 350 ) 875,000

Less : Cost of shares (2500 x 300 ) (750,000)

Taxable gain 125,000

In this case RPL will be treated as Private company because less than 50% of the shares are held by foreign
company

Examiner Comments:
(a) In this part of the question the students were required to define the terms “Security and
Derivative Products”. It was based on the requirements of Section 37A (3) of Income
Tax Ordinance, 2001 and Rule 13L of Income Tax Rules, 2002. Most of the students
were not aware of these terms and resorted to guesswork. Many candidates tried to
define a Bye- product instead of Derivative Product. Many candidates missed this part
altogether.
(b) This part required candidates to compute taxable gain/loss on disposal of shares under
correct head of income and also to identify giving reasons, the status of the company
i.e. private or public, under two scenarios. Most of the students correctly identified the
status of company in both the scenarios but many among them did not provide any
reasons. Other common mistakes were as follows:

127 | P a g e
▪ Consideration received on disposal of shares was taken as the price. Since break-up
value of the shares was more than the price, break-up value should have been
considered as the consideration.
▪ In case of (i) only few candidates specified that the gain is taxable as ‘separate block
of income’. In case of (ii), vast number of students restricted their answer to the fact
that the company in question is a ‘private company’. They did not compute the taxable
gain under this scenario. Many candidates calculated amount of tax also, which was
not required.

Marking Scheme:
(a) 1.5 marks for each definition 3.0
(b) (i)
▪ Identification of the correct head of income 1.0
▪ Identification of type of company, whether public or private 1.0
▪ Computation of taxable gain on sale of securities 2.0
(ii)
▪ Identification of the correct head of income 1.0
▪ Identification of type of company, whether public or private 1.0
▪ Computation of taxable gain on sale of securities 2.0

Answer # 7 Spring 2017 Q. 2(b)

(i) Since Saleha inherited paintings from her father, the fair market value of the painting on the date of its
acquisition/transfer would be treated to be its cost. Hence, cost of the painting would be Rs. 1,550,000 and
there is a loss of Rs. 300,000. But, according to the ITO-2001, no loss can be recognized on disposal of
painting.
(ii) The cost of the Jewellery would be Rs. 1,300,000 i.e. the value thereof at the time of gift. Therefore, the
gain of Rs. 1,000,000 should be recognized.
(iii) The car sold by Saleha was being used by her for business purposes and therefore depreciation was also
being charged on it. However, depreciable assets are specifically excluded from the definition of capital
assets. Therefore, no capital gain or loss would arise on the disposal of car.
(iv) No capital gain/loss will arise as any movable property held for personal use by the person is excluded
from the definition of capital assets.

Examiner Comments:
This part of the question required students to discuss the taxability of four different
transactions in the context of capital gain / loss as is covered in Section 37 and 38 of the
Income Tax Ordinance 2001. Each transaction was explained as a sub-part and comments
on each sub-part are given below:
i. Most of the students did not know that no loss can be recognized on disposal of certain
assets which include paintings. Moreover, many candidates treated the amount at
which the painting was purchased by Saleha’s father as the cost, instead of the fair
market value of the painting on the date of inheritance.

128 | P a g e
ii. Most of the candidates were able to correctly compute gain on disposal of jewelry.
Moreover, some students were of the incorrect opinion that jewelry being a personal
asset, the gain would not be taxed.
iii. Most of the students were unable to explain that depreciable assets are specifically
excluded from the definition of capital assets, and therefore, no capital gain or loss
would arise on the disposal of car.
iv. The overall performance was good as most of the students were able to explain
correctly that moveable property held for personal use is excluded from the definition
of capital assets, and no capital gain/loss will arise.

Marking Scheme:
(i)
▪ Identification of the cost that should be taken into account for determining the gains /
loss on sale of painting 1.0
▪ The taxability of the gain/loss on sale of painting 1.0
(ii)
▪ Identification of the cost that should be taken into account for determining the gains /
loss on sale of jewellery 1.0
▪ The taxability of the gain/loss on sale of jewellery 1.0
(iii)
▪ Explanation for not inclusion of car as capital asset 1.0
▪ The taxability of the gain/loss on sale of car 1.0
(iv)
▪ Discussion on the rule for treatment of movable property held for personal use 1.0
▪ The taxability of the gain/loss on sale of dining table 1.0

Answer # 8 Spring 2017 Q. 3(b)

The amount of Rs. 5 million forfeited by Najam in accordance with the terms of the agreement for the sale of
his house to Zameer is to be treated as rent received.
Najam should recognize the gain of Rs. 10,000,000 (30,000,000 – 20,000,000) on disposal of the house to Farid
under the head ‘Capital Gain’.

Examiner Comments:
▪ Examinees did not know the meaning of concealed assets and resorted to guess work.
▪ Examinees were of the opinion that the Commissioner-IR is empowered to sell the
concealed assets in the auction and transfer the amount received to Government treasury.
▪ Examinees failed to acknowledge that while issuing the assessment order the
Commissioner, shall take into account the computation of taxable income and tax
payable for the last completed tax year of the person during which the concealed asset
was accounted for.

129 | P a g e
Marking Scheme:
▪ Explanation of the tax treatment of the amount forfeited 1.0
▪ Explanation of the tax treatment of the amount received from the sale of the property 3.0

Answer # 9 Autumn 2017 Q. 3(b)

Taxable Income
Sale of painting Rupees
Consideration received 2,000,000
Less: Cost of painting (1,800,000)
Capital gain 200,000

Examiner Comments:
This part was based on the requirements of section 37(5d) and 38 of the Income Tax
Ordinance, 2001 and candidates were required to calculate gain on the sale of a painting
where the sale was made to the brother of the seller and the holding period was more than
one year.
The overall response was very good as a large number of the candidates secured full
marks. Some of the common mistakes were:
▪ It was stated that the sale to a brother comes under the ambit of non-recognition rule and
hence no gain/loss would be recognized.
▪ Consideration received on sale of capital gain of Rs. 2,000,000 was treated as taxable
gain and 75% of gain was adjusted against cost of painting.
▪ Some candidates were of the view that capital gain does not arise on disposal of painting,
being capital asset specified in section 38(5) of the Income Tax Ordinance, 2001. They
failed to appreciate that the said provision is applicable in case of loss on disposal only.

Marking Scheme:
Determination of taxable gain on disposal of painting 2.0

Answer # 10 Autumn 2019 Q. 3(b)

Under the ITO-2001, any movable property held for personal use are excluded from the definition of capital
assets. Therefore, income from sale of personal vehicles are not taxable under the head Capital Gains.

Examiner Comments:
Good performance was observed in this part of the question.

Marking Scheme:
▪ Discussion on provision relating to taxability on sale of movable property held for
personal use 1.5
▪ Conclusion 0.5

130 | P a g e
Answer # 11 Spring 2020 Q. 4(b)

Gain/ Loss on disposal of sculpture Sculpture I Sculpture II


Fair market value of sculpture when they lost 360,000 540,000
Consideration allocation ratio (360/900, 540/900) based on fair market value 40% 60%
Consideration on the basis of allocation ratio 376,000 564,000
Less: Cost of acquisition (410,000) (475,000)
(Loss)/Gain on disposal of sculpture *(34,000) 89,000
Taxable income - 89,000
*The loss shall not be recognized on the disposal of sculpture

Examiner Comments:
▪ Examinees failed to comprehend that the insurance claim received was to be apportioned
between the two sculptures in the ratio of their respective fair market values at the time
of loss and then the respective cost was to be reduced therefrom to determine the gain or
loss.
▪ Examinees instead of determining gain / loss separately for each sculpture, performed
calculation on aggregate basis by deducting cost of sculpture of Rs. 885,000 from
consideration received of Rs. 940,000.

Marking Scheme:
Computation of gain/loss on disposal of sculpture:
▪ Distribution of consideration and cost 2.0
▪ Exemption of 25% due to holding for more than one year 1.0
▪ Treatment of loss on disposal of sculpture 1.0

Answer # 12 Autumn 2022 Q. 3

(a)
Capital gain Cash
consideration Cost of Capital Taxable
/ FMV purchase gain/loss capital gain
(whichever is
higher)
----------------------- Rs. in million -----------------------
Investment in shares of a
public unlisted company 3.0 2.0 1.0 1.0
Investment in shares of a listed
company 3.5 2.1 1.4 1.4
Personal car (not covered under
the definition of capital asset)
6.0 3.8 - -
Painting (loss on painting not to be
recognized) 1.2 1.7 - -
Jewelry 8.0 4.8 3.2 3.2
5.6 5.6

Taxation of capital gain


Capital gain to be taxed under NTR:
Gain on investment in shares of public unlisted company 1.0

131 | P a g e
Gain on disposal of jewelry 3.2
4.2
Tax liability:
On Rs. 4 million 0.765
Above Rs. 4 million @35% 0.07
0.835

Capital gain to be taxed under separate block of income:


Tax on gain on disposal of investment in shares of listed company
(1.4×0%) -

(b) Shahid
Computation of total and taxable income for the tax year 2022
Rs. in million
Income from property
Forfeited deposit 6.00
Less: Repair allowance @ 1/5 (1.20)
4.80
Capital gain
Disposal of bungalow [(54–50)] 4

Income from other sources


Rent of factory building along with plant and machinery (1×12) 12.00
Less: Repair & maintenance (3.50)
8.50
Exempt income
Rental income of agricultural land [5÷2] 2.50
Total income 19.80
Less: Exempt income (rental of agricultural land) (2.50)
Less: Separate block of income (capital gain) (4.00)
Taxable income 13.30

Tax Liability
On 4 million 0.765
Above 4 million @ 35% 3.255
4.02
Tax on capital gain (4×5%) 0.2
Total tax liability 4.22

132 | P a g e
CHAPTER 5
INCOME FROM OTHER SOURCES
Scope:
 Income of every kind received by a person in a tax year,
 if it is not included in any other heading,
 other than income exempt from tax under this Ordinance,
 shall be chargeable to tax in that year under the heading
 “Income from Other Sources.
Chargeability:
Income from other sources is chargeable to tax on ‘receipt basis’. Thus, any income from other sources which
is accrued for a tax year but is not received is not chargeable in that tax year.
Income From Other Source
N.T.R F.T.R
1) Additional payment on delayed refund under any tax law
Melinda Gates paid tax Rs. 30 million to FBR for the tax year 2024, later on she realized she had paid an excess
payment of Rs. 5 million, therefore she filed a refund application. FBR agreed to refund the excess amount,
however the refund is delayed so the department paid her 6 million (1 million as additional payment on delayed
refund.) Comment on the Income Status….
Answer [ Rs. 1 million is paid in excess of Rs. 5 Million. So this 1 million will be a part of Income from Other
Source]
2) Ground Rent
Farrukh & Co. is in search of land for the construction of its head office. He approached Mr. Daniyal to
provide a land which is currently under the ownership of Daniyal at M.M Alam Road. As per mutual
agreement Farrukh & Co. decided to take the land on rent and will construct the head office and will pay the
annual rent of Rs. 3,000,000 to Daniyal. Comment on the Rent received by Daniyal….
Solution:
Ground rent will be allowed as an expense to Farrukh and will be income from other source of Daniyal
3) Rent from the sub-lease of land or a building;
Mr. Doremon has let out his house to Nobita for a monthly rent of Rs. 40,000 on 1st November 2023, the house
comprises of two floor (each floor rent is Rs. 20,000) on 1st January 2024 Nobita decided to let out the ground
floor to his friend Shizuka at a monthly rent of Rs. 25,000. Calculate the Income for the Tax year 2024 for
Doremon and Nobita.
Solution
Mr. Doremon Income [40,000 x 8 = 320,000]
Mr. Nobita Income:
Rent received for floor [25,000 x 6 = 150,000]
Rent paid for the floor [20,000 x 6 = 120,000]
Net Income = 30,000

133 | P a g e
4) Income from the lease of any building together with plant or machinery;

Rent Received XXX

Less: Expenses (Repairs & Maint.) (X)

Depreciation (X)

Initial Allowance (X)

Net Income XXX

(Visit Past Paper for the Practice of this rule)


5)Income from provision of amenities, utilities or any other service connected with renting of building

Income Received XXX

Less: Expenses (X)

Net Income XXX

To Do Question (Solve by Yourself) Mr. Jz rented his flat to Mr. AB for a monthly rent of Rs. 100,000. In
addition to the rent he also provided the laundry services to AB for which he charges 15,000 per month his
actual expenses on laundry was 8,000 per month. In addition to the laundry he also provided the generator to
AB at a monthly rent of Rs. 5,000 while he paid the rent of the generator the owner Rs. 3,000 per month. He
also provided the Security guard for which he charged Rs. 12,000, this amount is included in the rent. His annual
expenditure on the security guard is 130,000 Rs.
Calculate the income of Jz under the relevant head
6)- Income deemed u/s 111- unexplained assets or income;
7)- Any annuity or pension [Other Then Salary]
8)- Any amount received by a person from Approved Income Payment Plan or Approved Annuity Plan
under Voluntary Pension System Rules, 2005.
10)- Any other amount received as consideration for the provision, use or exploitation of property,
including from the grant of a right to explore for, or exploit, natural resources;
Mr. Ghareeb owns a land in the Middle of Balochistan, one day from nowhere a Mr Ameer approached him and
makes him aware that there might be a possibility of copper underneath his land. Mr. Ghareeb has no knowledge
about this and how to explore it, Mr Ameer ask his permission to explore the land with a monthly payment of
Rs. 200,000 to him.
(Ans: 200,000 x 12 = 2,400,000 will be the Income from other source of Mr Ghareeb)
11) The fair market value of any benefit, whether convertible to money or not, received in connection
with the provision, use or exploitation of property;
Continuing the above example :In Addition to the Amount Mr. Ameer, also agreed that Ghareeb will get the
20% of the copper extracted in a year. He received his share of copper on 31st March and sold the all copper to
GM Cables for Rs. 1,000,000.

134 | P a g e
(Ans: 1,000,000 will be a part of income from other source of Ghareeb)
12) Any amount received by a person as consideration for vacating the possession of a building or part
thereof.
Any amount received by a person as consideration for vacating the possession of a building or part thereof,
reduced by any amount paid by the person to acquire possession of such building or part thereof. This income
shall be taxable in 10 years from the year of receipt in equal proportion.
13) Any gift received without the consideration
any amount or fair market value of any property received without consideration or received as gift, other than
gift received from grandparents, parents, spouse, brother, sister, son or a daughter. (Gift Received in cash even
from relatives will be taxable)
During the tax year 2020, Sadiq received a flat as gift from his uncle, Mumtaz Alvi. The flat was located in posh
area of Lahore and its fair market value at the time of gift was Rs. 4.5 million. Discuss the tax treatment of the
flat received by Sadiq. (Ans : 4.5 million will be a part of his income)
14) Any Amount received as loan
Any amount received as a
 Loan
 Advance
 deposit for issuance of shares or
 gift by a person in a tax year from another person (Not being a banking Co. or financial institution)
Otherwise than by
➢ Cross check drawn on bank or
➢ Banking channel from a person holding a NTN Number
Shall be treated as “Income from Other Source”
Exception: This rule shall not be applied to
➢ Advance for Sale of Goods
➢ Advance For Supply of Services
If a Person Holds NTN If a person doesn’t holds NTN
He Should make the payment to anyone following He Should make the payment to anyone through
modes:- following modes:-
➢ Cross Cheques
➢ Cross Cheques
➢ Any Other Banking Channel

Mr. Atif Received the following amounts


1. Loan received from Grandfather in Cash [Part of Income from Other Source]
2. Loan From uncle through banking channel (Uncle have NTN) [Not Part Of Income]
3. Gift received from mother in Cash [Part of Income from Other Source]
To Do Question (Solve by Yourself) Mr. Abid is constructing his house and for the purpose of meeting the
construction expenses, he intends to obtain a loan of Rs. 500,000 from his friend professor, who is in the
business of money lending. He is advised that such loan shall be charged under his taxable income. You are
required to advise Abid under which circumstances this loan won’t be a part of his income

135 | P a g e
To Do Question (Solve by Yourself) Mr. Kachra Seth owner of Heera Pheri Kabariya received the following
Payments:
➢ Loan of Rs. 200,000 in cash from Raju (Having NTN)
➢ Loan of Rs. 250,000 from Totla Seth in Cash (Not Having NTN)
➢ Loan of Rs. 350,000 from Babu Bhaiya through crossed Cheque (Having No NTN)
➢ Loan of Rs. 450,000 from Sham through online transfer (Having No NTN)
➢ Received a Iphone 15 Pro Max (512 Gb) from Anjali. (Her employer)
Required: Comment on each transaction


15) Income splitting: Royalty income of Authors .
Where the time taken by an author of a literary or artistic work to complete the work
exceeds 24 months, the author may elect to treat any lump sum amount received by the
author in a tax year on account of royalties in respect of the work as having been received
in that tax year and the preceding two tax years in equal proportions.

Question
Mr. Danishwar, a renowned author, completed his book on "Human Behavior" in two and a half years
time. He received a lump sum amount of Rs. 900,000 in May 2010 on account of royalty. Briefly explain
the taxability of the above situation (Marks 3)

Question
Mr. Pansari provided further information for the tax year 20X4:
i. Received a royalty of Rs. 2,000,000 from K Publishing on a book written on Wild Hunting. Mr.
Pansari completed the book in 19 months and all the costs relating to its publication were borne
by the publisher. The applicable tax rates in tax years 20X2 and 20X3 were 16% and 18%
respectively.
Tax treatment:
Total royalty of Rs.2 million is required to be included in taxable income for the tax year 20X4 as the
completion time of the book does not exceed 24 months.

Income from Other Sources


Sr. Taxable
No Particulars Rs. Rs
1 Lecture, teaching and examination fee received X
2 Professional fee received X
3 Additional Payment on delayed Refund X
4 Ground Rent Received X
5 Rent from Sub-lease of land or building X
6 Income from lease of Building with Plant & Machinery
7 Rent Received X
• Less: Initial allowance and Depreciation charges (X)
• Less: Repairs and collection charges (X)

136 | P a g e
• Less: Property Tax paid (X)
• Less : Any expenditure related to this building and machinery (X) X
Income from provision of amenities, utilities or any other services connected with
8
them X
Less: Any expenditure actually paid in respect of this income (X) X
9 Any annuity or pension received from any person ( other than employer) X
Any amount received in respect for provision, use or exploitation of property,
10
including the grant for the use or exploit of of property X
The FMV of any benefit whether convertible to money or not, received in connection
11
with the provision, use or exploitation of property X
12 Any amount received for vacating the possession of the property
Taxable amount = (Received amount – amount paid) ÷ 10 years X
Any amount received by the person from Approved Income payment plan or
13
Approved annuity plan under the Voluntary pension system Rules X
14 FMV of any property received as gift without any consideration other than relative X
Any amount of loan, advance, deposit for issuance of shares or gift received from
15
another person
• Amount received from a person (having NTN) through crossed cheque ( Exempt) --
• Amount received from a person (having NTN) through a normal banking channel
(Exempt) --
• Amount received from a person (having NTN) in form of cash (Totally Taxable) X
• Amount received from a person (having no NTN) through crossed cheque ( Exempt) --
• Amount received from a person (having no NTN) through a normal banking channel
(Totally Taxable) X
• Amount received from a person (having no NTN) in form of cash (Totally Taxable) X
16 Prizes and Winnings
• Prize on prize bond and crossword puzzle FTR
• Raffle, Lottery, Prize on winning a quiz or prize offered by companies for
promotion of sales FTR
17 Dividend Income
• Pakistan Source dividend income @ 15% FTR
• Foreign Source dividend income X
18 Profit on Debt (Interest Income)
• Bank profit & Profit from PLS account FTR
• Profit on Saving certificates under NSC including Defense saving certificates
and post office certificates FTR
• Profit on certificates, debentures issued by a company or financial institution FTR
• Profit on securities issued by the government such as PIBs FTR
19 Royalty Income
• Pakistan source Royalty income or fee for technical services received by non-resident
person @ 15% FTR
• Pakistan source Royalty income or fee for technical services received by non-resident
person through permanent establishment in Pakistan X
• Royalty Income received by resident non-professional writer X
• Royalty income of authors for literary work completed in more than 24 months
[ amount received as royalty income ÷ 3 years]= amount taxable in current year and in two
preceding years] X

Total Income from other Source Excluding FTR XXX

137 | P a g e
PAST PAPER QUESTIONS
Question # 1 Spring 2014 Q. 4

Bashir and Jameel jointly own a house in Karachi. Bashir has 75% share in the house. On 1 September 20X3,
the house was let out at an annual rental value of Rs. 6,500,000. This amount includes Rs. 186,000 per month
for utilities, cleaning and security.

During the tax year 20X4, the owners incurred the following expenditures in relation to the house:

Rupees

Utilities, cleaning and security 650,000

Repair and maintenance 810,000

Insurance premium 240,000

Collection charges 25,400

Mark-up on amount borrowed for extension of the house 840,000

Bashir and Jameel have no other source of income. All the above expenses were incurred by them jointly.
Required:
Calculate taxable income of Bashir and Jameel under appropriate heads of income for the tax year 20X4. (10)

Question # 2 Spring 2017 Q. 3(a)

On 1 June 20X6 Dawood and Dewan jointly purchased a bungalow for Rs. 35 million. They paid the
amount in the ratio of 65:35 respectively. To arrange funds for the deal, Dawood borrowed Rs. 3,000,000 in cash
from Shameem who is in the business of lending money. The rate of interest is agreed @ 20% per annum.

On 1 July 20X6, the house was let out to a company at annual rent of Rs. 4,500,000 inclusive of an amount of Rs.
75,000 per month for utilities, cleaning and security. For providing these services Dawood and Dewan paid Rs.
35,000 per month. During the tax year 20X7 they also paid Rs. 10,000 as collection charges and Rs. 230,000 for
administering the property.

Required:
Compute taxable income of Dawood and Dewan under appropriate heads of income for the tax year 20X7. (08)

Question # 3 Spring 2019 Q. 3(c)

On 1 July 20X8, Zahid rented out his properties as follows:


(i) An apartment was rented to Abdul Qadir at a monthly rent of Rs. 40,000. Zahid received a non-
adjustable security deposit of Rs. 300,000 which was partly used to repay the non-adjustable
security deposit amounting to Rs. 175,000 received from the previous tenant in July 20X3. He also
spent Rs. 20,000 on repairs of the apartment in February 20X9.
(ii) A bungalow was rented to a bank. Zahid and his younger brother are joint owners of the bungalow
in the ratio of 60:40 respectively. The annual rent agreed with the bank was Rs. 6,000,000 which is
inclusive of Rs. 100,000 per month for utilities, cleaning and security. Zahid paid Rs. 35,000 per
month for providing these services.
Required:
Under the provisions of Income Tax Ordinance, 2001 compute total and taxable income of Zahid for the
tax year 20X9 under appropriate heads of income. (07)

138 | P a g e
Question # 4 Autumn 2019 Q. 2(b)

During the tax year 20X9, Amjad carried out the following transactions in respect of his properties:
(i) On 1 July 20X8, Amjad purchased a factory building in Sukkur along with the installed machinery at the
price of Rs. 9 million and Rs. 3 million respectively. To manage the shortage of funds of Rs. 2,000,000, he
borrowed the same on 1 July 20X8 from his friend Shamshad through a crossed cheque. The loan carries
interest at the rate of 18% per annum.
On 1 January 20X9, he let out this building along with the machinery to Basit at a monthly rent of Rs.
500,000 payable in advance.

(ii) On 1 July 20X8, Amjad let out his residential property situated in DHA Karachi to Mirza Limited at a
monthly rent of Rs. 300,000. Rent for the two years was received in advance on 1 August 20X8.

(iii) On 1 July 20X8, Amjad also entered into an agreement with Zeeshan for the sale of his plot situated in
Quetta for Rs. 50 million. The plot had been purchased for Rs. 40 million in 20X4. Under the terms of sale
agreement, he received Rs. 5 million at the time of signing the agreement and the balance was to be
received on 30 September 20X8. However, due to financial difficulties, Zeeshan failed to pay the balance
amount on the due date and consequently, Amjad forfeited the advance in accordance with the terms
of the agreement.
On 10 April 20X9, he finally sold the plot to Jamshed for Rs. 65 million.

(iv) Following expenditures were incurred by Amjad in respect of his properties in Sukkur and Karachi:

Property situated in
Details of expenditures
Sukkur Karachi
Repair & maintenance - building 270,000 70,000
- machinery 50,000 -
Ground rent 50,000 10,000
Insurance - building 150,000 20,000
Total 520,000 100,000

Required:
In view of the provisions of the Income Tax Ordinance, 2001 compute under appropriate head of income,
taxable income of Amjad for the tax year 20X9. (10)
Question # 5 Spring 2020 Q. 4(a)

Respond to the following independent situations, under the provisions of the Income Tax Ordinance, 2001:
(a) During the tax year 2020, Sadiq received a flat as gift from his uncle, Mumtaz Alvi. The flat was located
in posh area of Lahore and its fair market value at the time of gift was Rs. 4.5 million. Discuss the tax
treatment of the flat received by Sadiq. (02)

Question # 6 Autumn 2020 Q. 4(a)

Farheen is a resident filer and has provided following information pertaining to tax year 2020:
(i) She owns a bungalow situated in Multan which was given on rent to Abbas under a rental agreement of
five years which expired on 31 March 2020. Details of payments received as per the rent agreement are
given below.

Rent Rs. 175,000 per month


Security guards’ salaries Rs. 50,000

139 | P a g e
Non-adjustable security deposit Rs. 2,500,000

On expiry of the rental agreement, Farheen refunded the security deposit to Abbas and rented out the
bungalow to a new tenant Zafar on the same terms and conditions.

Farheen pays Rs. 40,000 per month to a security services company which provides security guards at the
bungalow.

(ii) She owns a residential plot in Karachi. On 1 March 2020, she decided to sell the plot to Mehreen for Rs.
2,200,000 and received a deposit of Rs. 220,000. On 1 June 2020, she forfeited the deposit on refusal of
Mehreen to purchase the plot.

(iii) On 1 December 2017, she had acquired a furnished office on monthly rent of Rs. 5,000 for her own use
and had paid a non-refundable amount of Rs. 2,000,000 to the previous tenant for vacating the office.
During the year, she received an offer of Rs. 2,400,000 from Shehroz to vacate this office which she
accepted and received the amount on 1 March 2020.

(iv) On 1 October 2019, she inherited a factory with plant and machinery from her father and let it out on 1
December 2019 at a monthly rent of Rs. 500,000.

(v) Legal and professional charges of Rs. 40,000 were paid for preparation of rental agreements.

(vi) On 15 November 2019, she received income tax refund of Rs. 180,000 related to tax year 2017. This
amount included Rs. 30,000 being additional payment on delayed refund.

Required:
Under the provisions of the Income Tax Ordinance, 2001 and Rules made thereunder, compute the total
income of Farheen under appropriate heads of income for the tax year 2020. (07)

140 | P a g e
ANSWERS

Question # 1 Spring 2014 Q. 4

Income of Bashir Rupees

Income from other source [w-2 1,210,000 x 75%] 907,500

Income from property w-1 1,934,950

Taxable Income under NTR 2,842,450

Tax Payable to Govt (315,000 + 25% x 10,613) 425,613

Income of Jameel

Income from other source [w-2 1,210,000 x 75%] 302,500

Income from Property 644,984

Taxable Income under NTR 947,484

Tax Payable to Govt (15,000 + 15% x 147,484) 37,123

W-1 Income from Property

Gross receipt 6,500,000

Less: Income charged under other Source (186,000 x 12) (2,232,000)

4,268,000

Less: Receipt relating to the next year (4,268,000 x 2/12) (711,333)

Rent chargeable to tax 3,556,667

Less: Admissible deductions

Repair allowance (3,556,667 x 20%) (711,333)

Insurance Premium (240,000)

Collection Charges (25,400 or (4% of 3,556,667) (25,400)

Net Income from Property 2,579,934

Share of Bashir (2,579,934 x 75%) 1,934,950

Share of Jameel (2,579,934 x 25%) 644,984

W-2 Income from Other Source

141 | P a g e
Rent charged for utiliies etc (186,000 x 10) 1,860,000

Less : Expenses (650,000)

1,210,000

Question # 2 Spring 2017 Q. 3(a)

Dawood

Income from other source Rupees

Cash loan 3,000,000

From Utilities, cleaning & security (480,000 (w-2) x 65%) 312,000

3,312,000

Income from Property

Rent (2,736,000 (w-1) x 65%) 1,778,400

Total/Taxable Income 5,090,400

Tax Liability (680,000 + 1,090,400 x 32.5%) 1,034,380

Dewan

From Utilities, cleaning & security (480,000 (w-2) x 35%) 168,000

Rent (2,736,000 (w-1) x 35%) 957,600

Total/Taxable Income 1,125,600

Tax liability (15,000 + 325,600 x 15 %) 63,840

W-1 Income from property

Annual Receipt 4,500,000

Less : Income from other source (75,000 x 12) (900,000)

Rent chargeable to tax (Gross) 3,600,000

Less: Deductions

Repairs ( 20% x 3,600,000) (720,000)

Collection charges (240,000 or 4% of 3,600,000) (144,000)

Income from Property (Net) 2,736,000

142 | P a g e
W-2 Income from Other Source

Rent for utilities 900,000

Less : Expenses (420,000)

480,000

Examiner Comments:

According to the scenario presented in this part of the question, Dawood and Dewan had
jointly purchased a bungalow and had rented it to a company for an amount which was
inclusive of utilities and other services. The requirement was to compute the taxable
incomes of both individuals under appropriate heads of incomes. Most of the candidates
performed in an average manner scoring between 3 to 6 marks. Some of the common
mistakes were as follows:

▪ Some candidates considered the joint ownership of the property as an AOP.


▪ Many candidates did not know that the amount included in rent for the provision of
amenities and utilities should be taxed under the head “Income from other sources”.
▪ A number of candidates deducted collection and administrative charges against rent
whereas such deductions against income from property are allowed only where the
property is owned and rented by a company.
▪ Expenditure incurred in relation to utilities, cleanliness and security was deducted from
income from property. Instead, it should have been claimed against amount received on
account of utilities, cleanliness and security i.e. against income from other sources.
▪ Only few candidates were able to identify that loan borrowed by Dawood is to be
included in his taxable income under the head “Income from other sources” because any
amount taken as advance from any person not being a banking company otherwise than
through banking channel is treated as income of the recipient.

Marking Scheme:

Computation of:

▪ income from property 4.0


▪ income from other sources 4.0

Question # 3 Spring 2019 Q. 3(c)

Income from property Rupees


Rental income (40,000 × 12) 480,000
Rental income from joint property (4,800,000 (W-1) × 60%) 2,880,000
Less: Repair charges -
Add: Un-adjustable security deposits (Rs. 212,500 (W-2) × 1/10) 21,250
3,381,250
Less: Allowable Deductions
Repair Expense (3,381,250 x 20%) (676,250)
Income from Property (Net) 2,705,000

Income from other sources

143 | P a g e
Income from utilities, cleaning and security
[(1,200,000 - 420,000) ×60%] 468,000
Taxable income ❑ 3,173,000
Tax Liability (465,000 + 30% 173,000) ❑ 516,900

❑ W-1: Determination of Income from joint property (bungalow) ❑ Rupees


❑ Total rental income ❑ 6,000,000
❑ Less: Amount relating to utilities, cleaning and security ❑ (1,200,000)
❑ Income from property ❑ 4,800,000

❑ W-2: Computation of un adjustable security deposit ❑ Rupees


❑ Received from new tenant ❑ 300,000
❑ Less: Amount charged to tax in July 2013 to June 2018 (175,000 × 5/10) ❑ (87,500)
❑ ❑ 212,500

Examiner Comments:

(i) Amount relating to utilities, cleaning and security, being income from other sources,
was not excluded from the rental income from joint property.

(ii) Incorrect amount of the un adjustable security deposit was worked out for inclusion in
the head of income from property.

(iii) 60% of the proportionate share of income from utilities, cleaning and security was
taxable as income from other sources. However, income from providing such amenities
was to be reduced by the sum paid by the individual in providing such services before
applying the proportionate share percentage. This point is generally missed in number
of scripts.

(iv) Income from property was not deducted from total income to arrive at taxable income.

Marking Scheme:

Computation of:

▪ income from property 5.0


▪ income from other sources 2.0

Question # 4 Autumn 2019 Q. 2(b)

Mr. Amjad
Computation of taxable income
For tax year 20X9 Mr. Amjad
❑ Rupees
❑ Income from property ❑
❑ (i) ❑ Residential property at DHA – Karachi (W-1) ❑ 2,850,000
❑ (ii) ❑ Amount forfeited from Zeeshan ❑ 5,000,000
❑ ❑
❑ ❑

144 | P a g e
❑ ❑
❑ Income from other sources ❑
❑ (i) ❑ Factory building at Sukkur – Basit (W-1) ❑ 950,000
❑ ❑
❑ Income from capital gain ❑
❑ (i) ❑ Sale of plot in Quetta – Jamshed ❑ -
❑ Since the plot was bought in 2015, therefore no tax is payable under the law disposal ❑
after 6 years
❑ Total Taxable income ❑ 8,800,000
❑ Tax Liability (765,000 + 4,800,000 x 35%) ❑ 2,445,000

❑ W-1: Income from Income other


property sources
DHA - Karachi Sukkur - Factory
❑ Rental Income (300,000×12), (500,000×6) 3,600,000 3,000,000
❑ Less: Admissible expenses
❑ Repair to building (allowed upto 1/5 of the rental
amount) (720,000) 70,000
❑ Repair to machinery (10,000) 50,000
❑ Ground rent - 50,000
❑ Insurance – Building (20,000) 150,000
❑ Depreciation: Building – Normal [Rs. 9m @ 10%] - 900,000
❑ Plant – Normal [Rs. 3m @ 15%] - 450,000
❑ Interest on loan from Shamshad
❑ Rs. 2 million @ 18% for 6 months - 180,000
❑ - 2,050,000
❑ Net income 2,850,000 950,000

Examiner Comments:

▪ Many examinees offered 11 months' rent to tax instead of 12 months.


▪ Expenses relating to residential property were considered to be admissible in case of
individuals.
▪ Rent from factory building together with plant and machinery was wrongly considered
under the head 'Income from property'.
▪ Many examinees computed initial depreciation on both building and plant and
machinery. Only few examinees correctly computed depreciation for the period of 6
months instead of the whole year.
▪ Gain arising on sale of plot in Quetta was wrongly treated as taxable gain.
▪ Some examinees considered the gain to be taxable under the head 'Income from
property'.
▪ Some examinees offered the rent received in advance for two years to be taxed under
the head Income from property.

Marking Scheme:

Computation of income from:


▪ property 4.0
▪ other sources 4.5
▪ capital gain 1.5

145 | P a g e
Question # 5 Spring 2020 Q. 4(a)

Any amount or fair market value of any property received as gift, other than gift received from grandparents,
parents, spouse, brother, sister, son or a daughter shall be chargeable to tax under the head ‘Income from
other sources’.

The flat received by Sadiq from his uncle will be chargeable to tax.

Examiner Comments:

▪ Examinees failed to comprehend that flat received by Sadiq as a gift from his uncle was
to be added to capital gain at the fair market value of Rs. 4,500,000.
▪ Examinees also failed to appreciate that non-recognition rule applies only when a gift is
received from grandparents, parents, spouse, brother, sister, son or a daughter.

Marking Scheme:

▪ Discussion on the provision relating to the treatment of property received as a gift 1.5
▪ Conclusion 0.5

Question # 6 Autumn 2020 Q. 4(a)

Farheen
Computation of total income, taxable income and net tax payable/refundable
For tax year2020
Rupees
Income from property
Rent from Bungalow - Abbas [175,000×9] 1,575,000
Rent from Bungalow - Zafar [175,000×3] 525,000
Non-adjustable security deposit - Zafar (W-1) 150,000
Forfeiture of deposit for sale of building 220,000
Total income from property 2,470,000
Less Allowable Deductions
Repair (2,250,000 x 1/5) (450,000)
Net Income from Property 2,020,000
Income from other sources
Security guard's salary - [50,000×12] 600,000
Deposit for vacating the office [(2,400,000–2,000,000)/10] 40,000
Rentals from lease of factory with machinery [500,000×7] 3,500,000
Additional payment of delayed refund [S.39(cc)] 30,000
Less: Payment for security services [40,000×12] (480,000)
Total income from other sources 3,690,000
Total income 5,710,000

W-1 Rupees
Non-adjustable security deposit received from Zafar 2,500,000
Less: Amount charged to tax till tax year 2019 [(2,500,000/10)×4] (1,000,000)
1,500,000
Amount chargeable to tax this year [1,500,000/10] 150,000

146 | P a g e
Examiner Comments:

(a)
▪ Net income from provision of security guards, rent received from inherited factory and
amount received for vacating the office was taken under 'Income from property'
instead of 'Income from other sources'.
▪ The amount previously paid for getting the office vacated was not deducted from the
amount received.
▪ The entire amount of tax refund was taken as taxable instead of just the additional
payment.
▪ Legal charges were taken as an admissible deduction.

Marking Scheme:

(a) Computation of total income from:


▪ property 3.5
▪ other sources 3.5

147 | P a g e
CHAPTER 06
FINAL TAX REGIME
Final tax regime (Gross income basis)
1. Income subject to final tax are those which are subject to collection or deduction of tax at source and
the tax so collected or deducted at source is treated as final tax on the income arising from such
transactions.
2. The tax collected or deducted on such transactions is commonly known as non-adjustable tax collected
or deducted at source. The taxation of income subject to final tax.
3. All transactions subject to collection or deduction of tax at source do not fall under income subject to
final tax. Different set of rules apply for each nature of income

Following rules apply to the income subject to final tax:


1. Such income is not chargeable to tax under any head of income in computing the taxable income;
2. No deduction is allowed for any expenditure incurred in deriving the income
3. The amount of the income is not reduced by
• Any deductible allowance; or
• The set off of any loss
4. The tax deducted is not reduced by any tax credit
5. There is no refund of the non-adjustable tax collected or deducted at source unless such tax is in excess
of the amount of final tax for which the taxpayer is chargeable and
6. An assessment is treated to have been made and the person shall be required to furnish a return of
income in respect such income.

Examples of FTR under the Income Tax Ordinance 2001


• Dividend
• Profit on Debt (Except Co.)
• Commission & Brokerage
• Royalty
• Exports
• Fee for Technical Services of Non-Resident
• Prizes, Winnings
• Lottery

Dividend [D.A.R.F.A.L.L]
“Dividend” includes
➢ Any distribution by a company to its shareholders out of accumulated profit. Dividend may be
distribution of:-[ DARFALL ]
❖ Debentures or deposit certificates
❖ Assets of company including money or
❖ Reduction of capital by the company
❖ After tax profit of a branch of Foreign company operating in Pakistan.
❖ Amount expended by a private company on behalf for benefit of shareholder.
❖ Loan or advance given by a private company (ITO) to its shareholders.
❖ Liquidation of company
➢ All distributions must be up to the extent of accumulated profits possessed by the company at date of
distribution.

148 | P a g e
➢ Following payments are not dividends;
• Loan or advance by a private company involved in lending business.
• Subsequent dividend if the payment is set-off against the loan already treated as dividend.
• Remittance of after tax profit by a branch of Petroleum Exploration and Production (E&P) foreign
company, operating in Pakistan.
• Distribution on liquidation of company or reduction of capital by company in respect of any share for
full consideration, or redemption of debentures or debenture stocks, where the holder of the share or
debenture is not entitled in the event of liquidation to participate in the surplus assets.

Definition of Accumulated Profit


"Accumulated Profits "in relation to distribution or payment of dividend include –
• Any reserve made up wholly or partly of any allowance, deduction or exemption all profits of the
admissible for tax purpose
• All profits of the company including income of a trust up to the date of distribution of dividend (in case
of liquidation, up to the date of its liquidation).

Question
You are a tax partner in a local firm of Chartered Accountants. You have been approached by some of the
clients for opinion on the possible tax implication/treatment of certain matters. Under the provisions of the
Income Tax Ordinance, 2001 and Rules made thereunder, discuss the tax implication/treatment in the
following independent case:
On 31.7.20X9 Noble (Pvt.) Ltd (NPL) received a notice from Officer of Inland Revenue asking it to deposit
withholding tax of Rs.3,750,000 in respect of a loan of Rs.25,000,000 which was provided by NPL to one
of its shareholders on 31.12.20X8. NPL's accumulated profits at the time of provision of loan amounted to
Rs.20,000,000. (Marks 3)
Answer
When a private company provides loan to any of its shareholders, the loan is treated as a dividend to the
extent the company's accumulated profits. Since NPL had accumulated profit of Rs.20 million at that time,
the entire amount of loan of Rs.25,000,000 shall not be considered as dividend. Therefore, NPL was
required to withhold tax of Rs.3,000,000 i.e. 15% of Rs.20 million instead of Rs.3,750,000 from the
payment.

• Tax is required to be deducted from dividend income which is taxable under


• FTR as full and final tax liability for a shareholder including a corporate shareholder and no
expense or deduction is allowable from such income.
• The general rate is 15% of gross dividend. However, different tax rates are applicable in
specific cases are as under:

Recipient of dividend Tax rate

Dividend paid by Independent Power Producers with specified conditions 7.5%


Dividend received from a company where no tax is payable by such company due 25%
to exemption, carry forward of loss or tax credits

149 | P a g e
Dividend received from Special Purpose Vehicle: 0%
0% in case of a Real Estate Investment Trust (REIT) scheme as recipient of
dividend; and 35%

35% in case of other recipients


Other dividend in case of corporate and non-corporate shareholders 15%

Zakat deducted from dividend income:


No deduction is allowed from FTR income.
However, zakat paid out of dividend income is deductible from normal taxable income, if any

Incorrect tax deduction from dividend income:


In case of incorrect tax deduction by a withholding agent, any excess or short deduction shall be adjusted accordingly
from normal tax liability.
Example: A company has deducted tax @ 10% from dividend income instead of 15% then tax@ 15% shall
be included in tax liability after rebates and withholding @ 10% shall be deducted from tax liability and
automatically be adjusted Same treatment in case of other FTR items.

Dividend received from any foreign company


Summary of relevant provisions are:

Section 5 [charging section]:


Dividend income is taxable @ 15% of the gross dividend under FTR

Section 150 [tax withholding section]:


Tax is deductible from gross dividend @ 15%.

Result:
Dividend income is taxable @ 15% general rate whether tax is deductible not such as dividend from
any foreign company.

The foreign company does not deduct tax from such dividend income as per Pakistan tax laws, tax @ 15% is therefore
required to be included in tax liability after rebates. [Part of NTR]

If the foreign company has deducted tax from dividend as per that foreign Country’s laws then foreign tax credit
shall not be allowed in Pakistan as the dividend income falls under FTR where no tax credit is allowed.

Bonus Shares

Bonus shares are taxable @ 10% of the specified value under FTR and the company issuing bonus
shares is obliged to collect tax from the shareholders as under:

a) A company issuing bonus shares shall deposit tax in the government treasury within 15 days
of the first day of closure of books @ 10% of the value of bonus shares.
b) Value of bonus shares in case of a listed company
Value of bonus shares in case of a listed company shall be day-end price on the first day of closure
of books.

1. c)Value of bonus shares in case of an unlisted company including private company

150 | P a g e
Value of bonus shares in case of an unlisted company including private company shall be the value
as prescribed under the law.
Rule 231G of the Income Tax Rules
Value of bonus shares issued by an unlisted company shall be the face value or break-up value whichever is
higher.
Break-up value shall be determined as ordinary paid-up capital + reserves (excluding capital reserves, share premium
reserves) divided by total number of ordinary shares after the issuance of bonus shares based on the latest financial
statements approved by the BOD for the purposes of issuance of bonus shares.

d) The tax deposited by the company shall be collected from the shareholders before the issue of bonus shares.
If any shareholder does not pay tax nor collects its bonus shares within 15 days of issuance Of bonus shares, the
company may dispose of such number of shares sufficient to recover The amount of tax deposited by it on behalf
of the shareholder.

Difference between bonus shares and dividend in specie is as under


• Bonus shares is a situation where A Ltd issues additional shares of A Ltd to its shareholders free of cost
in lieu of cash dividend
• Dividend in specie is a situation where A Ltd has shares of B Ltd and transfers the same to its
shareholders free of cost in lieu of cash dividend.
Taxability of dividend income and capital gain in case of bonus shares and dividend in specie can be Summarized
as under:
• The amount of dividend in specie (i.e. FMV of shares at the time of transfer to the shareholder) shall be
taxed as dividend under FTR at the rates specified for normal dividend income whereas value of bonus
shares is taxable under FTR @ 10% of the value as mentioned above at the time of issue of bonus shares.
• The difference between the consideration received and the cost (i.e. the amount of dividend income)
shall be taxable as capital gain both in cases of bonus shares and dividend in specie.
Cost of bonus shares shall be:
➢ Ex-bonus price if tax has already been paid in case of a listed company;
➢ The value prescribed (i.e. Break-up value or face value whichever is higher), if tax is paid in case of an
unlisted company; and
➢ Zero, if no tax is paid
➢ Capital gain on disposal of shares of a public company settled through NCCPL is taxable as separate
block of income at specified tax.
➢ Capital gain on disposal of shares of a private company is taxable under normal tax structure.

Payment on account of royalty, Fee for technical services and other services received by a non-resident
Pakistan source royalty, fee for technical services and other services received by a non-resident is subject to
withholding tax at the following % which is full and final tax liability:
15% on royalty income
10% on fee for offshore digital services, fee for money transfer operations, card network services, payment
gateway services, inter-bank financial telecommunication service fee for technical services
However, royalty income is taxable under normal tax structure as business income if it is effectively connected
with a Permanent Establishment (PE) in Pakistan of the non-resident person,
Likewise, services above mentioned are also taxable under normal tax structure as business income if the
services are rendered through a PE in Pakistan
.

151 | P a g e
Definition of Royalty
"Royalty" means any amount paid or payable, however described or computed, whether periodical or a lump
sum, as consideration for –

152 | P a g e
a) the use of, or right to use any patent, invention, design or model, secret formula or process, trademark
or other like property or right;
b) the use of, or right to use any copyright of a literary, artistic or scientific work, including films or
video tapes for use in connection with television or tapes in connection with radio broadcasting, but
shall not include consideration for the sale, distribution or exhibition of cinematograph films;
c) the receipt of, or right to receive, any visual images or sounds, or both, transmitted by satellite, cable,
optic fiber or similar technology in connection with television, radio or internet broadcasting;
d) the supply of any technical, industrial, commercial or scientific knowledge, experience or skill;
e) the use of or right to use any industrial, commercial or scientific equipment;
f) the supply of any assistance that is ancillary and subsidiary to, furnished as a means of enabling the
application or enjoyment of, any such and is property or right as mentioned in sub-clauses (a) through
(e): and
g) the disposal of any property or right referred to in sub-clauses (a) through (e):
Definition of Fee for Technical Services
"Fee for technical services" means any consideration, whether periodical or lump sum, for the rendering of any
managerial, technical or consultancy services including the services of technical or other personnel, but does
not include-
consideration for services rendered in relation to a construction, assembly or like project undertaken by the
recipient; or
consideration taxable under the head "Salary"

Definition of "Permanent establishment"


"Permanent establishment" means a place of business through which the business of the person is wholly or
partly carried on, and includes-
a) a place of management, branch. Office, factory or workshop premises foe soliciting orders, warehouse,
permanent sales exhibition or sales outlet, other than a liaison office except where the office engages in
the negotiation of contracts (other than contracts of purchase);
b) Mine. oil or gas well, quarry or any other place of extraction of natural resources
ba) an agricultural, pastoral or forestry property;
bb) virtual business presence in Pakistan including any business where transactions are
conducted through internet or any other electronic medium, with or without having any physical
presence
c) a building site, a construction, assembly or installation project or supervisory activities connected with
such site or project but only where such site, project and its connected Supervisory activities continue
for a period or periods aggregating more than ninety days within any twelve-months period;
d) the furnishing of services, including consultancy services, by any person through employees or other
personnel engaged by the person for such purpose;
e) a person acting in Pakistan on behalf of the person (hereinafter referred to as the ("agent"), other than
an agent of independent status acting in the ordinary course of business as such, if the agent-
i. has and habitually exercises an authority to conclude contracts on behalf of the other person;
ii. has no such authority, but habitually maintains a stock-in-trade or other merchandise from which
the agent regularly delivers goods or merchandise on behalf of the other person; or
f) Any substantial equipment installed, or other asset or property capable of activity giving rise to incom

153 | P a g e
Profit on debt i.e. interest income
"Profit on debt" whether payable or receivable, means
➢ Any profit, yield, interest, discount, premium or other amount, owing under a debt, other than a return
of capital; or
➢ Any service fee or other charge in respect of a debt, including any fee or charge incurred in respect of
a credit facility which has not been utilized.
The following interest incomes are covered in specific provisions:

• Profit on certificates under National Savings Scheme including Defense Saving Certificates (DSC) and
Post Office Saving Account.
• Bank profit including profit and loss sharing (PLS) account
• Profit on certificates, debentures etc. including Term Finance Certificates, Certificates of Investment
issued by a company or a financial institution.
• Profit on securities issued by federal, provincial or local government such as Pakistan Investment
Bonds.
The tax rates applicable on the above interest income are:

Tax rate

For a company (normal corporate tax rate) 29%

For an individual and AOP where profit on debt: 15% on gross amount
does not exceed Rs.5 million Normal slab rates
[separate block of income]
exceeds Rs.5 million

Note for students:


Profit on debt in each case is not covered in the above specific provisions e.g. payment on account of interest on loan
through loan agreement is not covered under specific provisions and therefore the same is taxable under normal tax
structure irrespective of any limit.
Therefore, if an individual gives loan to a company through loan agreement (other than through debentures etc.)
then no tax shall be deducted in this case and the interest income of the individual shall be taxable under the head
income from other sources at normal slab rates.

Interest income on Bahbood Saving Certificates, Pensioners' Benefit account and Shuhada Family
Welfare account is not taxable under separate block of income - refer chapter of rebates.
Interest income is exempt in certain cases and therefore no tax shall be deducted. Few examples are:
➢ Profit from foreign currency accounts in Pakistan by non-resident persons.
➢ Profit from rupee account by a non-resident individual holding a Pakistan Origin Card (POC) or
National ID Card for Overseas Pakistanis (NICOP) or CNIC where foreign exchange remittance is made
exclusively for this purpose.
➢ Profit received by a non-resident on a security issued by a resident where: (section 46)

154 | P a g e
➢ The persons are not associates;
➢ The security was widely issued outside Pakistan for raising a loan for a business in Pakistan;
➢ The profit was paid outside Pakistan; and
➢ The profit was paid on any security approved by the FBR.
Zakat deducted from interest income:
No deduction including zakat is allowed where profit on debt is taxable as separate block of income. (While
can be claimed as deductible allowance.

Question
GM Ltd is considering an option to issue TFC outside Pakistan for the purpose of raising funds for use in its business in
Pakistan. To induce the investors for acquiring
TFCs of the Company, it intends to advertise that the income on the same will not be taxable in Pakistan.
Required:
List down the conditions to be fulfilled for claiming exemption on profit on debt payable on TFCs being issued by the
company. (Marks 05)

Exercise:
(xiii) Omega (Pvt.) Limited is engaged in the business of trading and sale of fertilizers. The company has
extended loan of Rs. 2.5 million to one of its shareholders on 30 June 2023 when the accumulated
profits of the company were Rs. 1.8 million. Determine the amount to be treated as dividend.
Required: Determine the amount to be treated as dividend.
(xiv) Robin Petroleum International (RPI), a company incorporated in Netherlands, is operating in Pakistan
as a branch. RPI has entered into an agreement with the Government of Pakistan under which RPI has
been given the right to explore and produce crude oil and natural gas in specified areas of Sindh and
Baluchistan.
Required: Explain the tax implications on RPI’s branch in Pakistan of the remittance of the after tax profits
of the branch to its head office in the Netherlands.
(xv) Mr Z has received dividend Rs.75,000 net of Zakat Rs.10,000 and tax @15%.
Required: Determine the amount to be treated as dividend.
Answer
a) The amount of loan to the extent of accumulated profits will be treated as dividend i.e. Rs. 1,800,000
b) Remittance of after- t a x profit of a branch of a foreign company operating in Pakistan is considered
as dividend. However, remittance of after-tax profit by a branch of Petroleum Exploration and Production
(E&P) foreign company operating in Pakistan is excluded from the definition of dividend. As the Pakistan
branch of RPI is a branch of an E & P foreign company operating in Pakistan, the remittance of the after tax
profits of RPI’s branch in Pakistan would not be considered to be dividend income.
c) Total taxable dividend is Rs.100,000 i.e. (75,000+10,000)/85%

155 | P a g e
Exercise
Certain types of payments by a private company to its shareholders can be treated as "dividend" under the
Income Tax Ordinance, 2001.
State the conditions necessary for the application of this rule and the exceptions to it.

Answer
The following types of payment made by a private company as defined under the Companies Act, 2017 may
be treated as "dividend", to the extent to which the company possesses accumulated profits:
Payment by way of advance or loan to a shareholder; or
Any payment for the individual benefit of any shareholder.
EXCEPTIONS:
Any advance or loan made to a shareholder in the ordinary course of the business, where lending of money is
a substantial part of the business of the company.
Any dividend paid which is set off by the company against any amount previously paid and treated as dividend

Exercise
Rose Company Ltd has a paid up capital of Rs.5 million consisting 500,000 shares of Rs.10 each. On
30.6.2001 the company's balance sheet shows accumulated profits of Rs.1.5 million. Last year the company
also created a reserve of Rs.1 million for issue of bonus shares. The company has to be liquidated. The
official liquidator realized Rs.6.5 million and distribution among the shareholders was made at the rate of
Rs.13 per share. Shewani Group owns 200,000 shares in the company. How much of the amount received
by Shewani Group is dividend? Please explain your answer

Answer

3 x 200,000 = Rs.600,000 shall be treated as dividend.


According to the Income Tax Ordinance, any distribution by a company to its shareholders on
liquidation to the extent of accumulated profits immediately before its liquidation shall be
treated as dividend.
Therefore, distribution @ Rs.3 per share is taxable as dividend.

Exercise
A company engaged in manufacturing activities has decided to provide loan to one of its shareholders. Explain
the tax implication on the company as well as the shareholder if the Company:
i. Is registered under the Companies Act as a private limited company.
ii. Is an unlisted public company.

(i)If the company is a private company as defined in the Companies Act


Tax implications on shareholders
The definition of Dividend includes any payment by a private company by way of loan to its shareholder for
the individual benefit of the shareholder, to the extent of accumulated profits. Accordingly, amount received
by the shareholder shall be considered as dividend in the hands of the shareholder and the tax rate applicable
on dividend shall apply accordingly.
Tax implications on private limited company
Company being a withholding agent is responsible to deduct withholding tax at the specified rate from the
payment being considered as dividend.
(ii)If the company is unlisted public company
In this case loan to a shareholder is not included in the definition of dividend and therefore no tax implications
for the shareholder or the company.

156 | P a g e
Exercise
Tamba Pakistan (Pvt) Ltd is engaged in the manufacture of pharmaceutical products. Its board of directors
has approved a 3-year loan to one of its major shareholders.
Required:
Explain the tax implications of the above transaction on the company as well as the shareholder

Answer
Tax implications on shareholders
The definition of Dividend includes any payment by a private company by way of loan to its shareholder
for the individual benefit of the shareholder, to the extent of accumulated profits. Accordingly, amount
received by the shareholder shall be considered as dividend in the hands of the shareholder and the tax rate
applicable on dividend shall apply accordingly.
Tax implications on private limited company
Company being a withholding agent is responsible to deduct withholding tax at the specified rate from the
payment being considered as dividend.

Exercise
On 25.8.20X8, the Officer of Inland Revenue has issued a notice to Rahat Foods (Private)
Ltd (RFPL) to deposit withholding income tax of Rs.1,950,000 in respect of loan
amounting to Rs.13,000,000 given to Mr. Nadeem, a shareholder of RFPL, by treating the
amount of loan as dividend. The notice was served to the company on 30.8.20X8.
According to RFPL's records, the loan was given to Mr. Nadeem on 25.5.20X7 when
accumulated profit of the company was Rs.12,000,000.
In the light of the provisions of the Income Tax Ordinance, 2001 explain whether you
agree with the notice issued to RFPL by the Officer of Inland Revenue.
Solution
The definition of dividend as per Income Tax Ordinance, 2001 includes any payment by a
private company by way of advance or loan to a shareholder or any payment by any such
company on behalf, or for the individual benefit, of any such shareholder, to the extent of
accumulated profits of the company.
Therefore, the tax officer is correct to the extent of treating the loan as dividend. However, he
made error in treating the entire amount of Rs.13 million as dividend because the amount of
accumulated profit was Rs.12 million on that date. Therefore, only Rs.12 million can be treated
as dividend.

157 | P a g e
CHAPTER 07
DEDUCTIBLE ALLOWANCES AND
REBATES
Tax credits /rebates
What is rebate or tax credit?
Rebate or tax credit is a tax benefit whereby tax liability is reduced in specified situations according to the
specified percentage or formula such as donation given to an approved charitable organization.
The following tax credits / rebates are allowable:
a) Full time teacher allowance - clause 2 Part III 2nd Schedule
b) Tax credit on profit of Bahbood Savings Certificates or Pensioners' Benefit account and Shuhada Family
Welfare Account maintained with National Savings Centres -
c) Foreign tax credit (also called Unilateral relief or Cross Border Transaction Relief)-section 103
d) Tax credit:
i. on charitable donations - section 61
ii. on approved Pension Fund - section 63
iii. for point of sale machine - section 64D

Format & Summary of Rebates


Total Taxable Income Under NTR only XXX

Less: Deductible allowances


• Zakat Paid under Zakat & Ushr Ordinance (Sec-60) (X)
• Worker’s Welfare Fund (WWF) (Sec-60A) (X)
• Worker’s Profit Participation Fund (WPPF) (Sec-60B) (X)
• Deductible allowance for educational expense (Sec-60D)
➢ 5% of tuition fee paid by the individual
➢ 25% of Taxable Income
➢ Rs. 60,000 multiplied by the number of children of the individual
lower one will be allowed (X)

Total Taxable Income Under NTR only XXX

Taxation ( Salaried/ Non-Salaried)


Tax Liability Under NTR XXX

Tax Liability XXX


Less Tax Credits
Full Time Teacher or a Researcher Allowance (Separate Working) (X)
Foreign Tax Credit (X)

Average Relieves:

158 | P a g e
Donations to Charitable Institutions (61) K
Contribution to an Approved Pension Fund (63) M

Tax credit on point of sale machine (64D) Y

𝑇𝑎𝑥 𝐴𝑠𝑠𝑒𝑠𝑠𝑒𝑑
Average Relieves = 𝑥 (𝐾, 𝑀)
𝑇𝑎𝑥𝑎𝑏𝑙𝑒 𝐼𝑛𝑐𝑜𝑚𝑒

Add: Separate Block Income


Capital Gain U/S 37-A XXX
Gain on disposal of property XXX
Golden Handshake XXX
Add: Tax on FTR XXX
Total Tax liability XXX

Less : Tax Deducted at source (XXX)


Less: FTR (XXX)
Less: All WHT (XXX)
XXX
Add: Tax Credit disallowed (If any) (X)

Tax Payable XXX

Type of Credit Conditions Necessary Calculation of Credit


Full Time Teacher or He should a full time teacher or a researcher,
Researcher Employed in a nonprofit education or research Tax liability of salary x 25%
Allowance institution.
(Clause 2 part III of Institution is duly recognized by Higher Education
2nd Schedule) Commission (HEC),
(FTTA) a Board of Education or
a University recognized by the HEC, including
government training and research institution,

Foreign Tax Credit A person must be resident Lower of


u/s 103 (FTC) Foreign income is taxable in Pakistan Actual Tax Paid
There is no tax treaty between both countries. Or
Tax is paid within two years after the end of the tax The Pakistan tax payable in
year in which the foreign income to which the tax respect of the income at average
rate of tax:
relates. 𝐹𝑜𝑟𝑒𝑖𝑔𝑛 𝑆𝑜𝑢𝑟𝑐𝑒 𝐼𝑛𝑐𝑜𝑚𝑒
𝑥 𝑇𝑎𝑥
𝑇𝑜𝑡𝑎𝑙 𝑇𝑎𝑥𝑎𝑏𝑙𝑒 𝐼𝑛𝑐𝑜𝑚𝑒

Average Relieves

159 | P a g e
Type Conditions Amount Lower of Rs.
Paid by a crossed cheque or in kind to: Actual amount
Any board of education, educational institution Or
S 61
or any university in Pakistan run by Govt.
Donations to Charitable 30% of taxable income K
any govt. hospital or relief fund
Institutions
Any non-profit organization. (15%/10% for Associates)
Entities mentioned in 13th Schedule
An Individual holds NTN or Valid CNIC. Actual amount
Driving income under the head salary or or
S 63 20% of taxable income
income from business.
Contribution to an M
Contributes or pay premium in the year in
Approved Pension Fund
approved pension fund under the Voluntary
Pension System Rules, 2005.

S 64 Any Person who is required to integrate with Purchase cost of POS


Tax Credit on Point of FBR’s system for real time reporting of sale or machine
Y
Sales Machine receipt shall be entitled to tax credit on Rs. 150,000 per machine
purchase cost of POS machine

Explanation of Deductible Allowances and Rebates


Deductible allowances (Sec 60, 60A, 60B & 60D)
Definition:
“Deductible allowance” means an allowance that is deductible from total income
Zakat (Sec 60)
A person is entitled to a deductible allowance for the amount of any Zakat paid by the person in a tax year under
the Zakat and Ushr Ordinance, 1980.
Where the Zakat has been deducted out of the profit on debt (which is chargeable under the head “income from
other sources”), such Zakat shall not be deducted out of the total income, rather, it shall be allowed as a
deduction while computing income from other sources.
Where the amount of Zakat is more than total income, the excess amount shall not be refunded or carried forward
or carried back.
Worker’s welfare fund (Sec 60A)
A person shall be entitled to a deductible allowance for the amount of any Workers’ Welfare Fund (WWF) paid
by the person in the tax year under Workers’ Welfare Fund Ordinance, 1971 or under any law relating to the
Workers’ Welfare Fund enacted by Provinces after the Eighteenth Constitutional Amendment Act, 2010.
However, no deductible allowance will be allowed where any amount is paid to provinces by trans-provincial
organizations (a person having operations in more than one province).

Worker’s participation fund (Sec 60B)


A person shall be entitled to a deductible allowance for the amount of any Workers’ Participation Fund paid by
the person in a tax year in accordance with the provisions of the Companies Profit (Workers’ Participation) Act,
1968 under any law relating to the Workers’ profit participation Fund enacted by Provinces after the eighteenth
Constitutional Amendment Act, 2010. However, no deductible allowance will be allowed where any amount is
paid to provinces by trans-provincial organizations (a company having operations in more than one province).
Deductible allowance for education expenses (Sec 60D)
Every individual shall be entitled to a deductible allowance in respect of tuition fee paid by the individual in a
tax year provided that the taxable income of the individual is less than Rs.1,500,000.
(xvi) The amount of an individual ‘s deductible allowance allowed for a tax year shall not exceed the
lesser of —
• 5% of the total tuition fee paid by the individual in the year;

160 | P a g e
• 25% of the person’s taxable income for the year; and
• an amount computed by multiplying 60,000 with number of children of the individual.
(xvii) Any allowance or part of an allowance for a tax year that is not able to be deducted for the year shall
not be carried forward to a subsequent tax year.
(xviii) Allowance shall be allowed against the tax liability of either of the parents making payment of the
fee on furnishing national tax number (NTN) or name of the educational institution.
(xix) Allowance shall not be taken into account for computation of tax deduction from Salary under section
149.

Tax Credits and Rebates


Full time teacher allowance:
➢ A full-time teacher or a researcher of a recognized non-profit educational or research institution
including government research institutions shall be allowed a reduction of 25% of tax payable.
➢ The institution must be recognized by Higher Education Commission or a Board of Education or a
University recognized by Higher Education Commission.
➢ This additional tax reduction would be allowed on tax liability on taxable salary. Other income, if any,
would be excluded for this purpose.
➢ This rebate shall not apply to teachers of medical profession who derive income from private medical
practice or who receive share of consideration from patients.
Explanation
Full time teacher means a person employed purely for teaching and not performing any administrative or
managerial jobs e.g. principals, headmasters, directors, vice-chancellors, chairmen, controllers etc.
Similarly a full time researcher means a person purely employed for research job only in a research institution
and such institution is purely performing research activities.

Example
Mr. B has disclosed the following income for the tax year 20X8:
– Taxable salary as a full time teacher from an approved non-profit educational institution Rs.3,300,000
– Taxable other sources Rs.500,000
Calculate tax liability.

Solution
Mr. B
Tax year 20X8
Computation of taxable income and tax liability
Taxable salary as a full time teacher 3,300,000
Taxable other sources 500,000
Taxable income 3,800,000
Tax liability (Salaried Case)
Tax on Rs.3,600,000 435,000
Tax on Rs.200,000 @ 27.5% 55,000
490,000
Less: Full time teacher allowance (as per working) 91,875
Tax liability 398,125
Working of full time teacher allowance: Salary Income 3,300,000
Tax on Rs.2,400,000 165,000
Tax on Rs.900,000 @ 22.5% 202,500
367.500
Full time teacher allowance 25% of Rs.367,500 91,875

161 | P a g e
Example
Mr. Z has disclosed the following income for the tax year 20x8:
– Taxable salary as a full time teacher from an approved non-profit educational institution Rs.5,800,000
– Taxable other sources Rs.1,700,000
Calculate tax liability.
Solution
Rs. 1,360,000.

Tax credit for profit on Bahbood Savings Certificates or Pensioners' Benefit Account or Shuhada
Family Welfare Account
a) Profit on Certificates issued by, or accounts maintained with, National Savings Centres is taxable at
15% fixed tax and therefore normal slab rates do not apply on such profit.
Withholding tax shall also be deducted by the National Saving Centres at the time of payment of profit
on debt.
b) Profit on Bahbood Savings Certificates issued by, or Pensioners' Benefit Account and Shuhada Family
Welfare Account maintained with, National Savings Centres (such profit) is not subject to withholding
tax and normal slab rates apply on such profit.
However, tax rate shall not exceed 5% of such profit which means that if average rate of tax on income
including such profit is more than 5% then tax credit on such profit would apply to reduce the tax on
5% on such profit.
[Average rate of tax is A/B where A is the tax liability before tax credit for profit on Bahbood
Certificates etc. and B is the taxable income]

Example
Mr. E disclosed the following income during the tax year 20X8:
Income from business Rs.3,3000,000
Profit on Bahbood Savings Certificate Rs.1,000,000
Calculate tax liability.

Solution
Income from business 3,300,000
Profit on Bahbood Savings Certificate Taxable income 1,000,000
Tax liability (Non-Salaried Case) 4,300,000
Income tax on Rs.4,000,000 765,000
Income tax on Rs.300,000 @ 35% 105,000
870,000
Tax credit on Profit on Bahbood Savings Certificate
Tax on profit on certificate at average tax rate
(870,000/4,300,000) x 1,000,000 202,326
Tax @ 5% on profit on bahbood certificates 50,000
Tax credit 152,326
Tax liability 717.674

Example
Mr. F disclosed the following income during the tax year 20X8:
Salary income Rs.4,200,000
Profit on Bahbood Savings Certificate Rs.600,000
Calculate tax liability.
Solution
Tax liability Rs.699,375

162 | P a g e
Foreign Tax Credit
Section 103 of the Ordinance lays down different aspects of foreign tax credit in the following
manner:
1) In case a resident taxpayer derives foreign source income chargeable to tax in respect of which he has paid
foreign income tax, the taxpayer shall be allowed a tax credit of an amount equal to the lesser of:
a) the foreign income tax paid; or
b) the Pakistan tax payable in respect of the income.
2) The Pakistan tax payable in respect of foreign source income shall be computed by applying the average rate
of Pakistan income tax applicable to the taxpayer for the year against the taxpayer’s net foreign source income
for the year.
3) If a taxpayer has foreign income under more than one head of income, this principle shall apply separately to
each head of income.
4) Income derived by a taxpayer from carrying on a speculation business shall be treated as a separate head of
income.
5) If a person has more than one type of tax credits available to him, these shall be applied in the following
order:
a) Foreign tax credit u/s 103
b) Tax credit/rebate on donations, investment, enlistment, etc.,
c) Advance tax and tax deducted/collected at source
6) Any tax credit or part of a tax credit allowed for a tax year which is not credited shall not be refunded, carried
back to the preceding tax year, or carried forward to the following tax year.
7) A foreign tax credit shall be allowed only if the foreign income tax is paid within two years after the end of
the tax year in which the foreign income to which the tax relates was derived by the resident taxpayer.

Example C:
Mr. Junaid's income for the tax year 20X8 is as under:
Taxable Pakistan source income:
Salary 1,800,000
Other source 1,000,000
Foreign source business income taxable in Pakistan 1,500,000
Foreign tax paid in the foreign country 330,000
Calculate tax liability.
Solution C:
Rs.566,512

Definition: Average rate of Pakistan Income Tax


“Average rate of Pakistan income tax” in relation to a taxpayer for a tax year, means the percentage that the
Pakistani income tax (before allowance of the tax credit under this section) has of the taxable income of the
taxpayer for the year;

• “foreign income tax” includes a foreign withholding tax; and


• “net foreign-source income” in relation to a taxpayer for a tax year, means the total foreign-source
income of the taxpayer charged to tax in the year, as reduced by any deductions allowed to the taxpayer
for the year that:
• relate exclusively to the derivation of the foreign-source income; and
• are reasonably apportioned to the derivation of foreign-source income

Rebate on Donations - Section 61


A person shall be entitled to a tax credit / rebate on the donation, voluntary contribution or subscription in cash
through banking channel or in kind i.e. any property is given by the person to the following:

163 | P a g e
a) Any board of education or university in Pakistan established under any law.
b) Any educational institution, hospital or relief fund in Pakistan established or run by Federal, provincial or
local government.
c) Any non-profit organization, trust or welfare organization approved by the tax authorities established for the
purpose of religious, education, charitable, welfare or for promotion of sports.
d) Entities, organizations and funds mentioned in 13th Schedule to the Income Tax Ordinance, 2001 such as
The Citizen Foundation and Pakistan Disabled Foundation.
"Charitable purpose" includes relief of the poor, education, medical relief and the advancement of any other
object of general public utility.
"Non-profit organization" means any person other than an individual, which is:
(a) Established for religious, educational, charitable, welfare purposes for general public, or for the
promotion of an amateur sport;
(b) Registered under any law as a non-profit organization [such as under Trust Act or section 42 of the
Companies Act 2017];
(c) Approved by the Commissioner for specified period; and
(d) None of its assets confers a private benefit to any other person.

Limit on the amount eligible for rebate


Tax credit shall be allowed at average rate of tax on the lower of the following:
i. Actual amount of donation or FMV of property at the time it is given; or
ii. 30% of taxable income (20% in case of a company)

Note for students:


Rebate on donation is allowed at average rate of tax which is A/B where:
A is the tax liability before rebate on donation; and
B is the taxable income.
Same average rate of tax shall be taken for the purpose of rebates under sections 61 and 63 i.e. rebate on
donations and approved pension fund.

However, where donation is given to an associate by the donor then tax credit shall be allowed at average rate
of tax on the lower of the following:
i. actual amount of donation or FMV of property at the time it is given; or
ii. 15% of taxable income (10% in case of a company)

Value of donation in kind


Value of items donated in kind shall be taken as per Rule 228(4) of the Income Tax Rules 2002, summary of
which is as under:
a) Imported items:
Value for the purpose of custom duty along with all duties and charges paid by the donor
b) Items manufactured in Pakistan:
Purchase price along with duties and charges paid by the donor
c) Used depreciable items:
Tax Written down Value (WDV) i.e. cost - tax depreciation
d) Motor vehicles:
i. New vehicles imported by the donor shall be valued at CIF value (ie. Cost Insurance and Freight) plus
all duties and charges till their registration.
ii. New vehicles locally purchased shall be valued at price paid by the donor plus al duties and charges till
their registration.
iii. Used vehicles imported by the donor shall be valued at the import price adopted by the Custom
Authorities plus all charges and duties till their registration.

164 | P a g e
iv. Value adopted in the first year shall be reduced by 10% of the said value (e on straight line basis) for
each successive year up to a maximum of 5 years.
v. Used vehicles locally purchased shall be valued as:u
• If vehicles are up to 5 years old, value shall be original cost as reduced by 10% for every year
following the year in which it was imported or purchased
• If vehicles are more than 5 years old, value shall be purchase price paid by the donor for the
used car or 50% of the original value whichever is higher
e) Other items:
Fair market value as determined by the Commissioner. The most common example in this category is
used personal assets.

Example
Mr. Akhtar disclosed the following income and expenditures relevant to the tax year 20X8
❖ Taxable salary Rs.4,000,000 from a non-profit organization as a full time teacher
❖ Taxable other source Rs.1,020,000
❖ Cheque received under inheritance Rs.8,200,000
❖ Zakat paid Rs.148,000
❖ Donations to approved charitable organizations through banking channel Rs.1,500,000
Calculate tax liability of Mr. Akhtar for the tax year 20X8.
Solution
Mr. Akhtar
Tax year 20X8
Computation of taxable income and tax liability
Taxable salary 4,000,000
Taxable other source 1,020,000
Amount received under inheritance 8,200,000 exempt
Total income 5,020,000
Less: Zakat paid 148,000
Taxable income 4,872,000
Tax liability (Salaried Case) 435,000
Tax on Rs.3,600,000 349.800
Tax on Rs.1,272,000 @ 27.5% 784,800
Less: Full time teacher allowance as per working 136,250
648,550
Less: Rebate on donation
Amount eligible for rebate; lower of the following:
- Actual donation 1,500,000
-30% of taxable income 1,461,600
Rebate 648,550/4,872,000 x 1,461,600 194,565
Tax liability 453.985
Working of full time teacher allowance:
Salary Income 4,000,000
Tax on Rs.3,600,000 435,000
Tax on Rs.400,000 @ 27.5% 110,000
545,000
Full time teacher allowance 25% of Rs.545,000 136,250

Example
Mr. Fazal disclosed the following income and expenditures relevant to the tax year 20X8 # Taxable salary
Rs.1,700,000 from a non-profit organization as a full-time teacher
# Taxable other source Rs.1,320,000

165 | P a g e
# Cheque received under inheritance Rs.5,000,000
# Zakat paid Rs.148,000
# Donations to approved charitable organizations through banking channel Rs.900,000 Calculate tax liability
of Mr. Fazal for the tax year 20X8.
Solution
Rs.289, 538

Allowance for contribution to Approved Pension Fund (APF) - Section 63


An eligible person deriving income under the head "salary" or "income from business" is entitled to a tax
credit on contribution or premium paid to APF under Voluntary Pension System Rules at the average rate of
tax on the lower of:
i. total contribution or premium paid during the year;
ii.20% of taxable income of the current tax year.

Eligible person-
An individual Pakistani who holds a valid National Tax Number (NTN) or Computerized National
Identity Card (CNIC).
Transfer of existing balance of an account to another account under Voluntary Pension Scheme Rules shall
not qualify for rebate. It means that the original investment by a person shall be eligible for such rebate.

Tax credit for point-of-sale machine

(1) Any person who is required to integrate with FBR's computerized system for real time reporting of sale or
receipt, shall be entitled to tax credit on purchase cost of point-of-sale machine.
"Point of sale machine" means a machine meant for processing and recording the sale transactions for goods
or services, either in cash or through credit and debit cards or online payments in an internet enabled
environment.

(2) The amount of tax credit for a tax year in which point of sale machine is installed, integrated and
configured with the FBR's computerized system shall be lesser of -
i. Purchase cost of point-of-sale machine; or
ii. Rs.150,000 per machine.
If the amount of tax rebates is more than the tax liability then no refund shall be allowed nor is the same
allowed to be carried forward or carried back (except few cases as mentioned in chapter of losses).
However, if tax paid or deducted at source is more than the tax liability then the excess shall be refunded or
carried forward.

Transfer of rebate to an AOP by a member


If the person is a member of an AOP, any tax credit that could not be claimed by such member can be claimed
by the AOP of which he is member, in the same year.
For this purpose, a copy of written agreement between the member and the AOP shall be furnished along with
the return of income of AOP [A return of income is a document prescribed by the FBR which is used by the
taxpayer to declare his taxable income and tax liability - refer chapter 9].

Question 1:
Mr. Z, a resident person for tax purposes, requested you to calculate his tax liability from the following
information:

166 | P a g e
i. Taxable salary Rs.16,000,000 as a full-time teacher from an approved non-profit educational
institution
ii. Taxable other source Rs.2,000,000
iii. Taxable business income from a foreign country Rs.1,500,000 where Mr. Z paid income tax of
Rs.200,000. There is no tax treaty with that foreign country.
iv. Zakat was deducted at source Rs.70,000. He also paid zakat of Rs.230,000 to his relatives
v. Donation was paid by him Rs.110,000 to a private approved charitable institution in cash for which he
has a proper receipt
vi. He donated his household furniture to a government hospital. FMV is estimated at Rs.1,050,000

Question 2:
Mr. Y requested you to calculate his tax liability from the following information:
i. Taxable salary as a full-time teacher from a recognized non-profit educational institution
Rs.8,800,000
ii. Taxable income from other sources Rs.5,100,000
iii. Zakat deducted by the bank Rs.122,000
iv. Mr. Y received Rs.8,000,000 through inheritance
v. He made the following donations:
a) Donation to a government hospital in cash Rs.120,000
b) Donation to a government educational institution through cheque Rs.80,000
c) Donation to a private charitable approved institution through cheque Rs.1,200,000
vi. He joined an approved pension fund (APF) during the year and contributed Rs.2,850,000 towards
APF.

Answer to Q.1
Mr.Z-Resident person
Pakistan source income
Salary as a full time teacher 16,000,000
Income from other source 2,000,000
Foreign source income 1,500,000
Total income 19,500,000
Less: Zakat deducted at source 70,000
Taxable income 19,430,000
Tax liability (Salaried Case)
Tax on Rs.6,000,000 1,095,000
Tax on Rs.13,430,000 @ 35% 4,700,500
5,795,500
Less: Full time teacher allowance (as per working) 1,148,750
4,646,750
Less: Foreign tax credit
a) tax paid in foreign country 200,000
b) (4,646,750 ÷ 19,430,000) x 1,500,000 358,730
whichever is lower 200,000
4,446,750
Less: Rebate on donation
4,446,750 ÷ 19,430,000 x 1,050,000 240,303
Tax liability 4,206,447

Working of full time teacher allowance:


Salary Income 16,000,000
Tax on Rs.6,000,000 1,095,000

167 | P a g e
Tax @ 35% on Rs.10,000,000 3,500,000
4,595,000
Full time teacher allowance 25% of Rs.4,595,000 1,148,750
Notes:
-Zakat paid to relative is not an allowable deduction.
-Donation to a private approved charitable institution is not entitled for rebate/tax credit if paid in cash

Answer to Q.2
Mr. Y
Salary 8,800,000
Income from other sources 5,100,000
Total income 13,900,000
Less: Zakat deducted at source 122,000
Taxable income 13,778,000
Tax liability (Non-Salaried Case)
Tax on Rs.4,000,000 765,000
Tax on Rs.9,778,000 @ 35% 3.422,300
4,187,300
Less: Full time teacher allowance (as per working) 518,750
3,668,550
Less: Rebate on donation 340,815
3,668,550 ÷ 13,778,000 x 1,280,000 3,327,735
Rebate on APF
3,668,550 ÷ 13,778,000 x 2,755,600 733,710
Tax liability 2,594,025

Working of full-time teacher allowance:


Salary Income 8,800,000
Tax liability (Salaried case)
Tax on Rs.6,000,000 1,095,000
Tax on Rs.2,800,000 @ 35% 980,000
2,075,000
Full time teacher allowance 25% of Rs.2,075,000 518,750

Notes:
– Amount received through inheritance is capital receipt and therefore not taxable.
– Donation to a Government Hospital in cash is not entitled for rebate.
– Rebate on approved pension fund is allowable on the lower of the following amounts:
✓ Actual contribution Rs.2,800,000; or
✓ 20% of taxable income which is Rs.2,755,600

PAST PAPER QUESTIONS


Question # 1 Autumn 2009 Q. 2(b)

State the provisions of Income Tax Ordinance, 2001 pertaining to foreign tax credit available to a resident
taxpayer. (06)

Question # 2 Spring 2010 Q. 2

168 | P a g e
Mr. Qamar intends to donate an amount of Rs. 10 million to certain educational and welfare institutions. In
your capacity as his tax consultant, explain the tax relief which may be available in respect of such donation
and the conditions he must fulfill to avail such relief. (09)

Question # 3 Autumn 2017 Q. 2(b, c)

(c) Determine the amount of deductible allowance that a resident individual can claim on account of
education expenses, if his taxable income for the year was Rs. 800,000 and he paid monthly fee of Rs.
6,000 per child for his three children. (02)

Question # 4 Autumn 2020 Q. 3(b)

Differentiate between deductible allowances and admissible deductions. Give three examples of each. (06)

Question # 5 Spring 2021 Q. 4

(b) Briefly explain the provisions of the Income Tax Ordinance, 2001 and Rules made thereunder relating to:
(iii) order of application of various tax credits if a taxpayer is allowed more than one tax credit for a tax
year. (03)

Question # 6 Spring 2023 Q. 2


Taha who has recently joined a tax consultancy firm, prepares the following table with regard to
taxability of interest income earned by various persons during the tax year 2023:

Taxability of interest income


Amount of
Name of Mode of Admissibility
Status interest income Head of Tax
person investment of related
(Rs. in million) income regime
expense
Resident Term Deposit Income from
Aatif 1.1 FTR Yes
Individual Receipts of a bank other sources
Resident Term Deposit Income from
Bilal 5.5 NTR Yes
Individual Receipts of a bank other sources
Resident Bahbood Savings Income from
Seema 5.2 Exempt No
Individual Certificate other sources
Non- Securities issued
Income from
Kamal resident *3.3 by the resident NTR Yes
other sources
Individual person
Loan agreement
Resident with Association Income from
Sikandar 4.4 NTR Yes
Individual of Persons being business
its member
Dream Resident Term Finance
Income from
Bank Banking 10.1 Certificates of a NTR Yes
other sources
Limited Company company
*Profit was paid outside Pakistan on approved securities which were widely issued outside Pakistan.

Required:
Prepare the corrected ‘Taxability of interest income’ columns in the table. (07)

ANSWERS

169 | P a g e
Question # 1 Autumn 2009 Q. 2(b)

(i) In case a resident taxpayer derives foreign source income chargeable to tax in respect of which he has
paid foreign income tax, the taxpayer shall be allowed a tax credit of an amount equal to the lesser of:
▪ the foreign income tax paid; or
▪ the Pakistan tax payable in respect of the income.
(ii) The Pakistan tax payable in respect of foreign source income shall be computed by applying the average
rate of Pakistan income tax applicable to the taxpayer for the year against the taxpayer’s net foreign
source income for the year.
(iii) If a taxpayer has foreign income under more than one head of income, this principle shall apply
separately to each head of income.
(iv) Income derived by a taxpayer from carrying on a speculation business shall be treated as a separate head
of income.
(v) If a person has more than one type of tax credits available to him, these shall be applied in the following
order:
▪ Foreign tax credit u/s 103
▪ Tax credit/rebate on donations, investment, enlistment, etc.,
▪ Advance tax and tax deducted/collected at source

(vi) Any tax credit or part of a tax credit allowed for a tax year which is not credited shall not be refunded,
carried back to the preceding tax year, or carried forward to the following tax year.
(vii) A foreign tax credit shall be allowed only if the foreign income tax is paid within two years after the end
of the tax year in which the foreign income to which the tax relates was derived by the resident taxpayer.

Question # 2 Spring 2010 Q. 2

Qamar shall be entitled to a tax credit in respect of any sum paid in the tax year as a donation to
(i) Any board of education or any university in Pakistan established under a Federal or a Provincial
(ii) Any educational institution hospital or relief fund established to run in Pakistan by Federal Government
or a Provincial Government or a local authority; or
(iii) any non-profit organization or any persons eligible for tax credit under section 100C;
(iv) entities, organizations and funds mentioned in 13th Schedule of the Ordinance.

The amount of Mr. Qamar's tax credit allowed for the tax year shall be computed according to the
following formula, namely:-

(A/B) x C
A is the amount of tax assessed to him for the tax year before allowance of any tax credit
B is his taxable income for the tax year; and

C is the lesser of-


(i) the total amount of donations
(ii) (ii) 30% of the taxable income of the individual for the year

If amount is donated by an associate C shall be 15% of taxable income. The benefit of a cash donation
shall be allowed only if it is paid by a crossed cheque

Question # 3 Autumn 2017 Q. 2(b, c)

170 | P a g e
The amount allowed as educational expense should be the lesser of:
Rupees
(i) 5 % of the total tuition fee paid (6,000×12×3×5%) 10,800
(ii) 25% of the person’s taxable income (800,000×25%) 200,000
(iii) An amount equal to 60,000 × 3 children 180,000

Therefore, the amount allowed is Rs. 10,800.

Question # 4 Autumn 2020 Q. 3(b)

An admissible deduction is specific to a particular head of income and is deducted from income under that
head. If admissible deductions are greater than the income derived under a particular head it will create a
loss which may be set off against income under any other head or may carried forward to a subsequent year.
Some examples of admissible deductions are:
▪ Depreciation expense
▪ Rent expense
▪ Salary expense

A deductible allowance is deducted from the total income and if it is higher than the total income it will be
restricted to the amount of total income. Any deductible allowance in excess of the total income cannot be
carried forward to a subsequent tax year. Examples of deductible allowance are:
▪ Zakat
▪ Workers welfare fund
▪ Workers participation fund

Question # 5 Spring 2021 Q. 4

If at the time of calculating income tax payable, a person has more than one type of tax credits available,
these tax credits shall be applied in the following order:
(1) Foreign tax credit
(2) Tax credit on donations, investment, etc.,
(3) Quarterly advance tax and tax deducted/collected at source

Following are the tax credits available under Part X Chapter III of the Ordinance, for;
(i) Charitable donations (Sec 61)
(ii) Contribution to approved pension fund (Sec 63)
(iii) Certain person (Sec 65F)
(iv) Specified industrial undertakings (Sec 65G)
(v) Point of sale machines (Sec 64D)

Question # 6 Spring 2023 Q. 2

Admissibility
Name of person Head of income Tax regime of related expenses
Atif - FTR No
Income from other
Bilal NTR Yes
sources

171 | P a g e
Income from other
Seema NTR Yes
sources
Kamal - Exempt No
Exempt (Include in income
Sikandar - No
just for rate purpose)
Dream Bank Income from business NTR Yes

172 | P a g e
CHAPTER 08
INCOME TAX PRACTICAL
QUESTIONS OF INDIVIDUALS
Notes for students:
1. Certain dates and information have been changed to solve the questions based on the tax year 2024
2. An ambiguity exists as to whether income under separate block of income should be considered for rebate
purposes and to decide salaried or non-salaried case.
In this case two opinions may be formed as under:
• Separate block of income should be considered for rebate purposes and so on as there is no negative
provision in the Ordinance as compared to income taxable under Final Tax Regime (FTR)
• Separate block of income should not be considered for rebate purposes and so on as the rates on
separate block of income are charged normally on gross income such as tax on gross interest or
charged on income after considering specific expenses such as capital gain on disposal of immovable
property and capital gain u/s 37A.
• ICAP does not consider separate block of income for rebate purposes and so on and therefore we are
also not considering the same in our solutions. Reference for this purpose is: ICAP's Practice Kit for
CAF 2022 Q.27.
3. Basic knowledge and features of Final Tax Regime (FTR) are included in the syllabus. Detailed knowledge,
items covered under FTR and their rates are not included in the syllabus.
Therefore, the students are supposed to have basic knowledge of FTR before attempting the practical
questions of individuals. We are explaining FTR's features in this chapter to facilitate the solution of practical
questions of individuals

Salient features of FTR Section 8 and Section 169


a) Tax deductible or collectible at source related to the items of FTR shall become full and final discharge of
tax liability and normal slab rates do not apply in this case. However, any incorrect tax deduction shall be
adjusted accordingly.
b) No expense, deduction or allowance against presumed income is allowed.
c) Losses cannot be adjusted against presumed income.
d) Tax liability under FTR shall not be reduced by any tax credit / rebate.

173 | P a g e
Income Tax Numerical Format
Mr. Name
Resident Individual
Tax year 20XX

Sr. Taxable
No Particulars Rs. Rs
Calculation of Taxable Income and Tax Payable

1. Income from Salary


Allowances X
Perquisites X
Terminal Benefits X
Employee Share Scheme X
Total Income from Salary XXX

2 Income from Property


S.U.R.F X
Less: Allowable deduction (X)
Total Income from Property XXX

3 Income from Business


Revenue X
Less: Expenditure (X)
Total Income from Business XXX

4 Income from Capital Gain


Gain U/S 37 X
Gain U/S 38 X
Gain U/S 37-A (SBOI) X
Gain immovable property (SBOI) X
Total Income from Capital Gain (Including SBOI) XXX

5 Income from Other Source


Other source income under FTR X
Other Source income under NTR X
Income from other source (Including FTR) XXX

Total Income (NTR,FTR & SBOI only) XXX


Less : Separate Block of Income & FTR
Golden Handshake (X)
Capital Gain U/S 37-A (X)
Capital Gain on immovable property (X)

174 | P a g e
FTR Income (X) (XXX)
Total Income Excluding (SBOI and FTR) XXX

Less: Deductible allowances


Zakat Paid under Zakat & Ushr Ordinance (Sec-60) (X)
Worker’s Welfare Fund (WWF) (Sec-60A) (X)
Worker’s Profit Participation Fund (WPPF) (Sec-60B) (X)
Deductible allowance for educational expense (Sec-60D)
➢ 5% of tuition fee paid by the individual
➢ 25% of Taxable Income
➢ Rs. 60,000 multiplied by the number of children of the
individual lower one will be allowed (X)
Total Taxable Income (NTR Only) XXX

Taxation [ Sal/Non Sal]


(Income x Rate ) XXX

Total Tax Liability XXX

Less: Full time Teacher Allowance (XXX)


XXX
Less: Foreign tax credit (XXX)
XXX
Less: Rebates
➢ Charitable Donation U/S 61 X
➢ Approved Pension fund X

Add: Tax on FTR + SBOI XXX

Total Tax liability XXX

Less : Tax Deducted at source (XXX)


Less: FTR (XXX)
Less: All WHT (XXX)
XXX
Add: Tax Credit disallowed (If any) (X)

Tax Payable XXX

175 | P a g e
Key Concepts Tested in Numerical ICAP CAF

Q. No. Reference Marks Remarks i.e. special features in the question

1 Q.15 Sept 2004 17 Fee for attending BOD meetings


2 Q.2 Sept 2005 16 Foreign source salary
TV and VCR provided for use
3 Q.1 March 2006 15 Quiz Prize, Profit in Lieu of Salary
4 Q.2(a) March 2007 12 Forfeited deposit Foreign source salary
5 Q.1 March 2009 18 Foreign source salary
Tax borne by the employer
6 Q.1 Sept 2009 21 Specified amount of tax borne by the company & Two
pensions
7 Autumn 2010 Q. 1 21 Leave Encashment, Profit on Debt, Employee share
scheme
8 Spring 2011 Q. 1 20 Commission, gratuity, employment switching
9 Autumn 2011 Q. 1 19 Medical Facility not under the contract, Final Settlement
10 Spring 2013 Q. 1 20 Employee Share Scheme, Mutual Units
11 Autumn 2013 Q. 1 21 Tax borne by the employer
Employee share scheme
Best employee award
12 Autumn 2014 Q. 1 20 B.O.D Meeting fee, literary work, winning Prize
13 Autumn 2015 Q. 1 20 Car given for use and transfer, Appliances given for use,
Agriculture Income
14 Spring 2016 Q. 1 16 Provident Fund, Dividend, Contribution to Approved fund
15 Q.1 Sept 2016 15 Employment bond
Employee share scheme
Termination benefit taxable at last 3 years' average rate of
tax
16 Q.1 Sept 2017 16 Effect of annual increment, Loan from Employer
17 Q.1 Sept 2018 16 Specified amount of tax borne by the employer
18 Q.1 Sept 2019 17 Foreign source salary income
19 Q.1 Sept 2020 19 Full time teacher allowance
Interest free loan with the waiver of interest on provident
fund balance
Foreign tax credit
20 Q.1 Sept 2021 19 Relocation allowance
21 Q.1 March 2022 17 Allowance for office duties as a % of salary
Foreign source salary of a resident person
22 Q.2 Sept 2022 16 Expenses for work from home
Gifts received in kind and in cash
23 Q.1 March 2023 19 Employment bond
Deemed interest expense
24 Q.1 September 14 Free Food, Insurance, Provident fund, Employee training

176 | P a g e
ICAP CFAP Q. No. [For Advance Practice ] for jazbati Students

Q. No Reference Marks Remarks i.e. special features in the question


25 Q.1 Dec 2009 18 Specific amount of tax to be borne by the employer
Forfeited deposit
Loan received in cash
26 Q.1 June 2011 20 Compensation under the redundancy scheme taxable @last 3
years' average rate of tax
27 Q.1 Dec 2012 25 Returning expatriate
Refrigerator, cooking range and washing machine provided by
the employer for use Rent of agricultural land
Dividend in specie
28 Q.1 Dec 2013 22 Compensation for delayed refund
29 Q.6 June 2015 8 Voluntary waiver of leave encashment by the employee
Royalty income

Question # 1 Q.15 September 2004

Mr. A is the Chief Executive of a multinational company (unlisted).


Details of his emoluments are as follows:
(a) Basic salary 4,004,520
(b) Bonus 1,980,642
(c) Utility allowance 400,452
(d) Leave encashment 538,083
(e) Other allowance 90,000
(f) House rent allowance 1,802,040
Apart from the above, he is eligible to receive gross Director's Fee of Rs.52,000. The company has paid the
amount after deduction of tax @ 20%.
During the year he has sold shares that were acquired through exercise of a 'Stock Option' three years ago, The
gain on sale amounts to Rs.4,206,000.
He also owns a property which has been let out on rent. The details of rent received and expenses incurred are
as follows:
(a) Rent Rs.40,000 per month. The property was let out on rent for the whole year. The annual letting value of
the house is Rs.450,000. No income tax was deducted by the tenant.
(b) He has paid property tax amounting to Rs.11,500.
(c) During the year he has paid Rs.6,000 for repairs and maintenance.

He has also earned gross amount of profit of Rs.6,500 on PLS (profit and loss sharing) Account.
During the year the following amounts were withheld at source towards income tax.
(a) From salary income Rs.1,200,000 other than director's fee
(b) From profit on PLS Account Rs.650
You are required to compute the taxable income and tax liability for the tax year 20X8. (Marks 17)

Question # 2 Q.2 September 2005

Mr. Imran is a citizen of Pakistan. During the first nine months of the tax year, he worked as financial
controller of a Pakistan based unlisted subsidiary company of a multinational group.
After that he was transferred and employed as Head of Finance of the UAE based subsidiary of the Group.
Mr. Imran's family stayed in Dubai throughout the year.
The detail of income earned by him during the year is given below:

177 | P a g e
From the UAE company
Mr. Imran earned US $ 30,000 during the three-month's employment in the UAE. No tax is deducted from
salary earned and paid in the UAE.
To relocate Mr. Imran in UAE, the UAE Company incurred one time miscellaneous cost of
Rs.100,000 to move the household items of Mr. Imran from Pakistan to Dubai.
From Pakistan subsidiary
(a) Basic salary Rs.500,000 p.m.
(b) Medical allowance Rs.45,000 per month (no free medical or hospitalization facility is given to Mr. Imran
under the terms of employment).
(c) The company has provided Mr. Imran a TV and VCR costing Rs.40,000 on which the company charges
depreciation @ 20% in its books of accounts.
(d) Company has provided interest free loan to Mr. Imran amounting to Rs.5 million which remained
outstanding throughout his employment with the company (Pakistan subsidiary).
Mr. Imran acquired a flat from the amount of loan and rented it out @ Rs.100,000 per month with effect from
1st July. He also paid Rs.35,000 as property tax during the period.
(e) *His family's housing cost in Dubai, borne by the company amounts to Rs.30,000 per month.
(f) Mr. Imran's travelling and related cost borne by the Pakistan subsidiary to meet his family, amounts to
Rs.30,000 p.m.
(g) During the employment with the Pakistan subsidiary, Mr. Imran had exercised option to acquire 300
shares of the parent company @ US $8 per share.
At the time when the option was exercised, the value of the share was US $ 10 (Rs.580) per share.
Furthermore, during the year Mr. Imran sold 200 options previously received by him at a price of US $3 per
option (Rs.171) after holding it for more than a year.
Neither the Pakistan subsidiary nor Mr. Imran incurred any cost in this regard.
Required:
Compute the taxable income of Mr. Imran for the tax year 20X8 based on the data provided above.
(Marks 16)
Note for students:
Housing cost in Dubai paid by the Pakistani company is being considered as reimbursement of personal
expense of Mr. Imran and not as accommodation provided by the company.
If it is considered as accommodation provided by the company then 45% of the basic salary would be the
taxable amount in this respect.

Question # 3 Q.1 March 2006

Ms. FH was working as a Marketing Head with Consumer Products Ltd. (CPL) at following emoluments:
(i) Basic salary Rs. 100,000 per month
(ii) House rent allowance Rs.40,000 per month
(ii) Utilities allowance Rs.15,000 per month

In addition to the above cash emoluments, she was provided with a Honda Civic car, exclusively for official
use. The cost of car to the Company was Rs.1,000,000. As per Company's policy, the car was sold to Ms. FH
during the year at the WDV of Rs.100,000 whereas the FMV of the same at the time of sale was Rs.300,000.
In May 20X8, Ms. FH was approached by Pharma Industries (Pvt.) Ltd (PIL). They offered her employment
at a higher salary and some extra benefits, along with a onetime payment of Rs.200,000 as an inducement to
accept their offer. Ms. FH accepted PIL's offer by resigning from CPL with effect from 1.6.20X8. She joined
PIL from 1.7.20X8. The amount of Rs.200,000 was, however, paid to her on 29.6.20X8..
During the year, Ms. FH has also undertaken the following transactions:
i. Shares in Queens Pakistan (Pvt.) Ltd were sold for Rs.500,000. These shares were acquired in the
year 20X3 at a cost of Rs.200,000.

178 | P a g e
ii. A residential plot inherited on 30.4.20X7 was sold for Rs.1,000,000 on 15.6.20X8. The FMV of the
plot at the time of inheritance was Rs.300,000 (original purchase cost was Rs.200,000).
iii. A painting purchased at a cost of Rs.100,000 was sold for Rs.75,000.
iv. She had won a cash prize of Rs.250,000 in a quiz show. Tax of Rs.50,000 was deducted from the
prize money under section 156.
v. Gross dividend income of Rs.50,000 from a listed company. Dividend was received after deduction of
tax of Rs.7,500 under section 150.
vi. An amount of Rs.50,000 was donated to an approved charitable institution.
In the light of above information, compute the taxable income of Ms. FH for the tax year 20X8 by giving brief
explanation for the items not included in the taxable income. (Marks 15)

Question # 4 Q.2(a) March 2007

Explain the correct tax treatment in the tax year 20X8 in each of the following situations:
i. In 20X1, Mr. H inherited a rare sculpture of Buddha which had a FMV of Rs.300,000 on the date of
inheritance (original purchase cost Rs.200,000).
In February 20X8, the sculpture was sold by him at Rs.500,000.
ii. In July 20X7, Mr. Y entered into an agreement for sale of his residential plot to Mr. M, who paid an
advance of Rs.500,000.
According to the agreement, Mr. M was required to pay the balance by 28.8.20X7.
However, instead of paying the balance amount, he terminated the sale agreement. Mr. Y forfeited the
advance of Rs.500,000 in accordance with the terms of the agreement.
iii. In September 20X7, Mr. S sold his personal car, Toyota Corolla, to one of his cousins at a price of
Rs.50,000 whereas the FMV of the car was Rs.200,000. The car was purchased by him in the year
20X4 at a cost of Rs.300,000.
iv. Mr. I was working as a Chief Financial Officer in DW Pakistan (Pvt) Ltd, which is a wholly owned
subsidiary of DW AG, Germany.
According to the Company's policy, Mr. I was sent on secondment to Germany on 1.1.20X8 for a
period of five years.
During this period, half of his salary will be credited to his bank account in Pakistan, whereas the remaining
portion will be received by him in Germany. (Marks 12)

Question # 5 Q.1 March 2009

Mr. Manto worked as an employee in Berlin Hotel, Germany for a period of five years. During the said period
he did not visit Pakistan for a single day.
He returned to Pakistan on 1.7.20X7 and immediately joined as a General Manager in a well- reputed hotel,
based in Karachi.
Assume that the details of his income for the tax year 20X8 are as follows:
i. Basic salary (per month) Rs.100,000
House rent allowance (per month) Rs.30,000
Medical allowance (per month) Rs.10,000
Bonus for the year Rs.600,000
ii. Besides medical allowance, he is also entitled to free medical treatment at approved hospitals.
iii. He has been provided a company maintained 1600cc car which was used partly for official and partly
for personal purposes.
The hotel has leased the car from a bank. The gross lease rentals payable over the period of lease
amount to Rs.2,700,000.
The fair market value of the car at the time of lease was Rs.1,600,000.
The total lease rentals paid by the hotel during the year amounted to Rs.800,000.

179 | P a g e
iv. He is entitled to lunch at the hotel's restaurants where the usual charges are Rs.400 per person. He is
entitled to concessional rate of Rs.40 per day which is deducted from his salary. Assume that there are
300 working days in the year.
v. He went for a training course to Islamabad where boarding and lodging cost amounting to Rs.150,000
was borne by the hotel. He incurred a further expense of Rs.125,000 which was reimbursed by the
hotel.
vi. Provident fund was deducted @10% of his basic salary. An equal amount was contributed by the
hotel. Interest credited to his provident fund account amounted to Rs.48,000.
vii. As per terms of employment agreed with Mr. Manto, tax payable on salary will be borne by the hotel.
viii. On 15.7.20X7, he received a lump sum amount of Rs.4,000,000 through a normal banking channel as
final settlement from Berlin Hotel.
ix. On 1.8.20X7, he inherited 25,000 shares of a private limited company. The original cost in the hands
of transferor of the shares, on the date of inheritance, was Rs.42 per share.
x. He sold all the shares on 28.2.20X8 at Rs.62 per share.
Required:
(a) Compute Mr. Manto's taxable income and tax payable.
(b) Briefly explain the treatment of items which are not considered in the above computation. (Marks 18)

Question # 6 Autumn 2009 Q. 1

Mr. Zulfiqar, a senior executive of Mirza Petroleum Ltd (MPL), retired on 31" March after completion of 19
years of dedicated service.

The details of Mr. Zulfiqar's income for the year ended 30.6.20X8 are given below:
Income from MPL
Rupees
Basic salary 280,000
Medical allowance 45,000
Utilities allowance 45,000
Cost of living allowance 25,000
Total monthly salary 395,000

Market value of rent free accommodation provided 120,000

(ii) As per terms of employment, tax liability of Mr. Zulfiqar to the extent of Rs. 200,000 is to be borne by
MPL.
(iii) On his retirement, he received gratuity of Rs. 2,660,000 from an unapproved gratuity fund maintained by
MPL.
(iv) He is receiving pension amounting to Rs.50,000 per month from the date of his retirement.

Other Information
(v) He is also receiving pension amounting to Rs.12,000 per month from a multinational company where he
worked from 1975 to 1995.
(vi) A plot inherited from his father was sold for Rs.5,000,000. Fair market value of the plot at the time of
inheritance in the year 20X1 was Rs.1,000,000.
(vii) On 1st January, he rented out one of his residential bungalows for Rs.100,000 per month and received
advance rent for two years.
(viii) Rs.500,000 were invested in new shares offered by a listed company.
(ix) He paid mark up amounting to Rs.250,000 on a house loan obtained from a scheduled bank. The house is
being used for his residence.
(x) He incurred a loss of Rs.20,000 On sale of a painting.

Required:

180 | P a g e
(a) Compute taxable income and tax liability of Mr. Zulfiqar.
(b) Briefly comment on the items which are not considered in the above computation. (Marks 21)

Question # 7 Autumn 2010 Q. 1

Mr. Zameer Ansari is working as a Chief Executive Officer in Wimpy (Private) Limited (WPL). Following are
the details of his income / receipts during the tax year 2023:

(a) His monthly cash remuneration in WPL is as follows:


Rupees
Basic salary 200,000
Medical allowance 30,000
Utilities allowance 10,000

(b) In addition to the above, he was also provided the following benefits in accordance with his terms of
employment:
(i) Medical insurance for hospitalization and surgery, limited to Rs. 1,500,000 per annum.
(ii) Payment of his children’s school fees of Rs. 15,000 per month. The fee is deposited directly into the
school’s bank account.
(iii) Rent free furnished accommodation on 1000 square yards. The accommodation is located within the
municipal limits of Karachi.
(iv) Two company-maintained cars. One of the cars was purchased by WPL for Rs. 3,000,000 and is
exclusively for his business use. The second car was obtained on lease on February 1, 2017 and is
used partly for official and partly for personal purposes. The fair market value of the leased vehicle
at the time of lease was Rs. 1,800,000.
(v) Leave encashment amounting to Rs. 100,000 was paid to Mr. Zameer on July 5, 2023.
(vi) An amount equal to one basic salary was paid by WPL to an approved pension fund.

(c) Mr. Zameer had received 15,000 shares of WPL on December 1, 2020 under an employee share scheme.
He had the option to transfer the shares on or after January 1, 2022. However, he sold all the shares on
April 1, 2010.
Fair value of the shares were as follows:
▪ Rs. 35 per share on December 1, 2020
▪ Rs. 42 per share on January 1, 2022
▪ Rs. 48 per share on April 1, 2023

(d) An apartment owned by Mr. Zameer was rented on July 1, 2021 to Mr. Abdul Ghaffar at a monthly rent
of Rs. 22,000. He received a non-adjustable security deposit of Rs. 150,000 which was partly used to repay
the non-adjustable security deposit amounting to Rs. 90,000 received from the previous tenant in July
2020. He also incurred Rs. 20,000 on account of repairs to the apartment.

(e) He earned profit amounting to Rs. 750,000 on fixed deposit account maintained with a bank. The bank
withheld income tax amounting to Rs. 75,000 and Zakat amounting to Rs. 250,000.

(f) Tax deducted at source from his salary, amounted to Rs. 650,000.

Required:
Compute the taxable income, tax liability and tax payable by Mr. Zameer Ansari for the tax year 2023.
(21)

Question # 8 Spring 2011 Q. 1

Mateen was employed with Melody Ltd (ML) as an event organizer. On 30.6.2022 he resigned from his
employment without completion of notice period. On 1.7.2022 he joined another company Rock Star Ltd (RSL)

181 | P a g e
as a senior event organizer.

Following information is available relating to his assessment for the tax year 2023:
(a) On 1.7.2022 RSL paid Rs.280,000 to ML as compensation in lieu of un-served notice period by Mateen.
(b) On 15.7.2022 Mateen received a gratuity of Rs.350,000 from an unrecognized gratuity fund maintained
by ML. He also received Rs.150,000 as leave encashment.
(c) In accordance with the terms of his employment with RSL, Mateen was provided with the following
emoluments / benefits during the tax year 2023:
(i) Basic salary of Rs.245,000 per month and utility allowance of Rs 21,000 per month.
(ii) A reimbursement of personal medical expenses up to 15% of the annual basic salary and Rs.250,000
on account of hospitalization charges of his daughter were made after procuring hospital bills
showing the national tax number of the hospital. These bills were also attested and certified by RSL.
(iii) For the first two months of his employment, a pick and drop facility was provided to Mateen at a
monthly rent of Rs.25,000. On 1.9.2022 RSL provided a company maintained 1300 cc car which was
partly used for private purposes. The cost of the car was Rs.2,500,000.
(iv) Monthly salary of Rs.6,000 was paid to Mateen's house keeper. Mateen however, reimbursed 20%
of the house keeper's salary to RSL.
(v) A special allowance of Rs.50,000 was paid to meet expenses necessarily to be incurred in the
performance of his official duties. Actual expenditure was Rs.40,000.
(vi) On 1.1.2023, he was provided an interest free loan of Rs.1,500,000. The prescribed benchmark rate
is 10%per annum.
(vii) A commission of Rs.500,000 for introducing new clients to the company. Withholding tax was
deducted by RSL @ 12% from such payments.
(viii) The tax deducted at source from his salary by RSL for the tax year 2023 amounted to Rs.550,000
(other than tax deducted on commission).
(d) Apart from his employment with RSL, Mateen also organized events for private clients. He received a
total of Rs.1,000,000 from such clients. No tax was deducted from such receipts. However, he incurred an
overall loss of Rs.350,000 on organizing these events.
(e) On 31.5.2023 he received Rs.180,000 from Mr. Ali as consideration for vacating his bungalow.
(f) He also received a share of profit from a business in Malaysia equivalent to Pak. Rs.535,000. He paid
Rs.130,000 in taxes in Malaysia on such income.
(g) Mateen acquired 10,000 shares of a listed company from the Privatization Commission of Pakistan at a
price of Rs.100 per share on 31.5.2022. He was allowed a tax credit of Rs.100,000 in tax year 2022 against
this investment. On 20.5.2023 he sold all the shares for Rs.1,000,000.
(h) He paid Zakat of Rs. 250,000 to an approved institution by cross cheque

Required:
Compute the taxable income, mix liability and tax payable / refundable, if any, by Mateen for the tax year 2023.
(Marks 20)

Question # 9 Autumn 2011 Q. 1

Mr. Khursheed, Pakistani national, was employed as the chief financial officer in Zulfiqar Gas Company
(ZGC), since 2012. Following information pertains to his income for the tax year 2023. Assume that he has
provided the following information on 25.9.2023 to his tax consultant for the purpose of computation of taxable
income and tax liability:
(I) Income from ZGC
Khursheed was employed with ZGC up to 31.12.2022. During this period he received the following
emoluments:
▪ Basic salary of Rs.400,000 per month, medical allowance of Rs.75,000 per month and utility
allowance equivalent to 10% of basic salary.
▪ A company-maintained car for official and private use. The car was purchased two years ago at a cost
of Rs.5 million. According to the company's policy, ZGC deducted Rs.10,000 per month from his
salary, for private use of the car.

182 | P a g e
Khursheed had undergone a major surgery during the year and incurred an expenditure of ks,1,500,000.
ZGC reimbursed the entire amount as a special case as it was not covered under the terms of employment.

Due to Poor health, Khursheed opted for early retirement on 31.12.2022 under the company's voluntary
retirement scheme.
He received the following benefits on his retirement:
▪ Rs.7,500,000 as a golden handshake under the voluntary retirement scheme.
▪ Rs.9,100,000 from an unapproved gratuity fund maintained by ZGC.
▪ Transfer of company's car for Rs.2,600,000. The amount was deducted from his final settlement. The
fair market value of the car as of 31.12.2022 was Rs.2,800,000.

The tax deducted at source from the salary amounted to Rs.2,500,000.

(II) Other Information


▪ On 1.1.2023, Khursheed commenced business of marketing of horticultural plants and related items.
However, due to intense competition, he had to wind-up this venture on 31.5.2023. During this period,
he had incurred a loss of Rs.750,000.
▪ He purchased 5,000 shares for Rs.500,000 from initial public offering of a new listed company before
1.7.2013. He claimed a tax credit of Rs.60,000 on such investment, against the tax payable for the tax
year 2013. On 15.7.2022, he sold these shares for Rs.700,000. Incidental expenses are included in
these figures.
▪ He incurred a loss of Rs.500,000 on the sale of his shareholdings in a private company.
▪ He sold his personal car at a profit of Rs.300,000.
▪ On 1.3.2023, he purchased an apartment for Rs.5,000,000. 60% of this amount was financed by a
scheduled bank under housing finance scheme. During the tax year 2023, he paid markup amounting
to Rs.127,500. On 1.4.2023, he rented out the flat to Mr. AS at a monthly rent of Rs.100,000 and
received advance rent for eight months.
▪ His average tax rate for the preceding three years is 12%.

Required:
(a) Compute the amount of taxable income, tax liability and tax payable / (refundable), if any, for the tax year
2023. (13)
(b) Briefly comment on the items which are not considered by you in the above computation. (6)

Question # 10 Spring 2013 Q. 1

Mr. Creative is working as Director Human Resources at Artistic Technologies Ltd (ATL). Following are the
details of his income/receipts during the latest tax year:

(a) Monthly cash remuneration from ATL:

Basic salary Rs. 300,000


Utilities allowance 15% of basic salary
Medical allowance 12% of basic salary

(b) In addition to above, he was also provided the following benefits in accordance with his terms of
employment:
(i) Rent-free furnished accommodation in a bungalow situated on a 500 square yard plot of land. Rent
for a comparable accommodation facility in the vicinity is Rs.150,000 per month.
(ii) An 1800cc company-maintained car. The car was purchased two years ago at a cost of Rs.1,600,000
and is used both for official and personal purposes.

(c) A house owned by Mr. Creative had been leased-out by him at a monthly rent of Rs.50,000. The lease

183 | P a g e
expired on 31 December. Mr. Creative refused to renew the lease in spite of the tenant's offer to renew the
lease after increasing the rent by 10%. He returned the non-adjustable deposit of Rs.300,000 to the tenant,
which was received two years ago.
The house was immediately leased to his cousin without any security deposit on a monthly rent of
Rs.48,000.
(d) Five years ago, Mr. Creative had purchased 20,000 shares of Rs.10 each, of an unlisted public company
@ Rs.140 per share. After one year of acquisition, he received 8,000 bonus shares from the company. No
tax was paid at the time of issuance of bonus shares. During the latest tax year, he sold 75% of the bonus
shares at a price of Rs.145 per share.
(e) During the latest tax year, following investments were made: Rs. Approved voluntary pension fund
600,000 Open-end mutual fund 1,100,000
(f) During the latest tax year, he redeemed 4,000 units of mutual fund [other than stock fund] at Rs.58.6 per
unit. These units were purchased at the beginning of the previous tax year at Rs.50 per unit and Mr.
Creative had claimed a tax credit of Rs.40,000 on this investment. The given figures include incidental
expense.
(g) Donations of Rs.50,000 were paid to approved charitable institutions.
(h) Tax deducted at source from his salary was Rs.400,000.

Required:
Compute the taxable income, tax liability and tax payable for the latest tax year. (Marks 20)

Question # 11 Autumn 2013 Q. 1

Mrs. Aslam was employed with Sahal Ltd (SL) as a Marketing Manager. On 30.6.2022 she resigned from her
employment with SL On 1.7.2022, she joined H Pakistan Ltd (HPL), a quoted company, as a Marketing
Director.
She has provided you the following information in respect of the tax year 2023:

(i) In July 2022, she received following amounts from SL in final settlement:
▪ Leave encashment amounting to Rs.95,000.
▪ Gratuity of Rs.500,000 from an unrecognized gratuity fund maintained by SL.
▪ Reimbursement of Rs.100,000 against a health insurance policy. The insurance claim was lodged by
SL on behalf of Mrs. Aslam in January 2022.
(ii) In accordance with the terms of her employment, income tax related to her salary and benefits is to be
borne by HPL. Her emoluments/benefits during the tax year were as follows:
▪ Basic salary of Rs.200,000 per month.
▪ Meckal allowance of Rs.60,000 per month.
▪ Rent free accommodation with annual letting value of Rs.480,000.
▪ Traveling allowance of Rs.50,000 per month. 60% of the amount was spent in the performance of
official duties.
▪ Provident fund @ 10% of basic salary. An equal amount was contributed by HPL.
(iii) Under an employee share scheme, Mrs. Aslam was awarded 5,000 shares in HPL on 1.1.2023. Under the
scheme she was not allowed to sell the shares up to 31.3.2023. She sold all the shares in HR. on 1.5.2023.
Fair value of the shares on the above dates was as follows:
▪ Rs.20 per share on 1.1.2023
▪ Rs.28 per share on 31.3.2023
▪ Its.32 per share on 1.5.2023
(iv) On 31-12-2022, she received a loan of Rs.400,000 from HPL. The loan carries a mark-up of MO per arum
The prescribed benchmark rate is 10%.
(v) She won the best executive employee award of HPL and received a laptop having a fair market value of
Rs.100,000.
(vi) An amount of Rs.355,000 was received from her spouse as support payment, under an agreement to live
apart.
(vii) She paid Rs.105,000 as zakat under the Zakat and Ushr Ordinance, 1980.
(viii) Donation of Rs.70,000 was paid to an approved organization

184 | P a g e
Required:
Compute the taxable income, tax liability and tax payable for the tax year 2023.
Note: Show all relevant exemptions, exclusions and disallowances. (Marks 21)

Question # 12 Autumn 2014 Q. 1

Sultan is working as electronic engineer with Ansari Electrical Company Limited (AECL).
He has provided you with the following information for the tax year ended 30 June 2023:

(a) His monthly cash remuneration in AECL is as follows:


Rupees
Basic salary 480,000
Medical allowance 48,000
Utilities allowance 55,000

Market value of rent free accommodation 75,000

(b) He was also provided the following benefits in accordance with the terms of his employment:
(i) Leave encashment amounting to Rs. 300,000.
(ii) Hospitalization cost is covered by an insurance policy upto the amount of Rs. 1.5 million. The
insurance premium relating to this benefit amounted to Rs. 55,000.
(iii) He is allowed to use his personal car for office use. Reimbursement of car running and maintenance
expenses amounted to Rs. 550,000. 15% of these expenses pertain to personal use.

(c) Rs. 200,000 were received from a private limited company for attending board meetings.
(d) A lump sum amount of Rs. 1.2 million was received as the author of a literary work. Sultan took three
years to complete this literary work.
(e) Sultan is also a part time singer and owns a studio. He sold the premises in which the studio was situated
for Rs. 10 million and shifted his musical instruments to new premises which he purchased for Rs. 15
million. He received Rs. 2.5 million from his father in cash as loan to pay for his new studio. The previous
premises was purchased several years ago for Rs. 1.4 million and had a tax written down value of Rs.
600,000 at the time of disposal.
(f) The net income from the studio for tax year 2023 was Rs. 990,000. The expenses include salaries of two
workers at Rs. 15,000 and Rs. 18,000 per month and utility bills amounting to Rs. 110,000. All expenses
were paid in cash.
(g) He won a car, in a competition held by Star Motor Limited for promotion of its sales. The fair market
value of the car was Rs. 850,000.
(h) He gifted 40 fans having a fair market value of Rs. 100,000 to an approved charitable organisation.
(i) An amount of Rs. 500,000 was paid by him as contribution to an approved pension fund.

Required:
Under the provisions of the Income Tax Ordinance, 2001 compute the taxable income and tax thereon for the
tax year 2023. Tax rates are given on the last page. (20)

Question # 13 Autumn 2015 Q. 1

Mukarram is working as a Commercial Manager in Airmen Engineering Limited (AEL), an unlisted public
company, for the past many years. He derived following emoluments during the tax year ended 30 June 2023:

Rupees
Basic salary (per month) 250,000
Medical allowance (per month) 37,500

185 | P a g e
Housing allowance (per month) 25,000
Travel allowance (per month) 11,500

In addition to above, Mukarram was also provided the following:


(i) A used company maintained car for both business and personal use. This car was provided to him on 1 July
2022 in replacement of his previous car. This car was purchased three years ago at a price of Rs. 1,000,000.
However, the fair market value of the car on 1 July 2022 was Rs. 800,000. On 1 September 2022, in
accordance with the terms of his employment, AEL transferred the previous car to Mukarram free of cost.
The market value of the car at the time of transfer was Rs. 400,000 whereas its book value was Rs. 200,000.
On 1 June 2023, Mukarram sold this car to his neighbor at a price of Rs. 350,000.
(ii) Performance related bonus of Rs. 500,000. The bonus was however, paid to him on 5 July 2023.
(iii) Two free buffet dinner coupons per month, one each for Mukarram and his wife in a five star hotel. The
coupons were provided in line with AEL’s policy for its management employees. The dinner costs AEL
Rs. 2,000 per person.
(iv) Reimbursement of Rs. 20,000 in respect of telephone and internet charges. 20% of this amount was spent
by Mukarram in performance of his official duties.
(v) Two air-conditioners and a washing machine for use at home. The combined book value of these
appliances was Rs. 300,000. The appliances are returnable to AEL after three years’ time. AEL charged
10% depreciation on these appliances.
(vi) An option to purchase 20,000 shares in AEL on 1 May 2023 at Rs. 25 per share. The break-up value of
AEL on that date was Rs. 85 per share.

Other information relevant to tax year 2023 is as under:


(i) On 1 April 2023, Mukarram sold a diamond ring to his brother Zohaib for Rs. 250,000. The ring was
purchased on 1 January 20X3 at a price of Rs. 280,000.
(ii) Mukarram has 65 acres of agricultural land in Badin and a building in immediate vicinity of the land.
Mukarram rented out 30 acres of his land along with the building to Dino who is a cultivator. Dino uses
the building as a store house. Mukarram received annual rents of Rs. 750,000 and Rs. 325,000 in respect
of the land and building respectively.
Mukarram is also running a small rice husking unit in Badin. He uses entire agricultural produce in the
husking unit which is grown on the remaining portion of his land. During the year he brought 5,000
kilograms of raw rice from his land to the unit for husking. He would have earned Rs. 2,500 per 40
kilogram of raw rice had he sold it directly to the market. His sales from rice husking unit stood at Rs.
850,000 whereas other operating expenses were of Rs. 400,000.
(iii) On 31 May 2023 a painting was destroyed by heavy rains. Mukarram had purchased the painting on 30
June 2020 for Rs. 100,000. However, due to constant increase in the value of the painting, he had insured
it at a premium of Rs. 15,000. He received insurance claim of Rs. 275,000 on 15 June 2023.

Required:
Under the provisions of the Income Tax Ordinance, 2001 and Rules made thereunder, compute the taxable
income of Mukarram for tax year 2023. (20)

Question # 14 Spring 2016 Q. 1

Wajahat, aged 48 years, is a marketing manager in Nayaab (Pvt.) Limited (NPL), a company engaged in the
manufacture and supply of tissue papers. The details of his monthly emoluments during the year ended 30 June
2023 are as under:
Rupees
Basic salary 70,000
Dearness allowance 10,000
Conveyance allowance 8,000

In addition to the above, Wajahat was also provided the following:


(i) Provident fund (PF) contribution of Rs. 8,400 per month. An equal amount per month was contributed by

186 | P a g e
Wajahat to the fund. Interest income of Rs. 391,000 at the rate of 20% of accumulated balance of PF was
credited to his PF account.
(ii) Reimbursement of electricity bills during the year amounting to Rs. 60,000.

Following further information is also available:


(i) Wajahat received net dividend of Rs. 78,200 from BEE Limited, a company listed on Pakistan Stock
Exchange Limited. Withholding tax and zakat deducted from dividend amounted to Rs. 9,200 and Rs.
4,600 respectively. He also received dividend of Rs. 65,000 from a company in U.A.E through normal
banking channels. However, no tax was withheld either in Pakistan or U.A.E.
(ii) Wajahat contributed Rs. 890,000 in an approved pension fund under the Voluntary Pension System Rules,
2005.
(iii) On 1 September 2022, Wajahat started a tuition centre for the students of finance in a posh locality. He
received tuition fees of Rs. 2,198,000 and incurred following expenses:
▪ Monthly salary of Rs. 50,000 paid to himself and Rs. 35,000 to his friend Yousuf who taught financial
accounting at the centre.
▪ Travelling, boarding and lodging expenses of Rs. 300,000. These expenses were incurred by Wajahat
in Sri Lanka for attending teachers training workshop.
▪ Rs. 250,000 against purchase of used computers for the centre.
▪ Other miscellaneous expenses amounting to Rs. 195,000.
(iv) Wajahat’s total taxable income during the previous tax year was Rs. 1,850,000.

Required:
Under the provisions of the Income Tax Ordinance, 2001 and Rules made thereunder, compute the total income,
taxable income and net tax payable by/refundable to Wajahat during the tax year 2023. (16)
Note: Show all relevant exemptions, exclusions and disallowances.

Question # 15 Autumn 2016 Q. 1

Bader is working as General Manager Finance with HiFi Limited (HFL) for the past two years. The details of
his monthly emoluments during the year ended 30 June 2023 are as under:

Rupees
Basic salary 250,000
Medical allowance 28,000
House rent allowance 120,000

In addition to above, Bader was also provided the following:


(i) Rs. 900,000 for signing a bond with HFL. According to the bond Bader would not resign from his
employment before the expiry of 30 June 2025.
(ii) Company maintained car for both official and private use. The car was purchased on 1 August 2022 at a
fair market value of Rs. 1,500,000.
(iii) On 1 January 2023 HFL sold an item of inventory to Bader for Rs. 12,000. The net realizable value of the
item of inventory at the end of 31 December 2022 and 30 June 2023 was Rs. 22,000 and Rs. 24,000
respectively. HFL had acquired it in July 2014 at a cost of Rs. 35,000.
(iv) An option was granted to Bader in August 2021 to acquire 2,500 shares in HFL’s parent company, Mamoo
plc. (MP), listed on Hong Kong stock exchange. However, the option was exercisable after completion of
one year of service with HFL. Bader paid an amount equivalent to PKR 200,000 to acquire the option
when the fair market value of the option was PKR 250,000.

On 1 September 2015 he paid an amount equivalent to PKR 300,000 to acquire the shares in MP. The
shares were issued to him on 15 September 2022 when the market value of each share was equivalent to
PKR 375.

187 | P a g e
On 15 June 2023 Bader sold 2,000 shares in MP and received net proceeds equivalent to PKR 875,000 in
his bank account in Pakistan. This amount was received after deduction of bank charges of PKR 5,000
and brokerage commission equivalent to PKR 10,000.

Other information relevant to tax year 2023 is as under:


(i) On 1 July 2015 Bader received following payments from his previous employer Sultan Hospital Limited:
▪ Rs. 600,000 in respect of termination benefits under an agreement.
▪ Rs. 485,000 against gratuity under an unapproved scheme.
(ii) On 1 November 2022 Bader fell ill and was admitted to Sultan Hospital Limited. The hospital incurred
Rs. 65,000 on his treatment but did not charge anything to Bader.
(iii) On 1 December 2022 he paid a premium of Rs. 300,000 on a life insurance policy.
(iv) On 1 January 2023 Bader purchased 35,000 listed shares in Muft Limited (ML) at a price of Rs. 25 per
share. On 20 March 2023 he fully subscribed 15% right shares offered by ML to its existing shareholders
at a price of Rs. 20 per share.
(v) Withholding tax deducted from Bader’s salary during tax year 2023 amounted to Rs. 1,105,000.
(vi) His total assessed taxable income and total taxes paid thereon during the three preceding tax years
amounted to Rs. 10,500,000 and Rs. 1,260,000 respectively.

Required:
Under the provisions of the Income Tax Ordinance, 2001 and Rules made thereunder, compute the taxable
income and net tax payable by or refundable to Bader for tax year 2023. (15)

Question # 16 Autumn 2017 Q. 1

Taqi Ahmed is working as Director Marketing with Zee Textiles Limited (ZTL) for the last twenty five years.
Details of his monthly emoluments during the year ended 30 June 2023 are as under:

Rupees
Basic salary 440,000
Conveyance allowance 44,000
Medical allowance 44,000

In addition to the above, Taqi Ahmed has provided the following information:
(i) He and his family members are covered under the health insurance policy in accordance with the terms of
employment. The amount of annual premium paid by ZTL was Rs. 200,000.
(ii) During the year, daily allowance of Rs. 400,000 was received to meet the expenses for working on
assignments at ZTL’s factories located in Lahore and Multan.
(iii) On 31 July 2023, the HR Committee approved a performance bonus for all employees for the year ended
30 June 2023. Taqi received Rs. 1,200,000 as performance bonus on 15 August 2023.
(iv) On 31 March 2023, in recognition of completion of twenty five years of his service with ZTL, the board
of directors approved to waive the outstanding amount of loan taken by Taqi Ahmed. This interest free
loan of Rs. 2,500,000 was taken on 1 January 2021 and was repayable in fifty equal monthly instalments
commencing from May 20X5. The prescribed benchmark rate is 10% per annum.
(v) During the year, he received Rs. 100,000 for attending board meetings of ZTL. No tax was withheld from
this amount.
(vi) Amount of tax withheld by ZTL from his salary amounted to Rs. 2,000,000.

Other information relevant to tax year 2023 is as under:


(i) Salary is transferred to the bank account on 10th of the following month.
(ii) 10% annual increase was given to him effective 1st July in each of the last three years.
(iii) Taqi has given his house on rent to his cousin at annual rent of Rs. 1,500,000. The rent was inclusive of
amenities and utilities of Rs. 25,000 per month. However, annual rent for a similar house with same
amenities and utilities, in the vicinity, is Rs. 1,800,000.
(iv) He acquired 15,000 shares of a listed company from Privatization Commission of Pakistan at a price of

188 | P a g e
Rs. 60 per share on 15 January 2022. He claimed tax credit of Rs.90,000 on such investment, against the
tax payable for the tax year 2022. On 15 June 2023 he sold all the shares at the rate of Rs. 85 each.
(v) On 31 August 2022, he was entitled to receive 5,000 interim bonus shares from Arian Limited (AL) a
listed company. The market value of these shares on that date was Rs. 22 per share.
(vi) He also received Rs. 150,000 as cash dividend declared by AL. The share registrar incorrectly treated Taqi
as non-filer and deducted withholding tax accordingly.

Required:
Under the provisions of the Income Tax Ordinance, 2001 and Rules made thereunder, compute under correct
head of income, the total income, the taxable income and net tax payable by or refundable to Taqi Ahmed for
the year ended 30 June 2023. (16)

Question # 17 Autumn 2018 Q. 1

Ahmer Ghazi has been working as director production in Delta Pakistan Limited (DPL) for last three years. He
received following monthly emoluments from DPL during the year ended 30 June 2023:

Rupees
Basic salary 650,000
House rent allowance 95,000
Medical allowance 70,000

In addition to the above, the employer also provided following to Ahmer Ghazi:
(i) Health insurance for him and his family members. The amount of annual premium paid by DPL was Rs.
50,000.
(ii) Return air ticket for Dubai worth Rs. 180,000 for him and his family as a reward for achieving the
production target.
(iii) Loan of Rs. 5 million was given to him on 1 August 20X7 at 6% per annum.
(iv) Withholding tax of Rs. 1,500,000 deducted from his salary was reimbursed to him.

Other information relevant to the tax year 2023 is as under:


(i) Under an employee share scheme 10,000 shares of DPL were allotted to Ahmer Ghazi on 1 January 2021.
According to the scheme, he was not allowed to sell/transfer the shares up to 31 December 2021. On 1
April 2023, he sold 6,000 shares of DPL for Rs. 33 per share. The face value of each share is Rs. 10. Fair
market values of each share on different dates were as follows:
▪ Rs. 20 per share on 1 January 2021
▪ Rs. 23 per share on 1 January 2022
▪ Rs. 29 per share on 30 June 2023

(ii) On 30 October 2022 Ahmer Ghazi let out his apartment at a monthly rent of Rs. 30,000 to his friend. The
fair market rent of the apartment is Rs. 40,000 per month.
(iii) He is a part time singer and earned Rs. 225,000 by allowing a private TV channel to use his song in a TV
drama.
(iv) He purchased Sukuks of a listed company amounting to Rs. 1,400,000 as an original allottee, on 30 June
2023.

Required:
Under the provisions of the Income Tax Ordinance, 2001 and Rules made thereunder, compute the following
for the year ended 30 June 2023:
(a) Total income (10)
(b) Taxable income (01)
(c) Net tax payable or refundable (05)
Note: Ignore minimum tax under section 113

189 | P a g e
Question # 18 Autumn 2019 Q. 1

Saeed, a citizen of Pakistan, was working on a foreign vessel belonging to Delta Shipping Company (DSL)
based in Spain for the past three years. His monthly salary was USD 15,000 which was remitted to his Pakistani
bank account through normal banking channel. The amount received during the tax year 2023 was converted
to Pak Rupees at an average exchange rate of USD 1 = PKR 131.

On 1 October 2022, he resigned from DSL and joined Haris Pharma Limited (HPL) in Pakistan as a General
Manager. He was offered following monthly salary and allowance in HPL:

Rupees
Basic salary 600,000
Medical allowance 66,000

In addition to the above, he was also provided the following:


(i) Bonus equal to two monthly basic salaries. However, bonus amount was adjusted in proportion to the
duration of his stay in the company. The bonus amount was paid to him on 5 July 2023.
(ii) Two company maintained cars. Both cars were purchased on 1 October 2022. The car costing Rs.
3,500,000 was used for official purposes whereas the car costing Rs. 1,900,000 was used for personal
purposes.
(iii) Free lunch from the restaurant owned by one of HPL’s directors. The fair market value of food provided
to him during the year was Rs. 125,000.
(iv) A special allowance of Rs. 20,000 per month to meet expenses wholly and necessarily incurred in the
performance of his official duties. Actual expenses incurred by him during the year were Rs. 150,000.
(v) Provident fund contribution of Rs. 60,000 per month. An equal amount per month was also contributed
by Saeed to the fund.

Other information relevant to tax year 2023 is as under:


(i) On 1 December 2022, Saeed obtained a loan of Rs. 25 million from a scheduled bank at 15% mark-up per
annum to acquire a residential house.
(ii) During the year, he received dividends of Rs. 575,000 from a listed company. The amount was net of
withholding income tax at the rate of 15% and Zakat of Rs. 62,500 deducted under the Zakat and Usher
Ordinance, 1980.
(iii) Withholding tax deducted by HPL from Saeed’s salary during the tax year 2023 amounted to Rs.
1,300,000.

Required:
Under the provisions of the Income Tax Ordinance, 2001 and Rules made thereunder, compute under the
appropriate head of income, the total income, taxable income and net tax payable by or refundable to Saeed
for the tax year 2023. (17)

Question # 19 Autumn 2020 Q. 1

Sageer, a resident individual, is working as a full time professor at Knowledge Institute (KI) which is a non-
profit education and research institution and is duly recognized by Higher Education Commission. KI is entirely
owned and funded by Zinger Limited (ZL), a company listed on the Pakistan Stock Exchange.

Details of his monthly remuneration during the year ended 30 June 2023 are given below:

Rupees
Basic salary 200,000
Medical allowance 20,000
Fair market rent of accommodation 100,000

190 | P a g e
In addition to the above, he was also provided the following:
▪ Health insurance for Sageer and his dependents as per the terms of employment. For this purpose, KI is
paying annual insurance premium of Rs. 40,000.
▪ Provident fund contribution of Rs. 15,000 per month to a recognized provident fund.
▪ An equal amount was also contributed by Sageer to the fund.

Additional information
(i) On 1 July 2022, Sageer was granted an option to acquire 10,000 shares in ZL at a price of Rs. 105 per
share under an employee share scheme. Sageer bought the option on the same date by paying Rs. 175,000
to KI when the fair market value of the option was Rs. 200,000. He exercised the option on 30 September
2022 when the fair market value was Rs. 130 per share.

As per the scheme, he was not allowed to sell or transfer the shares before 31 December 2022. On 31
December 2022, the fair market value of ZL’s shares was Rs. 142. On 30 May 2023, he sold 5,000 of
these shares at Rs. 135 per share.

(ii) On 1 July 2022, Sageer obtained an interest free loan of Rs. 1,500,000 from KI in exchange for which he
agreed to waive the interest receivable on his provident fund balance maintained with KI. Interest provided
on provident fund balance for the year was 8%. The prescribed benchmark rate is 10%.

(iii) On 31 August 2022, he received leave encashment of Rs. 100,000 relating to previous year.

(iv) During the year, tax of Rs. 160,000 was deducted at source by KI.

Other information relevant to tax year 2023:


(i) On 15 January 2023, he sold a shop situated in Karachi for Rs. 15,000,000. He had purchased this shop in
2021 for Rs. 19,000,000 out of which Rs. 5,000,000 was paid in cash.
(ii) On 1 March 2023, he sold a residential plot situated in Faisalabad for Rs. 18,000,000. The plot was
inherited from his father in 2014. Fair market value of the plot at the time of inheritance was Rs. 7,000,000.
(iii) In June 2023, Sageer independently developed learning courses for sale through a web based marketplace
managed by a company situated outside Pakistan. On 25 June 2023, he received USD 4,260 into his dollar
account from sale of these courses. Withholding income tax @ 8% was deducted from the receipt as per
the income tax laws of the foreign country.

Relevant exchange rates were as follows:

25 June 2023 USD 1 = PKR 168


30 June 2023 USD 1 = PKR 169
Average exchange rate for June 2023 USD 1 = PKR 168.5

(iv) On 1 June 2023, Sageer paid Rs. 2,500,000 as donation to a non-profit organization listed in the Second
Schedule of the Income Tax Ordinance, 2001.

Required:
Under the provisions of the Income Tax Ordinance, 2001 and Rules made thereunder, compute the taxable
income and net tax payable by or refundable to Sageer for the year ended 30 June 2023. Show all relevant
exemptions, exclusions and disallowances. (19)

Question # 20 Autumn 2021 Q. 1

Nauman has been working as manager finance in Dua Limited (DL), a public listed company, for many years.
He received following monthly emoluments from DL during the year ended 30 June 2023:

191 | P a g e
Rupees
Basic salary 120,000
Medical allowance 20,000
House rent allowance 60,000

In addition to the above, the employer also provided him the following benefits:
(i) Company maintained car for both official and personal use. The car was purchased on 1 July 2018 at the
cost of Rs. 1,400,000. As per company policy, Nauman purchased this car at its book value of Rs. 450,000
on completion of five years i.e. 30 June 2023. Fair market value of this car on the date of sale to Nauman
was Rs. 1,000,000.
(ii) Provident fund contribution of Rs. 18,000 per month to a recognized provident fund. An equal amount
was also contributed by Nauman to the fund. Interest income of Rs. 540,000 at the rate of 18% of
accumulated balance of the fund was credited to Nauman’s account.
(iii) On 1 July 2022, he was transferred to Lahore and was paid relocation allowance of Rs. 300,000.
(iv) HR Committee approved a performance bonus for the year ended 30 June 2023 for all employees. Nauman
received Rs. 400,000 as performance bonus on 15 July 2023.
(v) On 1 April 2023, Nauman obtained a loan of Rs. 5,000,000 @ 6% per annum from DL to purchase a new
house for his own use. First instalment of the loan was paid on 30 June 2023. He incurred legal expenses
of Rs. 20,000 for obtaining the loan.

Other information relevant to tax year 2023:


a) During the year, Nauman received interest income of Rs. 510,000 on his investments in defence savings
certificates. The amount was net of withholding income tax at the rate of 15% and Zakat of Rs. 200,000
was deducted under the Zakat and Usher Ordinance, 1980.
b) On 1 October 2022, Nauman received advance rent of Rs. 1,200,000 for 12 months for renting office
premises. This amount includes Rs. 400,000 for utilities, cleaning and security. During the tax year 2023,
Nauman incurred following expenditures in relation to the premises:

Rupees
Repair and maintenance 70,000
Insurance premium 50,000
Administration and collection charges of rent 30,000
Utility, cleaning and security 250,000

Nauman has opted normal tax regime for chargeability of tax on income from property.

Required:
Under the provisions of the Income Tax Ordinance, 2001 and Rules made thereunder, compute total income
and taxable income of Nauman for the tax year 2023. Show all relevant exemptions, exclusions and
disallowances. (13)

Question # 21 Spring 2022 Q. 1

For the purpose of this question, assume that the date today is 31 August 2023. Basit, a senior manager at
Master Limited (ML), resigned on 31 January 2023 after completion of three and a half year of service. During
the tax year 2023, he received the following emoluments from ML:
(i) Salary of Rs. 610,000 per month.
(ii) Allowance of Rs. 60,000 per month for services of domestic servant. Out of which, he paid Rs. 36,000 per
month in respect of these services.
(iii) Allowance equal to 5% of salary solely expended in the performance of his duties of employment.

Additional information:
(i) On 1 July 2022, he leased a car having fair market value of Rs. 4,800,000 at a monthly rental of Rs.

192 | P a g e
120,000. He pays lease rentals from his own sources but has used this vehicle for both official and personal
purposes.
(ii) On 1 July 2022, 13000 shares of ML were allotted to Basit under an employee share scheme, against the
payment of Rs. 30 per share. According to the scheme, he was not allowed to sell transfer the shares upto
31 December 2022. On 31 May 2023, he sold 5000 shares of ML at its fair market value (FMV). FMV of
each share on different dates are as follows:

1 July 2021 31 December 2021 31 May 2022


Rs. 50 Rs. 90 Rs. 80

(iii) On 15 February 2023, he received the following payments from ML as final settlement:
▪ Rs. 320,000 on account of leave encashment.
▪ Rs. 2,200,000 under gratuity scheme approved by the board.
▪ Rs. 700,000 salary arrears related to tax year 2022.
(iv) Withholding tax deducted by ML from Basit’s salary during the tax year 2023 amounted to Rs.1,400,000.

Other information:
(i) On 31 January 2023, he received gold worth Rs. 200,000 as a gift from his old friend.
(ii) On 1 February 2023, he purchased mutual fund units of Rs. 2,500,000.
(iii) On 1 April 2023, he left for United Kingdom and joined Oliver Limited (OL) as an employee at a monthly
salary of GBP 3,200. He remained abroad till end of the tax year 2023. No withholding tax was deducted
by OL from his salary.
(iv) While residing in UK, Basit served as a visiting faculty member at a University. He earned GBP 1,500
from the university and incurred an expenditure of GBP 500 for providing services at the university.
Withholding tax deducted by the university amounted to GBP 225.
(v) Average exchange rate during 1 April 2023 to 30 June 2023 was GBP 1 = Rs. 250.

Required:
Under the provisions of the Income Tax Ordinance, 2001 and Rules made thereunder:
(a) compute the total income, taxable income and net tax payable by or refundable to Basit for the tax year
2023. (Show all relevant exemptions, exclusions and disallowances) (16)
(b) what other option is available to Basit for the taxation of salary arrears of Rs. 700,000 received from ML
as part of final settlement. (Revised computation is not required) (01)
(c) identify the additional statement that Basit needs to file in respect of his foreign source income. Also
briefly discuss the particulars to be mentioned in the additional statement. (02)

Question # 22 Autumn 2022 Q. 2

Nasir has been working as head of finance in Asaaish (Private) Limited (APL). He received following monthly
emoluments from APL during the year ended 30 June 2023:

Rupees
Basic salary 800,000
Medical allowance 100,000
Cost of living allowance 200,000

In addition to the above, APL also provided him the following benefits:
(i) Residential house owned by APL for no rent. The fair market value of the rent was Rs. 300,000 per month.
(ii) Company maintained car. The car was acquired on lease by APL on 1 July 2021 at an annual rental of Rs.
1,100,000. The fair market value of the car as on 1 July 2021 and 30 June 2023 were Rs. 4,000,000 and
Rs. 6,000,000 respectively. 70% of the car is used for office purpose while 30% is used for personal
purposes.
(iii) 250 liter of fuel every month. The average petrol price during the year was Rs. 180 per liter.
(iv) Reimbursement of car maintenance expenses upto Rs. 20,000 per month. During the year, APL reimbursed

193 | P a g e
Rs. 150,000 to him in this respect.
(v) Health insurance for Nasir and his dependents as per the terms of employment. For this purpose, APL is
paying annual insurance premium of Rs. 100,000. The insurance company incurred expenses of Rs.
500,000 on hospitalization of his dependents.
(vi) Ad-hoc relief allowance equal to one month’s basic salary keeping in view the increase in inflation.

Nasir incurred a monthly expenditure of Rs. 20,000 from July 2022 to November 2022 while working from
home under the COVID guidelines issued by APL’s management.

Withholding tax deducted by APL from his salary during the tax year 2023 amounted to Rs. 4,500,000.

Other information:
(i) During the year, he got married and received Rs. 1,000,000 in cash as gifts from various relatives and
friends including Rs. 400,000 from his parents. In addition, his parents gifted him a car worth Rs.
5,000,000.
(ii) During the year, he paid a cash donation of Rs. 480,000 to a non-profit organization listed in the Thirteenth
Schedule.
(iii) During the year, Nasir contributed Rs. 4,700,000 to an approved pension fund under the Voluntary Pension
System Rules, 2005.

Required:
Under the provisions of the Income Tax Ordinance, 2001 and Rules made thereunder:
(a) compute the total income, taxable income and net tax payable by or refundable to Nasir for the tax year
2023. (Show all relevant exemptions, exclusions and disallowances) (14)
(b) With reference to provision of residential house to Nasir by APL, discuss the tax implication for APL in
this regard. (02)

Question # 23 Q.1 March 2023

For the purpose of this question, assume that the date today is 30.9.2024.
Cheng, a Chinese citizen, has been in Pakistan since the year 2016. For career growth, he left his employment
with Hope Ltd (HL), an unlisted FMCG, on 30.9.2023 and joined a leading chain of hotels namely Desire
Hotels (DH) on 1.10.2023.
The details of his emoluments during the year ended 30.6.2024 are as follows:
HL DH
---- Rupees----
Basic salary per month 350,000 450,000
Medical allowance per month 35,000 60,000
Utilities allowance per month 20,000 30,000
Lunch provided by the employer at subsidized rate per month cost to the 10,000 25,000
employer
Company maintained car (for both official as well as personal purposes):
• Cost 3,000,000 5,000 ,000
• Fair market value as on 30.6.2024 2,500,000 6,000,000
Annual bonus related to the tax year 2023 350,000
Gratuity under an unapproved scheme 1,225,000

Additional information (other benefits provided by DH):


i. Two return air tickets to China to the extent of Rs.600,000 for Cheng and his spouse. During the year,
he incurred Rs.550,000 on account of his traveling to China.
ii. Rs.750,000 received for signing a bond with DH, according to which Cheng cannot leave the
organization before 30.6.2025.
iii. Rs.400,000 received from DH as commission for securing a large contract.

194 | P a g e
iv. Payment of the outstanding loan of Rs.3,800,000 by DH as per the terms of the employment contract.
Cheng had obtained this interest free loan from HL, for the construction of a house. On 1.7.2020, the
house was given on rent under a 5-year rental agreement at an annual rental of Rs.800,000.

Other information:
i. During the year, he received CNY 31,500 (net of 30% tax) equivalent to Rs.1,260,000 in his Pakistani
bank account being a share of profit from a business in China.
ii. During the year, he received cash dividend of Rs.97,750 (net of withholding tax @ 15%) and bonus
dividend of 2,000 shares from Ambitious Ltd (AL), a company listed on the Pakistan Stock Exchange.
The fair market value of AL's share on the date of entitlement of bonus shares and on 30.6.2024 were
Rs.25 and Rs.20 respectively.
iii. He transferred an amount of Rs.600,000 to the bank account of a non-profit organization as donation
for the flood affectees.
iv. On 25.6.2024, he paid an annual premium of Rs.300,000 on a life insurance policy.
Required:
Under the provisions of the Income Tax Ordinance, 2001 and Rules made thereunder, compute total income,
taxable income and tax payable by or refundable to Cheng for the tax year 2024 (Show all relevant
exemptions, exclusions and disallowances) (Marks 19)

Question # 24 Q.1 September 2023

Salman works as the head chef at Tasty Café (TC), which is owned by Zameendar Limited (ZL),
a company listed on the Pakistan Stock Exchange. He received the following emoluments from
TC during the year ended 30 June 2023:
Rupees
Basic salary 2,400,000
House rent allowance 600,000
Annual bonus 480,000

In addition to the above, TC also provided him with the following benefits:
(i) Free meals while on duty worth Rs. 450,000.
(ii) Staff discounts across all restaurants owned by ZL. Salman availed discount worth Rs.
40,000 during the year.
(iii) Health insurance up to the amount of Rs. 500,000 covering his spouse and two children
as per terms of his employment. The insurance premium related to this benefit amounted
to Rs. 70,000. During the year, the insurance company incurred
expenses of Rs. 300,000 on hospitalization of his dependents.
(iv) A company-maintained car which was purchased by TC for Rs. 2,000,000. The fair
market value (FMV) of the car on 30 June 2023 is Rs. 5,500,000. The car is used
partly for official purposes and partly for personal use.
(v) An option to acquire 12,000 shares of ZL at a price of Rs. 98 per share under an employee
share scheme. The option was granted on 1 February 2023 when the FMV of the shares
was Rs. 104 per share. Salman exercised the option on 1 March 2023 when the FMV was
Rs. 110 per share.
(vi) Provident fund contribution at 8% of the basic salary in a recognized provident fund.
During the year, 22% profit was credited to Salman’s account. His balance in the fund
amounted to Rs. 960,000.

195 | P a g e
(vii) Training of three-week culinary chef course. Salman attended this course in Dubai, for
which TC paid a course fee of Rs. 500,000. In relation to his stay in Dubai, he was
provided a travel allowance of Rs. 250,000. However, Salman opted to stay at a relative’s
house in Dubai and only spent Rs. 100,000 out of the total travel allowance provided by
TC.
Additional information:
(i) During the year, Salman received an interest income of Rs. 765,000 from his bank
account. The amount was net of withholding income tax at the rate of 15%.
(ii) Salman paid Rs. 300,000 as a donation to a non-profit organisation listed in the Thirteenth
Schedule of the Income Tax Ordinance, 2001. 80% of the amount was paid through a
crossed cheque, and the remaining amount was paid in cash.
(iii) During the year, TC withheld Rs. 400,000 in tax from his salary.
Required:
Under the provisions of the Income Tax Ordinance, 2001 and Rules made thereunder, compute
under the correct head of income, the total income, the taxable income, and net tax payable by
or refundable to Salman for the tax year 2023. (Show all relevant exemptions, exclusions and
disallowances) (14)

Question # 25 [CFAP Questions for Jazabati Students] Q.1 December 2009

Ms. Saima is a telecommunication engineer working with a leading GSM operator as their chief technical
officer for the last many years.
She has provided you with the following information relating to her assessment for the year ended 30.6.20X9:
i. Monthly salary of Rs.500,000 was paid to her by the company consisting of the following:
Basic Salary 400,000
Medical allowance 40,000
Conveyance allowance 60,000
The salary was credited to her bank account on the 25th of every month.
She incurred actual medical expenses of Rs.100,000 during the year. These expenses were reimbursed
to her by the company in accordance with the terms of her employment.
ii. She received a bonus of Rs.1 million. Employer also agreed to pay tax on such bonus to the extent of
Rs.100,000.
iii. Apart from her employment with a GSM operator, she also served as a visiting faculty member at a
local engineering university and received a total of Rs.445,000 net of tax deduction at source @ 11%.
Ms. Saima incurred an expenditure of Rs.70,000 towards this service.
iv. In August 20X8, she participated and won prize money of Rs.200,000 in a quiz competition arranged
by Pakistan Urdu Academy. The prize money was paid to her after tax deduction of Rs.40,000.
v. She inherited a plot of land from her father on his death 12 years ago. On 1.10.20X8 she entered into a
contract of sale with Mr. Moin for a consideration of Rs.50 million. Mr. Moin paid a deposit of Rs.1
million and agreed to pay the balance within one month of the date of contract.
On due date, Mr. Moin defaulted in making the payment upon which Ms. Saima forfeited the deposit
in accordance with the terms of the contract.
Later on, the plot was sold to Mr. Parkash at a price of Rs.50 million.
vi. Ms. Saima purchased another plot of land for a consideration of Rs.56 million. She borrowed Rs.5
million from her sister for the purchase of this plot. The amount was received in cash.
vii. Ms. Saima also inherited a painting from her father few years ago. The painting was valued at
Rs.500,000 at the time of inheritance. On 1.4.20X9 she gifted the painting to her brother who came
from Canada after five years. He went back to Canada after staying in Pakistan for a period of two
months. The value of the painting was Rs.1 million when it was gifted.

196 | P a g e
Required:
Compute the taxable income of Ms. Saima for the tax year 20X9. Give brief reasons under the
Income Tax Ordinance, 2001 in support of your treatment of each of the above items. (Marks 18)

Question # 26 [CFAP Questions for Jazabati Students] Q.1 June 2011

Mr. Khan has been working for a listed company Turtle Ltd (TL) for the last many years. The details of his
emoluments during the tax year ended 30.6.20X4 are as under:
Basic salary (per month) Rs. 350,000
Conveyance allowance (per month) Rs. 50,000
In addition to the above cash emoluments, Mr. Khan was also provided with the following:
a) A rent free furnished accommodation with a fair market rent of Rs.100,000 per month.
b) An 1800cc company maintained car, both for business and private use. The car was purchased by TL
on 1.7.20X1 at a fair market value of Rs.2,000,000.
c) On 1.7.20X3 he was provided with an interest free loan of Rs.2,500,000 which is repayable in lump
sum in December 20X4. The prescribed benchmark rate is 10% per annum.
In order to increase its operational efficiency, TL announced a redundancy scheme to its employees. Mr. Khan
opting for the scheme resigned from TL with effect from 1.1.20X4.
Upon resignation, 25% of his outstanding loan balance was waived by TL and the remaining loan amount was
adjusted from his final settlement.
He received the following payments from TL:
Compensation under the redundancy scheme 4,000,000
Gratuity under unapproved scheme 2,000,000
Following further information is also available:
i. Tax of Rs.837,000 was withheld by TL from the above payments.
ii. Mr. Khan was allowed to purchase the 1800cc car at an accounting book value of Rs.1,000,000 which
he sold immediately in the open market at a price of Rs.1,500,000.
iii. On 1.3.20X4 Mr. Khan rented out the ground floor of his bungalow to Mr. Riaz, for establishing a
departmental store, at a monthly rent of Rs.137,500. Due to the strategic location of the store, he also
received adjustable and non-adjustable deposits of Rs.600,000
iv. On 1.4.20X4 he rented out the residential portion of the bungalow to a Commercial Bank for their
marketing executive. He received gross amount of Rs.2,400,000 as two years' advance rent. The Bank
deducted tax of Rs.230,000 from such payment.
v. A donation of Rs.500,000 was made to an un-approved trust for the construction of mosque.
vi. Mr. Khan was issued shares in TL before 1.7.2013. The fair market value of shares at the and
Rs.500,000 respectively.
He disposed off these shares in June 20X4 at a gain of Rs.500,000.
Required:
Compute the taxable income, tax liability and tax payable/ refundable, if any, to Mr. Khan for the tax year
20X4. The average rate of tax of Mr. Khan for the last three years was 8%.
Note: Show all exemptions, exclusions and disallowances where relevant. (Marks 20)

Question # 27 [CFAP Questions for Jazabati Students] Q.1 December 2012

Mr. Yaqeen, a Pakistani citizen, returned to Pakistan on 30.6.20X1 after residing for six years in
Norway.
On 1.7.20X1 he joined a private hospital KKUH and received the following emoluments:
Basic salary (per month) 500,000
Leave fare assistance 60,000

197 | P a g e
Medical allowance (per month) 240,000
On 1.1.20X2 Mr. Yaqeen resigned from the hospital and joined Dil (Private) Ltd (DPL), a company engaged
in health care and production of dental products. Mr. Yaqeen received Rs.3,000,000 from DPL as
consideration for joining the company.

DPL agreed to pay following emoluments to Mr. Yaqeen for the tax year 20X2:
Basic salary (per month) 800,000
Medical allowance (per month) 80,000
Utilities allowance (per month) 100,000
• On 1.1.20X2 DPL provided him with refrigerator, cooking range and washing machine for his use at
home. The book value of these appliances was Rs.200,000 and these were returnable to the company
after four years. 15% depreciation was charged by DPL on these appliances.
• On 31.3.20X2 he was given an option to purchase 2,000 shares of DPL at Rs.50 per share. The break-
up value of the company on that date was Rs.150 per share. Mr. Yaqeen did not exercise this option
till 30.6.20X2.
• On 1.4.20X2 he received a loan of Rs.5,000,000 from DPL for the purchase of a house. The profit on
loan was payable @ 8% per annum. The prescribed bench mark rate is 10% per annum.
Other information relevant to Mr. Yaqeen for the tax year 20X2 is as under:
i. On 30.4.20X2 he received salary arrears of Rs.900,000 from his ex-employer in Norway.
ii. Mr. Yaqeen had 30 acres of agricultural land in Dheer which he did not cultivate himself. During tax
year 20X2 he received annual rent of Rs.600,000 from the tenant cultivating the land.
iii. On 1.5.20X2 he spent Rs.800,000 on the renovation of his residential house. The entire amount was
obtained as a loan from a scheduled bank on which a profit of Rs.20,000 was paid to the bank during
the tax year 20X2.
iv. On 15.6.20X2 he received insurance claim of Rs.600,000 against theft of a painting which was stolen
on 31.5.20X2. The painting was purchased by him on 1.1.20X1 for Rs.350,000. He had paid
insurance premium of Rs.24,000 and also paid lawyer's fee of Rs.50,000 who represented him in the
settlement proceedings.
v. On 15.7.20X1 Mr. Yaqeen received 20,000 shares in AB (Private) Ltd (ABL), a company
incorporated under the Companies Act, 2017 as a dividend in specie. On 30.6.20X2 he sold 15,000
shares in ABL for Rs.425,000. The fair market value of these shares, on the date of issue, was
estimated at Rs.25 per share.
Required:
Under the provisions of Income Tax Ordinance, 2001 compute the taxable income and net tax payable for the
tax year 20X2.
Give brief reasons for the treatment of items in (iv) and (v) above. Also explain the treatment of
any items that are not appearing in your computation. (Marks 25)

Question # 28 [CFAP Questions for Jazabati Students] Q.1 December 2013

Mr. Iqbal, aged 45 years, is working as a Chief Engineer in a listed company Tameer Ltd (TL). The company
is engaged in the manufacture of chipboards for the local market.
He derived following emoluments during the tax year ended 30.6.20X4:
Basic salary (per month) 300,000
Cost of living allowance (per month) 50,000
Milk allowance (per month) 10,000
In addition to the above emoluments, Mr. Iqbal was also provided the following:
i. Special bonus equal to one month's basic salary paid on 5.6.20X4.

198 | P a g e
ii. A new company maintained car for his personal use. The car was purchased on 1.3.20X4 at a cost of
Rs.1,800,000. However, the cost of the car would have been Rs.3,000,000 had the company obtained
it on finance lease.
Mr. Iqbal, in accordance with the terms of his employment, purchased his previous car from TL for
Rs.250,000. This car was provided to him solely for business purposes. The fair market value of the
car at the time of sale to Mr. Iqbal was Rs.600,000.
iii. A reimbursement of Rs.36,000 in respect of driver's salary. Mr. Iqbal paid Rs.60,000 to the driver for
four months.
iv. A fully furnished accommodation in DHA, Karachi. The fair market value of the rent was estimated
to be Rs.85,000 per month.
v. An option to acquire 4,000 shares in TL's parent company, Tameer Inc. which is listed on New York
Stock Exchange was granted to him in May 20X3. Mr. Iqbal exercised the option on 5.1.20X4 at a
price of USD 1.5 per share.
The market value of the shares at the close of business on 5.1.20X4 was USD 2.5 per share. He sold
3,000 shares on 30.6.20X4 at a price of USD 3 per share.
The dollar rupee parity on both the above dates was USD 1 = Rs.100.
vi. On 15.5.20X4 Mr. Iqbal was provided 800 shares in TL as a reward for his excellent performance.
However, he was restricted from selling or transferring these shares before 16.11.20X4. The market
value of these shares at the close of business on 15.5.20X4 was Rs.12.5 per share.
Mr. Iqbal received additional income from the following sources, for the tax year 20X4:
i. Brokerage fee of Rs.200,000 in connection with the transfer of two apartments in Islamabad. The
brokerage fee was received in cash. Mr. Iqbal incurred an expense of Rs.30,000 against telephone
costs and air travel to Islamabad in connection with the above deal. He also paid Rs.10,000 as a gift to
his brother for showing the apartments to his clients in Islamabad.
ii. Profit of Rs.150,000 on a savings account maintained with an Islamic bank. The bank deducted
withholding tax of Rs.15,000.
iii. Zakat of Rs.25,000 has been deducted by the bank.
iv. He also received an income tax refund of Rs.225,000 related to tax year 20X2. The amount included
Rs.25,000 being compensation for delayed refund.
v. Annual rent of Rs.800,000 from letting out a building to KK Enterprise. Following expenses were
incurred by Mr. Iqbal in relation to the building:
Repairs Rs.200,000, Fire insurance premium Rs.30,000, Ground rent Rs.10,000, Watchman's salary
Rs.8,000 and Interest of Rs.15,000 on a loan obtained for building renovation by creating first charge
on the building in favour of a scheduled bank.
Other related information is as under:
• TL deducted withholding tax of Rs.600,000 from Mr. Iqbal's salary during tax year 20X4.
• On 1.7.20X3, Mr. Iqbal acquired a life insurance policy and paid a premium of Rs.500,000. He also
contributed Rs.1,600,000 to an approved pension fund.
• During tax year 20X3 his assessed taxable income was Rs.3,000,000.
Required:
Under the Income Tax Ordinance, 2001 and Rules made thereunder, compute the taxable income and income
tax payable by or refundable to Mr. Iqbal for the tax year ended 30.6.20X4. Show all exemptions, exclusions
and disallowances where relevant. (Marks 22)

Question # 29 [CFAP Questions for Jazabati Students] Q.6 June 2015

Mr. Pansari, a Pakistani citizen, is working as a company secretary in Sukoon Ltd (SL), an un-listed public
company, engaged in the business of production and supply of olive oil. Following are the details of his
emoluments during the year ended 30.6.20X4:

199 | P a g e
Basic salary per month 450,000
Conveyance allowance per month 50,000
In addition to the above cash emoluments, Mr. Pansari was also provided with the following:
i. A 2000cc company maintained car both for business and private use. The car was purchased in tax
year 20X3 at a cost of Rs.3,000,000. However, the current market value of the car is Rs.3,500,000.
ii. A special payment of Rs.75,000 in lieu of leave was made available to him. Mr. Pansari however,
voluntarily waived his right to receive such payment.
iii. Free provision of two cans of olive oil per month. The market value of each can was Rs.500.
iv. In July 20X2 he was granted an employee stock option to purchase up to 15,000 shares in SL's
holding company Trio Ltd, situated in Bermuda, at an option price of USD 3 per share.
The shares were required to be purchased within 18 months from the option date. Mr. Pansari
exercised the option in September 20X3 to purchase 8,000 shares when the market price of the shares
was USD 5 per share.
After two months of the acquisition, Mr. Pansari sold 6,000 shares at a price of USD 8.5 per share.
[Assume the dollar rupee parity on the above dates was USD 1 = PKR 102].
Following further information is also available:
i. Received a royalty of Rs.2,000,000 from K Publishing on a book written on Wild Hunting. Mr.
Pansari completed the book in 19 months and all the costs relating to its publication were borne
by the publisher.
The applicable tax rates in tax years 20X2 and 20X3 were 16% and 18% respectively.
ii. Received a pension of Rs.50,000 from his ex-employer.
iii. Rs.200,000 gross fee for attending a directors' meeting of SL's associated company Nice (Pvt) Ltd
held in June 20X3 received in July 20X3 after tax deduction @ 20%.
iv. There was a brought forward capital loss of Rs.25,000. The loss was suffered by Mr. Pansari on
sale of shares in Ghareeb (Pvt) Ltd in the immediately preceding tax year.
Required:
Under the provisions of the Income Tax Ordinance, 2001 and Rules made thereunder, compute the taxable
income of Mr. Pansari for the tax year 20X4.
Note: Show all relevant exemptions, exclusions and disallowances. (Marks 8)

200 | P a g e
ANSWERS

Question # 1 Q.15 September 2004

Mr. A
Tax Year 20X8
Computation of taxable income and tax liability
SALARY
Basic salary 4,004,520
Bonus 1,980,642
Utility allowance 400,452
Leave encashment 538,083
Other allowance 90,000
House rent allowance 1,802,040
Director's fee 52,000
8,867,737
CAPITAL GAIN
Gain on sale of shares of an unlisted company i.e. private company 4,206,000

INCOME FROM PROPERTY


Actual rent Rs.480,000 or fair market rent Rs.450,000 whichever is higher 480,000
Repairs allowance 1/5th of chargeable rent- (96,000)
Property tax (11,500) 372,500

INCOME FROM OTHER SOURCES


Profit on debt 6,500
13,452,737
Less: Profit on debt taxable at separate rates 6,500
Taxable Income 13,446,237

Tax liability (Non-salaried Case)


Income tax on Rs.4,000,000 765,000
Income tax on Rs.9,446,237 @35% 3,306,183
4,071,183
Income tax on profit on debt 15% of Rs.6,500 975
Total tax liability 4,072,158
Less: Tax withheld from salary 1,200,000 + 10,400 1,210,400
Tax withheld from profit on debt 650 1,211,050
Tax payable with return of income 2,861,108

Question # 2 Q.2 September 2005

Mr. Imran
Tax Year 20X8
Computation of taxable income
SALARY from UAE Company
Exempt from tax in Pakistan (Note) --

201 | P a g e
SALARY from Pakistan subsidiary
Basic salary 500,000 x 9 4,500,000
Medical allowance 45,000 x 9 405,000
Less: Exempt up to 10% of basic salary 450,000 Nil
TV and VCR: 20% of 40,000 = 8,000 x (9/12) 6,000
Interest free loan: 5 million x 10% for 9 months 375,000
Housing cost in Dubai paid by the Pakistani company: 30,000 x 9 270,000
Traveling cost: 30,000 x 9 270,000
Employee Share Scheme
Disposal of option 200 shares x 171 34,200
- Shares acquisition ($10-$8) x 58 x 300 34,800 69,000
Taxable salary 5,490,000

Income from property


Chargeable rent 12 x 100,000 1,200,000
Repairs allowance 1/5th of chargeable rent (240,000)
Property tax (35,000)
Notional interest at benchmark rate 5m x 10% for 9 months (375,000) 550,000
Taxable Income 6,040,000

Note for salary from UAE company:


Salary earned outside Pakistan shall be exempt if a citizen of Pakistan leaves Pakistan during a tax year and
remains abroad during that tax year - Section 51(2). Therefore, foreign source salary of Mr. Imran would not
be taxable in Pakistan.

Question # 3 Q.1 March 2006

Ms. FH
Tax Year 20X8
Computation of taxable income
SALARY
Basic salary 100,000 x 11 1,100,000
House rent allowance 40,000 x 11 440,000
Utility allowance 15,000 x 11 165,000
Purchase of car from the employer 300,000 - 100,000 200,000
Payment to accept the offer of PIL 200,000
2,105,000
CAPITAL GAIN
Shares in Queens Pakistan (Pvt) Ltd (Rs.500,000-200,000) 300,000
Sale of residential plot (Rs.1,000,000-200,000) 800,000
3,205,000
Less: Capital gain on sale of plot taxable at separate rates 800,000
Income taxable at normal slab rates 2,405,000

Notes for items not included in taxable income:


a) Car for office use: No amount is taxable if the car is used exclusively for office use.

202 | P a g e
b) Painting: Painting is a capital asset. However, there is a specific category of capital assets including painting
on which capital gain, if any, is taxable but capital loss is not recognized. Therefore, capital loss on disposal of
painting is not considered in the computation of income.
c) Donation: Rebate is allowed on donation to an approved charitable institution through banking channel.

Notes on income covered under FTR:


The following items are covered under Final Tax Regime whereby tax deduction at source is considered as full
and final tax and therefore these items are not included in normal taxable income:
Gross amount Final Tax
Cash prize in a quiz show -20% 250,000 50,000
Dividend -15% 50,000 7,500

Question # 4 Q.2(a) March 2007

i. A rare sculpture is a capital asset and its disposal is taxable under the head 'Capital Gain'. Original cost
at the time of inheritance in the hands of the transferor shall be treated as deemed cost for the recipient.
Therefore, capital gain of Rs.500,000-200,000 = 300,000 is taxable.
ii. Forfeited deposit under a contract for sale of immovable property is included in the definition of rent
under the head income from property which is taxable as normal slab rates after considering allowable
deductions.
iii. Car held for personal use is not a capital asset and therefore its disposal does not fall within the ambit
of capital gain.
Sale of a personal car is a capital receipt which is not taxable under any head of income.
iv. Salary shall be Pakistan source income where the salary is received from any employment exercised in
Pakistan, wherever paid. As the services are performed outside Pakistan, salary in this case shall be
foreign source salary.
Foreign source salary shall be exempt if a citizen of Pakistan leaves Pakistan during a tax year and
remains abroad during that tax year -Section 51(2). Therefore, foreign source salary in this case would
not be taxable in Pakistan.

Question # 5 Q.1 March 2009

(a)
Mr. Manto
Tax year 20X8
Computation of Taxable Income and Tax payable
Salary
Basic salary 1,200,000
House rent allowance 360,000
Medical allowance 120,000
Bonus 600,000
Company maintained car: 5% of the FMV of Rs.1,600,000 80,000
Provident fund (assumed to be recognized)
Company's contribution 10% of basic salary 120,000
Less: Exempt up to Rs.150,000 or 10% of basic + --
Dearness allowance whichever is lower 120,000 Nil
Interest credited 48,000 / (120,000+ 120,000) = 20%
Interest credited @ 20% 48,000
Less: Interest @ 16% i.e. Rs.38,400 or 1/3rd of basic +

203 | P a g e
Dearness allowance whichever is lower 38,400 9,600
Taxable salary excluding tax borne by the company 2,369,600
Tax borne by the company (working note) 204,077
Taxable salary 2,573,677

Capital Gain u/s 37


Sale proceed 62 x 25,000 1,550,000
Less: Cost at the time of inheritance is deemed cost @42 1,050,000
500,000
Taxable income 3,073,677

Working for Tax borne by the company


Taxable salary excluding tax borne by the employer 2,369,600
(slab is being changed due to tax borne by the employer)
Taxable salary excluding tax borne by the employer 2,369,600
Initial tax of the next slab 165,000
Taxable salary excluding final figure of tax borne 2,534,600
Income tax on Rs.2,400,000 15,000
Income tax @ 12.5% on Rs.134,600 = 30,285/77.5% 16,825
Grossed up tax 31,825

Proof of the above grossed up figure


Taxable salary excluding tax borne by the employer 2,369,600
Tax borne by the employer 204,077
Taxable salary 2, 573,677
Tax on Rs.2,400,000 15,000
Tax @12.5% on Rs.173,677 21,709
36,709

Tax liability (Salaried Case)


Income tax on Rs.2,400,000 165,000
Income tax on Rs.673,677 @ 12.5% 151,577

Less: Tax borne by the employer 204,077


Tax payable with the return of income 112500

(b) Explanations for items not taken into consideration


1. Free medical treatment
Free medical treatment is exempt if it is in accordance with terms of employment where National Tax Number
of the medical practitioner along with employer's attestation are available. However, in this case medical
allowance is fully taxable.
2. Lunch
Certain perquisites without marginal cost to the 'employer are exempt which include free or subsidized food
provided by hotels and restaurants to its employees during duty hours.

3. Training course
It is assumed that the training course was required and beneficial for performance of office duty. Any perquisite
or allowance solely expended for office purpose is not a part of salary.
4. Final settlement from Berlin Hotel

204 | P a g e
If an individual citizen of Pakistan (returning expatriate) is resident in the current tax year but was non-resident
in the 4 preceding tax years, his foreign-source income shall be exempt in the current tax year and in the
following tax year.
Mr. Manto was out of Pakistan for the last 5 years and returned to Pakistan in the beginning of the tax year
20X8. Therefore, he is a returning expatriate as per Pakistan tax law and his foreign source income including
salary income would be exempt from tax

Question # 6 Autumn 2009 Q. 1

Mr. Zulfiqar
Tax year 20X8
Computation of Taxable Income and Tax payable

Rs.
Salary
Basic salary 280,000 × 9 2,520,000
Medical allowance 45,000 × 9 405,000
Less: Exempt up to 10% of basic salary 252,000 153,000
Utility allowance 45,000 x 9 405,000
Cost of living allowance 25,000 x 9 225,000
Rent free accommodation 45% of basic salary 1,134,000
Tax liability borne by the employer 200,000
Gratuity from unrecognised fund 2,660,000
Less: Exempt up to Rs.75,000 or 50% of amount (75,000) 2,585,000
Pension
From Mirza Petroleum (50,000 x 3) 150,000 Exempt
From multinational Company (12,000 x 12) 144,000 144,000
Taxable salary 7,366,000

Income from property


Chargeable rent 100,000 × 6 600,000
Less: Repairs allowance 1/5th of chargeable rent (120,000) 480,000

Capital gain
Disposal of Plot (5,000,000 – 1,000,000 = 4,000,000) Sold after 6 years Exempt
Loss on Sale of painting is not allowed as deduction N/A

Taxable income 7,846,000

Tax liability (Salaried Case)


Tax liability on Income under NTR 1,095,000
(1,846,000 x 32.5%) 646,100
Tax to be borne by the employer (200,000)
Tax lability of Mr. Zulfiqar 1,541,100

(b) Explanation for items not taken into consideration


Painting: Painting is a capital asset. However, there is a specific category of capital assets including
painting on which capital gain, if any, is taxable but capital loss is not recognized. Therefore, capital loss
on disposal of painting is not considered in the computation of income.

205 | P a g e
Question # 7 Autumn 2010 Q. 1

Mr Zameer Asari
Computation of taxable income and tax thereon
For the tax year 2023
Rupees
Income from salary (A)
Basic salary (Rs. 200,000 x 12) 2,400,000
Medical allowance (Rs. 30,000 x 12) [Note 2] 360,000
Utilities allowance (Rs. 10,000 x 12) 120,000
School fees (Rs. 15,000 x 12) 180,000
Rent free furnished accommodation (Rs. 2,400,000 x 45%) 1,080,000
Car used for business purposes only (exempted) -
Car used for personal as well as business purposes (Rs.1,800,000x5%) 90,000
Payment to an approved pension fund (exempted) -
Employee share option - charged to salary in tax year 2020 and Leave encashment (paid after -
tax year 2023)
Payment to approved pension fund-exempt -
Income from Salary 4,230,000

Income from property (B)


Rental income 264,000
Add: Un-adjustable security deposits {132,000 (W-1) x 1/10} 13,200
Income from property – gross 277,200
Less: 1/5th repair allowance (55,440)
Income from property - Net 221,760

Capital gain (C)

Gain on sale of shares [15,000 shares x Rs. (48-42) 90,000

Income from other sources (D)


Profit on fixed deposit (FTR) 75,000

Total income (A+B+C+D) 4,616,760

Less: Zakat (2,500)


Less: Profit on fixed deposit (FTR) (75,000)

Taxable income 4,539,260

Computation of tax liability


Tax on Rs. 4,539,2600 [435,000+(4,539,260 – 3,600,000) x 27.5%] 693,297
Tax liability on profit on debt under FTR (Rs.75,000 x 15%) 11,250
Total tax liability 704,547
Less: Tax deducted at source
On salary income (250,000)
On profit on debt (7,500)
Balance tax payable 447,047

W-1: Un-adjustable security deposits


Received from new tenants 150,000
Amount charged to tax in 2020 & 2021 (Rs. 90,000 x 2/10) (18,000)

206 | P a g e
132,000

Note 1: No addition in salary income is needed for car provided by the employer solely for business use.
Note 2: As medical facility was also available, medical allowance is fully chargeable.
Note 3: The deductible allowance u/s 60D on child’s educational expenses shall not be allowed as the taxable
income is more than Rs.1,500,000.

Question # 8 Spring 2011 Q. 1

Mr. Mateen
Tax Year 2023
Computation of taxable income and tax liability

Rs.
SALARY
Compensation in lieu of unserved notice 280,000
Gratuity received from unapproved fund maintained by ML 350,000
Less: Exempt up to lower of 50% of gratuity or Rs.75,000 (75,000) 275,000
Leave encashment 150,000
Basic salary (245,000 × 12) 2,940,000
Utility allowance (21,000 × 12) 252,000
Company maintained car (5% of Rs.1,500,000 × 10/12) (b) 62,500
Salary of house keeper (6,000 x 12 × 80%) 57,600
Special allowance to meet office expenditures 50,000 -

Interest free loan (1,500,000 × 10% × 6/12) (c) 75,000


Commission from ML (d) 500,000
Taxable salary 4,592,100

INCOME FROM BUSINESS


Foreign source business income from Malaysia 535,000
Pakistan source business loss (350,000) 185,000

INCOME FROM OTHER SOURCES


Consideration received for vacating possession (180,000 /10) 18,000
Taxable income 4,795,100

Tax liability (Salaried Case)


Income tax on Rs.3,600,000 435,000
Income tax on Rs. 1,195,100 @ 27.5% 328,653
763,653
Less: Tax credit on foreign source income on lower of:
Tax paid in Malaysia 130,000
Pakistan tax payable (763,653/ 4,795,100) × 185,000 29,463
Whichever is lower 29,463
734,190

Less: Tax withheld from salary 550,000


Tax withheld on commission receipts (500,000 × 12%) 60,000 (610,000)

207 | P a g e
Tax payable with return of income 124,190

Notes:
a) Profits in lieu of salary:
Compensation paid by RSL in lieu of un-served notice period by Mr. M is taxable as profits in lieu of salary
as the same is received as consideration to enter into employment agreement with RSL.
b) Company maintained car:
5% of the cost of car to the employer is taxable where the car is used for office and private purposes.
c) Interest free loan:
Since interest free loan is obtained from employer, such benefit at the benchmark rate at 10% is taxable
as salary.
d) Commission received:
Commission is received from the employer while performing employment duties it is therefore taxable
as salary and the tax withheld from commission is adjustable from tax payable. However, if commission
is received from any person other than employer, it is taxable under the head "Income from Business"
e) Receipts from Private clients:
Receipts from private clients is not a part of employment it is therefore taxable as "Income from
Business". Business loss can be adjusted against any head of income other than salary and FTR.

Explanations for items not taken into consideration


a) Reimbursement of medical expenses:
Reimbursement of medical expenses is exempt as the same is attested and certified by RSL and NTN of
hospital is also available.
b) Pick and drop facility:
Since the facility is provided exclusively for business purpose the same is therefore exempt

Question # 9 Autumn 2011 Q. 1

Mr. Khursheed
Tax Year 2022
Computation of taxable income and tax liability
Rs.
SALARY
Basic salary 400,000 × 6 2,400,000
Utility allowance 2,400,000 x 10% 240,000
Medical allowance 75,000 x 6 450,000
Less: 10% of basic salary (240,000) 210,000
Medical reimbursement not in accordance with terms 1,500,000
Company maintained car (5% of Rs.5,000,000 × 6/12) 125,000

Golden handshake payment 7,500,000


Gratuity from unapproved fund 9,100,00
Less: Rs.75,000 or 50% of gratuity whichever is lower (75,000) 9,025,000
Vehicle purchased from employer 2,800,000 - 2,600,000 200,000
Salary Income 21,200,000

INCOME FROM PROPERTY


Chargeable rent 25000 x 3 75,000
Repairs allowance 1/5th of chargeable rent (15,000)
Mark up 60,000 60,000

208 | P a g e
INCOME FROM BUSINESS
Business loss Rs.750,000
It cannot be adjusted against salary income and FT, but it can be adjusted against income from
property. Any unadjusted business loss shall be carried forward for 6 subsequent tax years.
Adjustment against property income (60,000)
Unadjusted business loss c/f 750,000 – 60,000 (690,000)

Income from Capital Gain (700k – 500k) 200,000

Total Income 21,400,000

Less : SBOI
Income from capital gains (200,000)
Golden handshake (750,000)

Total Taxable income under NTR 13,700,000

Less : Zakat (67,500)

Taxable Income under NTR 13,632,500

Tax liability (Salaried Case)


Tax liability on NTR income (1,095,000 + 7,632,500 x 35%) 3,766,375
Add : Tax on Golden Hand Shake ( 750,000 x 5%) 375,000
Add: Tax on gain on securities (200,000 x 12.5%) 25000

Total Tax Liablity 4,166,375


Less: Tax at Source (3750000)
Tax payable 416375

Question # 10 Spring 2013 Q. 1

Mr. Creative
Tax Year
Computation of taxable income and tax liability
Rs.
SALARY
Basic salary 300,000 × 12 3,600,000
Utility allowance 15% of basic salary 540,000
Medical allowance 12% of basic salary 432,000
Less: 10% of basic salary (360,000) 72,000
Accommodation 45% of basic salary 1,620,000
Company maintained car 5% of Rs. 1,600,000 80,000
Taxable salary 5,912,000

INCOME FROM PROPERTY


Chargeable rent (50,000 × 6) + (55,000 × 6) 630,000
Less: Repairs allowance 1/5th of chargeable rent (126,000) 504,000

CAPITAL GAIN

209 | P a g e
Disposal of bonus shares of unlisted company u/s 37
Sale proceeds 6,000 bonus shares x Rs.145 870,000
Cost of Share ( 6000 x 142) (852,000)
18000

Disposal of mutual fund unit’s taxable u/s 37A 4,000 × (Rs.58.6 - 50) 34,400
Total Capital Gain 52,400

Total income 6,468,400


Less: Capital gain u/s 37 being separate block of income 34,400
Taxable income 6,434,000
Less: Zakat (50,000)
Taxable Income Under NTR 6,348,000

Tax liability (Salaried Case)


Tax Liability (1,095,000 + 384,000 x 35%) 1,229,400

Tax rebate on donation (1,229,400 / 6,348,000 × 50,000) (9683)

Tax rebate on APF


Amount of eligible for rebate = actual amount or 20% of taxable income whichever
is lower = Rs.600,000
(1,229,400 / 6,348,000 × 600,000) (116,200)
1,103,517
Income tax on capital gain (34,400 x 7.5%) 2580
1,106,097
Less: Tax deducted from salary (400,000)
Tax payable with return of income 706,097

Question # 11 Autumn 2013 Q. 1

Mrs. Aslam
Tax Year 2023
Computation of taxable income and tax liability
Salary from HPL
Basic salary 2,400,000
Medical allowance 720,000
Less: exempt up to 10% of basic salary (240,000) 480,000
Rent free accommodation: 45% of 2,400,000 1,080,000
Travelling allowance Rs.20,000 × 12 240,000
Contribution to recognized provident fund 240,000
Less: exempt up to lower of Rs.150,000 or 10% of basic + DA (150,000) 90,000
Employee share scheme 5,000 shares × Rs.28 140,000
Laptop from employer 150,000
Taxable salary from HPL excluding tax borne by HPL 4,580,000
Tax borne by HPL as per working note 853,125
Taxable salary from HPL including tax borne by HPL (A) 5,433,125

210 | P a g e
Salary from SL
Leave encashment 95,000
Gratuity from unapproved fund 500,000
Less: Exempt up to lower of Rs. 75,000 or 50% of gratuity amount (75,000) 425,000
Reimbursement against health insurance policy Exempt
Total (B) 520,000

Total Salary (A+B) 5,953,125

Capital Gain under section 37A


Consideration received 5,000 shares × Rs.32 160,000
Less: amount taxed under the head salary (140,000) 20,000

Income From Other Source


Support Payment under an agreement to Live apart (Exempt) 350,000

Total Income 6,323,125


Less: Exempt Income (350,000)
Less: Capital gain being separate block of income (20,000) (370,000)
Total Income Under NTR 5,953,125
Less: Zakat (105,000)
Taxable Income Under NTR 5,848,125

Tax liability (Salaried Case)


Tax Liability ( 435,000 + 2,248,125 x 27.5%) 1,053,234

Rebate on donation (1,053,234/ 5,848,125) × 70,000 12,606


1,040,628
Income tax on capital gain @ 12.5% on Rs.20,000 2,500
Total tax liability 1,043,128
Less: Tax paid by HPL (853,125)
Tax Payable 189,976

W-1 Tax Borne By Employer


Step 1
Salary Income 4,580,000
Tax there on (405,000 + 980,000 x 25%) 650,000
5,230,000
Step 2 Salary Income 5,230,000
Tax there on (405,000 + 1,630,000 x 25%) 812,500

Step 3 Salary Income ( 5,230,000 + 812,500) 5,392,500


Tax there on (405,000 + 1,792,500 x 25%) 853,125

Question # 12 Autumn 2014 Q. 1

Rs.
Salary

211 | P a g e
Basic Salary 480,000 x 12 5,760,000
Medical allowance 48,000 x 12 576,000
Utilities allowance 55,000 x 12 660,000
Accommodation 45% of basic salary 2,592,000
Leave encashment 300,000
Reimbursement of running expenses of personal car 550,000 x 15% 82,500
Fee for attending BOD meeting 200,000
Taxable salary 10,170,500

Income from Business


Net profit from studio 990,000
Add: Tax gain on disposal of studio 800,000
Salary exceeding Rs. 25,000 paid in cash 28,000 x 12 336,000 2,126,000

Income from Other Sources


Literary work Rs. 1,200,000 / 3 400,000
Loan received in cash 2,500,000
Taxable income 15,196,500

Tax liability (Non-salaried Case)


Income tax on Rs. 4,000,000 765,000
Income tax on Rs. 11,196,500 @ 35% 3,918,775
4,683,775
Less: Rebate on donation
4,683,775 / 15,196,500 x 100,000 30821
Rebate on approved pension fund
4,683,775 / 15,196,500 x 500,000 154,107 184,928
4,498,847
Less: Tax deduction
from salary 1,500,000
from fee for attending BOD meetings 40,000 1,540,000
Tax payable with return on income 2,958,847

Notes:
(1) Medical allowance is fully taxable as it is in addition to medical facility in accordance with terms of
employment. Reimbursement of hospitalization cost from insurance company is exempt assuming that
NTN of medical practitioner and employer's attestation are available. Medical insurance premium paid
by the employer to the insurance company is not a part of salary income.
(2) Fee for literary work may be divided in 3 years for tax purposes if the work is done in more than 24
months
(3) Tax gain on disposal of studio:

Sale proceed 10,000,000


Cost equal to sale proceed 10,000,000
Less: Tax depreciation (1.4 - 0.6 million) (800,000) 9,200,000
Tax gain 800,000

(4) Salary can be paid in cash where it is up to Rs. 25,000 per month. Likewise, utility bills can be paid in cash
irrespective of any limit. Therefore, the said payments are allowable tax expenses.
(5) Winning a car in a competition for sales promotion by a company is taxable under FTR and the tax shall
be collected @ 20% of fair market value of the car which is fill and final tax liability on such income.

212 | P a g e
Examiner Comments:
This was a practical question and required the candidates to compute the taxable income
of an individual and tax thereon for the tax year 2014. Generally the candidates were
aware of the tax treatment of various receipts, expenses and other income. Some of the
mistakes that were more commonly observed were as follows:
▪ Many candidates were unaware that exemption of medical allowance up to 10% of the
basic salary does not apply where medical benefit is provided in addition to the medical
allowance. Consequently, medical allowance was incorrectly considered as exempt.
▪ Fair market value of accommodation amounting to Rs. 900,000 (Rs. 75,000 x 12) was
included in taxable income which was incorrect because rent free accommodation is
taxed at higher of fair market value or 45% of annual basic salary. Hence, Rs. 2,592,000
(being 45% of annual basic salary of Rs. 5,760,000) should have been included in
taxable income.
▪ Leave encashment was treated as exempt.
▪ Fee received for attending the BOD meetings was treated as 'Income from Other
Sources'. Since under section 12(a) directorship is covered under the definition of
employment, such income is to be included in salary income.
▪ A number of candidates did not appreciate that salary expense exceeding Rs 18,000
paid in cash was inadmissible and salary expense upto Rs. 15,000 and utility bills paid
in cash were admissible business expenses.
▪ Number of candidates simply ignored the loan received in cash from the father of the
individual, whereas under Section 39(3) it was taxable under the head Income from
Other Sources.
▪ Fair market value of car won in competition was considered as being subject to tax
under the normal tax regime, whereas it is to be taxed under the Final Tax Regime.
▪ Many candidates did not claim tax credit on 'Donation' and 'Contribution to Approved
Pension Fund'. Some calculated it incorrectly whereas there were a number of students
who considered these as direct deductions from taxable income.
▪ Tax credit on contribution to approved pension fund was calculated after deducting
tax credit on donation from taxable income, whereas both tax credit are calculated
simultaneously i.e. on the basis of average rate of tax.
▪ Some candidates allowed tax credit on insurance premium paid in respect of medical
insurance policy.
▪ Amount of gain on sale of studio was calculated by deducting tax written down value
from sale proceeds. Whereas according to Section 22(13)(d) where the consideration
received on the disposal of immovable property exceeds the cost of the property, the
consideration received has to be treated as the cost of the property. Hence, the amount
of accumulated depreciation becomes the gain.
Further, many candidates treated it as capital gain or income from other sources
whereas it should have been included in income from business.
▪ Income as singer was included under the head income from other sources instead of
income from business.

Question # 13 Autumn 2015 Q. 1

Mr. Mukarram
Tax Year 2023
Computation of Taxable Income
Rs. Rs.
Salary

213 | P a g e
Basic salary Rs. 250,000 x 12 3,000,000
Medical allowance Rs. 37,500 x 12 450,000
Less: Exempt 10% of basic salary (300,000) 150,000
Housing allowance Rs. 25,000 x 12 300,000
Travel allowance Rs. 11,500 x 12 138,000
Company maintained car 5% of the Rs. 1,000,000 50,000
FMV of the car transferred by the employer 400,000
Dinner coupons 2 x 12 x Rs. 2,000 48,000
Reimbursement of telephone and internet Rs. 20,000 x 80% 16,000
Depreciation of the appliances provided by the employer 10% of Rs. 300,000
30,000
Employee share scheme: taxable in the next year --
Taxable salary 4,132,000
Capital Gain
Insurance claim of painting as consideration received 275,000
Purchase cost of painting (100,000)
Insurance premium: incidental exp for insurance claim (15,000)
Capital gain on disposal of painting 160,000

Income from Business


Sale of rice 850,000
Purchase cost 5,000 kgs @ Rs. 62.5 per kg (312,500)
Other expenses (400,000) 137,500
Exempt Income (750,000 + 325,000) 1,075,000
Total Income 5,504,500
Less: Exempt Income (1,075,000)
Taxable Income 4,429,500

Notes:
(1) Company maintained car: 5% of the cost of car to the employer is a taxable perquisite irrespective of its
FMV in the current year.
(2) FMV of any property including car is taxable when the property is transferred to the employee as reduced
by any payment made by the employee in this respect. Therefore, FMV of the car (and not book value)
is taxable for Mukarram.
(3) Sale of car: Car is not a capital asset and therefore any gain or loss on disposal of a personal asset, which
does not fall within the ambit of capital gain, is not required to be included in taxable income.
(4) Bonus received on 5.7.20X5 is not taxable in the tax year 20x5 as the salary is taxable on receipt basis.
(5) Capital loss is not recognized on disposal of specified capital assets including jewelry and therefore loss
on disposal of diamond ring is not recognized as capital loss and is not deducted from capital gain on
disposal of painting.
(6) Rent of agriculture land used for agriculture purpose and building in the immediate vicinity of agriculture
land used as a store house for agriculture produce is included in the definition of agriculture income and
therefore exempt under the Income Tax Ordinance 2001.
(7) Agricultural produce used in-house for business purpose: FMV of the agricultural produce used by the
cultivator in-house for his business shall be considered as purchase cost.

Examiner Comments:
This practical question carrying 20 marks required the students to compute the taxable
income of a salaried individual also having business income, capital gains and agricultural
income (rent). Most of the students performed well and secured good marks. However,
following mistakes were commonly observed in many scripts:

214 | P a g e
▪ Considerable number of candidates were unaware that medical allowance is exempt
to the extent of 10% of the basic salary and treated the entire medical allowance as
taxable. This was a major shift because in all previous attempts this aspect was well
understood by almost all the students.
▪ Amount equal to 5% of cost of the vehicle was to be added to taxable income on account
of company maintained vehicle. This was ignored by many students. Further, many
students added 5% of Fair Market Value instead of cost.
▪ Transfer of car to Mr. Mukarram free of cost, was not treated as taxable perquisite,
whereas, some candidates treated book value of the car transferred as taxable instead
of the market value.
▪ Bonus which is treated as part of salary is taxed on receipt basis. However, many
students included the amount of unpaid bonus in the computation of taxable income.
▪ The free buffet lunch coupons received from the employer were also not included in
taxable income. Reimbursement by the employer of expenses incurred by employee are
taxable as perquisites unless these are incurred in the performance of official duty.
▪ Depreciation on assets given by the employer for use at home should have been
included in taxable income. Most of the students ignored it whereas some of them
included the entire amount in the taxable income.
▪ Many students added share option in taxable income for the year. Mere availability of
the option is not taxable. It is taxed when it is exercised.
▪ Income arising from rice husking unit was considered as agricultural income and
treated as exempt whereas it is a normal business income. Also, some students failed to
claim deduction for market value of agricultural produce used as raw material. On the
other hand many candidates treated rent from agricultural land and building as
taxable, whereas these are considered as agricultural income and are exempt under
section 41 of the ITO 2001.
▪ The requirement was only to compute the taxable income. Many candidates
unnecessarily computed tax liability also.
▪ Many students wrongly adjusted the capital loss arising on the sale of diamond ring
against taxable income. As per section 38(5), loss on sale of jewelry is not available for
adjustment.
▪ Loss on sale of personal car was set-off against other taxable income. Such loss is not
recognizable as movable personal assets do not fall under the definition of ‘capital
asset’.
▪ Some candidates were of the view that capital gain does not arise on disposal of
painting, being a capital asset specified in section 38(5) of the Income Tax Ordinance,
2001. They failed to appreciate that such provision is applicable only in case of ‘loss’
arising on disposal.
▪ A number of candidates did not realize that since painting was retained for more than
one year, the taxable gain should be reduced by 25%.
▪ Some candidates wrongly classified the gain on sale of painting under the head ‘Income
from Other Sources’ instead of ‘Capital Gains’.
▪ 80% of the reimbursed amount of internet and telephone charges should have been
treated as taxable as Mr. Mukarram had spent 20% of the amount for official duties.
Many candidates ignored it altogether whereas some of the students treated the entire
amount as taxable.

Marking Scheme:
▪ Computation of income from salary 9.0
▪ Computation of gain on disposal of painting 2.0
▪ Computation of income from business 3.0
▪ 01 mark for showing each correct exemption/inadmissible deduction 6.0

215 | P a g e
Question # 14 Spring 2016 Q. 1

Mr. Wajahat
Tax Year 2023
Computation of Taxable Income and Tax Liability

Salary Rupees Rupees Rupees


Basic Salary 70,000 x 12 840,000
Dearness allowance (DA) 10,000 x 12 120,000
Conveyance allowance 8,000 x 12 96,000
Recognized Provident Fund (PF)
Company's contribution 100,800
Less: Rs. 150,000 or 10% of basic salary + DA
Whichever is lower 96,000 4,800

Interest credited to RPF @ 20% 391,000


Less: Interest @ 16% 312,800
1/3rd of basic + DA 320,000
Whichever is Higher 320,000 71,000
Reimbursement of electricity bills 60,000
Taxable 1,191,800

Income from Business


Tuition fee 2,198,000
Allowable deduction:
Salary to Mr. Y 35,000 x 10 (350,000)
Travelling expenses (300,000)
Normal tax depreciation on computers 250,000 x 30% (75,000)
Other expenses (195,000)
Total Business Income 1,278,000

IFOS (FS dividend) 65,000


Divident (FTR) 92,000

Total Income 2,626,800


Less:
Dividend (FTR) 92,000
Total Income under NTR 2,534,800

Less: Zakat deducted 4,600


Taxable income under NTR 2,530,200

Tax liability (Non-Salaried Case)


Income tax on Rs. 2,400,000 315,000
Income tax on Rs. 130,200 @ 25% 32,550
347,550
Less: Rebate on contribution to APF 347,550/ 2,530,200 x
506,040
69,510
278,040
Add: Tax on gross dividend from Bee Ltd 92,000 x 15% 13,800
291,840

216 | P a g e
Less: Tax deducted from dividend by Bee Ltd 9,200
9,200
Tax payable with return of income 282,640

Notes:
(1) Dividend received from foreign company:
Dividend received from any foreign company is taxable is Pakistan
(2) Monthly salary paid to himself: Salary paid to himself is drawing and not salary expenses for the business
entity.
(3) Initial allowance is not allowable on used computer
(4) Contribution to APF: Contribution to APF is eligible for rebate at average rate of tax on the lower of the
following:
- Actual contribution; and
- 20% of current year's taxable income.
The amount eligible for rebate is 20% of taxable income of the current year i.e. Rs. 506,040 being the
lower amount.

Examiner Comments:
This practical question carrying 16 marks required the students to compute the total
income, taxable income and net tax payable in case of a salaried individual. Most of the
students performed well and secured good marks. Some students made unnecessary and
illogical assumptions which changed the context of the question altogether. Such
assumptions are not acceptable. Some of the common mistakes are discussed below:
▪ Dearness and Conveyance allowances were considered to be exempt.
▪ Interest on provident fund @ 16% or 1/3rd of Basic Salary plus Dearness Allowance
whichever is lower was considered as exempt whereas the higher of the two is actually
exempt.
▪ Initial allowance was claimed on computers. Many candidates did not appreciate the
fact that initial allowance is only admissible on such depreciable assets which have not
been previously used in Pakistan. Moreover, in case of additions during the year,
normal tax depreciation was computed for proportionate period in which the asset
remained in use, instead of the full year.
▪ Zakat was deducted from tax liability instead of deducting it while computing taxable
income. Some candidates were of the view that since Zakat has already been deducted
from dividend, it cannot be deducted from normal income.
▪ Salary received by Mr. Wajahat from his own business, i.e. the tuition center was
treated as salary for income tax purposes also and the amount was claimed as a
business expense.
▪ Tax on dividend was computed @ 10% instead of 15%. Moreover, a significant number
of candidates did not gross up the net amount of dividend by adding withholding tax
and zakat deducted at source, whereas, many students considered dividend received
from a company in UAE as being exempt from tax. Furthermore, many students did not
treat dividend income as a separate block of income under the final tax regime and
considered it as taxable under the normal tax regime.
▪ Tuition fee received was treated as income from other sources instead of income from
business.
▪ Withholding tax deducted from the dividend received from BEE Limited was treated as
full and final discharge of tax liability.
▪ Tax credit on investment in pension fund should have been calculated at average rate
applied on the lower of (i) actual contribution, (ii) 20% of taxable and (iii) 50% of last
tax assessed income. Most of the students ignored the third point. (ii). Further, many
students included dividend income in determining the average rate of tax.

217 | P a g e
Marking Scheme:
▪ Computation of income from salary 5.5
▪ Computation of income from business 3.0
▪ Computation of dividend income (separate block) 2.0
▪ Computation of taxable income 1.0
▪ Determination of net tax payable 4.5

Question # 15 Autumn 2016 Q. 1

Mr. Badar
Tax Year 2023
Computation of Taxable Income and Tax Liability
Rs.
Salary from HFL
Basic Salary 3,000,000
Medical allowance 28,000 x 12 336,000
Less: exempt up to 10% of basic 300,000 36,000
House rent allowance 1,440,000
Bond signing amount 900,000
Company maintained car Rs. 1,500,000 x 5% x 11/12 68,750
Inventory item Rs. 22,000 - 12,000 10,000
Employee share scheme
- Fair market value 2,500 shares x Rs. 375 937,500
Cost paid Rs. 300,000 + 200,000 500,000 437,500
Taxable salary from HFL 5,892,250

Salary from Sultan Hospital


Termination benefits 600,000
Gratuity from unapproved scheme 485,000
Less: Exempt up to lower of Rs. 75,000 or 50% of gratuity amount 75,000 410,000
Medical treatment exempt under clause 53A Part I 2nd Schedule --
Taxable Salary 6,902,250

Capital Gain: section 37


Gross consideration 875,000 + 10,000 + 5,000 890,000
Cost paid 500,000 / 2,500 x 2,000 (400,000)
Amount taxed under head of salary 437,500 / 2,500 x 2,000 (350,000)
Bank charges and commission being incidental expenses (15,000) 125,000
7,027,130
Less: Terminal benefits taxable at last 3 years' average tax rate (600,000)

Taxable Income 6,427,130

Tax liability (Salaried Case)


Tax on Rs. 6,000,00 1,095,000
Tax on Rs. 427,250 @ 35% 149,537
1,224,537

Total tax liability 1,224,537


Tax on terminal benefit (1,260,000 / 10,500,000) x 600,000 72,000

218 | P a g e
1,172,537
Less: Tax deducted from salary (1,105,000)
Tax payable 67,537

Notes:
(a) Bond signing amount is profit in lieu of salary and therefore taxable
(b) Termination benefits are taxable at the normal slab rates of current tax year or as separate block of
income at the last 3 years' average tax rate whichever is beneficial for the employee.

Examiner Comments:
This practical question carrying 15 marks required the students to compute the taxable
income and net tax payable or refundable in case of a salaried individual. Most of the
students performed fairly and secured reasonable marks. However, the following errors
were observed in many scripts:
▪ Medical allowance up to 10% of the basic salary is exempt. Majority of the candidates
ignored this rule and treated the entire amount of medical allowance as taxable.
▪ Candidates appreciated that 5 per cent of the FMV of the perquisite representing car
for both official and private use is to be included in the taxable income. However, the
amount to be included in the taxable salary was to be restricted with reference to
number of 'days' it was used in the tax year. Almost all candidates wrongly restricted
the same with reference to the number of 'months', rather than 'days'.
▪ Free medical treatment was provided by the hospital (employer) to the employee by
virtue of his employment; the same was exempt from tax. Many candidates did not
appreciate this fact.
▪ Amount chargeable under the head salary on acquisition of shares in MP was not
calculated correctly. Rs. 200,000 paid to acquire the option and Rs. 300,000 paid to
acquire the shares was not deducted from market value of shares. Market value of
shares was not calculated by multiplying market value of each share of Rs. 375 with
number of 2,500 shares acquired.
▪ Amount taxable on account of unapproved gratuity was not considered as a separate
block of income, to be taxed on the average rate of tax for the three preceding tax years.
▪ In respect of tax credit on investment in shares and life insurance premium, very few
candidates knew about the following:
- for determining the average rate of tax, income and tax liability in respect of
separate block of income were to be excluded; and
- for investment in shares, tax credit was to be calculated only on fully subscribed
right shares and not on the entire shareholding.

Marking Scheme:
▪ Computation of income from salary:
- Termination benefits 2.0
- Share options scheme 2.0
- Other computations of income from salary 3.0
▪ Computation of income from capital gain 3.0
▪ Computation of tax credit 2.0
▪ Determination of net tax payable 3.0

Question # 16 Autumn 2017 Q. 1

Mr. TA
Tax Year 2023

219 | P a g e
Computation of Taxable Income and Tax Liability
Rs.
Salary
Basic salary 400,000 + (440,000 x 11) 5,240,000
Conveyance allowance 40,000 + (44,000 x 11) 524,000
Medical allowance 40,000 + (44,000 x 11 524,000
Travel and daily allowance to meet office expenses 400,000
Deemed income on interest on loan (Working Note) 120,000
Performance bonus received on 15.8.20X7 taxable in the year 20X8 1,200,000 --
Waiver of outstanding loan 2,500,000 - (22 x 50,000) 1,400,000
Fee for attending BOD meetings 100,000
8,308,000
Income from Property
Actual rent or fair market rent whichever is higher 1,800,000
Less: Amount of amenities and utilities 300,000
1,500,000
Less: Repair allowance 1/5th of chargeable rent 300,000 1,200,000

Capital Gain under section 37A


Consideration received 15,000 x 85 1,275,000
Cost of shares 15,000 x 60 900,000 375,000

Income from other sources


Dividend Income (150,000/0.7) FTR 214,286
Amenities and utilities from tenant 25,000 x 12 300,000
514,286

Total Income 10,397,286

Less: Dividend Income 214,286


Capital gain under section 37A 375,000

Taxable Income 9,808,000

Tax liability (Salaries Case)


Income tax on Rs. 6,000,000 1,095,000
Income tax on Rs. 3,808,000 @ 35% 1,332,800
Income tax on capital gain 12.5% of Rs. 375,000 46,875
2,474,675

Tax under FTR on dividend Rs. 214,286 x 15% 32,143


2,506,818
Less: Tax deducted from salary 2,000,000
Tax deducted from dividend Rs. 214,286 x 30% 64,286 2,064,286
Tax payable with return of income 442,532

Notes:
(a) Medical insurance provided directly by the employee:
Medical facility provided directly by the employer or through insurance company is not taxable if NTN of
the medical practitioner and employer's attestation are available. Insurance premium paid by the
employer is not part of the salary income.
(b) Deemed / Imputed Interest on laon from the employer:

220 | P a g e
National interest income is included in taxable salary @ 10% where the loan exceeding Rs. 1,000,000 is
taken from the employer.
Working of notional interest @ 10% is as under:

Outstanding
Loan Interest
July 20X6 2.5m - (14 x 50,000) 1,800,000 15,000
August 20X6 1,750,000 14,583
September 20X6 1,700,000 14,167
October 20X6 1,650,000 13,750
November 20X6 1,600,000 13,333
December 20X6 1,550,000 12,917
January 20X6 1,500,000 12,500
Febraury 20X6 1,450,000 12,083
March 20X6 1,400,000 11,667
120,000

(c) Disposal of Shares of listed company:


Gain on disposal of a listed company is taxable under section 37A as separate block of income.
(d) Bonus shares:
Face value or market value of bonus shares os not taxable at the time of receipt of bonus shares
(e) Dividend income:
If a shareholder is not included in active taxpayers' list then the company is requred to deduct tax @ 30%
of gross dividend whereas dividend income is taxable under FTR @ 15%.

Examiner Comments:
This 16-mark question required the candidates to compute the total income, the taxable
income and net tax payable by or refundable, in case of a salaried individual including tax
payable relating to separate block income and final tax regime. The overall performance
was average as 37% candidates secured passing marks. The common mistakes were as
under:
▪ Many candidates only calculated taxable income and did not calculate total income.
▪ Total income was not computed head wise.
▪ Performance bonus of Rs. 1,200,000 was added in salary income on earned basis
whereas it is taxed on receipt basis.
▪ Bonus shares received were taxed under normal tax regime.
▪ Daily allowance of Rs. 400,000 was received to meet expenses of working assignment,
but was included in taxable income.
▪ Imputable interest on interest free loan was computed on Rs. 2,500,000 i.e. the entire
amount of loan instead of the balance outstanding. Moreover, many students
calculated it for twelve months despite that the balance loan was waived off on 30
March 20X7. Considerable number of candidates ignored it altogether.
▪ Rs. 100,000 received for attending board meeting was treated as separate block
income and taxed at the rate of 20%.
▪ Cash dividend received from AL after deduction of withholding tax was not grossed up.
▪ Capital gain on disposal of shares was taxed under normal tax regime whereas some
candidates claimed 25% of capital gain as exempt and 75% taxable under normal tax
regime.
▪ Income from property was calculated on the basis of actual rent instead of using
annual rent prevalent in the vicinity for similar house. Moreover, amount pertaining to
amenities and utilities was also included in income from property whereas it should
have been included in income from other sources. Furthermore, many candidates

221 | P a g e
treated it as normal income instead of considering it as a separate block of income.
▪ Some candidates used incorrect tax rates probably because they didn't use the tax rate
table carefully.

Marking Scheme:
▪ Computation of income from:
- Salary 5.5
- Property 0.5
- Other Sources 1.5
- Capital Gain 1.0
▪ Determination of total income and taxable income under NTR 2.0
▪ Tax liability under NTR 1.0
▪ Tax liability under separate block 2.5
▪ Adjustment of withholding tax and tax credit 2.0

Question # 17 Autumn 2018 Q. 1

Mr. Ahmer Ghazi


Tax Year 2023
Computation of Taxable Income and Tax Liability
Rs.
Salary 7,800,000
Basic salary 650,000 x 12 1,140,000
House rent allowance 95,000 x 12 840,000
Health insurance --
Award for meeting sales target 180,000
Concessional loan benefits Rs. 5 million x 4% for 11 months 183,333
Tax paid by the employer 1,500,000
Taxable salary 11,643,333

INCOME FROM PROPERTY


Chargeable rent 320,000
Less: Repair allowance 1/5th of chargeable rent (64,000) 256,000

CAPITAL GAIN
Sale proceeds 6,000 shares x Rs. 33 198,000
Less: Cost of shares 6,000 x Rs. 23 (138,000) 60,000

INCOME FROM OTHER SOURCES


Amount received for song being royalty income 225,000

Taxable Income 12,184,333

Tax liability (Salaried Case)


Income tax on Rs. 6,000,000 1,095,000
Income tax on Rs. 6,184,333 @ 35% 2,164,516
3,259,517
Less: Tax paid by the employer 1,500,000
Tax payable with return of income 1,759,517

222 | P a g e
Examiner Comments:
This practical question carrying 16-mark required the candidates to compute total income,
taxable income and net tax payable by or refundable to a salaried individual also having
income from property, capital gain and income from other sources. The overall
performance was very encouraging with excellent presentation, structured layout and 70%
of the students secured passing marks. The common mistakes were as follows:
▪ Many candidates did not pay attention to the requirement of calculating the income
under correct head of income.
▪ Medical allowance in excess of 10% of basic salary was considered as taxable; whereas,
since health insurance was being provided, the entire medical allowance should have
been treated as taxable. In some cases, health insurance premium of Rs. 50,000 was
included in total income.
▪ While computing the gain on disposal of shares, the market value on the date of
allotment was considered as the cost; instead of the market value on the date when
restriction to sell was removed. Moreover, in many cases, the gain was included in
salary income. These candidates did not appreciate that salary income is recorded in
the year in which the restriction on transfer of shares is removed, at the fair value of
the shares on that date and that is why when the shares are actually sold, the amount
already included in salary is treated as cost, for the purpose of calculation of capital
gain.
▪ Income as singer amounting to Rs. 225,000 was treated as business income instead of
income from other sources. Some students treated it as taxable under final tax regime.
▪ Majority of the candidates calculated tax credit incorrectly using total income as the
denominator instead of taxable income.
▪ Tax rates for non-salary tax payer were applied.
▪ Mark up on concessional loan of Rs. 5 million was ignored.

Marking Scheme:
(a) Computation of income from:
▪ salary 5.5
▪ property 1.0
▪ capital gain 2.5
▪ other sources 1.0
(b) Deduction of separate block of income from total income 1.0
(c)
▪ Computation of:
− tax liability under NTR 1.0
− tax liability under separate block 1.5
− tax credit for investment in Sukuks 2.0
▪ Adjustment of withholding tax 0.5

Question # 18 Autumn 2019 Q. 1

Mr. Saeed
Tax Year 20X9
Computation of Taxable Income and Tax Liability
Rs.
Salary from DSL:
Foreign sourse salary income exempt being returning expatriate --
Salary from HPL
Basic salary 600,000 x 9 5,400,000

223 | P a g e
Medical allowance 66,000 x 9 595,000
Less: Exempt up to 10% of basic salary (540,000) 54,000
Bonus: not received in the tax year 20X9 --
Company maintained car for personal use 1,900,000 x 10% x 9/12 142,500
Free lunch provided through an arrangement by employer 125,000
--
Special allowance to meet office expenditure Exempt 180,000 --
Contribution to PF (assumed to be recognized)
Actual contribution by HPL 60,000 x 9 540,000
Less: 10% of basic salary or Rs. 150,000
whichever is lower 150,000 390,000
Taxable Salary 6,111,500

Received from DSL


Salary received (USD 15,000 x 3 = 45,000 @ 170 Rs)

INCOME FROM OTHER SOURCES


Gross dividend taxable under FTR (575,000 + 62,500) / 85% 750,000 750,000

Total income 6,861,500


Less: FTR Income (750,000)
Total Income Under NTR 6,111,500

Less: Zakat deducted under Zakat Ordinance 62,500


Taxable Income 6,049,000

Tax liability (Salaried Case)


Income tax on Rs. 6,000,000 1,095,000
Income tax on Rs. 49000 @ 35% 17,150
1,112,150
Income tax on gross dividend @ 15% of Rs. 750,000 112,500
1,224,650
Less: Tax deducted by HPL 1,300,000
Tax deducted from dividend 112500 1,412,500
Tax refundable (187,850)

Examiner Comments:
▪ Incomes were not classified under appropriate heads.
▪ Salary received by Saeed from DSL and car facility provided to him for personal use
were offered to tax on the basis of 12 months instead of 9 months of actual receipt /
use.
▪ Examinees also failed to acknowledge that exempt salary does not form part of salary
income.
▪ Car provided wholly for official purposes was wrongly offered to tax at the rate of 5%
of the value of car.
▪ Bonus received after the year-end was included in taxable income.
▪ Free food provided in lunch was treated as exempt.
▪ Many examinees, without appreciating the rule for the computation of deductible
allowance, considered the entire amount of mark-up on loan obtained from a
scheduled bank, as direct deduction.
▪ Exemption on employer’s contribution to provident fund was wrongly considered at Rs.
75,000 instead of Rs. 150,000.

224 | P a g e
▪ Withholding tax deducted at source on dividend was wrongly deducted from total
income.
▪ Majority of the examinees failed to compute the correct amount of gross dividend.
▪ Some of the examinees wrongly adjusted the tax withheld u/s 149 from total salary.

Marking Scheme:
▪ Computation of:
− income from salary 7.0
− exempt income 1.0
− FTR income (separate block) 1.5
▪ Deduction of separate block and exempt income 2.0
▪ Computation of deductible allowance and its treatment:
− Zakat 1.0
− Mark-up paid to the bank 2.0
▪ Computation of tax liability 1.5
▪ Adjustment of withholding tax 1.0

Question # 19 Autumn 2020 Q. 1

Mr. Sageer
Tax Year 2023
Computation of Taxable Income and Tax Liability
Rs.
SALARY
Basic salary 200,000 x 12 2,400,000
Medical allowance 20,000 x 12 240,000
[Medical allowance is fully taxable if it is in addition to medical facility in
accordance with terms of employment

Rent free accommodation


Fair market rent of accommodation 100,000 x 12 1,200,000
45% of basic salary 1,080,000
Whichever is higher is taxable 1,200,000

Health insurance premium 40,000 --


Health insurance premium paid by the employer is not income of the
employee. Amount received by the employee from insurance company is
income but the same is exempt subject to the specified conditions

Contribution to recognized provident fund (PF) 15,000 x 12 180,000


Less: Exempt up to Rs. 150,000 or 10% of [basic salary + dearness
allowance], whichever is lower (150,000) 30,000

Employee Share Scheme


FMV on 31st December 10,000 shares x Rs. 142 1,420,000
Cost of shares paid 10,000 shares x Rs. 105 (1,050,000)
Cost of option paid (175,000) 195,000

[FMV of option is irrelevant]

Benefit of interest free loan Exempt

225 | P a g e
[Benefit of interest free loan is exempt in the following cases:
Where the amount of loan does not exceed Rs. 1 million; or

Where such benefit is extended by the employer due to the waiver of


interest by such employee on his account (e.g. provident fund etc,)
maintained with the employer.]

Leave encashment 100,000

[Salary is taxable on receipt basis]

Taxable salary 4,165,000

CAPITAL GAIN under section 37A


Sale proceed of 5,000 shares of ZL @ Rs. 135 675,000
Cost of shares paid 5,000 shares x Rs. 105 (525,000)
Cost of option paid 175,000 / 10,000 x 5,000 (87,500)
Amount taxed under the head salary 195,000 / 10,000 x 5,000 (97,500)
Capital loss c/f (35,000) --

[Capital loss under section 37A cam only be adjusted against capital gain
under section 37A. Alternatively, it will be c/f for 3 subsequent tax year.]

CAPITAL GAIN under section 37A


Sale of shop
Sale proceed 15,000,000
(14,000,000
Cost Rs. 19 million - Rs. 5 million paid in cash )
Capital Gain 1,000,000

Sale of residential plot


Sale proceed 18,000,000
FMV is deemed cost at the time of inheritance (7,000,000)
11,000,00
Capital gain 0
12,000,00
0

INCOME FORM BUSINESS


Gross receipt from courses outside Pakistan ($ 4,260 x Rs. 168) / 92% 777,913
16,942,91
Total income 3
12,000,00
Less: Capital gain on Property 0
Taxable income 4,942,913

Tax liability (Salaried Case)


Income tax on Rs. 3,600,000 435,000
Income tax on Rs. 1,342,913 @ 27.5% 369,301
804,301
Less: Full time teacher allowance as per working 136,563

226 | P a g e
667,738
Less: Foreign tax credit
Tax paid in the foreign country Rs. 777,913 x 8% 62,233
Tax in Pakistan at average rate of tax
667,738 / 4,942,913 x 777,913 105,088
whichever is lower 62,233
605,505
Less: Rebate on donation
Amount eligible for rebate: Actual donation Rs. 2.5 million or 30% of
taxable income, whichever is lower 1,482,874

Rebate on donation 605,505/ 4,942,913 x 1,482,874 181,652


423,853
Tax on disposal of Shop (1,000,000 x 10%) 100,000
Tax on Plot (11,000,000 x 2.5%) 275,000
798,853
Less: Tax withheld from salary 160,000
Tax payable with return of income 638,853

Working: full time teacher allowance


Salary income 4,165,000

Income tax on Rs. 3,600,000 435,000


Income tax on Rs. 565,000 @ 27.5% 155,375
590,375

Full time teacher allowance @ 25% of Rs. 590375 147,594

Examiner Comments:
▪ Medical allowance was treated as ‘exempt’ despite the fact that it was given in addition
to free hospitalization.
▪ In some cases, the benefit under share option scheme was not calculated as income.
▪ While computing the benefit under employee share option scheme, Rs.142 was taken
as fair market value instead of Rs.130.
▪ Examinees failed to identify that capital loss on sale of shares can only be adjusted from
capital gain from securities.
▪ Tax on capital gain from sale of immoveable property was computed separately for
each property instead of computing it as the total capital gain from immovable
property.
▪ The deductible allowance on account of donation paid to non-profit organization
covered under 2nd Schedule of the Income Tax Ordinance, 2001 was calculated and
deducted from tax liability instead of from taxable income. Further, examinees ignored
the limit of 30% of taxable income applicable on such donations.
▪ In computation of foreign tax credit, the tax payable in Pakistan was computed without
considering income under all the heads. Some examinees directly deducted the paid
foreign tax from tax liability.
▪ Full time teacher allowance was computed using the basic salary instead of total salary
income. Some examinees computed the allowance by applying the rate of 25% to tax
payable.
▪ Cost of the inherited plot was taken zero instead of taking the fair market value at the
time of inheritance as its cost.
▪ The deductible allowance pertaining to donation was deducted before deduction of

227 | P a g e
separate block of income.

Marking Scheme:
▪ Computation of total income from:
− salary 7.0
− business 1.0
− capital gain from sale of properties 2.0
− capital gain from sale of securities 2.5
▪ Deduction of separate block and exempt income 0.5
▪ Computation of deductible allowance and its treatment:
− Donation to non-profit organization 1.5
− Computation of tax liability under NTR 1.5
− Computation of tax credit 1.5
− Computation of foreign tax credit 1.0
− Adjustment of withholding tax 0.5

Question # 20 Autumn 2021 Q. 1

Mr. Nauman
Computation of total income, taxable income and net tax payable/refundable
For tax year 2023
Rupees
Income from salary
Basic salary [120,000×12] 1,440,000
Medical allowance [240,000(20,000×12) –144,000(1,440,000×10%)] 96,000
House rent allowance (60,000×12) 720,000
Company maintained car for both official and personal use (1,400,000×5%) 70,000
Purchase of car on book value (1,000,000 – 450,000) 550,000
Employer’s contribution to provident fund
[18,000×12=216,000–144,000(1,440,000×10%) (Allowed limit is 1/10 of the basic
72,000
salary OR 150,000 whichever is lower)
Interest on provident fund [540,000–480,000{higher of: interest @ 16% i.e 480,000
60,000
(540,000÷18%)×16 OR 480,000(1/3rd of basic salary i.e. (1,440,000÷3)}]
Relocation allowance -
Bonus – [not taxable in TY2023 as it is received in July 2023) -
Loan obtained on concession rate [5,000,000×4%(10%-6%)×(3÷12) 50,000
Legal expenses – Not deductible being no deduction shall be allowed for expenses
-
incurred in earning salary income
Total income from salary 3,058,000
Income from property

Rent income 800,000(1,200,000‒400,000)×9/12 600,000


Less:Repair Allowance (600,000÷5) (120,000)
Insurance premium (50,000)
Administration and collection charges to the extent of 4% of chargeable rent
(24,000)
600,000×4%
406,000
Income from other sources
Interest income (510,000÷0.85)+200,000 800,000
Income from utility, cleaning and security 400,000
Less: Expenditure (250,000)

228 | P a g e
150,000
950,000

Total income from all sources 4,414,000


Less: Separate block of income (interest income) (800,000)
3,614,000
Less: Deductible allowances
Zakat (200,000)
Taxable income under NTR 3,414,000

Examiner Comments:
▪ Legal expenses for obtaining the loan were not shown as inadmissible expenses.
▪ Interest income was grossed up incorrectly.
▪ Deductible allowance for profit on debt related to purchase of new house was ignored.
▪ Relocation allowance was considered as taxable.
▪ Rent amount was not proportionated for nine months.
▪ Income from utilities, cleaning and security was taxed under the head ‘income from
property’.
▪ The entire amount of administration and collection charges of rent was taken as an
admissible expense instead of restricting it to the extent of 4% of chargeable rent.
▪ Time was wasted on computation of tax liability which was not required in the
question.

Marking Scheme:
▪ Income from salary 7.0
▪ Income from property 2.0
▪ Income from other sources 2.0
▪ Deductible allowances 2.0

Question # 21 Spring 2022 Q. 1

Mr. Basit
Computation of total income, taxable income and net tax payable/refundable
For tax year 2023
Salary Rupees
Pakistan source income:
Salary [610,000×7] 4,270,000
Allowance for services of domestic servant [60,000×7] 420,000
Allowance @ 5% of salary solely expended in the performance of his duties
213,500
of employment (4,270,000×5%)
Acquired car on lease -
Shares acquired under employee share scheme [1,170,000(13,000×90)
780,000
390,000(13,000×30)]
Leave encashment 320,000
Gratuity (2,200,000–300,000) 1,900,000
Salary arrears of tax year 2022 700,000
Foreign source income:
Salary (3,200×250×3) 2,400,000
Total income from salary 11,003,500
Capital gain

229 | P a g e
Loss on sale of ML’s shares [400,000(5,000×80)–450,000(5,000×90)] (50,000)
Income from other sources
Pakistan source income:
Gift received 200,000
Foreign source income:
Income earned from university [1,000(1,500–500)×250] 250,000
450,000
Total income 11,403,500
Less: Foreign source salary – Exempt (2,400,000)
Add: Capital loss (Separate block of income) 50,000
Taxable income 9,053,500
Tax liability
On Rs. 6,000,000 1,095,000
On remaining Rs. 3,053,500 @ 35% 1,068,725
2,163,725
Less: Foreign tax credit [250,000×22.06% (1,997,388/9,053,500×100) = 55,155] OR
(55,155)
[225×250 = 56,250] whichever is lower.
2,108,570
Less: Withholding tax (1,400,000)
Tax refundable 708,570

(b) Arears amount may be taxed at the rates of tax year 2022 that would have been applicable if the salary
had been paid to the Basit in tax year 2022.
(c) Basit is required to furnish a foreign income and assets statement giving particulars of:
▪ his total foreign assets and liabilities as on 30 June 2023;
▪ any foreign assets transferred by him to any other person during tax year 2023 and the consideration
for the said transfer; and
▪ complete particulars of foreign income, the expenditure derived during the tax year 2023 and the
expenditure wholly and necessarily for the purposes of deriving the said income

Examiner Comments:
(a)
▪ Allowance as fixed percentage of salary was considered as exempt.
▪ Entire gratuity amount was considered as exempt.
▪ Lease rental of car was taken as an allowable expense without appreciating the fact
that no expense is allowed against 'Income from salary'. Similarly, 5% of fair value of
car was considered as subject to tax by some examinees.
▪ Amount in respect of shares acquired under employee share scheme was computed
incorrectly.
▪ Income from foreign university was considered exempt.
▪ Gift was considered as subject to tax under capital gain.
▪ Total income including exempt and separate block of income was not shown as a
separate line.
▪ Foreign tax credit was either ignored or computed incorrectly.

(b) Arrears were considered as subject to tax at the average rate of last three years.

(c)
▪ Wealth statement was incorrectly identified as additional statement.
▪ All particulars were not mentioned.

230 | P a g e
Marking Scheme:
(a)
▪ Income from salary
– Pakistan source 6.0
– Foreign source 2.0
▪ Capital gain 2.0
▪ Income from other sources
– Pakistan source 1.0
– Foreign source 2.0
▪ Tax credits 3.0
(b) Other option available for taxation of salary arrears 1.0
(c)
▪ Identification of additional statement 0.5
▪ 0.5 mark for each particular to be mentioned in the additional statement 1.5

Question # 22 Autumn 2022 Q. 2

Mr. Nasir
Computation of total income, taxable income and net tax payable/refundable
For tax year 2022
Rupees
Salary
Income from salary:
Basic salary (800,000×12) 9,600,000
Medical allowance (100,000×12) 1,200,000
Cost of living allowance (200,000×12) 2,400,000
Housing [360,000{300,000 OR 360,000(45%×800,000) whichever is higher}×12] 4,320,000
Company maintained car (4,000,000×5%) 200,000
Fuel [(250×180) ×12×30%] 162,000
Maintenance of car -
Hospitalization born by insurance company -
Ad-hoc relief allowance 800,000
Expenses incurred for work from home -
18,682,000
Income from other sources
Gift - Cash 1,000,000
- Car -
1,000,000
Total income / Taxable income 19,682,000
Tax liability
On Rs. 6,000,000 1,095,000
On remaining 13,682,000 (19,682,000 – 6,000,000) @ 35% 4,788,700
5,883,700
Less: Tax credit on donations (being donation in cash form) -
Less: Tax credit on contribution to approved pension fund, lower of following:
▪ Total contribution paid by Nasir i.e. Rs. 4,700,000
▪ 20% of taxable income (19,682,000×20%) i.e. Rs. 3,936,400
Tax credit on contribution 5,883,700/19,682,000×3,936,400 (1,176,740)
4,706,960
Less: withholding tax (4,500,000)

231 | P a g e
Tax Payable 206,960

(b) APL will not be considered to have earned any rental income from this property and accordingly there
will be no tax consequences for it.

Question # 23 Q.1 March 2023

Mr. Cheng
Tax Year 2024
Computation of taxable income and tax liability
SALARY from HL:
Basic salary 350,000 x 3 1,050,000
Medical allowance 35,000 x 3 105,000
Less: Exempt up to 10% of basic salary 105,000 --
Utilities allowance 20,000 x 3 60,000
Lunch provided by the employer 30,000 --
Company maintained car 3,000,000 x 5% x 3/12 37,500
Bonus of previous year received in the current year 350,000
Unapproved gratuity scheme 1,225,000
Less: Exempt up to Rs.75,000 or 50% of amount receivable
whichever is lower 75,000 1,150,000
Benefit of interest free loan 3,800,000 x 10% x 3/12 95,000

SALARY from DH
Basic salary 450,000 x 9 4,050,000
Medical allowance 60,000 x 9 540,000
Less: Exempt up to 10% of basic salary 405,000
Utilities allowance 30,000 x 9 135,000
Lunch provided by the employer 270,000
Company maintained car 5,000,000 x 5% x 9/12 225,000 --
Air tickets for trip to China 187,500
Consideration for signing a bond 550,000
Commission 750,000
Repayment of loan by the employer 400,000
3,800,000
Taxable salary 12,885,000

INCOME FROM PROPERTY


Rent of the house 800,000
Repairs allowance @ 1/5th of chargeable rent (160,000)
Interest deemed to have been paid (95,000) 545,000

INCOME FROM BUSINESS


Business income from China 1,260,000/ 70% 1,800,000

INCOME FROM OTHER SOURCES


Bonus shares taxable under FTR 2,000 shares x Rs.25 115,000 --
Dividend taxable under FTR 97,750 / 85% 50,000 --
Taxable income 15,230,000
Tax liability (Salaried Case)
Income tax on Rs.6,000,000 1,095,000

232 | P a g e
Income tax on Rs.9,230,000 @ 35% 3,230,500
4,325,500
Foreign tax credit
Tax paid in the foreign country 1,800,000 x 30% 540,000
Tax in Pakistan on foreign source income at average tax rate
(4,325,500/15,230,000) x 1,800,000 511,221
Whichever is lower 511,221
3,814,279
Rebate on donation (3,814,279 / 15,230,000) x 600,000 150,267
3,664,012
Tax on dividend 115,000 x 15% 17,250
Tax on bonus shares 50,000 x 10% 5,000
Total tax liability 3,686,262
Less: Tax deducted from dividend 17,250
Tax paid on bonus shares 5,000 22,250
Tax payable 3,664,012

Question # 24 Q.1 September 2023

Salman
Computation of total income, taxable income and net tax payable by or refundable For the
tax year 2023
Income from salary: Rupees
Basic salary 2,400,000
House rent allowance 600,000
Annual bonus 480,000
Free meals while on duty - exempt -
Staff discount 40,000
Health insurance – exempt -
Company maintained car (2,000,000 × 5%) 100,000
Employee share scheme [12(110–98) × 12,000] 144,000
PF contribution (2,400,000 × 8%) 192,000
Less: Exempt (150,000 OR 1/10 of basic salary i.e 240,000,
whichever is lower) (150,000) 42,000
Profit on provident fund [211,200(960,000×22%) – Exempt: Higher of interest @ 16%
i.e. 153,600(960,000×16%) OR [800,000(1/3rd of basic salary i.e. (2,400,000÷3)] -
Culinary chef course -
Travelling allowance – not spent on behalf of the employer (250,000 – 100,000) 150,000
3,956,000

Interest income (765,000÷0.85) 900,000


Total income 4,856,000
Less: interest income being FTR (900,000)
Taxable income 3,956,000

Tax liability
on Rs. 3,600,000 435,000
on excess amount (356,000 × 27.5%) 97,900 532,900
Tax on interest income (900,000×15%) 135,000
Less: Tax credit on donations:
Donations amount through cheque (300,000×80%) 240,000

233 | P a g e
30% of taxable income (3,956,000×30%) 1,186,800
Tax credit [(532,900 ÷ 3,956,000) × 240,000, being lower] (32,256)
Tax payable 635,644
Less: Tax withheld by TC (400,000)
Less: Tax withheld by Bank (135,000)
Net Tax Refundable 100,644

Question # 25 [CFAP Questions for Jazbati Students] Q.1 December 2009

Ms. Saima
Tax Year 20X9
Computation of taxable income
SALARY
Basic salary 400,000 x 12 months 4,800,000
Medical allowance 40,000 x 12 480,000
Conveyance allowance 60,000 x 12 720,000
Reimbursement of medical expenses: exempt
Bonus 1,000,000
Tax borne by the employer 100,000
Taxable salary 7,100,000

Income from Property


Forfeited deposit against contract for sale of plot as rent 1,000,000

Income from Business


Service income 445,000/ 89% 500,000
Less: Expenditures related to service income 70,000 430,000

Income from Other sources


Loan received in cash 5,000,000
Taxable Income 13,530,000

Notes:
1. Medical allowance is fully taxable where medical expenses are reimbursed as per policy. However,
reimbursement of medical expenses is exempt assuming that NTN of medical practitioner and
employer's attestation are available.
2. Tax borne by the employer is a taxable perquisite.
3. Service income as a visiting faculty member is taxable under normal tax structure.
4. Prize winning in a quiz competition is taxable under FTR @ 20% and the tax deduction is the full and
final discharge of tax liability.
Therefore, it is not included in normal taxable income.
5. Sale of plot inherited is exempt as the holding period is more than 6 years.
However, deposit forfeited under a contract for sale of immovable property is included in the definition
of rent and therefore taxable under the head income from property after considering admissible
deductions.
However, repairs allowance @ 1/5th of chargeable rent is not admissible against rent of open plot.
6. Loan received in cash is taxable under the head income from other sources.
7. Gift of painting, being a capital asset, to brother is a disposal under the head capital gain.
However, certain transactions including gift are covered within the ambit of non-recognition rule and
therefore no capital gain or loss arises on this transaction.

234 | P a g e
Question # 26 [CFAP Questions for Jazabati Students] Q.1 June 2011

Mr. Khan
Tax Year 20X4
Computation of taxable income and tax liability
SALARY
Basic salary 350,000 x 6 months 2,100,000
Conveyance allowance 50,000 x 6 300,000
Rent free accommodation 45% of basic salary 945,000
Company maintained car 2,000,000 x 5% for 6 months 50,000
Interest free loan 2,500,000 x 10% for 6 months 125,000
Waiver of loan 25% of Rs.2,500,000 625,000
Compensation under redundancy scheme (separate block of income) 4,000,000
Gratuity from unapproved scheme 2,000,000
Less: exempt up to Rs.75,000 or 50% of amount receivable
whichever is lower 75,000 1,925,000
Car purchased from the employer: 500,000
FMV Rs.1,500,000-payment Rs.1,000,000 10,570,000
Less: Compensation being separate block of income 4,000,000
Taxable salary (taxable at normal slabs) 6,570,000

Income from property


Rent of ground floor Rs.137,500 x 4 months 550,000
10% of non-adjustable deposit of Rs.500,000 50,000
Rent of residential portion Rs.2,400,000 x 3/24 300,000
Chargeable rent 900,000
Less: Repairs allowance 1/5th of chargeable rent 180,000 720,000

Capital Gain
Gain on disposal of listed company: 0% tax as the shares were
acquired before 1.7.2013 500,000 exempt
Taxable Income 7,290,000

Tax liability (Salaried Case)


Tax on Rs.7,290,000
Tax on Rs.6,000,000 1,095,000
Tax on Rs.1,290,000 @ 35% 451,500
Tax on compensation @ 8% of Rs.4,000,000 320,000
Total tax liability 1,866,500
Less: Tax deducted from salary 837,000
Tax deducted by the tenant 230,000 1,067,000
Tax payable with return of income 799,500

Notes:
1. Interest free loan: The benchmark rate 10% is included in taxable salary as a taxable perquisite.
2. Compensation under redundancy scheme may be taxable at the last three years' average rate of tax at
the option of the taxpayer which is 8% in this case and beneficial for Mr. Khan.

235 | P a g e
3. Property income is taxable on accrual basis and therefore rent for the actual rent out period of the
relevant tax year is taxable and the advance rent adjustable against future rentals is not taxable in the
current tax year.
4. Donation to unapproved institution is not eligible for rebate.

Question # 27 [CFAP Questions for Jazabati Students] Q.1 December 2012

Mr. Yaqeen
Tax Year 20X2
Computation of taxable income and tax liability
Salary from KKUH
Basic salary 500,000 x 6 months. 3,000,000
Medical allowance 60,000 x 6 360,000
Less: exempt up to 10% of basic salary 300,000 60,000
Leave fare assistance 240,000

Salary from DPL


Consideration for joining DPL 3,000,000
Basic salary 800,000 x 6 4,800,000
Medical allowance 80,000 x 6 480,000
Less: exempt up to 10% of basic salary 480,000 --
Utility allowance 100,000 x 6 600,000
Use of refrigerator, cooking range and washing machine
15% depreciation on Rs.200,000 for 6 months 15,000
Concessional loan 5,000,000 x (10% -8%) for 3 months 25,000
Taxable salary 11,740,000

Capital Gain
Consideration received i.e. insurance' claim for painting 600,000
Admissible deductions incidental to purchase and sale of painting
Purchase cost (350,000)
Insurance premium (24,000)
Lawyer's fee for insurance claim (50,000)
Capital gain 176,000
Disposal of shares in ABL
Sale proceed 425,000
Less: taxed as dividend income 15,000 shares x Rs.25 375,000 50,000
Taxable income 11,966,000

Tax liability (Salaried Case)


Tax on Rs.11,966,000
Tax on Rs.6,000,000 1,095,000
Tax on Rs.5,966,000 @ 35% 2,088,100
Total tax liability 3,183,100

Notes:
1. Option under employee share scheme is not taxable unless it is disposed off. As the same is not yet
disposed of no taxable benefit arises.

236 | P a g e
2. Salary received from ex-employer in Norway is exempt as Mr. Yaqeen is a returning expatriate and
therefore his foreign source income is exempt.
3. Rent of agricultural land used for agricultural purpose is exempt as this amount is part and parcel of
agricultural income which is exempt.
4. Profit on debt utilized for house is not eligible to be deducted from total income.
5. Where a capital asset is lost or destroyed, consideration received shall be the scrap value along with any
compensation, indemnity or damages received under an insurance policy, agreement, settlement or
judicial decision.
Therefore, insurance claim shall be treated as consideration received and insurance premium and
lawyer's fee for insurance settlement shall be treated as incidental expenses for the insurance claim.
6. The amount of dividend in specie (i.e. FMV of shares at the time of transfer to the shareholder) shall be
taxed as dividend under FTR @ 15% and at the time of disposal the difference between the consideration
received and the amount of dividend shall be taxable as capital gain.

Question # 28 [CFAP Questions for Jazabati Students] Q.1 December 2013

Mr. Iqbal Tax Year 20X4


Computation of taxable income and tax liability
SALARY
Basic salary 300,000 x 12 months 3,600,000
Cost of living allowance 50,000 x 12 600,000
Milk allowance 10,000 x 12 120,000
Bonus 300,000
Company maintained car for personal use 1,800,000 x 10% x 122/365 60,164
Purchase of car from the employer 600,000-250,000 350,000
Reimbursement of driver's salary 36,000
Rent free accommodation 45% of basic salary 1,620,000
Employee share scheme ($2.5 - $1.5) x 4,000 shares x Rs.100 400,000
Taxable salary 7,086,164

Income from Property


Chargeable Rent 800,000
Repairs allowance 1/5th of chargeable rent (160,000)
Insurance premium (30,000)
Ground rent (10,000)
Interest expense (15,000)
Other expense i.e. watchman's salary (8,000) 577,000

Income from Business


Brokerage fee 200,000
Less: expenses Rs.30,000+10,000 40,000 160,000

Capital Gain u/s 37


Disposal of shares of TL's parent company
$3-$1.5-$1 = $0.5 x 3,000 x 100 150,000

Income from Other Sources


Profit on saving bank account (separate block of income) 150,000
Compensation on delayed refund 25,000
Total income 8,148,164

237 | P a g e
Less: Zakat deducted by the bank 25,000
Profit on saving bank account taxable at separate rates 150,000 175,000
Taxable Income 7,973,164

Tax liability (Salaried Case)


Tax on Rs.7,973,164
Tax on Rs.6,000,000 1,095,000
Tax on Rs.1,973,164 @ 35% 690,607
1,785,607
Less: Tax credit on approved pension fund
1,785,607/7,973,164 x (1,600,000 or 20% of 7,973,164
Whichever is lower i.e. 1,594,633) 357,121
1,428,486
Tax on profit on saving account 15% of Rs.150,000 22,500
Total tax liability 1,405,986
Less: Tax deducted from salary 600,000
Tax paid on profit on saving account 15,000 615,000
Tax payable with return of income 835,986

Notes:
1. Company maintained car exclusively for business purpose is not a taxable perquisite. Company
maintained car for personal use is taxable at 10% of the cost to the employer i.e. Rs.1,800,000
proportionate to the days available for use i.e. 1st March to 30th June (122 days).
2. Capital gain on disposal of shares in Tameer Inc is taxable under section 37 as Tameer Inc is not a
public company being not listed in Pakistan.
3. Shares in TL as a reward for his excellent performance is taxable at the FMV.
However, as the employee does not have right to transfer the shares year the same is not taxable in the
current tax year.
4. Commission paid to brother Rs.10,000 as a gift is allowable tax expense as the brother has performed
an activity for the business.
Any expense whether paid voluntary is also allowable tax expense if related to business e.g. bonus
paid to employees on voluntary basis in addition to the terms of employment contract is also
allowable tax expense.
5. Profit on saving account with an Islamic bank is taxable @ 15% as a separate block of income.

Question # 29 [CFAP Questions for Jazabati Students] Q.6 June 2015

Mr. Pansari
Tax Year 20X4
Computation of taxable income
SALARY
Basic salary 450,000 x 12 5,400,000
Conveyance allowance 50,000 x 12 600,000
Company maintained car 5% of Rs.3,000,000 150,000
[Company maintained car is taxable @ 5% of the cost of car for
The employer instead of fmv in the current year]
Leave encashment 75,000
[Salary is taxable on receipt basis. However, the definition of
Receipt includes an amount made available to the employee and

238 | P a g e
Therefore leave encashment made available to mr. Pansari and
Waived by him voluntarily is treated as received by him and taxable]
Cans of olive oil Rs.500 x 2 x 12 12,000
Employee share scheme
FMV of 8,000 shares x $5 x Rs.102 4,080,000
Cost paid by Mr. Pansari 8,000 shares x $3 x Rs.1022, 448,000 1,632,000
Pension from ex-employer Exempt --
Fee for attending BOD meeting 200,000
Taxable salary 8,069,000

Capital Gain u/s 37


Sale value of 6,000 shares x $8.5 x Rs.102 5,202,000
Cost of 6,000 shares x ($3 + $2) x Rs.102 3,060,000
2,142,000
Less: Capital Loss of the last year 25,000 2,117,000

OTHER SOURCES
Royalty income 2,000,000
[Where completion time of a literary or artistic work exceeds
24 months, the author may elect to treat any lump sum royalty
Received in a tax year in respect of the work as having been
Received in that tax year and the preceding two tax years in
Equal proportions.
Time of completion in this case is 19 months and therefore
The royalty is fully taxable in the current year]
Taxable income 12,186,000

239 | P a g e
CHAPTER 09
BUSINESS INCOME
BASIC CONCEPTS & TAX
STRUCTURE
A. Definitions
a) Business Income ( S – 18)
b) Speculative Income ( S – 19)
B. Deductions
a. General
i. Admissible Deductions (S – 20)
ii. In-admissible Deductions (S – 21)
b. Capital Expenditures
i. Depreciation (S – 22)
ii. Initial Allowance (S – 23)
iii. Intangibles (S – 24)
iv. Amortization (S – 25)
c. Disposal of Assets
i. Disposal and acquisition of assets (S – 75)
ii. Cost (S – 76)
iii. Consideration received (S – 77)
iv. Non-arm's length transactions (S – 78)
v. Non-recognition rules (S – 79)
d. Specific Deductions
i. Scientific Research Expenditure ( S – 26)
ii. Employee Training and Facilities ( S – 27)
iii. Profit on debt ( S – 28)
iv. Bad Debts ( S – 29)
e. Method of accounting
i. Method of accounting ( S – 32)
ii. Cash-basis accounting ( S – 33)
iii. Accrual-basis accounting ( S – 34)
iv. Stock-in-trade ( S – 35)

240 | P a g e
Income from business

1. The following incomes of a person for a tax year, other than income exempt from tax under this Ordinance,
shall be chargeable to tax under the head “Income from Business”–
a. the profits and gains of any business carried on by a person at any time in the year;
b. any income derived by any trade, professional or similar association from the sale of goods or provision
of services to its members;
c. any income from the hire or lease of tangible movable property;
d. the fair market value of any benefit or perquisite, whether convertible into money or not, derived by a
person in the course of, or by virtue of, a past, present, or prospective business relationship
Example

Mr. Malik Riaz contracted with DHA to build phase- 120 in Lahore. DHA decided to give them Rs.100
million and 1 canal plot in the same phase. The fair market value of the plot was Rs.10 million. The total
business income is Rs.110 million (100+10).

Explanation (ITO).-
For the purposes of this clause, it is declared that the word ‘benefit’ includes any benefit derived by way of
waiver of profit on debt or the debt itself under the State Bank of Pakistan, Banking Policy Department,
Circular No.29 of 2002 or in any other scheme issued by the State Bank of Pakistan.

e. any management fee derived by a management company (including a modaraba management


company)

2. Any profit on debt derived by a person where the person’s business is to derive such income shall be
chargeable to tax under the head “Income from Business” and not under the head “Income from Other
Sources”.
Example:
Mr. Sam deposited Rs.1 million in HBL bank. HBL issued loan to Mr. Man Rs.1 million. HBL charged
interest from Mr. Man Rs.200, 000/- and paid interest to Mr. Sam Rs. 100,000/-. The taxable income of
HBL under the head BUSINESS shall be Rs. 200,000/- and taxable income of Mr. Sam shall be Rs.
100,000/- under the head INCOME FROM OTHER SOURCES. Mr. Sam is running a business of cloth
in Nawaz cloth market.

3. Any amount received by a banking company or a non-banking finance company, where such amount
represents distribution by a mutual fund or a Private Equity and Venture Capital Fund out of its income
from profit on debt, shall be chargeable to tax under the head “Income from Business” and not under the
head “Income from Other Sources”

4. Where a lessor, being a scheduled bank or an investment bank or a development finance institution or a
modaraba or a leasing company has leased out any asset, whether owned by it or not, to another person, any
amount paid or payable by the said person in connection with the lease of said asset shall be treated as the
income of the said lessor and shall be chargeable to tax under the head “Income from Business”.

241 | P a g e
Examples:
1- MCB bank (scheduled bank) owned a building in Lahore and let out the same building to Khadim
Enterprises at ALV of Rs. 500/- and also leased 5 cars at annual rent of Rs.200/-. MCB also-taken
a building on rent in Karachi for his office purposes. The upper portion of that building is given on
rent to Mr. Bhai at an annual rent of Rs.50/-. Total income (Rs. 750/-) received from Lahore
building (owned), Karachi building (being a tenant) and lease of car shall be taxed under the head
BUSINESS income.
2- Mr. Ghareeb is running a business of rent-a-car and also owned a building in Lahore. He let out the
same building to Khadim Enterprises at ALV of Rs. 500/- and also leased 5 cars at annual rent of
Rs.200/-. Mr. Ghareeb also-taken a building on rent in Karachi for his office purposes. The upper
portion of that building is given on rent to Mr. at an annual rent of Rs.50/-. The detail of taxable
income shall be as follows
Income from Property Rs. 500
Income from Business Rs.200
Income from Other Sources Rs.50
Total Taxable Income Rs.750

Important aspects of income from business


General principles relating to taxation of business income are summarized below:
• A resident person is taxed on his worldwide business income whereas a non-resident person is
liable to tax in respect of his income to the extent it is Pakistan-sourced.
• Income may be recorded using the cash or accrual basis of accounting. Companies are, however,
required to follow the accrual system of accounting for its business income.
• Expenditure incurred wholly and exclusively for the purpose of business is generally allowable in
tax except for certain specific provisions of law which are discussed later in the chapter.
• The profits which are taxed under section 18 are the real profits and not notional profits. Therefore,
gain arising from revaluation/impairment of fixed assets / investments or unrealized gain or loss
arising from revaluation of foreign currency related debtors and creditors is not taxable as these
are not real profits and can only be taxed when these are actually realized.
• Profits can arise only out of the trading (revenue) receipts. Capital receipts are not taxable unless
expressly made taxable under the Income Tax Ordinance, 2001.
• Those profits and gains of a business are chargeable to tax which are carried on by the taxpayer at
any time during the tax year. However, following receipts are taxable even if the taxpayer does
not carry on business during the tax year:
• Recovery in cash or kind against a deduction/loss allowed previously against business
income
• Gain on sale of depreciable assets
• Recovery made out of bad debts allowed in preceding year
• Trading liabilities or any portion thereof which is found not to have been paid within the
expiration of three years of the end of the tax year in which it was allowed
• Sum received after discontinuance of a business
Speculation business

Definition:
In this section, “speculation business” means any business in which a contract for the purchase and sale of any
commodity (including stocks and shares) is periodically or ultimately settled otherwise than by the actual
delivery or transfer of the commodity, but does not include a business in which –
a. a contract in respect of raw materials or merchandise is entered into by a person in the course

242 | P a g e
of a manufacturing or mercantile business to guard against loss through future price fluctuations
for the purpose of fulfilling the person’s other contracts for the actual delivery of the goods to
be manufactured or merchandise to be sold;
b. a contract in respect of stocks and shares is entered into by a dealer or investor therein to guard
against loss in the person’s holding of stocks and shares through price fluctuations; or
c. a contract is entered into by a member of a forward market or stock exchange in the course of
any transaction in the nature of jobbing arbitrage to guard against any loss which may arise in
the ordinary course of the person’s business as such member.
Example:
In August 2015, LT signed a future contract with Mubarak Enterprises (ME) for the purchase of 500 metric
tons of maize at Rs. 15,800 per metric ton. The delivery was expected to be made in October 2015. ME also
agreed to repurchase the entire lot at the price prevailing on the date of sale. In October 2015 price of maize
increased to Rs. 18,240 per metric ton and LT sold the entire lot to ME without taking delivery. (CA-Inter,
Spring 2016)
(Ans: The gain Rs.1,220,000 [500 x (18,240 – 15,800)] is taxable under the head speculative income.)

Where a person carries on a speculation business:


(a) That business shall be treated as distinct and separate from any other business carried on by the person;
(b) This part shall apply separately to the speculation business and the other business of the person;
(c) Apportionment of deductions shall apply as if the profits and gains arising from a speculation business
were a separate head of income (Sec 67);
(d) Any profits and gains arising from the speculation business for a tax year computed in accordance
with this part shall be included in the person`s income chargeable to tax under the heading “Income
from Business”
(e) Any loss of the person arising from the speculation business sustained for a tax year shall be set off
only against the income of the person from any other speculation business of the person chargeable to
tax for that year;
(f) If a speculation loss sustained by a person for a tax year is not wholly set off, then the amount of the loss
not set off shall be carried forward to the following tax year and applied against the income of any
speculation business of the person in that year and so on, but no speculation loss shall be carried
forward to more than six tax years immediately succeeding the tax year for which the loss was first
computed.
Exercise: Speculation Business
M/s XYZ Enterprises deals in cloth trading. Total revenue from cloth trading was Rs. 10,000,000 during
the year 20YY. The gross profit from the trading business was amounting to Rs. 2,000,000.
During the year, the price fluctuations were very high in cloth market. Considering this trend, M/s XYZ
Enterprises also made forward purchasing of cloth to reap the benefit of price fluctuations. In April 20YY,
The enterprises agreed to purchase Bengali cloth of 20,000 bundles at the rate of 100 per bundle, the
delivery of which was expected in June 20YY. The seller agreed to purchase the same goods at the rate
ruling at the date of sale. In June the price of Bengali cloth has been increased to Rs. 120 per bundle. M/s
XYZ Enterprises disposed of that cloth of 20,000 bundles to the seller at the market prevailing rate without
taking any delivery of stocks, Total revenue from sale of cloth aggregates to Rs. 2,400,000, Therefore, the
Enterprise earned income of Rs. 400,000.
Total administrative and general expenses of Rs. 1,000,000 were incurred during the year 20YY. Compute
the taxable income and tax liability.
It is worth mentioning that carry forward loss of the Enterprise was Rs. 1,000,000 in respect of business.
Whereas speculation loss was Rs. 250,000.

243 | P a g e
Answer

Particulars Speculation Business Trading Business Total


Gross Revenue 2,400,000 10,000,000 12,400,000
Gross Profit 400,000 2,000,000 2,400,000
Expenditure (1,000,000 x 193,548 806,452 1,000,000
2,400,000/12,400,000)

Net income 206,452 1,193,548 1,400,000


Carry forward loss 250,000 1,000,000 1,250,000
Taxable Income/ (loss) for the year *(43,548) 193,548

*Speculation loss carried forward


** Loss of speculation business cannot be set off with trading business.
Deductions in computing income chargeable under the head

A. Expenditures incurred for business purpose:


A deduction shall be allowed for any expenditure incurred by the person in the year wholly and exclusively for
the purposes of business.

Some of the examples of expenditures that may be allowed as deductions are given below:

1. Cost of the goods purchased and utilized for business.


2. Cost of goods manufactured and sold.
3. Rent for premises used for the business.
4. Any tax, charge or rate other than income tax or based on profit. (Custom
tax, road tax, property tax, municipal tax.
5. Repair charges of any asset used for the business.
6. Insurance premium paid for the insurance of assets.
7. Any sum paid to the employee (few exceptions).
8. Actual number of bad debts.
9. Expenditure incurred on any educational institution or hospital for the
benefits of the employee or their dependents.
10. Expenditure incurred on training of the employees.
11. Annual Subscription paid to a registered trade association.
12. Expenditures on foreign delegations.
13. Any expenditure wholly and exclusively for business.

B. Animals which used for business:


Animals which have been used for the purposes of the business or profession otherwise than as stock-in-trade
and have died or become permanently useless for such purposes, the difference between the actual cost to the
taxpayer of the animals and the amount, if any, realized in respect of the carcasses or animals.

Actual cost of the animals XXX

Less: Amount realized in respect of carcass or (XXX)


animal

244 | P a g e
Amount to be allowed as deduction XXX

Example:
• Habib Dairy Ltd purchased three cows for dairy business at Rs.100,000/- each in tax year 20X1.
Vaccination cost of each cow was Rs.2,000/-. One cow became permanently disabled and was sold
at Rs.20,000/- in tax year 20X3. Compute the deductions allowed under the business income (ANS:
In tax year 20X1 total business expenditures will be Rs.6,000/- (2,000 x 3). Similarly, no
expenditures will be allowed in 20X2. In tax year 20X3 Rs.80,000 (100,000-20,000) will be charged
as business expense. )
• Cost of sale included Rs. 400,000 in respect of the cost of two cows as they became permanently
useless for milking purposes. These cows were originally purchased for TL’s dairy farm in
Faisalabad for Rs. 200,000 each. TL sold these cows in the market for Rs. 80,000 each, for which
no entry has been made in the accounts. (Cost, net of sale, is an allowable deduction, which means
400,000 – 160,000 (80kx2) = 240,000 should be deducted from account. Rather 240k the company
has deducted Rs.400k. The additional cost of Rs.160,000 should be added back in the profits of the
company)

C. Depreciation & Amortization


Where expenditures incurred in acquiring a depreciable asset or an intangible, with a useful life of more than
one year or is pre-commencement expenditure, the person must depreciate or amortize the expenditures.
(because capital expenditures are not allowed).

Examples:
• Office building purchased at Rs.50 million. This is not a business expenses rather depreciation shall
be charged.
• Administrative expenses included Rs.4.8 million, paid against purchase of industrial software
having a useful life of three years. This amount will be added back in the accounting profit and
amortization for the year will be deducted to reach at taxable income.
• The Company incurred an expenditure of Rs.2, 000, 000 on sales promotion. It has been estimated
that the benefit of such expenditure will extend to 3 years and, therefore, the same is being amortized
over a period of 3 years. However, for tax purposes, the whole of the expenditure has been claimed.

D. Amalgamation Expense
Where any expenditure is incurred by an amalgamated company on legal and financial advisory services and
other administrative cost relating to planning and implementation of amalgamation, a deduction shall be allowed
for such expenditure.

Examples:
Alpha Ltd. and Beta Ltd. merged to form a new company Xeta Ltd. Xeta Ltd. paid Rs.5 million for
the merger. 5 million is allowed as an expense to Xeta Ltd. These expenditures are not allowed to
Alpha Ltd and Beta Ltd.

Note for the student


• Section 21 provides a list of deductions which are not admissible for tax purpose e.g. penalty paid for
the violation of any law, rules or regulation is not admissible for tax purpose even though charged in
the accounts.

245 | P a g e
• There may be a situation where an expense has been charged in the accounts but the same is not
admissible for tax purpose as per section 21 or any other provision of the Ordinance.
• In this case the expenditure needs to be added back in the accounting profit while determining taxable
income e.g. salary of an employee is required to be paid through banking channel if it exceeds the
specified amount per month and if paid in cash would become inadmissible for tax purpose.
• There may be a situation where an income has been credited in the accounts but not taxable due to
exemption provided in the tax laws. This needs to be deducted from accounting profit.
• In certain cases, an amount may have the same treatment for accounting purpose as well as tax purpose
e.g. rent paid for business premises through banking channel is admissible tax deduction but no
adjustment is required as the same has already been charged in the accounts.
• There may be a timing difference despite the fact that an item has the same treatment for accounting
and tax purposes e.g. provision for doubtful debt is an accounting expense in the year when provision
is created but allowable tax expense in the year when the same is written off as per section 29 of the
Ordinance.
• In this case this amount shall be added back in the year when provision is made and shall deducted in
the year when it is written off.
• An expenditure which is allowable for tax purpose in the year in which it is charged in the accounts is
termed as against revenue expenditure.
• As against revenue expenditure, capital expenditures are those expenditures which provide benefit to
the business entity for more than one year and they are depreciated or amortized.
• Tax laws provide rules for calculation of tax depreciation which are normally different from the
accounting concepts of depreciation. Therefore, accounting depreciation shall be added back in the
accounting profit and tax depreciation including initial allowance etc. shall be deducted from accounting
profit while determining taxable profit
Question
A Ltd has disclosed the following particulars and requested you to compute
taxable income of the company: Rs.
a) Accounting profit before tax 5,000,000
b) Charged to the above accounting profit
i. Accounting depreciation 1,400,000
ii. Penalty paid on contravention of the Companies Act
(Inadmissible for tax purpose) 57,000
iii. Advertisement (admissible for tax purpose) 2,000,000
iv. Provision for doubtful debts during the year (inadmissible) 25,000
c) Other information
i. Depreciation as per tax provisions 1,650,000
ii. Doubtful debts written off during the year against provision in
the previous year Rs.18,000 which was added back in the
previous year.
Iii. Sale of wastages during production Rs.400,000 included in
the accounting income also taxable income as per tax
provisions.
Answer: Rs.
Accounting profit 5,000,000
Accounting depreciation 1,400,000
Penalty in violation of law 57,000
Provision for doubtful debts 25,000

Less: Tax depreciation 1,650,000

246 | P a g e
Provision for doubtful debts written off 18,000 1,668,000
4,814,000

Apportionment of expenditure, deductions and allowances – Section 67

Where expenditures, deductions and allowances relate to –


(1) the derivation of more than one head of income; or
(2) derivation of income comprising of taxable income on net income basis and any
(3) the derivation of income chargeable to tax under a head of income and to some other purpose,
The expenditures, deductions and allowances shall be apportioned on any reasonable basis taking into account
of the relative nature and size of the activities to which the amounts relate.

Rules for apportionment of expenditures, deductions and allowances (Rule 13)

Common expenditure, deductions and allowances means any expenditure, deductions and allowances that is
not clearly allocable to any particular class or classes of incomes. Rules regarding apportionment of such
expenses, deductions and allowances are given as follows:
• Any expenditure, deduction or allowance that is incurred for particular class of income (chargeable
income, FTR income or exempt income) shall be allocated/apportioned to that class only.
• Any common expenditure shall be apportioned amongst each class of income according to the
following formula:
• Amount of expense x Gross receipts for the class of income
Gross receipts for all class of income
Note: Gross receipts means net off receipts or turnover of sales tax of FED paid.

• While allocating common expenditure, deductions and allowances (particularly selling expenses) the
nature and source of each class of income must be taken into account.
• The basis determined for allocation of expenditure, deductions and allowances should be certified by
a Chartered Accountant or a Cost and Management Accountant. This certificate shall be accepted by
CIR unless there is significant variations (10% + beyond the limits) from allocation under the rules.

• Where in case of certain transaction the net gain, brokerage, commission and other income is taken as
turnover, then the gross profit from business shall be taken as gross receipts for the purpose of
apportionment of expenditure

247 | P a g e
CHAPTER 10
DEDUCTIONS NOT ADMISSIBLE
FROM BUSINESS INCOME
Deductions not admissible - General concept
An expense may be an accounting expense but inadmissible for tax purpose which shall be added back in the
accounting profit while calculating taxable income. There may also be a situation where a particular item is a
tax expense but not charged in the accounts e.g. initial allowance of a depreciable asset which needs to be
deducted from accounting profit.
Section 21 of the Income Tax Ordinance specifies deductions which are not admissible for purpose. There are
other inadmissible deductions as well such as provision for doubtful debts [section 29] which are discussed in
a later chapter.

Tax, charge or levy paid to the government - Section 21(a)


Any cess, rate or tax paid or payable in Pakistan or a foreign country on the profits of the business as a % or
otherwise on the basis of such profits.
Taxes and duties which are not based on profit are admissible such as custom duties on import of raw material
and property tax on business premises.
Tax payable under the Income Tax Ordinance is not allowable deduction. Normal presentation in the accounts
is as under:
Net profit before tax 8,000,000
Provision for taxation 2,100,000
Net Profit after tax 5,900,000
In the above case, we can take net profit before tax as a starting figure for the purpose of calculation of taxable
income and in this case we do not need to add back provision for taxation However, if we start with the figure
of net profit after tax only then provision for taxation shall be added back.

Examples:
Following tax payments are not allowed as business expense:
• Tax paid at the time of filing of tax return;
• Tax assessed by the commissioner of inland revenue;
• Sales tax claimable as input tax
• Tax payable on foreign source income.
Following payments are allowed as business expense:
• Custom duty paid at import stage
• Short Sales tax liability paid of previous year
• Vehicle tax paid in cash amounting to Rs. 55,000 for eight office cars

Tax deduction from payment received by a taxpayer - Section 21(b)


Any amount of income tax deduction from the taxpayer is not allowable tax expense.
This amount is advance payment of tax and shall be deducted from tax liability e.g. tax deduction from payment
to A Ltd (a manufacturing company) on account of supply of goods is not a Profit and Loss account item for A
Ltd but deductible from tax liability to arrive at tax payable by /refundable to A Ltd.

Example:
Payment received against supply of goods Rs.95, 500/- net of tax deducted @ 4.5%. The total taxable income
shall beRs.100, 000/- (95,500/95.5x100) because tax deducted shall not be allowed as an expense.

248 | P a g e
Tax deduction from payments made by a taxpayer as business expenditures- Section 21(c)
Any expenditure, payment of which is subject to withholding tax is not allowable tax expense unless the person
has paid or deducted and paid the required withholding tax

However, disallowance in respect of purchases of raw materials and finished goods shall not exceed 20% of
purchases of raw materials and finished goods.
It means that default in case of withholding tax will result in disallowance of such expenditure. However, the
Commissioner has power to recover, from the recipient or the payer, the amount of tax not deducted and / or
paid and in this case section 21(c) shall not apply.
It should also be considered whether a taxpayer is obliged to deduct tax or not e.g. an individual or AOP having
turnover of less than Rs.100 million is not obliged to deduct tax while making payment on account of
commission [section 233] and in this case section 21(c) shall not apply and therefore commission expense would
remain admissible tax deduction.

Few more examples where tax is not required to be deducted are:


➢ where salary of an employee does not exceed the basic exemption
➢ where payment in aggregate during a financial year is less than:
▪ Rs.75,000 for purchase of goods; and
▪ Rs.30,000 on account of services.
➢ where payment to a non-resident is not chargeable to tax in Pakistan
➢ Individual and AOP (not included in specific category such as educational institutions) paying rent of
less than Rs.1.5 million in a year

Example:
• Zahoor Ltd business expenses included rent of branch office Rs.600, 000/- and manager salary Rs.
350,000/-. Income tax has not been withheld from salaries and rent. Rent expenses are not allowed as
an expense because payment is made without deduction of tax whereas, salary is allowed as tax
expense because tax should not be deducted as it is below Rs.600, 000/-.
• Hameed paid Rs. 50,000/- as consultancy to a non-resident without deduction of tax this amount is
not allowed as an expense
• Bashir paid Rs. 50,000/- as consultancy to Mr. Michael (a non-resident) without deduction of tax
because he has taken exemption certificate from tax department. This amount is allowed as an expense
because was not deductible.
Example:
Stock Enterprises, purchased stock (raw material and finished goods) of Rs.100,000/- and made a
payment of Rs.100,000/- against services. The payments were made through cheques but tax was not
deducted. Total expenditures disallowed will be:

Stock (100,000 x 20%) Rs.20,000/-


Services Rs.100,000/-

Tax payable by any other person borne by the taxpayer


An employer is allowed to pay the tax payable by an employee in addition to normal salary and in this case the
said tax payment shall be considered as:
-salary income for the employee [Tax borne by employer] and
-salary expense in the hands of employer and is allowable tax expense. Any obligation of an employee
paid by the employer is an employment perquisite.

Cases other than salary:


In other cases, tax payable by any other person is not legally allowed to be paid by the taxpayer. It is a considered

249 | P a g e
opinion that if a taxpayer pays tax liability of any other person then the said tax payment would not be allowed
as tax expense.

Example:
• Hameed paid Rs. 50,000/- as consultancy to a non-resident without deduction of tax (@10%) but he
deposited Rs.5,000/- as tax in government treasury. Total expense allowed is Rs.50,000/-. Tax deposited
Rs.5,000/- is not allowed as an expense.
• Mr. Raees paid salary of Rs. 650,000/- to his manager for the current year without deduction of tax of
Rs.2,500/- but he deposited Rs.2,500/- as tax in government treasury. Total expense allowed is
Rs.652,500/-. Tax deposited Rs.2,500/- is allowed as an expense because section 13 says, tax born by
the employer shall be a part of salary.

Tax deduction from interest paid to non-resident


▪ Profit on debt shall be Pakistan source income if the debt is used for business in Pakistan. Therefore,
interest income earned by a non-resident person is taxable in Pakistan irrespective of the fact whether
such non-resident person has a permanent establishment in Pakistan or not.
▪ However, 2nd Schedule provides exemptions in specified cases. Likewise, interest income may be
exempt under any tax treaty or taxable at lower tax rate.
▪ It means that any person making payment on account of interest to a non-resident person shall deduct
tax unless any specific exemption is available. In case of default, interest expense would be disallowed.
▪ Almost similar concept is applicable for payment to Non-resident on account of 'fee for technical
services', 'fee for offshore digital services' and 'royalty' etc. with reference to business in Pakistan.

Commission to inactive persons on items covered under 3rd Schedule of the Sales Tax Act, 1990
Commission expense shall be inadmissible if it is in excess of 0.2% of gross amount of supplies of items listed
in 3rd Schedule of the Sales Tax Act where the recipient of commission is not appearing in the active taxpayers'
list under the Income Tax Ordinance.
Notes for Students:
Few examples of 3rd Schedule items are: cigarettes, juices, ice cream, beverages, bottled water, shampoo, soap,
toothpaste, cosmetics, tea, toilet paper and tissue paper, spices sold in retail packing with brand name and trade
mark, cement, household electrical goods and gas appliances etc.

Entertainment Expenditure - Section 21(d)


Entertainment expenditure in excess of the prescribed limit is not allowable deduction.
The types of entertainment expenditures that are allowable tax expense are mentioned in Rule 10 of the Income
Tax Rules. A summary of which is as under:
Entertainment expenditure. -
1) A deduction for entertainment expenditure shall be limited to expenditure incurred by a person which
is –
a. Outside Pakistan on entertainment in connection with business transactions;
b. In Pakistan on entertainment of foreign customers and suppliers;
c. On entertainment of customers and clients;
d. on entertainment at a meeting of shareholders, agents, directors or employees; or
e. on entertainment at the opening of branches.

2) A person shall be allowed a deduction only for expenditure incurred on the entertainment of persons
related directly to the person's business.
3) "Entertainment" means the provision of meals, refreshments, and reasonable leisure facilities in
accordance with the tradition of business and subject to overall norms and customs of business in
Pakistan.

250 | P a g e
Examples:
• Administration expenditures includes entertainment expenditure of Rs. 128,000 incurred on arrival of foreign
customers for business purposes are allowed as an expense
• Administration expenditures includes entertainment expenditure of Rs. 130,000 incurred by the director for
personal hoteling purposes are not allowed as an expense

Contribution to Provident, Gratuity or Superannuation Fund 21(e) & (f)


▪ Any contribution made by the person to a fund that is not a recognized provident fund, approved
superannuation fund, or approved gratuity fund or approved pension fund;
▪ An amount in excess of fifty percent of total contribution made by a person (employer) to approved
gratuity fund, approved pension fund or an approved superannuation fund.
▪ Any contribution made by the person to any provident or other fund established for the benefit of
employees of the person, unless the person has made effective arrangements to secure that tax is
deducted from any payments made by the fund in respect of which the recipient is chargeable to tax
under the head "Salary";

FBR circular on tax calculation is as under:


▪ Provision for the year is inadmissible (add)
▪ Payment to employees is admissible for tax purpose (less)
It means that expense in the above cases is allowable tax expense on payment basis.

Penalty or Fine - Section 21(g)


Any fine or penalty for the violation of any law, rule or regulation is not admissible.
Certain penalties are allowable tax expenses which are not for the violation of any law, rule or regulation e.g.
▪ Penalty for late repayment of loan installment or late payment of interest
▪ Demurrage to custom authorities
▪ Late payment surcharge with utility bills
▪ Liquidated damages or penalty for breach of a contract in the ordinary course of business

Note for students:


Incorrect declaration / affidavit given to any government department such as custom authorities is a violation
of law and any penalty in this respect is inadmissible.

Personal Expenses - Section 21(h)


Any personal expense by the taxpayer is not admissible
However, any asset e.g. car given to an employee for use is treated as business purpose and any personal use by
the employee would not render the expense inadmissible.

Amounts transferred to Reserves - Section 21(i)


Any amount carried to a reserve fund or capitalized in any way is not admissible.

Certain payments to a member by an Association of Persons (AOP) - Section 21(j)


Profit on debt, brokerage, commission, salary or other remuneration by an AOP to its members is not admissible
for tax purpose.
These items should be added back on the assumption that the same have been charged in the accounts unless
otherwise specified. However, if it appears from the question that these items have been charged in the
appropriation account then the same shall not be added back.
Rent paid by a firm to a partner for his residence is a part of salary expense of the firm and therefore not
admissible for tax purpose.
However, if a partner's premises is used by the firm and rent is paid to the partner then the same shall be treated
as rent expense which is allowable tax expense and need not be added back.

251 | P a g e
Illustration:
An AOP having two partners Mr. A and Mr. B with equal share:
Accounting profit 1,800,000

Accounting depreciation 200,000


Salary to partner A 900,000
Commission to partner B 50,000
2,950,000
Less: Tax depreciation 550,000
Taxable income 2,400,000
Income tax 315,000
Divisible income 2,085,000

Share of profit from AOP is exempt in the hands of individual members but the same shall be included for rate
purpose in other taxable income except income under FTR.
One should observe that salary is included in the share of profit from AOP in the above calculation. In fact, if a
member of an AOP receives salary or other remuneration, interest, commission or brokerage from the same
AOP it becomes part and parcel of share of profit from AOP and not considered as a separate income.
However, any other payment received from the firm shall be considered separately e.g. rent of partner's premises
used by the firm is taxable as property income in the hands of the partner.

Payment of Salary through banking channel Section 21(m)

Salaries and wages are required to be paid through crossed cheque or direct transfer of funds to the employee's
bank account or through digital mode.
However, this condition is not applicable where the salary of an employee does not exceed Rs.32,000 per month.
Payment of salary and wages is inadmissible in the following cases:
▪ Taxable salary paid without tax deduction
▪ Salary to a member by an AOP
▪ Salary paid in cash where it exceeds Rs.32,000 per month

Payment of Business expenditures (other than salary) by an individual or AOP through banking channel
- Section 21(1)
Other expenditures are also required to be paid by a business entity other than a company through crossed
banking instrument showing transfer of the amount from business bank account of the taxpayer.
However, this condition is not applicable in the following cases:
▪ Where aggregate of a single account head does not exceed Rs.250,000 for the year; or
▪ Single payment up to Rs.25,000; or
▪ Payment on account of freight, travel fare, postage, utility and other government dues
Online transfer of payment from the business account of the payer to the business account of banking channel,
subject to the condition that such transactions are verifiable from the bank payee as well as payments through
credit card shall be treated as transactions through the bank statements of the respective payer and the payee.
This clause shall also apply for a company unless section 21(la) has been made effective by the FBR.

Payment of Business expenditures (other than salary) by a company through banking channel - Section
21(la)
Other expenditures are required to be paid by a company through digital means from business bank account
notified to the Commissioner.
Definition of digital means- section 2(17B)
"Digital means" means digital payments and financial services including but not limited to-
a) online portals or platforms for digital payments/receipts;
b) online interbank fund transfer services;
c) online bill or invoice presentment and payment services;
d) over the Counter digital payment services or facilities;

252 | P a g e
e) card payments using Point of Sale terminals, QR codes, mobile device, ATMs, Kiosk or any other
digital payments enabled services; or
f) any other digital or online payment modes.
However, this condition is not applicable in the following cases:
Where aggregate of a single account head does not exceed Rs.250,000 for the year; or Single payment
up to Rs.25,000; or
Payment on account of freight, travel fare, postage, utility and other government dues
Note for students:
According to the proviso to section 21(la), this clause shall be effective from such date as the FBR may notify
in the official Gazette. It means that this clause is not effective for a company unless notified by the FBR.
However, till that date section 21(1) as above is applicable for a company as well.

Sales promotion expenses by a pharmaceutical manufacturer - Section 21 (O)


Sales promotion, advertisement and publicity expenses by a pharmaceutical manufacturer in excess of 10% of
turnover are inadmissible.

Utility bills in excess of limits -Section 21(p)


Utility bills in excess of limits as may be prescribed shall be inadmissible.

Sr. Transaction Decision

1 Total office expenditures during the year were Rs.450,000/-. All payments were 40,000 is
made through cross cheques except Rs.40,000/- and Rs.9,000/- in cash. Disallowed

2 Total office expenditures during the year were Rs.249,000/-. All payments were Nothing is
made through cross cheques except Rs.20,000/- and Rs.9,000/- in cash. disallowed

3 Total Freight charges during the year were Rs.150,000/-. The whole expenditures Nothing is
were incurred in cash. disallowed

4 Director of the company paid printing charges Rs.150,000/- through his personal Disallowed
bank account.

Expenditures attributable to sales made by an industrial undertaking to persons not registered under the
Sales Tax Act, 1990-Section 21(q)
Expenditures attributable to sales made by an industrial undertaking to persons required to be registered but not
registered under the Sales Tax Act, 1990 shall be inadmissible according to the following formula: Where-
(A/B) x C
A is the total deductions i.e. expenditures
B is the turnover for the tax year; and
C is the sales exclusive of sales tax and federal excise duty to persons required to be registered but not registered
under the Sales Tax Act, 1990 where sales is Rs.100 million or more per person.

However, disallowance of expenditures under this clause shall not exceed 10% of total expenditures.

Example:
Total sales of A Ltd is Rs.800 million including sales to:
Mr. Y, an unregistered wholesaler, Rs.120 million; and Mr. Z, an unregistered wholesaler, Rs.70 million.
Total expenditures of A Ltd Rs.650 million.

Calculate the amount of disallowance under section 21(q) of the Income Tax Ordinance, 2001.

Solution:

253 | P a g e
Sales to Mr. Z does not fall within the ambit of section 21(q) as the sales is not Rs.100 million or more.
Expenditures attributable to sales to Mr. Y are as under:
(650/800) x 120 = 97.5 million
Therefore, the expenditures disallowed under this clause are lower of the following:
▪ Expenditures as per formula above Rs.97.5 million; or
▪ 10% of the total expenditures Rs.65 million.
Disallowed expenditures are Rs.65 million.

Expenditures attributable to sales claimed by any person fails to integrate with FBR - Section 21(r)
Expenditures attributable to sales claimed by any person who is required to integrate with FBR but fails to
integrate his business with FBR.
However, disallowance of expenditures under section 21(r) shall not exceed 8% of allowable deductions.

254 | P a g e
Chapter 11 Depreciation & Capital
Expenditures
• Method of depreciation
• Rate of depreciation (dep.) and initial allowance (IA)
• Computation of dep. and IA (asset fully and partially used for business)
• Written down value of a depreciable asset (taxable entity and exempt entity)
• Computation of gain/loss
o Normal / general case
o Partial used asset disposed off
o Passenger Transport Vehicle NOT plying for hire (Value > Rs. 7.5 million)
o Disposal of Immoveable Property (SP>Cost)
o Export of depreciable assets
• Cost and Consideration of an asset
Depreciable asset

Depreciable asset means any tangible movable property, immovable property (other than unimproved land), or
structural improvement to immovable property, owned by a person that —

a) has a normal useful life exceeding one year;


b) is likely to lose value as a result of normal wear and tear, or obsolescence; and
c) is used wholly or partly by the person in deriving income from business chargeable to tax,
but shall not include any tangible movable property, immovable property, or structural improvement to
immovable property in relation to which a deduction has been allowed under another section of this Ordinance
for the entire cost of the property or improvement in the tax year in which the property is acquired or
improvement made by the person;

Provided that where a depreciable asset is jointly owned by a taxpayer and an Islamic financial institution
licensed by the State Bank of Pakistan or Securities and Exchange Commission of Pakistan, as the case may be,
pursuant to an arrangement of Musharika financing or diminishing Musharika financing, the depreciable asset
shall be treated to be wholly owned by the taxpayer.

Structural improvement

Structural improvement in relation to immovable property, includes any building, road, driveway, car park,
railway line, pipeline, bridge, tunnel, airport runway, canal, dock, wharf, retaining wall, fence, power lines,
water or sewerage pipes, drainage, landscaping or dam

Normal Depreciation

1. Subject to this section, a person shall be allowed a deduction for the depreciation of the person‘s
depreciable assets used in the person‘s business in the tax year.
2. Subject to sub-section the depreciation deduction for a tax year shall be computed by applying the rate
specified in Part I of the Third Schedule against the written down value of the asset at the beginning of
the year.

255 | P a g e
Sr. Assets Rate of
No Depreciation

1 Building (all types) 10%

2 Furniture (including fittings) 15%

3 Machinery and plant (not otherwise specified), Motor vehicles (all types), 15%
ships, technical or professional books, Ships
4 A ramp built for disable persons not exceeding Rs. 250,000 each. 100%

5 Below ground installation in mineral oil concerns. 100%

6 Computer hardware, allied items, machinery and equipment used in 30%


manufacture of IT products.
7 Aircrafts, aero-engines and aerial photographic apparatus 30%

8 Offshore platforms and production installations in mineral oil concerns. 20%

3. Normal depreciation on an asset acquired during a tax year shall be calculated on full year’s basis (where
the asset is commissioned for use) on the cost of the asset as reduced by initial allowance (or first year
allowance or accelerated depreciation) while no depreciation shall be calculated in the year of disposal.

4. The written down value of a depreciable asset of a person at the beginning of the tax year shall be
a. Where the asset was acquired in the tax year, the cost of the asset to the person as reduced by any initial
allowance in respect of the asset under section 23; or
b. In any other case, the cost of the asset to the person as reduced by the total depreciation deductions
(including any initial allowance under section 23) allowed to the person in respect of the asset in
previous tax years.
Explanation,- For the removal of doubt, it is clarified that where any building, furniture, plant or machinery is used for
the purposes of business during any tax year for which the income from such business is exempt, depreciation admissible
under sub-section (1) shall be treated to have been allowed in respect of the said tax year and after expiration of the
exemption period, written down value of such assets shall be determined after reducing total depreciation deductions
(including any initial allowance under section 23) in accordance with clauses (a) and (b) of this sub-section.

Example:

Exempt Ltd. was incorporated in 2017. Its business income was exempted for two years under Income Tax
Laws. It purchased plant and a car in 2017. The depreciation for the year 2019 shall be as follows: (Assume
same tax rules in the previous years)

Year Assets Plant-AB Plant-A Car-A Tax


Allowances
Depreciation Rates 15% 15% 15%
2017 Cost 5,000,000 3,000,000 1,500,000
Initial allowance (1,250,000) (750,000) (2,000,000)
Depreciable Value 3,750,000 2,250,000 1,500,000
Depreciation (562,500) (337,500) (225,000) (1,125,000)
WDV – 2017 3,187,500 1,912,500 1,275,000 (3,125,000)

256 | P a g e
2018 Depreciation (478,125) (286,875) (191,250) (956,250)
WDV – 2018 2,709,375 1,625,625 1,083,750 (956,250)
2019 Depreciation (406,406) (243,844) (162,563) (812,813)
WDV - 2019 2,302,969 1,381,781 921,188 (812,813)

Initial Allowance Sec-23

The WDV of an asset shall be computed after the deduction of the initial allowance. The provision regarding
the initial allowance is discussed under the section 23 of the Ordinance and that section states that:-

1. A person who places an eligible depreciable asset into service in Pakistan for the first time in a tax year
shall be allowed a deduction for initial allowance, when the asset is used by the person for the purposes
of his business for the first time or the tax year in which commercial production is commenced,
whichever is later.
eligible depreciable asset means a depreciable asset other than —

a) any road transport vehicle unless the vehicle is plying for hire;
b) any furniture, including fittings;
c) any plant or machinery that has been used previously in Pakistan; or
d) any plant or machinery in relation to which a deduction has been allowed under another section
of this Ordinance for the entire cost of the asset in the tax year in which the asset is acquired

2. The amount of the initial allowance of a person shall be computed @ 25% for plant and machinery. The
rules containing for the measurement of the cost of the asset is discussed under section 76.
3. An initial allowance allowed to lessor in respect of assets owned by him and leased to another person
shall be deductible only against the lease rental income derived in respect of such assets

Definition: Industrial Undertaking


“Industrial undertaking” means
a) an undertaking which is set up in Pakistan and which employs,
i. ten or more persons in Pakistan and involves the use of electrical energy or any other form of energy which
is mechanically transmitted and is not generated by human or animal energy; or
ii. twenty or more persons in Pakistan and does not involve the use of electrical energy or any other form of
energy which is mechanically transmitted and is not generated by human or animal energy: and which is
engaged in—
i. the manufacture of goods or materials or the subjection of goods or materials to any process which
substantially changes their original condition; or
ii. ship-building; or
iii. generation, conversion, transmission or distribution of electrical energy, or the supply of hydraulic power;
or
iv. the working of any mine, oil-well or any other source of mineral deposits;
v. from the 1st day of May, 2020, a person directly involved in the construction of buildings, roads, bridges
and other such structures or the development of land, to the extent and for the purpose of import of plant and
machinery to be utilized in such activity, subject to such conditions as may be notified by the Board;
vi. from the first day of July, 2020 a resident company engaged in the hotel business in Pakistan;
vii. telecommunication companies operating under the license of Pakistan Telecommunication Authority
(PTA)

5. Where a depreciable asset is used in a tax year partly in deriving income from business chargeable to tax and
partly for another use, the deduction allowed under this section for that year shall be restricted to the fair

257 | P a g e
proportional part of the amount that would be allowed if the asset was wholly used to derive income from
business chargeable to tax.
Example –: (A)
Mr. Depreciable purchased a building at a price of Rs. 10 million, for house and his office purposes and started
using a portion of his house for business office. The area covered by his office is almost 60%.
100% Use 60%
Cost of Building 10,000,000
Initial Allowance (-----------)
10,000,000
Depreciation (1,000,000) (600,000)
WDV - Year 1 9,000,000 (600,000)

Year – 2 Depreciation (900,000) (540,000)


WDV - Year 2 8,100,000
Year -1 Tax allowances - fully business used = (1,000,000)
Tax allowances - 60% business used = (600,000)
(Dep. Not allowed 400k)
Year -2 Tax allowances - fully business used = (900,000)
Tax allowances - 60% business used = (540,000)
(Dep. Not allowed 360k)
Total Depreciation not allowed in two years (400+360) = 760,000
(6) Where, in any tax year, a person disposes of a depreciable asset, no depreciation deduction shall be allowed
under this section for that year and gain / loss shall be = Consideration – Tax WDV

(7) Where the asset is partly used for business purposes, the written down value of the asset to compute gain,
shall be increased by the amount that is not allowed as a deduction, because of partial use for business purposes.

Example:

Assume the data given in example A, the asset was sold at Rs. 9million. Computation of gain or loss when the
asset was fully used for business or if used 60% for the business.

Full business Use 60% business use


Sale Price 9,000,000 9,000,000
WDV (8,100,000) (8,100,000)
Depreciation not allowed 0 (760,000)
Taxable Gain 900,000 140,000

Special Cases:

Passenger Transport Vehicle NOT plying for hire (Value > Rs. 7.5 million)

The maximum cost for depreciation purposes of a passenger transport vehicle not plying for hire, is 2.5
million rupees and gain on that vehicle shall be computed as follows:

7.5 𝑚𝑖𝑙𝑙𝑖𝑜𝑛
𝐺𝑎𝑖𝑛 = 𝑆𝑃 𝑥 − 𝑇𝑎𝑥 𝑊𝐷𝑉 (𝑏𝑎𝑠𝑒𝑑 𝑜𝑛 7.5 𝑚𝑖𝑙𝑙𝑖𝑜𝑛)
𝐴𝑐𝑢𝑡𝑎𝑙 𝐶𝑜𝑠𝑡

258 | P a g e
Actual Price = 10,000,000 Vehicle Value

Allowed Cost 7,500,000

Initial Allowance -

7,500,000

Depreciation (1,125,000)

WDV - Year -1 6,375,000

Depreciation Year -2 (956,250)

5,418,750

Sale Price 9,300,000

Gain 1,556,250

7.5 𝑚𝑖𝑙𝑙𝑖𝑜𝑛
1,556,250 = 9,300,000 𝑥 − 5,418,750
10,000,000

259 | P a g e
To DO Question

Q # 1 PEL. Ltd disposed one of its vehicles for Rs. 8,000,000 in tax year 20X2. The vehicle was purchased
in tax year 20X1. Cost of the vehicle was Rs. 7,800,000.

Compute the amount of taxable gain?

Q#2 Mr. JZ disposed of a car used in business on May 18, 20X1 for Rs. 6,300,000. It was purchased for Rs
7,650,000 and its tax WDV on June 30, 20X0 was Rs. 6,200,000.

Compute gain/(loss) on disposal?

Disposal of Immoveable Property (SP>Cost)

Where the consideration received on the disposal of immovable property exceeds the cost of the property, the
consideration received shall be treated as the cost of the property.

Example: Assume the data in example A, building was fully used for business purposes and was sold in third
year at 15 million.

𝐺𝑎𝑖𝑛 = 𝑆𝑎𝑙𝑒 𝑃𝑟𝑖𝑐𝑒 − (𝑆𝑎𝑙𝑒 𝑃𝑟𝑖𝑐𝑒 − 𝑇𝑎𝑥 𝐴𝑙𝑙𝑜𝑤𝑎𝑛𝑐𝑒)

𝐺𝑎𝑖𝑛 = 15,000,000 − (15,000,000 − 1,900,000)

Gain 1,000,000+900,000 = 1,900,000

Export of depreciable assets

Where a depreciable asset that has been used by a person in Pakistan is exported or transferred out of
Pakistan, the person shall be treated as having disposed of the asset at the time of the export or transfer for a
consideration received equal to the cost of the asset.

𝐺𝑎𝑖𝑛 = 𝐶𝑜𝑠𝑡 𝑜𝑓 𝐴𝑠𝑠𝑒𝑡𝑠 − (𝐶𝑜𝑠𝑡 𝑜𝑓 𝐴𝑠𝑠𝑒𝑡𝑠 − 𝑇𝑎𝑥 𝐴𝑙𝑙𝑜𝑤𝑎𝑛𝑐𝑒)

The gain shall be equal to accumulated tax depreciation and initial allowance

TO Do Question

Q#1 A company disposed of its factory building for Rs. 10,000,000. Cost of the building was Rs.
8,000,000. And WDV of the building at the time of the disposal is Rs. 6,500,000.

Compute the gain or loss on disposal?

Q#2 Mr. J disposed of its factory building on May 18, 20X1 for Rs. 650,000. It was purchased for Rs.
400,000 and its WDV on 30 June 20X0 was Rs. 277,000.

Compute gain or (loss) on disposal?

Q#3 Mr. J exported a computer which was previously used as a fixed asset in his business to Bangladesh
against consideration of Rs. 300,000. The cost of the asset was 200,000. And its WDV at the time of
disposal was Rs. 20,000.

260 | P a g e
Compute gain or loss on disposal?

Question
During the tax year 2024, CFG (Pvt.) Limited disposed of the following assets:
(a) Immoveable property was sold for Rs. 150 million. The cost of the property was Rs. 100 million. Upto
tax year 2022, tax depreciation of Rs. 30 million had been allowed on the immoveable property.
(b) A machine used in the business in Pakistan, was exported to USA. The export proceeds amounted to Rs.
45 million. The cost and written down value of the machinery was Rs. 35 million and 28 million respectively.
(c) Two buses were disposed of for Rs. 2.5 million. They were acquired in tax year 2021. The tax written
down value of buses at the beginning of the tax year 2023 was Rs. 2.4 million. The buses were being used
partly i.e. 60% for business purpose. Tax rate of depreciation is 15%.
Required:
Calculate tax gain on loss on disposal of above asset
Solution

(a) Rs. In Million


Sale Proceed 150
Cost (Note) 150
Depreciation allowed (30)
WDV at the time of disposal (120)
Gain on disposal 30
Note: For computing gain on disposal of immoveable property, the consideration received shall be
treated as the cost of property if the consideration exceeds its cost (Gain on disposal shall be equal
to the depreciation allowed).
b)
Consideration received equal to actual cost 35
WDV at the time of disposal Gain on disposal (28)
Gain on Disposal 7

For computing gain on disposal of depreciable asset by way of export that has been used previously in
Pakistan, the consideration received shall be treated as the cost of asset (Gain on disposal shall be equal to
depreciation allowed)

(c) Rs. In Million


Sale proceed 2.5
Less: WDV at beginning of the year (2.4)
Depreciation not allowed (2.4/0.85 x 0.15 x 0.40) (0.17)
Loss on disposal (0.07)
WDV of the asset, in case asset is used partly for business and party for non-business purpose, shall be
computed on the basis that the asset has been solely used to derive business income. It means that depreciation
allowed as well as disallowed shall be deducted from the cost of the asset in arriving at the WDV. However,
the WDV of the asset shall be increased by the amount of depreciation disallowed on account of non-business
use at the time of disposal.

261 | P a g e
Asset
Disposal [D.E.S.E.R.T S.T.A. R L.C.D]
Disposal includes
1 Sold 2 Exchanged
3 Transferred 4 Distributed
5 Cancelled 6 Redeemed
7 Relinquished 8 Destroyed
9 Lost 10 Expired
11 Surrendered. 12 The transmission of an asset by
succession or under a will
13 The application of a business asset to
personal use or vice versa

▪ Transmissions of asset by succession or by will, shall be treated as a disposal of asset by deceased at


the time asset is transmitted.
▪ The application of a business asset to personal use shall be treated as a disposal of asset.
▪ Where a business asset is discarded or ceases to be used in business, it shall be treated to have been
disposed of.
▪ A disposal shall include the disposal of a part of an asset.

Disposal of an Asset in Parts


In a case where a part of an asset is disposed of, the cost of the asset shall be apportioned between the part of
the asset disposed and part of the asset retained in the business. The cost of the asset is apportioned on the
basis of the FMV of both parts at the time of acquisition.

Acquisition of an Asset
A person shall be treated as having acquired an asset at any of the following times:
• When he begins to own an asset
• When he is granted any right to own an asset; or
• When a personal asset is applied for a business use.
Cost of an Asset
The cost of an asset acquired by a person shall be determined in accordance with the following rules.

Purchase of an Asset
Where a person purchases an asset, its cost of acquisition shall be including the following amounts;
1. Consideration paid for the asset. The consideration may be in Cash or in Kind. Where the
consideration is in kind then the fair market value of an asset given as consideration (on the date when
new asset is acquired) shall be taken as consideration for new asset.
2. Expenses incurred on acquisition of an asset.
i. Broker’s Commission
ii. Registration Charges
iii. Tax such as CVT
iv. Cost of valuation report by a valuers in respect of acquisition of asset of a capital nature
3. Expenses incurred on alteration or improvement of an asset.
4. Expenses incurred on disposal of an asset.

262 | P a g e
Example

UBL Limited purchased a vehicle for Rs. 190,000 and incurred Rs. 10,000 on its registration, etc. after three
years the company sold out the vehicle for Rs. 160,000. The total Accumulated depreciation till the time of
disposal was Rs. 97,600. An amount of Rs. 5,000 will be expended on the disposal
Required: Compute the gain/ (loss) on disposal of vehicle.

Answer:
Rs. Rs.
Disposal Consideration 160,000
Less: WDV at time of disposal
Cost:
Purchase price 190,000
Expenses of registration 10,000
Expenses on disposal 5,000
Total 205,000
Less: Accumulated Depreciation (97,600) (107,400)
Gain on disposal of vehicle 52,600

Purchase of Assets through banking channel

The following assets are required to be purchased through banking channel:


• Immovable property having FMV exceeding Rs. 5 Million (FMV fixed by FBR or value fixed by the
provincial authority for the purpose of stamp duty, whichever is higher); and
• Other asset having FMV exceeding Rs. 1 Million
If the above asset is not purchased through banking channel then:
• The asset shall not be eligible for tax depreciation or amortization; and
• Cost shall be treated as zero for computation of any gain on sale of such asset
The above provision is applicable for all types of asset.

Personal Asset treated as business asset

Where a personal asset is applied for business and treated as acquired by the business, then the fair market
value at the time the asset is applied for business shall be taken as cost of asset

Asset produced or Constructed by a person

Where a person has himself produced or constructed an asset, then the following amount shall be included in
the cost of such asset:
▪ Total cost incurred in producing or constructing the asset
▪ Expenses incurred on bringing the asset in usable condition
▪ Expenses incurred for alteration or improvement of an asset
▪ Expenses incurred on disposal of an asset.

263 | P a g e
Example

PPL Limited purchased a machine for Rs. 1,350,000. The company incurred the following expenses in respect
of the machine:
Rs
Freight, Octroi and other charges 15,000
Loading and unloading charges 2,000
Spares purchased for the erection of the machine 3,000
Labor and other expenses connected with erection 5,000
Wages paid to the worker during the month of erection of machine 5,000
Oil and lubricants during the month 500
Required: Compute the cost of the machine

Solution
Cost of the Machine Rs. Rs.
Purchase Price 1,350,000
Add: Expenses on machine to bring it in useable condition
Freight etc. 15,000
Loading unloading 2,000
Spares 3,000
Labor Charges 5,000 25,000
Total cost of Machine 1,3750,000

Asset Acquired with a Foreign Currency Loan

Where an asset is acquired with a loan in foreign currency and exchange rate fluctuation increases or decreases
the liability of the person in Pak Rupees, then any increase or decrease in the tax liability, before full and final
repayment of the loan shall also be added to or deducted from the cost of the asset.

Notes:
▪ For the purpose of depreciation, difference on account of foreign currency fluctuation, if any shall be
taken into account in the year of occurrence.
▪ The terms and conditions of any hedging agreement shall be considered in determining whether the
exchange rate fluctuations have any effect on person’s liability.
▪ When an asset is acquired against foreign currency loan and as result of hedging agreement there is no
change in the liability of person in local currency terms, so there will be no change in the cost of asset.

264 | P a g e
Example:
Bill Limited acquired a foreign currency loan amounting to US$ 40,000 in the beginning of the year 2010. The
loan was repayable in eight half yearly installment of US$ 5,000. The loan was utilized for import of a plant
valuing equal to the amount of loan. At the time of obtaining the loan and importing the plant 1US$= 100 PKR.
The company incurred the following expenses in connection with the plant:

Insurance and sea freight Rs. 20,000


Port expenses 5,000
Custom duty and excise duty on import 55,000
Inland freight and octroi charges 10,000
Expenses on installation of plant 10,000
After payment of four installments the exchange rate fluctuated and next two installments were paid @ Rs.
105 and Rs. 110, respectively per USD.
Required:
Compute the cost of the plant prior to and after the change in exchange rate. Also compute the depreciation
for the year 2010, 2011, and 2012 assuming the depreciation is charged at 15%.

Solution
Cost of plant prior to change (Tax year 2010)

Purchase price ( US $ 40,000 @ Rs. 100) Rs.


4,000,000
Add: Other Costs:
Insurance and sea freight 20,000
Port Expenses 5,000
Custom duty and excise duty 55,000
Inland freight and octroi 10,000
Installation expense 10,000
Total Cost 4,100,000

Depreciation for the year 2010 and 2011

Total Cost Rs.


4,100,000
Less: Initial Allowance ( 4,100,000 * 25% ) (1,025,000)
Depreciation ( 4,100,000 – 1,025,000) * 15% (461,250)
WDV at the year end 2,613,750

WDV at the beginning of year 2011 2,613,750


Depreciation for the year (2,613,750 * 15%) (392,063)
WDV at the year end 2,221,687

Cost of plant after change (Tax Year 2012)

WDV at the Beginning of the year 2,221,687


Add: increase in liability due to change in exchange rate
At 5th Installment ( $ 5,000 * Rs. 5 ( 105-100) 25,000
At 6th Installment ( $ 5,000 * Rs 10( 110-100) 50,000
Revised WDV 2,296,687

Depreciation for the year (2,296,687 * 15%) (344,503)


WDV at the Year End 1,952,184

265 | P a g e
Asset acquired from any Subsidy, Etc.

The amount of the grant, subsidy, rebate, commission or any other assistance received or receivable in respect
of acquisition of an asset shall not be included in the cost of such asset. However where the amount of grant,
etc. is chargeable to tax under the ordinance then such amount shall also be included in the cost of the asset.

Example

A charitable institution received a grant of Rs. 150,000 for purchase of some equipment. The required asset was
purchased against Rs. 170,000. Compute the cost of the Asset in each of the following conditions;
1. The grant is not chargeable to tax
2. The grant is chargeable to tax.
Answer
1. Grant not chargeable to tax
Purchase value of equipment Rs. 170,000
Less: grant received for equipment (150,000)
Cost of the Asset 20,000

2. Grant chargeable to tax


In this case the cost of the asset will be Rs. 170,000 as the grant received is chargeable to tax.

Where Acquisition of an Asset is derivation of an Amount chargeable to tax/ Exempt to tax

• Where the acquisition of an asset by a person is the derivation of an amount chargeable


to tax, the cost of the asset shall be the amount so charged plus any amount paid by the person for the asset.
• Where the acquisition of an asset by a person is the derivation of an amount exempt from tax, the cost
of the asset shall be the exempt amount plus any amount paid by the person for the asset

Exercise
Burewala Express Limited (BEL) is in the business of manufacturing and sale of component parts
for automobile assembly industry. On 1 January 2022, BEL took a loan of US$ 500,000 from GHI
Bank, USA, which was utilized for purchasing the plant. The loan is repayable in 5 equal instalments
in US Dollars. The rate of exchange on 1 January 2022 was US$ 1 equal to Rs.196 and the loan
liability was recorded in the books of account of BEL at Rs. 98,000,000 (US$ 500,000 x Rs.196).
Other relevant information is as follows:
1. The Project was completed in June 2023, but was only commissioned for use on 31 July 2023.
The total amount spent by BEL on the plant was Rs.200,000,000
2. On 1 July 2023, the Government of Pakistan (GOP) voluntarily paid BEL Rs.10,000,000 as a
subsidy in respect of the plant installed in the Project.
3. The first instalment of US$ 100,000 towards repayment of the US Dollar loan was paid to GHI
Bank on 30 June 2024 when the exchange rate was US$ 1= Rs.198.
Required:
Calculate the initial allowance, depreciation and written down value of the plant on 30 June 2024
for preparing the tax return for tax year 2024

266 | P a g e
Solution
Description Note Amount
Cost of the plant 200,000,000
Subsidy 1 (10,000,000)
Exchange fluctuation 2 200,000
Cost of the plant 190,200,000
Less: Initial allowance @ 25% (47,550,000)
Written down value 142,650,000
Less: Depreciation @ 15% (21,397,500)
Written down value 121,252,500
Notes:
N-1:
In determining the cost of an asset for tax purposes the actual amount spent by a person in acquiring an
asset is required to be reduced by the amount of any grant, subsidy, rebate, commission or any other
assistance received or receivable by the person in respect of the acquisition of the asset except where the
said amount received is chargeable to tax . Further the amount of Rs. 10 million
is not income for tax purpose but is a capital receipt on the grounds that
(a) The amount was voluntarily paid by GOP without any consideration
(b) The company did not ask for the subsidy
(c) Amount received did not arise out of any legal or contractual obligation
(d) The amount is not traceable nor even remotely connected to any source of income
N-2:
An amount of Rs. 200,000 will be added to the cost of the asset due to the depreciation of the Pakistani
Rupee from Rs. 98 to Rs. 100 against the dollar.
Where a person has acquired an asset with a foreign currency loan (repayable in foreign currency) and
before the loan is fully repaid, there is an increase or decrease in the loan liability of the person in terms of
Pakistan rupees, due to a change in the rate of exchange of the foreign currency, the amount by which the
liability has increased or decreased is to be added to or reduced from the cost of the asset. In other words,
the cost of the asset acquired with the foreign currency loan is recomputed for tax purposes

Cost of leased Asset


Cost of depreciable asset acquired by the lessee on maturity or pre-mature termination of finance lease
agreement is the residual value or bargain purchase price as the case may be.

Question
Motor vehicle Rs.
Cost at the time acquired 1,000,000
Finance obtained under finance lease 1,000,000
Bargain purchase price:
Before payment of 7th installment 1,000,000
After payment of 7th but before payment of 11th installment 750,000
After payment of 11th but before payment of 17th installment 500,000
After payment of 17th but before payment of 22th installment 250,000
Residual value on maturity of lease 70,000
Monthly lease rentals 60,000
No. of installments 22

267 | P a g e
Solution

Cost of motor vehicle for the purpose of depreciation deduction


Before payment of 7 installments 1,000,000
After paying 7 installments 750,000
After paying 11 installments 500,000
After paying 17 installments 250,000
On maturity of lease i.e. after paying 22 installments 70,000

Consideration Received

268 | P a g e
24.Intangibles.

Definitions u/s 24

Cost in relation to an intangible, means any expenditure incurred in acquiring or creating the intangible,
including any expenditure incurred in improving or renewing the intangible; and

Intangible means any patent, invention, design or model, secret formula or process, copyright 1icensce, trade
mark, scientific or technical knowledge, computer software, motion picture film, export quotas, franchise,
license, intellectual property, or other like property or right, contractual rights and any expenditure that provides
an advantage or benefit for a period of more than one year (other than expenditure incurred to acquire a
depreciable asset or unimproved land).

Examples:

1. Administrative expenditures include Computer software of Rs. 2,800,000. The software was acquired
on 25 February 2016 with an estimated useful life of 4 years. Special year ended September.
(Answer: Add back Rs.2.8 million in profit and loss and deduct Rs.418, 082 (2,800,000 ÷4) x (218/365))
2. Administrative expenditures include Rs. 5,000,000 being the cost of a right to use a formula for the
development of a new chemical compound. TL obtained the rights on 1 March 20X5 from High Tec
Inc. USA for twelve years. Special year ended December.
(Answer: Add back Rs.5 million in profit and loss and deduct Rs. 37,742,000= (5,000÷10×306 ÷365))
Amortization

1) A person shall be allowed an amortization deduction in accordance with this section in a tax year for
the cost of the person‘s intangibles–
i. that are wholly or partly used by the person in the tax year in deriving income from
business chargeable to tax; and
ii. that have a normal useful life exceeding one year.
2) No deduction shall be allowed under this section where a deduction has been allowed under another
section of this Ordinance for the entire cost of the intangible in the tax year in which the intangible is
acquired.
3) The amortization deduction of a person for a tax year shall be computed as
𝐶𝑜𝑠𝑡 𝑜𝑓 𝐼𝑛𝑡𝑎𝑛𝑔𝑖𝑏𝑙𝑒
𝐴𝑚𝑜𝑟𝑡𝑖𝑧𝑎𝑡𝑖𝑜𝑛 = 𝑥 𝑁𝑜. 𝑜𝑓 𝑑𝑎𝑦𝑠 𝑢𝑠𝑒𝑑 𝑖𝑛 𝑡ℎ𝑒 𝑦𝑒𝑎𝑟
𝑌𝑒𝑎𝑟𝑠 𝑥 365

269 | P a g e
4) Where an intangible does not have an ascertainable useful life then the same shall be treated as 25
years.
5) Where an intangible is used in a tax year partly in deriving income from business chargeable to tax
and partly for another use, the deduction allowed under this section for that year shall be restricted to
the fair proportional part of the amount that would be allowed if the intangible were wholly used to
derive income from business chargeable to tax.
Where, in any tax year, a person disposes of an intangible, no amortization deduction shall be allowed under
this section for that year and gain/loss shall be computed as = Consideration – WDV
(Written down value = Cost – Accumulated amortization)

25. Pre-commencement expenditure.

Definition
pre-commencement expenditure means any expenditure incurred before the commencement of a business
wholly and exclusively to derive income chargeable to tax, including the cost of feasibility studies, construction
of prototypes, and trial production activities, but shall not include any expenditure which is incurred in acquiring
land, or which is depreciated or amortized under section 22 or 24.
Example
Cost of sales included Rs. 80,000/- incurred for test run.
(Answer: Add back Rs.80, 000 profit and loss and deduct Rs. 16,000 (80,000 x 20%))

1. A person shall be allowed a deduction for any pre-commencement expenditure in accordance with this
section.
2. Pre-commencement expenditure shall be amortized on a straight-line basis @ 20% per annum.
3. The total deductions allowed under this section in the current tax year and all previous tax years in
respect of an amount of pre-commencement expenditure shall not exceed the amount of the expenditure.
4. No deduction shall be allowed under this section where a deduction has been allowed under another
section of this Ordinance for the entire amount of the pre-commencement expenditure in the tax year in
which it is incurred.

270 | P a g e
PAST PAPER QUESTIONS
Question 1
Define the term 'Intangibles'.

Question 2
Sun & Moon have recently registered as partnership. They have incurred the following expenditure:
• Fees paid to consultants for preparation of registration deed Rs.50,000
• Preparation of feasibility report Rs.100,000
• Purchase of office equipment Rs.150,000
• Purchase of machinery Rs.1,000,000
• Trial run cost Rs.200,000
• Installation cost Rs.50,000
Required:
You are required to explain the tax treatment by computing the amount allowable as deduction in
accordance with the provisions of Income Tax Ordinance 2001.

Question 3
Define the term 'Pre-commencement expenditure' in the light of Income Tax Ordinance, 2001.

Question 4
List down the assets on which 'Initial allowance' cannot be claimed.

Question 5
What are the prescribed rates of normal depreciation on the following assets as per the Third Schedule to
the Income Tax Ordinance, 2001?
i. Factory building.
ii. Residential quarter for labor.
iii. Furniture.
iv. Plant and machinery.
v. Computer and hardware.
vi. Technical books.
vii. New ships.
viii. Motor vehicle.

Question 6
Discuss the common rules with regard to 'Non-arms' length transactions of disposal of asset under the
Income Tax Ordinance, 2001.

Question 7
Define the term 'Depreciable Assets' with reference to the Income Tax Ordinance, 2001.

Question 8
During the tax year 20X9, AA Enterprise disposed of the following assets:
i.An immovable property was sold for Rs.200 million. The cost of immovable property was Rs.100 million. Up
to tax year 20X8, tax depreciation of Rs.10 million had been allowed on the immovable property.
ii.A plant was exported to Nepal. The export proceeds amounted to Rs.28 million. The cost and written down
value of the plant was Rs.25 million and Rs.18 million respectively.
iii.Three trucks were disposed of for Rs.2.5 million. They were acquired in tax year 20X8. The tax written
down value of trucks at the beginning of tax year 20X9 was Rs.2.4 million. The trucks were being used partly
i.e. 60% for business purposes. Assume that the rate depreciation for tax purposes is 20% [ignore initial
allowance].

271 | P a g e
Required:
Compute the tax gain or loss on disposal of each of the above assets.

Question 9
In the context of Income Tax Ordinance 2001, discuss the rules relating to claiming of amortization
deduction on intangibles.

Question 10
Gulzar is a Pakistani resident and operates various businesses.
He disposed of the following assets during the tax year 20X4:
a)An immovable property was sold for Rs.50 million. The cost of the immovable property was Rs. 25 million.
Tax depreciation of Rs. 4 million had been allowed on the immovable property up to the tax year 20X3.

b) A car was disposed of for Rs.1.2 million. The car was acquired on 1.7.20X2. The tax written down value of
the car at the beginning of tax year 20X4 was Rs.0.9 million. The car was being used partly (70%) for business
purposes. Assume that the rate of depreciation for tax purposes is 20%.

c) An antique sculpture was purchased for Rs.350,000 on 30.8.20X1. It was sold for Rs.1,500,000 on
28.2.20X4 through auction. The auctioneer was paid a commission Rs.150,000. Tax was deducted and paid
by Gulzar from the amount of commission within due date.
Required:
Compute capital gain or loss on the disposal of the specified transactions.

Question 11
Sarwar Enterprises sold an immovable property for Rs.50 million. The cost of the immovable property was
Rs.30 million. Tax depreciation of Rs.6 million had been allowed on the immovable property up to the tax
year 20X6.Under the provisions of the Income Tax Ordinance, 2001 compute taxable income or loss, under
the correct head of income for tax year 20X7.

Question 12
Shams Industries Ltd (SIL) sold and exported one of its plants to a Nigerian Company. The sa proceeds
received in SIL's account amounted to Rs.25 million. The cost and tax WDV of the plant was Rs.20 million and
Rs.7 million respectively.
Under the provisions of the Income Tax Ordinance, 2001 compute taxable income or loss, under the correct
head of income for tax year 20X7.

Question 13
Under the provisions of the Income Tax Ordinance, 2001 compute taxable income or loss under correct head
of income for tax year 20X8, in the following case:
•Shaoor is the sole proprietor of Shaoor Enterprises (SE). On 31.1.20X8 SE sold a factory building including
land for Rs.10 million. At the time of disposal, the fair market values of the land and building were Rs.3
million and Rs.5 million respectively.
•The land and building were acquired on 1.7.20X6 at a cost of Rs.2 million and Rs.6 million respectively. The
tax WDV of the building on 1.7.20X7 was Rs.5.4 million.

Question 14
Following transaction pertains to S Ltd (SL) which took place during the tax year 20X9: A machine costing
Rs.1,800,000, being used in SL's Karachi factory was transferred to its subsidiary in Ghana. The fair market
value and tax written down value of the machine on the date of transfer were Rs.2,500,000 and Rs.600,000
respectively.
Required:

272 | P a g e
Explain the taxability of the above transaction.

Question 15
Sikandar has revalued his factory building in accordance with International Financial Reporting Standards
and consequently charged depreciation on the revalued amount.
Explain the tax implication of the revaluation.

Question 16
Shahbaz has acquired machinery for his new factory against a loan repayable in USD.
Discuss what would be the cost of machinery for the purpose of depreciation deduction.

Question 17
During the tax year 20X9, Salman Shahid sold the following assets:
a)A vehicle used by manager-in-charge of his garment factory for Rs.2.8 million. The vehicle was purchased
for Rs.3.1 million in tax year 20X6 (maximum allowable cost was Rs.2.5 million.
b)A machine for Rs.350,000 on 1.6.20X9, which he had imported from Malaysia for Rs.1,900,000 on
1.5.20X9, to start a new business. The machine was badly damaged during the shipment from Malaysia,
rendering it unfit for use. He received insurance claim of Rs.1,840,000 as damages on 15.5.20X9. Charges
incurred in connection with the submission of claim with insurance company were Rs.38,000.
Required:
Under the provisions of the Income Tax Ordinance, 2001 compute under the appropriate head of income,
the amount to be included in the taxable income of Salman Shahid for the tax year 20X9.

Question 18
For the purpose of this part of the question, assume that the date today is 31.8.2024.During the year ended
30.6.2024, Faster & Co. (FC) started a new project.
Following information is available:
• Incurred Rs.5 million on feasibility study of the project.
•Obtained a 3% loan of AED 2 million from a UAE bank on 1.1.2024 for the purchase of plant and machinery.
The interest is payable annually and principal amount is repayable at the end of third year.
•Installed the plant and machinery at a cost of Rs.150 million on 14.3.2024.
The exchange rates of 1 AED to PKR on different dates are as follows:
•Rs.50 on 1.1.2024
•Rs.55 on 30.6.2024
•Rs.53 average between 1.1.2024 to 30.6.2024
Required:
Compute the amount of allowable deduction in determining the taxable income of FC for tax year 2024.

273 | P a g e
ANSWERS
Answer to Q.1
'Intangible' means any patent, invention, design, secret formula, copyright, trade mark, scientific or technical
knowledge, computer software, motion picture film, export quota, franchise license or like property or right
and any expenditure that provides an advantage for a period of more than one year.
Self-generated goodwill shall not be treated as intangible asset.

Answer to Q.2
The amount of tax deduction that can be claimed by Sun & Moon is as under:
Fee paid for preparation of registration deed: Inadmissible
Pre-commencement expenditures amortized over 5 years
Preparation of feasibility report 100,000
Trial run cost 200,000
300,000
Amortization @ 20% 60,000
Normal tax dep. on office equipment @ 15% of Rs.150,000 22,500
Cost of machinery 1,000,000
Installation cost 50,000
Total cost of machinery 1,050,000
Initial allowance on machinery @ 25% 262,500
Normal tax dep. on machinery @ 15% of (1,050,000-262,500) 118,125
463,126

Answer to Q.3
'Pre-commencement expenditure' means any expenditure incurred before the commencement of business (i.e.
before the commencement of commercial production) exclusively for business purpose including the cost of
feasibility studies, prototypes and trial production activities (net of sale proceed of trial production, if any) but
excluding cost of land, any expenditure incurred before the depreciable assets and intangibles.

Answer to Q.4
Initial allowance is allowable @ 25% of the cost of asset other than the following:
• Immovable property or structural improvement to immovable property
• Road transport vehicle not plying for hire
• Furniture and fittings
• Any plant and machinery that has been used previously in Pakistan
• Assets allowed as tax expense

Answer to Q.5
The prescribed rates of normal tax depreciation as per 3rd Schedule are:
Asset Rate
Factory building 10%
Residential quarters for labor 10%
Furniture 15%
Plant and machinery 15%
Computer and hardware 30%
Technical books 15%
New ships 15%
Motor vehicle 15%

Answer to Q.6
If an asset is disposed of in a non-arms' length transaction then the Fair Market Value at the time of disposal
shall be treated as its sale proceed. The purchaser shall be treated to have acquired the said asset at the same
FMV and not at the cost paid by him.

274 | P a g e
Answer to Q.7
Depreciable asset means any tangible property or structural improvement (excluding cost of unimproved land)
owned by a person that:
✓ has a normal useful life exceeding one year;
✓ is likely to lose value as a result of normal wear and tear, or obsolescence; and
✓ is used wholly or partly for taxable business purpose.
Structural improvement includes any building, road, driveway, car park, railway line, pipeline, drainage,
bridge, tunnel etc.

Answer to Q.8
(i) Disposal of immovable property
Sale proceed 200
Deemed cost of immovable property 200
Less: Tax depreciation allowed 10
Tax WDV 190
Tax gain on disposal of immovable property 10

Where sale proceed of an immovable property being depreciable asset exceeds its cost then the sale proceed
shall be treated as the cost of such immovable property.

(ii) Export of plant


Sale proceed 25
Less: Tax WDV 18
Gain on export of plant 787

Where a depreciable asset is exported or transferred outside Pakistan then sale proceed shall be equal to its
original cost.

(iii) Disposal of trucks


Cost of trucks in the tax year 20X8 2.4/80% 3.00
Tax depreciation calculated @ 20% 0.60
Tax WDV at the beginning of tax year 20X9 2.40
Tax depreciation disallowed on account of non-business use 0.24

Tax WDV 2.40


Add: Depreciation disallowed in the tax year 20X8 0.24
Total tax WDV 2.64
Sale proceed 2.50
Tax loss on disposal of trucks 0.14

Answer to Q.9
1. Where an intangible having useful life of more than one year is used for business purpose,
amortization deduction in a tax year is allowable as under:
Cost of intangible
𝑁𝑜𝑟𝑚𝑎𝑙 𝑢𝑠𝑒𝑓𝑢𝑙 𝑙𝑖𝑓𝑒 𝑖𝑛 𝑤ℎ𝑜𝑙𝑒 𝑦𝑒𝑎𝑟𝑠
Where an intangible does not have an ascertainable useful life then the same shall be treated as 25 years.
2. If an intangible is used partly for business purpose and partly for other use in a tax then amortization
deduction shall be allowed proportionately.
3. If an intangible is not available for use for the whole tax year then amortization deduction shall be
calculated proportionately based on number of days available for use divided by number of days in
the tax year.

275 | P a g e
4. Amortization deduction is not allowed in the year of disposal of intangible. Any gain or loss on such
disposal shall be considered for tax purpose and shall be calculated by deducting tax WDV from
consideration received.

Answer to Q.10
Capital gain or loss on the disposal of specified transactions are as under:
a. Immovable property being a depreciable asset:
The definition of capital asset excludes certain assets and therefore gain or loss on disposal of such excluded
assets does not fall within the ambit of capital gain. A depreciable asset inter alia is in the list of exclusions
and therefore disposal of a depreciable immovable property is not taxable under the head capital gain.
Moreover, section 22(8) specifically provides that gain on disposal of a depreciable asset is taxable under the
head income from business.
b. Car being a depreciable asset:
The definition of capital asset excludes certain assets and therefore gain or loss on disposal of such excluded
assets does not fall within the ambit of capital gain. A depreciable asset inter alia is in the list of exclusion and
therefore disposal of a depreciable car is not taxable under the head capital gain.
Moreover, section 22(8) specifically provides that gain on disposal of a depreciable asset is taxable under the
head income from business.
c. Antique:
Consideration received 1,500,000
Less: Cost 350,000
Commission paid 150,000 500,000
Taxable capital gain 1,000,000

Answer to Q.11
Sale proceed 50,000,000
Deemed cost of immovable property 50,000,000
Less: Tax depreciation allowed 6,000,000
Tax WDV 44,000,000
Tax gain on disposal of immovable property 6,000,000
Where sale proceed of an immovable property being depreciable asset exceeds its cost then the sale proceed
shall be treated as the cost of such immovable property.

Answer to Q.12
Sale proceed 20,000,000
Less: Tax WDV 7,000,000
Gain on export of plant 13,000,000
Where a depreciable asset is exported or transferred outside Pakistan then sale proceed shall be equal to its
original cost.

Answer to Q.13
Note 1:
Sale proceed of two or more assets disposed of in a single transaction is apportioned in proportion to their
FMV at the time of disposal as under:
Sale value of land and building 10,000,000
FMV at the time of disposal:
Land 3,000,000
Building 5,000,000
Therefore sales proceed shall be taken:
Land 10 x 3/8 3,750,000
Building 10 x 5/8 6,250,000
Note 2:
Land is not a depreciable asset and therefore any gain on disposal within specified period is taxable under the
head capital gain as a separate block of income at specified tax rate.
Note 3:

276 | P a g e
Gain on disposal of any depreciable building is taxable under the head income from business. Where the
consideration received of a depreciable building exceeds its original cost, the consideration received shall be
treated as cost of the property.
Shaoor Enterprises
Tax year 20X7
Computation of Taxable Income
Income from Business
Sale proceed of building 6,250,000
Cost equal to sale proceed 6,250,000
Less: Tax depreciation 6,000,000 - 5,400,000 600,000
Tax WDV 5,650,000
Tax gain on disposal of building taxable under NTR 600,000

Capital Gain
Sale proceed of land 3,750,000
Less: Cost of land 2,000,000
Taxable capital gain as separate block of income 1,750,000

Answer to Q.14
If a depreciable asset is exported or transferred out after use in Pakistan then its original cost shall be taken as
its sale value instead of actual sale value or FMV.
Therefore, the gain on disposal of the machine transferred to its subsidiary in Ghana shall be worked out as
under:
Rs.
Consideration received (equal to cost) 1,800,000
Tax WDV at the time of disposal 600,000
Tax gain on disposal 1,200,000

Answer to Q.15
Tax laws in Pakistan do not consider revaluation effects of assets. Consequently, depreciation will be allowed
on tax WDV of building without taking into account the effect of revaluation.

Answer to Q.16
The machine has been purchased with a loan repayable in foreign currency. The exchange difference, if any,
in repayment of loan in the year of occurrence shall be added to or reduced from the cost/ WDV of the
machine, as the case may be. Cost of hedging arrangement, if any, with the State Bank of Pakistan to avoid
exchange rate fluctuation shall be included in the cost of machine.

Answer to Q.17
(i) Disposal of Vehicle
Cost Rs. 3,100,000
Cost restricted to 2,500,000
Less: Tax depreciation in 20X6 @ 15% of cost 375,000
Tax depreciation in 20X7 @ 15% of WDV 318,750
Tax depreciation in 20X8 @ 15% of WDV 270,937 964,687
Tax WDV at the beginning of 20X9 1,535,313
Less: Proportionate sale proceed 2.5/3.1 x 2,800,000 2,258,065
Tax gain on disposal of vehicle under business income 722,752

(ii) Disposal of Machine


Insurance claim 1,840,000
Scrap value 350,000
Total consideration received 2,190,000
Less: Cost of import 1,900,000
Insurance claim charges 38,000 1,938,000

277 | P a g e
Capital gain 252,000

Note:
Machine was not a depreciable asset in this case as the same was not commissioned for use. The gain on
disposal is therefore taxable under the head capital gain.

Answer to Q.18
The amount of allowable deductions is as under:
Amortization of feasibility study being pre-commencement expenditure
20% of Rs.5 million 1,000,000
Profit on debt AED 2 million @ 3% for 6 months = 30,000.x Rs.53 1,590,000
Initial allowance on plant Rs.150 million x 25% 37,500,000
[Interest cost is not a part of cost of plant and it is allowable as revenue expenditure.
Whereas exchange loss is included in the cost of plant in the year of
occurrence. No instalment of loan is paid in the current year
and therefore no exchange loss is recognized for tax purposes.]
Normal tax depreciation 15% of (Rs.150 million - 37.5 million) 16,875,000
Total 56,965,000

278 | P a g e
CHAPTER 12
BUSINESS INCOME: OTHER
ASPECTS INCLUDING TURNOVER
TAX
26. Scientific research expenditure.

(1) A person shall be allowed a deduction for scientific research expenditure incurred in Pakistan in a tax year
wholly and exclusively for the purpose of deriving income from business chargeable to tax.

Definitions
Scientific research means any activity undertaken in Pakistan in the fields of natural or applied science for the
development of human knowledge;

Scientific research expenditure means any expenditure incurred by a person on scientific research undertaken
in Pakistan for the purposes of developing the person‘s business, including any contribution to a scientific
research institution to undertake scientific research for the purposes of the person‘s business, other than
expenditure incurred –
(a) in the acquisition of any depreciable asset or intangible;
(b) in the acquisition of immovable property; or
(c) for the purpose of ascertaining the existence, location, extent or quality of a natural deposit; and

Scientific research institution means any institution certified by the Board as conducting scientific research in
Pakistan.

27. Employee training and facilities.


A person shall be allowed a deduction for any expenditure (other than capital expenditure) incurred in a tax year
in respect of—
(a) any educational institution or hospital in Pakistan established for the benefit of the person‘s employees and
their dependents;
(b) any institute in Pakistan established for the training of industrial workers recognized, aided, or run by the
Federal Government or a Provincial Government or a Local Government; or
(c) the training of any person, being a citizen of Pakistan, in connection with a scheme approved by the 4Board
for the purposes of this section

Interest expense, Lease rentals and Special purpose vehicle - Section 28


a) Followings are tax deductible from business income if related to taxable business income:
b) Profit on debt if the debt is utilized for business purpose (for capital expenditure as well as for working
capital requirement) However, exchange loss if any is a capital expenditure in respect of a loan in
foreign currency utilized for capital expenditure.
c) Share of profit under musharika scheme to a bank
d) Financial cost of securitization of receivable to Special Purpose Vehicle (SPV) by an originator
e) Profit on debt on Term Finance Certificates or other interest-bearing certificates issued by a company
Assets taken on lease:
Tax depreciation on leased assets is not admissible instead lease rentals to the approved financial Institution,
modaraba or leasing company or SPV on behalf of an originator are allowable tax expense for the lessee where
the asset is used for business purpose.

279 | P a g e
In case of finance lease, accounting depreciation and finance charge shall be added back to the purpose
accounting profit and the lease rentals shall be deducted to arrive at taxable income.
Security/initial deposit paid in respect of lease is not allowable tax expenditure. However, if the security deposit
is forfeited on transfer of asset to the lessee then the said initial deposit additional payment, if any, shall be
considered as the cost of asset for the lessee for the, of tax depreciation.

Lease of passenger transport vehicle not plying for hire.


Lease rentals to the extent of principal amount shall not exceed Rs.2.5 million in respect of lease If cost of a
passenger transport vehicle not plying for hire.

Example:
A Ltd, a manufacturing company acquired a car for its chief executive from a leasing company. Cost of the car
is Rs.3 million.
Lease rentals for the tax year paid to the leasing company Rs.900,000 including repayment of principal amount
Rs.675,000.
Calculate the amount of lease rentals that can be claimed as allowable deduction.
Solution Lease Rentals allowed as deduction are 787,500

Question
You are a tax partner in a local firm of Chartered Accountants. You have been approached by some of the clients
for opinion on the possible tax implication/treatment of certain matters.
Under the provisions of the Income Tax Ordinance, 2001 and Rules made thereunder, discuss the tax
implication/treatment in the following independent case:
On 1.9.20X6 Mega (Pvt) Ltd (MPL), engaged in the business of manufacturing plastic bottles, had obtained a
machinery on finance lease. For the year ended 30.9.20X9 MPL charged interest of Rs.228,000 to the profit and
loss account on account of such finance lease. Total lease rentals paid during the year amounted to Rs.475,000.
At the end of the lease term which expired on 31.8.20X9, the machinery was transferred to MPL at a residual
value of Rs.764,000. The market value of the machinery on the date of its transfer amounted to Rs.868,000.

280 | P a g e
Answer
In case of a finance lease, accounting depreciation and interest charged to the accounts are inadmissible
deductions. However, the lease rentals of Rs.475,000 are admissible deduction.
After transfer of machinery to MPL, tax depreciation would be admissible at residual value of Rs.764,000.
For the purpose of tax depreciation, the residual value of the machinery (and not its market value) shall be
treated as its tax WDV, as residual value is the consideration that was paid by MPL.
The machinery would not be eligible for initial allowance as it was already in use of MPL.

Special Purpose Vehicle (SPV)


• A public company having the prescribed amount of paid up capital, a trust or a body corporate may be
registered as SPV with SECP for the purpose of Securitization of receivables under the Companies
(Assets Backed Securitization) Rules.
• An Originator including a leasing company and modaraba may transfer its receivables to SPV in
consideration of a mutually agreed payment by SPV who is entitled to collect the receivables.
• Lease rentals of an asset, used by a lessee expense for his taxable business income, to SPV on behalf of
an originator is allowable tax expense for the lessee.
• Financial cost of securitization of receivables by an Originator in respect of SPV is an allowable tax
expense.
• Payment by SPV to an Originator in respect of securitization of receivables shall not be subject to tax
deduction at source.
• Income of special purpose vehicle is exempt if at least 90% of accounting profit other than capital gain
is distributed as dividend other than issue of bonus shares.

29. Bad debts.


Conditions to claim Bad Debts
A person shall be allowed a deduction for a bad debt in a tax year if the following conditions are satisfied,
namely:—
(a) the amount of the debt was –
(i) previously included in the person‘s income from business chargeable to tax; or
(ii) in respect of money lent by a financial institution in deriving income from business
chargeable to tax;
(b) the debt or part of the debt is written off in the accounts of the person in the tax year; and
(c) there are reasonable grounds for believing that the debt is irrecoverable.
Bad Debts allowed recovered
Where a person has been allowed a deduction in a tax year for a bad debt and in a subsequent tax year the person
receives in cash or kind any amount in respect of that debt, the following rules shall apply, namely:–

Actual bad debts < Bad debts allowed


(a) where the amount received exceeds the difference between the whole of such bad debt and the amount
previously allowed as a deduction under this section, the excess shall be included in the person‘s income
under the head ―Income from Business‖ for the tax year in which it was received; or

Actual bad debts > Bad debts allowed


(b) where the amount received is less than the difference between the whole of such bad debt and the amount
allowed as a deduction under this section, the shortfall shall be allowed as a bad debt deduction in computing
the person‘s income under the head ―Income from Business‖ for the tax year in which it was received.

Example:
Good Debts Enterprises (GDE) made a sale of Rs.100/- to Bad Debts Ltd. (BDL) in tax year 2018. This sale
was shown in the taxable income of 2018. In 2019, DGE received nothing from BDL and booked it as bad debts
in its books of accounts and claim bad debts from tax department. Tax department only allowed Rs.60/- as bad

281 | P a g e
debts by assuming that Rs.40/- will be recovered from the BDL. Following are the possible situations and their
tax treatment in 2020

Situation Actual Receipts Actual Bad debts Bad debts Tax Treatment (Rupees)
from BDL (Rupees) allowed (Rupees)
(Rupees)
1 2 3 4 5 = 3-4
A 10 90 (100-10) 60 30 Expenses
B 30 70 (100-30) 60 10 Expenses
C 40 60 (100-40) 60 No Treatment
D 60 40 (100-60) 60 20 Income
E 80 20 (100-80) 60 40 Income
F 100 NIL (100-100) 60 60 Income

To do Question
Ms. Shagufta is running a business in the name of Al Nafay Business Solutions. In the tax year 2023,
she claimed bad debts of Rs. 1,000,000 and Rs. 1,500,000 from its clients Mr. Junaid and Mr.
Nawaz. She was allowed deduction of bad debts of Rs. 750,000 and Rs. 800,000 with respect of
receivable from Mr. Junaid and Mr. Nawaz in Tax year 2023. During 2024, she received following
sums from these two debtors:
Mr. Junaid Rs.900,000
Mr. Nawaz Rs. 500,000
Work out the amount to be added/allowed on account of bad debts in the tax year 2024.

Method of accounting: Section 32 and stock in trade


a) There are two methods of accounting; accrual-basis accounting (also called mercantile accounting
system) and cash-basis accounting.
b) A company is required to follow accrual-basis accounting. A business entity other than a company may
follow cash-basis accounting or accrual-basis accounting at its discretion regularly applied.
c) Change in the method of accounting:
A person may apply for a change in the method of accounting and the Commissioner may, by order
in writing, approve such an application but only if satisfied that the change is necessary to clearly
reflect the person's taxable business income.
If a person's method of accounting has changed, the person shall make adjustments to items of
income, deduction, credit or any other items affected by the change so that no item is omitted and
no item is taken into account more than once.

d) The FBR has authority to prescribe any specific manner of maintaining books for any particular business
or profession.
e) Accrual-basis accounting:
• Income is taxable when it is due to the person i.e. when the person is entitled to receive it even
if the time for discharge of the entitlement is postponed or the amount is payable by
installments.
• An expenditure is allowable when it is payable by the person i.e. when all the events have
occurred that determine liability and the amount can also be determined with reasonable
accuracy.
Notes for students:
Tax treatment of provision for slow moving stock, wastage, discount or warranty is the same as in case of provision
for doubtful debts.
If liability is determined against a provision, then it would be allowed as tax expense such as provision for bonus
to employees.

282 | P a g e
• An expenditure period of 3 from the end of the basis accounting is required to be paid within a period
of 3 years from the end of the tax year in which it was allowed.
• Any unpaid liability against such expense shall be chargeable to tax in the 4th year. However, if the said
amount is paid subsequently then it shall be allowed as a tax deduction. This is usually termed as trading
liability outstanding for more than 3 years.
- This provision is not applicable in the following cases:
- Where the debit side of a liability is not a tax expense e.g. loan payable or advance from
customers or where the expenditure was disallowed under any provision of the Income Tax
Ordinance, 2001 such as penalty for the violation of any law, rule or regulation.
- Where the expense was against exempt income therefore the expense was not allowed as tax
expense.
- Where the expense was against income taxable under FTR and the expense was not allowable
as tax expense e.g. exports
- Where the expense was disallowed by the tax department after conducting tax audit etc.
35. Stock-in-trade.
Cost of sales shall be computed as follows:

opening value of the person ‘s stock-in-trade


Add: cost of stock-in-trade acquired
Less: closing value of stock-in-trade

Valuation of Closing Stock


Cost or NRV
Closing stock shall be computed at cost or NRV whichever is lower.
Absorption Cost Method / Prime Cost Method
• Prime cost or absorption cost method may be adopted by a person who is preparing its accounts
on cash basis and
• Absorption cost method accounting shall be adopted by a person who is preparing its accounts
on accrual basis.
FIFO / Weighted Average
Where particular items of stock-in-trade are not readily identifiable, a person may account for that stock on the
first-in-first-out method or the average-cost method but, once chosen, a stock valuation method may be changed
only with the written permission of the Commissioner and in accordance with any conditions that the
Commissioner may impose.

Where the person commenced to carry on business in the year, the fair market value of any stock-in-trade
acquired by the person prior to the commencement of the business.

Definitions u/s 35
Absorption-cost method means the generally accepted accounting principle under which the cost of an
item of stock-in-trade is the sum of direct material costs, direct labour costs, and factory overhead costs;
Average-cost method means the generally accepted accounting principle under which the valuation of
stock-in-trade is based on a weighted average cost of units on hand;
Direct labor costs means labor costs directly related to the manufacture or production of stock-in-trade;
Direct material costs means the cost of materials that become an integral part of the stock-in-trade
manufactured or produced, or which are consumed in the manufacturing or production process;
Factory overhead costs means the total costs of manufacturing or producing stock-in-trade, other than
direct labour and direct material costs;
First-in-first-out method means the generally accepted accounting principle under which the valuation of
stock-in-trade is based on the assumption that stock is sold in the order of its acquisition;

283 | P a g e
Prime-cost method means the generally accepted accounting principle under which the cost of stock-in-
trade is the sum of direct material costs, direct labour costs, and variable factory overhead costs;
Stock-in-trade means anything produced, manufactured, purchased, or otherwise acquired for
manufacture, sale or exchange, and any materials or supplies to be consumed in the production or
manufacturing process, but does not include stocks or shares; and
Variable factory overhead costs means those factory overhead costs which vary directly with changes in
volume of stock-in-trade manufactured or produced.

Method of Accounting

Individual / AOP Companies

Cash Basis Accrual Basis

Prime Cost
Absorption Cost

Closing Stock Valuation

Concept of small company


Small company has been defined to mean a company registered under the Companies Act
on or after 1.7.2005 which:
a) has paid-up capital plus undistributed reserves not exceeding Rs.50 million;
b) has employees not exceeding 250 at any time during the year;
c) has annual turnover not exceeding Rs.250 million;
d) is not formed by the splitting up or the reconstitution of company already in existence; and
e) is not a small and medium size enterprise.

▪ Benefit for a small company


Income tax rate of a small company is 20% of taxable income

Women enterprise and its taxation


Definition
woman enterprise means a start-up established on or after 1/7/2021
• As sole proprietorship concern owned by a woman or
• An AOP all of whose members are women or
• A company whose 100% shareholding is held or owned by women
Taxation
Tax payable by women enterprise on profits and gains derived from business chargeable to tax under the head
income from business shall be reduced by 25%,
However, this benefit will not be available to business that is formed by the transfer or reconstitution or splitting
up of an existing business.

284 | P a g e
Minimum tax regime
Turnover is defined as
A) Gross sales or gross receipts, exclusive of
i. Sales Tax and Federal Excise duty or
ii. any trade discounts shown on invoices,
iii. and also excluding sales / receipts taxable under FTR
B) Gross fees for the rendering of services, commissions and gross receipts from contracts; except covered
under FTR
C) the company‘s share of the amounts stated above of any association of persons of which the company is a
member.
[Note: Early Payment discount shall not be excluded from turnover & receipts from all business activities includes sales
of immoveable property taxable under business Income]

Certain types of incomes are subject to minimum tax under the ITO, 2001 to assure that certain portion of tax
is paid by the taxpayer.
Various incomes which are treated as minimum tax under the Income Tax Ordinance, 2001 are:
1. Minimum tax under section 113 on turnover.
2. Commercial importer under section 148(7).
3. Tax deduction at source @ 11% from gross amount of service rendered under section 153 for all persons
other than a company.
4. Tax collected upto the electricity bill amount of Rs.360,000 per annum for a person other than company
under section 235.

➢ Under the minimum tax regime, tax already deducted at source is treated as minimum tax which is then
compared with liability under the normal tax regime.
➢ In case liability under normal tax regime is greater than tax already deducted, the person is required to
pay liability under normal tax regime.
➢ In case tax already deducted is greater than liability under normal tax regime, then same is treated as
final tax.

Minimum tax on the income of certain persons [Sec 113] or turnover taxation
This section shall apply to
➢ A resident company,
➢ An individual or an AOP (having turnover of Rs. 100 million or above in the tax year 2017 or in any
subsequent tax year) and
➢ Permanent establishment of a non-resident company

• The above entities shall pay tax @ 1.25% of turnover if their tax liability is nil or less than 1.25% of
turnover.
• Even if the income of the business entity is exempt from income tax or no tax is otherwise payable on
account of loss for the tax year, brought forward loss, tax credit, depreciation etc.

Reduced rates of minimum tax have been specified in certain cases such as:-
• 0.75% for Oil marketing companies and poultry farming
• 0.25% for distributors of pharmaceutical products, FMCGs and cigarettes and flour and rice mills
• Minimum tax in excess of normal tax liability shall be c/f and adjusted against the tax liability of
subsequent 3 tax years
• This provision doesn’t applies on certain cases like NPO

285 | P a g e
Example: for carry forward of minimum tax in excess of normal tax liability (ignore withholding
tax in this example):
Year Normal tax liability 1.25% of turnover
1 10,000 60,000
2 90,000 63,000
3 120,000 75,000

Solution:
Year 1
Tax liability 60,000
(Excess minimum tax c/f Rs.50,000)

Year 2
Normal tax liability 90,000
Less: b/f excess minimum tax of year 27,000
Tax liability 63,000
(Excess minimum tax of year 1 c/f to year 3 Rs.23,000)

Year 3
Normal tax liability 120,000
Less: b/f excess minimum tax of year 123,000
Tax liability 97,000

Fair Market Value - Section 68


(a) The FMV of any property or rent, asset, service, benefit or perquisite at a particular
time stal be the price which the property or rent, asset, service, benefit or perquisite
would ordinarily fetch on sale or supply in the open market at that time.
(b) The FMV of any property or rent, asset, service, benefit or perquisite shall be
determined without regard to any restriction on transfer or to the fact that it is not
otherwise convertible cash.
(c) Where the price, other than the price of immovable property, referred to in (a) above
is n ordinarily ascertainable, such price may be determined by the Commissioner.
(d) FMV of immovable properties: The FBR is empowered to notify, through
notifications, the FMV of immovable property situated in different parts of
Pakistan.
Where the FMV of immovable properties of an area is not notified by the FBR, the value
fixed by the District Officer (Revenue) or provincial or any other authorized authority for
the purpose of stamp duty shall be deemed to be the FMV of that immovable property.

Foreign currency transactions i.e. currency conversion - Section 71


(a) Every amount taken into account under this Ordinance shall be in Rupees.
(b) Where an amount is in a foreign currency, the amount shall be converted to the Rupee
at State Bank of Pakistan rate applying between the foreign currency and the Rupee on the
date the amount is taken into account for the purposes of this Ordinance.

Agricultural produce cultivated and used as raw material in the business


Agricultural income is exempt under section 41 of the Income Tax Ordinance 2001.
If a cultivator or receiver of agricultural produce as rent-in-kind uses agricultural produce
raised or received by him as raw materials in a business then the market value of the
agricultural produce shall be allowed as a deduction from business. It means that the same
shall be considered as purchase cost for the business.

286 | P a g e
However, where the agricultural produce is not ordinarily sold in the market i.e. the market value is not
determinable then the cultivation expenses and the land revenue taxes / rent paid for the area are
allowable tax deductions from business

PAST PAPER QUESTIONS


Question # 1 Autumn 2009 Q. 4

(a) State the conditions which a tangible asset should meet to qualify as a depreciable asset. (04)

(b) During the tax year 2009, Ishaq Enterprise disposed off the following assets:
(i) an immovable property was sold for Rs. 200 million. The cost of immovable property was Rs. 100
million. Up to tax year 2008, tax depreciation of Rs. 10 million had been allowed on the immovable
property.
(ii) a plant was exported to Nepal. The export proceeds amounted to Rs. 28 million. The cost and written
down value of the plant was Rs. 25 million and Rs. 18 million respectively.
(iii) three trucks were disposed off for Rs. 2.5 million. They were acquired in tax year 2008. The tax
written down value of trucks at the beginning of tax year 2009 was Rs. 2.4 million. The trucks were
being used partly i.e. 60% for business purposes. The rate of depreciation for tax purposes is 15%.

Required:
Compute the tax gain or loss on disposal of each of the above assets. (06)

Question # 2 Autumn 2010 Q. 4(b)

You are the tax consultant of Ideal Associates who are engaged in the business of manufacture and sale of
electronic goods for the last twenty years. The firm has requested for your opinion in respect of the following:
(i) Provision for bad debts.
(ii) Payment against a liability which was outstanding since 2006 and had been added back into the taxable
income of the firm in 2009.
(iii) Initial depreciation allowance on a three-year old plant, which has been imported from China. The
remaining useful life of the plant is 7 years.

Required:
Advise the management on the treatment of the above transactions, under the Income Tax Ordinance, 2001.
(07)

Question # 3 Spring 2011 Q. 3

Carrot Ltd (CL) is engaged in the manufacture, import and sale of electronic appliances for the past twenty
years. When reviewing the company’s tax provisions, you noticed the following amounts appearing in the tax
calculation for the year ended June 30, 20X2.
I. Expenditure of Rs. 450,000 on promotion of a product which is expected to generate revenue for twelve
years.
II. Bad debt in respect of a staff loan, Rs. 25,000.
III. Initial allowance of Rs. 4,000,000 on a used equipment acquired locally from MSD Limited.
IV. Financial charges amounting to Rs. 100,000 and depreciation amounting to Rs. 300,000 on a vehicle
acquired on finance lease from Radish Leasing. Lease rentals paid during the year amounted to Rs.
400,000.

Required:
Under the provisions of Income Tax Ordinance, 2001 discuss the admissibility of the above amounts for tax
purposes. (15)

287 | P a g e
Question # 4 Autumn 2012 Q. 6

In the context of Income Tax Ordinance 2001,


(a) state the meaning of “Intangible”. (04)
(b) discuss the rules relating to claiming of amortization deduction on intangibles. (07)

Question # 5 Spring 2013 Q. 5

(a) Describe the methods of accounting that may be adopted under the Income Tax Ordinance, 2001 by the
following persons deriving income chargeable to tax under the head ‘Income from Business’.
(i) A company
(ii) Any person other than a company (04)

(b) State the provisions of the Income Tax Ordinance, 2001 relating to the change in method of accounting
for income chargeable to tax under the head ‘Income from Business’. (03)

Question # 6 Autumn 2014 Q. 4

In Income Tax Ordinance, 2001 the term “disposal” has a wider connotation than sale because it includes
exchange, relinquishment, and extinguishment.
List the situations under which an asset owned by a person shall be treated to have been disposed of.
(05)

Question # 7 Autumn 2015 Q. 2

Under the provisions of the Income Tax Ordinance, 2001 what would be the cost of an asset for the purpose of
depreciation deduction in each of the following circumstances?
(a) Mr. Aamir acquired a new machine partly in exchange for an old machine. He paid freight to bring the
old machine to the seller’s location and also purchased cooling equipment which was attached to the new
machine for its smooth functioning. (04)
(b) Mr. Saulat acquired production machinery by utilizing a loan repayable in euro. The loan is expressed in
rupees and is repayable in two years’ time. Mr. Saulat also received 20% subsidy on such machinery from
the Provincial Government. (04)
(c) On 1 July 2015 Mr. Talha started using his personal computer for business purposes. He also had to
upgrade the operating system to comply with his business needs. (02)
(d) Mr. Rahi constructed a furnace for his factory in Korangi Industrial Area. (02)

Question # 8 Spring 2016 Q. 2

Akram has recently established an advertising agency in the name and style of Azad Advertising. For
introducing his business to both international and local clients, he has allocated considerable chunk of his
marketing budget to entertainment expenditures. Under the Income Tax Ordinance, 2001 and Rules made
thereunder, advise Akram about the prescribed limits/conditions for the deduction of entertainment
expenditure. (07)

Question # 9 Spring 2017 Q. 2(a)

(a) Explain the term ‘disposal of assets’ as referred to in the Income Tax Ordinance, 2001. (05)

Question # 10 Autumn 2017 Q. 3(c,d)

Under the provisions of the Income Tax Ordinance, 2001 compute taxable income or loss, under the correct
head of income for tax year 2017, in each of the following cases:

288 | P a g e
(c) Sarwar Enterprises sold an immovable property for Rs. 50 million. The cost of the immovable property
was Rs. 30 million. Tax depreciation of Rs. 6 million had been allowed on the immovable property up to
the tax year 2016. (2.5)
(d) Shams Industries Limited (SIL) sold and exported one of its plants to a Nigerian Company. The sale
proceeds received in SIL’s account amounted to Rs. 25 million. The cost and tax written down value of
the plant was Rs. 20 million and Rs. 7 million respectively. (2.5)

Question # 11 Autumn 2017 Q. 5

Under the Income Tax Ordinance, 2001 certain persons are required to pay minimum tax amounting to 1% of
their turnover from all sources.
(a) Explain the term ‘Turnover’ for the purpose of determining the minimum tax. (05)
(b) List the persons who are required to pay minimum tax. (03)
(c) Discuss the provisions relating to carry forward of minimum tax paid to the subsequent years.
(02)

Question # 12 Autumn 2018 Q. 3

(a) Hirani & Company (HC), a resident AOP, is engaged in the manufacturing of various consumer products
and is assessed under normal tax regime. During the year ended 30 June 20X8, HC’s sales was Rs.
140,000,000. It includes sales tax of Rs. 10,000,000 and excise duty of Rs. 5,000,000. The taxable income
for the year is Rs. 6,170,000.
Compute HC’s tax liability for tax year 20X8, under the provisions of the Income Tax Ordinance, 2001.
(Tax rates are given on the last page) (03)

(b) The accounting profit before tax of Bashir Associates (BA) for the year ended 30 June 20X8 is Rs.
1,200,000.

Last year, BA had written off balances outstanding from two of its debtors namely Pulse International (PI)
and Hussain Global (HG) which were partly allowed by the tax authorities. Details are as follows:

PI HG
---------Rupees --------
Amounts written off 1,150,000 925,000
Allowed by tax authorities 825,000 240,000

During the current tax year, BA received Rs. 652,000 from PI and Rs. 346,000 from HG, in full settlement
of their debts.

In the light of the Income Tax Ordinance, 2001 compute BA’s taxable income for the tax year 20X8. (05)

Question # 13 Spring 2019 Q. 3(b)

Following transactions pertain to Salam Limited (SL) which took place during the tax year 20X9:
(i) A machine costing Rs. 1,800,000, being used in SL’s Karachi factory was transferred to its subsidiary in
Ghana. The fair market value and tax written down value of the machine on the date of transfer were Rs.
2,500,000 and Rs. 600,000 respectively. (02)
(ii) On 1 January 20X9, SL entered into a forward contract for the purchase of raw materials to be used in its
business to guard against loss through price fluctuations. On the date of maturity of the forward contract,
SL did not take the delivery of the raw materials but the contract was settled by making a payment of Rs.
500,000. (03)
Required:
Explain the taxability of the above transactions.

289 | P a g e
Question # 14 Autumn 2019 Q. 5

(a) Identify any three situations in which the fair market value of the assets shall be treated to be the cost of
the asset. (03)

(b) During the tax year 20X9, Salman Shahid sold the following assets:
(i) A vehicle used by manager-in-charge of his garment factory for Rs. 7.8 million. The vehicle was
purchased for Rs. 8.1 million in tax year 20X6. (03)

Required:
Under the provisions of the Income Tax Ordinance, 2001 compute under the appropriate head of income,
the amount to be included in the taxable income of Salman Shahid for the tax year 20X9.

Question # 15 Spring 2020 Q. 3(a)

Under the provisions of the Income Tax Ordinance, 2001 and Rules made thereunder, discuss:
(a) the prescribed limits/conditions for the deduction of entertainment expenditure. (06)

Question # 16 Spring 2021 Q. 2(b)

Gillani and Company (GC), a sole proprietor, is dealing in various consumer products in Pakistan. During the
year ended 30 June 20X2, GC’s taxable income for the year was Rs. 1.6 million.

Required:
Under the provisions of the Income Tax Ordinance, 2001 compute the amount of net income tax payable by
GC and amount of income tax to be carried forward, if any, for the tax year 20X2, in each of the following
situations:
(i) GC’s sales were Rs. 120,500,000 inclusive of sales tax.
(ii) GC’s sales were Rs. 110,000,000 inclusive of sales tax. (05)

Question # 17 Autumn 2021 Q. 3(a)(ii)

State the provisions of the Income Tax Ordinance, 2001 relating to each of the following:
(ii) Change in the method of accounting for income chargeable to tax under the head ‘income from business’
(03)

Question # 18 Spring 2022 Q. 2

(a) Under the provisions of the Income Tax Ordinance, 2001 discuss the tax implication/treatment in each of
the following independent matters:
(i) Purchase of immovable property in cash. (03)

(b) For the purpose of this part of the question, assume that the date today is 31 August 2022. During the year
ended 30 June 2022, Faster & Co. (FC) started a new project. Following information is available:

Incurred Rs. 5 million on feasibility study of the project.


Obtained a 3% loan of AED 2 million from a UAE bank on 1 January 2022 for the purchase of plant and
machinery. The interest is payable annually and principal amount is repayable at the end of third year.

Installed the plant and machinery at a cost of Rs. 150 million on 14 March 2022.
The exchange rates of 1 AED to PKR on different dates are as follows:

Average between
1-Jan-2022 30-Jun-2022 1-Jan-2022 to 30-Jun-2022

290 | P a g e
Rs. 50 Rs. 55 Rs. 53

Required:
Compute the amount of allowable deduction in determining the taxable income of FC for tax year 2022.
(04)

Question # 20 Spring 2021 Q. 2

(a) What do you understand by the term ‘Turnover’ as provided in section 113 of the Income Tax Ordinance,
2001? List the persons who are required to pay minimum tax on the basis of turnover. (08)

(b) Gillani and Company (GC), a sole proprietor, is dealing in various consumer products in Pakistan. During
the year ended 30 June 20X2, GC’s taxable income for the year was Rs. 1.6 million.

Required:
Under the provisions of the Income Tax Ordinance, 2001 compute the amount of net income tax payable
by GC and amount of income tax to be carried forward, if any, for the tax year 20X2, in each of the
following situations:
(i) GC’s sales were Rs. 120,500,000 inclusive of sales tax.
(ii) GC’s sales were Rs. 110,000,000 inclusive of sales tax. (05)

291 | P a g e
ANSWERS
Answer # 1 Autumn 2009 Q. 4

(a) A tangible movable or immovable asset will be considered a depreciable asset when all the following
condition are met:
▪ It has a normal useful life exceeding one year;
▪ It is likely to lose value as a result of normal wear and tear, or obsolescence;
▪ It is used wholly or partly by the person in deriving income from business chargeable to tax.
(b)
(i) Rs. In ‘000’
Consideration Received 200,000
Less: Tax WDV
Cost 200,000
Accumulated Depreciation (10,000) 190,000
Tax gain on disposal 10,000
Rs. In ‘000’
Consideration Received 25,000
Less: Tax WDV (18,000)
Tax gain on disposal 7,000

Sale Proceeds 2500


Less : WDV at the beginning of year (2400)
Less : Depreciation not allowed [(2400/0.85) – 2400] x 40% (169)
Loss in Disposal (69)

Answer # 2 Autumn 2010 Q. 4(b)

(i) The firm cannot claim deductions on account of bad debts unless the following conditions are satisfied
▪ the amount of debt was previously included in the person's income from business chargeable tax;
▪ the debt or part of the debt is written off in the accounts of the person in the tax year,
▪ there are reasonable grounds for believing that the debt is irrecoverable.
(ii) Since the liability pertaining to the year 2006 has been charged/ (added back) to tax therefore subsequent
payment thereof shall be allowed as a deduction, in the year in which the payment is made i.e. 2009.
(iii) The firm can claim the initial allowance against the imported used plant as:
▪ it is used in Pakistan for the first time in a tax year.
▪ it is used by the firm for the purposes of its business it falls in the definition of eligible depreciable
asset

Answer # 3 Spring 2011 Q. 3

Comments on the deductibility of expenditures charged by CL:


(i) Amortization of intangibles:
Any expenditure that provides an advantage or benefit for a period of more than one year is included in
the definition of intangibles and is required to be amortized over the period of expected benefit that is 12
years.
As such CL would be allowed to charge Rs. 37,500 (450,000/12) in tax year 20X2.
(ii) Bad debts:
Only those bad debts are allowed as admissible deductions which have previously been included in the
taxpayer's business income chargeable to tax
Since the staff loan was not previously offered to tax as business income, it would not be admissible.

292 | P a g e
(iii) Initial depreciation:
Initial allowance is only admissible on such plant and machinery which was not previously used in
Pakistan.
Since in this case, the equipment was previously used in Pakistan, the initial allowance is not admissible
(iv) Vehicle on finance lease:
Entire lease rentals paid during the year, on leased assets, ie, Rs. 400,000 shall be allowed as admissible
deduction.

Following expenditures however, would not be admissible:


▪ Finance Charges 100,000
▪ Depreciation 300,000

Answer # 4 Autumn 2012 Q. 6

(a) A tangible movable or immovable asset will be considered a depreciable asset when all the following
condition are met:
▪ It has a normal useful life exceeding one year;
▪ It is likely to lose value as a result of normal wear and tear, or obsolescence;
▪ It is used wholly or partly by the person in deriving income from business chargeable to tax.

(b) The amortization deduction of a person for a tax year shall be computed according to the following
formula, namely: A/B

Where:
A is the cost of the intangible; and
B is the normal useful life of the intangible in whole years.

An intangible which does not have an ascertainable useful life, shall be treated as if it had a normal useful
life of twenty five years.

If an intangible is used in a tax year partly in deriving income from business chargeable to tax and partly
for another use, the deduction allowed for that year shall be restricted to the fair proportional part of the
amount that would be allowed if the intangible were wholly used to derive income from business
chargeable to tax.

Where an intangible is not used for the whole of the tax year in deriving income from business chargeable
to tax, the deduction allowed under this section shall be computed according to the following formula,
namely: A x B/C

Where:

A is the amount of amortization computed in accordance with the above provisions


B is the number of days in the tax year the intangible is used in deriving income from business chargeable
to tax; and
C is the number of days in the tax year.

Answer # 5 Spring 2013 Q. 5

(a)
(1) Company shall account for income under the head "Income from Business" on an accrual basis.
(2) Other persons may account for such income on a cash or accrual basis.
(b)
(1) A person may apply for a change in the person's method of accounting and the Commissioner may,

293 | P a g e
by order, approve such an application but only if satisfied that the change is necessary to reflect the
person's income under the head "Income from Business".
(2) If a person's method of accounting has changed, the person shall make adjustments to items of income,
deduction and credit, so that no item is omitted and no item is taken into account more than once.

Answer # 6 Autumn 2014 Q. 4

Following assets will be considered as disposed off:


(1) An asset will be considered as disposed off when its ownership comes to an end, including when it is:
(a) sold, exchanged, transferred or distributed; or
(b) cancelled, destroyed, lost, expired or surrendered.
(2) The transmission of an asset by succession or will shall be considered as disposal at the time of
transmission.
(3) The application of a business asset to personal use will be considered as disposal when asset is put to
personal use. Where a business asset is discarded or ceases to be used, it will be considered that it is
disposed off.
(4) A disposal shall include the disposal of a part of an asset.

Examiner Comments:
This was a simple question in which the students were required to demonstrate their
understanding of the term ‘disposal’ by listing the situations under which an asset owned by
a person shall be treated to have been disposed of. The response was below average as many
students tried to answer the question on the basis of their general knowledge. Only few
students were able to list down all the situations contained in Section 75 of the Ordinance.
Majority of them listed two or three points only.

Answer # 7 Autumn 2015 Q. 2

(a) The cost of new purchased machine shall be the sum of following amounts:
(a) The total amount given for asset, including FMV of consideration in kind determined at the time the
asset is acquired.
(b) Any incidental expenditure paid in acquiring and disposing of asset.
(c) Expenditure paid to alter or improve the asset. For example, the cooling equipment purchased here.
(b) If an asset is purchased with a loan from foreign currency, then any increase/decrease in liability due to
currency fluctuation will be added or deducted in the cost of the asset. For the calculation of depreciation,
currency fluctuation will be accounted for in year of occurrence.
The cost of an asset does not include the amount of any subsidy received in respect of the acquisition of
the asset. However, if subsidy is chargeable to tax, then it will be added in cost.
(c) If a personal asset is put to business use, its cost will be the fair market value on the date it is put to business
use and the cost shall include expenditure paid to alter or improve the asset. Therefore, the cost of personal
computer shall be sum of FMV on 1" July, 2015 and cost of up gradation.
(d) The cost of self-constructed furnace shall be the total cost incurred in producing or constructing the asset
plus any incidental expenditure paid to alter or improve the asset.

Examiner Comments:
This question consisted of four parts. In each part there was a situation related to purchase
of an asset. The candidates were required to discuss as to what would be the cost of the
asset for the purpose of determining the tax depreciation. The performance in each part is
discussed below:
(a) According to the given scenario, a new machine was acquired partly in exchange of an old
machine. Freight was paid to send the old machine to the supplier. Further, cooling equipment
was also purchased and was attached to the new machine.
Only few candidates were able to correctly identify all the components of cost of new machine.
The common mistakes were as follows:
▪ A number of candidates did not specify the most obvious component of the cost i.e. the
‘consideration paid in cash’.

294 | P a g e
▪ Many candidates did state that fair market value (FMV) of the old machine would form part
of the cost, however, they did not mention that the determination of such FMV would be at the
time the new machine is acquired.
▪ While computing the cost, many candidates considered the FMV of the new machine instead
of FMV of the old machine.
▪ Some students failed to comprehend that instead of adding to the cost, the FMV of the old
machine would be deducted from the cost of new machine.
▪ Many candidates could not identify that freight charges paid to bring the old machine to the
seller’s location would also be included in the cost of new machine.

(b) This part of the question was based on the requirements of Section 76(5), (6) and (10) of the
Income Tax Ordinance, 2001 and tested candidates’ knowledge with regard to the treatment of
subsidy received and exchange rate fluctuations related to foreign currency loan, while computing
the cost of a fixed asset. Generally, the candidates were aware that under the tax laws, subsidy
should be deducted in arriving at the cost of an asset for tax purposes. However, many students
added the amount of subsidy instead of deducting it. Further, only few candidates were aware that
subsidy will not be deducted if such subsidy is chargeable to tax.
Most of the candidates correctly explained the effect of increase or decrease in foreign loan
liability due to exchange rate fluctuations, on the cost of the asset. However, most of the
candidates did not state that the difference is to be taken into account in the year in which it arises.
(c) This part was well answered by most of the candidates as they were aware that the FMV of the
computer at the time it is put to business use and the cost incurred on its up- gradation would
form part of the cost of the asset. However, few students thought that it would be the written down
value instead of FMV which would be added to the cost of an asset. Some of the students were of
the opinion that cost of up-gradation should be amortized separately.
(d) Total cost incurred on construction of the furnace was correctly identified as cost of furnace.
However, many candidates did not state that such cost will also include any incidental expenditure
incurred for acquiring the asset. Further, many students wasted time in explaining pre-
commencement expenditures and rules related to initial depreciation, which were not relevant.

Marking Scheme:
(a)
▪ Up to 01 mark for inclusion of the following into cost of machine:
- cash given for the asset
- fair market value of old machine
- freight and other incidental expenditure
- purchase – cooling equipment
(b)
▪ Explaining the treatment of:
- subsidy received 1.5
- exchange rate fluctuation 2.5
(c)
▪ 01 mark each for identification of each of the following components of cost of asset:
- fair value and date on which it is to be determined
- up-gradation cost
(d)
▪ 01 mark for inclusion of following into cost of machine:
- construction cost
- incidental expenditure

Answer # 8 Spring 2016 Q. 2

Entertainment expenditure:
Akram would be allowed a deduction for entertainment expenditure if the following conditions are satisfied:

The expenditure should be incurred in deriving income from business chargeable to tax and should be limited
to expenditure incurred which satisfies the following conditions:

295 | P a g e
▪ expenditure incurred outside Pakistan on entertainment in connection with business transactions or is
allocated as head office expenditure;
▪ expenditure incurred in Pakistan on entertainment of foreign customers and suppliers;
▪ expenditure incurred on entertainment of customers and clients at the person’s business premises;
▪ expenditure incurred on entertainment at a meeting of shareholders, agents, directors or employees; or
▪ expenditure incurred on entertainment at the opening of branches.

A deduction shall only be allowed for expenditure incurred on the entertainment of persons related directly to
Akram’s business.

Examiner Comments:
This question carrying 7 marks was designed to examine the students’ understanding of the
provisions of the Income Tax Ordinance, 2001 and Rules made thereunder, regarding the
admissibility of entertainment expenditure incurred by a taxpayer. The candidates were
generally able to respond well to this question and the response was above average.

However, three important conditions, i.e. entertainment expenditure is admissible as a


deduction only when it is incurred on (i) deriving business income chargeable to tax (ii)
entertainment of persons directly related to the business (iii) entertainment of foreign
customers and suppliers in Pakistan only, were missed by a number of candidates. Further,
many candidates failed to appreciate the importance of the words contained in the law and
used inappropriate replacements thereof which often conveyed a very different meaning.
They are advised to be careful in the choice of words in such questions.

Marking Scheme:
01 mark for explaining each condition under which entertainment expenditure would be
allowed under the head income from business 7.0

Answer # 9 Spring 2017 Q. 2(a)

A person who holds an asset shall be treated as having made a disposal of the asset at the time the person parts
with the ownership of the asset, including when the asset is:
▪ sold, exchanged, transferred or distributed; or
▪ cancelled, redeemed, relinquished, destroyed, lost, expired or surrendered.

The transmission of an asset by succession or under a will shall be treated as a disposal of the asset by the
deceased at the time asset is transmitted.

The application of a business asset to personal use shall be treated as a disposal of the asset by the owner of the
asset at the time the asset is so applied.

Where a business asset is discarded or ceases to be used in business, it shall be treated to have been disposed
of. A disposal shall include the disposal of a part of an asset.

Examiner Comments:
This part of the question required the students to explain the term ‘disposal of assets’.
Apparently, many students had not studied it and used guesswork which was of little use.
However, a significant number of students also secured good and even full marks.

Marking Scheme:
(a) Explaining the term ‘disposal of assets’ 5.0

Answer # 10 Autumn 2017 Q. 3(c,d)

296 | P a g e
(c) Rs. in million
Sale proceeds (equivalent to cost) 30
Less: Cost of land and WDV of immovable property (30 - 6) (24)
Gain on disposal (Income from business) 6

(d) Rs. in million


Cost (equivalent to sale proceeds) 20
Less: WDV of plant (7)
Gain on disposal (Income from business) 13

Examiner Comments:
(c) This part was based on the requirements of section 22(13)(d) of the Income Tax Ordinance, 2001
and candidates were required to calculate gain on the sale of immovable property. There was just
one testing point in the scenario i.e. gain on disposal of immovable property cannot exceed the
amount of tax depreciation claimed thereon till the end of the previous tax year.
This is achieved because according to Income Tax Ordinance, the sale proceeds of an immovable
property for the purpose of computation of the gain are restricted to its cost or the cost is
considered to equal the sale proceeds. In either case, the result is the same. Those who understood
this rule well were easily able to secure full marks whereas the others could not secure any mark.

(d) This part was based on the requirements of section 22(14) of the Income Tax Ordinance, 2001 and
candidates were required to calculate gain on the sale and export of plant. Under this subsection,
the rule explained in (c) above i.e. section 22(13)(d) has also been made applicable for computing
the gain on sale of plant used in Pakistan but exported out of Pakistan. Most of the candidates
who performed well in part (c) also performed well in this part and vice versa. Some of the
candidates stated the rule correctly but did not compute the amount of gain.

Marking Scheme:
(c)
▪ Determination of sale proceeds 1.5
▪ Computation of gain/loss under correct head of account 1.0
(d)
▪ Determination of sale proceeds 1.5
▪ Computation of gain/loss under correct head of account 1.0

Answer # 11 Autumn 2017 Q. 5

(a) For the purpose of minimum tax liability, turnover is defined as:
(i) The gross sales/gross receipts, exclusive of sales tax and federal excise duty or any trade discounts
shown on invoices or bills, derived from the sale of goods and also excluding any income taken as
deemed.
(ii) The gross fees for the rendering of services, including commissions.
(iii) The gross receipts from the execution of contracts.
(iv) The company’s share of the above stated amounts of an association of persons of which the company
is a member.

In case of (i), (ii) and (iii) above, it does not include any amount covered by final discharge of tax liability
for which tax is separately paid or payable.

(b) Following persons are required to pay minimum tax:


(i) a resident company
(ii) an individual having turnover of Rs. 100 million or above in the tax year 2017 or in any subsequent
tax year
(iii) an association of persons having turnover of Rs. 100 million or above in the tax year 2017 or in any
subsequent tax year

297 | P a g e
(c) Where minimum tax exceeds the actual tax payable, the excess amount of tax paid shall be carried forward
for adjustment against normal tax liability of the subsequent tax year.
However, the amount shall be carried forward and adjusted against tax liability for a maximum of five tax
years immediately succeeding the tax year for which the amount was paid.

Examiner Comments:
(a) In this part the candidates were required to about explaining the term ‘turnover’ for the purpose
of determining the minimum tax. The overall performance remained poor as generally the
candidates were unaware of the concept as contained in the Income Tax Ordinance, 2001. Instead,
most of them used guesswork and tried to narrate the dictionary meaning of the term.
(b) In this part candidates were required to list the persons who are required to pay minimum tax. The
performance remained average. Many candidates were unaware of the changes that have been
made during the past few years especially the inclusion of individuals. A major error was to
include all companies under the purview instead of resident companies only. Many candidates
also included persons whose income is subject to FTR. Many candidates produced a long list
which included various types of tax payers such as cottage industries, those who have failed to
file return of income, those who have committed fraud, etc.
(c) In this part candidates were required to discuss the provisions relating to carry forward of
minimum tax paid to the subsequent years. This part was based on the requirements of section
113(1)(c) of the Income Tax Ordinance, 2001. Most of the candidates were unable to reply
correctly and the performance was quite poor. Some of the common errors were as follows:
▪ Provisions of sales tax regarding excess of input tax over output tax were discussed.
▪ A large number of candidates were of the opinion that minimum tax cannot be carried
forward.

Marking Scheme:
(a) Explanation of the term ‘Turnover’ 5.0
(b) 0.5 to 1.5 marks for identification of each person who is required to pay minimum
tax 3.0
(c) Discussion with regard to carry forward of minimum tax paid 2.0

Answer # 12 Autumn 2018 Q. 3

(a) Minimum tax on income will be applicable as Hirani & Company [HC) is a Resident AOP having turnover
more than Rs.100 million.

Rupees
Income Tax payable under Normal Tax Regime (NTR)
Taxable income 6,170,00
Income tax on above (Table 1) (1,330,000 + 35% x 170,000 A 1,389,500

Income Tax payable under minimum tax regime


Gross (140,000,000 – 10,000,000 – 5,000,000) 125,000,000
Sales
Tax u/s 133 @ 1.25% on Rs. 125,000,000 B 1,562,500

Tax liability of HC (higher of A and B) 1,562,500

Tax of Rs. 173,000 (1.562,500 - 1,389,500) will be carried forward for adjustment against actual tax
liability 1,562,500 of the subsequent 3 tax years.
(b) Rupees
Accounting profit before adjustments 1,200,000
Less: Amount received from Pulse International (652,000)
Less: Amount received from Hussain Global (346,000)
Bad debts amount previously allowed but recovered by PI 327,000
Bad debts short allowed (339,000)

298 | P a g e
Taxable income 190,000

Pulse International (PI)


Amount received 652,000
Bad debts claimed 1,150,000
Less: Bad debts allowed as deduction 825,000
325,000
Excess income to be added in the income for the tax year 2018 327,000
Hussain Global (HG)
Amount received 346,000
Bad debts claimed 925,000
Less: Bad debts allowed as deduction 240,000
685,000
Amount short received to be allowed as deduction for the TY 20X8 (339,000)

Examiner Comments:
This question consisted of three parts. The overall performance was reasonable as 51%
candidates secured passing marks. Part wise comments are given below.
(a) This part of the question tested the concept of minimum tax. The given scenario was about a
resident AOP engaged in manufacturing activities; which was assessed under normal tax regime.
The AOP’s sale, taxable income and sales tax and excise duty included in the sale were given.
Candidates’ demonstrated weak knowledge as majority of them only calculated tax liability under
normal tax regime. Some of the other common mistakes were as follows:
▪ Turnover tax was computed on gross sales i.e. without deducting sales tax and excise duty
included in the sales.
▪ Sales Tax and Federal Excise Duty were considered as not allowable tax deductions and were
added to taxable income.
▪ Minimum (turn over) tax was calculated at the rate of 1% instead of 1.25%.

(b) In this part the candidates were required to specify the time limits during which the Commissioner
of Income Tax could make the first amendment to an assessment and also the time limit for any
subsequent amendment. Most of the students stated correctly about the time period of five years
but did not mention or correctly mention as from when the period of five years would be counted.
The same issue was observed with regard to the subsequent amendment also.

Marking Scheme:
(a)
▪ Income tax payable under normal tax regime 1.0
▪ Income tax payable under minimum tax regime 1.0
▪ Determination of tax liability 1.0
(b)
▪ 0.75 mark for each adjustment in accounting profit 3.0
▪ 01 mark for determination of short amount to be allowed/excess income to be added in the income
for the tax year 2018 2.0

Answer # 13 Spring 2019 Q. 3(b)

(b)(i) Rs.in 000


Consideration received (equal to the cost of the asset) 1,800
WDV at the time of disposal 600
Gain on disposal 1,200

(ii)The forward contract entered into by SL for the purchase of raw materials is in the nature of a hedging
contract which was entered into to guard against loss from future price fluctuations. Such contracts have
specifically been excluded from the definition of speculative business. Therefore, Rs. 500,000 paid to
settle the forward contract is an expenditure incurred in the normal course of business and is a deductible

299 | P a g e
expenditure.

Examiner Comments:
(i) Instead of Rs. 1,800,000, the amount of Rs. 2,500,000 was treated as consideration received.
(ii)
(i) Students considered the given forward contract under ‘Speculative Business’. In fact, such
contract was specifically excluded from the definition of speculative business.
(ii) Students failed to mention that the amount paid to settle the hedging contract is an expenditure
incurred in the normal course of business and is a deductible expenditure.

Marking Scheme:
(i) Determination of the correct amount relating to:
▪ consideration received 1.0
▪ written down value 1.0
(ii)
▪ Discussion on hedging contract 2.0
▪ Conclusion 1.0

Answer # 14 Autumn 2019 Q. 5

(a) The fair market value of the assets shall be treated as cost of the assets when received:
▪ under a gift from a relative, bequest or will;
▪ by succession, inheritance or devolution;
▪ on a distribution of assets on dissolution of an association of person; or
▪ on a distribution of assets on liquidation of a company
The cost of a personal assets treated as acquired by the business shall be the fair market value of the asset
determined at the date it is applied to business use
(b)
(i) Loss on disposal of a vehicle used by manager Rupees
Consideration received on disposal of passenger transport vehicle not plying for hire
A×B/C Where (7.8×7.5/8.1) 7,222,222
(A is the amount received on disposal of the vehicle; B is the amount allowed as per
law i.e. Rs. 7 .5 million; and C is the actual cost of acquiring the vehicle.)
Written down value of vehicle (7.5×85%×85%×85%) (4,605,938)
Income from business 2,616,284

Examiner Comments:
(a) Majority of the answers revolved around one or two points. Many examinees failed to appreciate
that distribution of assets on dissolution of AOP and on liquidation of company are also the
situations in which fair market value is deemed to be the cost of the asset.
(b) (i)
▪ Examinees failed to identify the correct head of income.
▪ Many examinees failed to restrict the cost of vehicle to Rs. 7.5 million.
▪ Errors were made in restricting the sales proceeds in the same ratio in which the cost was
restricted.

Marking Scheme:
(a) Up to 01 mark for identification of each situation in which fair market value of the asset shall be
treated to be the cost of the asset 3.0
(b)(i)
▪ Computation of:
− consideration received on disposal of vehicle 1.5
− written down value 1.0
▪ Mentioning correct head of income 0.5

Answer # 15 Spring 2020 Q. 3(a)

300 | P a g e
Entertainment expenditure:
The prescribed limits / conditions for the deduction of entertainment expenditure are as under:

The expenditure should be incurred in deriving income from business chargeable to tax and should be limited
to expenditure incurred which satisfies the following conditions:
▪ expenditure incurred outside Pakistan on entertainment in connection with business transactions or is
allocated as head office expenditure;
▪ expenditure incurred in Pakistan on entertainment of foreign customers and suppliers;
▪ expenditure incurred on entertainment of customers and clients at the person’s business premises;
▪ expenditure incurred on entertainment at a meeting of shareholders, agents, directors or employees; or
▪ expenditure incurred on entertainment at the opening of branches.

A deduction shall only be allowed for expenditure incurred on the entertainment of persons related directly to
person’s business.

Examiner Comments:
Most of the examinees failed to appreciate that entertainment expenditure is allowed as
admissible deduction only when incurred on:
▪ deriving business income chargeable to tax
▪ entertainment of persons directly related to the business
▪ entertainment of foreign customers and suppliers in Pakistan

Marking Scheme:
Up to 01 mark for mentioning each prescribed limits/conditions for the deduction of
entertainment expenditure 6.0

Answer # 16 Spring 2021 Q. 2(b)

Computation of tax liability of Gillani & Co.


For the tax year 20X1
Situation 1 Situation 2
Income tax payable under normal tax regime
Taxable income 1,600,000 1,600,000
Income tax
(60,000 + (1,600,000 – 1,200,000) x 17.5% A 130,000 130,000

Income tax payable under minimum tax


Gross sales 120,500,000 110,000,000
Less: Sales tax (17/117) 17,508,547 15,982,906
102,991,453 94,017,094

Turnover tax @ 1.25% B 1,287,393 N/A

Tax liability of GC (Higher of A or B) 1,287,393 130,000


Carry Forward of Excess tax 1,157,393

Examiner Comments:
(b)
▪ Examinees were not able to compute the correct figure of sales tax from the gross sales.
▪ Minimum tax was calculated in situation 2 without considering the fact that the turnover is less
than Rs. 100 million.
▪ Carried forward amount of minimum tax was not computed.

301 | P a g e
Marking Scheme:
(b)
▪ Computation of income tax payable under normal tax regime 1.0
▪ Computation of income tax payable under minimum tax 2.0
▪ Determination of tax liability and carried forward of excess tax 2.0

Answer # 17 Autumn 2021 Q. 3(a)(ii)

A person may apply in writing for a change in the person’s method of accounting to the Commissioner. The
Commissioner may, by notice in writing, approve such an application but only if satisfied that the change is
necessary to clearly reflect the person’s income chargeable to tax under the head “Income from Business”. If a
person’s method of accounting has changed, the person shall make adjustments to items of income, deduction,
or credit, or to any other items affected by the change so that no item is omitted, and no item is taken into
account more than once.

Examiner Comments:
▪ In point no. (ii), examinees failed to mention that items affected by the change shall be adjusted
in a manner that no item is omitted and no item is taken into account more than once.

Marking Scheme:
0.5 mark for each provision related to change of tax year from special to normal 2.0

Answer # 18 Spring 2022 Q. 2

(a) In case any immovable property having fair market value (FBR value or DC rate whichever is higher)
greater than five million rupees is purchased in cash, then it will have following implications:

Such asset shall not be eligible for initial allowance or depreciation.

Such amount shall not be treated as cost for computation of any gain on disposal (sale value will be treated
as capital gain).

Such person shall pay a penalty of 5% of the FBR value or DC rate whichever is higher.

(b) Faster & Co.


Computation of amount of allowable deduction in determining taxable income

Rs. in million
Initial allowance on plant and machinery [160(W-1)×25%] (40.00)
Depreciation on plant and machinery [18(160(W-1)×75%×15%)] (18.00)
Amortization of pre-commencement expenditure (5×20%) (1.00)
Finance charges [(2×3%÷2)×53] (1.59)
(60.59)

W-1: Cost of plant and machinery Rs. in million


Acquisition cost 150
Change in AED rate [2×(55-50)] 10
160

Answer # 20 Spring 2021 Q. 2

302 | P a g e
(a) For the purpose of minimum tax liability, turnover is defined as:
(i) The gross sales/gross receipts, exclusive of sales tax and federal excise duty or any trade discounts
shown on invoices or bills, derived from the sale of goods and also excluding any income taken as
deemed.
(ii) The gross fees for the rendering of services, including commissions.
(iii) The gross receipts from the execution of contracts.
(iv) The company’s share of the above stated amounts of an association of persons of which the company
is a member.

In case of (i), (ii) and (iii) above, it does not include any amount covered by final discharge of tax liability
for which tax is separately paid or payable.

Following persons are required to pay minimum tax:


(i) a resident company
(ii) an individual having turnover of Rs. 100 million or above in the tax year 2017 or in any subsequent
tax year
(iii) an association of persons having turnover of Rs. 100 million or above in the tax year 2017 or in any
subsequent tax year
(b) Computation of tax liability of Gillani & Co.
For the tax year 20X1
Situation 1 Situation 2
Income tax payable under normal tax regime
Taxable income 1,600,000 1,600,000
Income tax
(60,000 + (1,600,000 – 1,200,000) x 17.5% A 130,000 130,000

Income tax payable under minimum tax


Gross sales 120,500,000 110,000,000
Less: Sales tax (17/117) 17,508,547 15,982,906
102,991,453 94,017,094

Turnover tax @ 1.25% B 1,287,393 N/A

Tax liability of GC (Higher of A or B) 1,287,393 130,000


Carry Forward of Excess tax 1,157,393

Examiner Comments:
(a)
▪ Examinees tried to define the term 'Turnover' merely on the basis of their general knowledge,
which was entirely different from the specific definition given in the Income Tax Ordinance, 2001.
▪ Examinees did not include commission in the definition of turnover.
▪ Examinees failed to appreciate that turnover for the purposes of minimum tax liability did not
include any amount covered by final discharge of tax liability for which tax was separately paid
or payable.
▪ Examinees did not mention that only a resident company is required to pay minimum tax.
(b)
▪ Examinees were not able to compute the correct figure of sales tax from the gross sales.
▪ Minimum tax was calculated in situation 2 without considering the fact that the turnover is less
than Rs. 100 million.
▪ Carried forward amount of minimum tax was not computed.

303 | P a g e
CHAPTER 13 ASSOCIATION OF
PERSON
1. Treatment of share of profit in the hands of members
• AOP includes a firm, a Hindu undivided family, any artificial juridical
person and anybody of persons formed under a foreign law, but does not
include a company
Note for students: Partnership firm and joint venture are the common examples of AOP.
• AOP pays income tax at the prescribed rates on its income taxable under normal tax regime.
• Share of profit from AOP is exempt in the hands of members but the same shall
be included in taxable income only for rate purpose.
• Share of loss from AOP is not adjustable against income of the members nor is
it considered for rate purpose. AOP itself carries forward its losses.
• If income of AOP is exempt then share of profit of exempt income shall
not be included in taxable income of members for rate purposes.
Individual as a member of AOP
If, for a tax year, an individual has taxable income besides exempt share of profit from AOP, then the amount
of tax payable on the taxable income of the individual shall be computed in accordance with the following
formula:
(A/B) x C
A = is the amount of tax that would be assessed to the individual for the year if the amount or amounts exempt
from tax (i.e. share of profit from AOP) were chargeable to tax
B = is the taxable income of the individual for the year if the amount or amounts exempt from tax under (i.e.
share of profit from AOP) were chargeable to tax; and
C = is the individual‘s actual taxable income for the year
Key points for solving AOP numerical
Following points should be kept in mind when solving AOP numerical:
• Any profit on debt, brokerage, commission, salary or other remuneration paid by an association of
persons to a member of the association is not allowed as an expense; hence it should be added back to
the taxable income of AOP.
• “FBR has clarified that it is the profit before tax of AOP that will be included in the taxable income
of its members for rate purpose”
• Share of loss from AOP is neither adjustable against income of its members nor it is considered for
rate purpose.
• If an AOP has any income that falls under final tax regime (FTR) then members share from such
income shall not be added in the taxable income of the member.
Example 1
Taxable income of AOP 1,800,000
Tax on Rs.1,200,000 75,000
Tax on Rs.600,000 @ 20% 120,000 195,000
Divisible income. 1,605,000

Mr. A and Mr. B are the partners with equal share. Therefore, share of profit is Rs.802,500
for each partner.
Mr. A also provided the following information:

304 | P a g e
Taxable other sources 1,650,000
-Taxable capital gain on disposal of private companies shares 1,210,000
Zakat deducted by the bank 10,000
-Taxes deducted / paid at source 125,000
Required:
Calculate tax payable by Mr. A with the return of income.

Solution
Taxable other sources 1,650,000
Taxable capital gains 1,210,000
Total income 2,860,000
Less: Zakat deducted at source 10,000
Taxable income 2,850,000
Add: Share of profit from AOP for rate purpose 900,000
Taxable income for rate purpose 3,750,000
Tax liability (Non-Salaried Case)
income tax on Rs.3,000,000 465,000
Income tax on Rs.750,00 @ 30% 225,000
690,000
Income tax on Rs.2,850,000
𝐴 𝑋 𝐶= 690,000/3750,000 x 2,850,000 524,400
Less: Taxes deducted / paid at source 125,000

Example 2
Mr. C has disclosed the following information:
-Taxable capital gain on disposal of private companies shares 1,440,000
-Taxable other sources 600,000
- Share of profit from a firm 660,000
- Zakat deducted at source by the bank 45,000
-Taxes deducted / paid at source 38,000
Required:
Calculate tax payable with the return of income.

Solution
Tax payable with return of income Rs.246,597

305 | P a g e
(a) Notes for students:
If an individual is a member of AOP and has other normal taxable income as well then
Section 65(1) specifies a specific treatment for rebates, if any, in respect of donation and
APF that the components A and B i.e. average rate of tax for such rebates would include
the effect of share of profit from AOP.
It is a considered opinion that share of profit from AOP shall not be considered i.e. actual
taxable income and actual tax liability would be considered for other aspects such as:
• To decide salaried or non-salaried case (i.e. taxable salary would be divided
by actual taxable income)
• Foreign tax credit
• Amount of limits for rebates such as maximum 30% for donation and so on
• Limits on deductions for education expenses

Example 3
Mr. D has disclosed the following information:
-Taxable salary 2,800,000
-Taxable capital gain on disposal of private companies' shares 300,000
-Foreign source business income not exempt in Pakistan 360,000
-Tax paid in the foreign country equivalent to Pak Rupees 27,000
-Share of profit from AOP 320,000
- Zakat deducted at source by the bank 17,000
- Donation to an approved charitable institution through cheque 50,000
- Taxes deducted / paid at source 16,000
Required:
Calculate tax payable with the return of income.

(b) Solution
Taxable salary 2,800,000
Taxable capital gain 300,000
Foreign source income 360,000
Total income 3,460,000
Less: Zakat deducted at source 17,000
Taxable income 3,443,000
Add: Share of profit from AOP for rate purpose 320,000
Taxable income for rate purpose 3,763,000 Tax liability (Salaried Case)
Income tax on Rs.3,763,000
Tax on Rs.3,600,000 435,000
Tax on Rs.163,000 @ 27.5% 44,825
479,825
Income tax on Rs.3,443,000
𝐴 𝑋 𝐶 = 479,825 x 3,443,000 439,021
𝐵 3,763,000

306 | P a g e
Less: Foreign tax credit
Tax paid in the foreign country 27,000 Tax in Pakistan at average rate
of tax
439,021/3,443,000 x 360,000 45,904
Whichever is lower 27,000
412,021
Less: Rebate on donation (479,825 / 3,763,000) x 50,000 6,376 Less: Taxes deducted / paid at
source 405,645
Tax payable with return of income 16,000 389,645

(c) Transfer of rebate from a member to its AOP


If the person is a member of an AOP, any tax credit that could not be claimed by such
member can be claimed by the AOP of which he is member, in the same year. For this
purpose, a copy of written agreement between the member and the AOP shall be
furnished along with the return of income of AOP.

(d) Share of profit from AOP in the hands of a company - Section 92


(a) If a company is a member in an AOP (e.g. in a joint venture) then:
• The share of such company shall be excluded from taxable income of aop;
• The share of the company shall be included in the taxable income of the
company; and therefore it is taxable at the applicable corporate tax rate which is
29%.
It means that the share of a company being a member in an AOP shall not be taxable in the
hands of AOP. The company is required to tax its share in the normal manner applicable for a
company.

(e) Loss incurred by AOP:


In case of loss incurred by an AOP and a company is a member then share of loss shall
not be excluded from tax loss as the AOP shall c/f its losses and no member is entitled to
set off or c/f the loss of AOP as per section 59A.
(f) Example 4
(a) Gross revenue of AOP 25,000,000
(b) Allowable tax expenses of AOP 11,000,000
(c) Taxes paid at source by AOP 2,000,000
(d) The AOP has three members with the following share:
• A Ltd 30% share
• Mr. Y 35% share
• Mr. Z 35% share
(e) A Ltd.’s taxable business income is Rs.3,000,000 other than share of profit from
AOP. Advance tax paid by A Ltd is Rs.1,300,000
(f) Mr. Y earned non-salaried taxable income Rs.4,000,000 in his personal capacity.
(g) Mr. Z does not have any other income.
(g) Required:
Calculate tax liability of each member.

307 | P a g e
Solution
Gross revenue of AOP 25,000,000
Less: Allowable tax expenses 11,000,000
Taxable income of AOP including company's share 14,000,000
Less: A Ltd's share for separate consideration 30% of 14m 4,200,000
Taxable income excluding A Ltd's share 9,800,000
Income tax on Rs.4,000,000 765,000
Income tax on Rs.5,800,000 @ 35% 2,030,000 2,795,000
Income divisible between Mr. Y and Mr. Z 7,005,000
Tax payable by AOP 2,795,000
Tax liability as above 2,000,000
Advance tax paid 600,000
Less: claimed by A Ltd 1,400,000
Tax payable with return of income 1,395,000
Share of Mr. Y and Mr. Z Rs.3,502,500 each

Tax liability of A Ltd


29% of Rs.4,200,000+ 3,000,000 2,088,000
Less: Advance tax paid by A Ltd 1,300,000
Tax Payable 788,000

Tax liability of Mr. Y (Non-Salaried Case)


Taxable income 4,000,000
Add: share of profit from AOP for rate purpose 3,502,500
Taxable income for rate purpose 7,502,500
Income tax on Rs.7,502,500
Income tax on Rs.4,000,000 765,000
Income tax on Rs.3,502,500 @ 35% 1,225,875
1,990,875
Income tax on Rs.4,000,000
A xC = 1,990,875 x 4,000,000 1,061,446
B 7,502,500

Tax liability of Mr. Z is nil

1. Co-ownership of property - Section 66


Where any property is owned by two or more persons and their respective shares in that
property are definite and ascertainable then co-ownership of property is not considered as an
AOP for the purpose of

308 | P a g e
rental income and therefore rental income is not taxable as a separate tax entity. Share of taxable
rental income is taxable in the hands of each co-owner.
This provision is applicable in each case other than income from business. Examples may be:
• Rental income under the head income from property
• Rental income under the head income from other sources
• Capital gain on disposal of shares with joint ownership
(h) Note for students in respect of co-ownership:
Co-ownership of property shall not be considered as an AOP in respect of rental income.
However, if an AOP (already established as an AOP) buys any property from its resources then
the rental income shall be taxable in the hands of AOP despite the fact that the property is
registered in the name of one or more partners.
In this case the AOP shall be the beneficial owner of the property.
Similarly, if any property is owned by two or more persons and their respective shares in that
property are not definite and ascertainable, the property will be considered as being jointly
owned by an AOP and taxable income and tax payable thereon will be computed as per the
principles of taxation for AOP.

Other areas relating to AOP

2. Change in the constitution of an AOP - Section 98A


Where a change occurs in a tax year in the constitution of an AOP, liability of filing the
return on behalf of the AOP for the tax year shall be on the AOP as constituted at the time
of filing of such return but the income of the AOP shall be apportioned among the
members who were entitled to receive it.
However, where the tax assessed on a member cannot be recovered from him it shall be
recovered from the AOP as constituted at the time of filing the return.

3. Discontinuance of business or dissolution of an AOP-Section 98B


Where any business carried on by an AOP has been discontinued, or where an AOP is
dissolved, all the provisions of the income tax ordinance shall apply as if no such
discontinuance or dissolution had taken place.
Every person, who was, at the time of such discontinuance or dissolution, a member of
such AOP and the legal representative of any such person who is deceased, shall be
jointly and severally liable for the amount of tax payable by the AOP.

4. Succession to business, otherwise than on death-Section 98C


(1) Where a person carrying on any business (predecessor) has been succeeded in any
tax year by any other person (successor), otherwise than on the death of the
predecessor, and the Successor continues to carry on that business,-
(a) the predecessor shall be liable to pay tax in respect of the income of the tax
year in which the succession took place up to the date of succession and of any
previous tax year; and
(b) The successor shall be liable to pay tax in respect of the income of such tax
year after the date of succession.
(2) Where the predecessor cannot be found, the tax liability in respect of the tax
year in which the succession took place up to the date of succession and of any
previous tax year shall be that of the successor and all the provisions of the income
tax ordinance shall apply accordingly.
(3) Where any tax payable in respect of such business cannot be recovered from the
predecessor, it shall be recoverable from the successor, who shall be entitled to recover
it from the predecessor.

309 | P a g e
Income of a minor child (Sec 91)

Section 91 of the Ordinance states the procedure for levy of tax on the income of a minor child in the
following manner:
(xx) Any income of a minor child for a tax year chargeable under the head "Income from
Business" shall be chargeable to tax as the income of the parent of the child with the
highest taxable income for that year.
(xxi) The above provisions shall not apply to the income of a minor child from a business
acquired by the child through an inheritance.
Exercise
Associated Consultants is a joint venture (JV) of Mr. Ghulam Rasool and Consultancy Enterprises, a sole
proprietorship of Mr. Ahsan. The JV is not registered with registrar of firms. The proportion of interest of
the members in the JV is 70:30 between Mr. Ghulam Rasool and Mr. Ahsan respectively. The JV is engaged
in the providing of accounting, taxation and other services to different departments. During the year, total
Income of the JV under the normal tax regime was Rs.2,000,000 against the total revenue of Rs.30,000,000.
It is worth mentioning that Mr. Ahsan earned following income during the year:
Salary from Joint Venture = Rs. 450,000
Profit on debt from joint venture = Rs. 400 000
Compute the taxable income and tax liability of the Joint Venture for tax year 2024 (Ignore the
minimum tax provision.)
Answer

Profit as per accounts 2,000,000


Add Inadmissible deductions
Salary paid to members 450,000
Profit on debt paid to members 400 000
Taxable income 2,850,000
Tax on taxable income (Rs. 2,850,000) of JV i.e. Rs. 315,000 + 25% on 427,500
income exceeding Rs. 2.4m
Share of members out of profits of JV

Nature of income Mr.Ghulam Rasool Mr.Ahsan Total


Share in Profit 70% 30% 100%
Salary received - 450,000 450,000
Profit on debt received - 400,000 400,000
600,000
Balance Share 1,400,000 2,000,000
(30% x
(70% x 2,000,000) 2,000,000)
Income in hands of members of AOP 1,400,000 1,450,000 2,850,000

310 | P a g e
Question # 1 Spring 2010 Q. 1

Sohail, Khaled and Qazi are members of an association of persons (AOP) and share profit and loss the ratio of
2:2:1. The principal activity of the AOP is trading of various products.

Following are the details of the annual income / (loss) of the AOP and its members for the tax year 20X8:

(i) The AOP suffered loss before tax amounting to Rs.1,500,000. The loss has been arrived at after adjusting
rental income earned by the AOP, the details of which are as follows:

RS.
Rental income 2,000,000
Related Expenses:
Property tax 40,000
Depreciation 475,000 497,500
Net rental Income 1,502,500

No tax was withheld on the rental income or on trading activity.

(ii) The expenses debited to profit and loss account include the following amounts paid to the members of the
AOP:
Sohail Khaled Qazi
Salary (Rs.) 900,000 600,000 -
Interest on capital (Rs.) 300,000 300,000 500,000

(iii) Sohail earned Rs. 1,800,000 from another business, of which he is the sole proprietor.
(iv) Khaled received an amount of Rs.255,000 as share of income after tax from another AOP. He also earned
income of Rs.1,900,000 from a sole proprietorship concern owned by him.
(v) Qazi runs a part time business. His gross revenue is Rs.1 million whereas total business expenses are
Rs.150,000. He also paid taxes in advance amounting to Rs.100,000.

Required:
Compute the taxable income and tax liability of the AOP and each of its members. (19)

Question # 2 Spring 2015 Q. 5

(a) Under the provisions of the Income Tax Ordinance, 2001 state the rules relating to residential status of an
Association of Person (AOP). Also explain the taxability of income of AOP, in the hands of the firm and
its members. (05)

(b) State the rules relating to set-off and carry-forward of losses of AOP and its members. (02)

Question # 3 Spring 2016 Q. 8

Baqir, Asad and Rahi are members of an association of persons (BAR) and share profits and losses in the ratio
of 5:3:2 respectively. BAR is engaged in the business of trading consumer electronics and has two independent
branches one each in Tehran and Dubai. Following information has been extracted from BAR’s profit and loss
account for the year ended 31 December 2015:

Rupees
Sales 30,000,000
Cost of sales (20,500,000)
Gross profit 9,500,000

311 | P a g e
Administrative and selling expenses (4,732,000)
Financial charges (980,000)
Other income 1,700,000
Profit before taxation 5,488,000

Additional information:
Cost of sales includes:
(i) Closing stock which has been valued at net realizable value of Rs. 1,820,000. The cost of closing stock
under absorption costing was Rs. 1,950,000.
(ii) Provision of Rs. 75,000 against slow moving stores and spares.
(iii) Freight charges of Rs. 260,000. These were paid in cash to Momin Goods Transport for transporting goods
to customers in Multan.

Administrative and selling expenses include:


(i) Commission of Rs. 290,000 paid to Baqir, annual performance award of Rs. 310,000 paid to Rahi and Rs.
455,000 paid to AB Bank Limited in final settlement of a loan obtained by Asad for the construction of
his house in Muree.
(ii) Provision for bad debts of Rs. 735,000. The opening and closing balances of provision for bad debts
amounted to Rs. 1,100,000 and Rs. 1,435,000 respectively. Bad debts written off include a loan of Rs.
285,000 provided to a supplier.
(iii) Sales promotion expenses of Rs. 275,000. These expenses were paid by Rahi through his personal credit
card.
(iv) Rs. 86,000 paid to an institution operated by Federal Government for the training of industrial workers in
Punjab.

Further information:
For the year ended 31 December 2015 Dubai branch made a profit of Rs. 1,500,000 and Tehran branch made
a loss of Rs. 1,800,000. These figures are not included in the above profit and loss account.

Required:
Under the provisions of the Income Tax Ordinance, 2001 and Rules made thereunder, compute the taxable
income, net tax payable by BAR and the amount to be carried forward, if any, for tax year 2016. Assume tax
and accounting depreciation is same. (12)
Note: Your computation should commence with the profit before tax figure of Rs. 5,488,000.
Show all relevant exemptions, exclusions and disallowances.

Question # 4 Spring 2017 Q. 3(a)

On 1 June 20X6 Dawood and Dewan jointly purchased a bungalow for Rs. 35 million. They paid the amount
in the ratio of 65:35 respectively. To arrange funds for the deal, Dawood borrowed Rs. 3,000,000 in cash from
Shameem who is in the business of lending money. The rate of interest is agreed @ 20% per annum.

On 1 July 20X6, the house was let out to a company at annual rent of Rs. 4,500,000 inclusive of an amount of
Rs. 75,000 per month for utilities, cleaning and security. For providing these services Dawood and Dewan paid
Rs. 35,000 per month. During the tax year 20X7 they also paid Rs. 10,000 as collection charges and Rs. 230,000
for administering the property.

Required:
Compute taxable income of Dawood and Dewan under appropriate heads of income for the tax year 20X7. (08)

Question # 5 Spring 2019 Q. 1

Mustafa, Ali and Zain are partners of a resident firm in Pakistan, under the name and style MAZ Enterprises
(MAZE) which is engaged in manufacturing and local supply of auto spare parts. All partners have equal share

312 | P a g e
of profits and losses in the firm.

Following information has been extracted from accounting records of MAZE for the tax year 20X9:

Rs. in '000
Sales 140,400
Cost of goods sold (91,260)
Gross profit 49,140
Administrative and selling expenses (21,430)
Financial charges (15,740)
(37,170)
Other income 1,900
Profit before tax 13,870

Additional information:
(i) The above accounts have been prepared on cash basis and stock-in-trade has been valued on the prime-
cost method. However, the partners want to change the method of accounting from cash basis to accrual
basis. In this respect, following information has been gathered:

Opening balances Closing balances


-------------- Rs. in ‘000 -------------
Stock-iii-trade using prime-cost method 5,200 7,500
Stock-in-trade using absorption-cost method 5,900 8,800

(ii) Cost of goods sold includes cost of used machinery imported from China on 31 July 20X8 amounting to
Rs. 2,110,000. The cost includes payment of custom duty of Rs. 90,000 and income tax of Rs. 110,000 to
the Collector of Customs.
(iii) Administrative and selling expenses include:
▪ payment of Rs. 380,000 to a local hotel for holding annual eid-milan party for the employees, key
customers and their families.
▪ payment of a fixed monthly remuneration of Rs. 150,000 to each partner.
▪ payment of Rs. 180,000 for purchase of accounting software on 1 January 20X9. The software is
expected to be used for fifteen years.
(iv) Financial charges are net of interest income of Rs. 360,000 (net of tax @ 10% deducted by the bank),
earned by the firm on its savings accounts.

Required:
Under the provisions of Income Tax Ordinance, 2001 and Rules made thereunder, compute the total income,
taxable income and tax payable by MAZE using accrual basis of accounting. (10)
Note: Show all the relevant exemptions, exclusions and disallowances.

Question # 6 Spring 2020 Q. 2

Farhan, Kamran and Rehan are members of an association of persons (AOP) and share its profit and loss in
the ratio of 2:2:1 respectively.
Following information is available with regard to AOP and its members for the tax year 2020:
▪ During the year, AOP earned a profit before tax of Rs. 2,000,000 after making following payment to its
members:
Farhan Kamran Reha
n
------------------- Rupees -------------------
Salary 1,666,666 800,000 600,000
Interest on capital 566,666 400,000 300,0

313 | P a g e
00

▪ Kamran is running a business as a sole proprietor from which he earned Rs. 800,000. Kamran is also a
member of another AOP where his share of profit or loss is 60%. During the year, the other AOP incurred
a loss after tax of Rs. 350,000 and paid Rs. 150,000 on account of income tax.
▪ Rehan received net dividend of Rs. 102,000 from a listed company after deduction of withholding tax @
15%.
▪ Farhan has no other source of income.

Required:
Under the provisions of the Income Tax Ordinance, 2001 compute taxable income and tax liability of AOP
and each of its members for the tax year 2020. (11)

Question # 7 Autumn 2020 Q. 2

Libas & Co. is an association of persons (AOP) with three members, Saba, Junaid and Akram, sharing profit
and loss in the ratio of 1:1:2 respectively.

During the year, AOP earned profit before tax of Rs. 8,500,000 from its principal business i.e. trading of
garments. In addition, AOP is also involved in purchase and sale of following securities listed on the Pakistan
Stock Exchange:

Name of Details of purchase Details of sale


investee Date No. of Price per Date No. of Price per
company shares share (Rs.) shares share (Rs.)
XOK Limited 1 Oct 2016 200,000 200 29 June 2020 200,000 225
[Note A]
PBB Limited 18 Aug 2017 55,000 145 20 Dec 2019 100,000 180
10 Jan 2018 100,000 150
OOI Limited 15 Feb 2020 150,000 86 15 March 2020 150,000 78
[Note B]

Note A: Sale proceed from disposal of these shares was credited to the AOP’s bank account on 2 July 2020.
Note B: Due to shortage of funds for making this purchase, AOP borrowed Rs. 5,000,000 in cash from Imran,
who is in the business of lending money at 15% per annum.

Other information related to Saba:


▪ During the year, she earned Rs. 1,500,000 by working as a freelance photographer.
▪ On 1 April 2020, Saba received Rs. 1,100,000 from Zafar in full settlement of a loan. The loan was provided
on 1 April 2019 at 10% per annum interest through proper banking channel.

Required:
Under the provisions of the Income Tax Ordinance, 2001, compute taxable income and tax liability of AOP and
Saba for the tax year 2020. (13)

Question # 8 Autumn 2021 Q. 5

Kamkaj & Co. is an association of persons (AOP) with three members namely Baqir, Omer and Sadabahar
(Pvt.) Limited (SPL), sharing profit and loss in the ratio of 20:30:50 respectively.

Following information is available with regard to AOP and its members for the tax years 2020 and 2021:
(i) AOP’s income for tax years 2020 and 2021:

2020 2,021

314 | P a g e
------------------ Rupees ---------------
Income from business* (18,000,000) 25,000,000
Dividend income - 4,000,000
*Net of annual fixed commission of Rs. 7,000,000 to SPL

(ii) On 1 February 2021, Baqir earned capital gain of Rs. 5,200,000 on sale of his property which was
purchased on 1 January 2018.
(iii) Omer also operates a sole proprietor business from which he earned profits of Rs. 6,000,000 and Rs.
2,500,000 in tax years 2020 and 2021 respectively.

Required:
Under the provisions of the Income Tax Ordinance, 2001 compute the following for the tax years 2020 and
2021:
▪ Taxable income of AOP
▪ Taxable income and tax liability of Baqir and Omer (10)

315 | P a g e
ANSWERS
Question # 1 Spring 2010 Q. 1

Sohail, Khaled and Qazi (AOP)


Computation of Taxable and Divisible Income
Business income
Accounting Loss (1,500,000)
Less: Income from property (taxable under
separate head) 1,502,500

Add: Salary to Sohail 900,000


Salary to Khaled 600,000
Interest to Sohail 300,000
Interest to Khaled 300,000
Interest to Qazi 500,000 2,600,000
Business income/loss (402,500)

Income from property


Chargeable rent 2,000,000
Repair allowance 1/5th of chargeable rent (400,000)
Property tax (40,000)
Depreciation: Inadmissible Nil 1,560,000
Taxable income 1,157,500

Tax liability on income under NTR (10,000 +


357,500 x 12.5%) 54,688

Divisible Profit 1,157,500

Share of profit from AOP


Sohail Khaled Qazi Total
Salary 900,000 600,000 -- 1,500,000
Interest 300,000 300,000 500,000 1,100,000
(577000
Balance (2:2:1) (577000) ) (288,500) (1,442,500)
Share of profit 623,000 323,000 211,500 1,157,500

Computation of Taxable income of each member Sohail Khaled Qazi


Income from business 800,000 900,000 --
Income from consultancy ( 1000000 - 150000) 850,000
Taxable Income 800,000 900,000 850,000
Divisible profit from another AOP 255,000
Share of AOP 623,000 323,000 211,500
Income Including Share from AOP 1,423,000 1,478,000 1,061,500

Income tax 99,025 108,650 42,688


[Income tax / Total Income ] x Taxable Income 55,671 66,160 34,182

Answer # 2 Spring 2015 Q. 5

(a)

316 | P a g e
▪ An association of persons is considered a separate entity from its members (partners). Accordingly,
the AOP is liable to tax separately from the members of the association.
▪ An AOP is treated as tax resident for a tax year if the control and management of its affairs is situated
wholly or partly in Pakistan at any time during that tax year.
▪ A resident AOP is taxable on its worldwide income.
▪ If the AOP has paid tax for a tax year, the amount received by a member of the association in the
capacity as member out of the income of the association shall be exempt from tax. However, such
share of member is included in income of the member for determination of tax rate i.e. for rate purpose
(discussed below in detail).
▪ If at least one of the members of an AOP is a company, the share of such company or companies shall
be excluded for the purpose of computing the total income of the AOP and the company or companies
shall be taxed separately at the rates applicable to the companies, according to their share.
▪ The AOP is entitled to set off and carry forward its losses as per normal procedure. However, share
of loss from AOP is neither adjustable against the income of its members nor it is considered for rate
purpose.
▪ Where a change occurs in the constitution of a firm during a tax year, the firm’s income is apportioned
amongst the members on a time basis.
▪ The responsibility for filing the tax return of a firm for any tax year rests with the persons who are
members in the firm on the date the firm is required to file its tax return.
▪ Where a firm dissolves or discontinues carrying on business, any tax payable by the firm is recoverable
from any person who was a member in the firm at the time of dissolution or discontinuance of
business. Tax payable can also be recovered from legal heirs of the deceased partners.

(b)
▪ An AOP whose profits are chargeable to tax under Income Tax Ordinance, 2001 shall be entitled to a
set off or carry forward and set-off any loss of AOP against the income of AOP in the manner
explained above.
▪ Any members of an AOP shall not be entitled to set off or carried forward and set off loss of AOP
against his income.
▪ Any person who has succeeded in the business of another person otherwise than by inheritance, shall
not be entitled to set off or carry forward and set off loss of predecessor.

Examiner Comments:
(a) This was a straightforward question requiring the candidates to state the provisions
related to determination of the residential status of an Association of Persons [AOP].
Majority of the candidates did not know that residential status is always with
reference to a ‘tax year’. They also ignored a very important provision i.e. that an AOP
is considered a resident even if the control and management of its affairs is situated
in Pakistan ‘at any time during the tax year’ as many students stated incorrectly that
an AOP would be considered as resident if its control and management is situated in
Pakistan for more than 183 days.

Further, as regards taxability of income of AOP in the hands of its members, majority
of the candidates confined their answers to the taxability of income received by
individuals but did not mention the provisions of Income Tax Ordinance, 2001
regarding taxability of income of AOP where a company is a member of the AOP.

Majority of the candidates were aware of taxability of income of AOP in the hands of
its members. However, they confined their answers to the taxability of income
received by individual members of AOP. Only few candidates were able to elucidate
the taxability of share of income received by a company in its capacity as member of
the AOP.

(b) In this part, rules relating to set-off and carried forward of losses of AOP were

317 | P a g e
required to be explained. Candidates were generally aware that an AOP is entitled to
set-off and carry forward its losses against its income only. However, many
candidates also mentioned that losses of a member can be adjusted against the
income of AOP and vice versa which was incorrect. Further, only few candidates were
able to explain that share of loss from AOP is not considered even for rate purposes.

Question # 3 Spring 2016 Q. 8

BAR Associates
Tax Year 20X6
Computation of Taxable Income and Tax Liability
Rs.
Net profit before tax 5,488,000
Add: Provision for slow moving stock 75,000
Commission to a partner, Baqir 290,000
Performance award to a partner, Rahi 310,000
Loan of a partner, Asad paid by the firm 455,000
Provision for bad debts 735,000
Sales promotion expenses paid other than business bank a/c 275,000
7,628,000
Less: Trading debt written off against provision 115,000
Pakistan source business income 7,513,000

Foreign source business income


Profit from Dubai branch 1,500,000
Loss from Tehran branch (1,800,000)
Net loss c/f [not adjustable against foreign source income] (300,000) --
Taxable income 7,513,000
Income tax on Rs. 4,000,000 765,000
Income tax on Rs. 3,513,000 @ 35% 1229,550
1,994,550

Working of bad debts written off:


Opening balance of provision account 1,100,000
Add: Provision for the year 735,000
1,835,000
Less: Closing balance of provision account (1,435,000)
Debts written off during the year 400,000
Less: Loan to supplier written off - inadmissible 285,000
Bad debt written off allowed for tax purpose 115,000

Examiner Comments:
This practical question carrying 12 marks required the students to compute taxable income,
net tax payable and the amount to be carried forward by an AOP having two independent
branches, one each in Tehran and Dubai. The overall performance was good. However,
some of the common mistakes are discussed below:
▪ An important instruction in the question is that the computation should commence with
the profit before tax figure was ignored. Instead, adjustments were made in various line
items and the entire Profit & Loss Account was prepared which was a time consuming
exercise.
▪ Profit and loss of foreign branches were combined with profit from Pakistan source
income. Some student adjusted Dubai branch profit against Pakistan source profit and
carried forward Tehran branch loss whereas profit of one foreign branch could have been

318 | P a g e
adjusted from the loss of the other foreign branch to arrive at net loss from foreign
operations; and carried forward. Some students mentioned that profit and loss of Dubai
and Tehran branch could not be adjusted because they would be subject to final tax.
▪ Many students rightly mentioned that freight charges could be paid in cash without any
limit and expenses incurred on training of employee to Federal Government run
institution were allowable but deducted these expenses from profit as admissible expense
as they did not realize that these expenses had already been deducted from income. Some
students treated Rs. 86,000 paid to an institution operated by the Federal Government for
training of industrial workers in Punjab as donation and calculated tax credit thereon.
▪ Majority of the students failed to calculate the amount of bad debt written off and allowed
for tax purpose. Most of those who calculated them correctly, did not realize that loan to
the supplier written off was not an admissible expense as it had not been included in
business income previously.
▪ Difference between the value of stock under absorption costing and net realizable value
was deducted from accounting profit instead of treating it as an addition.
▪ Surprisingly, some students could not apply the income tax slabs correctly.

Marking Scheme:
▪ Computation of the admissible / inadmissible expenses
- Bad debts expense 3.0
- Stock 1.5
- Up to 01 mark each for other items 4.5
▪ Foreign loss to be carried forward 2.0
▪ Determination of net tax liability 1.0

Question # 4 Spring 2017 Q. 3(a)

Dawood
Income from other source Rupees
Cash loan 3,000,000
From Utilities, cleaning & security (480,000 (w-2) x 65%) 312,000
3,312,000

Income from Property


Rent (2,736,000 (w-1) x 65%) 1,778,400
Total/Taxable Income 5,090,400
Tax Liability (765,000 + 1,090,400 x 35%) 1,146,640

Dewan
From Utilities, cleaning & security (480,000 (w-2) x 35%) 168,000
Rent (2,736,000 (w-1) x 35%) 957,600
Total/Taxable Income 1,125,600
Tax liability (15,000 + 325,600 x 15 %) 63,840

W-1 Income from property


Annual Receipt 4,500,000
Less : Income from other source (75,000 x 12) (900,000)
Rent chargeable to tax (Gross) 3,600,000
Less: Deductions
Repairs ( 20% x 3,600,000) (720,000)
Collection charges (240,000 or 4% of 3,600,000) (144,000)
Income from Property (Net) 2,736,000

319 | P a g e
W-2 Income from Other Source
Rent for utilities 900,000
Less : Expenses (420,000)
480,000

Examiner Comments:
According to the scenario presented in this part of the question, Dawood and Dewan had
jointly purchased a bungalow and had rented it to a company for an amount which was
inclusive of utilities and other services. The requirement was to compute the taxable
incomes of both individuals under appropriate heads of incomes. Most of the candidates
performed in an average manner scoring between 3 to 6 marks. Some of the common
mistakes were as follows:
▪ Some candidates considered the joint ownership of the property as an AOP.
▪ Many candidates did not know that the amount included in rent for the provision of
amenities and utilities should be taxed under the head “Income from other sources”.
▪ A number of candidates deducted collection and administrative charges against rent
whereas such deductions against income from property are allowed only where the
property is owned and rented by a company.
▪ Expenditure incurred in relation to utilities, cleanliness and security was deducted from
income from property. Instead, it should have been claimed against amount received on
account of utilities, cleanliness and security i.e. against income from other sources.
▪ Only few candidates were able to identify that loan borrowed by Dawood is to be
included in his taxable income under the head “Income from other sources” because any
amount taken as advance from any person not being a banking company otherwise than
through banking channel is treated as income of the recipient.

Marking Scheme:
Computation of:
▪ income from property 4.0
▪ income from other sources 4.0

Question # 5 Spring 2019 Q. 1

MAZE (AOP)
Tax Year 20X9
Computation of Taxable and Divisible Income
Rs.
Net profit as per accounts 13,870,000
Add: Adjustment on account of change in accounting method
Closing stock under Absorption costing method 8,800,000
Closing stock under Prime cost method (7,500,000) 1,300,000
Cost of machinery charged in the accounts 2,110,000
Salary to partners 150,000 x 3 x 12 5,400,000
Accounting software charged in the accounts 180,000
22,860,000
Less: Initial allowance on machinery 25% of (2,110,000 -
110,000) 500,000
Normal depreciation on machinery @ 15% 225,000
Amortization of software (180,000 / 15) x 181 /
365 5,950
Interest income as separate block of income 360,000 1,090,950
21,769,050
Business Income

320 | P a g e
Income from Other Sources
Interest income as separate block of income
360,000 / 90% 400,000 --
Taxable Income 21,769,050
Income tax on Rs. 4,000,000 765,000
Income tax on Rs. 17,769,050 @ 35% 6,219,168 6,984,167
Add: Tax on Profit (360,000/.90) x 15% 60,000
Total tax liability 7,044,167
Advance Tax (110,000)
Tax deducted on Profit (40,000)
Tax Payable 6,894,167

Profit after Tax 14,874,883

Share of Profit from AOP


M A Z Total
1,800,00 1,800,0
Salaries from MAZE 0 00 1,800,000 5,400,000
5,456,35 5,456,3
Balance (equal share) 0 50 5,456,350 16,369,050
7,256,35 7,256,3
Share of profit for the year 0 50 7,256,350 21,769,050

Taxable income and tax liability of Mr. Z


Salary
Monthly salary 200,000 x 12 2,400,000
Reimbursement of medical 3,500,000 exempt
Purchase of car 250,000 - 110,800 139,200
Taxable Income 2,539,200
Add: Share of profit from AOP for rate purpose 7,256,350
Taxable Income for rate purpose 9,795,550

Tax liability (Salaried Case)


Tax Liability [1,095,000 + (9,795,550 - 6,000,000)
x 35%] 2,423,425
Tax liability on actual income [2423425/9795550 x
2539200] 628,199

Examiner Comments:
(a)
(i) Students were aware that profit before taxation was to be adjusted on account of
change in the accounting method from cash basis to accrual basis. However, they failed
to consider that only the difference between the values of closing stock under
absorption costing and prime cost method was to be added to the accounting profit.
Further, difference between the opening balances of the stock-in-trade under the two
methods required no adjustment against the accounting profit.
(ii) Payment for holding annual eid-milan party was treated as inadmissible expense.
(iii) Students did not add back interest income, being a separate block of income, to the
profit before taxation. Similarly, they did not deduct the same from the total income in
order to arrive at the taxable income for the year. Consequently, they failed to correct
total income.
(iv) Adjustment relating to income tax paid at import stage in the cost of machinery.
Further, few candidates wrongly excluded the custom duty from the cost of machinery

321 | P a g e
in order to arrive at the depreciable value of machinery.
(v) Normal depreciation on machinery was calculated on cost instead of cost less initial
allowance.
(vi) Correct amount of initial allowance on imported machinery was calculated in the
working but this was not shown as a deduction / admissible expenditure in the main
calculation.
(vii) Amortisation of computer software was computed based on number of months instead
of number of days in use.
(viii) Tax on interest income at the rate of 10% being a separate block income was not
computed.
(ix) Tax deducted at source was not adjusted against the tax liability.
(b)
(i) While calculating divisible profit for each partner, the students considered both NTR
and FTR instead of taking NTR only for rate purpose.
(ii) Reimbursement of actual cost of medical services was treated as taxable.
(iii) Share of profit from AOP was not included in the taxable income of the partner for rate
purposes. Among those who considered the share of profit, failed to exclude monthly
remunerations from the partner’s share of firm’s profit.

Marking Scheme:
(a)
▪ Computation of total income:
− 0.5 mark for each adjustment relating to inadmissible expenses/admissible income 4.5
− 0.5 mark for each adjustment relating to admissible expenses /exempt (or FTR )
income 3.5
▪ Determination of taxable income 0.5
▪ Determination of tax liability 1.5
(b)
▪ Computation of taxable income without share of AOP 1.5
▪ Computation of share of AOP 3.5
▪ Determination of tax liability 2.0

Question # 6 Spring 2020 Q. 2

M/S Farhan , Kamran and Rehan


Computation of Taxable Income and Tax Liability of
AOP
For the tax year 2020
Income from business Rupees
Business profit for the year 2,000,000

Add: Inadmissible expenses


Salary to Farhan 1,000,000
Salary to Kamran 800,000
Salary to Rehan 600,000
Interest to Farhan 500,000
Interest to Kamran 400,000
Interest to Rehan 300,000
3,600,000
Taxable income for the year 5,600,000

Tax liability
Tax on Rs. 4,000,000 750,000

322 | P a g e
@ 35% on amount exceeding Rs. 4,000,000 560,000
1,310,000
Amount available for distribution among the members of AOP 4,400,000

Computation of Taxable Income and Tax Liability of each Member


Farhan Kamran Rehan
------------------- Rupees -------------------
Income under NTR
Salary 1,000,000 800,000 600,000
Interest 500,000 400,000 300,000
Share of profit
(2,000,000÷5=180,000×2:2:1) 800,000 800,000 400,000
Share of profit from FKR 2,000,000 1,300,000
2,300,000
Share of loss from another AOP(Not - - -
allowed)
Income from sole proprietorship - 800,000 -
businesses
Taxable income 2,300,000 2,800,000 1,300,000

Income for rate purposes 2,300,000 2,800,000 1,120,000

Tax liability of Kamran on income from other business


Tax on Rs. 2,400,000 - 315,000 -
@ 25% on amount exceeding Rs. 2,400,000 - 100,000 -
Total tax payable - 415,000 -

Tax rate to be charged 0% 14.82% 0%


Tax liability of Kamran - -

Income under FTR


Dividend income (Rs.102,000 /0.85) 120,000

Tax on dividend income (Rs. 120,000×15%) 18,000


Tax deducted at source (18,000)
Tax liability of Rehan -

Examiner Comments:
▪ In case of AOP, salaries and interest paid on capital to partners were not considered as
inadmissible deductions. Similarly, in the hands of individual members of an AOP, these
amounts were not considered as part of their taxable income for rate purposes.
▪ Kamran’s share of loss from second AOP was adjusted against his share of profit from
first AOP for computing taxable income for rate purposes. On the other hand, many
examinees did not add his income from sole proprietorship business for calculating his
total taxable income.
▪ Tax liability of Kamran on income from other sources was ignored.
▪ The amount of dividend received by Rehan was not considered as FTR income and the
net amount of dividend instead of the gross amount was added to his taxable income.
▪ Examinees failed to appreciate that members of AOP who did not have any other source

323 | P a g e
of income, were not liable to pay any tax on their income from AOP.
▪ Examinees were of the opinion that Farhan and Rehan were also liable to pay tax on
their income.

Marking Scheme:
▪ Computation of income from business of AOP and amount available for distribution
among its members 2.0
▪ Computation of taxable income and tax liability of:
− Farhan 1.5
− Kamran 5.0
− Rehan 2.5

Question # 7 Autumn 2020 Q. 2

Libas & Co.


Computation of taxable income and tax liability of AOP
For the tax year 2020

Rupees
Income from business
Business profit for the year 8,500,000

Income from capital gain


Gain from sale of shares of XOK Limited [200,000×(225-200)] 5,000,000
Gain from sale of shares PBB Limited [55,000×(180–145)] 1,925,000
Gain from sale of shares PBB Limited [45,000×(180–150)] 1,350,000
Loss from sale of shares of OOI Limited [150,000×(78–86)] (1,200,000)
Total capital gain 7,075,000

Income from other sources


Loan received in cash 5,000,000

Total income 20,575,000


Less: Separate block of income - income from sale of securities (7,075,000)
Taxable income of AOP under NTR 13,500,000

Tax liability
On Rs. 4,000,000 765,000
On remaining Rs. 9,500,000 @ 35% 3,325,000
4,090,000
Tax on capital gain - separate block of income [7,075,000×12.5%] 884,375
Total tax payable 4,974,375

Saba
Computation of taxable income and tax liability of Saba
For the tax year 2020
Rupees
Profit on debt at 10% [1,100,000×(10/110)] 100,000
Freelance income 1,500,000
Taxable income 1,600,000
Share of profit from AoP (13,500,000×1/4) 3,375,000
Taxable income for rate purpose 4,975,000

324 | P a g e
Tax rate applicable on Saba
Tax on 4,000,000 765,000
Tax on remaining 975,000 @ 35% 341,250
Total tax liability 1,106,250

Tax rate to be charged (1,106,250/4,975,000) 22.23%

Tax liability of Saba (1,600,000×22.23%) 355,779

Examiner Comments:
▪ FIFO method was not applied correctly in computation of gain from sale of shares of
PBB. Consequently, examinees were not able to compute the cost of shares and amount
of gain correctly.
▪ Gain from sale of shares of XOK was not computed.
▪ Loan received in cash from Imran was not taken as income under the head 'Income
from other sources'.
▪ Tax liability of AOP was computed using slab rates of salaried individual.
▪ Average rate of tax was not calculated for computation of Saba's tax liability.

Marking Scheme:
▪ Computation of:
− taxable income of AOP from business 0.5
− taxable income of AOP from capital gain 4.0
− taxable income of AOP from other sources 1.0
− tax liability 2.5
▪ Computation of:
− taxable income of Saba 2.0
− share of profit from AOP 1.0
− tax liability 2.0

Question # 8 Autumn 2021 Q. 5

Kamkaj & Co.


Computation of Taxable Income
Tax Year 2020 Tax Year 2021
----------- Rupees -----------
Income from Business (18,000,000) 25,000,000
Add: Commission to SPL 7,000,000 7,000,000
(11,000,000) 32,000,000
Dividend income - 4,000,000
Total income (11,000,000) 36,000,000
Less: FTR income - (4,000,000)
Taxable income (11,000,000) 32,000,000
B/F loss (11,000,000)
Taxable income 21,000,000
Less: SPL’s share of 50% (10,500,000)
Taxable income of AOP after deducting SPL’s share 10,500,000

Tax liability of AOP: Rupees


Upto 6,000,000 1,330,000
On balance @ 35% 1,575,000

325 | P a g e
2,905,000

Computation of taxable income and tax liability of Baqir:


Tax Year 2020 Tax Year 2021
----------- Rupees -----------
Capital gain on sale of property (5,200,000) - 5,200,000

Tax liability on capital gain related to sale of immoveable property [5,200,000×5%] 260,000

Computation of taxable income and tax liability of Omer:


Tax Year 2020 Tax Year 2021
----------- Rupees -----------
Income from Business 6,000,000 2,500,000

Taxable income 6,000,000 2,500,000


Share of profit of AOP - 5,823,000
9,705,000[12,500,000(25,000,000×50%)–
2,795,000]×60%
Taxable income for rate purpose 6,000,000 8,323,000

On 6,000,000 1,3300,000 1,330,000


On balance @ 35% - 813,050
1,330,000 2,143,050
Tax rate to be charged 24.43%
(2,143,050÷8,323,000)
Tax liability of Omer 1,330,000 643,713
(2,500,000×25.74%)

Examiner Comments:
▪ While computing taxable income of AOP, commission to SPL was treated as an
admissible expense.
▪ Brought forward losses from tax year 2020 was not adjusted with taxable income of
tax year 2021.
▪ In order to compute tax liability of AOP, SPL's share was not deducted from taxable
income of AOP.
▪ Share of profit from AOP in tax year 2021 was allocated to Baqir whereas he had only
separate block of income i.e. capital gain on sale of property in tax year 2021.
▪ Capital gain on sale of property by Baqir was not reduced by 75% despite that the
holding period of property was between 3 to 4 years.
▪ Omer's share of profit from AOP in tax year 2021 was calculated incorrectly.

Marking Scheme:
▪ 2020
- Taxable income of AOP 1.0
- Taxable income and tax liability of Omer 2.0
▪ 2021
- Taxable income of AOP 3.0
- Taxable income and tax liability of Baqir 1.0
- Taxable income and tax liability of Omer 3.0

326 | P a g e
Chapter 14 LOSSES & Unabsorbed
Depreciation
This chapter deals with the following issues:
1. Set off of losses
2. Carry forward of losses including unabsorbed depreciation and amortization
Set off of losses
If a person sustains a loss under any head of income in a tax year, the same can be set-off against the income
from any other head of income except for “income under the heads ‘salary’
For example, a business loss can be set-off against income under the head “Income from other source” or any
other head of income.
Exception to the Normal Rule
• Loss in speculation business cannot be set-off against any other income. Capital loss cannot be set-off
against any other income.
• Loss from any head of income falling under final tax regime.
• Loss in a case where the income would have been an exempt income e.g. loss of agriculture income
cannot be set-off against any taxable income.
• No loss except loss under the head “Income from Business” (including income from speculation
business) and “Capital gain” can be carried forward.
• If a person sustains loss under the head “Income from Business” in addition to loss under any other
head, the loss under the head “Income from Business” shall be set-off last.
Carry forward of losses
Carry forward of Business loss
➢ If business loss (other than depreciation, amortization and speculation loss) sustained in a tax year
cannot be fully set-off in that year with income under any other head, it can be carried forward to
subsequent tax years.
➢ No loss shall be carried forward to more than six tax years immediately succeeding the tax year in which
the loss was first computed.
➢ If a person has a business loss carried forward for more than one tax years, the loss of earliest tax year
shall be set-off first.
➢ Where a loss, referred above relating to a tax year commencing on or after the First day, of July, 2020
is sustained by a resident company engaged in the hotel business in Pakistan the said loss shall be carried
forward for a period of eight years.
Carry forward of speculation business losses
➢ If a person sustains a loss in a tax year from any speculation business ,he can set off such loss only
against profits of any other speculation business carried on by him during the same tax year.
➢ If any loss cannot be so set-off, either wholly or partly, such portion of loss can be carried forward up
to six tax years immediately succeeding the tax year in which it was sustained.
➢ In the subsequent years too, the speculation loss can be set-off against income of any speculation
business only. It means that loss from speculation business cannot be adjusted against income under
any other head.
➢ If a person has a speculation loss carried forward for more than one tax years, the loss of earliest tax
year shall be set-off first.

327 | P a g e
Carry forward of capital losses
➢ A capital loss sustained by a person during a tax year cannot be set-off against income of that person
under any other head of income, for that tax year.
➢ The amount of capital loss which cannot be set-off shall be carried forward up to six tax years
immediately succeeding the tax year in which the loss was sustained.
➢ In the subsequent years too, capital loss can be set-off against capital gain only. It means that capital
loss cannot be adjusted against income under any other head.
➢ If a person has a capital loss carried forward for more than one tax year, the loss of earliest tax year
shall be set-off first
Limitations on set off and carry forward of losses
➢ An AOP whose profits are chargeable to tax under Income Tax Ordinance, 2001 shall be entitled to a
set off or carry forward and set-off any loss of AOP against the income of AOP in the manner explained
above.
➢ Any members of an AOP shall not be entitled to set off or carried forward and set off loss of AOP
against his income.
➢ Any person who has succeeded in the business of another person otherwise than by inheritance, shall
not be entitled to set off or carry forward and set off loss of predecessor.
Head-wise situation of c/f is as under
Heads of income C/f Provisions
Salary Loss cannot arise under the head salary.
Any other loss cannot be adjusted against salary income
Income from property Loss in a tax year can be set-off against any other head of income other than
salary and FTR. Any unadjusted loss under the head income from property
cannot be carried forward.
Other source Loss in a tax year can be set-off against any other head of
income other than salary and FTR. Any unadjusted loss under the head other
sources cannot be carried forward.
Capital gain Section 37
Capital loss can be carried forward only against future capital gains up to 6
years next following the tax year in which the loss occurred.
Section 37A
Loss on disposal of securities under section section 37A (shares of public
company etc.) shall be set off only against the gain from any other securities
under section 37A and any unadjusted loss shall be carried forward to 3
subsequent tax years only against gain under section 37A.
However, loss under section 37 can be adjusted against taxable
capital gain under section 37A
The taxpayer will be entitled for adjustment of prior years' capital loss on
FIFO basis subject to condition that the capital loss for the current year, if
any, will be set-off first.
Speculation Business Speculation loss can be carried forward only against future speculation
gains up to 6 years next following the tax year in which the loss occurred.

Speculation business means any business in which purchase and sale


transaction of any commodity including shares is settled otherwise than
actual delivery or transfer of the commodity but does not include a contract:

a) to cover future price fluctuation of any commodity to be manufactured or


sold in future through actual delivery or transfer of the commodity.
b) to cover loss on account of future price fluctuation of shares held by an
investor or dealer.

328 | P a g e
c) by a member of forward market or stock exchange to cover any loss which
may arise in the ordinary course of his business as a member.
Business - other than Normal business loss in a tax year can be adjusted against any head of
speculation (i.e. normal income other than salary and FTR but it can be carried forward only against
business) future business income up to 6 years next following the tax year in which
the loss occurred. Loss in hotel business by a resident company in the tax
year
2021 and onwards shall be c/f up to 8 years next following the tax year in
which the loss occurred.
Example: Mr. A has declared his income / loss for the current tax year as under:
Business loss Rs.900,000
Taxable salary income Rs.1,200,000
Taxable other source Rs.800,000
Answer:
Taxable salary income 1,200,000
Taxable other sources 800,000
Business loss 900,000
Business loss c / f 100,000
[It cannot be adjusted against salary income]
Taxable income 1,200,000

Summary of losses

Heads of Income Treatment of Loss Loss Adjustment Adj. of gain against other
heads
Set off Carry
Forward

Salary Income N/A N/A N/A No Loss Can adjust

Income From Property setoff Biz + CG+IFO N/A No Loss Can adjust

Business Income Setoff / Carry Spec. +CG+ Biz +Spec IFP+IFO


Forward IFO

Speculative Income Carry Forward N/A Spec. IFP+BIZ+IFO

Capital Gain

u/s 37 Carry Forward N/A CG IFP+BIZ+IFO

U/s 38 N/A N/A N/A IFP+BIZ+IFO

u/s 37A (securities) Setoff / Carry N/A US 37-A IFP+CG, BIZ+IFO


Forward

Income from other Setoff CG +Biz N/A IFP+BIZ


Sources

329 | P a g e
Foreign Can only be adjusted against foreign income of same head. However, Pakistan losses can be
Losses adjusted against foreign gains.

Example of set off and c/f of normal business loss


Tax year 20X7 - Taxable other source 2,000,000
Normal business loss. 3,800,000
Tax year 20X8 - Taxable other source 2,400,000
Business income 1,100,000
Solution:
TAX YEAR 20X7
Taxable other source 2,000,000
Normal business loss 3,800,000
Business loss of tax year 20X7 c/f to tax year 20x8 1,800,000
TAX YEAR 20X8
Taxable other source 2,400,000
Business income 1,100,000
Less: B/f business loss 1,800,000 nil
Taxable income 2,400,000
Business loss of tax year 20X7 c/f to tax year 20X9 700,000

Unabsorbed Depreciation
Unabsorbed depreciation and amortizations of intangible or pre-commencement expenditure
➢ Tax deprecation (including initial allowance or accelerated depreciation) and amortization for tax year
or any Portion thereof which could not be absorbed against available profits represent unabsorbed
depreciation / amortization which is part of business loss and normal rues for set-off and c/f shall apply
except that there is no time limit for the purpose of c/f of this amount.
➢ However, where taxable income of a tax year is Rs.10 million or more then unabsorbed depreciation
and amortization shall be adjusted only against 50% of the taxable business income of that year after
adjusting any b/f normal business loss.
➢ Depreciation and amortization for the tax year shall be set off last.

Format to Solve the Question


Taxable income of year 2 before tax depreciation and amortization xxx
Lest: Income other than business income (xxx)
xxx
Less: Normal business loss b/f from year 1 (xxx)
xxx
Less: Unabsorbed depreciation and amortization of year 1 (xxx)
xxx
Less: Tax depreciation and amortization of year 2 (xxx)
Taxable business income xxx
Add: Income other than business income xxx
Taxable income xxx

330 | P a g e
Illustration:
A Ltd's business loss of year 1 was Rs.20,000,000 including unabsorbed depreciation of Rs. 12,000,000. The
company's taxable income before depreciation de year 2 is Rs.28,000,000 including income from other source
Rs.1,500,000. Tax depreciation of year 2 is Rs.8,200,000.
Compute the taxable income, if any, and the amount of loss to be carried forward by the company in year 2.

Answer:
Taxable income of year 2 before tax depreciation 28,000,000
Less: Income from other source 1,500,000
Taxable business income of year 2 before tax depreciation 26,500,000
Less: B/f normal business loss of year 1 8,000,000
18,500,000
Less: Unabsorbed depreciation of year 1 [50% of 18.5m] 9,250,000
Less: Tax depreciation of year 2 8,200,000
Taxable business income 1,050,000
Other source 1,500,000
Taxable income 2,550,000
Unabsorbed tax depreciation of year 1 c/f 12m - 9.25m 2,750,000
Foreign source income - section 104
a) foreign loss under a particular head of income shall be treated as a separate head of Income. Speculation
business shall also be treated as a separate head of income.
b) foreign loss cannot be adjusted against any Pakistan-source income. However, Pakistan source loss can be
adjusted against foreign source income.
c) Any foreign source loss under a head of income shall be carried forward only against foreign source income
under that head up to 6 years next following the tax year in which the loss occurred.
d) foreign tax credit, if applicable, shall be calculated separately for each head of foreign source income.
Other areas (Rebates Not Discussed Earlier)
Tax credit for certain persons - section 65F
(A) Following persons / incomes shall be allowed a tax credit equal to 100% of the tax payable including
minimum tax and FTR:
(a) persons engaged in coal mining projects in Sindh supplying coal exclusively to power generation projects;
(b) a start-up for the tax year in which the start-up is certified by the Pakistan Software Export Board (PSEB)
and the next following two tax years;
(i) Definition of startup - section 2(62A)
A business of a resident person that commenced on or after 1.7.2012 and the person is engaged in or intends to
offer technology driven products or services provided that the person is registered with and duly certified by
PSEB and has turnover of less than Rs.100 million in each of the last 5 tax years.
(ii) Startup established by woman
Tax payable by woman enterprise on business income shall be reduced by 25%
Definition of woman enterprise
Woman enterprise means a startup established on or company wed by a woman or AOP where all members are
women or a company whose 100% shareholding is held by women.

331 | P a g e
This benefit shall not be available to a business formed by the transfer or reconstitution or reconstruction or
splitting up on an existing business.
Note for students:
The benefit of 25% reduction in tax liability is available where 100% tax credit is not available for any reason.
(B) Conditions for tax credit
(a) return has been filed;
(b) withholding tax statements for the relevant tax year have been filed, if required; and
(c) sales tax returns have been filed, if required.

Definition of Online marketplace


"online marketplace" means an information technology platform run by e-commerce entity over an
electronic network that acts as a facilitator in transactions that occur between a buyer and a seller.
Benefits for Online marketplace:
• Minimum tax shall be 0.25% of turnover instead of 1.25% in other cases
• Tax rate on commission and brokerage shall be 5%

Tax credit for specified industrial undertakings - section 65G


(A) Capital investment in purchase and installation of new machinery, buildings, equipment, hardware and
software (excluding self-created software and used capital assets) shall be allowed an investment tax credit of
25% of the investment amount against tax payable including FTR and minimum tax by the following persons:
(a) green field industrial undertaking engaged in –
(i) manufacture of goods which substantially changes their original condition; ог
(ii) ship building:
Provided that the person incorporated between 30.6.2019 and 30.6.2024 and the person is not formed
by the splitting up or reconstitution of an undertaking already in existence or by transfer of machinery
or building from an undertaking established in Pakistan prior to commencement of new business and is
not part of an expansion project; and
(b) industrial undertaking set up by 30.6.2023 and engaged in the manufacture of machinery, equipment
and items with dedicated use (no multiple uses) for generation of renewable energy from sources like
solar and wind, for a period of five years beginning from the date such industrial undertaking is set up.
(B) Any unadjusted tax credit shall be carried forward to the two subsequent tax years.
Definition of industrial undertaking
Industrial Undertaking means –
(a) an undertaking which is set up in Pakistan and which employs, -
(i) ten or more persons in Pakistan and involves the use of electrical energy or any other form of energy which
is mechanically transmitted and is not generated by human or animal energy; or
(ii) twenty or more persons in Pakistan and does not involve the use of electrical energy or any other form of
energy which is mechanically transmitted and is not generated by human or animal energy: and which is engaged
in the manufacture of goods, ship-building, generation or distribution of electrical energy, or the supply of
hydraulic power or the working of any mine, oil-well or any other source of mineral deposits

332 | P a g e
(b) a person directly involved in the construction of buildings, roads, bridges and other such structures
or the development of land, to the extent and for the purpose of import of plant and machinery to be
utilized in such activity
(c) a resident company engaged in the hotel business in Pakistan; and
(d) telecommunication companies operating under the Telecommunication Authority (PTA).
Definition of greenfield industrial undertaking
"greenfield industrial undertaking" means [section 2(27A)] –
(a) a new industrial undertaking which is -
(i) set up on land which has not previously been utilized for any commercial, industrial or manufacturing
activity and is free from constraints imposed by any prior work;
(ii) built without demolishing, revamping, renovating, upgrading, remodeling or modifying any existing
structure, facility or plant;
(iii) not formed by the splitting up or reconstitution of an undertaking already in existence or by transfer
of machinery, plant or building from an undertaking established in Pakistan prior to commencement of
the new business and is not part of an expansion project;
(iv) using any process or technology that has not earlier been used in Pakistan and is so approved by
the Engineering Development Board; and
(b) is approved by the Commissioner on an application made in the prescribed form and manner, accompanied
by the prescribed documents and, such other documents as may be required by the Commissioner:

333 | P a g e
PAST PAPER QUESTIONS
Question # 1 Spring 2015 Q. 6

Aslam is a resident taxpayer who operates his business from Lahore (LHR) and Paris (PAR). In August 2014, he
established a new branch in Berlin (BER).

Following information is available in respect of his business operations for tax year 2015:

LHR PAR BER


-----Rs. in million-----
Income / (loss) from business 29 40 (15)
Advance taxes paid in respective countries during the year 10 5 3
Income from capital gain (net of income tax of Rs. 3 million) - 27 -
Carried forward losses:
Loss from business - 55 -
Capital loss - 6 -

The following amounts paid by Aslam in respect of BER have been charged to LHR:
(i) salaries for the first three months amounting to Rs. 5 million.
(ii) rent expense for the year amounting to Rs. 7 million.

Required:
Under the provisions of the Income Tax Ordinance, 2001 calculate the tax payable by Aslam in the tax year
2015 and foreign tax losses to be carried forward to next year, if any. (09)

Question # 2 Autumn 2016 Q. 8

On 1 July 2015 Mehreen joined a local newspaper as an investigative journalist at a salary of Rs. 300,000 per
month. Tax deducted u/s 149 from her salary amounted to Rs. 40,000 per month.

Following are the details of her income received from Germany; tax paid thereon and brought forward foreign
losses for tax year 2016:

Foreign income/ Foreign losses


Foreign tax paid
Heads of income (loss) brought forward

Speculation business 600,000 110,000 (380,000)


Non-speculation business 1,480,000 187,600 -
Other sources (1,500,000) - -
Capital gain 950,000 76,000 (1,800,000)

On 1 May 2016 Mehreen resigned from her current job and joined Akhbar Merhaba (AM), an Arabic
newspaper in Dubai, as editor-in-chief on a monthly salary equivalent to PKR 1,200,000. AM paid 50% of her
salary in Dubai and remitted the remaining 50% to her bank account in Pakistan through normal banking
channel. Mehreen remained in Dubai during the rest of the tax year 2016.

Required:
Under the provisions of the Income Tax Ordinance, 2001 and Rules made thereunder, compute the taxable
income, net tax payable by or refundable to Mehreen for tax year 2016 and the amount of foreign losses or

334 | P a g e
foreign tax credit, if any, to be carried forward. (10)

Question # 3 Autumn 2020 Q. 4(b)

Ahmed has completed his MBA from a university in USA. He had been living there since August 2013 for his
education and came to Pakistan only once in 2017 i.e. from 10 March 2017 to 30 September 2017 and then
went back to USA to complete his MBA. Along with his studies, he was also doing a part time job at a
restaurant in USA till November 2019. He returned to Pakistan on 1 December 2019 and commenced a trading
business from 1 January 2020.

Below is the computation for taxable income/loss for the tax year 2020:

Pakistan source Foreign source


Total
income income
Income from Salary -------------------Rupees-------------------
Salary from restaurant in USA 840,000 840,000

Income from business


Revenue 4,000,000 4,000,000
Less: Deductions
Cost of goods sold (2,200,000) (2,200,000)
Selling and administrative expenses [Note A] (2,820,000) (2,820,000)
Donation [Note B] (600,000) (250,000) (850,000)
Taxable income/(loss) (1,620,000) 590,000 (1,030,000)

Note A: Selling and administrative expenses include the following:


(i) Salaries of Rs. 840,000 paid to two employees equally in cash. Withholding income tax was deducted as
required under Income Tax Ordinance, 2001.
(ii) Rs. 600,000 in respect of the feasibility study which was conducted before commencement of the business.

Note B: Donation of Rs. 600,000 was paid to a charitable hospital in Pakistan and Rs. 250,000 was paid to a
non-profit organization in USA.

Required:
Under the provisions of the Income Tax Ordinance, 2001 and Rules made thereunder, comment on the above
tax computation for tax year 2020. Give suggestion(s) wherever required. (08)

Question # 4 Spring 2012 Q. 2(c)

List the persons or incomes that are allowed a tax credit equal to 100% of the tax payable. Also specify the
conditions/limitations which are required to be fulfilled for availing the said tax credit. (Ignore tax credit
available to charitable organization) (04)

Question # 5 Autumn 2021 Q. 2(b)

Following information pertains to Ms. Ayesha for the tax year 2021:

Rs. in million
Income from non-speculation business 15.0
Income from property 3.0

335 | P a g e
Gain on sale of jewellery 2.5
Gain on sale of listed securities 4.0
Loss from speculation business (4.5)
Loss on sale of shares of a private company (3.6)
Loss on sale of antique (3.6)
Loss on sale of listed securities (6.0)
Loss from agriculture Loss from other sources (19.0)

Required:
Under the Income Tax Ordinance, 2001, discuss how the above losses can be set off against her aforesaid
incomes. Also discuss the amount of losses that can be carried forward for adjustment against her future
incomes. (08)

336 | P a g e
ANSWERS
Answer # 1 Spring 2015 Q. 6

Mr. Aslam
Income and Tax thereon
Ty 2015
Rs in million
Income from business Pakistan Source (LHR) w-1 41
Income from business foreign source (w-3) 13
Less: b/f business losses (55)
Income from capital gain foreign income 30
Less : b/f business losses (6)
Taxable Income 65

Tax Liability (765,0000 + 35% x 61,000,000) 22.11


Less: Foreign tax credit (Lower of)
Atual foreign tax paid 3 (3)
Pakstan tax paid (22.11/65) x 24 8.16
Tax paid in Pakistan 19.11

W-1 Income from Business – (LHR)


Income from business 29
Add : expense wrongly charged
Salaries 5
Rent expense 7
41

W-2 Income from Business – (BER)


Income (15)
Add : Expense wrongly charged
Salaries (5)
Rent expenses (7)
(27)

W-3 Foreign source income from business


PAR 40
BER (w-2) (27)
13

Examiner Comments:
This question pertained to a resident taxpayer who operates his business from Lahore
and Paris and had established a branch in Berlin. The requirement was to calculate the
tax payable by the resident taxpayer in the tax year 2015 and foreign losses to be carried
forward to next year, if any.
A very poor performance was witnessed in this question as the candidates seemed to lack
conceptual understanding of the related law. Few candidates did not attempt the
question altogether. Following mistakes were commonly seen in the answer scripts of
those who did attempt:
▪ Majority of the candidates did not appreciate that foreign source income under each
head of income and for each city is a separate block and tax liability under each head

337 | P a g e
for each city, if any, has to be worked out independently. Majority of the students
calculated the business loss from Paris and Berlin and adjusted these against the
capital gains from Paris and business income from Lahore Branch to determine the
taxable income.
▪ According to the question salaries and rent expense of Berlin Branch had been charged
to Lahore Branch. Only about 50% of the candidates made appropriate adjustments in
the profits of the respective branches.
▪ Capital gains of Paris Branch were taken as Rs. 27 million i.e. the net amount after
payment of tax amounting to Rs. 3 million, instead of taking the gross amount of Rs. 30
million.

Answer # 2 Autumn 2016 Q. 8

Mehreen
Computation of taxable income and income tax liability
For the tax year 2016

---------------- Rupees ----------------


Pakistan source Foreign source Total
Income from salary:
Salary[300,000 × 10], [1,200,000 × 2] (a) 3,000,000 2,400,000 5,400,000

Income from business:


Speculation business - 600,000 600,000
Less: brought forward speculation loss - (380,000) (380,000)
Net income from speculation business - 220,000 220,000

Non-speculation business - 1,480,000 1,480,000


Net business income (b) 1,700,000 1,700,000

(Loss) from other source (Carried forward) - (1,500,000) (1,500,000)

Capital gain - 950,000 950,000


Less: brought forward loss - (1,800,000) (1,800,000)
Net capital loss (Carried forward) (850,000) (850,000)

Total income (a+ b) 3,000,000 4,100,000 7,100,000

Less: exempt income


salary received from AM - (2,400,000) (2,400,000)
Taxable income 3,000,000 1,700,000 4,700,000

Since salary income is less than 75% of the taxable income, therefore, the slab applicable to Non-salaried
individuals shall be applied.

Computation of net-tax liability:

Tax on Rs. 4,000,000 765,000


Tax @ 35% on the amount exceeding Rs. 4,000,000 (i.e. on 700,000) 245,000
Tax payable 1,010,000
Less: Tax credit

338 | P a g e
Which is lesser of (A) or (B): Business
Speculation Non-Speculation
Foreign taxes paid (A) 110,000 187,600
Proportionate Pakistan tax
[(1,010,000÷4,700,000) × 220,000](B) 47,277 - (47,277)
Proportionate Pakistan tax
[(1,010,000÷4,700,000) × 1,480,000] (B) - 318,042 (318,042)
(230,078)
Total tax liability for the year 644,680
Less: Tax withheld at source (40,000 × 10) (400,000)
Net tax payable for tax year 2016 244,680

Un-adjusted foreign tax credit shall not be refunded, carried back to preceding year or carried to the following
year.

Examiner Comments:
The question pertained to an individual who had earned Pakistan source as well as foreign
source income under four different heads of income. The requirement was to compute the
taxable income, net tax payable or refundable and the amount of foreign losses or foreign
tax credit, if any, to be carried forward. Poor performance was witnessed in this question
as the candidates seemed to lack conceptual understanding of the related law. The
common mistakes were as follows:
▪ Pakistan source salary was computed for 12 months instead of 10 months.
▪ Only 50% of the foreign source salary was considered exempt; salary remitted to
Pakistan was considered taxable, which was incorrect. The entire foreign source salary
was exempt from tax by virtue of section 51 of the Income Tax Ordinance, 2001.
▪ Foreign loss under other sources was required to be carried forward. Candidates wrongly
adjusted the same against foreign source non-speculative business income. Candidates
did not realize that loss under specific foreign source head can only be adjusted against
income under the same head and not with income under any other head.
▪ Some candidates treated foreign tax paid as an expense.
▪ Since salary income was less than 75% of the taxable income, therefore, the slab
applicable to Non-salaried individual should have been applied. Many candidates
applied the salaried slab.
▪ Many candidates did have a fair idea that tax credit was to be allowed on an amount
equal to the lesser of the foreign income tax paid, or the amount arrived at by applying
the average rate of tax (applicable to Pakistan source income) on foreign source income.
However, they struggled / failed to apply these concepts practically.
▪ In many scripts, proportionate Pakistan tax on foreign source income was calculated on
aggregate basis for comparison with total foreign taxes paid, instead of calculating the
Pakistan tax in respect of each foreign source and comparing the same with foreign tax
paid under each head separately.

Marking Scheme:
▪ Computation of income from salary – Pakistan as well as foreign source 1.0
▪ Computation of income from business – Foreign source 1.5
▪ Computation of loss from other sources – Foreign source 1.0
▪ Computation of income from capital gain – Foreign source 2.5
▪ Computation of tax credit for foreign taxes paid 2.0
▪ Computation of tax liability and net tax payable 2.0

339 | P a g e
Answer # 3 Autumn 2020 Q. 4(b)

(i) Ahmed is a returning expatriate because he was not a resident in any of the preceding four tax years.
Therefore, his foreign source income for the year from the restaurant is exempt.
(ii) Salary exceeding Rs. 15,000 per month should be paid through cheque or direct transfer to the
employees’ bank account after deduction of tax (if any), to claim it as a deduction from income from
business. In this case, although Ahmed had deducted tax at source from salaries, he paid the monthly
salary Rs. 70,000 to each employee in cash. Therefore, this expense will be disallowed.
(iii) Cost of feasibility study conducted before commencement of the business falls under the definition of
pre-commencement expenditure which is subject to 20% amortization on straight line bases. Therefore,
the full amount of this cost will be disallowed and Ahmed will only be allowed a deduction of Rs. 120,000
(600,000×20%) as amortization.
(iv)
▪ No tax credit will be allowed for donation paid to the non-profit organization in USA because it does
not fall under the definition of non-profit organization as per the income tax ordinance, 2001.
▪ Donation expense paid to local organization will be a deductible allowance only when it is paid to an
institution named under clause 61 of 2nd schedule.
▪ Otherwise, Ahmed can avail a tax credit under section 61 on the donation paid to the government
hospital on average rate of tax.

Examiner Comments:
▪ Revised computation was prepared by some examinees even though the question
specifically mentioned that it was not required.
▪ Examinees could not identify the cost of feasibility study as a pre-commencement
expenditure.
▪ Donation paid in USA was considered eligible for tax credit and also for direct deduction
in some cases.

Marking Scheme:
Suggesting the correct treatment of:
▪ income from salary 1.5
▪ income from business 2.0
▪ cost of feasibility studies 1.5
▪ tax credit 3.0

Question # 4 Spring 2012 Q. 2(c)

Tax credit for certain persons


Following incomes or taxpayers shall be allowed a tax credit equal to one hundred per cent of the tax payable:
(i) Persons engaged in coal mining projects in Sindh supplying coal exclusively to power generation projects;
(ii) A start-up for the tax year in which the startup is certified by the Pakistan Software Export Board and for
the following two years;
(iii) Persons deriving income from exports of computer software or IT services or IT enabled services up to
the period ending on 30 June 2025 if 80% of the export proceeds are brought into Pakistan in foreign
exchange through normal banking channels.

The above tax credit shall be available subject to fulfillment of the following conditions:
(i) Annual return of income has been filed;
(ii) Withholding tax statements for the relevant tax year have been filed, where the person is a withholding
agent; and
(iii) Sales tax returns for the tax periods corresponding to the relevant tax year have been filed, if the person

340 | P a g e
is required to file sales tax return under any of the Federal or Provincial sales tax laws.

Question # 5 Autumn 2021 Q. 2(b)

Rs. in
Nature of loss Set off Carried forward
million
Loss from (4.5) It cannot be set off against any It can be carried forward against future
speculation other head of income. speculation gain upto next 6 tax years
business OR following the tax year in which the loss
It can only be set off against any occurred.
other gain from speculation
business.
Loss on sale of (3.6) This is a capital loss and it can be The loss of Rs. 1.1m (3.6–2.5) can be
shares of set off against capital gain on sale carried forward only against future
private of jewellery of Rs. 2.5m. capital gain upto 6 tax years next
company following the tax year in which the loss
occurred.
Loss on sale of (1.6) This loss shall not be recognized. So no question of set off or carried
antique forward of this loss arises.
Loss on sale of (6.0) This is a capital loss and it can The loss of Rs. 2m(6–4) can be carried
listed securities only be set off against capital gain forward only against future capital gain
on sale of listed securities of Rs. on disposal of securities under section
4m. 37A upto 3 subsequent tax years.
Loss from (8.0) Since agriculture income is exempt from tax, this loss cannot be adjusted
agriculture against any other income.
Loss from other (19) It can be set off with income from The loss of Rs. 4m (19– 15) cannot be
sources normal (other than speculation) carried forward.
business of Rs. 15m but cannot
be set off with income from
property.

Examiner Comments:
▪ Carry forward period for loss on sale of listed securities was mentioned as six years.
▪ Loss from other sources was considered as eligible for carrying forward to next years.

Marking Scheme:
Discussion on set off and carried forward of:
▪ loss from speculation business 1.0
▪ loss on sale of shares of a private company 1.0
▪ loss on sale of antique 1.0
▪ loss on sale of listed securities 2.0
▪ loss from agriculture 1.0
▪ loss from other sources 2.0

341 | P a g e
CHAPTER 15
INCOME TAX PRACTICAL
QUESTIONS [AOP & BUSINESS]
Income from Business (Summary)
Taxable
Rs.
Sr. No Particulars Rs
Accounting profit/ Loss After (Before tax) XXX
ADD
BACK
a) In-Admissible expenses already charged to Profit/Loss Account
Tax Charge or levy paid to the Govt. on profits
• Cess, rate or income tax calculated and paid as a percentage of Profit
• Income tax deducted at source
• Provision for income tax
Note: XX
a) Custom duties on import of raw material and property tax on business premises is allowed
expense because these taxes are not based on profit.
b) Professional tax or subscription payments to trade associations working or welfare of
business is allowed expense

b) Amount of any tax deducted from an income earned by person XX


c) Tax deductions from payments
• Taxable Salaries {more than Rs. 600,000 p.a.) paid without deduction of tax at source
• Any other expenditure subject to withholding tax is paid without tax deduction
XX
• Purchases of raw materials and finished goods are paid without deduction of tax -
Disallowance should not exceed 20% of purchases cost.

d) Entertainment expenditure, XX
e) Contribution to Employee welfare funds XX
f) fine or penalty for the violation of any law, rule or regulation
• Penalty for late payment of sales tax
• Penalty for violation of environmental law XX
Note: Penalties for breach of contract in ordinary course of business, Late payment surcharge
with utility bills is an allowable expense.
g) Personal expenses (Drawings) of owner and his dependents charged as expense in P&L
• Repairs cost of personal vehicle ·.
• Medical expenses of himself' and any other family member · XX
• Personal legal expenses.
• School fee of children.
h) Amount transferred to a Reserve fund or capitalized
• Amount transferred to general reserve
XX
• Amount transferred to dividend equalization fund
• Amount transferred to debenture sinking fund
i) Payment to a member (partner) by an AOP
• Salary given to partner (member) of AOP XX
• Interest on loan (profit on debt) given to partner (member) of AOP

342 | P a g e
• Interest on capital given to partner (member) of AOP
• Brokerage (commission) given to partner (member) of AOP
• Any other remuneration of partner (member) of AOP
Any expense under single head more than Rs. 250,000 as well as individual payment exceeding
j) XX
Rs. 25,000 paid in cash
k) Salaries exceeding Rs. 32,000 per month paid through cash XX
l) Capital expenditures
• Purchase cost of depreciable asset charged as expense in P&L
• Construction cost of depreciable asset charged as expense in P&.L
• Installation cost of depreciable asset charged as expense in p &L
XX
• Additions of depreciable asset charged as expense in P&L
• Improvement cost of depreciable asset charged as expense in P&L
• Acquisition/Renewal/Development cost of intangible assets charged as expense in
P&L
Advertisement-and sales promotion expense by. a pharmaceutical manufacturer in
m) XX XXX
excess or 10% of sales revenue ·

Depreciable assets (Sec 22 & 23)


Add Accounting Depreciation of depreciable assets - disallowed expense XX
Add Accounting Depreciation of asset taken under finance lease - disallowed expense XX
Add Accounting loss on disposal of depreciable assets - disallowed expense XX
Add Tax gain on sale of depreciable assets - allowed income not credited in P&L
Sale proceeds
Less: Tax Written Down Value , XX
{Written down value + Tax depreciation for non-business use}

Add Tax gain on sale of immovable business properly (Sales proceeds are more than its original
cost)
Sale Proceeds
XX
Less: Tax Written Down Value
Original cost (taken equal to sales .proceeds)
Less: Accumulated tax depreciation
Add Tax gain on export (transfer out) of depreciable asset outside Pakistan
Consideration received (taken equal to its original cost) XX
Less:' Tax Written Down· Value
Accounting gain on disposal of depreciable assets- disallowed incomes
Less (XX)
Less Tax depreciation of depreciable assets allowed expense not charged in P&L (XX)
Less Tax loss on sale of depreciable assets - allowed expense not charged in P&L
Scrap proceeds
(XX) XXX
Less: Tax Written Down Value
{Written down value+ Tax depreciation for non-business use]

Intangibles (Sec 24)


Accounting amortization of intangibles - disallowed expense
Add XX
Add Accounting loss on disposal of intangibles - disallowed expense XX
Tax gain on disposal of intangibles (Sale proceeds - Tax WDV of intangibles) -
Add allowed income not credited in P&L XX
Note: Ignore non - business use for the purpose of tax gain.
Less Accounting gain on disposal of intangibles (XX)
Tax amortization of intangibles - allowed expense not charged in P&L ·
Less (XX)
Tax loss on disposal of intangibles (Sale proceeds - Tax WDV of intangibles}-allowed expense
Less (XX) XXX
not charged in P&L

343 | P a g e
Pre-commencement expenditures (Sec 25)
Add Accounting amortization of pre-commencement expenditures – disallowed Expense XX
Less Tax amortization of Pre-commencement expenditure – allowed expense not charged in P&L
(XX) XXX
(Cost of Expenditure * 20% on straight line basis)

Scientific research expenditure (Sec 26)


Scientific research expenditure incurred and paid to institution outside Pakistan -
Add XX
disallowed expense
Scientific research expenditure paid to Pakistani institution not certified by FBR- disallowed
Add XX
expense·
Add Computer purchased for scientific research for business - disallowed expense XX
Cost of land purchased for construction of scientific research institution in Pakistan-disallowed
XX
Add expense
Cost of scientific research expenditure .not used for development of business disallowed
XX
Add expense
Cost of research conducted to ascertain quality of natural deposits in Baluchistan disallowed
XX XXX
Add expense

Bad Debts (Sec 29)


Add Increase in Provision for doubtful debts charged as expense in P&L XX
Add Actual bad debts written off.in P& L but not allowed by Commissioner XX
Add Actual bad debts written off in P& L but previously debt was not included in taxable income
• Advance given to employee written off
• Loan to employee written off• XX
• Advance given to supplier for purchase of raw material Written off
• Loan to associates written off
Less Decrease in Provision for doubtful debts credited as income in P& L (XX)
Less Accounting recovery of bad debts previously written off (XX)
Tax Recovery of bad debt previously written off
Add/ Amount received against bad debts previously written off
Less Less: Previously inadmissible bad debt expense XX/
Actual Bad debts expenses charged in P&L (XX)
Las: Actual Bad debts allowed as per tax law

Add Accrual basis of Accounting (Sec 34)


Expense charged in P& L but outstanding for more than 3 yea.rs
Note: Any outstanding amount (e.g., Advance from customer) not charged as expense in P&L is
outside the scope of this provision. XX XXX
Note: Where unpaid liability, for more than 3 years, added in income (as above) 'and the
person subsequently pays the liability, a dedication will be allowed in year of payment

Profit on debt financial cost and lease payments (Sec 28)

Add Interest clement paid in lease rental under finance lease - disallowed expense XX
Add Profit on debt (Interest expense) acquired for personal use of owner XX
Less Full payment of lease rental under finance lease - allowed expense but not
(XX) XXX
charged in P&L

INADMISSIBLE INCOMES INCLUDED & CREDITED IN PROFIT AND LOSS


Less ACCOUNT
Income chargeable under any other head of income but credited in P& L
(XXX) (XXX)
• · Rental income of land/building

344 | P a g e
• Rental income of personal property together with plant and machinery
• Gain on sale of shares (securities) of other listed or private Ltd. company
· • Interest income from bank deposits

Total Income from business XXX

FIXED ASSETS SCHEDULE

Dat Particular Building Plant & Furnitur Compute Motor Motor Total Tax
e s s Machiner e& r& Vehicle Vehicl Depreciatio
y Fittings allied (plying e (Not n
Items for hire) plying
for
hire)
Rs. Rs. Rs. Rs. Rs. Rs. Rs.
Opening Tax
WDV X X X X X X
Less: WDV
of disposal (X) (X)
X X

Add:
Additions
Purchase
Cost X X X X X (X)
Less: Initial - (X) - (X) (X) - (X)
allowance
X X X X X X
Total TAX
WDV X X X X X X

Less: Normal (X) (X) (X) (X) (X) (X)


Depreciation

Ending Tax X X X X X X
WDV
(X)

345 | P a g e
Key Concepts Tested in Numerical ICAP CAF

Q. No. Reference Marks Remarks i.e. special features in the question

1 Spring 2010 Q. 1 19 Divisible Income calculation


2 Spring 2012 Q. 1 20 Calculation of Income from PL, Segregation of Income
and Apportionment of expenses
3 Autumn 2012 Q. 1 17 Apportionment of expenses, B.O.D Meeting Fee
4 Autumn 2012 Q. 5(b Unabsorbed Depreciation
5 Spring 2014 Q. 1 (15) Personal Expenses, Advance Taxes, Depreciation
6 Spring 2015 Q. 1 18 Bad Debts, Amortization, Initial Allowance
7 Spring 2016 Q. 4 10 Speculation Business
8 Spring 2016 Q. 8 12 Provisions, bad debts, commissions
9 Spring 2017 Q. 1 16 Salaries, bad debts, assets exported
10 Spring 2018 Q. 1 18 Lease, Donation, Fixed Deposits
11 Autumn 2018 Q. 5 12 Securities, Vehicle Not Plying for Hire
12 Spring 2019 Q. 1 (10) Costing Methods, Amortization, entertainment
expenditures
13 Spring 2020 Q. 1 18 Costing methods, withholding taxes, financial cost
14 Spring 2021 Q. 3 19 Penalty, dividend, pre-commencement expenditures
15 Autumn 2021 Q. 4 18 In-house research, partly used assets, donations
16 Spring 2022 Q. 3 18 Bad debts, lease, commission of third schedule items
17 Autumn 2022 Q. 1 15 Minimum tax, exchange of assets, partly used leased
assets
18 Spring 2023 Q. 3 20 Unabsorbed Losses, foreign exchange, penalties
19 Autumn 2023 Q. 4 19 Women Enterprises, P.O.S, Amortization, Agriculture
Produce

346 | P a g e
Question # 1 Spring 2010 Q. 1

Sohail, Khaled and Qazi are members of an association of persons (AOP) and share profit and loss the ratio of
2:2:1. The principal activity of the AOP is trading of various products.

Following are the details of the annual income / (loss) of the AOP and its members for the tax year 20X8:

(i) The AOP suffered loss before tax amounting to Rs.1,500,000. The loss has been arrived at after adjusting
rental income earned by the AOP, the details of which are as follows:

RS.
Rental income 2,000,000
Related Expenses:
Property tax 40,000
Depreciation 475,000 497,500
Net rental Income 1,502,500

No tax was withheld on the rental income or on trading activity.

(ii) The expenses debited to profit and loss account include the following amounts paid to the members of the
AOP:
Sohail Khaled Qazi
Salary (Rs.) 900,000 600,000 -
Interest on capital (Rs.) 300,000 300,000 500,000

(iii) Sohail earned Rs. 1,800,000 from another business, of which he is the sole proprietor.
(iv) Khaled received an amount of Rs.255,000 as share of income after tax from another AOP. He also earned
income of Rs.1,900,000 from a sole proprietorship concern owned by him.
(v) Qazi runs a part time business. His gross revenue is Rs.1 million whereas total business expenses are
Rs.150,000. He also paid taxes in advance amounting to Rs.100,000.

Required:
Compute the taxable income and tax liability of the AOP and each of its members. (19)

Question # 2 Spring 2012 Q. 1

Dr. Sona is a Leading Eye Specialist and is listed on the panels of two hospitals. He also manages a private
clinic. Summary of his receipts and payments for the latest tax year is as follows:

Note Rs.
Receipts
Consultation fees
- Hospitals (i) 1,800,000
- Clinic 4,400,000
Income from surgery
- Hospitals (i) 1,440,000
- Clinic 4,350,000
Property income (ii) 1,012,000
Other income (iii) 75,000
Payments
Rent of clinic 300,000
Household expenses 1,960,000

347 | P a g e
Purchase of car 640,000
Purchase of surgical equipment 500,000
Salary to assistant 180,000
Clinic running expenses 240,000
Car expenses (iv) 200,000
Donation (v) 300,000

Notes to the receipts and payments are presented below:


(i) The amount received from hospitals is net of withholding tax @ 10% being independent service provider.
(ii) Dr. Sona owns a commercial building which he has has rented out. Detail of net receipts is followings:

Rs.
Rent for the year 870,000
Non-adjustable security deposit:
- received from.a new tenant 700,000
- paid to old tenant (received three years ago) (500,000)
Income Tax withheld (50,000)
Property tax on building (8,000)
Net receipts 1,012,000

(iii) The amount was received for writing an article in a magazine on World Health Day.
(iv) 60% of the motor car expenses were incurred in connection with his personal use.
(v) Donation was given to a Government medical college for upgrading its library.
(vi) Tax Depreciation on motor car and surgical equipment is Rs 96,000 and Rs 75,000 respectively

Required:
Compute the taxable income, tax liability and tax payable by Dr. Sona for the latest tax year. Provide
appropriate comments on the items which are not relevant for your computations. (Marks 20)

Question # 3 Autumn 2012 Q. 1

Beena is a lawyer and owns a law firm under the name Beena & Co. She is also Director Legal Affairs at AY
Foods Ltd. Details of her income for the tax year 20X8 are as follows:

(A) INCOME FROM BEENA & CO.


Income Statement
Note Rs.
Revenue (i) 8,500,000
Less: Expenses
Salaries (ii) 2,000,000
Gifts and donations (iii) 400,000
Lease charges (iv) 900,000
Professional fee (v) 400,000
Property expenses (vi) 350,000
Travel expenses 150,000
Other expenses (vii) 800,000 5,000,000
Net profit 3,500,000

Notes to the Income Statement


(i) Revenue includes Rs.750,000 recovered from Rafia in respect of bad debts that had been written off
while calculating the taxable income for the tax year 20X6. The amount was receivable against
professional services rendered to Rafia

348 | P a g e
(ii) Salary expenses include amounts of Rs.50,000 and Rs.75,000 per month paid to Beena and her
brother respectively. Her brother looks after administration and financial matters of the firm.
(iii) Gift and donations include gift to client, gift to her son and donation to Edhi Foundation amounting
to Rs.100,000, Rs.50,000 and Rs.250,000 respectively.
(iv) A vehicle was obtained solely for official purpose on operating lease, from a bank. The lease
commenced on 1.3.20X8. Lease charges include Rs.500,000 paid as security deposit to the bank.
(v) The professional fee includes an amount of Rs.150,000 paid to a legal firm for defending a law suit
filed against Beena, in a family court.
(vi) Beena lives in an apartment situated above her office, and 2/5" of the total property expenses relates
to this apartment.
(vii) Other expenses include an amount of Rs.150,000 paid for Beena’s Golf Club membership which she
exclusively used to promote her business interests. The payment to the club was made in cash. It also
includes Rs 200,000 tax deducted by the clients

(B) DIRECTOR'S REMUNERATION FROM AY FOODS LTD (AFL)


(i) Beena received monthly remuneration of Rs.100,000 from AFL.
(ii) During the year, she also received two bonus payments of Rs.100,000 each. One of the bonus
payments pertains to tax year 20X7. It was announced last year but disbursed to her in current year.
(iii) Beena has also been provided a vehicle, by AFL, for her personal as well as business use.The car
was acquired by AFL in May 20X5 at a cost of Rs.2,000,000. The fair market value of the car as at
30.6.20X8 was RS.1,500,000.
(iv) She received a fee of Rs.150,000 from AFL for attending the meetings of the Board of Directors
(BOD).
(v) Tax deducted by AFL from salary is Rs.390,000 and BOD meeting fee 9,000

Required:
Compute the taxable income, tax liability and tax payable by Beena for the tax year 20X8. Provide appropriate
comments on the items appearing in the notes which are not considered by you in your computations. (17)

Question # 4 Autumn 2012 Q. 5(b)

On 1 July 2021, Ms. Kashmala and Ms.Shumaila formed an Association of Persons (AOP) with the objective
of providing information technology support services to corporate clients. They contributed Rs. 1.2 million and
Rs. 0.8 million respectively in their capital accounts and agreed to share profits and losses in the ratio of their
capitals.

For the year ended 30 June 2022, business loss and unabsorbed depreciation of Rs. 0.4 million and Rs. 0.3
million respectively were assessed and carried forward. The total turnover of the AOP in 2022 was Rs. 40
million.

During the year ended 30 June 2023, the AOP incurred a net loss of Rs. 0.8 million on a turnover of Rs. 50
million. The loss for the year was arrived after adjustment of the following:
▪ Salaries amounting to Rs. 0.5 million and Rs. 0.3 million were paid to Ms.Kashmala and Ms.Shumaila
respectively.
▪ Accounting depreciation on office assets amounted to Rs. 0.3 million
▪ The taxes withheld by the clients, for the year ended 30 June 2023 amounted to Rs. 0.55 million. AOP is
entitled to claim tax depreciation of Rs. 0.25 million in respect of the office assets.

Required:
Calculate the taxable income, net tax payable and unabsorbed losses (including unabsorbed depreciation), if
any, to be carried forward by the AOP for the year ended 30 June 2023. Ignore any working of minimum tax.
(08)

349 | P a g e
Question # 5 Spring 2014 Q. 1

Qamar is engaged in the business of manufacturing and repair of electric motors. His accountant has prepared
the following tax computation for the tax year 20X4:

Rs.000
Sale of manufactured motors 45,000
Less: Cost of sales and administrative expenses
(excluding depreciation for the year) 33,000
Income before depreciation 12,000
Less: Tax depreciation for the year 9,000
3,000
Less: Brought forward business loss from tax year 20X3
(Total business loss was Rs.4 million) 3,000
Business income after adjusting business loss -
Add: Interest income received from a commercial bank 500
* Income from other sources 850 1350
Taxable income 1,350

Computation of tax
Tax liability 125
Less: Tax deducted by bank on interest income (500,000 x 10%) 50
Tax payable 75

Following expenses are included in the cost of sales and administrative expenses:

Description Rs.000
Travelling expenses include travel and hotel expenses of Qamar’s visit to Malaysia for 100
attending a trade fair
Electricity c charges paid for Qamar’s residence 150
Damages paid to a distributor for delayed supplies 200
Donations to a non-profit organization 300
Salary paid to Bari who is Qamar’s brother. Advance tax has been deducted from the 720
salary
Fine paid to the Ministry of Environment for infringement of environmental safety laws 200
Unabsorbed depreciation brought forward from previous tax year 500

Qamar is not satisfied with the tax return prepared by his accountant and has requested you to review the return.

Required:
Compute the revised taxable income of Qamar, and tax payable by or refundable to him for the tax year 20X4.
Briefly comment on treatment of the above items of expenses in your tax computation. (15)

Question # 6 Spring 2015 Q. 1

On 1 July 20X4, Tahir commenced business of manufacturing garments. Income statement of the business for
the year ended 30 June 20X5 is as follows:

Notes Rs. in 000


Sales 49,330
Less: Cost of sales (i) (39,150)
Gross profit 10,180
Less: Administrative and selling expenses (ii) (9,140)

350 | P a g e
Financial charges (iii) (2,500)
Other charges (iv) (1,358)
(2,818)
Add: Other income 3,875
Profit before taxation 1,057

Notes to the income statement:


(i) On 15 July 20X4, used machinery was imported from China valuing Rs. 1,500,000. Depreciation @ 15%
was charged on machinery for the whole year and is included in cost of sales.
(ii) Administrative and selling expenses include:
▪ Rs. 975,000 paid for the purchase of computer software. The software is likely to be used for twelve
years.
▪ Cost of preparation of a feasibility study amounting to Rs. 250,000 which was issued prior to the
commencement of business.
▪ Salary of Rs. 50,000 per month was paid to Tahir’s brother who handles the financial matters of the
business.
(iii) Financial charges include Rs. 80,000 pertaining to a vehicle obtained on lease from a leasing company.
The cost of vehicle was Rs. 1,300,000. Depreciation of Rs. 260,000 has been included in administrative
and selling expenses. Lease rentals paid during the year amounted to Rs. 300,000.
(iv) Other charges include:
▪ running and maintenance expenses of vehicle amounting to Rs. 295,450. Use of vehicle for personal
purposes was approximately 20%.
▪ provision for bad debts amounting to Rs. 25,000.

Other information:
(i) Tahir was working in UAE for the past five years and had come back to Pakistan in April 20X4. He received
an amount equivalent to Rs. 150,000 from his ex-employer as differential amount on his final settlement
in August 20X4.
(ii) He sold a plot for Rs. 3,500,000 which was inherited from his father in 20W9. Fair market value of the plot
at the time of inheritance was Rs. 1,500,000.
(iii) 5,000 shares were purchased for Rs. 600,000 from initial public offering of a new listed company.
(iv) Premium of Rs. 300,000 was paid on Tahir’s life insurance policy.

Required:
Under the provisions of the Income Tax Ordinance, 2001 compute the taxable income and tax liability of Tahir
for the tax year 20X5. Provide comments in respect of items which do not appear in your computation. (18)

Question # 7 Spring 2016 Q. 4

Lone Traders (LT), a sole proprietorship, is engaged in the business of buying and selling of Maize and Wheat
in bulk quantities. Following information has been extracted from LT’s records for the year ended 31 December
2015:
(i) Wheat sold to food companies in Punjab amounted to Rs. 13,000,000. The sale was made after allowing
discount of Rs. 680,000 to some of the new customers. The gross profit margin was 25% on gross sales.
(ii) LT paid Rs. 600,000 to a research institute for the development of a formula which is likely to improve
the quality of wheat it purchases from the growers.
(iii) In August 2015, LT signed a future contract with Mubarak Enterprises (ME) for the purchase of 500 metric
tons of maize at Rs. 15,800 per metric ton. The delivery was expected to be made in October 2015. ME
also agreed to repurchase the entire lot at the price prevailing on the date of sale.
(iv) In October 2015 price of maize increased to Rs. 18,240 per metric ton and LT sold the entire lot to ME
without taking delivery.
(v) LT incurred expenditure of Rs. 25,000 in respect of above future contract.
(vi) Administrative, selling and distribution expenses amounted to Rs. 2,500,000. These included a penalty of
Rs. 45,000 which was imposed due to late payment of sales tax on wheat.

351 | P a g e
(vii) Assessed losses brought forward from previous year were as follows:

Rupees
Trading business loss 550,000
Speculation business loss 300,000
Capital loss 250,000

Required:
Under the provisions of the Income Tax Ordinance, 2001 and Rules made thereunder, compute LT’s taxable
income/(loss) and the amount of loss to be carried forward, if any, for tax year 2016. (10)

Question # 8 Spring 2016 Q. 8

Baqir, Asad and Rahi are members of an association of persons (BAR) and share profits and losses in the ratio
of 5:3:2 respectively. BAR is engaged in the business of trading consumer electronics and has two independent
branches one each in Tehran and Dubai. Following information has been extracted from BAR’s profit and loss
account for the year ended 31 December 2015:

Rupees
Sales 30,000,000
Cost of sales (20,500,000)
Gross profit 9,500,000
Administrative and selling expenses (4,732,000)
Financial charges (980,000)
Other income 1,700,000
Profit before taxation 5,488,000

Additional information:
Cost of sales includes:
(i) Closing stock which has been valued at net realizable value of Rs. 1,820,000. The cost of closing stock
under absorption costing was Rs. 1,950,000.
(ii) Provision of Rs. 75,000 against slow moving stores and spares.
(iii) Freight charges of Rs. 260,000. These were paid in cash to Momin Goods Transport for transporting goods
to customers in Multan.

Administrative and selling expenses include:


(i) Commission of Rs. 290,000 paid to Baqir, annual performance award of Rs. 310,000 paid to Rahi and Rs.
455,000 paid to AB Bank Limited in final settlement of a loan obtained by Asad for the construction of
his house in Muree.
(ii) Provision for bad debts of Rs. 735,000. The opening and closing balances of provision for bad debts
amounted to Rs. 1,100,000 and Rs. 1,435,000 respectively. Bad debts written off include a loan of Rs.
285,000 provided to a supplier.
(iii) Sales promotion expenses of Rs. 275,000. These expenses were paid by Rahi through his personal credit
card.
(iv) Rs. 86,000 paid to an institution operated by Federal Government for the training of industrial workers in
Punjab.

Further information:
For the year ended 31 December 2015 Dubai branch made a profit of Rs. 1,500,000 and Tehran branch made
a loss of Rs. 1,800,000. These figures are not included in the above profit and loss account.

Required:
Under the provisions of the Income Tax Ordinance, 2001 and Rules made thereunder, compute the taxable
income, net tax payable by BAR and the amount to be carried forward, if any, for tax year 2016. Assume tax

352 | P a g e
and accounting depreciation is same. (12)
Note: Your computation should commence with the profit before tax figure of Rs. 5,488,000.
Show all relevant exemptions, exclusions and disallowances.

Question # 9 Spring 2017 Q. 1

Mushtaq is a sole proprietor of Mushtaq Enterprises (ME) engaged in the business of manufacturing of different
products. ME’s profit and loss account shows profit before taxation of Rs. 1.8 million for the year ended 30
June 20X7. A review of ME’s records has revealed the following information.
(i) ME employs five salesmen. Rs. 22,000 per month were paid to each salesman in cash which includes
reimbursement of Rs. 6,000 per month incurred on entertainment of customers at the business premises.
(ii) Administrative expenses include Rs. 150,000 which were paid to a research institute in China for the
purpose of developing a new product.
(iii) Accounting loss on the sale of patents was Rs. 65,000. The tax written down value of these patents at the
beginning of the year was Rs. 430,000 and these were sold for Rs. 524,000. Amortization charged to the
profit and loss account on these patents for the current year was Rs. 25,000.
(iv) Receivables from Atif and Aslam which had been written off in the previous year were recovered. Details
are as follows:
Atif Aslam
----------- Rupees ------------
Claimed bad debts in last tax return 800,000 1,200,000
Allowed by tax authorities last year 550,000 600,000
Amount recovered during the year 700,000 400,000

(v) ME has opened a sales office in Dubai. In this respect, furniture costing Rs. 850,000 with written down
value (WDV) of Rs. 650,000 was shifted to Dubai office. The tax WDV of the furniture at the beginning
of the year was Rs. 610,000.
(vi) Accounting depreciation for the year is Rs. 580,450. However, no depreciation has been provided on the
following fixed assets purchased on 1 March 20X7:

Rupees
Furniture 200,000
Used machinery imported from Germany 500,000

(vii) Tax depreciation for the year, prior to the adjustments mentioned in (vi) above, amounted to Rs. 456,400.
(viii) Advance tax paid u/s 147 was Rs. 200,000.
(ix) The assessed business losses of tax year 20X1 brought forward in year 20X7 are Rs. 830,000. These
include unabsorbed tax depreciation amounting to Rs. 705,000.

Other transaction of Mushtaq


On 1 June 20X7, he sold 6,000 shares for Rs. 432,000 out of 15,000 shares which he received on 1 May 20X4,
on the death of his father. The fair market value of shares on the date of transfer to Mushtaq was Rs. 25 per
share.

Required:
Under the provisions of Income Tax Ordinance, 2001 and rules made thereunder, compute taxable income and
net tax payable by or refundable to Mushtaq for the year ended 30 June 20X7. (16)

Question # 10 Spring 2018 Q. 1

Mr. Qateel, a resident individual, is engaged in the manufacture of various consumer goods under the name and
style ‘Qateel Enterprises (QE)’. The following information has been extracted from the records of QE for the
financial year ended 30 June 20X8.
Rupees

353 | P a g e
Total turnover 28,500,000
Cost of sales (26,155,000)
Gross profit 2,345,000
Operating expenses (4,500,000)
Operating loss (2,155,000)
Finance charges on lease of machinery (35,703)
Other income 5,000,000
Profit before tax 2,809,297

Additional information:
(i) Cost of sales includes:
▪ Rs. 45,000 paid as fine for violation of contract with a customer for delay in supply of goods.
▪ accounting depreciation of Rs. 1,900,000 (including depreciation on leased assets).
(ii) Operating expenses include:
▪ Rs. 450,000 paid for renewal of a manufacturing licence for fifteen years.
▪ vehicle tax paid in cash amounting to Rs. 55,000 for eight office cars.
▪ Rs. 200,000 paid as security deposit to K-Electric (KE) for replacement of transformer at the factory.
▪ Rs. 300,000 collected by KE as advance tax through monthly electricity bills.
▪ cash donation to poor families amounting to Rs. 64,600 and donation of Rs. 2,000,000 paid through
cheque to Edhi Foundation, which is listed in Part 1 of the Second Schedule of the Income Tax
Ordinance, 2001.
▪ penalty of Rs. 25,000 imposed by the Commissioner Inland Revenue for late filing of
▪ annual return of income for the tax year 20X7.
▪ entertainment expenditure of Rs. 128,000 incurred on arrival of foreign customers for business
purposes.
(iii) Other income includes:
▪ dividend of Rs. 580,000 received from listed companies. The amount is net of income tax at the rate
of 15% and Zakat of Rs. 100,000 deducted under the Zakat and Usher Ordinance, 1980.
▪ Capital gain of Rs. 1,200,000 from sale of shares of a private limited company.
▪ Shares were acquired on 1 August 20X3.
(iv) On 30 June 20X8, leased machinery was transferred to Qateel on maturity of lease. The leasing company
was asked to adjust the amount of security deposit against the residual value of Rs. 100,000. The date of
commencement of lease was 1 July 20X3.
Lease rentals paid during the year amounted to Rs. 270,000.
On the date of maturity, the accounting written down value and market value of the machinery was Rs.
590,490 and Rs. 800,000 respectively.
(v) During the year, a warehouse was constructed for storage of goods at a cost of Rs. 1,040,000. No
accounting depreciation has been recorded on it.
(vi) Tax depreciation for the tax year 20X8 without considering the effect of para (iv) and (v) above, amounted
to Rs. 1,560,000.
(vii) Advance income tax paid during the year amounted to Rs. 480,000.

Required:
Under the provisions of the Income Tax Ordinance, 2001 and Rules made thereunder, compute the total income,
taxable income and net tax payable by or refundable to QE for the year ended 30 June 20X8. (18)
Note: Ignore minimum tax under section 113.
Show all the relevant exemptions, exclusions and disallowances.

Question # 11 Autumn 2018 Q. 5

Saleem is a resident taxpayer and runs a fitness centre in DHA Karachi. He files his return of income regularly.
Following information pertains to his business for the tax year 2026:
i. Accounting profit before tax amounted to Rs. 2162500
ii. Administrative expenses include annual rent of the premises used for fitness centre amounting to Rs.

354 | P a g e
1,560,000. Withholding tax of Rs. 144,000 was deducted from the rent payment but was not deposited in
the government treasury.
iii. A passenger transport vehicle used for pick and drop of employees of fitness centre was disposed of for
Rs. 8,000,000. The vehicle was purchased for Rs. 8,500,000 in tax year 2025. No accounting depreciation
was provided during the year 2026. Accounting gain of Rs. 400,000 has been recorded in the profit or loss
account
iv. On 1 July 2025, a car was acquired on finance lease for Rs. 3,000,000. Advance tax paid at the time of
acquisition and registration of vehicle aggregated Rs. 85,000. The vehicle has been used 70% for business
purposes and 30% for Saleem’s personal use.
Accounting depreciation of Rs. 600,000 and financial charges of Rs. 462,000 were recorded in the profit
or loss account. Lease rentals paid during the year amounted to Rs. 857,000.
v. During the year, Saleem recorded gain of Rs. 50,000 on disposal of shares. Details are as under:

Name of investee company Sold on Purchased on Gain/(loss) on disposal (Rs.)


Sun (Private) Limited 1 Aug 2025 1 Sep 2020 500,000
Moon Limited - a listed company 15 Sep 2025 1 Jan 2023 (700,000)
Planet Limited - a listed company 1 Feb 2026 1 Jan 2023 250,000
50,000

Required:
Compute Saleem’s taxable income under appropriate head of income and tax liability for the tax year 2026.
(12)

Question # 12 Spring 2019 Q. 1

Mustafa, Ali and Zain are partners of a resident firm in Pakistan, under the name and style MAZ Enterprises
(MAZE) which is engaged in manufacturing and local supply of auto spare parts. All partners have equal share
of profits and losses in the firm.

Following information has been extracted from accounting records of MAZE for the tax year 20X9:

Rs. in '000
Sales 140,400
Cost of goods sold (91,260)
Gross profit 49,140
Administrative and selling expenses (21,430)
Financial charges (15,740)
(37,170)
Other income 1,900
Profit before tax 13,870

Additional information:
(i) The above accounts have been prepared on cash basis and stock-in-trade has been valued on the prime-
cost method. However, the partners want to change the method of accounting from cash basis to accrual
basis. In this respect, following information has been gathered:

Opening balances Closing balances


-------------- Rs. in ‘000 -------------
Stock-iii-trade using prime-cost method 5,200 7,500
Stock-in-trade using absorption-cost method 5,900 8,800

(ii) Cost of goods sold includes cost of used machinery imported from China on 31 July 20X8 amounting to
Rs. 2,110,000. The cost includes payment of custom duty of Rs. 90,000 and income tax of Rs. 110,000 to
the Collector of Customs.

355 | P a g e
(iii) Administrative and selling expenses include:
▪ payment of Rs. 380,000 to a local hotel for holding annual eid-milan party for the employees, key
customers and their families.
▪ payment of a fixed monthly remuneration of Rs. 150,000 to each partner.
▪ payment of Rs. 180,000 for purchase of accounting software on 1 January 20X9. The software is
expected to be used for fifteen years.
(iv) Financial charges are net of interest income of Rs. 360,000 (net of tax @ 10% deducted by the bank),
earned by the firm on its savings accounts.

Required:
Under the provisions of Income Tax Ordinance, 2001 and Rules made thereunder, compute the total income,
taxable income and tax payable by MAZE using accrual basis of accounting. (10)
Note: Show all the relevant exemptions, exclusions and disallowances.

Question # 13 Spring 2020 Q. 1

For the purpose of this question, assume that the date today is 31 August 2020.
Shahid is engaged in the business of manufacturing and supplying of auto parts. Following is the extract of his
profit or loss statement for the tax year 2020:
Rs. in ’000
Sales 29,058
Cost of goods sold (18,724)
Gross profit 10,334
Operating expenses (3,137)
Financial charges (2,030)
Other income 1,260
Profit before tax 6,427

Additional information:
(i) The above accounts have been prepared on cash basis and stock-in-trade has been valued on prime cost
method. However, Shahid wants to change the method of accounting from cash basis to accrual basis. In
this respect, following information has been gathered:
Opening balances Closing balances
---------Rs. in ’000---------
Stock-in-trade using prime cost method 1,800 2,800
Stock-in-trade using absorption cost method 2,300 3,200

(ii) Cost of goods sold includes:


▪ purchase of packing material of Rs. 440,000 from Nasir Traders. No withholding tax was deducted at
the time of payment.
▪ freight charges of Rs. 85,000. These were paid in cash for transporting goods from suppliers.
(iii) Operating expenses include:
▪ salary of Rs. 80,000 per month paid to Shahid’s brother who handles administrative matters of the
business.
▪ expenditure of Rs. 950,000 incurred on the development of a product which is expected to generate
revenue for five years.
▪ penalty of Rs. 15,000 for late filing of income tax return.

(iv) Financial charges include profit on debt of Rs. 450,000 earned on fixed deposit account maintained with
a bank. The bank withheld income tax and Zakat amounting to Rs. 45,000 and Rs. 93,750 respectively.

(v) Other income includes:


▪ capital gain of Rs. 45,000 received, net of withholding tax of Rs. 6,750, on sale of 20,000 shares in
Metal Limited (ML) in November 2019. ML is listed on PSX. On 1 January 2018, Shahid purchased

356 | P a g e
these shares for Rs. 200,000 at initial public offering. He had claimed a tax credit of Rs. 15,000 on
such investment in tax year 2018.
▪ rent of Rs. 980,000 received from an agriculture land in Badin. No withholding tax was deducted at
the time of receipt.
(vi) Tax depreciation for the year amounts to Rs. 680,000.
(vii) Tax deducted at source by customers amounts to Rs. 875,000.
(viii) The unabsorbed tax depreciation brought forward from tax year 2019 amounts to Rs. 568,000.

Required:
Under the provisions of the Income Tax Ordinance, 2001 and Rules made thereunder, compute total income,
taxable income and net tax payable by or refundable to Shahid for the tax year 2020. (Use accrual basis of
accounting) (18)
Note: Your computation should commence with profit before tax figure.
Ignore minimum tax under section 113.
Show all relevant exemptions, exclusions and disallowances.

Question # 14 Spring 2021 Q. 3

Muhammad Asghar owns an industrial undertaking under the name and style of Asghar & Company (AC)
which is engaged in the business of manufacturing pharmaceutical products. Following information is available
for the year ended 31 December 20X1:
Rs. in '000
Turnover 324,850
Cost of goods sold (217,197)
Gross profit 107,653
Administrative and distribution expenses (88,980)
Marketing expenses (19,765)
Other income 3,560
Profit before tax 2,468

Additional information:
(i) Cost of goods sold includes:
▪ Raw materials of Rs. 7,800,000. No withholding tax was deducted at the time of payment.
▪ accounting depreciation of Rs. 2,100,000 on plant and machinery.
▪ provision for slow moving inventory of Rs. 1,800,000.
(ii) Administrative and distribution expenses include:
▪ Rs. 676,500 paid to a local hotel for holding annual Eid-Milan party for the employees and their
families.
▪ Rs. 1,235,000 paid as penalty to a customer in settlement of his claim for damages under a contract
for the supply of a batch of vaccines. Laboratory tests and in-house investigations revealed that the
level of impurities in the vaccines exceeded the acceptable level as agreed in the contract.
▪ Rs. 2,300,000 paid as donation to a hospital established by the local government.
(iii) Marketing expenses include a reward of Rs 500,000. The reward was paid in cash to one of the salesmen
for exceeding his sales target.
(iv) Other income includes:
▪ dividend of Rs. 174,000. This amount was received from a listed company after deduction of income
tax at the rate of 15% and Zakat of Rs. 30,000 deducted under the Zakat and Usher Ordinance, 1980.
▪ gain of Rs. 660,000 on sale of shares in Akash (Pvt) Limited (APL) in November 20X1. 60% of the
shares in APL are owned by the Federal Government. AC purchased these shares in June 20X0.
Other information:
(i) A second hand plant was imported from France at a cost of Rs. 2,500,000. Withholding tax of Rs. 150,000
was deducted at import stage. The plant was installed in the month of September 20X1. AC incurred Rs.
375,000 on the installation of plant which is included in administrative and distribution expenses.
(ii) Pre-commencement expenditures of Rs. 3,400,000 were charged to accounting profit and loss for the year

357 | P a g e
ended 31 December 20X0. However, for tax purposes, it has to be amortized over the period of five years.
(iii) Tax depreciation other than imported plant amounted to Rs. 1,900,000.
(iv) Income tax deducted by the customers u/s 153 and advance income tax paid u/s 147 during the year
amounted to Rs. 1,400,000 and Rs. 200,000 respectively.

Required:
Under the provisions of the Income Tax Ordinance, 2001 and Rules made thereunder, compute total income,
taxable income and net income tax payable by or refundable to AC for the tax year 20X2. (19)

Note: Your computation should commence with profit before tax figure of Rs. 2,468K.
Ignore minimum tax under section 113.
Show all relevant exemptions, exclusions and disallowances.

Question # 15 Autumn 2021 Q. 4

Abbas, a resident individual, is engaged in the business of manufacturing various consumer goods under the
name and style of ‘Kamyab Enterprises (KE)’. Following information has been extracted from KE’s records
for the year ended 30 June 2021:

Rupees
Sales 43,089,000
Cost of Sales (26,042,000)
Gross Profit 17,047,000
Administrative & Selling Expense (7,800,000)
Financial charges (2,100,000)
Other income 5,560,000
Profit before tax 12,707,000

Additional information:
Cost of sales includes:
(i) accounting depreciation of Rs. 1,200,000. The tax written down values of KE’s fixed assets on 1 July 2022
were:
Rupees
Plant & Machinery 6,860,000
Computer & related products 800,000
Motor Vehicles (80% for the business purpose) 3,000,000

A new computer was purchased on 1 April 2023 for Rs. 150,000.


Motor vehicle which was purchased on 15 June 2021 at the cost of Rs. 1,000,000 was sold for Rs. 750,000
on 31 May 2023. Carrying value of this motor vehicle was equal to sale proceeds

(ii) an amount of Rs. 40,000 paid to factory supervisor on 23 March 2023 as advance salary for the month of
April. Since he was in urgent need of the amount and the banks were closed on 23 March 2023 due to the
Pakistan Day, he was paid in cash.

Administrative and selling expenses include:


(i) expenditure on ‘In-house scientific research’ related to KE’s business. It includes salaries of Rs. 880,000
paid to scientists, material of Rs. 230,000 used in the research and Rs. 700,000 paid to a company in China
for supporting KE’s scientists in the research work. This expenditure was not recorded as intangible asset
as it could not provide an advantage for a period of more than one year.
(ii) An expense of Rs. 650,000 paid as an instalment towards the purchase price of an industrial plot.

(iii) purchase of goats worth Rs. 225,000 for sacrifice on Eid-ul-Azha. The payment was made through cross

358 | P a g e
cheque.
(iv) Donations of Rs. 1,000,000 to approved non-profit organisations. 40% of this amount was donated to
organisations listed on the Thirteenth Schedule of the Income TaxOrdinance, 2001. All donations were
made through crossed cheques.
(v) an insurance premium of Rs. 200,000 paid to a registered insurance company for health insurance of Abbas
and his dependents.

Other income includes:


(i) An amount of Rs. 720,000 received from income tax department on account of tax refund related to tax
year 2020. This amount includes an additional payment of Rs. 80,000 due to delay in tax refund.
(ii) capital gains of Rs. 430,000 and Rs. 250,000 on sale of investments in shares of Manzil Limited, a public
unlisted company and Himmat Limited, a public listed company respectively on 20 June 2023. Both
investments were made on 1 January 2021.

Required:
Under the provisions of the Income Tax Ordinance, 2001 and Rules made thereunder, compute total income,
taxable income and net income tax payable by or refundable to Abbas for the tax year 2023. (18)

Note: Your computation should commence with profit before tax figure of Rs. 12.707 million.
Ignore minimum tax under section 113.
Show all relevant exemptions, exclusions and disallowances.

Question # 16 Spring 2022 Q. 3

For the purpose of this question, assume that the date today is 31 August 2023.
Aakash Kumar owns an industrial undertaking under the name and style of Premjee & Co. (PJC) which is
engaged in the business of manufacturing fast moving consumer goods. Following information is available
from PJC’s records for the year ended 30 June 2023:

Loss before tax for the year was Rs. 87 million.

i. Operating expenses include:


▪ a penalty of Rs. 2 million for late delivery of goods to a customer.
▪ commission of Rs. 2.5 million which was paid to a distributor, Liaquat Bashir, on sale of PJC’s
products of Rs. 50 million. These products are covered in the Third Schedule of the Sales Tax Act,
1990. Name of Liaquat Bashir is not appearing in the active taxpayers’ list under the Income Tax
Ordinance, 2001
▪ freight charges of Rs. 1.2 million which were paid in cash to a freight forwarding company in Karachi
▪ accounting depreciation of Rs. 40 million

ii. Other income includes:


▪ an insurance claim of Rs. 6 million, equivalent to accounting book value, received on 8 November
2022 in respect of a vehicle which was completely destroyed by fire. The cost and fair market value
of the vehicle before fire incident were Rs. 10 million and Rs. 8 million respectively. This vehicle was
purchased on 1 October 2020
▪ amounts recovered during the year from two debtors i.e. Shameem and Faheem. These amounts had
been written off in the last year. Details are as follows:
Shameem Faheem
---- Rs. in million ----
Bad debts claimed in the last tax return 19.2 28.8
Bad debts allowed by tax authorities last year 13.2 14.8
Amounts recovered during the year 16.8 10.6

▪ Rent of Rs. 21.6 million. On 1 July 2022, Aakash leased one of its factory buildings along with the
plant to Kamran at a monthly rent of Rs. 1.8 million, payable in advance. The building was purchased

359 | P a g e
for Rs. 85 million on 16 August 2020 whereas a second hand locally purchased plant was installed at
a cost of Rs. 34 million on 1 July 2022. During the year, Aakash incurred Rs. 3.2 million on repair
and maintenance of the factory building.

iii. PJC’s liabilities include amounts of Rs. 14 million and Rs. 17 million in respect of purchases made on 18
March 2020 and 1 August 2020 respectively. These purchases were allowed as admissible deductions
while computing income from business in their relevant tax years.
iv. During the year, outstanding financial charges of Rs. 2.8 million were waived by the bank on rescheduling
the loan. These charges were claimed as admissible deduction in the tax year 2021.

Tax depreciation for the year on all fixed assets, other than factory building and plant which were leased
out to Kamran, amounted to Rs. 48 million.

Other information:
1) On 15 August 2021, Aakash entered into a derivative contract for the purchase of gold. The contract was
to be expired on 15 November 2022. Aakash sold the contract before the settlement date and earned a net
gain of Rs. 23 million on the contract.
2) On 30 June 2023, Aakash earned capital gains of:
▪ Rs. 20 million on sale of his immovable property which was purchased on 1 June 2020.
▪ Rs. 3.6 million on sale of shares in a private company. These were acquired on 1 June 2022
3) During the year, Aakash received his share of profit from an AOP of Rs. 70 million.

Required:
Under the provisions of the Income Tax Ordinance, 2001 and Rules made thereunder, compute total income,
taxable income and net income tax payable by or refundable to Aakash for the tax year 2023. (18)

Note: Ignore minimum tax under section 113.


Show all relevant exemptions, exclusions and disallowances.

Question # 17 Autumn 2022 Q. 1

Azaadi & Co. (AC) is an association of persons engaged in the business of manufacturing disposable products.
Following information has been extracted from AC’s records for the year ended 30 June 2022:

Rs. in million
Sales 380
Less: Sales Tax (45)
Less: Trade Discount (15)
Net Sales 320
Less Cost of Sales (240)
Gross Profit 80
Less: Operating Expenses (146)
Loss before tax (66)

Additional information:
Cost of sales includes:
▪ payment of Rs. 12 million (including sales tax of Rs. 2 million) to Hashim Limited (HL) for the purchase
of a new machine for making plastic container in exchange of an old machine having a book value and a
fair market value of Rs. 5 million and Rs. 4 million respectively on exchange date. The transaction was
carried out on 1 July 2021.
▪ Old machine was purchased on 1 January 2020. Disposal of old machine and depreciation of new machine
were not recorded in AC’s books of accounts. Tax WDV of old machine was the same as accounting WDV
on the disposal date.
▪ purchase of raw materials of Rs. 40 million against which no withholding tax was deducted at the time of

360 | P a g e
payment. During the year, AC purchased total raw material of Rs. 120 million.
▪ Closing inventory of damaged finished goods of Rs. 18 million. Due to heavy rain, these goods were
damaged and it is expected to fetch Rs. 12 million only.

Operating expenses include:


▪ salary of Rs. 0.275 million paid to office boy in cash. He was paid a monthly salary of Rs. 25,000 for eleven
months.
▪ warehouse rent of Rs. 1.6 million paid through credit card linked to notified business bank account.
▪ bad debt of Rs. 2.8 million written off against receivable from a customer on his insolvency.
▪ commission of Rs. 3.2 million paid to one of the members of AC, by crediting the amount to his bank
account.
▪ scholarship of Rs. 4.5 million paid to Sara, one of AC’s staff members, for higher studies in accordance
with the terms of the employment
▪ payment of Rs. 6.4 million for the purchase of a specialized software for AC’s manufacturing department
on 1 April 2022. The software is expected to be used for five years. Although the software is available for
use from the date of purchase, AC’s members have decided to implement this software from 1 July 2022.
▪ depreciation and financial charges of Rs. 2.1 million and Rs. 1.5 million respectively in respect of a car
which was acquired on financial lease
▪ On 1 July 2021, AC entered into a lease agreement with a bank for a car of Rs. 10.5 million against the
annual lease rentals of Rs. 3.0 million, payable in arrears. The car has been used 80% for business purposes
and 20% for personal use of members.

Required:
Under the provisions of the Income Tax Ordinance, 2001 and Rules made thereunder:
(a) compute the total income, taxable income and tax liability of Azaadi & Co. for the tax year 2022 (15)
(b) discuss the tax implication in respect of scholarship received by Sara in her return of income for tax year
2022. (02)
Note: Show all relevant exemptions, exclusions and disallowances

Question # 18 Spring 2023 Q. 3

For the purpose of this question, assume that the date today is 31 August 2023.
Faith Brothers (FB) is engaged in the business of manufacturing tools and equipment. Following
information has been extracted from FB’s records for the year ended 30 June 2023:

Rs. in million
Revenue 1,400
Expenses (1,270)
Other income 47
Net profit 177

Additional information:
(i) Expenses include:
accounting depreciation of Rs. 188 million.
cash payment of Rs. 1 million for purchasing the ten air tickets.
payment of Rs. 4 million to shipping line on account of demurrages for containers
blocked at port. Moreover, penalties of Rs. 9 million were also paid to various clients
on delaying their orders because of closure of production plant for two weeks during
the year.
payment of Rs. 50 million to a builder for the construction of a building. The work
is still in progress at the year end.

361 | P a g e
payment of AED 100,000 equivalent to Rs. 5 million to a research institute in
UAE for the purpose of developing a new product.
(xxii) a foreign exchange loss of Rs. 15.8 million in respect of amount payable to FB’s
associated company in Qatar. This liability arose on 1 January 2022 from the import of
a second-hand plant at a cost of QAR 400,000. As per the agreement, the payment would
be made on 1 January 2024. Relevant exchange rates are as follows:
1 January 2022 30 June 2022 30 June 2023
Rs. per QAR 45 40 79.5
(ii) Other income comprises of:
▪ recovery of bad debts of Rs. 16 million. The amount of Rs. 30 million was written off
three years ago, out of which only Rs. 10 million was allowed by the tax authorities.
▪ capital gain of Rs. 40 million on sale of following securities on 1 March 2023:
Capital gain (Rs.
Date of purchase
in million)
Modaraba certificates 10 July 2022 8
Shares of an unlisted company 20 February 2022 12
Shares of a listed company 15 January 2019 20
▪ a loss of Rs. 9 million in respect of an insurance claim. The claim was lodged against
damage of a new machinery during the shipment that rendered it unfit for use.
(iii) Tax depreciation for the year on all fixed assets, other than imported second-hand plant,
amounted to Rs. 214 million.
(iv) Following are the details of losses brought forward from previous years:
Rs. in million
Loss from business relating to tax year 2021 52
Loss from speculation business relating to tax year 2022 14
Unabsorbed tax depreciation 168
Capital losses on sale of listed securities relating to:
▪ tax year 2019 8
▪ tax year 2020 6

Required:
Under the provisions of the Income Tax Ordinance, 2001 and Rules made thereunder:
(a) compute total income, taxable income and income tax liability of FB for the
tax year 2023. (17)
▪ Ignore minimum tax under section 113.
▪ Show all relevant exemptions, exclusions and disallowances.
(b) determine the amount of FB’s remaining unutilised tax losses alongwith the tax year
upto which such losses may be carried forward. (03)

Question # 19 Autumn 2023 Q. 4

(a) Sweet Bakers (SB) is a bakery business owned by Mariam and her two brothers, Ehsan
and Ghulam, who share profits in the ratio of 60:20:20, respectively. SB has three retail
outlets located in Karachi, and it also owns agriculture land that is rented to a chicken
farmer. SB is registered with sales tax authorities as a Tier-1 retailer.

362 | P a g e
The following information has been extracted from the records of SB for the year ended
30 June 2023:
Rs. in million
Net sales 500
Less: Cost of sales (350)
Gross profit 150
Less: Operating expenses (73)
Profit before tax 77

Additional information:
(i) Net sales include rental received from the chicken farmer, as detailed below:
A monthly payment of Rs. 0.5 million.
Supply of 120 paitis of eggs (agriculture produce) every month. Each paiti
holds a market value of Rs. 7,200. SB consumed these eggs in the production
of various bakery items, but they are not accounted for in the abovementioned
cost of sales.
(ii) Cost of sales include:
purchase of various raw materials worth Rs. 24 million on which no
withholding tax was deducted at the time of payment. SB made total purchases
of Rs. 200 million during the year.
purchase of milk powder worth Rs. 10 million, of which 10% is delivered to
the homes of the three partners for their personal use.
salaries of Rs. 8.2 million, Rs. 6 million, and Rs. 4.8 million to Mariam,
Ehsan, and Ghulam, respectively.
purchase of a new bakery plant worth Rs. 15 million.
(iii) Operating expenses include:
purchase of point-of-sale machines worth Rs. 0.4 million which were installed
on 1 July 2022 in all outlets to integrate with FBR’s computerized system for
real time reporting of sales.
payment of Rs. 5.5 million on 1 January 2023 to an IT company for the
development of an application to facilitate online orders from customers. The
useful life of this application is expected to be five years and is available for
use from 1 October 2023.
(iv) On 1 February 2023, Mariam contributed her personal van to the business, which
she had purchased at a cost of Rs. 6 million on 1 January 2022. The fair market value
of the van at the time of transfer was Rs.9 million.

Required:
Under the provisions of the Income Tax Ordinance, 2001 and Rules made
thereunder, compute under the correct head of income, the total income, taxable
income, and tax liability of SB for the tax year 2023. (16)
(xxiii) Ignore minimum tax under section 113.
(v) Show all relevant exemptions, exclusions and disallowances.

(b) Under the provisions of the Income Tax Ordinance, 2001, the tax payable by women
enterprises on profit and gains derived from business chargeable to tax under the head
‘Income from Business’ shall be reduced by 25%.

Required:
Explain the term ‘Women enterprise’. Also, discuss whether the abovementioned
reduction in tax liability is available to Sweet Bakers. (03)

363 | P a g e
ANSWERS
Question # 1 Spring 2010 Q. 1

Sohail, Khaled and Qazi (AOP)


Computation of Taxable and Divisible Income
Business income
Accounting Loss (1,500,000)
Less: Income from property (taxable under separate
head) 1,502,500

Add: Salary to Sohail 900,000


Salary to Khaled 600,000
Interest to Sohail 300,000
Interest to Khaled 300,000
Interest to Qazi 500,000 2,600,000
Business income/loss (402,500)

Income from property


Chargeable rent 2,000,000
Repair allowance 1/5th of chargeable rent (400,000)
Property tax (40,000)
Depreciation: Inadmissible Nil 1,560,000
Taxable income 1,157,500

Tax liability on income under NTR (15,000 +


357,500 x 15%) 68,625

Divisible Profit 1,157,500

Share of profit from AOP


Sohail Khaled Qazi Total
Salary 900,000 600,000 -- 1,500,000
Interest 300,000 300,000 500,000 1,100,000
Balance (2:2:1) (577000) (577000) (288,500) (1,442,500)
Share of profit 623,000 323,000 211,500 1,157,500

Computation of Taxable income of each member Sohail Khaled Qazi


Income from business 800,000 900,000 --
Income from consultancy ( 1000000 - 150000) 850,000
Taxable Income 800,000 900,000 850,000
Divisible profit from another AOP 255,000
Share of AOP 623,000 323,000 211,500
Income Including Share from AOP 1,423,000 1,478,000 1,061,500

Income tax 99,025 108,650 42,688


[Income tax / Total Income ] x Taxable Income 55,671 66,160 34,182

364 | P a g e
Question # 2 Spring 2012 Q. 1

Income from Business Rupees Rupees


From hospital (1.8m + 1.44m) / 90% 3,600,000
From clinic 8,750,000
12,350,000
Less:
rent of clinic 300,000
salary to assistant 180,000
clinic running exp 240,000
car exp 200,000 x 40% 80,000
Tax Depreciation on Car (96,000 x 40%) 38,400
Depreciation on Equipment 75,000
913,400
11,436,600
Income from property
Rent 870,000
Unadjustable deposit [700,000 - (550,000/10 *3) = 550,000 x 1/10] 55,000
Chargeable rent 925,000
Repair allowance 1/5th of chargeable rent (185,000)
Propert tax on building (8,000) 193,000
Total Income from Property 732,000

Income form other Source (Article writing) 75,000

Total Income under NTR 12,243,600

Tax liability (Non-Salaried Case)


Tax liability under NTR ( 765,000 + 35% x 8,243,600) 3,650,260
Less:
rebate on donation 3,650,260 / 12,243,600 x 300,000 89,441
3,560,819
Less:
tax deduction by hospital 360,000
tax deduction by tenant 50,000 410,000
Tax payable with return of income 3,150,819

Question # 3 Autumn 2012 Q. 1

Ms. Beena
Tax Year 20X8
Computation of Taxable Income and Tax Liability
Rs. Rs.
Salary
Basic salary 100,000 x 12 1,200,000
Bonus 200,000
Car benefit 2,000,000 x 5% 100,000
Fee for attending BOD meeting 150,000
1,650,000

365 | P a g e
Income from Business
Net profit as per accounts 3,500,000
Add: Inadmissible
Salary to Ms. Beena 50,000 x 12 600,000
Gift to Beena's son 50,000
Donation to Edhi Foundation eligible for rebate 250,000
Security deposit for lease 500,000
Legal charges for a personal law suit 150,000
Property expenses 2/5th of Rs. 350,000 140,000
Tax Deducted by the clients 200,000
Club Membership paid in cash 150,000 5,540,000
Taxable income 7,190,000

Tax liability (Non-Salaried Case)


Income tax on Rs. 4,000,000 765,000
Income tax on Rs. 3,190,000 @ 35% 1,116,500
1,881,500
Less: Rebate on donation 1,881,500 / 7,190,000 x 250,000 65,420
1,816,080
Less: Taxes deduction
From clients 200,000
from salary 390,000
from fee from attending BOD meeting 9,000 599,000
Tax Payable with return of income 1,217,080

Notes:
(1) Bonus pertaining to tax year 20X7:
Salary is taxable on receipt basis. Therefore, last year's bonus received in the current year is taxable in the
current year.
However, Where salary is paid in arrears and as a result the employee is chargeable at higher rate, the
employee may elect for the amount to be taxed at the rates of tax that would have been applicable if the
salary had been paid in the tax year to which it relates
(2) Bad debts recovered: The recovered bad debt is treated as income because it was claimed as expense in
the previous year
(3) Salary to brother: Salary paid to her brother is an allowable expense as he is working as an employee in
the business entity.
(4) Gifts to clients are allowable business expense and therefore not added back. Likewise, lease rental paid
to the bank is allowable business expense.

Question # 4 Autumn 2012 Q. 5(b)

Computation of taxable income & tax there on for the tax year 2023
Net loss (800,000)
Add: Inadmissible expenses
Salary to members of AOP 800,000
Accounting depreciation 300,000
300,000
Less: Brought forward business loss from the year 2022 (400,000)
Un-adjusted Business loss (100,000)
Business loss carried forward to next year (100,000)
Tax depreciation for the year 250,000
Unabsorbed depreciation bf 300,000

366 | P a g e
Unabsorbed depreciation to be carried forward 550,000
as there is loss under NTR & minimum tax liability ignored therefore no tax is payable by
the AOP under the given case

Question # 5 Spring 2014 Q. 1

Mr. Qamar
Tax Year 20X4
Computation of Revised Taxable Income and Tax Liability
Income before depreciation 12,000,000
Add: Electricity charges for residence 150,000
Donation to non-profit organization (eligible for rebate) 300,000
Fine paid to Ministry of Environment 200,000
Unabsorbed depreciation (deduction as a last item) 500,000
Taxable business income before depreciation 13,150,000
Less: B/f normal business loss for the tax year 20X3 4,000,000
9,150,000
Unabsorbed tax depreciation 500,000
Tax depreciation for the year 9,000,000 9,500,000
Business loss (350,000)

Income from Other Sources


Interest income: separate block of income 500,000
Other income 850,000
Taxable income 500,000

Since Total Income is less then minimum threshold income so there is no need to calculate the tax liability

Notes:
(1) Travelling expenses:
Travelling of Qamar to Malaysia to attend the trade fair was entirely for business purposes and the same
is allowable for tax purposes
(2) Electricity bills for residence:
Personal expense incurred by a person is not allowable expense for tax purposes and therefore the
electricity charges paid for the residence of Mr. Qamar are inadmissible
(3) Damages paid to a distributor:
It is an allowable tax expense in view of the fact that the same is not incurred for any fine or penalty paid
for the violation of any law, rule or regulation. The said expenditure has been incurred for the purpose of
business and therefore allowable
(4) Salary paid to brother:
Salary is allowable subject to two conditions as under:
(i) payment of salary is required through banking channel if it exceeds Rs. 25,000 per month; and
(ii) tax is required to be deducted if annual taxable salary exceeds basic exemption
It is irrelevant that the employee is a close relative or not. Therefore, salary to brother paid to Mr. Bari of
Rs. 720,000 is allowable expense irrespective of the fact that Bari is Qamar's brother.
(5) Donation:
Donation is eligible for rebate at average rate of tax on the lower of actual donation or 30% of taxable
income.
(6) Fine:
Any fine paid for the violation of any law, rule or regulation is not admissible for tax purposes. Since it is
incurred in breach of law imposed by the government, the same of not allowable expense.

367 | P a g e
Question # 6 Spring 2015 Q. 1

Rs. Rs.
Income from Business
Net profit as per accounts 1,057,000
Add: Inadmissible
Accounting depreciation on machinery 225,000
Computer software: intangible asset 975,000
Cost of feasibility study: pre-commencement expenditure 250,000
Finance charge on leased asset 80,000
Accounting depreciation on leased asset 260,000
Personal element in running and maintenance of vehicle
Rs. 295,450 x 20% 59,090
Provision of bad debt 25,000 1,874,090
2,931,090
Less: Initial allowance on machinery Rs. 1,500,000 x 25% 375,000
Normal depreciation on machinery (Rs. 1,500,000 - 375,000) x 15% 168,750
Amortization of computer software 975,000 / 12 81,250
Amortization of feasibility study 250,000 / 5 50,000
Lease rentals 300,000 975,000
Taxable income 1,956,090

Tax liability (Non-salaried Case)


Income tax on Rs. 1,200,000 75,000
Income tax on Rs. 756,090 @ 20% 151,218
226,218

Notes:
(1) Salary to brother:
Salary paid to Tahir's brother is allowable expense as he is working as an employee in the business and
the salary is not added back in the assumption that the salary has been paid through banking channel and
the tax has been duly deducted from salary.
(2) Salary from UAE:
Mr. Tahir was a returning expatriate in the tax year 20X4 as he was non-resident in the preceding 4 years
and being a returning expatriate, his foreign source income is exempt on the tax year 20X4 and 20X5
Therefore, salary received in the tax year 20X5 from UAE is exempt and not included in his taxable
income
(3) Plot is covered in the definition of capital asset under section 37A. Any capital gain on immovable
property being a capital asset is exempt if the holding period is more than 6 years.
Therefore, capital gain on disposal of plot is not taxable.

Examiner Comments:
This question required the computation of taxable income and tax liability of an individual involved in
garment manufacturing business with income from other sources also. The candidates were also
required to comment in respect of items which did not appear in their computation. The overall
performance in this question was average as only few students could score high marks.
A major issue in the performance in this question was that students tried to reconstruct the profit and
loss account of the business based on admissible expenses. This approach led to a number of mistakes
as inadmissible expenses were added to cost of sales instead of being subtracted. The proper approach
in such questions is to start from accounting profit before taxation and add inadmissible deductions
and subtract admissible deductions to arrive at taxable income. This approach requires much lesser
time and errors which occur on account of confusion with regard to the sign of the admissible/in-
admissible expense are also avoided.

368 | P a g e
Some of the common mistakes were as follows:
▪ Initial depreciation was not calculated on imported machinery from China because it was 'used
machinery' whereas this rule is applicable in case of machinery used in Pakistan.
▪ Normal depreciation on imported machinery was calculated on cost instead of cost less initial
allowance.
▪ Some candidates calculated correct amount of initial allowance on imported machinery in notes /
working of the question but this was not shown as a deduction/admissible expenditure in the main
calculation.
▪ Some students with weaker concepts added the value of imported machinery to accounting profit
in arriving at the taxable income.
▪ All sorts of errors were made in dealing with the leased vehicle. Very few students knew the correct
treatment i.e. financial charges and depreciation claimed as an expense in the profit and loss
account are to be added and rentals paid have to be deducted in arriving at the taxable income.
▪ Many candidates did not know that gain on sale of plot was exempt as the sale had taken place
after more than two years from the date of purchase. On the other hand, many candidates
mentioned that gain on sale of plot was not a capital gain and did not include it under any head.
▪ Some candidates considered salary paid to Tahir's brother as inadmissible. They failed to
appreciate that the same was admissible as Tahir's brother was an employee in the business.
▪ Majority of the candidates were aware of the fact that cost of feasibility study was inadmissible.
However, many candidates considered it as an 'intangible' instead of 'pre-commencement
expenditure'. Consequently, they allowed amortization at the rate of 10 per cent instead of 20 per
cent.
▪ Cost of the computer software was treated as fixed asset instead of an intangible and depreciation
was calculated thereon instead of amortizing it over 10-year period. Some candidates calculated
amortization on the basis of twelve years.
▪ With respect to the amount received from employment in UAE, a number of candidates restricted
their answers to the fact that such amount is exempt from tax being a foreign source income. They
were expected to fully explain that since Tahir was a citizen of Pakistan and was a non-resident
for the last four years, his foreign source salary is exempt from tax in the current as well as the
next year.
▪ Tax payable was calculated by applying rates of tax for salaried individuals.
▪ Majority of the students ignored an important requirement i.e. to comment on the items which did
not appear in the computation; instead, items involved in calculation were explained which was
not required.

Question # 7 Spring 2016 Q. 4

Loan Traders
Tax Year 2023
Computation of Taxable Income
Speculation Trading
Total
Business Business
Net sales after special discount 13,000,000
Add: Discount 680,000
Gross sales before special discount 22,800,000 9,120,000 13,680,000

Gross profit on speculation 500 x (18,240 - 15,800) 1,220,000


Gross profit on trading 25% of gross sales 3,420,000
Research expense (600,000)
Direct exp for speculation contract (25,000)
Common expenses 2,500,000
Less: Penalty being inadmissible 45,000
Apportioned on the basis of gross sales 2,455,000 (982,000) (1,473,000)
Net business income 213,000 1,347,000
B/f losses (300,000) (550,000)
Taxable income / loss (87,000) 797,000

369 | P a g e
Notes:
(1) Speculation loss of Rs. 87,000 can be carried forward to the next period but it can not be adjusted against
any income other than speculation.
(2) Capital loss is adjustable only against capital gain. In the absence of any capital gain for the year, capital
loss os c/f to the next tax period.
(3) Common expenses are apportioned on the basis of gross sales as the discount given is not as per normal
practice of LT. In case of normal practice of discount, the common expenses should be apportioned on the
basis of net sales after discount.

Examiner Comments:
This practical question carrying 10 marks required the students to compute the taxable income and
brought forward losses under various heads of a trader who was also involved in speculative business.
The overall performance was below average. The layout and presentation were also disappointing.
Some of the common mistakes were as follows:
▪ Sale of maize should have formed part of speculative business but was treated as part of trading
business.
▪ All the expenditures were deducted from trading business income.
▪ Loss against trading business was adjusted against speculative business income.
▪ Rs. 600,000 paid to research institute for development of formula was treated as intangible asset
whereas it was an admissible expense.
▪ Speculative loss and capital loss were added together and carried forward as a single item.

Marking Scheme:
▪ Computation of income from trading business 2.5
▪ Computation of income from speculation business 2.0
▪ Apportionment of common expenses 2.5
▪ Determination of taxable income / (loss) for the year 1.0
▪ Determination of the amount of loss to be carried forward 2.0

Question # 8 Spring 2016 Q. 8

BAR Associates
Tax Year 20X6
Computation of Taxable Income and Tax Liability
Rs.
Net profit before tax 5,488,000
Add: Provision for slow moving stock 75,000
Commission to a partner, Baqir 290,000
Performance award to a partner, Rahi 310,000
Loan of a partner, Asad paid by the firm 455,000
Provision for bad debts 735,000
Sales promotion expenses paid other than business bank a/c 275,000
7,628,000
Less: Trading debt written off against provision 115,000
Pakistan source business income 7,513,000

Foreign source business income


Profit from Dubai branch 1,500,000
Loss from Tehran branch (1,800,000)
Net loss c/f [not adjustable against foreign source income] (300,000) --
Taxable income 7,513,000
Income tax on Rs. 4,000,000 765,000
Income tax on Rs. 3,513,000 @ 35% 1229,550

370 | P a g e
1,994,550

Working of bad debts written off:


Opening balance of provision account 1,100,000
Add: Provision for the year 735,000
1,835,000
Less: Closing balance of provision account (1,435,000)
Debts written off during the year 400,000
Less: Loan to supplier written off - inadmissible 285,000
Bad debt written off allowed for tax purpose 115,000

Examiner Comments:
This practical question carrying 12 marks required the students to compute taxable income, net tax
payable and the amount to be carried forward by an AOP having two independent branches, one each
in Tehran and Dubai. The overall performance was good. However, some of the common mistakes are
discussed below:
▪ An important instruction in the question is that the computation should commence with the profit
before tax figure was ignored. Instead, adjustments were made in various line items and the entire
Profit & Loss Account was prepared which was a time consuming exercise.
▪ Profit and loss of foreign branches were combined with profit from Pakistan source income. Some
student adjusted Dubai branch profit against Pakistan source profit and carried forward Tehran
branch loss whereas profit of one foreign branch could have been adjusted from the loss of the
other foreign branch to arrive at net loss from foreign operations; and carried forward. Some
students mentioned that profit and loss of Dubai and Tehran branch could not be adjusted because
they would be subject to final tax.
▪ Many students rightly mentioned that freight charges could be paid in cash without any limit and
expenses incurred on training of employee to Federal Government run institution were allowable
but deducted these expenses from profit as admissible expense as they did not realize that these
expenses had already been deducted from income. Some students treated Rs. 86,000 paid to an
institution operated by the Federal Government for training of industrial workers in Punjab as
donation and calculated tax credit thereon.
▪ Majority of the students failed to calculate the amount of bad debt written off and allowed for tax
purpose. Most of those who calculated them correctly, did not realize that loan to the supplier
written off was not an admissible expense as it had not been included in business income
previously.
▪ Difference between the value of stock under absorption costing and net realizable value was
deducted from accounting profit instead of treating it as an addition.
▪ Surprisingly, some students could not apply the income tax slabs correctly.

Marking Scheme:
▪ Computation of the admissible / inadmissible expenses
- Bad debts expense 3.0
- Stock 1.5
- Up to 01 mark each for other items 4.5
▪ Foreign loss to be carried forward 2.0
▪ Determination of net tax liability 1.0

Question # 9 Spring 2017 Q. 1

Mr. Mushtaq
Tax Year 20X7
Computation of Taxable Income and Tax Liability
Rs.
Profit before taxation 1,800,000
Add: Inadmissible

371 | P a g e
Paid to a research institute in China: Admissible as per section 20 150,000
Accounting loss on disposal of patent 65,000
Tax gain on disposal of patent 524,000 - 430,000 94,000
Accounting amortization of patent 25,000
Bad debt recovered from Atif 700,000 - (800,000 - 550,000) 450,000
Tax gain on transfer of furniture to Dubai 850,000 - 610,000 240,000
Accounting depreciation 580,450
3,404,450
Less: Deficiency in bad debt recovered from Aslam 400,000 - (1,200,000 -
600,000) 200,000
1,100,00
Bad debts recovered credited in the a/cs 700,000 + 400,000 0 1,300,000
2,104,450
Less: Normal business loss of tax year 20X1 830,000 - 705,000 125,000
1,979,450
Less: Unabsorbed tax dep of the tax year 20X1 705,000
1,274,450
Less: Normal tax dep on furniture 200,000 x 15% 30,000
Initial allowance on imported machinery 500,000 x 25% 125,000
Normal tax dep on machinery (500,000 - 125,000) x 15% 56,250
Tax dep on other assets 456,400 667,650
Taxable business income 606,800

Capital gain u/s 37


Consideration received 432,000
Deemed cost 6,000 x 25 (150,000) 282,000
Taxable Income 888,800

Tax liability (Non-Salaried Case)


Income tax on Rs. 800,000 15,000
Income tax on Rs. 288,800 @ 15% 43,320
Tax liability 58,320

Less: Advance tax paid (200,000)


Tax refundable (141,680)

Examiner Comments:
This practical question carrying 16 marks required the students to compute the taxable income and
net tax payable by or refundable to an individual who was primarily engaged in the business of
manufacturing of different products but had other sources of income also. The overall performance
was good and about 64% of the students secured passing marks.
The common mistakes were as follows:
▪ Gain on sale of shares is a capital gain and should be taxed as a separate block, but many students
added it in income from business, whereas some of them added it for rate purposes only.
▪ Fair market value of the shares at the time of transfer was treated as capital gain.
▪ All sorts of errors were made in respect of amounts recovered from Atif and Aslam. The candidates
are advised to refer to ICAP's suggested answer to understand the working.
▪ Few candidates did not offer any treatment for transfer of furniture to Dubai. Transfer of furniture,
being a depreciable asset, out of Pakistan, should have been treated as disposal for a consideration
equal to cost of the furniture.
▪ Amortization on patent charged in the year of sale was deducted for arriving at tax written down
value.
▪ Full year's depreciation is to be allowed in the year of purchase. Many candidates restricted the
depreciation on furniture and imported machine to 4 months.
▪ Normal depreciation on imported machinery was calculated on cost instead of cost less initial

372 | P a g e
allowance. Many students did not claim initial allowance probably on the assumption that the
machinery was used whereas the same was allowed because the machinery was being used in
Pakistan for the first time.
▪ Initial allowance was claimed on furniture.
▪ Amount paid to the research institution in China for development of a new product was considered
as an admissible expense.
▪ Amount of advance tax was added back in profit before tax as inadmissible expense and was not
deducted from tax payable.
▪ The entire amount paid to salesmen in cash was considered inadmissible. Part of the amount which
represented reimbursement on account of entertainment expenses was admissible.
▪ Many candidates did not set-off brought forward business loss against the business income for the
year; perhaps on the assumption that the adjustment was subject to time limitation. They failed to
recognize that it was the 6th successive year following the year in which the loss first arose and
was adjustable.
▪ Some students did not read the question carefully and deducted assessed loss of tax as well as the
unabsorbed tax depreciation whereas it was clearly mentioned in the question that brought
forward losses were inclusive of unabsorbed depreciation.

Marking Scheme:
▪ Computation of the admissible / inadmissible expenses and income
− Bad debts 3.0
− Depreciation – accounting and tax 3.0
− Intangibles 3.0
− Up to 01 mark each for all other items 3.5
▪ Computation of income from capital gain 2.0
▪ Computation of net tax payable 1.5

Question # 10 Spring 2018 Q. 1

Mr. Qateel (Qateel Enterprises)


Tax Year 20X8
Computation of Taxable Income and Tax Liability
Rs.
Profit before taxation 2,809,297
Add: Inadmissible
Finance charge on lease machine 35,703
Fine for violation of a business contract: Admissible --
Accounting depreciation 1,900,000
Renewal of manufacturing license: intangible 450,000
Vehicle tax paid in cash: Admissible being government dues --
Security deposit to K-Electric 200,000
Advance income tax through electronic bills 300,000
Donation in cash to poor families 64,600
Donation to NPO: eligible for rebate 2,000,000
Penalty for late filing of income tax return 25,000
Entertainment expenses for business purpose: Admissible --
7,784,600
Less: Amortization of renewal fee 450,000 / 15 30,000
Dividend taxable under FTR 580,000
Gain on disposal of shares under taxable under capital gain 1,200,000
Lease rentals for machine 270,000
Normal depreciation on machine already in use of Qateel
Enterprises 100,000 x 15% 15,000
Normal depreciation on warehouse

373 | P a g e
10% of Rs. 1,040,000 104,000
Tax depreciation on assets other than above 1,560,000 3,759,000
Taxable business income 4,025,600

Capital Gain
Gain on disposal of a private company's shares 1,200,000

IFOS (FTR)
Dividend (580,000+100,000)/.85 =800,000 800,000

Total income 6,025,600


Less: Dividend Income (800,000)

Total Income under NTR 5,225,600

Less: Zakat deducted from dividend 100,000


Taxable income 5,125,600

Tax liability (Non-Salaried Case)


Tax on Rs. 4,000,000 765,000
Tax @ 35% on Rs. 1,125,600 393,960
1,158,960
Rebate on donation of Rs. 1,537,680 (i.e. actual donation Rs. 2 million or
20% of taxable income, whichever is lower)

1,158,960 / 5125600 x 1,537,680 347,688


1,506,648
Tax on dividend (580,000 + 100,000) / 85% x 15% 120,000
1,626,648
Less: Advance income tax with electric bills 300,000
Tax paid on dividend (580,000 + 100,000) / 85% x 15% 120,000
Advance tax paid 480,000 900,000
Tax refundable (726,648)

Examiner Comments:
This practical question carrying 20-marks required the candidates to compute total income, taxable
income and net tax payable by or refundable to a resident individual under the provisions of the
Income Tax Ordinance, 2001 and rules made thereunder. This type of question covers a number of
areas ranging from easy to difficult. Overall performance of the students in this question was
reasonable as 41% candidates secured passing marks. A major issue in most cases was that total
income was not computed. The other common mistakes noted were as follows:
▪ Fine for the violation of contract with a customer was considered as inadmissible whereas only
the fine for violation of law is inadmissible.
▪ Cash donation to poor families amounting to Rs. 64,600 was considered eligible for tax credit.
▪ Some candidates were of the view that entertainment expenditure incurred on arrival of foreign
customers for business purposes is inadmissible.
▪ All sorts of errors were made in dealing with the leased machinery. Few candidates knew the
correct treatment i.e. financial charges shown as expense in the profit and loss account were to be
added and rentals paid were to be deducted in arriving at the taxable income. Many candidates
failed to claim depreciation on leased machinery transferred to the company on maturity of lease
whereas some of them claimed it on the market value instead of residual value. Some of them
computed initial depreciation also which was incorrect as initial depreciation is allowed only when
the asset is used for the purpose of business for the first time or the tax year in which commercial
production is commenced, whichever is later.
▪ Zakat deducted under the Zakat and Usher Ordinance, 1980 at source was treated as inadmissible

374 | P a g e
expenditure and added back in profit before tax. Some students deducted zakat from tax payable.
▪ Vehicle tax of Rs. 55,000 was added back as inadmissible expense on the ground that it was paid
in cash. Some students treated it as advance income tax and adjusted it against tax liability.
▪ Rs. 200,000 paid as security to K-Electric for replacement of transformer at the factory was treated
as admissible expense.
▪ Dividend was grossed up by the amount of tax deducted i.e. 15% before adding the amount of
Zakat deducted at source. Moreover, in some cases dividend was shown as taxable under normal
tax regime whereas some students calculated withholding tax on net amount of dividend. In some
cases dividend was taxed at the rate of 12.5% instead of 15%.
▪ Capital gain of Rs. 1,200,000 from sale of shares of a private limited company was treated as
exempt because these shares were held for more than three years whereas some students treated
the entire amount as taxable. In fact, it should have been reduced by 25% as the shares were held
for more than one year. Some students treated the gain as separate block of income.
▪ Useful life of renewal of a manufacturing license (intangible asset) was considered as fifteen years
instead of 10 years whereas when an intangible has a useful life of more than 10 years, it shall be
treated as if it had a normal useful life of ten years.

Marking Scheme:
▪ Computation of taxable income:
- 0.5 mark for each adjustment relating to inadmissible expenses / admissible
income 6.5
- up to 1.5 marks for each adjustment relating to admissible expenses / exempt
(or FTR) income 5.0
- determination of total income and taxable income 2.5
▪ Determination of total tax liability 2.5
▪ Determination of income tax refundable 1.5

Question # 11 Autumn 2018 Q. 5

Mr. Saleem
Tax Year 2026
Computation of Taxable Income and Tax Liability
Rs.
Profit before taxation 2,162,500
Add: Inadmissible
Rent: tax withheld but not deposited into Government treasury 1,560,000
Depreciation on leased asset 600,000
Financial Charges 462,000
Tax Gain on disposal of vehicle 683,824
3,305,824
Less: Accounting profit on sale of vehicle 400,000
Lease rentals Rs. 857,000 x 70% 599,900
Accounting gain on disposal of shares taxable under capital gain 50,000 1,049,900
Taxable business income 4,418,424

Capital Gain under section 37


Gain on disposal of shares of Sun (Private) Ltd 500,000

Capital Gain under section 37A


Loss on disposal of shares of Moon Ltd (700,000)
Gain on disposal of shared of Planet Ltd 250,000
Loss u/s 37A can be adjusted only against gain u/s 37A (450,000) --
Taxable income 4,918,424

375 | P a g e
Tax liability (Non-Salaried Case)
Tax Liability (765,000 + 918,424 x 35%) 1,086,488

Less: Advance income tax on vehicle 85,000


Tax payable with return of income 1,001,488

Working: Disposal of vehicle


Consideration on disposal [8,000,000 x (7,500,000/8,500,000)] 7,058,824
Less Tax WDV on Disposal [7500000 x 0.85%] (6,375,000)
683,824

Examiner Comments:
This question required calculation of taxable income under appropriate heads and tax liability of a
resident individual driving income from a fitness club. The performance remained below average as
only 34.4% of the candidates secured passing marks. The common errors were as follows:
▪ Written down value was incorrectly calculated as most of the students seemed unaware of the rule
that for tax purposes, cost of passenger transport vehicle is restricted to Rs 7.5 million and when
the same vehicle is sold, the sale proceeds are reduced in the same ratio in which the cost was
restricted, for the purpose of computing tax gain/loss.
▪ Lease rental was not pro-rated between business and personal use. Some candidates considered
the lease rentals as inadmissible.
▪ Capital gains on listed securities was not computed correctly;
▪ Some students did not compute the tax correctly as applicable to individuals - who are not salaried.
Amount of lease rental paid was added as inadmissible expense. Some candidates deducted the
whole amount of lease rental as admissible expense without adjustment of 30% use of vehicle for
personal purposes.
▪ Loss on the disposal of Moon Limited was considered as inadmissible and added to accounting
profit instead of adjusting it against gain on disposal of shares of Planet Limited.
▪ Depreciation and financial charges on car acquired on finance lease were considered as
admissible.
▪ Tax rates applicable to salaried individuals were used instead of tax rates applicable to non-
salaried individuals.
▪ Total income was not computed head wise.

Marking Scheme:
▪ Computation of taxable income
− Income from business:
o inadmissible expenses 2.0
o admissible expenses 3.0
o loss on disposal of passenger transport vehicle 3.0
− Income from capital gain 2.5
▪ Computation of tax liability 1.5

Question # 12 Spring 2019 Q. 1

MAZE (AOP)
Tax Year 20X9
Computation of Taxable and Divisible Income
Rs.
Net profit as per accounts 13,870,000
Add: Adjustment on account of change in accounting method
Closing stock under Absorption costing method 8,800,000
Closing stock under Prime cost method (7,500,000) 1,300,000
Cost of machinery charged in the accounts 2,110,000

376 | P a g e
Salary to partners 150,000 x 3 x 12 5,400,000
Accounting software charged in the accounts 180,000
22,860,000
Less: Initial allowance on machinery 25% of (2,110,000 -
110,000) 500,000
Normal depreciation on machinery @ 15% 225,000
Amortization of software (180,000 / 15) x 181 /
365 5,950
Interest income as separate block of income 360,000 1,090,950
21,769,050
Business Income
Income from Other Sources
Interest income as separate block of income
360,000 / 90% 400,000 --
Taxable Income 21,769,050
Income tax on Rs. 4,000,000 765,000
Income tax on Rs. 17,769,050 @ 35% 6,219,168 6,984,167
Add: Tax on Profit (360,000/.90) x 15% 60,000
Total tax liability 7,044,167
Advance Tax (110,000)
Tax deducted on Profit (40,000)
Tax Payable 6,894,167

Profit after Tax 14,874,883

Share of Profit from AOP


M A Z Total
1,800,00 1,800,0
Salaries from MAZE 0 00 1,800,000 5,400,000
5,456,35 5,456,3
Balance (equal share) 0 50 5,456,350 16,369,050
7,256,35 7,256,3
Share of profit for the year 0 50 7,256,350 21,769,050

Taxable income and tax liability of Mr. Z


Salary
Monthly salary 200,000 x 12 2,400,000
Reimbursement of medical 3,500,000 exempt
Purchase of car 250,000 - 110,800 139,200
Taxable Income 2,539,200
Add: Share of profit from AOP for rate purpose 7,256,350
Taxable Income for rate purpose 9,795,550

Tax liability (Salaried Case)


Tax Liability [1,095,000 + (9,795,550 - 6,000,000)
x 35%] 2,423,425
Tax liability on actual income [2423425/9795550 x
2539200] 628,199

Examiner Comments:
(a)
(i) Students were aware that profit before taxation was to be adjusted on account of change in the
accounting method from cash basis to accrual basis. However, they failed to consider that only

377 | P a g e
the difference between the values of closing stock under absorption costing and prime cost method
was to be added to the accounting profit. Further, difference between the opening balances of the
stock-in-trade under the two methods required no adjustment against the accounting profit.
(ii) Payment for holding annual eid-milan party was treated as inadmissible expense.
(iii) Students did not add back interest income, being a separate block of income, to the profit before
taxation. Similarly, they did not deduct the same from the total income in order to arrive at the
taxable income for the year. Consequently, they failed to correct total income.
(iv) Adjustment relating to income tax paid at import stage in the cost of machinery. Further, few
candidates wrongly excluded the custom duty from the cost of machinery in order to arrive at the
depreciable value of machinery.
(v) Normal depreciation on machinery was calculated on cost instead of cost less initial allowance.
(vi) Correct amount of initial allowance on imported machinery was calculated in the working but
this was not shown as a deduction / admissible expenditure in the main calculation.
(vii) Amortisation of computer software was computed based on number of months instead of number
of days in use.
(viii) Tax on interest income at the rate of 10% being a separate block income was not computed.
(ix) Tax deducted at source was not adjusted against the tax liability.
(b)
(i) While calculating divisible profit for each partner, the students considered both NTR and FTR
instead of taking NTR only for rate purpose.
(ii) Reimbursement of actual cost of medical services was treated as taxable.
(iii) Share of profit from AOP was not included in the taxable income of the partner for rate purposes.
Among those who considered the share of profit, failed to exclude monthly remunerations from
the partner’s share of firm’s profit.

Marking Scheme:
(a)
▪ Computation of total income:
− 0.5 mark for each adjustment relating to inadmissible expenses/admissible income 4.5
− 0.5 mark for each adjustment relating to admissible expenses /exempt (or FTR ) income 3.5
▪ Determination of taxable income 0.5
▪ Determination of tax liability 1.5
(b)
▪ Computation of taxable income without share of AOP 1.5
▪ Computation of share of AOP 3.5
▪ Determination of tax liability 2.0

Question # 13 Spring 2020 Q. 1

Mr Shahid
Computation of total income, taxable income and net tax payable/refundable
For tax year 2023
Computation of profit under accrual basis of accounting
Profit as given in the question - on cash basis 6,427,000
Adjustment on account of:
- closing stock under absorption cost method 3,200,000
- closing stock under prime cost method (2,800,000)
400,000
Profit under accrual basis of accounting 6,827,000
Income from business
Profit before taxation 6,827,000
Add: Inadmissible expenses/admissible income
Purchases of packing material (440,000×20%) 88,000
Freight charges on goods - allowed expenditure -
Salary allowed as paid for business activities (brother) -
Penalty for late filing of income tax return 15,000

378 | P a g e
Expenditure on promotion of a product 950,000
1,053,000
Less: Admissible expenses/inadmissible income
Expenditure made for promotion of a product = 950,000/5 (190,000)
Tax depreciation (680,000)
Gain on sale of shares (45,000)
Agriculture income - Exempt income (980,000)
Profit on debt (450,000)
(2,345,000)
5,535,000
Less: Unabsorbed tax depreciation - brought forward (568,000)
Total business income for the year 4,967,000
Capital gain
Gain on the sale of 20,000 shares 51,750
Income from other sources
Profit on fixed deposit account (FTR income) 450,000
Exempt income
Rent received for the agriculture land 980,000
Total income 6,448,750
Less:
Capital gain on sale of shares (Separate block of income) 51,750
Profit on fixed deposit account (FTR) 450,000
Rent received for the agriculture income (Exempt) 980,000
1,481,750
4,967,000
Less: Deductible allowance
Zakat paid / deducted 93,750
Taxable income for the year 4,873,250

Tax liability
Tax on Rs. 4,000,000 765,000
Tax on amount exceeding Rs. 4,000,000@ 35% 305,637
1.070.637
Profit on debt taxable as separate block Rs.450,000 x 15% 67,500
Tax on capital gain @ 12.5% (Rs. 51,750 @ 0.125) 6,469
1,144,606
Less: Tax deducted
by customers 875,000
on capital gain 6,750
on fixed deposit account 45,000
926,750
Net tax payable 217,856

Examiner Comments:
▪ Sources of income were not classified under appropriate heads.
▪ Examinees failed to comprehend that only the difference between the values of closing stock under
absorption cost and prime cost were to be added to the accounting profit. Whereas, difference
between the opening balances of the stock-in-trade under the two methods required no adjustment.
▪ The whole amount of packing material was considered as inadmissible deduction whereas only
20% of the amount was inadmissible.
▪ Freight charges and salary paid to brother were considered as inadmissible whereas penalty of
Rs. 15,000 was treated as admissible deduction.
▪ Examinees considered Rs. 950,000 incurred on the development of a product as admissible
deduction in the same period whereas it was to be amortized over the period of five years.

379 | P a g e
▪ Unabsorbed tax depreciation was adjusted against ‘Total income’ instead of ‘Income from
business’.
▪ Examinees failed to appreciate that the value of profit on debt was gross of withholding income
tax and zakat. Moreover, in some of the cases, profit on debt was not reclassified as “Income from
other sources” being FTR income.
▪ Examinees failed to gross up the amount of capital gain at Rs. 51,750.
▪ Examinees did not reclassify agriculture income as ‘exempt income’.
▪ Tax deducted at source by customers of Rs. 875,000 was treated as admissible deduction whereas
no tax credit was claimed against the taxes deducted at source by customers on capital gain and
fixed deposit account while computing the final tax liability.

Marking Scheme:
Computation of taxable income
▪ Computation of profit under accrual basis of accounting 2.0
▪ Up to 01 mark for each adjustment relating to:
− Income from business 7.0
− Income from capital gain 1.25
− Income from other sources 1.25
− Exempt income 1.0
− Deductible allowance 1.0
▪ Computation of tax liability 3.0
▪ Adjustment of withholding tax 1.5

Question # 14 Spring 2021 Q. 3

Muhammad Asghar
Computation of total income, taxable income and net tax payable/refundable
For tax year 2021
Rs.
Income from business
Profit before tax 2,468,000
Add: Inadmissible expenses / admissible income
Raw material and finished goods disallowed 20% 7,800,000×20% 1,560,000
Accounting depreciation 2,100,000
Provision for slow moving inventory 1,800,000
Expenditure on Eid-Milan party -
Penalty paid to a customer -
Donation paid to hospital established by local government 2,300,000
Installation charges of imported plant 375,000
Reward paid in cash to salesmen 500,000
8,635,000
Less: Admissible expenses / inadmissible income
Amortization of pre-commencement expenditure 3,400,000×20% (680,000)
Tax depreciation (1,900,000)
Initial allowance of imported plant (2,500,000+375,000)×25% (718,750)
Depreciation of imported plant (2,500,000+375, 718,750)×15% (323,438)
Dividend received from a listed company (174,000)
Gain on sale of shares in APL (660,000)
(4,456,188)
Income from Business 6,646,812
Income from other sources
Dividend received - FTR income (174,000+30,000=204,000/0.85) 240,000
Capital gain

380 | P a g e
Gain on disposal of APL (Treated as a public company because 60% shares of APL are held
by Fed Govt.) 660,000
Total income for the year from all sources 7,546,812
Less: Separate block of income
Dividend received - FTR income 240,000
Gain on disposal of public company 660,000
900,000
6,646,812
Less: Deductible allowance
Zakat paid/deducted (30,000)
Taxable income under NTR 6,616,812

Tax liability
Tax on Rs. 4,000,000 765,000
On balance 2,616,812 x 35% 915,884
Tax liability under normal tax regime 1,680,884
Less: Tax credit on donation
Lesser of 2,300,000 or 30% of the taxable income i.e. Rs. 19 85 04 3 (504,265)
(1680884/6,616,812×1985043)
1,176,619
Dividend income: Tax at the rate of 15% - FTR 36,000
Gain on sale of shares of public company (660,000 x 12.5 %) 82500
Total tax payable 1,295,119
Less: Advance tax paid (200,000)
Withholding tax deducted (1,600,000)
Withholding tax deducted (Dividend) (36,000)
Tax collected at import stage (150,000)
Tax refundable 690,881

Examiner Comments:
▪ Examinees failed to appreciate that 20% of raw material and finished goods were inadmissible on
account of failure to withhold tax whilst making payment.
▪ Expenditure on Eid-Milan party and penalty paid to a customer were considered inadmissible.
▪ Pre-commencement expenditure was also added back without realizing that the expense was
incurred in the prior year. Only 20% amortization was to be allowed thereon.
▪ Income tax paid at import stage was deducted from the cost of plant.
▪ Dividend was grossed up by 85% without adding the amount of Zakat deducted at source.
▪ Zakat was deducted from total income.
▪ Donation was taken as a straight deduction from taxable income instead of claiming tax credit
based on average rate of tax.

Marking Scheme:
▪ Computation of:
− income from business 9.0
− income from capital gain 1.0
− income from other sources 1.5
▪ Deduction of separate block of income 1.0
▪ Computation of:
− deductible allowance and its treatment 1.0
− tax liability under NTR 1.0
− tax credit 1.5
− tax liability under FTR 1.0
▪ Adjustment of withholding tax 2.0

381 | P a g e
Question # 15 Autumn 2021 Q. 4

Kamyab Enterprises
Computation of total income, taxable income and net tax payable/refundable
For tax year 2023
Rupees
Income from business
Profit before tax 12,707,000
Add: Inadmissible expenses / admissible income
Accounting depreciation 1,200,000
Salary paid in cash 40,000
Cost related to scientific research incurred in Pakistan -
Expenditure paid to Chinese company for research work 700,000
Instalment of industrial plot being capital payment in nature 650,000
Purchase of goats for Eid-ul-Azha 225,000
Donation to approved NPO 1,000,000
Health insurance premium 200,000
4,015,000
Less: Admissible expenses and inadmissible / FTR income
Tax refund received from Income tax department (720,000)
Tax depreciation (W-1) (1,613,550)
Loss on disposal of motor vehicle (W-2) (28,000)
Capital gain not to be taxed under business income (430,000+250,000) (680,000)
(3,041,550)

Income from business 13,680,450

Capital gain
Capital gain on sale of securities (430,000+250,000) 680,000

Income from other sources


Additional payment received on delayed tax refund 80,000
Total income for the year from all sources 14,440,450

Less: Capital gain on disposal of investment in Himmat Limited as separate block of income (250,000)
Taxable income under NTR 14,190,450

Tax Liability [765,000 + (14,190,450 – 4,000,000) x 35%] 13,555,450

Tax Credit on Donation (13,555,450/14190450 x 1,000,000 (295,738)


Tax on Securities (250,000 x 12.5%) 31,250

Tax Payable 13,290,962


*being donation amount is less than 30% of taxable income.

W-1 Tax Depreciation Rupees


P& M (6860000 x 15%) 1,029,000
Initial allowance 37,500
Dep of new computer (150,000 – 37,500) x 30% 33,750
Computer (800,000 x 30%) 240,000
Motor Vehicle (3,000,000 – 722,500) x 15 % x 80% 273,300
1,613,550

W-2 Computation of tax loss on sale of motor vehicle Rupees


Cost
Depreciation TY 2021 (1,000,000×15%) (150,000)

382 | P a g e
TY 2022 (850,000×15%) (127,500)
TY 2023 -
(277,500)

Tax WDV 722,500


Disallowed depreciation (277,500×20%) 55,500
778,000
Sale proceeds 750,000
Loss on disposal 28,000

Examiner Comments:
▪ In order to compute tax loss on sale of motor vehicle, disallowed depreciation on account of use
of motor vehicle for personal purpose was not added to its tax written down value.
▪ Initial allowance on new computer was not calculated.
▪ Normal depreciation on new computer was not reduced by 50%.
▪ Payment to Chinese company related to 'In-house scientific research' was treated as an admissible
expense.
▪ Capital gain on sale of investment in shares of unlisted company was considered as separate block
of income.
▪ While calculating tax credit for investment in health insurance, maximum limit of Rs. 150,000 was
ignored.
▪ Tax refund received from income tax department was not deducted from profit before tax. Further,
it was adjusted from income tax liability for the year.
▪ Additional payment received from income tax department due to delay in tax refund was not taxed
under income from other sources.

Marking Scheme:
▪ Income from business (including computation of tax depreciation and tax loss on disposal of fixed
asset) 9.0
▪ Income from capital gain 3.0
▪ Income from other sources 1.0
▪ Deductions from total income 2.0
▪ Tax credits 2.0
▪ Tax liability 1.0

Question # 16 Spring 2022 Q. 3

Computation of total income, taxable income and net tax payable/refundable


For tax year 2023
Income from business Rs. in million
Loss before tax (87.0)
Add: Inadmissible expenses / admissible income
Commission expense disallowed due to sale to inactive tax payer
2.4
[2.5−0.1(50×0.2%)]
Accounting depreciation 40.0
Bad debts recovered from Shameem [16.8−6(19.2−13.2)] 10.8
Outstanding payments for more than 3 years 14.0
Financial charges waived by the bank 2.8
70.0
Less: Admissible expenses and inadmissible / FTR income
Penalty -
Freight charges paid in cash -
Tax depreciation (48.0)
Insurance claim received (6.0)
Loss on disposal of vehicle (W-1) (0.92)
Reversal of Bad debts recovered recorded as other income (16.8+10.6) (27.4)

383 | P a g e
Bad debts recovered from Faheem [10.6 14(28.8 14.8)] (3.4)
Rental income – Chargeable under income from other sources (21.6)
(107.6)
Income from non-speculation business (124.32)
Income from speculation business Net gain from derivative contract 23.0
Income from business (A) (101.89)

Capital gain
Sale of property (20) 20
Sale of private company shares (3.6) 3.6
(B) 23.6
Income from other sources
Rental income from leasing of property comprised of building and 2nd hand
21.6
locally purchased plant (1.8×12)
Less: Deductions
Repair and maintenance (actual) (3.2)
Depreciation of building (85×90%×90%×10%) (6.9)
Depreciation of plant (34×15%) (5.2)
(C) 6.3
Total income (A+B+C) (71.99)
Less: Capital gain on sale of property (separate block of income) (20)
Taxable income (91.99)

Since Aakash’s taxable income for tax year 2022 is negative, his share of profit from associate is ignored.

Tax Liability Rs. in million


Tax on capital gain on sale of property (separate block of income) (20×7.5%) 1.5

W-1: Loss / Gain on disposal of vehicle


Rs. in million
Insurance claim (7.5/10 x 6) 4.5
Cost 7.5
Depreciation TY 2021 (7.5× 15%) (1.125)
TY 2022 (7.5×85%×15%) (0.956)
TY 2023 -
WDV (5.42)
Loss on disposal of vehicle (0.92)

Examiner Comments:
▪ Entire commission expense was considered as admissible expense rather it should be restricted to
0.2 percent of sales being distributor's name is not appearing in the active taxpayers' list.
▪ Freight charges paid in cash were considered as an inadmissible expense.
▪ Unpaid liabilities in respect of purchases made on 1 August 2018 were subject to tax in tax year
2022, considering 2022 as a first year following the end of the three years.
▪ Incorrect amounts of debt were included/excluded from loss before tax following partial recovery
of bad debts.
▪ Gain on selling the derivative contract before its settlement date was not identified as income from
speculation business.
▪ Rental income from leasing of factory building along with the plant was subject to tax under the
head 'Income from property'. Consequently, instead of deducting actual repair and maintenance
as an allowable expense, 1/5th of rent was deducted. Further depreciation on building as well as
plant was calculated incorrectly.
▪ Capital gain on sale of shares of private company was considered as separate block of income.

384 | P a g e
Marking Scheme:
▪ Income from business
– Non-speculation (including computation of loss on disposal of vehicle) 11.0
– Speculation 1.0
▪ Capital gain 2.0
▪ Income from other sources 3.0
▪ Tax liability on separate block of income 1.0

Question # 17 Autumn 2022 Q. 1

Azadi and Co.


Computation of total taxable income and tax liability for tax year 2022
Rs. in million
Loss before tax (66.00)
Add: Inadmissible expenses / admissible income
Payment for purchase of machine 12.00
Purchase of raw material for which no withholding tax was deducted (120×20%) 24.00
Salary paid to office boy in cash -
Warehouse rent -
Bad debt written off -
Commission paid to member of AOP 3.20
Payment for purchase of accounting software 6.40
Depreciation on leased car 2.10
Lease financial charges 1.50
49.20
Less: Admissible expenses and inadmissible income
Tax depreciation on machine [3.5(W-1)+1.575(W-1)] (5.075)
Loss on disposal of old machine (4–5) (1.00)
NRV adjustment (18–12) (6.00)
Scholarship given to Sara -
Amortization of ERP software [6.4÷5×(91÷365)] (0.32)
Lease rentals [0.71(3×2.5÷10.5)×80%] (0.57)
(12.9)
Total income (28.7)

Computation of minimum tax:


Turnover (380 – 45 – 15) 320.00
Tax @ 1.25% 4.00

W-1: New Machine


Cost: Rs. in million
Payment through cheque 12.00
Sales tax on purchase (2.00)
Fair market value of old machine 4.00
14.00
Initial allowance @ 25% (3.50)
10.50
Depreciation for the year @ 15% (1.575)
8.425

(b) As Sara is an employee of AC and employee is not considered as an associate of employer, the scholarship
received by Sara from AC is exempt from tax.

385 | P a g e
Question # 18 Spring 2023 Q. 3

(a) Faith Brothers


Computation of total income, taxable income and tax payable/refundable For the tax
year 2023
Rs. in million
Income from business
Profit before tax 177.0

Add: Inadmissible expenses


Accounting depreciation 188.0
Payment to builder 50.0
Payment to a research institute in UAE 5.0
Foreign exchange loss 15.8
Loss on disposal of a capital asset (new machinery damaged during
shipment) 9.0
267.8
Less: Admissible expenses
Cash payment for the purchase of ten air ticket -
Payment of demurrages -
Payment of penalties -
Bad debts recovered not allowed in the previous year (16.0)
Shortfall in recovery of bad debts 16–(30–10) (4.0)
Reversal of capital gain (40.0)
(60.0)
Income before tax depreciation 384.8

Less: b/f business loss (52.0)


Tax depreciation (214.0)
Tax depreciation on plant (W-1) (3.9)
(217.9) (269.9)
Income before unabsorbed tax depreciation 114.9
Less: Unabsorbed tax depreciation 114.9×50% (57.5)
Total business income 57.4

Capital gain
Modaraba certificates 8.0
Shares of an unlisted company 12.0
Loss on disposal of a capital asset (new machinery damaged during
shipment) (9.0)
3.0
Shares of a listed company 20.0
Less: B/f capital loss (6.0)
14.0
Total income 82.4

Less: Separate block of income


Capital gain on Modaraba (8.0)
Capital gain on listed company share (14.0)

386 | P a g e
(22.0)
Taxable income 60.4
Tax liability
Upto 4 million 0.765
Above 4 million 56.4*35% 19.74
20.505
Tax on capital gain – on madaraba 8×15% 1.2
– on listed company shares 14×12.5% 1.8
23.505

W-1: Cost of plant and depreciation thereon


Rs. in million
Purchase cost (400,000×45) 18.0
Foreign exchange gain – June 2022 400,000×5(45–40) (2.0)
16.0
Initial allowance @ 25% (4.0)
12.0
Depreciation – tax year 2022 12×15% (1.8)
10.2
Foreign exchange loss – June 2023 400,000×39.5(79.5–40) 15.8
26.0
Depreciation – tax year 2023 26×15% (3.9)
22.1

(b) Amount of unutilized losses alongwith maximum period to which these losses can be carried
forward:
Given Utilized c/f Maximum
------- Rs. in million ------- period (TY)
Loss from business - Tax year 2021 52 52 - -
Speculation - Tax year 2022 14 - 14 2028
Unabsorbed tax depreciation 168 57.5 110.5 No time limit
Capital loss on sale of listed securities:
- Tax 2019 8 - - Already lapsed
- Tax 2020 6 6 - -

Question # 19 Autumn 2023 Q. 4

(a)Sweet Bakers
Computation of total income, taxable income and tax liability For tax year 2023

Rs. in million
Income from business:
Profit before tax 77.00
Less: Rental income
- in cash (0.5×12) (6.00)
- in kind (120×7,200×12) (10.37)

387 | P a g e
(16.37)
Less: Market value of eggs received as a rent and used as a raw
material (10.37)
Add: Purchase of various raw material on which no withholding tax was
deducted [24 or 40 (20% of 200 i.e. total purchase, whichever is
lower] 24.00
Add: Purchase of milk powder for personal use (10×10%) 1.00
Add: Salaries to partners (8.2+6+4.8) 19.00
Add: Purchase of new bakery plant 15.00
Less: Initial allowance (15×25%) (3.75)
Less: Normal depreciation [11.25(15–3.75)×15%] (1.69)
Add: Purchase of POS machines 0.40
Less: Depreciation on POS machines (cost of POS machines are
allowed as a tax credit) -
Add: Payment to IT company for development of an app. 5.50
Less: Amortization expense (App is available to use subsequent to the
year-end) -
Less: Depreciation of van (*9×15%) (1.35)
108.37
Exempt income:
Rent from agriculture land 16.37

Total income 124.74


Less: Rent from agriculture land (16.37)
Taxable income 108.37

Tax liability:
- On Rs. 4 million 0.765
- On excess amount @ 35% 36.53
37.295
Less: Tax credit u/s 64D [Amount invested i.e. Rs. 0.4 OR maximum limit
i.e 0.45(0.15×3), whichever is lower] (0.40)
36.895

*Rs. 7.5 million is also considered correct.

(b) Definition
Woman enterprise means a start-up established on or after first day of July 2021 as sole
proprietorship concern owned by a woman or an AOP all of whose members are women or
a company, whose 100% shareholding is held or owned by women.
Reduction in tax liability:
Since all partners of SB are neither women nor this is the startup business, the given
reduction in tax liability shall not be available to Sweet Baker.

388 | P a g e
CHAPTER 16
INCOME TAX OTHER ASPECTS [Not
Discussed In Earlier Chapters]
2. SECTION WISE INCOME TAX IMPORTANT NOTES
The following issues are covered in this chapter:
1. Diplomatic and United Nations - exemption
2. Foreign government officials - exemption
3. International agreements - exemption
4. Honors by the President
5. Scholarship
6. Support payment under an agreement to live apart
7. Recouped expenditures - section 70
8. Cessation of source of income - section 72
9. Associates - definition
10. Representative of a deceased individual-section 87
11. Exemptions and tax concessions in the Second Schedule-section 53
12. Exemptions and tax provisions in other laws - section 54
13. Limitation of exemption - section 55
14. Definitions [not covered in earlier chapters / paras]
15. Agriculture Produces as raw material
16. Exemption under foreign investment (Promotion and Protection) Act, 2022

Exemption - Diplomatic and United Nations - Section 42


Income of an individual is exempt if he is entitled to privileges under:
- Diplomatic and Consular Privileges Act
- United Nations (Privileges and Immunities) Act
Any pension received by a citizen of Pakistan from United Nations or its specialized
agencies is exempt if his salary from such employment was exempt in Pakistan.

Exemption - Foreign Government Officials - Section 43


Salary of a foreign government official is exempt in respect of services in Pakistan to such government
if:
a. The employee is a citizen of the foreign government and not a citizen of Pakistan
b. Services are similar to those performed by Pakistan government officials in foreign countries;
and
c. That foreign government grants similar exemption to Pakistan government officials

Exemptions under International Agreements - Section 44


Certain incomes are exempt under tax treaties i.e. an agreement for the avoidance of
double taxation. Tax treaties shall apply in case of any contradiction between any tax
treaty and the local laws.
Income of a foreign national is exempt to the extent provided in an Agreement between
Pakistan and a foreign country or a public international organization subject to certain
conditions.

Exemption under Foreign Investment (Promotion and Protection) Act, 2022 (the Act 2022)

389 | P a g e
Section 44A
(a) Taxes on income (including capital gains), advance income tax, minimum and final
taxes under the Income Tax Ordinance, 2001 shall be exempt in respect of qualified
investment (such as Reko Diq project in Balochistan) under the Act 2022.

390 | P a g e
(b) All investors and shareholders of the qualified investment, their associates and
companies specified in the Act 2022 including third party lenders on account of any loan
shall also be exempt from taxes and other provisions of the Income Tax Ordinance.

Honors by the President -Section 45


Any monetary award or allowance attached to any honor, award or medal awarded by the
president of Pakistan is exempt.

Scholarship-Section 47
Any scholarship granted to meet education cost is exempt other than scholarship paid
directly or indirectly by an associate.
Education expense paid by an employer to an employee or his dependents is a taxable perquisite.

Support payment under an agreement to live apart - Section 48


Amount received by a spouse as support payment under an agreement to live apart is exempt.

Recouped expenditure - Section 70


Any amount subsequently received in cash or in kind in respect of any tax expenditure or
tax loss shall be treated as income.

Cessation of source of income - Section 72


If any taxable activity has ceased and subsequently any benefit is derived in cash or in
kind from this activity then it shall be taxable in the normal manner and all the provisions
of the Ordinance shall apply accordingly.

Associates-Section 85
1. Two persons shall be associates where one may reasonably be expected to act in
accordance with the intentions of the other, or both persons may reasonably be
expected to act in accordance with the intentions of a third person.
2. One person sufficiently influences, either alone or together with associate, the
other person i.e. economic or financial dependency
3. One person enters into a transaction, directly or indirectly, with the other who is a
resident of zero tax area.
4. Two persons shall not be associates solely by reason of employment.
5. Following shall be treated as associates:*
a) An individual and his relative
b) Members of AOP
c) A member and his AOP where the individual either alone or together
with his associates controls 50% or more share in profit or loss of AOP
d) A trust and its beneficiaries
e) A shareholder and the company where the shareholder either alone or
together with his associates controls 50% or more voting power or rights
to dividend or capital.
f) Two companies where a person either alone or together with his associates
controls in both the companies 50% or more voting power or rights to dividend
or capital

391 | P a g e
Definition of Relative:
"Relative" in relation to an individual, means
(a) An ancestor, a descendant of any of the grandparents, or an adopted child, of
the individual, or of a spouse of the individual; or
(b) A spouse of the individual or of any person specified in clause (a).

Legal representative of a deceased individual-Section 87


a. The legal representative of a deceased individual shall be liable for any tax that
the individual would have become liable for if the individual had not died and
any tax payable in respect of the income of the deceased's estate.
b. The liability of a legal representative shall be limited to the extent to which the
deceased's estate is capable of meeting the liability which shall be the first
charge on the deceased's estate.
c. Any tax proceeding taken against the deceased before his death shall be treated as
taken against the legal representative and may be continued from the stage at
which the proceeding stood on the date of the deceased's death.
d. Any tax proceeding which could have been taken against the deceased if
the deceased had survived may be taken against the legal representative
of the deceased.
e. "Legal representative" means a person who in law represents the estate of a
deceased person, and includes any person who intermeddles with the estate of
the deceased and where a party sues or is sued in representative character the
person on whom the estate devolves on the death of the party so suing or sued.

Exemptions and tax concessions in 2nd Schedule to the Income Tax Ordinance-
Section 53
1. The income or persons specified in 2nd Schedule shall be -
a. Exempt from tax, subject to specified conditions, if any;
b. Subject to lower rate of tax as are specified therein;
c. Allowed a tax reduction, subject to any conditions and extent specified therein; or
d. Exempt from the operation of any provision of the income tax
ordinance, subject to any conditions and extent specified therein.
2. The Federal Government or FBR with the approval of the Federal Minister-in-
charge may add, omit or change any clause of 2nd Schedule, whenever
circumstances exist to take immediate action for the purposes of:
- National security;
- Natural disaster;
- National food security in emergency situations;
- Protection of national economic interests in situations arising out of
abnormal fluctuation in international commodity prices;
- Implementation of bilateral and multilateral agreements;
- Granting an exemption from any tax imposed including a reduction in tax liability
- An exemption from the operation of any provision of this Ordinance to
any international financial institution or foreign Government owned
financial institution operating under an agreement.
3. All amendments in the 2nd Schedule in a financial year shall be placed before
the National Assembly. If not placed before the National Assembly, the
amendment shall stand rescinded on the expiry of the financial year in which it
was issued.

Exemptions and tax provisions in other laws - Section 54


No provision in any other law providing for-

392 | P a g e
a. An exemption from any tax imposed under this Ordinance;
b. A reduction in the rate of tax imposed under this Ordinance;
c. A reduction in tax liability of any person under this Ordinance; or
d. An exemption from the operation of any provision of
this Ordinance, Shall have legal effect unless also provided
for in this Ordinance.

Limitation of exemption - Section 55


Where any income is exempt from tax, the exemption shall be limited to the original
recipient of that income and shall not extend to any person receiving any payment
wholly or in part out of that income. It means that if income of a company is exempt
then salary income received by the employees and dividend income received by
shareholders would be taxable unless specifically exempt.

Agriculture produces as raw material (Rule-11)


Where a person who is a cultivator or receiver of agricultural produce as rent in kind uses agriculture produce
as raw materials in his business, then the income chargeable to tax under the head income from business shall
be computed as follows:
Total income xxx
Market value of agricultural produce used in
the business as raw material (xxx)
xxxx
Only the market value of agriculture produce is deducted. No further deduction shall be allowed in respect of
any expenditure incurred by a taxpayer as cultivator or as a receiver of rent in kind.
The market value shall be calculated as follows:
(xxiv) where the agricultural produce is ordinarily sold in the market in its raw state or after application
of any process ordinarily employed by a cultivator or receiver of agricultural produce as rent-in-
kind to render it fit to be taken to market, the market price for the produce at the time it is used as
raw materials in the person’s business
(xxv) in any other case, the sum of the following amounts, namely
• the expenses of cultivation; and
• the land revenue rent paid for the area in which the produce is grown.

Exemption under foreign investment (Promotion and Protection) Act, 2022 (Sec 44A)
(xxvi) In December 2022, ‘Foreign Investment (Promotion and Protection) Act, 2022 [‘FIPPA’] was passed
with a view to attract, encourage, and protect, large scale investments in Pakistan. Through section 3
of FIPPA, the Federal Government has been empowered to notify any investment, sector, industry or
project as ‘Qualified Investment’ by listing it in the First Schedule to FIPPA. Presently, the Reko Diq
Project in the Province of Balochistan, including the Reko Diq Mining Company (Private) Limited and
its associated companies, have been listed in the First Schedule to the FIPPA as Qualified Investment.
(xxvii) The FIPPA further elaborately defines the term ‘Investment Incentive’ to include inter alia exemptions
from and reduction/concessions in the rates of any Federal, Provincial or local duties, charges, taxes,
levies, fees and cesses as may be mutually agreed by an investor and a concerned government through
an investment agreement.
(xxviii) The Second and Third Schedules to FIPPA, as passed by the parliament on December 13, 2022,
contained a list of certain amendments in the Income Tax Ordinance, 2001, Sales Tax Act, 1990,

393 | P a g e
Federal Excise Act, 2005 and the Customs Act, 1969 vis-à-vis the Reko Diq Project, Reko Diq
Minining Company (Private Limited) and its associated companies. Whilst as a result of the above
enactment already passed by the Parliament, the respective amendments have already been made and
consequently all concessions and exemptions envisaged in FIPPA have been legally enforceable.
However, in order to avoid procedural hassle relating to implementation of these concessions with
regard to FBR’s system, all income tax concessions are now incorporated in Income Tax Ordinance,
2001 in the following manner:
• Taxes on income (including capital gains), advance tax, withholding taxes, minimum
and final taxes under this Ordinance shall, for the period and to the extent provided in
the Second and Third Schedules to the Foreign Investment (Promotion and Protection)
Act, 2022 in respect of qualified investment as specified at Sr. No.1 of the First Schedule
to the said Act or investors, be exempt or subject to tax at the rate and in the manner
specified under the said Act.
• All investors and shareholders of the qualified investment, their associates and
companies specified in the Second and Third Schedules to the said Act including third
party lenders on account of any loan shall also be exempt from taxes and other
provisions of this Ordinance or subject to tax at the rate and in the manner specified
under the said Act for the period and to the extent provided in the Second and Third
Schedules to the said Act.
• Provisions of this Ordinance relating to Anti-Avoidance, for the period and to the extent
specified in the said Act including sections 106, 106A, 108, 109 and 109A, shall not
apply to the persons and amounts mentioned in (i) and (ii).
• Rates of depreciation, initial allowance and pre-commencement expenditure under
sections 22, 23 and 25 as on the 20th day of March, 2022 shall continue to be applicable
for thirty years as provided in the Third Schedule to the said Act in respect of persons
mentioned in (i) and (ii) above.
• For the purpose of this section, the terms defined under the Second and Third Schedules
to the said Act shall apply mutatis mutandis to this Ordinance.

Definitions [not covered in earlier paragraphs / chapters]


"Income" includes:
- Any amount chargeable to tax;
- Any amount subject to collection or deduction of tax under final tax regime;
- Any amount treated as income; and
- Any loss of income.

"Iris" means a web based computer program for operation and management of Inland
Revenue taxes and laws administered by the FBR.

"Officer of Inland Revenue" means any Additional Commissioner, Deputy Commissioner


Assistant Commissioner, Inland Revenue Officer, Inland Revenue Audit Officer, District
Taxation Officer, Assistant Director Audit or any other officer however designated or
appointed by theBoard for the purposes of this Ordinance.

"Principal Officer" used with reference to a company or AOP includes -


a. A director, a manager, secretary, agent, accountant or any similar officer; and
b. Any person connected with the management or administration of the company or
aop upon whom the commissioner has served a notice of treating him as the
principal officer thereof.

394 | P a g e
"Taxpayer" means any person who derives taxable income, and includes-
a. Any person who is required to deduct or collect tax as withholding agent; or
b. Any representative of a person who derives taxable income;
c. Any person required to furnish a return of income or pay tax under this ordinance.
A. above

395 | P a g e
Practice Questions [Read the Chapter for the Answers of these Theoretical Questions]

Question 1

Define the term 'Taxpayer' under the Income Tax Ordinance.

Question 2
Define the term 'Total Income' under the Income Tax Ordinance.

Question 3
What does the term 'Income' mean under the Income Tax Ordinance?

Question 4
Define the term "Principal Officer" with reference to the Income Tax Ordinance.

3. Question 5
Who is liable to discharge the tax liability of a deceased person, and to what extent?

Question 6
One of your clients, Mr. Nadir who is the legal representative of his deceased uncle Mr.
Ather, has approached you seeking your views with regard to his legal obligations under
section 87 on the following matters:
i. Taxation of income earned by Mr. Ather prior to his death and the extent of tax
liability of Mr. Nadir in respect of such income.
ii. Legality of the tax assessment proceedings pending against Mr. Ather at the time of his death.
Question 7
What are the circumstances in which two or more persons shall be considered as
associates under the Income Tax Ordinance 2001?

Question 8
Identify the situations in which two companies shall be considered to be associates within
the meaning of the Income Tax Ordinance 2001.

4. Question 9
Briefly explain the term "legal representative" with reference to the Income Tax Ordinance, 2001,

396 | P a g e
Question 10
Mr. Zia's father expired in March 2009. Being the only heir, he received all his father's business
and assets. In August 2009, a notice was received from the income tax department in the name
of his father to pay unpaid tax liabilities along with penalty and default surcharge.
Mr. Zia is of the view that since his father expired, the notice is irrelevant
Required:
In the light of Income Tax Ordinance, explain the correct legal position of Mr. Zia
with regard to his father's income tax liabilities and the related income tax
proceedings. (Marks 6)

Question 11
Mr. Hyder is the legal representative of his deceased uncle since 5.1.2010 and manages his
estate worth Rs.10 million approximately.
On 10.8.2010, he received two notices from the Income Tax Department requiring him to:
- Submit details of his uncle's income for the tax year 2009.
- Make payment of Rs.12 million against his uncle's income for the tax year 2007 and 2008.
Required:
Advise Mr. Hyder about the extent of his tax liability in respect of the income earned by
his uncle before 5.1.2010. Also advise him about his obligations relating to the tax
assessment proceedings pending/arising against his uncle.
Question 12
Under the Income Tax Ordinance 2001, where a person is reasonably expected to act in
accordance with the intentions of another person, both persons are considered as
associates.
5. Required:
(1) Explain the term "person" in the above context. (Marks 3)
(2) State the circumstances in which a company and its shareholder shall be considered as associates.
6. (Marks 4)
Question 13
Mr. A is responsible for managing the property of his uncle who died on 5.2.20X3. The
approximate worth of the property is Rs.7 million.
In August 20X3, a notice was received from income tax department in the name of his
uncle requiring details of his income for the tax year 20X2 along with demand for
payment of tax in respect of previous year amounting to Rs.8.5 million.
7. Required:
Advise Mr. A as regards the following:
(a) Extent of A's liability in respect of the income earned by his uncle before 5.2.2013.
(b) His obligations relating to the tax assessment proceedings pending/arising against his uncle.
8. (Marks 5)

397 | P a g e
Question 14
Under the Income Tax Ordinance 2001, explain the term "Associate".
State the circumstances under which a shareholder in a company and the company
may be regarded as associates.

9. Question 15
The Income Tax Department initiating a proceeding against Mobeen, issued a demand
note requiring him to pay the outstanding amount of his tax liability for tax year 20X5
along with default surcharge.
However, before settlement of his tax liability, Mobeen died in a car accident.
10. Required:
Under the provisions of the Income Tax Ordinance, 2001:
(1) Describe whether tax authorities would be able to recover the amount of tax after
Mobeen's death and what would be the extent of such recovery. (Marks 3)
(2) Comment on the status of the proceedings initiated against Mobeen. (Marks 2)

11. Question 16
Under the provisions of the Income Tax Ordinance, 2001 describe the following:
(1) Meaning of the term 'Associates'. (Marks 2)
(2) Circumstances in which a member of an association of persons and the
association may be regarded as associates. (Marks 2)
(3) Situation in which members of an association of persons may not be regarded as associates.
12. (Ma
rks 2)
Question 17
Under the provisions of the Income Tax Ordinance, 2001 compute taxable income or loss
under correct head of income for tax year 20X8, in the following case:
Mrs. Raees separated from her spouse due to certain disagreements. Under an agreement
to live apart, her spouse provided her a house and paid cash of Rs.150,000 per month as
support payment. The fair market rent of the house is Rs.50,000 per month. (Marks 2)

13. Question 18
Haider, included in active taxpayers' list, was carrying on business as a cloth trader. On
28.10.20X7 there was a fire in his shop and the entire stock of clothes costing
Rs.1,550,000 was destroyed. The insurance company refused to pay the claim.
Consequently, Haider ceased his business on 31.1.20X8.
After cessation of business, Haider filed an appeal against the insurance company and was
able to recover Rs.1,300,000 as full and final settlement from the insurance company in
tax year20X9.
14. Required:
Under the Income Tax Ordinance, 2001:
(1) State the requirements that Haider should comply with, on cessation of his
business on 31.1.20X8.(Marks 3)
(2) Briefly discuss the treatment of the recovered amount in the tax year 20X9. (Marks 2)

398 | P a g e
15. Question 19
Abid is the legal representative of his grandfather since his death on 10.7.20X7 and
manages his estate worth Rs.28 million. On 22.1.20X9, he received a notice from the
Income Tax Department requiring him to make payment of Rs.0.8 million against his
grandfather's income for the tax year 20X7. The notice also required him to submit details
of his grandfather's income for the tax year 20X8.
16. Required:
Advise Abid about his obligations relating to the tax assessment proceedings pending /
arising against his grandfather. (Marks 5)

17. Question 20
On 1.7.20X1, Mrs. A separated from her spouse and decided to live apart with her six
years old son. Below are the extracts of clauses from the agreement to live apart:
(1) Mr. A will pay Rs.50,000 in cash every month to his spouse.
(2) Mr. A will continue to pay his son's monthly school fee of Rs.10,000.
(3) Mr. A will transfer the ownership of a shop in her spouse's name. The shop was
already in use by a tenant at a monthly rent of Rs.88,000. Mrs. A will be entitled
to receive the rent from the date of transfer of ownership in her name.
On 1.9.20X1, the ownership of the shop was transferred in her name.
18. Required:
Under the provision of the Income Tax Ordinance, 2001 briefly explain the tax treatment
of the above arrangement in the income tax return of Mrs. A for the tax year 20X2. Also
specify the head of income under which each of the above receipts will be classified.
(Computation is not required). (Marks 4)

399 | P a g e
Answer 1
"Taxpayer" means any person who derives taxable income, and includes -
i. Any representative of a person who derives taxable income;
ii. Any person who is required to deduct or collect tax as withholding agent; or
iii. Any person required to furnish a return of income or pay tax under the Income Tax Ordinance, 2001.

Answer 2
Total income of a person for a tax year shall be the sum of the –
Person's income under all heads of income for the year; and
Person's income exempt from tax under the Income Tax Ordinance

Answer 3
"Income" includes:
- Any amount chargeable to tax;
- Any amount subject to collection or deduction of tax under final tax regime;
- Any amount treated as income; and
- Any loss of income.
Answer 4

"Principal Officer" used with reference to a company or AOP includes -


a) A director, a manager, secretary, agent, accountant or any similar officer; and
b) Any person connected with the management or administration of the company or aop upon whom the
commissioner has served a notice of treating him as the principal officer thereof.

Answer 5
Refer to Answer 13

Answer 6
Refer to Answer 11
Answer 7
1. Two persons shall be associates where one may reasonably be expected to act in accordance with the
intentions of the other, or both persons may reasonably be expected to act in accordance with the
intentions of a third person.
2. One person sufficiently influences, either alone or together with associate, the other person i.e.
economic or financial dependency
3. One person enters into a transaction, directly or indirectly, with the other who is a resident of zero tax
area.
4. Two persons shall not be associates solely by reason of employment.
5. Following shall be treated as associates:
a) An individual and his relative
b) Members of AOP
c) A member and his AOP where the individual either alone or together with his associates
controls 50% or more share in profit or loss of AOP
d) A trust and its beneficiaries
e) A shareholder and role 50% or more re the shareholder either alone or together with his
associates controls 50% or more voting power or rights to dividend or capital.
f) Two companies where a person either alone or together with his associates controls in both
the companies 50% or more voting power or rights to dividend or capital

Answer 8
Two companies shall be considered as associates where a person either alone or together with

400 | P a g e
his associates controls in both the companies 50% or more voting power or rights to dividend
or capital

Answer 9
"Legal representative" means a person who in law represents the estate of a deceased person, and includes any
person who intermeddles with the estate of the deceased and where a party sues or is sued in representative
character the person on whom the estate devolves on the death of the party so suing or sued.

Answer 10
Refer to Answer 15

Answer 11
Mr. Hyder is the legal representative of his deceased uncle and is liable for any tax, which would have been
payable by his uncle, if he had not died. However, such liability is limited to the extent of Rs.10 million i.e.
value of his deceased uncle's estate.
Any proceeding taken against his uncle shall be continued against Mr. Hyder from the stage at which it stood
on the date of his uncle's date. Further, any proceeding which could have been taken against the deceased if he
had survived may be taken against the legal representative.

Answer 12
(i) "Person" the following shall be treated as persons for tax purposes, namely:-
a. An individual;
b. A company or AOP incorporated, formed, organized or established in Pakistan or elsewhere;
c. The Federal Government, a foreign government, a political sub-division of a foreign government, or
public international organization.
(ii) A shareholder and the company shall be treated as associates where the shareholder either alone or
together with his associates controls 50% or more voting power or rights to dividend or capital.

Answer 13
(a) As a legal representative, Mr. A is liable for any tax, which would have been payable by his deceased
uncle. However such liability is limited to the extent of Rs.7 million i.e. value of his deceased uncle's state.
(b) Any proceedings pending against his uncle shall continue against Mr. A from the stage at which it stood
on the date of his uncle's death. Further, any proceedings which could have been initiated against the deceased
if he had survived, may now be initiated against Mr. A.

Answer 14
Two persons shall be associates where one may reasonably be expected to act in accordance with the
intentions of the other, or both persons may reasonably be expected to act in accordance with the intentions of
a third person.
A shareholder and the company shall be treated as associates where the shareholder either alone
or together with his associates controls 50% or more voting power or rights to dividend or
capital

Answer 15
(i) The tax authorities would be able to recover the amount of outstanding liability from the legal
representative of Mobeen.
The legal representative is liable for any tax that Mobeen would have become liable for if he had not died.
However, the liability of the legal representative shall be limited to the extent to which Mobeen's estate is
capable of meeting the liability and such liability shall be the first charge on Mobeen's estate, in preference to
any other outstanding liability of the deceased.
(ii) Any proceeding taken against Mobeen before his death shall be treated as taken against the
legal against him from the stage at which the proceeding stood on the date of Mobeen's death

401 | P a g e
Answer 16
(i) Associate
Two persons are associate where the relationship between the two is such that one may reasonably be
expected to act in accordance with the intentions of the other, or both persons may reasonably be expected to
act in accordance with the intentions of a third person.

(ii) Circumstances in which a member of an association of persons (AOP) and the AOP may be regarded as
associates:
Where the member, either alone or together with an associate or associates under another application of the
Income Tax Ordinance, 2001, controls 50% or more of the rights to income or capital of the AOP.

(iii) Situation in which members of an AOP may not be regarded as associates:


Members of an AOP may not be regarded as associates where the Commissioner is satisfied
that neither person may reasonably be expected to act in accordance with the intentions of the
other

Answer 17

Answer 18.
i. Haider should give a notice in writing to the Commissioner regarding the discontinuation of business
within 15 days of such discontinuation i.e. by 15.2.20X8.
He is also required to furnish the return of income on his own or on being required by the
Commissioner by notice in writing.
The return should cover the period commencing from 1.7.20X7 to 31.1.20X8
ii. If any taxable activity has ceased and subsequently any benefit is derived in cash or in kind from this
activity then it shall be taxable in the normal manner and all the provisions of the Ordinance shall
apply accordingly.
Therefore, the receipt of Rs.1,300,000 shall be included in his income from business for the tax
year 20X9, provided that the write off has been allowed in the previous year

Answer 19
As Mr. Abid is the legal representative of his deceased grandfather, any proceedings pending against Abid's
grandfather shall continue against Abid from the stage at which it stood on the date of his grandfather death.
Further, any proceedings which could have been initiated against the deceased if he had survived, may now be
initiated against Abid.
Abid is also liable for any tax, which would have been payable by his deceased grandfather. However, such
liability is limited to the extent of Rs.28 million i.e. value of his deceased grandfather's estate.

Answer 20
Monthly cash allowance and payment of school fees
Any income / asset received by a spouse as support payment under an agreement to live apart shall be exempt
from income tax.
Therefore, monthly cash amount of Rs.50,000 and school fees of Rs.10,000 paid by Mr. A would be
considered as support payment.
Both amounts will be shown as exempt income under the head 'Income from other sources' in Mrs. A's income
tax return.

Rent received from the property


Since the ownership of the shop was transferred to Mrs. A, the rental income from the shop will not be
considered as support payment. Rental income of Rs.88,000 per month with effect from 1st September will be
taxable under the head 'Income from property in Mrs. A's income tax return.
Fair market value of the shop will be considered as support payment under an agreement to live
apart and would not be taxable

402 | P a g e
CHAPTER: 17 RETURNS & Wealth
Statement
Return of income
Persons required to file Return:
The following persons are required to file return of income:
a) Every company including an approved non-profit organization;
b) A person other than a company whose taxable income exceeds basic exemption.
c) A person whose income for the year is subject to FTR;
d) Who has been charged to tax for any of the two immediately preceding tax years,
e) Who claims a loss carried forward;
f) A person who owns immovable property of 500 square yards or more or a flat in urban areas;
g) A person who owns immovable property of 500 square yards or more in a rating area,
h) A person who owns a flat of 2,000 square feet or more in a rating area;
i) A person who owns a vehicle of above 1,000 cc;
However, the provisions of clauses (f) to (i) are not applicable for: [Not Liable to File the return][D.O.W.N]
➢ Non-resident person;
➢ Widow;
➢ Orphan below 25 years of age; and
➢ Disabled person.
j) A person who has obtained National Tax Number;
k) A person who holds commercial or industrial connection of electricity where
the annual bills exceed Rs. 500,000.
l) A resident person who is registered with any Chamber of Commerce and
Industry or any trade association or market committee or any professional body
including:
➢ Pakistan Engineering Council;
➢ Pakistan Medical and Dental Council;
➢ Bar Council;
➢ Institute of Chartered Accountants of Pakistan; or
➢ Institute of Cost and Management Accountants of Pakistan.
m) Every resident person required to file foreign income and assets statement as
required under section 116A.
(1) Declaration of business bank account
Every taxpayer is required to declare bank account utilized for business transactions
through original or modified registration form prescribed by the FBR. In case of default, a
penalty shall be imposed. [i.e expenses for business purposes shall be in-admissible
in case of not declaration of bank account]
(2) Electronic filing of return of income and other documents
Return of income can be filed electronically on the web and by way of magnetic media or
any computer readable media. In the following cases electronic filing is mandatory:

Documents to be filed electronically


a) Company and AOP Return of income and withholding tax statements.
b) A person registered under the Sales Return of income and withholding tax statements
Tax Act

403 | P a g e
c) An individual having salary income Return of income along with:
• Proof of tax deduction / payment
• Wealth statement and reconciliation

404 | P a g e

Foreign income and assets statement if required
under section 116A.
d) An individual declaring taxable Return of income and withholding tax statements
income of Rs.1 million and above or
turnover exceeding Rs.50 million
e) Federal government departments Withholding tax statements

f) A non-corporate entity claiming Return of income


refund
g) Any person claiming refund Application for refund
Notice by the Commissioner for filing of return
The Commissioner may give a notice to file return of income within 30 days or any longer
or shorter time from the date of notice to any person who was required to file return in the
opinion of the Commissioner but has failed to do so.
Time limit for issuance of notice by the Commissioner
Such notice may be issued only in respect of any of the last 5 tax years. However, where a,
person has not filed return for any of the last 5 completed tax years, such notice may be
issued in respect of one or more of the last 10 completed tax years.
The above time limit shall not apply if the Commissioner is satisfied that a person who
failed to file his return has foreign source income or owns foreign assets.
Q. On October 17, 2022 Commissioner issued notice to Mr. Bajwa to file his return of income for the
TY 2017. Is the commissioner justified in issuing the notice?

A. Commissioner is not justified in issuing the notice as the above mentioned notice may be issued in
respect of any of the last 5 completed tax years. In the given question, commissioner may issue a notice
for tax year 2018, 2019, 2020, 2021 or 2022.

Power to enforce filling of return


1. FBR shall have powers to issue income tax general order in respect of
persons who are not appearing on ATL but are liable to file return of
income.
2. The income tax general order issued may entail any or all of the following
consequences for the persons mentioned therein, namely:-
a) Disabling of mobile phones or mobile phone SIMS;
b) Discontinuance of electricity connection; and
c) Discontinuance of gas connection.
3. FBR or the Commissioner may order restoration of mobile phones, mobile
phone SIMS and connections of electricity and gas, in cases where he is
satisfied that -
(a) The return has been filed; or
(b) Person was not liable to file return of income
4. No person shall be included in the general order unless following conditions have been met with:
a) Notice to file return of income has been issued;
b) Date of compliance of the notice has elapsed; and
c) The person has not filed the return.
Notice for the return of income for a period less than 12 months
The Commissioner may give notice to any person or his representative to file return of
income for a period less than 12 months where the person
• has died,
• become bankrupt or
• gone into liquidation or

405 | P a g e
• about to leave Pakistan permanently.
Notice of discontinued business
Any person discontinuing his business shall give a notice in writing to the Commissioner
to this effect within 15 days of the discontinuation along with return of income up to the
date of discontinuance.
If such notice is not given by the person and the Commissioner has reason to believe that a
business has been discontinued then he may give a notice to that person to file return of
income.
In view of the difficulty in filing of return within this short period of 15 days, the requirement of
filing of return has been dispensed with. A notice within this period shall continue to be
obligatory.
Due date of filing of return and extension therein
Due date for filing:
Following are the different dates for tax reporting purposes

Section Return Filer Return Period Due Date

118(1) A return of income of a Tax year ending On or before 31 December next


company between 1st January to 30 following the end of tax year
June
118(1) All other cases of person All year ends On or before 30 September next
&(3) filing returns following the end of tax year
118(5) Return in response to notice Year end as specified Due date fixed for
under section 117
(discontinuance of in notice submission of tax return.
business)
114(4) Return in response to notice Year end as specified Due date specified in the
under section 114(5) (return
liable to be filed but not in notice notice for submission of
filed) tax return or thirty days from the
date of issuance of notice.

Extension of time for filing of return


For the purpose of extension of time for filing of return of income, the taxpayer is required
to file an application to the Commissioner and the Commissioner may grant an extension
of time up to 15 days if he is satisfied that the applicant is unable to furnish the return by
the due date because of -
a) Absence from Pakistan;
b) Sickness or other misadventure; or
c) Any other reasonable cause.
Under exceptional circumstances a longer time extension may also be granted by the Commissioner.
Refusal of extension by the Commissioner
If a commissioner refuses to extend the time for filing of return of income then the taxpayer
may file an application to the Chief Commissioner who may allow such extension of time
for 15 days and a longer time under exceptional circumstances.

406 | P a g e
Return not filed within due date
Where a person fails to file a return of income by the due date or within time extended by
the Commissioner or FBR, such person shall not be:
• Included in the of active taxpayers list (ATL);
• Allowed to carry forward any loss for that tax year;
• Issued refund during the period the person is not included in ATL; and
• Entitled to additional payment for delayed refund during the period the person is
not included in ATL.
However, the person shall be included in ATL on filing return of income after the due date
if the person pays surcharge, in addition to any other liability, as under:
• Rs. 20,000 in case of a company
• Rs. 10,000 in case of an AOP
• Rs. 1,000 in case of an individual.
Revised return of income
If a person discovers any omission or wrong statement in the return of income, he may
furnish a revised return subject to the following conditions:
a) It is accompanied by the revised accounts or revised audited accounts; (The Commissioner may waive this
condition if filing of revised accounts or revised audited accounts is not necessary in his opinion.)
b) The reason of revision of return duly signed is filed therewith;
c) It is accompanied by the Commissioner's approval in writing for revision of return.
This condition is not applicable in the following cases [Means No approval required for
revision]
• If the revised return is filed within 60 days of filing of return; or
• Where the revised taxable income is more than the taxable income previously
declared or the revised loss is less than the loss previously declared; or
• Where the Commissioner does not give his approval / rejection within 60
days of filing of application for revision of return; and
d) Taxable income is not less than the income, or loss is not more than the loss, determined
by the Commissioner in an order (best judgment, amendment or rectification of
assessment) or by any appellate authority (decision by Commissioner (Appeals), Appellate
Tribunal, High Court).
The Commissioner shall grant approval in case of a bonafide omission or wrong statement.

Reduction of penalty on revision of return of income


The filing of revised income tax return has been liberalized and an option has been made
available to a taxpayer to file revised return voluntarily, at any stage of the proceedings,
along with the payment of tax involved and default surcharge.
However, penalties shall be reduced as under:
1. A taxpayer can file revised return along with payment of short paid tax and default
surcharge. If such voluntary revision is made before receipt of notice for audit or
amendment of assessment, no penalty shall be imposed.
2. In case a taxpayer revises his return voluntarily and pays tax demand during the
process of audit (before issuance of notice of amendment) along with the default
surcharge, only 25% of the penalty is required to be paid in this case.
3. In case of such voluntary revision of return after issuance of notice of amendment,
along with tax sought to be evaded and the default surcharge, only 50% of the
penalty is required to be paid where he files a revised return at this point.

407 | P a g e
Wealth statement [Sec 116]
A. Wealth statement is a statement of assets and liabilities, for any year, which a person is required to
file, in the prescribed form and verified in prescribed manner giving particulars of
(xxix) the person’s total assets and liabilities as on the date or dates specified in such notice;
(xxx) the total assets and liabilities of the person’s spouse, minor children, and other dependents as
on the date or dates specified in such notice;
(xxxi) any assets transferred by the person to any other person during the period or periods specified
in such notice and the consideration for the transfer; and
(xxxii) the total expenditures incurred by the person, and the person’s spouse, minor children, and
other dependents during the period or periods specified in the notice and the details of such
expenditures; and
(xxxiii) the reconciliation statement of wealth.
B. Every resident taxpayer being an individual, filing a return of income for any tax year shall furnish a
wealth statement and wealth reconciliation statement for that year along with such return.
C. Further, every member of an association of persons shall also furnish wealth statement and wealth
reconciliation statement for the year along with return of income of the association.
D. Where a person, who has furnished a wealth statement, discovers any omission or wrong statement
therein, he may, without prejudice to any liability incurred by him under any provision of this
ordinance, furnish a revised wealth statement along with the revised wealth reconciliation and the
reasons for filing revised wealth statement, under intimation to the commissioner in the prescribed
form and manner, at any time before the receipt of notice for amendment in assessment, for the tax
year to which it relates, is made;
E. Provided that where the Commissioner is of the opinion that the revision under this sub section is
not for the purpose of correcting a bona fide omission or wrong statement, he may declare such
revision as void through an order in writing after providing opportunity of being heard.
Explanation: For the removal of doubt it is clarified that wealth statement cannot be revised after the expiry
of five years from the due date of filing of return of income for that tax year.

Foreign Income and Assets Statement [Sec 116A]


(i) Every resident taxpayer being an individual having foreign income of not less than ten thousand
United States dollars or having foreign assets with a value of not less than one hundred thousand
United States dollars shall furnish a statement, hereinafter referred to as the foreign income and assets
statement, in the prescribed form and verified in the prescribed manner giving particulars of
• the person’s total foreign assets and liabilities as on the last day of the tax year;
• any foreign assets transferred by the person to any other person during the tax year and
the consideration for the said transfer; and
• Complete particulars of foreign income, the expenditure derived during the tax year and
the expenditure wholly and necessarily for the purposes of deriving the said income.
(ii) The Commissioner may by a notice in writing require any person being an individual who, in the
opinion of the Commissioner on the basis of reasons to be recorded in writing, was required to furnish
a foreign income and assets statement but who has failed to do so to furnish the foreign income and
assets statement on the date specified in the notice.”;

How to prepare a wealth statement


(i) Preparation of wealth statement is a technical task but familiarity with the method of preparation of
financial statements helps a lot in solving such problems. The manner of preparation of wealth
statements is given hereunder:

408 | P a g e
(ii) Wealth statement is primarily a balance sheet of an individual taxpayer where personal assets and
liabilities of a person are reflected on any given date i.e. the date on which the taxpayer closes his
accounts or the date as demanded through a notice in writing. It is worthwhile to mention here that
wealth statement only gives details of personal assets and liabilities held by a person but does not
reflect the status of business assets and liabilities. However, it does show net equity or shareholding
of that person in any business. A wealth statement is incomplete without reconciliation statement
showing accretion, no change or decrease in wealth.
(iii) Cash and bank reconciliation statement is derived from the cash and bank account. It starts from
taking opening balance of cash and bank balances and after adding cash inflows and subtracting cash
outflows, the remaining amount portrays closing balances of cash and bank account(s). This closing
balance is included in the assets of the wealth statement. It is important to bear in mind that where
expenditure side is not explained through the cash receipt side, then the difference could be un-
explained investment as income for the year. To avoid this treatment, figures need to be tallied with
investments worked out through wealth reconciliation statement.
(iv) Now after taking the figure of the cash and bank reconciliation, wealth statement for the current year
is complete and a person can easily calculate the figure of increase/decrease in the net wealth by
subtracting the last year’s net wealth figure from the current year’s net wealth figure shown in the
current year wealth statement.
(v) While preparing wealth statement assets and liabilities are recorded at historical cost irrespective of
their present market value.
Exercise:
Mr. Nadeem has filled following wealth statement as on 30.06.2023
Rupees
Plot at DHA, Lahore 3,500,000
Capital in ABC & Co 2,500,000
Jewelry 500,000
Shares in XYZ (Pvt.) Ltd 1,000,000
Bank 3,500,000
TOTAL 11,000,000
Personal Loan 1,000,000
TOTAL 10,000,000
During the year following information is provided:
He earned salary income of Rs. 1,300,000 and paid tax Rs.100,000. He sold
share of Rs. 200,000 for a consideration of Rs. 350,000.
He settled his personal loan of Rs. 500,000.
His household expenses aggregates to Rs. 850,000.
He has given gift of Rs. 400,000 to his brother Kamran through crossed cheque.
He has earned profit on ABC & Co of Rs.450,000. His drawings from the firm during the year was Rs.
275,000. He paid tax of Rs. 40,000on firm income.
He purchased a new plot at EME society for total consideration Rs. 2,000,000 payable in 20
installments. During the year he paid Rs. 700,000 in installments.
On 30th June 2024, his bank balance was Rs. 475,000.
Required:
Prepare the wealth reconciliation statement and wealth statement as on June 30, 2024.

Answer
MR. NADEEM

409 | P a g e
Wealth reconciliation statement
Amount
Opening Wealth 10,000,000
Add: Sources
Salary Income 1,300,000
Gain on sale of shares 150,000
Profit on ABC & CO 450,000
Total 11,900,000
Less:
Gift to brother 400,000
Tax deducted from salary 100,000
Tax on profit of ABC & Co. 40,000
Household expenses 850,000
1,390,000
Total (Rs. 11,900,000–1,390,000) 10,510,000
Note 2
Cash and Bank Reconciliation
Opening bank 3,500,000
Add:
Salary 1,300,000
Drawings 275,000
Sale of shares 350,000
1,925,000
Total 5,425,000
Less:
House hold expenses 850,000
Taxes 140,000
Gift 400,000
Plot instalments 700,000
Loan instalment 500,000 2,590,000
2,835,000
Wealth statement 2024
Plot at DHA 3,500,000
Capital in ABC (Note 1) 2,675,000
Advance for plot at EME 700,000
Jewellery 500,000
Shares in XYZ 800,000
Bank 2,835,000
Total 11,010,000
Less:
Loan (500,000)
Closing wealth 10,510,000
Note 1: Capital
Opening 2,500,000
Profit 450,000
2,950,000
Drawings 275,000
Total 2,675,000

410 | P a g e
Chapter: 18 Assessments
Definition of Assessment
"Assessment" includes provisional assessment, re-assessment and amended assessment.

Assessment:
A return of income (other than a revised return) filed by a taxpayer shall be considered as an assessment of
taxable income.

However, if a return of income fled is not complete, the taxpayer shall be required by the Commissioner to
complete the return of income.
A return of income shall be treated as a complete return if the following conditions are fulfilled:
• In the prescribed form
• Duly signed
• State the prescribed information together with a declaration of the records kept
• In the context of business, copy of accounts and any other prescribed documents are furnished
• Accompanied with evidence of payment of due tax as per return of income
• Accompanied with a wealth statement and wealth reconciliation statement; and
• Accompanied with foreign income and assets statement as required under section 116A.
In case of any deficiency, the Commissioner shall issue a notice within 180 days from the end of the financial
year in which the return was furnished to complete the return by the date mentioned in the notice. In the case
of non-compliance of the notice, return filed be treated as invalid and the taxpayer shall be treated as not
having furnished a return of income by the due date.
Adjustments to be made in declared respective amounts of the return [Section 120(2A)]
A return of income furnished under sub-section (2) of section 114 shall be processed through automated system
to arrive at correct amounts of total income, taxable income and tax payable by making adjustments for—
• any arithmetical error in the return;
• any incorrect claim, if such incorrect claim is apparent from any information in the return;
• disallowance of any loss, deductible allowance or tax credit as specified; and
• disallowance of carry forward of any loss under clause (b) of sub-section (1) of section 182A.
Provided that no such adjustments shall be made unless a system generated notice is given to the taxpayer
specifying the adjustments intended to be made:
Provided further that the response received from the taxpayer, if any, shall be considered before making any
adjustment, and in a case where no response is received within 30 days of the issue of such notice, adjustments
shall be made.
Provided also that where no such adjustments have been made within 6 month of filing of return, the amounts
specified in the return as declared by the taxpayer shall be deemed to have been taken as adjusted amounts on
the day the return was filed and the taxpayer shall be intimated automatically through IRIS.

Provided also that the provisions of this sub-section shall apply from the date notified by the Federal Board of
Revenue in the official Gazette.

Note: FBR has thus far not notified any date. Resultantly, the concept of self- assessment is still applicable.

For the purposes of this section,—


a. “arithmetical error” includes any wrong or incorrect calculation of tax payable including any minimum or
final tax payable.
b. "an incorrect claim apparent from any information in the return" shall mean a claim, on the basis of an entry, in
the return,—
i. of an item, which is inconsistent with another entry of the same or some other item in such return;

411 | P a g e
ii. regarding any tax payment which is not verified from the collection system; or
iii. in respect of a deduction, where such deduction exceeds specified statutory limit which may have been
expressed as monetary amount or percentage or ratio or fraction.

However, in addition to above deemed assessment, the Commissioner has powers to conduct audit of
income tax affairs of any person under section 177 and all the provisions of that section shall apply
accordingly.

Audit by the Commissioner: Section 177


The Conduction of Audit:
✓ The Commissioner may call for any documents including books of account of any of the previous 6
years for conducting audit of any person.
✓ The Commissioner or any other authorized officer is allowed access to electronic data and use of
machine and software on which such data is kept.
✓ Such machines and duly attested hard copies of data may be taken into possession for the purpose of
investigation.
✓ The Commissioner shall conduct an audit (including examination of accounts and records. Enquiry
into revenues, expenditures, assets and liabilities) and may call for other appropriate information.
✓ After obtaining taxpayer's explanation on all the issues raised in the audit report, the Commissioner
may amend the assessment.
✓ The Commissioner may conduct audit electronically through video links.
✓ Where a taxpayer has not furnished the required documents or sufficient explanation, the
Commissioner shall determine taxable income on the basis of sectoral benchmark ratios prescribed by
the FBR.
✓ "Sectoral benchmark ratios" means standard business sector ratios notified by the FBR on the basis of
comparative cases and includes financial ratios, production ratios, gross profit ratio, net profit ratio,
recovery ratio, wastage ratio and such other prescribed ratios.

The FBR may appoint a firm of Chartered Accountants or a firm of Cost and Management Accountants to
conduct an audit of any person selected for audit and the scope of such audit shall be as determined by the FBR
on a case to case basis.

Audit by the Commissioner subject to time limit:


A person may be selected for audit in a year by the Commissioner even if audit has been done in his case in,
any previous year. The Commissioner is not required to get permission of the FBR for the selection of a case
for audit.
However, the Commissioner shall take approval of FBR for audit of a taxpayer if income tax audit of the
taxpayer has been done in any of the preceding four tax years.

Special audit panel - Section 177


➢ The FBR may appoint special audit panels, to conduct audit including forensic audit of the income tax
affairs, comprising two or more members from the following:
- Any officer of Inland Revenue;
- A firm of chartered accountants;
- A firm of cost and management accountants; or
- Any other person including a foreign expert or specialist as directed by the FBR.
➢ Chairman of the panel shall be an officer of Inland Revenue. If any one member of the panel, other
than the chairman, is absent from conducting audit, the proceedings of audit may continue and the
audit shall not be invalid or be called in question merely on the ground of such absence.
➢ The scope of such audit by the panel shall be as determined by the Commissioner or FBR on case-to-
case basis.
➢ In case of a member other than officer of Inland Revenue, an agreement of confidentiality shall be
made between FBR and the person or international tax authority.

412 | P a g e
Best judgement assessment (also called ex-parte assessment): Section 121
The Commissioner is empowered to make an assessment of income and tax based on any available
information and to the best of his judgement in the case of failure by the taxpayer to:
o Furnish return of income in response to a notice issued by a commissioner
o File a wealth statement
o Produce accounts, documents, and records required to be maintained under section 174 or any other
documents that the Commissioner or special audit panel may have required.
This assessment order shall indicate taxable income, tax due and paid, if any, and the time, manner and place
of appealing the assessment order.
Best judgement assessment can be issued within 6 years after the end of tax year to which it relates. However,
where a person has not filed any return during the last 5 years then the Commissioner may issue notice to file
return of income for any of the last 10 years and in this case best judgment, if required, may be passed within 2
years from the end of the tax year in which such notice is issued.

Amendment of assessment: Section 122 (also termed as re-opening of assessment or additional


assessment)
The Commissioner has power to amend an assessment by making such alterations as considered necessary to
ensure that the taxpayer is liable for the correct amount of tax.

An assessment can only be amended within 5 years from the end of the financial year in which the
Commissioner has issued or treated to have issued the original assessment order.

The Commissioner has power to make further amendments, as many times as may be necessary, within the
later of:
a) 5 years from the end of the financial year in which the Commissioner has issued or treated to have
issued the original assessment; and
b) 1 year from the end of the financial year in which the amendment was made.
Before making an amendment or further amendment an opportunity of being heard shall be provided to the
taxpayer.

The Commissioner shall pass amended or further amended order within 180 days of the issuance of show
cause notice (may be extended by maximum 90 days by the Commissioner for the reasons to be recorded in
writing). Time shall be excluded from this limit on account of:
- Adjournment of hearing on the request of the taxpayer not exceeding 60 days,
- Adjournment on account of stay order;
- Adjournment on account of alternative dispute resolution committee proceedings.
- Adjournment on account of agreed assessment proceedings.

An assessment can be amended or further amended only where:


i. The Commissioner is of the view that the assessment order is prejudicial to the interest of revenue or
the income tax ordinance has incorrectly been applied in the original assessment including:
▪ Misclassification of income under a head of income
▪ Incorrect payment of tax with the return of income
▪ Incorrect claim for any tax relief, exemption or refund
ii. The Commissioner has definite information that income declared is incorrect including concealment
of income or furnishing of inaccurate particulars.
Definite information includes information in respect of:
➢ Taxable sales, purchases, services and other receipts
➢ Acquisition of any asset or investment made
➢ Expenditure incurred

413 | P a g e
Agreed assessment: Section 122D
1. Where a taxpayer, in response to a notice for amendment of assessment, intends to settle his case, he
may file offer of settlement before the assessment oversight committee (Committee), in addition to
filing reply to the Commissioner.
2. Committee may call for the record of the case and after affording opportunity of being heard to the
taxpayer, may decide to accept or modify the offer of the taxpayer through consensus and
communicate its decision to the taxpayer.
3. Where the taxpayer is satisfied with the decision of Committee,
a) The taxpayer shall deposit tax, penalty and default surcharge as per decision of Committee;
b) The Commissioner shall amend assessment in accordance with the decision of Committee;
c) The taxpayer shall not file an appeal against such amended assessment.
4. Where Committee has not been able to arrive at a Consensus or where the taxpayer is not satisfied
with the decision of Committee, the case shall be referred back to the Commissioner for decision on
the basis of reply of the taxpayer in response to notice for amendment of assessment.
5. Committee shall consist the Chief Commissioner, Commissioner and Additional Commission having
jurisdiction over the taxpayer.
6. Agreed assessment shall not be made in cases involving concealment of income or where
interpretation of question of law is involved having effect on other cases

Revision by the Commissioner: Section 122A


• The Commissioner is empowered to call for the record of any proceedings in which an order was
passed by any authority sub-ordinate to him for making an enquiry on his own motion or on an
application made by the taxpayer for revision.
• The Commissioner may revise any order passed by any Officer of Inland Revenue if the assessment is
not under any appeal.
• The said revision shall not be prejudicial to the person to whom the order relates. It means that
Commissioner does not have power of enhancement of income under this section.
• If any order is remanded back to any sub-ordinate by the Commissioner for modification, the revised
order shall be issued within 120 days.

Revision by the Chief Commissioner: Section 122B


Commissioner is empowered to issue a certificate in respect of exemption from withholding tax or a lower rate
certificate subject to certain conditions.
If there is any dispute or hardship faced by a taxpayer then the Chief Commissioner may review the said matter
at his own discretion or on an application made by the taxpayer.

Provisional assessment: Section 123


➢ Where a concealed asset of a person is impounded by any government agency which, in the opinion of
the Commissioner, was acquired from any taxable income, the Commissioner has power to pass a
provisional assessment order before making a final assessment.
➢ Where an offshore asset, not declared earlier, is discovered by the Commissioner or any government
department / agency then the Commissioner may issue a provisional assessment order before making a
final assessment.
➢ The Commissioner shall finalize the provisional assessment as soon as practicable.
➢ "Concealed asset" means any property or asset which, in the opinion of the Commissioner, was
acquired from any taxable income.
concealment of income includes;
(xxxiv) the suppression of any item of receipt liable to tax in whole or in part, or failure to disclose
income chargeable to tax;
(xxxv) claiming any deduction or any expenditure not actually incurred;
(xxxvi) any act referred to in sub-section (1) of section 111; and
(xxxvii) claiming of any income or receipt as exempt which is otherwise taxable.

414 | P a g e
Explanation - For removal of doubt it is clarified that none of the aforementioned acts would constitute
concealment of income unless it is proved that taxpayer has knowingly and willfully committed these acts;

415 | P a g e
Assessment giving effect to an order by the appellate authority:
a) If an assessment or amendment is to be made on the instructions of any appellate authority then the
Commissioner shall issue the order within 2 years from the end of the financial year in which the
Commissioner is served with the appellate order.
b) Where direct relief is provided in an appellate order, the Commissioner shall issue appeal effect order
within 2 months of the date the Commissioner is served with the appellate order.

Powers of tax authorities to modify orders: Section 124A


Where a question of law has been decided by the Appellate Tribunal or High Court in the case of a taxpayer,
the Commissioner may follow the said decision including in respect of an identical situation in the Subsequent
years of the said taxpayer even if the tax department has filed an appeal against such decision.
If the higher appellate authority reverses such decision of the Appellate Tribunal or High Court, the
Commissioner is empowered to modify the assessment within 1 year from the date of receipt of the decision of
the higher appellate authority and in this case normal time limit for amendment shall not apply.

Assessment in relation to disputed property: Section 125


If the ownership of any property is in dispute in any civil court, income from such property shall be assessed
within one year from the end of the financial year in which the decision of such court is made

Evidence of assessment: Section 126


Production of assessment or its certified copy shall be conclusive evidence of assessment and that the amount
and all particulars of the assessment are correct.
An assessment, notice or other document shall not be quashed / void for want of form or shall not be affected
by reason of mistake or defect if it is, in substance and effect, in conformity with the Ordinance.
However, an order shall be quashed or void:
• if it is in substance and effect, not in conformity with Income Tax Ordinance, 2001; or
• the person assessed or intended to be assessed or effected by the document is not designated in it according to
common understanding
National Database and Registration Authority (Sec 175B)
• The National Database and Registration Authority shall, on its own motion or upon application by the
Board, share its records and any information available or held by it, with the Board, for broadening of
the tax base.
• The National Database and Registration Authority may identify in relation to any person, (whether a
taxpayer or not) income, receipts, assets, properties, liabilities, expenditures, or transactions that have
escaped assessment or are under-assessed or have been assessed at a low rate, or have been subjected
to excessive relief or refund or have been mis declared or misclassified under a particular head of
income.
• The Board may use and utilize any information communicated to it by the National Database and
Registration Authority and forward such information to an income tax authority having jurisdiction.
• The National Database and Registration Authority may compute indicative income and tax liability of
anyone mentioned above by use of artificial intelligence, mathematical or statistical modelling or any
other modern device or calculation method.
• The indicative income and tax liability computed by the National Database and Registration Authority
shall be notified by the Board to the person in respect of whom such indicative income and tax liability
has been determined, who shall have the option to pay the determined amount on such terms,
conditions, instalments, discounts, reprieves pertaining to penalty and default surcharge, and time
limits that may be prescribed by the Board.
• In case the person against whom a liability has been determined, does not pay such liability within the
time prescribed, the Board shall take action under this Ordinance, upon the basis of tax liability
computed above.

416 | P a g e
• If the person against whom the liability has been determined pays such liability, such payment shall
be construed to be an amended assessment order

Records -
• Every taxpayer shall maintain in Pakistan 'such accounts, documents and records as may be prescribed.
• The Commissioner may disallow or reduce a taxpayer's claim for a deduction if the taxpayers unable,
without reasonable cause, to provide a receipt or other evidence of the transaction.
• The prescribed records shall be maintained for 6 years after the end of the tax year to which they
relate.
• However, where any proceeding is pending before any authority or court the taxpayer shall maintain
the record till final decision of the proceedings.
• The above time limit is not applicable for any record related to asset; income or expenditure outside
Pakistan.
• The Commissioner may require any person to install and use a prescribed Electronic Tax Register for
the purpose of storing and accessing information regarding any transaction related to tax liability.
• Records to be kept at the specified place
• The records required to be maintained by a taxpayer shall be kept at the business place or, where the
business is carried on in more places than one, at the principal place of business or at each of such
places if separate books of account are maintained in respect of each place.
• Where a person derives income from sources other than from business, the records shall be kept at the
person's place of residence or such other place as may be so declared by such person.
• The place where the records are kept shall be clearly stated on the tax return form.

Books of accounts, documents and records to be kept at specified place (Rule 33)

1 Income from business The books of accounts, documents and records required to be
maintained by a taxpayer shall be kept at the place where the
taxpayer is carrying on the business or, where the business is
carried on in more places than one, at the principal place of
business or at each of such places if separate books of accounts
are maintained in respect of each place.
2 Income from sources Where a person derives income from sources other than from
other than business business, the books of accounts, documents and records shall be
kept at the person’s place of residence or such other place as
may be so declared by such person.

3 Place to be clearly The place or places where the books of accounts, documents
stated on tax returns and records are kept shall be clearly stated on the tax return form
in the column requiring the details of the records maintained.

Prescribed books of accounts

S.No Taxpayer required to maintain


proper books of account, Records to be kept
documents and records

1 Every taxpayer deriving income money received and expended by the tax payer
from business all sales and purchases of goods
all services provided and obtained by the taxpayer.
all assets of the taxpayer
all liabilities of the taxpayer; and

417 | P a g e
For Manufacturer all items of cost relating to the utilization of
materials, labor and other inputs.
the books of account, documents and records.
2 Every taxpayer other than
companies, deriving income
chargeable under the head
Income from business
3 Taxpayers with business income
• Serially numbered and dated cash-memo / invoice/ receipt
upto Rs.500,000 and new
for each transaction of sale or receipt containing the
taxpayers deriving income from
following
business
(a) taxpayer’s name or the name of his business, address,
national tax number or CNIC and sales tax registration
number, if any
(b) the description, quantity and value of goods sold or
services rendered;
• Where each transaction does not exceed Rs. 100, one or
more cash-memos per day for all such transactions may be
maintained
• Daily record of receipts, sales, payments, purchases and
expenses a single entry in respect of daily receipts,
• Vouchers of purchases and expenses

4 Taxpayers with business income Serially numbered and dated cash-memo / invoice/ receipt for each
exceeding Rs. 500,000 and transaction of sale or receipt containing the following
wholesalers, distributors, dealers (a) taxpayer’s name or the name of his business, address, national
and commission agents tax number or CNIC and sales tax registration number, if any
(b) the description, quantity and value of goods sold or services
rendered; and
(c) in case of a wholesaler, distributor, dealer and commission
agent, where a single transaction exceeds Rs. 10,000, the name and
address of the customer Provided that where each transaction does
not exceed Rs.100, one or more cash-memos per day for all such
transactions may be maintained
Cash book and/or bank book or daily record of receipts, sales,
payments, purchases and expenses; a single entry in respect of daily
receipts, sales, purchases and different heads of expenses will
suffice.
• General ledger or annual summary of receipts, sales, payments,
purchases and expenses under distinctive heads.
• Vouchers of purchases and expenses and where a single
transaction exceeds Rs. 10,000 with the name and address of the
payee; and
• Where the taxpayer deals in purchase and sale of goods, quarterly
inventory of stock-in-trade showing description, quantity and value

5 Professionals like medical • Serially numbered and dated patient-slip / invoice/receipt for each
practitioners, legal practitioners, transaction of sale or receipt containing the following
accountants, auditors, architects, (a) taxpayer’s name or the name of his business or profession,
engineers etc. address, national tax number or CNIC and sales tax registration
number, if any
(b) the description, quantity and value of medicines supplied or
details of treatment/ case/ services rendered (confidential details are

418 | P a g e
not required) and amount charged
(c) the name and address of the patient / client Provided that the
condition of recording address of the patient on the patient slip
under this clause shall not apply to general medical practitioners
• Daily appointment and engagement diary in respect of clients and
patients provided that this clause shall not apply to general medical
practitioners
• Daily record of receipts, sales, payments ,purchases and expenses;
a single entry in respect of daily receipts, sales, purchases and
different heads of expenses will suffice
• Vouchers of purchases and expenses

6 Manufacturers (with turnover Serially numbered and dated cash-memo / invoice /receipt for each
exceeding Rs. 2.5 million): transaction of sale or receipt containing the following
[30(4)] (a) taxpayer’s name or the name of his business address, national
tax number or CNIC and sales tax registration number, if any
(b) the description, quantity and, value of goods sold
(c) where a single transaction exceeds Rs. 10,000 with the name
and address of the customer
• Cash book and/or bank book
• Sales day book and sales ledger (where applicable)
• Purchases day book and purchase ledger (where applicable)
• General ledger
• Vouchers of purchases and expenses and where a single
transaction exceeds Rs. 10,000 with the name and address of the
payee;
• Stock register of stock-in-trade (major raw materials and finished
goods) supported by gate in-ward and outward records and
quarterly inventory of all items of stock-in-trade including work-in-
process showing description, quantity and value.
7 Every taxpayer deriving income Salary
chargeable under the head Salary certificate indicating the amount of salary and tax deducted
income from salary, property, there from.
capital gains or other sources
Income from property
Tenancy agreement, if executed
Tenancy termination agreement, if executed
Receipt for amount of rent received
Evidence of deductions claimed in respect of premium paid to
insure the building, local rate, tax, charge or cess, ground rent,
profit/interest or share
in rent on money borrowed, expenditure on collecting the rent,
legal services and unpaid rent.

Capital gain
Evidence of cost of acquiring the capital asset
Evidence of deduction for any other costs claimed
Evidence in respect of consideration received on disposal of the
capital asset.

Income from other sources

Dividends (Dividend warrants)

419 | P a g e
Royalty (Royalty agreement)

Profit on debt
• Evidence and detail of profit yielding debt
• Evidence of profit on debt and tax deducted thereon, like
certificate in the prescribed form or bank account statement; and
• Evidence of Zakat deducted, if any.

Ground rent, rent from the sub-lease of land or building, income


from the lease of any building together with plant or machinery and
consideration for vacating the
possession of a building or part thereof
(a) Lease agreement
(b) Lease termination agreement.

Annuity or Pension
• Evidence of amount received. Prize money on bond, winning
from a raffle, lottery or cross word puzzle
• Evidence of income and tax deducted thereon, like certificate in
the prescribed form. Provision, use or exploitation of property
• Agreement. Loan, advance, deposit or gift
• Evidence of mode of receipt of a loan, advance, deposit or gift
i.e., by a crossed cheque or through a banking channel.

General
• Evidence of deduction for any other expenditure claimed.

420 | P a g e
Chapter: 19 Appeals
Circumstances giving rise to appeal to the Commissioner (Appeal)
Circumstances giving rise to an appeal may include:
➢ A best judgment assessment (ex-parte assessment) based on any available information or material to
the best of the Taxation Officer’s / Commissioner’s judgment.
➢ An amendment of assessment
➢ An order holding an individual personally liable to pay the amount of tax, which was required to be
collected or deducted by him/her or having collected or deducted fails to pay the same as required by
the law
➢ An order declaring or treating a person as a representative of a non-resident person
➢ An order refusing to rectify the mistake, either in full or in part
➢ An order having the effect of enhancing the assessment or reducing a refund or otherwise increasing
the tax liability
No appeal in above cases, shall be made by a taxpayer against an order of assessment unless the taxpayer has
paid the amount of tax due under subsection (1) of section 137 (tax due at time of furnishing the return).

Forums of appeals
Following forums of appeal are available to an aggrieved person:
(i) Commissioner (Appeals)
(ii) Appellate Tribunal
(iii) High Court
(iv) Supreme Court of Pakistan (against decision of reference to High Court). (Although the Ordinance
does not provide anything on the subject, but any person can prefer to SC under the Constitution); and
(v) Alternative Dispute Resolution

Types of decision in appeals


The decisions by any appellate authority against additions made by the assessing authorities may
be as under:
• Confirm (i.e. additions made by the assessing officer may be confirmed
by an appellate authority and therefore the taxpayer may file an appeal to
the higher appellate authority)
• Reduce (i.e. additions made by the assessing officer may be reduced by an
appellate authority and therefore the income tax department may file an appeal
to the higher appellate authority)
• Enhance (taxpayer shall be provided reasonable opportunity of being heard)
• Annul the assessment. An assessment may be annulled where:
a) The notice has been issued without jurisdiction i.e. by a wrong officer
b) The notice has not been properly served
c) Assessment has been framed on the wrong person
d) Assessment has been made in the wrong tax year.

In the above cases, re-assessment can be made by issue of proper notice, served on the proper
person for the proper tax year subject to time limitation allowed by the Ordinance.

421 | P a g e
e) Assessment has been made in respect of a capital receipt or income which is exempt
f) Assessment was time barred.

Appeals to Commissioner
A. Time limit for filing of appeal
30 days from the date of receipt of order.
Late filing may be admitted by the commissioner (Appeals) on any reasonable ground.
B. Documents to be submitted with appeal
a) Prescribed form of appeal
b) Grounds of appeal (i.e. a brief summary of objections)
c) Notice of demand and copy of order
d) Power of attorney in favor of the authorized representative
e) Challan for appeal fee as under:
• For a company: Rs. 5,000;
• For individual and AOP: Rs. 2,500 for appeal against an assessment order and Rs.
1,000 for appeal against any other order.
C. Condition to be fulfilled before filing of appeal
The appeal shall become invalid if the person has not paid the tax which was required to be paid
at the time of filing of return of income.

D. Procedure in Appeal and Stay of tax demand


o Commissioner (Appeals) shall fix a day for hearing of appeal and shall give a notice to
the appellant and the concerned Commissioner.
o The Commissioner (Appeals) may adjourn the hearing of the appeal from time to time.
o The Commissioner (Appeals) may, before the hearing of an appeal, allow an appellant
to file any new ground of appeal where he is satisfied that the omission of the ground
from the form of the appeal was not willful or unreasonable,
o The Commissioner (Appeals) may, before disposing of an appeal, call for such particulars
as he may require or cause further enquiry to be made by the Commissioner.
o The Commissioner (Appeals) shall not admit any documentary evidence which was not
produced before the Commissioner unless he is satisfied that the appellant was prevented
by sufficient cause from producing such material or evidence before the Commissioner.

E. Automatic stay of tax demand


If a taxpayer has paid 10% of the tax demand and the appeal is pending before Commissioner
(Appeals) then no recovery proceeding shall be initiated by the tax department.
F. Stay of tax demand by the Commissioner (Appeals)
Commissioner (Appeals) has power to grant stay of tax demand for a period up to 30 days after
providing an opportunity of being heard to the tax department.
Another stay of tax demand may also be granted for a further period of 30 days by the
Commissioner (Appeals) after providing an opportunity of being heard to the tax department
provided that the order on appeal shall be passed by the Commissioner (Appeals) within the said
period of 30 days.

422 | P a g e
G. Time limit for Commissioner (Appeals)
The Commissioner (Appeals) shall decide the appeal within 120 days from the filing of
appeal. This time limit may be extended by 60 days for reasons to be recorded in writing.
A period during which the hearing of an appeal is adjourned at the request of the appellant or is
postponed due to any reason shall be excluded for the purpose of this time limit.

Appeals to Appellate Tribunal

A. Time limit for filing of appeal


60 days from the date of receipt of decision order of Commissioner (Appeals). Late filing
may be admitted by the Appellate Tribunal on any reasonable ground.

B. Documents to be submitted with appeal


n) Prescribed form of appeal
o) Grounds of appeal (brief summary of objections) including copy of grounds of first appeal
p) Copy of order
q) Copy of Commissioner (Appeals) decision order
r) Power of attorney in favor of the authorized representative
s) Challan for appeal fee of Rs. 5,000 for a company and Rs. 2,500 for others.
C. Procedure in Appeal
o Power of attorney and challan for appeal fee are not required if appeal is filed by the tax
department.
o Appellate Tribunal shall decide an appeal within 6 months of its filing.
o Appellate Tribunal may grant stay of tax demand up to 180 days in aggregate after providing an
opportunity of being beard to the tax department.
D. Composition of Tribunal
Appellate Tribunal shall consist of a chairperson and other judicial and accountant members appointed
in the manner as the Prime Minister may prescribe by the rules.
A person may be appointed as judicial member if the person:
• Has been a Judge of a High Court;
• Is or has been District Judge;
• Is an advocate of a High Court with a standing of at least 10 years; or
• Possesses such other qualification as may be prescribed.
A person may be appointed as accountant member if he is:
• An officer of Inland Revenue equivalent to the rank of Chief Commissioner;
• A Commissioner or Commissioner (Appeals) having at least 3 years' experience; or
• A person who has, for a period of not less than 10 years, practiced professionally
as a chartered accountant or as a cost and management accountant.

Question
Under the provisions of the Income Tax Ordinance, 2001 who may be appointed judicial and accountant
member of the Appellate Tribunal? (Marks 6)

423 | P a g e
Reference to High Court (HC) -Section 133
(i) An appeal can be filed to HC only in respect of question of law after the decision
of the Appellate Tribunal. Whether a question of law arises or not shall be decided
by the HC
(ii) For this purpose, a reference application is required to be filed within 90 days by
the taxpayer (along with fee of Rs.100) or by the tax department.
(iii) This reference application shall be heard by a Bench of not less than two judges of
HC and if the bench is satisfied that a question of law arises, it may proceed to hear
the case.
(iv) If a taxpayer files a reference application to HC even then he is required to pay tax
in accordance with the Appellate Tribunal order. However, HC may order for the
stay of tax demand for a maximum period of six months.
(v) If tax demand is reduced as a result of HC judgment, then the tax paid by the
taxpayer is refundable. However, the Commissioner may postpone the refund
subject to the following conditions:
• Tax department intends to file an appeal to the Supreme Court
• Tax department files an application to the HC within 30 days of the
receipt of HC judgment to get permission from HC to postpone the
refund; and
• HC makes an order authorizing the Commissioner to postpone the
refund till the decision of the Supreme Court.

Alternative Dispute Resolution (ADR) – Section 134A


(a) ADR is a legal forum for resolution of an aggrieved person's grievances related to:
• Tax liability of Rs. 100 million or above;
• Admissibility of refund
• Waiver of default surcharge or penalty; or
• Any other specific relief

(b) A person may apply to FBR to refer the issue to the committee which is under
litigation in an appellate authority including court of law except where criminal
proceedings have been initiated.
The application shall be accompanied by an initial proposition for dispute
resolution, including an offer of tax payment

(c) Composition of the forum


The FBR shall, upon examination of an application, appoint ADRC within 15 days and
refer the case to ADRC. The ADRC shall comprise:
1. A retired judge of a High Court or Supreme Court who shall also be the chairperson
of ADRC, to be nominated by the FBR from a panel notified by the Law and
Justice Division
2. Chief Commissioner having jurisdiction over the case.
3. A person to be nominated by the taxpayer from a panel notified by the FBR comprising;
a) Chartered accountants, cost and management accountants and advocates
having minimum 10 years of experience in the field of taxation. The taxpayer
shall not nominate a chartered accountant or advocate if the said person is or
has been an auditor or an authorized representative of the taxpayer.
b) Retired officers of the inland revenue service in bs-21 or above; or
c) Reputable businessman as nominated by chamber of commerce and industry.

424 | P a g e
.

(d) Stay of tax demand


The recovery of tax related to the dispute referred to ADRC shall be stayed upon withdrawal
as pending appeal till the final decision of ADRC or dissolution of ADRC, whichever is
earlier.

(e) Mode of proceedings and disposal


ADRC shall undertake an examination of the issue and may make enquiry, obtain expert
opinion and cause an audit by any income tax authority or any other person. Based on the
findings, ADRC shall decide the issue by majority within 45 days which may be extended
to further 15 days for the reasons to be recorded in writing.
The decision of ADRC shall not be cited or taken as precedent in any other case or in the
same case in a different tax year.
The decision of ADRC shall be binding on the Commissioner when the taxpayer has
withdrawn the appeal pending before the appellate authority including court and the
withdrawal is communicated to the Commissioner within 60 days of the service of decision
order of the ADRC.
The Commissioner shall also withdraw his appeal, if any, before the appellate authority
including court within 30 days of the order of withdrawal communicated by the taxpayer
subject to the condition that the taxpayer has paid tax as decided by ADRC.

(f) Effect of failure to decide the case by ADRC


If ADRC fails to decide the case within the time specified then FBR shall dissolve ADRC
and the matter shall then be decided by the appellate authority including court where the
dispute was pending.
FBR shall communicate the order of dissolution of ADRC to the taxpayer, Commissioner
and the appellate authority including court of law which shall decide the appeal within 6
months of the communication of dissolution order.
Assessment giving effect to an order [Sec 124]
The various situations and time limitations are tabulated below:

Decision of appellate authority Time within which the new assessment order has to be made

Direct relief provided to taxpayer Two months from the date the order is served on the
commissioner

One year from the end of the financial year in which the
commissioner is served with the order provided no further appeal
Assessment order wholly or partly
or reference is preferred against the order of the appellate
set aside
authority either by the commissioner or the taxpayer

Two years from the end of the financial year in which the
Any other decision
commissioner is served with the order

In case of an assessment order is set aside or modified, the proceedings may commence from the stage next
preceding the stage where the setting aside or modification took place. In these cases, Commissioner shall not
be entitled to re-issue any notice which was earlier issued or shall not require refurnishing or re-filing of any
return, statement or other particulars which were earlier furnished or filed.

425 | P a g e
Where in consequence of an order of any appellate forum or court any income is excluded from the
computation:
• of taxable income of a person for any year and included in the computation of taxable
income for another year; or
• of taxable income of one person is included in the taxable income of other person.
the assessment or amended assessment made as above shall be treated as assessment in
consequence of such order.
In case of an assessment order passed under section 124 (Appeal effect), the tax payable shall become payable
immediately instead of payment within 30 days.

Burden of Proof [Sec 136]


Section 136 of the Ordinance points to one very important aspect of appeal and states that in any appeal by a
taxpayer, the burden shall be on the taxpayer to prove, on the balance of probabilities:
(vi) In the case of an assessment order, the extent to which the order does not correctly reflect the
taxpayer’s tax liability for the tax year; or
(vii) In the case of any other decision, that the decision is erroneous.

426 | P a g e
Chapter: 20 Questions ICAP CAF
Returns, Assessment and Appeals
Question # 1 [Q.6 May 1994]

State different provisions under which assessment can be framed.


(Marks: 07)

Question # 2 [Q.7 May 1994] Marks 08 [Q.2 March 1999] Marks 05 [Q.6 March 2000]Marks 10

What are the various options available to a taxpayer who is not satisfied with the assessment
order framed by the income tax assessing officer?

Question # 3 [Q.6 (a) April 1995]

What are the appeal forums available to an aggrieved person?


(Marks: 03)

Question # 4 [Q.6 (b) April 1995] Marks 07 [Q.8 September 2003]


Marks 05

What are the conditions to be fulfilled before filing of appeals?

Question # 5 [Q.6 (ii) May 1997]

Write short note on Appellate Authorities.


(Marks: 04)

Question # 6 [Q.4 October 1997] Marks 09 [Q.3 September 1999]


Marks 10 [Q.1 March 2001]
Marks 09 [Q.3 (a) March 2003]
Marks 05 [Q.7 September 2005]
Marks 06 [Q.2 (a) September 2008]
Marks 06 [Q.2 (a) March 2015]
Marks 06 [Q.5 (a) March 2017]
Marks 06

Who is required to file a return of income? What are the dates of filing the return? And
Extension in date for filing return.

Question # 7 [Q.2 (ii) April 1998] Marks 04 [Q.6 (a) September 1999] Marks 08

What are the powers of the Commissioner for revision of the assessment order? Also explain
the conditions for revision applications.

Question # 8 [Q.5 March 2000] Marks 10 [Q.6 (b) September 2006] Marks 05 [Q.2 (b) September 2010]
Marks 05 [Q.4 (b) September 2017] Marks 07

Can assessment once finalized be re-opened or modified or further modified? Discuss with
basis and reasons.

Question # 9 [Q.8 September 2000]

427 | P a g e
An assessing officer has completed the assessment proceedings by making the following
additions in the income of a taxpayer:
1. Depreciation disallowed as claimed by the taxpayer;
2. Disallowance of printing and stationery expenses to the extent of 10% of the claim;
3. Non-acceptance of trading results by estimating sales from Rs.1 million to Rs.1.1
million and enhancing gross profit rate from 12% to 15%.
The aggrieved taxpayer has approached you and requested to let him know his rights
available under the law to protect his interest.
You are required to address him a reply including stating the deadlines within which such
legal rights can be
availed. (Ma
rks: 05)

Question 10
[Q.6 September 2001] Marks 10
[O.8 March 2003] Marks 10

Describe briefly the provisions relating to re-opening (i.e. amendment and further amendment)
of a completed assessment, including period of limitation, if any?

Question 11 [Q.9 March 2002]

You have received a letter from Mr. ZA who is seeking your advice regarding the mode and
procedure of filing an income tax appeal. Please draft a suitable reply briefly describing the
appellate procedure and incorporate the following chart in your reply:
Name of Authority whose Filing Limitation Decision in Limitation
Appellate order may be Fee period for appeal Period for
authority appealed against filing appeal decision

(Marks; 12)
Question 12 [Q.3 (b) March 2003]

Can the commissioner order a "person" to file a return of Income? Please explain. (Marks: 05)

Question 13 [Q.7 September 2003]

Briefly state the time frame for filing the return of income by:
1. a company;
2. Persons other than a company. (Marks: 05)

Question 14 [Q.9 September 2003]

Briefly describe the minimum books of accounts, documents, and records that are required to
be maintained by the following taxpayers:

428 | P a g e
1. Taxpayer (other than a company) deriving business income up to Rs. 500,000. (Marks: 04)
2. Taxpayer (other than a company) deriving business income exceeding Rs. 500,000.(Marks: 04)

Question 15 [Q.7 (b) March 2004]

For how many years the tax payer is required to maintain accounts and documents under the
relevant provision of the Income Tax Ordinance, 2001.(Marks: 02)

Question 16
[Q.8 March 2004] Marks 05
[Q.2(c) March 2014] Marks 05
[Q.5 (a) (ii) September 2022] Marks 02

Describe the requirements of Income Tax Ordinance, 2001 for persons who are about to
discontinue their business.

Question 17 [Q.9 (a) September 2004]

Briefly state the time limit within which the Commissioner is permitted to further amend an assessment.
(Marks: 02)

Question 18 [Q.9 (b) September 2004]

Briefly state the time limit within which the Commissioner is required to pass an order to
give effect to the finding or directions of the Commissioner (Appeals). (Marks: 02)

Question 19 [Q.10 September 2004]

Briefly state the time limit for filing an Appeal/Reference before the following forums:
1. Commissioner (Appeals) (Marks: 01)
2. Appellate Tribunal (Marks: 01)
3. Reference to the High Court (Marks: 01)

Question 20 [Q.8 March 2005]

What is the status of a complete return of income filed under the Income Tax Ordinance, 2001? (Marks: 05)

429 | P a g e
Question 21
[Q.6 March 2006] Marks 04
[Q.3(c) March 2008] Marks 04
[Q.5 (b) (i) March 2017] Marks 03

Describe the circumstances under which the Commissioner is empowered to issue a notice
requiring a person to furnish a return of income for a period of less than twelve months.

Question 22 [Q.8 March 2006]

What are the time limits prescribed by the Income Tax Ordinance 2001, within which the
Commissioner is required to pass an order to give effect to the decision of Appellate Tribunal
under the following circumstances?
1. T
he Appellate Tribunal has set aside the assessment and order of the Appellate Tribunal
was received by the Commissioner on 30.11.20X4. (Marks: 03)
2. T
he Appellate Tribunal has deleted the additions made by the assessing officer and
the order of the Appellate Tribunal was received by the Commissioner on
15.112.20X4.(Marks: 03)

Question 23 [Q.6 (a) September 2006]

One of your client has received a notice from the Taxation officer demanding payment of tax in
respect of an order issued by the Commissioner against which your client intends to file an
appeal before the Appellate Tribunal.
You are required to explain the provisions contained in the Income Tax Ordinance, 2001
regarding stay of demand by the Appellate Tribunal.

(Marks: 06)

Question 24 [Q.4 (b) September 2018]

Specify the circumstances under which the Commissioner of Income Tax has powers to issue
notice demanding a return of income from certain person(s) for a period of less than twelve
months.
Also state the powers of the Commissioner if such person fails to furnish the return as required,
within the specified time.

(Marks: 06)
Question 25 [Q.4 (a) March 2019]

430 | P a g e
Under the Income Tax Ordinance, 2001 identify four situations under which an appeal may be
filed with the Commissioner (Appeals). (Marks: 04)

Question 26 [Q.4 (b) March 2019]

Mr. SA has received an ex-parte assessment order from the income tax department under which
he is required to pay Rs.5.2 million on account of tax not withheld from certain payments. He
does not agree with the contention of the income tax department and would like to file an
appeal to the Commissioner (Appeals).

Required:
State the procedure that he should follow for fling of appeal to the Commissioner (Appeals).
(Marks: 03)

Question 27 [Q.4 (a) September 2019]

Under the provisions of the Income Tax Ordinance, 2001 and Rules made thereunder, briefly
explain the requirement of books of account to be maintained by a taxpayer who has business
income up to Rs.500,000. (Marks: 04)

Question 28 [Q.4(c) September 2019]

Under the provisions of the Income Tax Ordinance, 2001 and Rules made thereunder, briefly
explain the provisions regarding Special Audit Panel. (Marks: 05)

Question 29 [Q.3 (a) September 2020]

On 2.7.2019, Rubina received a show cause notice u/s 122 from the Commissioner Inland
Revenue (CIR) for amendment of the assessment order for tax year 2018. Due to lack of
knowledge about tax matters, she did not respond to it.
On 1.8.2019, she received a demand notice under which she was required to pay Rs. 610,00
within 30 days on account of undeclared income and an amended assessment order for tax year
2018 under section 122 from the CIR.
Rubina is dissatisfied with the order issued by the CIR and wants to file an appeal to the
Commissioner (Appeals) because payment of this amount will cause hardship to her.
Required:
Under the provisions of the Income Tax Ordinance, 2001:
1. State the time period within which an appeal may be filed by Rubina to the
Commissioner (Appeals).(Marks: 01)
2. Discuss different types of orders that the Commissioner (Appeals) may make for
disposing of an appeal.(Marks: 02)

431 | P a g e
3. Explain what action(s) the Commissioner (Appeals) may take for ensuring that no
undue hardship will caused to Rubina because of the payment of this
demand.(Marks: 03)
4. Discuss the option(s) available to Rubina for defending her case, if the
Commissioner (Appeals) issues an order confirming the amended
assessment order issued by the CR. (Marks: 02)

Question 30 [Q.4 (b)(i) March 2021]

Briefly explain the provisions of the Income Tax Ordinance, 2001 and Rules made thereunder
relating to requirement of books of account to be maintained by a manufacturer having
turnover exceeding Rs.2.5 million. (Marks: 04)

Question 31 (Q.5 March 2021]

Star Garments Ltd (SGL) had filed its tax return for the tax year 2015 on 30.9.2015.
On 25.2.2021, the Commissioner, on the basis of definite information, issued a notice under section 122
(5) to SGL for the audit of books of account for the tax year 2015.
The accountant informed the chief executive officer that tax audit for the tax year 2015 had
already been conducted in 2019 and an amended assessment order under section 122(5A) was
issued by the Commissioner on 24.2.2020.
Required:
Under the provisions of the Income Tax Ordinance, 2001:
1. Explain the term ‘Definite information'. (Marks: 02)
2. Discuss whether the Commissioner is empowered to make further amendment in the
assessment order issued on 24.2.2020. (Marks: 07)

Question 32 [Q.4 (a) March 2022]

Briefly explain the term 'Sectoral benchmark ratios'. Also, explain the circumstances in which a
Commissioner shall determine taxable income on the basis of sectoral benchmark ratios.
(Marks: 03)

Question 33 [Q.4 (b) (i) March 2022]

Riaasat Ltd (RL) is a manufacturing company. With effect from 1.7.2022, RL İs considering to
change its tax year from the normal to the special tax year ending on 31 December.
Required:

432 | P a g e
1. Identify the due/last date of filing of RL's tax return in respect of the following:
1. Filing of tax return for the year ended 30 June 2022.
2. Filing of tax return for the transitional period,
3. Filing of first tax return for the special tax year.
Assume that RL has changed its tax year from normal to special and filed its tax returns
for relevant tax years, as discussed above. (Marks: 03)
2. Identify the due/last date of amendment of assessment related to;
1. Normal tax year for the year ended 30.06.2022
2. First special tax year. (Marks: 02)

Question 34 [Q.5 (a) (i) September 2022]

Under the provisions of the Income Tax Ordinance, 2001 and Rules made thereunder, briefly
discuss the terms 'Normal assessment' and 'Best judgement assessment'.(Marks: 03)
Question 35 [Q.5 (a) (iii) September 2022]

Under the provisions of the Income Tax Ordinance, 2001 and Rules made thereunder
additional records which are required to be kept by a sole proprietor whose business income
exceeds Rs. 500,000 as compared to a sole proprietor whose business income is up to Rs.
500,000. (Marks: 02)

Question 36 [Q.4 (b) March 2023]

On 1.7.20X2, Kulsoom, a widow, established an online garment retail business and employs
various ecommerce platforms to market a diverse range of garments to customers through
Pakistan. She operates the business from her residential house.
During the year, the sales from online business were Rs. 6,000,000 and total expenditure
including her personal expenses were Rs. 5,800,000.
At the year end, her sole assets consisted of a 1300cc personal car and Rs. 800,000 as cash on hand.
Required:
Under the provisions of the Income Tax Ordinance, 2001 and Rules made thereunder, advice
the requirement of filing of return of income and tax treatment of the above to Kulsoom.
(Calculation of taxable income and tax liability is not required) (Marks: 05)

Question 37 [Q.5 (a) March 2023]

Assurance & Co., a partnership firm, has filed an appeal against the order of the Commissioner
(Appeals) to the Appellate Tribunal Inland Revenue (ATIR) which is pending before ATIR.

433 | P a g e
Recently, the firm has appointed a new tax adviser who proposed seeking relief through
Alternative Dispute Resolution (ADR) mechanism provided by the Income Tax Ordinance,
2001.
Under the provisions of the Income Tax Ordinance, 2001, advise Assurance & Co. in respect of
the following matters:
Cases eligible for settlement through ADR mechanism. (Marks: 02)
Composition of the ADR committee. (Marks: 03)

Question 38 [Q.5 (b) March 2023]

State the provisions of the Income Tax Ordinance, 2001 relating to the filing of revised return
of income by a taxpayer. (Marks: 04

ANSWERS

Question # 1 [Q.6 May 1994]

The different provisions under which assessment can be framed are as under:
1. Self-assessment scheme where return of income filed by a taxpayer is
deemed to be an assessment order which may be subject to audit etc. at any
subsequent stage
2. Assessment under audit
3. Assessment under amendment or further amendment
4. Best judgement assessment
5. Provisional assessment
6. Agreed assessment.

Question # 2 [Q.7 May 1994] Marks 08 [Q.2 March 1999] Marks 05 [Q.6 March 2000]Marks 10

Various options available to a taxpayer who is not satisfied with the assessment order framed by
the income tax assessing officer are as under:
1. He may file an appeal before the Commissioner (Appeals) within 30 days of
receiving the assessment order;
2. After filing an appeal before the Commissioner (Appeals), the taxpayer may file an
application to the FBR to refer his case to an Alternative Dispute Resolution
Committee;
Or

434 | P a g e
3. If the assessment order was passed by an authority sub-ordinate to Commissioner then
the taxpayer may file a revision application to the Commissioner only where the
taxpayer decides not to file an appeal to the Commissioner (Appeals).

Question # 3 [Q.6 (a) April 1995]

Appeal forums available to an aggrieved person are as under:


1. Commissioner (Appeals);
2. Appellate Tribunal;
3. High Court; and
4. Supreme Court.
An application may also be given to the FBR to refer the case to be decided
by Alternative Dispute Resolution Committee.

Question # 4 [Q.6 (b) April 1995] Marks 07 [Q.8 September 2003]


Marks 05

If a person has not paid the tax which was required to be paid at the time of filing of return of
income, the appeal shall become invalid.
Further the taxpayer is required to file appeal before the Commissioner (Appeals) with
prescribed documents and appeal fee within 30 days of receipt of the order against which appeal
is being filed.

Question # 5 [Q.6 (ii) May 1997]

The Appellate Authorities and short notes thereon are as under:


Commissioner (Appeals):
A taxpayer may file an appeal to the Commissioner (Appeals) against an order passed by any
income tax authority within 30 days of receipt of order with specified documents and appeal
fee.
Appellate Tribunal:
A taxpayer or the tax department may file an appeal to the Appellate Tribunal against the
order passed by the Commissioner (Appeals) within 60 days of the receipt of appellate
order with specified documents and appeal fee.
High Court:
A taxpayer or the tax department may file an appeal to a High Court against the decision
of the Appellate Tribunal only against a question of law for which a reference application
is required to be filed to the High Court. The Appellate Tribunal is the final authority for
a question of facts.
Supreme Court:
A taxpayer or the tax department may file an appeal to the Supreme Court against the decision
of a High Court only against a question of law

Question # 6 [Q.4 October 1997] Marks 09 [Q.3 September 1999]


Marks 10 [Q.1 March 2001]

435 | P a g e
Marks 09 [Q.3 (a) March 2003]
Marks 05 [Q.7 September 2005]
Marks 06 [Q.2 (a) September 2008]
Marks 06 [Q.2 (a) March 2015]
Marks 06 [Q.5 (a) March 2017]
Marks 06

Persons required to file return of income:


The following persons are required to file return of income:
1. Every company including an approved non-profit organization;
2. A person other than a company whose taxable income exceeds basic exemption;
3. A person whose income for the year is subject to FTR;
4. Who has been charged to tax for any of the two immediately preceding tax years;
5. Who claims a loss carried forward;
6. A person who owns immovable property of 500 square yards or more or a flat in urban areas;
7. A person who owns immovable property of 500 square yards or more in a rating area;
8. A person who owns a flat of 2,000 square feet or more in a rating area;
9. A person who owns a vehicle of above 1,000 cc;
However, the provisions of clauses (f) to (i) are not applicable for a non-resident
person, widow, orphan below 25 years of age and disabled person.
10. A person who has obtained National Tax Number;
11. A person who holds commercial or industrial connection of electricity where
the annual bills exceed Rs. 500,000;
12. A resident person who is registered with any Chamber of Commerce
and Industry or any trade association or market committee or any professional
body including Pakistan Engineering Council, Pakistan Medical and Dental
Council, Bar Council, 1CAP or 1CMAP:
13. Every resident person required to file foreign income and assets statement.
Date for filing of return of income:
1. 30th September
1. Return of income by an individual or AOP
2. Return of income by a company having year end between 1 July to 31 December.
2. 31st December
Return of income by a company having year end between 1 January to 30h June.

Extension of time for filing of return of income:

436 | P a g e
The taxpayer is required to file an application to the Commissioner and the
Commissioner may grant an extension of time up to 15 days if he is satisfied that the
applicant is unable to furnish the return by the due date because of:
1. absence from Pakistan;
2. Sickness or other misadventure; or
3. Any other reasonable cause.
Under exceptional circumstances a longer time extension may also be granted by the Commissioner.
If a Commissioner refuses to extend the time for filing of return of income then the
taxpayer may file an application to the Chief Commissioner who may allow such
extension of time for 15 days and a longer time under exceptional circumstances.

Question # 7 [Q.2 (ii) April 1998] Marks 04 [Q.6 (a) September 1999] Marks 08

Revision by the Commissioner:


The Commissioner is empowered to call for the record of any proceedings in which an order
was passed by any authority sub-ordinate to him for making an enquiry on his own motion or on
an application made by the taxpayer for revision.
The Commissioner may revise any order passed by any Officer of Inland Revenue if the
assessment is not under any appeal.
The said revision shall not be prejudicial to the person to whom the order relates. It means that
the Commissioner does not have power of enhancement of income under this provision.

Question # 8 [Q.5 March 2000] Marks 10 [Q.6 (b) September 2006] Marks 05 [Q.2 (b) September 2010]
Marks 05 [Q.4 (b) September 2017] Marks 07

The Commissioner has power to amend or to further amend an assessment, within the
specified time limits, by making such alterations as considered necessary to ensure that the
taxpayer is liable for the correct amount of tax,
Before making an amendment or further amendment an opportunity of being heard shall be
provided to the taxpayer.
An assessment can be amended or further amended only where:
1. The Commissioner is of the view that the assessment order is prejudicial to the interest of
revenue or the income tax ordinance has incorrectly been applied in the original
assessment including:

1. Misclassification of income under a head of income


2. Incorrect payment of tax with the return of income
3. Incorrect claim for any tax relief, exemption or refund

437 | P a g e
2. The Commissioner has definite information that income declared is incorrect
including concealment of income or furnishing of inaccurate particulars. Definite
information includes information in respect of:
1. Taxable sales, purchases, services and other receipts
2. Acquisition of any asset or investment made
3. Expenditure incurred

Question # 9 [Q.8 September 2000]

Our observations on the issues raised are as under:


Depreciation disallowed as claimed by the taxpayer:
If an asset is covered under the definition of depreciable asset and the payment is made through
banking channel where the cost of an immovable property exceeds Rs.5 million and in case of
other asset Rs.1 million then the Commissioner has no right to disallow depreciation claimed
by a taxpayer under the provisions of law including 3rd Schedule to the Income Tax Ordinance,
2001.

Disallowance of printing and stationery expenses to the extent of 10% of the claim:
If the taxpayer has evidences of purchase of printing and stationery items and uses them for
business purpose then the Commissioner has no right to disallow the expenses in this respect
claimed by the taxpayer.
Non-acceptance of trading results by estimating sales from Rs.1 million to Rs.1.1 million and
enhancing gross profit rate from 12% to 15%:
If the trading results are verifiable from evidences then the Commissioner cannot increase the
sales or gross profit rate on arbitrary basis.
If the Commissioner has made the above disallowances in the assessment order and the
taxpayer has complied with the requirements above mentioned then he should file an appeal to
the Commissioner (Appeals) within 30 days of the receipt of such assessment order along with
the specified documents and appeal fee.

Question 10
[Q.6 September 2001] Marks 10
[O.8 March 2003] Marks 10

The Commissioner has power to amend or to further amend an assessment, within the
specified time limits, by making such alterations as considered necessary to ensure that
the taxpayer is liable for the correct amount of tax.
Before making an amendment or further amendment an opportunity of being heard shall
be provided to the taxpayer.
An assessment can be amended or further amended only where:
1. The Commissioner is of the view that the assessment order is prejudicial to the interest of
revenue or the income tax ordinance has incorrectly been applied in the original
assessment including:

438 | P a g e
1. Misclassification of income under a head of income
2. Incorrect payment of tax with the return of income
3. Incorrect claim for any tax relief, exemption or refund
2. The Commissioner has definite information that income declared is incorrect
including concealment of income or furnishing of inaccurate particulars. Definite
information includes information in respect of:

1. Taxable sales, purchases, services and other receipts


2. Acquisition of any asset or investment made
3. Expenditure incurred
Time limit for amendment or further amendment
An assessment can only be amended within 5 years from the end of the financial year in
which the Commissioner treated to have issued the original assessment order.
The Commissioner has power to make further amendments, as many times as may be
necessary, within the later of:
1. 5 years from the end of the financial year in which the Commissioner treated to
have issued the original assessment; and
2. 1 year from the end of the financial year in which the amendment was made.
The Commissioner shall pass amended or further amended order within 180 days of the
issuance of show cause notice (may be extended by maximum 90 days by the
Commissioner for the reasons to be recorded in writing). Time shall be excluded from this
limit on account of:
1. Adjournment of hearing on the request of the taxpayer not exceeding 60 days;
2. Adjournment on account of stay order;
3. Adjournment on account of alternative dispute resolution committee proceedings;
Adjournment on account of agreed assessment proceedings

Question 11 [Q.9 March 2002]

We have received your letter whereby you have sought our advice regarding the mode and
procedure of filing an income tax appeal.
We have reviewed the relevant provisions of the Income Tax Ordinance, 2001 and our reply on
the issues raised by you is as under:
A taxpayer has a right to file an appeal where he is aggrieved by any order passed by the
assessing officer inter alia making additions to his income or imposing any penalty.

The following chart would provide solutions to your queries:

439 | P a g e
Name of Authority whose Filing Fee Limitation Limitation
Appellate order may be period for Period for
Authority appealed against filing appeal decision
Commissioner Commissioner or In case of a 30 days 120 days from
(Appeals) any officer Company Rs. 5,000. the filing of
subordinate to him In case of an appeal may be
individual and AOP extended by
Rs. 2,500 for an 60 days
assessment order
and Rs. 1,000 for
other order
Appellate Commissioner Rs.5,000 for a 60 days 6 months from
Tribunal (Appeals) Company and filing of
Rs.2,500 for others appeal

Reference Appellate Tribunal Rs.100 90 days Not specified


application to Answer to Q.14
High Court passed by the
following minimum
books of accounts
and records
Supreme Court High Court Not specified Not specified Not specified

Question 12 [Q.3 (b) March 2003]

The Commissioner may give a notice to file return of income within 30 days or any longer
or shorter time from the date of notice to any person who was required to file return in the
opinion of the Commissioner but has failed to do so.
Such notice may be issued only in respect of any of the last 5 tax years. However, where a
person has not filed return for any of the last 5 completed tax years, such notice may be
issued in respect of one or more of the last 10 completed tax years.
The above time limit shall not apply if the Commissioner is satisfied that a person who
failed to file his return has foreign source income or owns foreign assets

440 | P a g e
Question 13 [Q.7 September 2003]

1. A company is required to file its return of income on or before 31st December next
following its financial year end where the company has year-end between 1st January
to 30th June.
30th September is the due date for a company having year end between 1st
January to 31st December.
2. Due date for filing return of income is 30 September for a person other than a company.

Question 14 [Q.9 September 2003]

1. Taxpayers other than a company with business income up to Rs. 500,000 shall
maintain the following minimum books of accounts and records:
1. Serially numbered and dated cash-memo/ invoice/ receipt for each
transaction of sale or receipt containing the following:
1. taxpayer's name or the name of his business, address, NTN /
CNIC and sales tax registration number, if any; and
2. The description, quantity and value of goods sold or services rendered.
2. Daily record of receipts, sales, payments, purchases and expenses; a single
entry in respect of daily receipts, sales, purchases and different heads of
expenses will suffice; and
3. Vouchers of purchases and expenses.

2. Taxpayers other than a company with business income exceeding Rs. 500,000
shall maintain the following minimum books of account and records:
1. Serially numbered and dated cash-memo/ invoice / receipt for each transaction of
sale or receipt containing the following:
1. Taxpayer's name or the name of his business, address, NTN / CNIC and sales tax
2. Registration number, if any;
3. The description, quantity and value of goods sold or services rendered; and
4. In case of a wholesaler, distributor, dealer and commission agent,
where a single transaction exceeds Rs. 10,000, the name and address
of the customer.
2. Cash book and/or bank book or daily record of receipts, sales, payments, purchases
and expenses; a single entry in respect of daily receipts, sales, purchases and
different heads of expenses will suffice;

441 | P a g e
3. General ledger or annual summary of receipts, sales, payments, purchases and expenses
under distinctive heads;
4. Vouchers of purchases and expenses and where a single transaction exceeds Rs.
10,000 with the name and address of the payee; and
5. Where the taxpayer deals in purchase and sale of goods, quarterly inventory of stock-
in-trade showing description, quantity and value.

Question 15 [Q.7 (b) March 2004]

The prescribed records shall be maintained for6 years after the end of the tax year to
which they relate. However, where any proceeding is pending before any authority or
court the taxpayer shall maintain the record till final decision of the proceedings.

Question 16
[Q.8 March 2004] Marks 05
[Q.2(c) March 2014] Marks 05
[Q.5 (a) (ii) September 2022] Marks 02

Any person discontinuing his business shall give a notice in writing to the Commissioner
to this effect within 15 days of the discontinuation along with return of income up to the
date of discontinuance.
If such notice is not given by the person and the Commissioner has reason to believe that
a business has been discontinued then he may give a notice to that person to file return of
income.
However, the FBR has provided relaxation of filing return of income in this short period
of 15 days. Notice of discontinuance is required to be given within 15 days.

Question 17 [Q.9 (a) September 2004]

The Commissioner has power to make further amendments, as many times as may be
necessary within the later of:
1. 5 years from the end of the financial year in which the Commissioner treated to
have issued the original assessment; and
1 year from the end of the financial year in which the amendment was made

Question 18 [Q.9 (b) September 2004]

1. If an assessment or amendment is to be made on the instructions of any


appellate authority then the Commissioner shall issue the order within 2 years from

442 | P a g e
the end of the financial in which the Commissioner is served with the appellate
order.
2. Where direct relief is provided in an appellate order, the Commissioner shall
issue appeal effect order within 2 months of the date the Commissioner is served with
the appellate order.

Question 19 [Q.10 September 2004]

1. 30 days
2. 60 days
3. 90 days

Question 20 [Q.8 March 2005]

A complete return of income (other than a revised return) filed by a taxpayer shall be
considered as an assessment of taxable income. However, if a return of income filed is not
complete, the taxpayer shall be required by the Commissioner to complete the return of
income.
A return of income shall be treated as a complete return if the following conditions are fulfilled:
1. In the prescribed form
2. Duly signed
3. State the prescribed information together with a declaration of the records kept
4. In the context of business, copy of accounts and any other prescribed documents are furnished
5. Accompanied with evidence of payment of due tax as per return of income
6. Accompanied with a wealth statement and wealth reconciliation statement; and
7. Accompanied with foreign income and assets statement, if required.

Question 21
[Q.6 March 2006] Marks 04
[Q.3(c) March 2008] Marks 04
[Q.5 (b) (i) March 2017] Marks 03

The Commissioner may give notice to any person or his representative to file return of
income for a period less than 12 months where the person has died, become bankrupt or
gone into liquidation or is about to leave Pakistan permanently.

Question 22 [Q.8 March 2006]

443 | P a g e
1. If an assessment or amendment is to be made on the instructions of any appellate
authority then the Commissioner shall issue the order within 2 years from the end of
the financial year in which the Commissioner is served with the appellate order.
2. Therefore, the Commissioner is required to pass the order by 30.6.20X7.Where direct
relief is provided in an appellate order, the Commissioner shall issue appeal effect
order within 2 months of the date the Commissioner is served with the appellate order.
Therefore, the Commissioner is required to pass order by 14.2.20X5

Question 23 [Q.6 (a) September 2006]

A taxpayer may file an application to the Appellate Tribunal for granting stay of tax
demand. If the Appellate Tribunal is of the opinion that the recovery of tax shall cause
undue hardship to the taxpayer, the Tribunal may stay the recovery of such tax for a
period not exceeding 180 days in aggregate.

For issuing an order of stay of tax demand the Appellate Tribunal shall provide an
opportunity of being heard to the tax department.

Question 24 [Q.4 (b) September 2018]

The Commissioner may give notice to any person or his representative to file return of income
for a period less than 12 months where the person has died, become bankrupt or .gone into
liquidation or is about to leave Pakistan permanently.
If the person fails to file return of income required as above then the Commissioner has power
to pass an assessment order under best judgement assessment

Question 25 [Q.4 (a) March 2019]

An appeal may be filed with the Commissioner (Appeals) if a person is not satisfied with
the order passed by the tax department such as:
1. Best judgment assessment (ex parte assessment)
2. Recovery order in respect of an alleged default ton account of withholding tax
3. Amendment or further amendment of an assessment
4. Order imposing any penalty or default surcharge.

444 | P a g e
Question 26 [Q.4 (b) March 2019]

Mr. SA is required to file the appeal in the prescribed form, verified in the prescribed manner,
be accompanied by the prescribed fee and the specified documents, including ground of appeal.
The appeal should be filed within 30 days of the service of intimation of the order. However
Commissioner (Appeals) may condone the delay in filing of an appeal upon application in
writing.

Question 27 [Q.4 (a) September 2019]

Taxpayers other than a company with business income up to Rs. 500,000 shall maintain the following
minimum books of accounts and records:
Serially numbered and dated cash-memo/ invoice/ receipt for each transaction of sale or receipt containing
the following:
•taxpayer's name or the name of his business, address, NTN / CNIC and sales tax registration number, if any;
and
•The description, quantity and value of goods sold or services rendered.
Daily record of receipts, sales, payments, purchases and expenses; a single entry in respect of daily receipts,
sales, purchases and different heads of expenses will suffice; and
Vouchers of purchases and expenses.

Question 28 [Q.4(c) September 2019]

The FBR may appoint special audit panels, to conduct audit including forensic audit of
the income tax affairs, comprising two or more members from the following:
1. Any officer of Inland Revenue;
2. A firm of chartered accountants;
3. A firm of cost and management accountants; or
4. Any other person including a foreign expert or specialist as directed by the FBR.

445 | P a g e
Chairman of the panel shall be an officer of Inland Revenue. If any one member of the
pane, other than the chairman, is absent from conducting audit, the proceedings of audit
may continue and the audit shall not be invalid or be called in question merely on the
ground of such absence.
The scope of such audit by the panel shall be as determined by the Commissioner or FBR
on case to case basis.
In case of a member other than officer of Inland Revenue, an agreement of confidentiality
shall be made between FBR and the person or international tax authority.

Question 29 [Q.3 (a) September 2020]

1. Within 30 days from the date of service of notice of demand which is 31.8.2019.
2. In disposing of an appeal, the Commissioner (Appeals), in respect of additions made
by the tax department, may:

1. Confirm the additions


2. Reduce the additions
3. Annul or set-aside the assessment.
4. Enhance (taxpayer shall be provided reasonable opportunity of being heard)
3. Stay of tax demand by the Commissioner (Appeals)
Commissioner (Appeals) has power to grant stay of tax demand for a period up to
30 days after providing an opportunity of being heard to the tax department.
Another stay of tax demand may also be granted for a further period of 30 days
by the Commissioner (Appeals) after providing an opportunity of being heard to
the tax department provided that the order on appeal shall be passed by the
Commissioner (Appeals) within the said period of 30 days.
Automatic stay of tax demand
Another option available to Rubina is automatic stay of tax demand in case if she
pays 10% of the tax demand and the appeal is pending before Commissioner
(Appeals) then no recovery proceeding shall be initiated by the tax department.
4. Rubina may file an appeal to the Appellate Tribunal against the order issued
by the Commissioner (Appeals) within 60 days of the date of service of order
of Commissioner (appeals).
Question 30 [Q.4 (b)(i) March 2021]

446 | P a g e
A Manufacturer having turnover exceeding Rs.2.5 million is required to maintain the
following by the Commissioner (Appeals)
1. Serially numbered and dated cash-memo/ invoice / receipt for each transaction of sale
or receipt containing the following:

1. Taxpayer's name or the name of his business, address, NTN or CNIC;


2. The description, quantity and, value of goods sold;
3. Where a single transaction exceeds Rs. 10,000 with the name and
address of the customer.
2. Cash book and/or bank book;
3. Sales day book and sales ledger (where applicable);
4. Purchases day book and purchase ledger (where applicable);
5. General ledger;
6. Vouchers of purchases and expenses and where a single transaction exceeds Rs.
10,000 with the name and address of the payee; and
Stock register of stock-in-trade (major raw materials and finished goods) supported by gate in- ward and
outward records and quarterly inventory of all items of stock-in-trade including work- in-process showing
description, quantity and value

Question 31 (Q.5 March 2021]

1. Definite information includes information in


respect of:
1. Taxable sales, purchases, services and other
receipts
2. Acquisition of any asset or investment made
3. Expenditure incurred
2. The Commissioner has power to make further amendments, as many times as may be
necessary, within the later of:
1. 5 years from the end of the financial year in which the Commissioner has
issued or treated to have issued the original assessment; and
2. 1 year from the end of the financial year in which the
amendment was made.

447 | P a g e
Question 32 [Q.4 (a) March 2022]

Where a taxpayer has not furnished the required documents or sufficient explanation, the
Commissioner shall determine taxable income on the basis of sectoral benchmark ratios
prescribed by the FBR.

"Sectoral benchmark ratios", means standard business sector ratios notified by the FBR on the
basis of comparative cases and includes financial ratios, production ratios, gross profit ratio, net
profit ratio, recovery ratio, wastage ratio and such other prescribed ratios.

Question 33 [Q.4 (b) (i) March 2022]

(i) Due Date

Filing of tax return for the year ended 30.6.2022 31.12.2022

Fling of tax return for the transitional period [i.e. 1.7.2022 to 30.9.2023
31.12.2022]

Filing of first tax return for the special tax year [i.e. 1.1.2023 30.9.2024
to 30.9.2024
31.12.2023]

(ii) Amendment of assessment related to Last Date


Normal tax year for the year ended 30.6.2022 30.6.2028

First special tax year 30.6.2030

Question 34 [Q.5 (a) (i) September 2022]

Normal assessment:
If a taxpayer has furnished a complete return of income, other than a revised return, the
Commissioner shall be treated to have assessed the income and tax due thereon.

448 | P a g e
Best judgment assessment:
The Commissioner is empowered to make an assessment of income and tax based on any
available information and to the best of his judgement in the case of failure by the
taxpayer to:
1. Furnish return of income in response to a notice issued by a Commissioner
2. File a wealth statement
3. Produce accounts, documents, and records required to be maintained under section
174 or any other documents that the Commissioner or special audit panel may have
required.
This assessment order shall indicate taxable Income, tax due and paid, if any, and the
time, manner and place of appealing the assessment order.

Question 35 [Q.5 (a) (iii) September 2022]

Following additional records are required to be kept by sole proprietor whose business
income exceeds Rs. 500,000 as compared to a sole proprietor whose business Income is
up to Rs. 500,000.
4. In case of a wholesaler, distributor, dealer and commission agent, where a single
transaction exceeds Rs. 10,000, the name and address of the customer;
5. Cash book and/or bank book;
6. General ledger or annual summary of receipts, sales, payments, purchases and
expenses under distinctive heads;
7. Where a single transaction exceeds Rs. 10,000 with the name and address of the payee; and
8. Where the taxpayer deals in purchase and sale of goods, quarterly inventory of
stock-in-trade showing description, quantity and value.
Question 36 [Q.4 (b) March 2023]

As a widow, she has been exempt from filing an income tax return despite owning a 1300
cc car. However, starting a new business during the tax year 20X3 obligates her to file an
income tax return if her income from business exceeds Rs. 600,000.
Although the reported income in the question is Rs. 200,000, which is within the basic
exemption of Rs. 300,000, the expenditure amount is comprised of both personal as well
as business expenses. However, for computing taxable income, only business expenses

449 | P a g e
shall be allowed. Therefore, by considering business expenses only, her income may
exceed Rs. 600,000 and consequently, she will be required to file the income tax return.

Question 37 [Q.5 (a) March 2023]

1. ADR is a legal forum for resolution of an aggrieved person's grievances related to:
1. Tax liability of Rs. 100 million or above;
2. Admissibility of refund
3. Waiver of default surcharge or penalty; or
4. Any other specific relief
A person may apply to FBR to refer the issue to the committee which is under litigation in
an appellate authority including court of law except where criminal proceedings have
been initiated.
2. Composition of the forum
The FBR shall, upon examination of an application, appoint ADRC within 15 days and
refer the case to ADRC. The ADRC shall comprise:
1. A retired Judge of a High Court or Supreme Court who shall also be the chairperson of
ADRG, to be nominated by the FBR from a panel notified by the Law and Justice
Division
2. Chief Commissioner having jurisdiction over the case
3. Person to be nominated by the taxpayer from a panel notified by the FBR comprising:
1. Chartered accountants, cost and management accountants and advocate
having minimum 10 years of experience in the field of taxation
2. The taxpayer shall not nominate a chartered accountant or advocate if the
said person is or has been an auditor or an authorized representative of the
taxpayer
3. Retired officers of the Inland Revenue Service in BS-21 or above; or
4. Reputable businessman as nominated by Chamber of Commerce and Industry

Question 38 [Q.5 (b) March 2023]

450 | P a g e
Any person who has furnished a return, discovers any omission or wrong statement
therein, may file a revised return subject to the following conditions:
1. It is accompanied by the revised accounts or revised audited accounts, as the case may be.
.
The Commissioner may waive this condition if he is satisfied that filing of
revised accounts or audited accounts is not necessary.
2. The reasons for revision of the return, in writing, duly signed by the taxpayer
are filed with the return.
A revised return may be filed within 60 days from the date of filling return without the
commissioner’s approval.
However, after 60 days, a revised return may be filed with the approval of the
Commissioner in writing for revision of the return.

Approval from Commissioner shall be deemed to have been granted if:


1. The Commissioner has not made an order of approval in writing for revision of
return within 60 days from the date of application for revision; or
2. Revised taxable income declared is more than the income previously declared or
the revised loss declared is less than the loss previously declared.

451 | P a g e
SECTION- B SALES TAX
Syllabus Weightage

Syllabus Teaching
Grid Weightage
Ref. Hours
C Sales Tax Laws 30-35 20-30

Syllabus Proficiency Testing


Learning Outcomes
Ref. levels levels
C. Sales Tax Laws

A Scope and Payment of Tax

Calculate sales tax (output and input) on taxable P2 T2


supplies (including zero-rated and exempt
supplies).

Discuss the time and manner of sales tax liability P2 T2


and its paymenT

Calculate apportionment of input tax and carry P2 T2


forward/refund thereof

B Registration

Describe the types, requirements and procedures P2 T2


involved for registration, de-registration and
returns.

C Book Keeping and Invoicing Requirements

List the records to be kept by a registered person P2 T2


and explain the related retention requirements and
procedures involved in the audit.

State the significance of tax invoice, debit and P1 T1


credit notes and their related requirements.

Explain the procedure for the destruction of goods P2 T2

452 | P a g e
CHAPTER 21
SALES TAX DEFINITION
1. History of Sales Tax in Pakistan
Sales Tax was a provincial subject at the time of partition. It was being administered in the provinces of Punjab
& Sindh as provincial levy. Sales tax was declared a federal subject in 1948 vide General Sales Tax Act, 1948
and this levy was transferred permanently to the Central Government in 1952. Sales tax was levied at the
standard rate of 6% at every stage of sales.

Later on, system of licensed manufacturers & wholesalers was instituted through the Sales Tax Act 1951
whereby they were allowed to purchase goods free of sales tax from each other and pay tax on sales to unlicensed
traders. Imports were chargeable to Sales Tax but the licensed manufacturers & wholesalers were allowed to
import goods without payment of Sales Tax. Later on Sales Tax became chargeable on locally produced &
imported goods at the time of their sales and import, respectively. The sales tax was collected, under the Finance
Ordinance 1956, on goods which were chargeable to Excise Duty, as if it were a duty of Excise. In 1981, by
virtue of an amendment in the Sales Tax Act 1951, the collection of Sales Tax on non-excisable goods was also
entrusted to the Excise Department.

In the late eighties Sales Tax was replaced with the Value Added Tax (VAT) in the country and accordingly
new enactment titled Sales Tax Act 1990 replaced Sales Tax Act 1951 with effect from 1.11.1990.

2. Sales Tax Definitions

Sales Tax
Sales tax means
a) The additional text or default surcharge leave under the sales tax act
b) A fine penalty or fee imposed or charged under the sales tax and
c) Any other some payable under the provisions of the sales tax act or the rules.

Output Tax:
Output Tax in relation to a registered person means
a) Tax levied under the sales Tax Act on a supply of goods made by the person
b) Tax levied under the federal excise Act 2005 in sales tax mode as a duty of excise on the manufacture
of the goods or rendering the services by the person
c) Sales tax levied on services rendered by the person under Islamabad capital territory (tax on services),
ordinance 2001
Input Tax
Input tax in relation to a registered person means
a) Tax Levied under sales Tax Act on supply of course to the person
b) Tax levied under sales Tax Act on the import of goods by the person
c) Tax levied under the federal excise Act 2005 in sales tax mode as a duty of excise on manufacture of
the goods rendering the services
453 | P a g e
d) Tax levied on services provided to the person under sales tax laws of provinces including Islamabad
and
e) Tax levied under sales Tax Act as adopted in the state of Azad Jammu and Kashmir on the supply of
goods received by the person
Person
Person means:
• Individual
• A company or association of persons incorporated formed organized or established in Pakistan or
elsewhere;
• The federal government
• A provincial government
• A local authority in Pakistan or
• A foreign government, a political subdivision of a foreign government or public international
organization

Registered Person:
It means a person who is registered or is liable to be registered under the Sales Tax Act. A person liable to be
registered but not registered shall not be entitled to any benefit available to a registered person.

Note: In view of the above definition, sales tax may be demanded by the sales tax department from an unregistered
person (who was required to be registered) despite the facts that he did not charge sales tax from his customers
and no input tax adjustment is available to him being unregistered

Taxable Activity
Taxable activity means any economic activity carried on by a person whether or not-for-profit and includes
a) An activity carried on in the form of business, trade or manufacture
b) An activity that involves the supply of goods or rendering services or both to another person
c) A one-off adventure or concern in the nature of a trade and
d) Anything done or undertaken during the commencement or termination of the economic activity.
But does not include
a) The activities of an employee providing services in that capacity to an employer
b) An activity carried on by an individual as a private recreational pursuit or hobby and
c) An activity carried on by a person other than an individual which, if carried on by an individual, would
fall within sub clause (b).

Taxable Supply
Taxable supply means a supply of taxable goods made by an importer; manufacturer, wholesaler (including
dealer), distributor or retailer other than a supply of goods which is exempt and includes a supply of goods
chargeable to tax at the rate of zero percent.

Exempt supply
Exempt supply" means a supply which is exempt from tax

Zero rated supplies


Zero-rated supply means a taxable supply which is charged to tax at the rate of zero percent

454 | P a g e
Goods
They include every movable property other than actionable claims, money, stocks and shares and securities.
• Immovable property is not subject to sales tax
• Actionable claim means a claim to a debt, promissory note etc.

Another example of actionable claim is:


A registered person issued discount cards/ coupons to its customer’s @Rs. 400 on the basis of which the
customer can avail discount of 20% on subsequent purchase of a specified quantity.
Discount cards/ coupons are actionable claims and therefore not subject to sales tax and sales tax on
goods shall be charged on discounted sales as the discount is in conformity with normal business practice.

Taxable Goods
All goods other than exempt goods. It means that all goods are subject to sales tax unless specifically exempt.
On the other hand, services are not all inclusive and only specified services are subject to sales tax under
provincial ordinances.

Supply
Supply means a sale or other transfer of the right to dispose of goods as owner including such sale or transfer
under a hire purchase agreement and also includes;
a) Putting to private business or non-business use of goods produced in the course of taxable activity for
purposes other than those of making a taxable supply (For Manufacturer only)
Explanation (Sale = Dispatch to customer),(Transfer of right to dispose = Gift), (Hire Purchase =
Installment basis), (Private Use = Drawing/taken to home), (Business use = use in office admin etc),
(Non Business use = Charity)
b) Auction or disposal of goods to satisfy adept owned by a person and
c) Possession of taxable goods held immediately before a person ceases to be a registered person
Mr. Kamran registered in Jan 2023 and later he got de registered-on 1st Sep 2023, he has 1000 units
in closing stock on 31st Aug 2023, these 1,000 units will be assumed as supply on deregister
d) Manufacture of goods belonging to another person the transfer of the goods to the owner or to a person
nominated by him
Co. A buys white cloth, later they contacted the Co B for printing on behalf of Co. A, this printing
by Co. B will be considered as supply (Toll Manufacturing)

Notes for students:


a) The term supply represents a ‘sale’ or ‘other transfer of right to ownership’. Thus, a
person making a ‘gift’ to his son would be regarded as a ‘supply’ although it will not be
subject to sales tax as there is no ‘taxable activity’ involved.
b) b) Since, under leasing arrangement, the title of goods continues to vest with the lessor,
there is no ‘supply’ unless the asset is transferred to the lessee on termination of lease.
c) Putting to personal use of goods manufactured constitutes a ‘supply’

Example 1:
The employees of the manufacturer of fridge are entitled to take a fridge for their home
consumption. This is putting to personal use of goods manufactured and therefore it is a supply
and subject to sales tax.
455 | P a g e
On the other hand, if a distributor of fridge provides a fridge to his employee under employment
package it will not constitute supply as the fridge is not being manufactured but procured.
Example 2:
A restaurant, operating on commercial basis, has the practice of supplying food to the needy
individuals free of charge. This constitutes a ‘supply’. On the contrary, if Mr. A purchases food
from a hotel and provides the same free of charge to the needy individuals is a supply but not a
taxable activity and therefore not subject to sales tax on the part of Mr. A.

e) Supply of taxable goods, through auction or otherwise, to satisfy a debt constitute a ‘supply’. A person
is not paying his income tax liability and the Commissioner disposes his taxable goods after attachment
to satisfy his debt. This constitutes a supply and subject to sales tax. Similarly, a bank can dispose of
any taxable goods to recover its loan or interest due by the registered person.

Supply chain
It means the series of transactions between buyers and sellers from the stage of first purchase or import to the
stage of final supply.

Importer
Means any person who imports any goods into Pakistan.

Manufacturer or Producer:
It means a person who engages in the production of goods
• whether or not the raw materials are owned by him and
• includes an assignee or trustee in bankruptcy, liquidator etc. or
• any manufacturer who disposes his assets in any fiduciary capacity.
Any person who holds or uses any patents, proprietary or other right to goods being manufactured shall also be
considered as manufacturer.

Toll Manufacturing
It shall be considered as manufacturing and subject to sales tax i.e. if a person is involved in providing services
such as textile printing, coloring, dying of raw materials of another person is considered as toll manufacturing
and sales tax shall be charged on conversion charges billed by the person as manufacturer. Another common
example is printing of books whether or not paper as raw material is owned by the printer or not. The principal
providing raw material for further processing is also considered as manufacturer.

Manufacture or Produce:
It includes:
a) Any process in which an article is converted into another distinct article
b) Any process in which an article, is so changed that it becomes capable of being put to use differently
c) Any process incidental to the completion of a manufactured product
d) Printing, publishing, lithography and engraving
e) Assembling, mixing, cutting, diluting, bottling, packaging, repacking.

Distributor:

456 | P a g e
Means a person appointed by a manufacturer, importer or any other person for a specified area to purchase
goods from him for further supply and includes a person who is also engaged in supply as a wholesaler or
retailer.
Note: It is essential to note that the distributor is ‘purchasing’ the goods and then supplying it. This means that he
is a person who is ‘buying’ and ‘selling’ goods of other persons. It means that if a distributor is working on
commission basis i.e. he is not purchasing goods then he is not liable to sales tax. In this case, the principal shall
directly charge sales tax from the customers

Wholesaler
It includes a dealer and means any person who carries on, whether regularly or otherwise the business of buying
and selling goods by wholesale or of supplying or distributing goods, directly or indirectly, by wholesale for
cash or deferred payment or for commission or other valuable consideration or stores such goods belonging to
others as an agent for the purpose of sale; and includes a person supplying taxable goods to a person who deducts
income tax at source under the Income Tax Ordinance, 2001.
Retail Price:
It means the price fixed by the manufacturer or importer in case of imported goods inclusive of all charges and
taxes excluding sales tax at which a particular variety of items should be sold to the general body of consumers
or, if more than one price is so fixed for the same variety, the highest of such prices.
The FBR may specify areas or zones for determination of highest retail price for any brand or variety of goods.
Note: The definition of retail price is important specially in the case of 3rd Schedule items.

Retailer:
It means a person supplying goods to general public for consumption provided that if he combines the business
of import and retail or manufacture with retail he shall notify and advertise wholesale price and retail price
separately and declare the address of his retail outlets.

Cottage industry
cottage industry” means a manufacturing concern, which fulfils each of following conditions,
(a) does not have an industrial gas or electricity connection;
(b) is located in a residential area;
(c) does not have a total labour force of more than ten workers; and
(d) annual turnover from all supplies does not exceed eight million rupees;

Company
(a) a company as defined in the Companies Act
(b) a body corporate formed by or under any law in force in Pakistan;
(c) a modaraba;
(d) a body incorporated by or under the law of a country outside Pakistan relating to incorporation of companies;
(e) a trust, a co-operative society or a finance society or any other society established or constituted by or under
any law for the time being in force; or
(f) a foreign association, whether incorporated or not, which the Board has, by general or special order, declared
to be a company for the purposes of the Income Tax Ordinance 2001

Tax Fraction (Formula):

457 | P a g e
% 𝑜𝑓 𝑡𝑎𝑥
% 𝑜𝑓 𝑡𝑎𝑥+ 100
𝑥 𝑣𝑎𝑙𝑢𝑒

Note: Where sales tax is not separately charged then the amount of tax is calculated on the basis of tax fraction
formula e.g. An invoice shows an amount of Rs.2,360 without identifying the amount of sales tax. Sales tax rate
is 18% and the amount of sales tax works out to Rs.360 (i.e. 2,360 x 18 / 118).

Tax Fraud
It means doing the following knowingly, dishonestly or fraudulently without any lawful excuse:
• An act or omission to take any action in contravention of law with the intention of:
• Understating tax liability for two consecutive tax periods
• Overstating tax credit or refund to cause loss of Government revenue
• Making taxable supply without getting registration
• Falsifying the sales tax invoices

Tax Invoice
A sales invoice, which includes the following particulars in Urdu or English language:
- Must be gapless serially numbered
- Name, address and registration number of recipient and in case of supplies by a manufacturer or
importer to unregistered distributor, the NIC or NTN of such unregistered distributor.
- Date of issue
- Quantity of goods and description including count, denier and construction in case of textile yarn and
fabric
- Value exclusive of tax
- Amount of sales tax
- Value inclusive of tax
- Not more than one tax invoice shall be issued for a taxable supply. A registered person may issue
invoices to another registered person electronically containing the above particulars.

Tax Period:
One month or such other period as the FBR may specify with the approval of Federal Minister-in- charge.

e-intermediary
Means a person appointed for filing of electronic returns and such other documents as may be prescribed by the
FBR on behalf of a registered person.

Open Market Price:


Consideration in money which that supply or similar supply would generally fetch in an open market.

Similar Supply
“Similar supply”, in relation to the open market price of goods, means any other supply of goods which closely
or substantially resembles the characteristics, quantity, components and materials of the aforementioned goods.

Active Taxpayers List (ATL)


“Active taxpayer” means a registered person who does not fall in any of the following categories:
a) Who is blacklisted or whose registration is suspended; and
458 | P a g e
b) Who fails to file:
- Sales tax return by the due date for two consecutive tax periods;
- Return of income
- Quarterly or annual withholding tax statement
Rules 12A and 12B of the Sales Tax Rules 2006
(1) A non-active taxpayer shall not be entitled to-
a) File Goods Declarations for import or export;
b) Issue sales tax invoices;
c) Claim input tax or refund; or
d) Avail any concession under the Act or rules made thereunder.

(2) No input tax can be claimed in case of purchase from any non-active taxpayer.

(3) A non-active taxpayer may be restored as active taxpayer, if -


a) The registered person files the return or statement along with payment of any tax due;
b) The RTO or LTU, on satisfying itself after conducting audit / investigation recommends to FBR for
restoration; and
c) The FBR issues an order to such effect.

Associates / associated persons


1. Two persons shall be associates where one may reasonably be expected to act in accordance with the
intentions of the other, or both persons may reasonably be expected to act in accordance with the
intentions of a third person.
2. One person sufficiently influences, either alone or together with associate, the other person i.e.
economic or financial dependency
3. One person enters into a transaction, directly or indirectly, with the other who is a resident of zero tax
area.
4. Two persons shall not be associates solely by reason of employment.
5. Following shall be treated as associates: *
a) An individual and his relative
b) Members of AOP
c) A member and his AOP where the individual either alone or together with his associates’ controls
50% or more share in profit or loss of AOP
d) A trust and its beneficiaries
e) A shareholder and the company where the shareholder either alone or together with his associates
controls 50% or more voting power or rights to dividend or capital.
f) Two companies where a person either alone or together with his associates’ controls in both the
companies 50% or more voting power or rights to dividend or capital
Definition of Relative:
"Relative" in relation to an individual, means
(a) An ancestor, a descendant of any of the grandparents, or an adopted child, of the individual, or of a
spouse of the individual; or
(b) A spouse of the individual or of any person specified in clause

Question
a) Sarfraz is a sole proprietor. Under the provisions of the Sales Tax Act, 1990, discuss whether each of
the following individuals/entities is an associate of Sarfraz:

459 | P a g e
(i) Jehanzeb: He has been working as an accountant for the past twenty years and has been responsible
for filing the tax returns of both Sarfraz and his business.
(ii) Falah Limited (FL): Sarfraz owns 20% shares in FL, while his mother owns 30% shares in FL.
(iii) Sarah: She is the adopted daughter of Sarfraz’s wife, Fatima. Sarah was adopted by Fatima before
her marriage to Sarfraz.
(iv) Umeed Trust (UT): Sarfraz is one of the trustees of UT. The trust was established to oversee the
operations of an orphanage. (06)

Solution
(i) Two persons cannot be associates solely by reason of the fact that one is employee
of another. Therefore, Jahanzeb is not an associate of Sarfraz.

(ii) A shareholder in a company and the company can be associates where the
shareholder either alone or together with an associate controls fifty percent or
more of the voting power in the company.
In the given scenario, Sarfraz together with his mother holds 50% shareholding
in FL so FL is the associates of Sarfraz.
(iii) Adopted child falls under the definition of relative. Therefore. Sarah is the
associates of Sarfraz.
(iv) Trust and any person who benefits or may benefit under the trust shall be
associates. In the given scenario, although the beneficiaries are orphans, UT may
be an associate of Sarfaraz, if he is deriving any benefit from the trust.

Time of Supply
This definition is very important as it determines as to when the sales tax incidence arises.
Time of supply depends upon different situations as under:

Situation Time of supply


Normal Time at which goods are delivered or made available to the
recipient of the supply
Supply through hire purchase agreement Date of agreement
Services the time at which the services are rendered or provided
(There will be no treatment of Advance, it will be ignored unless the goods are available or supplied)
If any part payment is received for a supply in a tax period
(a) it shall be accounted for in the return for that tax period; and
(b) In respect of exempt supply, it shall be accounted for in the return for the tax period during which the
exemption is withdrawn from such supply.

Question
a) Following are the independent transactions carried out by different enterprises:
(i) In November 2022, an agreement for the acquisition of machine on hire purchase was signed. In
December 2022, the machine was acquired under this agreement against 25% down payment of Rs. 30
460 | P a g e
million. The remaining balance is to be paid in 24 equal monthly installments of Rs. 4.5 million each.
(ii) In December 2022, an advance of Rs. 12 million was received against delivery of goods to be made in
March 2023.
(iii) A machine of Rs. 38 million was purchased in July 2022 but it was put to use in November 2022.
(iv) Goods of Rs. 7 million were sold in September 2022 but due to limited storage capacity at buyer’s
premises, the goods were delivered in January 2023.
Required:
In the light of the provisions of the Sales Tax Act, 1990 and Rules made thereunder, identify and discuss
the time (month) of supply for the chargeability of sales tax in respect of the above transactions. (05)

Solution

Value of Supply

Definition
Value of supply, in respect of taxable supply, is the consideration in money including all Federal and Provincial
duties which the supplier receives in respect of the supply excluding the amount of sales tax.

Normal Rule
The FBR has power to fix the value of any imported goods or taxable supplies.
Actual Value or FBR Value [higher will be value of Supply]
However, if the import or supply is made at a value higher than the value fixed by the FBR then the actual value
shall be considered.

Conditions to Claim Discount


Trade discount shall be excluded provided that
(i) the tax invoice shows the discounted price; and
(ii) (ii) the amount of discount is in conformity with the normal business practice.
The different situations are as under

461 | P a g e
Situation Value of supply
Normal case Consideration in money including all Federal and
Provincial duties which the supplier receives in respect
of the supply excluding amount of sales tax.

Mark up on credit sales shall not be taken into valueof


supply
Consideration is partly or fully in kind Open market price excluding sales tax

Sale is made on installment basis wherethe price Open market price excluding sales tax
includes mark up or surcharge [It means that mark up or surcharge included in credit
sales or installment sale is not subject to salestax]

Supply between associated persons Value as in the normal case or open market price
excluding sales tax whichever is higher

In case of imported goods excluding 3rd Schedule Value determined under the Customs Act including
items custom and excise duty levied thereon.

If there is reason to believe that the value is under Value determined by the Valuation Committee
declared in the tax invoice comprising representatives of trade and the sales tax
department
Manufacture of goods belonging to another person Actual consideration for value addition to such goods
i.e. toll manufacturing

Supply of electricity by an independent power Energy purchase price only.


producer or WAPDA
[The amount of capacity purchase price, energy
purchase price premium, excess bonus, supplement
charges etc. shall not be included in the value of
Supply]
Supply of electric and gas by a distribution Total amount billed including price of electricity or gas
company charges, rent, commission and all local, provincial and
federal duties and taxes but excluding:
- late payment surcharge
- sales tax; and
- any subsidy provided by the federal or
provincial government
Supply of used vehicles after value addition on Difference between sale price and purchase price
which sales tax has already been paid at the time of
import or
manufacturing

462 | P a g e
Taxable Supply with reference to retail tax The price fixed by the manufacturer or board

Question
Following are the independent transactions carried out by different enterprises during the month of
February 2022:
(i) Taxable goods of Rs. 800,000 were sold to one of the dealers. The amount was net of 20% trade
discount which was in accordance with market norms. The discounted price was not shown on the
tax invoice.
(ii) Taxable goods of Rs. 1,500,000 were used for internal testing and evaluation purposes. 40% of
these goods were locally procured while remaining 60% of these goods were own manufactured.
(iii) Advance of Rs. 600,000 was received for goods to be delivered in April 2022.
(iv) 1,000 units of taxable goods listed in the Third Schedule were sold at a unit price of Rs. 5,000.
Retail price of each unit was Rs. 6,000.
(v) New parts of Rs. 1,200,000 were issued free of cost to replace the defective parts under warranty.
(vi) Taxable goods of Rs. 400,000 were sold at credit terms of 2/10, n/30. Customer paid the amount
within ten days and availed the discount.
Required:
In the light of the provisions of the Sales Tax Act, 1990 and Rules made thereunder, state the value of
supply chargeable to tax for the month of February 2022. Also state the reason for your treatment. (08)
Solution
Value of Supply Reason
1 1,000,000 Discount can be claimed if it is as per market norms and has been shown on
(800,000 ÷ 80%) tax invoice. Since the amount of discount has not been shown on tax invoice,
it shall be chargeable to tax at gross amount.
2 900,000 Use of own manufactured items for in-house consumption will be subject to
(1,500,000×60%) sales tax. However, goods locally procured is not deemed to be supply.
3 Nil Time of supply is the time at which goods are delivered or make available to
the recipient. Since goods were not delivered in February, this was not
chargeable to tax in the month of February.
4 6,000,000 For taxable supplies specified in third schedule, sales tax is charged on the
(1000×6,000) retail price of goods.
5 Nil Free replacement of defective parts is considered as original supply and not a
separate supply so this was not chargeable to tax in February return.
6 400,000 Cash discount shall not be deducted while computing value of supply, so
gross amount shall be chargeable to tax.

463 | P a g e
CHAPTER 22
SALES TAX PROVSIONS
Basic concepts including VAT
Overview of Sales Tax System
Sales tax is a Value Added Tax (VAT) system. It is an indirect tax collectable from the whole supply chain i.e.
importers, manufacturers, wholesalers (including dealers and distributors) and retailers with certain exceptions.
Therefore, the sales tax is a multi-stage tax payable on the value of:
- Taxable supplies by a registered person in respect of any taxable activity carried on by him;
- Goods imported into Pakistan; and
- Specified taxable services

VAT is a percentage tax levied on the price each registered person charges for goods supplied or taxable services
rendered by him.
VAT normally utilizes a system of tax credit (called input tax adjustment) to place the ultimate and real burden
of tax on the final consumer and to relieve the intermediaries (i.e. the persons other than the final consumer)
from any tax burden.

Sales tax rates and who is liable to pay sales tax


Tax Rates
Normal Sales tax rate is 18%, it means when we supply or buy goods 18% tax will be charged

How Sales Tax Mechanism works


It is assumed in this example that every person in the supply chain is a registered person for sales tax purpose
and subject to sales tax @ 18% (restriction on input tax and special provisions for commercial importers and
retailers are not considered for this example):
S. Input Output Pay to
No. Transactions Tax Tax FBR
1 Importer’s import value Rs.9,200 1,656 1,656
Importer sells to a wholesaler of raw materials forRs.11,000 1,980
+ 1,980 sales tax 1,656 324
= margin is Rs.1,800
2 Wholesaler of raw materials buys at Rs.11,000 + 1,980 input
tax and sells to a manufacturer for Rs.11,600 + 2,088 sales 1,980 2,088 108
tax
= margin is Rs.600
3 Manufacturer buys at Rs.11,600 + 2,088 input tax (his other
manufacturing expenses are Rs.9,000) and sells to a
wholesaler of finished product at Rs.23,600 + 4,248 sales 2,088 4,248 2,160
tax
= margin is Rs.3,000

464 | P a g e
4 Wholesaler of finished product buys at Rs.23,600 + 4,248
input tax and sells to a retailer at Rs.24,000 + 4,320 salestax 4,248 4,320 72
= margin is Rs.400

5 Retailer buys at Rs.24,000 + 4,320 input tax and sells to


consumer at Rs.24,600 + 4,428 sales tax 4,320 4,428 108
= margin is Rs.600
TOTAL 4,428

In VAT system every person in a supply chain is supposed to be a registered person but it is very difficult in
Pakistan due to certain problems e.g. chips manufacturer may be a company for which registration, record
keeping, input-output adjustment etc. are not a big issue but a chips manufacturer may be an individual running
a small bakery who cannot be expected to comply all such legal requirements.
Likewise, every retailer in Pakistan is not expected to comply with all the legal requirements.
Therefore, a structure has been developed in Pakistan whereby two types of exemptions have been given as
under:
• Turnover based exemption i.e. small manufacturers termed as cottage industry and retailers (other than
specified retailers i.e. Tier 1 retailers) are exempt from registration and they do not charge sales tax on
their supplies; and
• Items based exemption i.e. certain products are exempt without any turnover limit e.g. books,
newspapers, locally manufactured computers and laptops
The following chart explains the situation

Importer Registration is required.


Importer shall pay sales tax on import stage and
subsequently charge sales tax on value of taxable supply
However, commercial importers shall pay sales tax as per
relevant provisions
Wholesaler / Distributor Registration is required and sales tax shall be chargedon
value of taxable supply
Retailer Only Tier 1 retailers (i.e. specified retailers) are requiredto
be registered.
Manufacturer
Cottage industry Exempt as per serial 3, Table 2, 6th Schedule
Other than cottage industry Registration is required and sales tax shall be chargedon
value of taxable supply

Further Tax
Further tax @ 4% shall also be charged when the goods are supplied to unregistered persons or inactive
taxpayers.
It means that the tax rate in this case is 18% + 4%.
However, further tax shall not be charged in the following cases [Exceptions]
i. Supplies to Government, semi-government and statutory regulatory bodies
ii. Supply of goods directly to end consumers including supplies by a retailer
iii. Items falling under 3rd Schedule i.e., items on which sales tax is chargeable on retail price
iv. Electric supplied to domestic and agricultural consumers
465 | P a g e
v. Natural gas supplied to domestic consumers and CNG stations
vi. Supply of second-hand worn clothing and other worn articles
vii. Goods falling under zero rating;
viii. Foam products including spring mattresses; and
ix. White crystalline sugar, fertilizers, jet fuel etc.

Further tax shall not become part of output tax which means that further tax is payable to the FBR as a bottom-
line figure.

“The FBR has power to fix a lower or higher rate on specified items”
Example of higher rate on imports
25% sales tax rate is applicable on import and supply of various items such as:
Juices Vehicles in CBU condition Chocolates
Cigarettes Tissue papers Shampoos
Jewelry

Example of higher rate on supply of locally manufactured goods


25% sales tax rate is applicable on the following locally manufactured goods:
• Vehicles of 1400 cc and above
• Double cabin picks up vehicles

Reduced rates
8th Schedule specifies import or supply of certain goods on which sales tax is chargeable at reduced rates subject
to certain conditions. Few examples are:
• 1% on locally manufactured electric vehicles such as electric buses, electric rickshaw and electric
motorcycle
• 12.5% on locally manufactured or assembled cars of up to 850cc
• 8.5% on locally manufactured Hybrid Electric Vehicles up to 1800 cc
• 12.5% on electric vehicle in CBU (completely built unit) condition of up to 50 kwh battery
• 1% on manufacture or import of drugs under the Drugs Act, 1976 including raw materials for the
production of pharmaceutical products.
• 3% on supply of locally manufactured jewelry articles of precious metal
• 5% on import of computers, laptop and notebook in CBU (completely built unit) condition

Capacity Tax
The FBR has authority to levy and collect sales tax on fixed basis or on the basis of capacity of plant in lieu of
sales tax on the basis of value of supply of goods [may also be called as capacity tax].
The tax shall be levied and collected, in the mode and manner specified therein on-:
a. production capacity of plants, machinery, undertaking, establishments or installations producing or
manufacturing such goods; or
b. fixed basis, as it may deem fit, from any person who is in a position to collect such tax due to the nature of
the business.

Tax on steel products/ship plates


In respect of goods, specified in the Thirteenth Schedule, the minimum production for a month shall be
determined on the basis of a single or more inputs as consumed in the production process as per criterion
466 | P a g e
specified in the Thirteenth Schedule and if minimum production so determined exceeds the actual supplies for
the month, such minimum production shall be treated as quantity supplied during the month and the liability to
pay tax shall be discharged accordingly.

Tax on supply to CNG stations


▪ In case of supply of natural gas to CNG stations, the Gas Transmission and Distribution Company shall
charge sales tax from the CNG stations at the rate of 18% on the value of supply to the CNG consumers.
▪ Value for the purpose of levy of sales tax shall include price of natural gas, charges, rents, commissions
and all local, provincial and Federal duties and taxes but excluding the amount of sales tax

Who is liable to pay sales tax?


Liability to pay the sales tax to the sales tax department shall be of the person:
• making the supply, in the case of supply of goods [It means that the purchaser, who pays sales tax, does
not pay sales tax to FBR instead he pays sales tax to the supplier and the supplier pays sales tax to FBR
after making his input tax adjustment];
• importing the goods, in the case of goods imported into Pakistan; and
• providing taxable services.
However, the FBR may specify the goods in respect of which liability to pay sales tax to FBR shall be of the
person receiving the supply.

When to pay sales tax


Sales tax shall be paid at the time of:
- payment of custom duty in the case of import of goods; and
- filing of sales tax returns in the case of supplies made or services provided in Pakistan

Change in the tax rates


1.Taxable supply in Pakistan shall be charged to tax at such rate as is in force at the time of supply

2.Import of goods shall be charged at such rate as is in force at the time of declaration is presented whether for
home consumption or for clearance from warehouse as the case may be except:
a) Where goods declaration is presented in advance of the arrival of conveyance, the tax shall be charged at the
rate as is in force on the date of the manifest of the conveyance is delivered.
b) In case of clearance from warehouse if the tax is not paid within 7 days of presenting the declaration, the tax
shall be charged at the rate in force on the date of actual payment.

Zero Rated Supplies – [5th schedule]

a) Goods falling under this category are chargeable to sales tax at 0%. It means that their output tax is 0%
however, their corresponding purchases may not necessarily zero rated and therefore, input tax would be
suffered which is reclaimable as input tax.

Export of goods falls under this category other than the following [Exception to Zero Rating]
• Export to any country as notified by the Federal Government [examples are: export to Afghanistan, Iran
or China; and (ii) export to Afghanistan by land route]
• Export intended to be re-imported into Pakistan

467 | P a g e
• Goods held for export but not exported

Examples of other items under this category:


• Supply to diplomats, diplomatic missions and privileged persons
• Supply of raw materials and components for further manufacture of goods in Export Processing Zone
(EPZ)
• Imports or supplies made to Gwadar Special Economic Zone excluding vehicle
• Supply of raw materials and machinery to registered exporters under Export Facilitation Scheme, 2021
or any other government scheme for this purpose
• Supply of stores and provisions for consumption aboard conveyance proceeding outside Pakistan e.g.
international flight or ship
• Packing materials used for zero rated supplies
• Electric and gas consumed by manufacturer-exporters
• other specified items subject to certain conditions including pencils, pens, milk etc.

b) Refund of input tax on zero rated supply


i. Refund of input tax on zero rated supplies shall be made within 45 days of filing of return.
ii. If a registered person is liable to pay any tax, default surcharge or penalty payable under any law
administered by the FBR, the refund of input tax shall be made after adjustment of unpaid outstanding
amount of tax, default surcharge and penalty.
iii. Where there is reason to believe that a person has claimed incorrect input tax credit or refund, the
proceedings against him shall be completed within 60 days may be extended up to 120 days by an
officer not below the rank of an Additional Commissioner and may be extended by the Board up to 9
months for reasons to be recorded in written.
iv. In case of delayed refund, the FBR shall pay an additional amount to the registered person @ Karachi
Inter-bank Offered Rate (KIBOR) per annum if there is no dispute in the claim of the refund.

Note for students:


Additional amount, if any, received on delayed refund from the sales tax department istaxable for income tax purpose
under the head “income from other sources”.

Exempt Supplies [6th schedule]


a) Certain imports and supplies of goods falling under this category are outside the scope of sales tax and
therefore not subject to sales tax.

b) An example is publication of books and newspapers where paper purchases suffer sales tax but their supply
does not and in this case input tax cannot be reclaimed.

Other important exempt items are:


1) Local supply of live 2) Agricultural produce 3) Holy Quran and other
animals and live poultry not subject to any holy books or recorded
[sales tax shall be paid on further manufacture in audio or video
import stage] cassettes
4) Goods imported by 5) Goods, excluding 6) Goods temporarily
diplomats, diplomatic electricity and natural imported into Pakistan

468 | P a g e
missions or privileged gas, supplied to meant for subsequent
persons hospitals run by export
charitable hospitals of
50 or more beds
7) Goods manufactured and 8) Fertilizers 9) Tractors
exported from Pakistan
which are subsequently
imported in Pakistan
within one year
10) Local supply of breads, 11) Machinery, equipment 12) Import of raw materials
nans and chapattis and materials imported and machinery by
for use in EPZ registered exporters
authorized under Export
Facilitation Scheme,
2021 or any other
government scheme for
this purpose
13) Supply of fixed assets 14) Specified goods
otherwise than stock in including locally
trade against which input manufactured laptops,
tax adjustment is not computers and
available e.g. resale of notebooks and import of
vehicles, furniture or parts for manufacturing
office equipment being a of laptops, computers
depreciable asset and notebooks

c) Difference between zero rated supplies and exempt supplies:


No output tax shall be charged and collected on both zero rated and exempt supplies but input tax, if leviable,
can be reclaimed only in respect of zero rated supplies.

Differences between zero rated supplies and exempt supplies in detail are as under:
Zero rated supply Exempt supply
Definition “Zero rated supply” means a taxable “Exempt supply” means a supply which is
supply which is chargeable to sales tax at not chargeable to sales tax.
0%.

Goods Goods exported or goods listedin 5th Goods specified by FBR through
Schedule notifications and goods listed in 6th
Schedule
Invoice Tax invoice shall be raised butsales tax No sales tax invoice is required
shall be charged at 0%

Input tax credit Input tax on zero rated supplies is Input tax on exempt supplies is not
refundable from FBR adjustable nor refundable

469 | P a g e
Registration Sales tax registration is required where a Sales tax registration is not required where a
person wants to claim refund person is engaged exclusively in exempt
Supplies

Third Schedule [Retail Item]


Sales tax is charged @ 18% (or at a reduced rate as specified in 8th Schedule) in respect of goods falling under
this category on the recommended retail price which shall be legibly printed on the label etc along with the
amount of sales tax.
After charging / paying such sales tax, the same amount of sales tax will be charged on subsequent supply.

Retail Price:
It means the price fixed by the manufacturer or importer in case of imported goods inclusive of all charges and
taxes excluding sales tax at which a particular variety of items should be sold to the general body of consumers
or, if more than one price is so fixed for the same variety, the highest of such prices.
The FBR may specify areas or zones for determination of highest retail price for any brand or variety of goods.
Note: The definition of retail price is important specially in the case of 3rd Schedule items.

Retailer:

It means a person supplying goods to general public for consumption provided that if he combines the business
of import and retail or manufacture with retail he shall notify and advertise wholesale price and retail price
separately and declare the address of his retail outlets.
Retailers are divided into 2 categories
1) Tier -1 Retailer (Specified Retailer)
2) Other than Specified retailers

1. Tier 1 retailers i.e. specified retailers are:


(a) a retailer operating as a unit of a national or international chain of stores;
(b) a retailer operating in an air-conditioned shopping mall, plaza or centre, excluding kiosks;
(c) a retailer whose cumulative electricity bill during the immediately preceding 12 months exceeds
Rs.1,200,000;
(d) a wholesaler-cum-retailer, engaged in bulk import and supply of consumer goods on wholesale basis to
the retailers as well as on retail basis to the general body of the consumers;
(e) a retailer who has acquired point of sale accepting payment through debit or credit cards or any other
digital payment service;
(f) a retailer whose deductible withholding tax under sections 236G or 236H of the Income Tax Ordinance,
2001 during the immediately preceding 12 consecutive months has exceeded the threshold as may be specified
by the FBR;
(g) any other person as prescribed by the FBR.

Tier 1 retailers [i.e. specified retailers]


Tier 1 retailers are required to be registered and all the provisions shall apply in the normal manner including
charge of sales tax, filing of monthly return, input tax adjustment / apportionment, debit / credit note, audit and
so on.

470 | P a g e
All Tier-1 retailers shall integrate their retail outlets with FBR’s computerized system for real time reporting of
sales. In case of default, input tax claim would be reduced by 60%.

Service charges by Tier – 1 retailers


Service charge @ Re.1 per invoice shall be collected by Tier – 1 retailers integrated with the FBR and the said
service charge shall be deposited along with the filing of monthly sales tax return without adjustment of any
input tax against the said service charge.

Retailers other than Tier 1 retailers


Retailers other than Tier 1 retailers are not required to be registered and they shall pay sales tax with their
monthly electric bills as under:
- 5% where the monthly bill does not exceed Rs.20,000; and
- 7.5% where the monthly bill exceeds Rs.20,000.
The above sales tax is the final discharge of their sales tax liability and they are not allowed to claim input tax
adjustment. Monthly sales tax return is not required to be filed and they are not subject to audit.
The above sales tax in case of unregistered retailer with electric bill is in addition to the sales tax otherwise
chargeable with electric bills which is 18% normal sales tax + 4% further tax + extra tax at specified rates..

Items covered under 3rd Schedule are


• Cigarettes
• Juices, ice cream, syrups, aerated water and beverages
• Mineral / bottled water
• Detergents, Shampoo, soap, toothpaste, shaving cream, cosmetics, shoe polish / cream
• Tea, powder drinks, milky drinks
• Toilet paper and tissue paper
• Spices sold in retail packing with brand name and trade mark
• Cement sold in retail packing
• Household electrical goods, including ACs, refrigerators, deep freezers, TV, recorders and players,
bulbs, tube-lights, electric fans and irons, washing machines and telephone sets.
• Household gas appliances, including cooking range, ovens, geysers and gas heaters
• Foam products including spring mattresses.
• Paints, lubricating oils, brake fluids etc. sold in retail packing
• Auto-parts in retail packing, storage batteries, tyres and tubes excluding those sold to automotive
manufacturers or assemblers
• Motorcycles and Auto rickshaws
• Biscuits in retail packing with brand name
• Tiles
Note for students: Further tax is not applicable on 3rd Schedule items, and no discount is allowed

471 | P a g e
Federal Government has directed to charge sales tax @ 25% on import and subsequent supply of following third
schedule goods

Adjustment of input tax


a) A registered person is entitled to deduct his input tax during the tax period for the purpose of taxable supplies
made or to be made (e.g. stocks not yet sold) from his output tax liability and for this purpose he must hold:

I. tax invoice in his name bearing his NTN or in case of supply of electricity or gas, a bill bearing his
registration number and the address where the connection is installed;
II. goods declaration (i.e. bill of entry) in case of goods imported by him; or
III. in case of goods purchased in auction, treasury challan in his name bearing his NTN

Input tax can be claimed on accrual basis subject to payment within a prescribed period (i.e 6 tax periods)

Input tax paid (not on accrual basis) with electric and gas bills can be claimed by a registered consumer and in
this case the gas or electric bill shall be regarded as tax invoice for sales tax purposes provided the bill contains
NTN and address of the business premises declared to the Commissioner of such consumer.

Where the electric or gas connection is not in the name of such person, NTN of such person is mentioned on
such bill along with the address of such person as given by him in the application for registration for the purpose
of sales tax.

Question
Please explain whether sales tax can be claimed in respect of sales tax paid on electricity billswhich are
not in the name of the company being a tenant. (Marks 3)

b) Where a registered person did not deduct input tax within the relevant period, he may claim such input tax in
the return for any of the next 6 tax periods.
Alternatively, the registered person may apply for the refund within one year u/s 66. The Commissioner has
power to grant extension of these time limits in special cases.

Question
472 | P a g e
Explain the procedure for the admissibility of the input tax which is not claimed by omission inthe
relevant tax period. (Marks 5)

c) In the following cases a registered person is not entitled to reclaim/ deduct his input tax:
i. Supply of exempt goods and services
ii. Goods and services not related to taxable supplies or acquired for personal or non-business use e.g.,
tissue papers purchased by a manufacturer for his own use. In this case he will be a direct consumer of such
goods who cannot reclaim input tax.
iii. Sales tax on services in respect of which input tax adjustment is barred under the respective provincial
sales tax laws.
iv. Input tax on fake invoices
v. Goods in respect of which sales tax has not been deposited into the government treasury by the supplier
vi. Purchases in respect of which a discrepancy is indicated by the CREST or input tax of which is not
verifiable in the supply chain. CREST (i.e. COMPUTERISED RISK-BASED EVALUATION of SALES TAX)
is the computerized program of the sales tax department for analyzing and cross-matching of sales tax returns.
vii. Goods and services, may be specified by the FBR, which have not been declared by the supplier in his
return at the time of filing of return by the buyer

Note for students:

Input tax is not allowed where a discrepancy is indicated by the automated system of FBR
i.e. computerized system for cross-matching of input tax

However, a discrepancy would be allowed on provisional basis and the registered person would be
advised by the FBR to contact the supplier and persuade him to disclose his relevant output tax.

If the supplier did not declare his relevant output tax in the next tax period then input tax allowed
earlier provisionally would be adjusted or recovered.

Important point: If a registered person does not have tax invoice then provisional adjustment would
not be allowed as existence of tax invoice bearing his name and NTN isa basic condition for claiming
input tax.

It means that if a registered person has tax invoice for his purchases and a discrepancy is indicated by
the computerized system then provisional adjustment of input tax would be allowed.

viii. Input tax paid on purchases if he fails to furnish the information required by the FBR
ix. Extra tax paid cannot be adjusted as input tax
x. Vehicles other than stock in trade
[Fork lifting vehicle is treated as machinery as per appellate authorities’ decision and therefore input tax can be claimed
on fork lifting vehicle]
xi. Building material including cement, paints, electric and gas appliances, pipes sanitary fitting etc. otherwise
than stock in trade. However, input tax can be claimed on pre-fabricated buildings.
xii. Input tax related to supplies of goods and services to unregistered distributor for which sales invoices do
not bear NIC or NTN of the recipient.
xiii. Any goods which the FBR may specify. The FBR has specified the following goods acquired otherwise
than stock in trade by a registered person in respect of which input tax shall not be reclaimed:
• Food, beverages, garments and consumption on entertainment
473 | P a g e
• Gifts and giveaways including dairies and calendars
• Supply of electricity and gas to residential colonies
• Office equipment (excluding electronic fiscal cash registers), furniture and fixtures
• Crockery, cutlery etc.
Question

The Sales Tax Act, 1990 specifies the principle for determining the tax liability whereby the eligible
input tax is deducted from the output tax on taxable supplies of a registered person.
Does the law specify any departure from the said principle relating to the levy of tax on taxable supplies?
Please discuss by narrating the relevant provisions of law. (Marks 6)

Question
A registered person is entitled to deduct input tax paid or payable during the tax period from the output tax
subject to compliance of section 7 and 73 of the Sales Tax Act. Section 8 of the Act however places certain
restrictions on goods on which input tax is not deductible.
You are required to specify those goods on which input tax is not allowable under section 8.(Marks 8)

Payment through banking channel


Payment for a transaction exceeding value of Rs.50,000 (other than utility bills) must be –
• made by a crossed banking instrument
• made within 180 days of the date of the issuance of the tax invoice [this period may be extended by the
Commissioner on any reasonable ground]
• from the business bank account of the buyer to the business bank account of the supplier

If above conditions are not met –


• The buyer would not be allowed any input tax credit, zero rating etc.
• The supplier will not be allowed input tax credit, zero rating etc. if the amount received on account of
supply is not deposited in his business bank account already declared to the sales tax department

Input tax can be claimed on accrual basis. However, payment through banking channel is required within 180
days. If payment is made in cash or the payment is not made within 180 days then input tax adjustment earlier
made would be reversed.

On-line transfer of payment as well as payments through credit card is also allowed.
Explanation. — “business bank account” shall mean a bank account utilized by the registered person for
business transactions, declared to the concerned Commissioner through Form STR 1 or change of particulars
in registration database.

Note: this section is not applicable on registered person supplying goods to unregistered person. However,
supplier is required to deposit cash in his business bank account to claim input tax.

Adjustment of payment against receivable


If payment is adjusted against any amount receivable from the same party then payment shall be deemed to have
been made subject to the following conditions:
- Sales tax shall be charged and paid by both parties, wherever applicable; and
- Approval of the Commissioner for such adjustment is required.

474 | P a g e
Limit on supplies to unregistered person
A registered person shall make all taxable supplies to registered persons excluding supplies not exceeding
Rs.100 million in a financial year and Rs.10 million in a month, failing which the supplier shall not be entitled
to claim input tax adjustment as attributable to such excess supplies to unregistered person.

The provisions of section 73(4) shall not apply to supplies made to:
(a) Government departments, authorities, etc. not engaged in making of taxable supplies;
(b) Foreign Missions, diplomats and privileged persons;
(c) All other persons not engaged in supply of taxable goods.

Moreover, it is clarified by the FBR that the limit of Rs.10 million per month / Rs.100 million per year is
applicable on goods supplied to one specific person.

Apportionment of Input Tax Rules


These rules apply to the registered person supplying taxable and exempt goods simultaneously. Input tax relating
wholly to taxable supplies is fully adjustable / reclaimable. If it relates wholly to exempt supplies it is not
admissible.
Sales tax on goods and services including utilities used for both taxable and exempt supplies shall be apportioned
according to the following formula:
Residual input tax credit =
Value of taxable supplies x Residual input tax
Value of taxable + exempt supplies
Residual input tax is sales tax on goods and services being used for taxable as well as exempt supplies but does
not include sales tax paid related wholly to taxable supplies or wholly to exempt supplies.
Monthly apportionment of input tax shall be treated as provisional adjustment and at the end of each financial
year the registered person shall make final adjustment on the basis of taxable and exempt supplies of that year.

Note for students: According to the above rule, apportionment formula is applied where inputtax is common for
two categories i.e. taxable supplies and exempt supplies.

However, in practice there are three categories i.e. taxable at normal rate, taxable at zero rateand exempt
supplies. Zero rate supplies is a separate category due to the following reasons:
• a separate refund application is processed for zero rated supplies; and
• Restriction on input tax is not applicable for zero rated supplies.

475 | P a g e
Apportionment Table

Value of Supply Input Tax Input tax on Fixed Asset

Taxable Supplies [Net] XXX [e] aaa bbb

Zero rates Supplies [Net] XXX [f] aaa’ bbb’

Exempt Supplies [Net] XXX [g] aaa’’ bbb’’

Total XXX [Y] AAA [Q] BBB [W]

𝑄 𝑄 𝑄 𝑊 𝑊 𝑊
∗ 𝑒 = 𝑎𝑎𝑎 ∗ 𝑓 = 𝑎𝑎𝑎’ ∗ 𝑔 = 𝑎𝑎𝑎’’ ∗ 𝑒 = 𝑏𝑏𝑏 ∗ 𝑓 = 𝑏𝑏𝑏’ ∗ 𝑔 = 𝑏𝑏𝑏’’
𝑌 𝑌 𝑌 𝑌 𝑌 𝑌

For Input Tax Against Supplies For Input Tax Against Fixed Assets

Restriction on input tax


A registered person other shall not be allowed to adjust input tax in excess of 90% of the output tax for a
particular tax period. Therefore, in case of lower profit margin he is required to pay 10% of his output tax to
FBR.

It means that if his input tax during a tax period exceeds his output tax as a result of loss or overbuying (closing
stock), he is not entitled to get refund instead he will pay 10% of his output tax to FBR.
Input tax disallowed due to this restriction shall be carried forward to the next period and shall be treated as
input tax of that period.
Input tax on acquisition of fixed assets or capital goods, if any, is claimable in the same tax period and restriction
of the said 90% is not applicable in this case.
FBR has power to increase the limit from 90% to 95% in any particular case including all Tier-1 retailers who
have integrated all their point of sales (POS) with the FBR.
Exceptions:
This restriction of 90% is not applicable in the following cases i.e. they can adjust input tax from output tax
without any restriction:
1. Persons registered in electrical energy sector and gas distribution companies
2. Oil marketing companies, petroleum refineries and Pakistan Steel Mills
3. Distributors
4. Commercial importers provided the value of imports subjected to 3% value addition tax exceeds 50%
of value of all taxable purchases in a tax period.
5. Persons making zero rated supplies provided value of such supplies exceeds 50% of value of all taxable
supplies during a tax period.
6. Registered persons other than manufacturers, making supplies of 3rd Schedule items on which sales tax
has been paid on retail price, provided the value of such supplies exceeds 80% of all taxable supplies
7. CNG dealers and petroleum dealers
8. Telecommunications

Note for students:


This restriction u/s 8B is not applicable in case of zero-rated supply and commercial imports. The above
exceptions shall apply for the local purchase / supply where the conditions above mentioned are fulfilled.

Excess input tax

476 | P a g e
Input tax disallowed due to restriction u/s 8B or excess input tax where the said restriction is not applicable may
be carried forward to the next tax period and treated as input tax of that period.

Note for students:


Refund of zero-rated items including exports may be claimed at the time of filing of return and need not be
carried forward.

Debit and Credit Note and Destruction of Goods


(a) Where a registered person has issued a tax invoice and the tax return or tax invoice needs to be modified
as a result of:
• cancellation of supply;
• return of goods;
• change in the nature of supply;
• change in the value of supply; or
• any other such event
Within 180 days then the registered person may issue a debit / credit note indicating specified information and
adjust accordingly. [Period of 180 days may be extended for any special reason]
Note for students:
Where tax liability increases as a result of issuance of debit note then the time limit of 180 days shall not apply.

EXAMPLE:
Mr. A, a registered person, supplied goods of Rs.100,000 to Mr. B who is also a registered person and received
Rs.118,000 from Mr. B (including sales tax of Rs.18,000). Goods returned to Mr. A. Mr. B will now issue a
debit note.
Mr. A: Mr. A has already received Rs.18,000 from Mr. B and paid to FBR as his output tax. Now he will pay
back Rs.18,000 to Mr. B and reclaim this amount from FBR.
In the case, Mr. A is allowed to deduct Rs.18,000 from his output tax.

Mr. B: Mr. B has already paid Rs.18,000 to Mr. A and reclaimed this amount from FBR as his input tax. Now
he will receive Rs.18,000 from Mr. A and he is required to pay the said amount to FBR.
In this case, Mr. B is required to deduct Rs.18,000 from his input tax.

(b) The debit / credit note shall show the following particulars:
i. Name and NTN of the recipient and the supplier
ii. Number and date of the original sales tax invoice including quantity, value and the amount of sales tax
iii. The reason of issuance of the note
iv. Signature and seal of the authorized person issuing the note
v. Quantity being returned or the supply of which has been cancelled (in case of return of goods or
cancellation of supply)
vi. Original, revised and difference in the value and sales tax (in case of change of value)

(c) Where the buyer and supplier both are registered persons and sales tax liability is reduced as a
consequence of credit note then the adjustment is allowed only where the other party accepts the credit note by
issuing corresponding debit note.
However, if a corresponding debit note is not issued by the other party then provisional adjustment would be
allowed to the registered person by the automated system of FBR and he would be advised by the FBR to contact
and persuade the other party to issue corresponding debit note. If it is not done in the next tax period then input
tax allowed earlier provisionally would be adjusted or recovered.

(d)If the other party is unregistered then adjustment shall be made only by the registered person on the basis of
credit / debit note issued by him.

477 | P a g e
(e)Where such goods are subsequently supplied then sales tax shall be charged in the normal manner.

Goods need to be destroyed:


Where such goods are returned by the buyer on the ground that the same are unfit for consumption and are
required to be destroyed then the same shall be destroyed under the supervision of the sales tax department and
the input tax credit in respect of goods so destroyed shall not be admissible.

However, in case of companies manufacturing perishable food items having an expiry date, if such items are
returned on account of being unfit for consumption and are then destroyed, the credit note may be issued with
15 days of the return of goods and adjustment may be made accordingly.

Supply to unregistered person


The Federal Government has power to specify any goods which cannot be supplied by a registered person to
any unregistered person.
It means that if a registered person makes such supplies then he shall not be allowed to take credit of input tax.
The specified goods that cannot be supplied by a registered person to an unregistered person:
i. Polypropylene granules;
ii. Artificial filament tow;
iii. Filter rods for cigarettes; and
iv. Air-conditioning, chilling and humidification plants, cranes, propane storage tank, heat exchanger and
gas separator.

Purchase of Stocks before Registration


Purchaser of stocks who has paid sales tax on such goods is required to apply for registration within 30 days of
such purchases if he wants to take credit of his input tax provided that he holds tax invoice and such goods
constitute verifiable unsold stock on the date of registration.
This period is 90 days in case of imports.

Commercial importers
A commercial importer shall pay sales tax @ 18% on import value in the normal manner.
However, sales tax on account of minimum value addition shall be collected at import stage @ 3% of the value
of goods imported in addition to the sales tax paid in the normal manner.
The above concept of minimum value addition is not applicable on import of raw materials and fixed assets
imported by a manufacturer for in-house consumption and few specified goods.

The commercial importer shall charge sales tax @ 18% from his customers. The value addition tax paid at
import stage shall form part of input tax and claimable against output tax for determining his net liability.
The excess of input tax, if any, over output tax shall be carried forward to the next tax period.
However, the refund of excess input tax over output tax in respect of such commercial imports shall not be
allowed to a registered person.

Excess Tax Collection


If any person collected output tax which was not collectable or collected excess output tax by mistake and the
incidence of such tax has been passed on to the consumer, he shall pay the amount of such tax to the Government
and no claim for refund in respect of such amount shall be admissible.

The burden of proof that the incidence of tax has not been passed to the consumer shall be on the person
collecting the tax.

478 | P a g e
Question
DEF Ltd supplied goods valuing Rs.1,000,000 to one of its distributors, Mr. Pink who is also registered
for sales tax purposes. Sales tax invoice was issued for the said amount plus sales tax at the prescribed
rate. The transaction was recorded in the monthly sales tax return for the month of January 2007.

In February 2007 the internal auditor of the Company observed that the accountant had applied
incorrect selling prices. As a result, the Company will have to refund the excess amount to Mr. Pink.

Required:

Advise the procedure to be adopted by the Company and the distributor to adjust the excess
amount of sales tax. (Marks 5)

Joint & several liability of registered person in supply chain where tax unpaid.
A registered person receiving taxable supply from another registered person is in the knowledge or has
reasonable grounds to suspect that some or all tax payable in respect of that supply or any previous or subsequent
supply of the goods would remain unpaid [burden to prove this fact is on the tax department], then such person,
as well as the person making the taxable supply shall be jointly and severally liable for payment of such unpaid
tax.
However, FBR may exempt any transaction from the provisions of this section.

Q.9 June 2008 ICAP CFAP


Zohaib & Co., a partnership firm, plans to purchase raw material from Mr. SA who has the reputation of evading
sales tax. Mr. K, the managing partner of the firm, however is of the view that failure to deposit sales tax by SA
would not have any bearing for his firm.
Required:
Offer your comments on the views expressed by Mr. K. (Marks 3)

Supply of parts free of cost under warranty


(a) Price charged to the customers for the vehicle includes the cost of warranty period related claims.
(b) The FBR also clarified that in such cases, where the final product is taxable, no sales tax is payable
except on supply of final product.
Therefore, no sales tax should be charged on replacement, free of charge, of the defective parts under warranty.

479 | P a g e
CHAPTER 23
SALES TAX REGISTRATION AND DE-
REGISTRATION
1. Registration, Compulsory registration and De-registration (Section 14 & Chapter I – Sales Tax Rules 2006)

Abbreviations used:
LTU Large Taxpayers Unit
RTO Regional Tax Office

A) Requirement of registration – section 14:


The following persons engaged in making taxable supplies in Pakistan (including zero rated supplies) are
required to be registered, namely:

i. a manufacturer not being a cottage industry


ii. a Tier 1 retailer;
iii. an importer
iv. a wholesaler, dealer or distributor
v. An exporter who intends to obtain sales tax refund against his zero-rated supplies
vi. a person who is required under any Federal or Provincial law to be registered for the purpose of any
duty or tax collected or paid as if it were a sales tax under the Act.

Discontinuation of gas and electricity connections


Gas and electricity connection may be discontinued through general order by the FBR in the following cases:
- any person, including tier – 1 retailers, who fails to register for sales tax purpose; or
- notified tier – 1 retailers registered but not integrated with the FBR
Upon registration or integration, the FBR shall notify the restoration of gas or electricity connection.

B) Application for registration – rule 5:


A person required to be registered shall apply for registration on computerized system before making any
taxable supply in the prescribed form indicating jurisdiction of RTO as per the following criteria:

Person Area where registration required


1 in case of public company the place where the registered office is located
2 in case of public company (i) if the company is primarily engaged in manufacture, the
place where the factory is situated; and
(ii) if the company is primarily engaged in business other
than manufacture, the place where main business
activities are actually carried on;
3 in case of a person not the jurisdiction where the business is actually carried on
incorporated
4 in case of a person not the jurisdiction where the manufacturing unit is located:
incorporated, having a single
manufacturing unit and whose
business premises and
manufacturing unit are located in
different areas

480 | P a g e
The FBR may transfer the registration of any registered person to a jurisdiction where the place of business or
registered office or manufacturing unit is located.

Attachments to the application:


The applicant having NTN shall, using his login credentials, upload the following information and documents:
(a) bank account certificate issued by the bank, in the name of business;
(b) registration or consumer number with gas and electricity supplier;
(c) particulars of all branches in case of multiple branches at various locations;
(d) GPS-tagged photographs of the business premises;
(e) In case of manufacturer, also the GPS-tagged photographs of machinery and industrial electricity or gas
meter installed; and
(f) Where an applicant has unsold / unused stock of tax-paid inputs on which he desires to claim the benefit
u/s 59, he shall declare such stock in a statement with his application for registration.

The system shall register the applicant on furnishing of the above documents and thereafter the applicant or his
authorized representative shall visit e-Sahulat Centre of NADRA within a month for bio-metric verification. In
case of default the registered person’s name shall be excluded from the active taxpayers’ list.
In case of manufacturer, the FBR may require post-verification. If document is missing or non- genuine the
registered person would be required to provide the same within 15 days. In case of default the registered person’s
name shall be excluded from the active taxpayers’ list.

(C) Temporary registration – rule 5A:


1. Where a manufacturer applies for registration without having installed machinery, temporary
registration as manufacturer shall be allowed within 72 hours to him for a period of 60 days subject to
furnishing of the complete list of machinery to be imported along with import documents.

2. After temporary registration, the person is allowed to import machinery, raw materials, etc. as a
manufacturer but he will submit a post-dated cheque (equal to the difference in duties and taxes to be availed
as a manufacturer i.e. 4% value addition tax which is payable by a commercial importer)

3. If the machinery is not installed within 60 days of issuance of the temporary registration, such temporary
registration shall be disabled and the post-dated cheques submitted shall be en cashed.

4. A person holding temporary registration shall file monthly return but shall not issue a sales tax invoice
and if such invoice is issued, no input tax credit shall be admissible against such invoice.

5. No sales tax refund shall be paid to the person during the period of temporary registration and the
amount of input tax may be carried forward to his returns for subsequent tax periods.

(D) Compulsory registration:


1. If a person, who is required to be registered, does not apply for registration, a notice shall be issued to
such person, giving an opportunity of being heard. After receiving a written reply, and personal hearing
if so desired by the person, the Commissioner shall pass an order whether or not such person is liable
to compulsory registration.

2. Where the person does not respond within the time specified in the notice, the Commissioner shall
transmit the particulars to computerized system, which shall compulsorily register the said person.

3. A compulsory registered person is required to comply with all the provisions of sales tax laws. In the
case of failure to do so, the Commissioner may issue notice for production of records and appearance
in person to assess the amount of sales tax payable and take any other legal action against such person.

481 | P a g e
4. If it is subsequently established that a person was not liable to be registered but was wrongly registered,
computerized system on the recommendation of the Commissioner, shall cancel such registration and
such person shall not be liable to pay any tax, default surcharge or penalty.

(E) Change in the particulars of registration


(1) In case of a change in any particulars, the registered person shall notify the change within 14 days and
the computerized system shall issue the revised registration certificate.

Where a person is unable to file application directly in computerized system, he may submit the application to
the Commissioner RTO who shall ensure entry of the application in computerized system within 3 days.
(2) The change of business category from non-manufacturer to a manufacturer shall be allowed after
fulfilling the requirements which are applicable for registration as a manufacturer such as verification of
machinery and confirmation of status as industrial consumer from electric and gas distribution companies.

(F) Transfer of Registration:


The registration may be transferred from one Commissioner to another or to LTU or RTO. If a registered person
intends to shift his business activities from the jurisdiction of one Commissioner to another or has any other
valid reason, he may apply for the transfer of his registration which is subject to the approval by the sales tax
department.

The Commissioner / RTO / LTU in whose jurisdiction the registration is now transferred shall exercise the
jurisdiction over such person in the manner as if it always had such jurisdiction.

G) Option to file application with Commissioner:


The person applying for registration, change in particulars or transfer of registration may, in exceptional cases,
file an application in the RTO. The Commissioner at RTO will then send such application to the computerized
system within 3 days.

H) Cancellation of multiple registrations:


In case of multiple registrations, the registered person shall retain only one registration and surrender all other
registrations.

The FBR may, in special cases, allow multiple registrations of manufacturing units located in different LTU or
RTO.
I) De-registration:
Every registered person
• who ceases to carry on his business or
• whose supplies become exempt or
• who sell his business
• who merges his business with another
• failed to file return for 6 tax periods
shall apply to the Commissioner for de-registration
The Commissioner may, on such application or on its own initiative, recommend to the computerized system to
cancel the registration with the later of
• 90 days from the date of application or
• the date all the dues are cleared

• A registered manufacturer who, after registration, is covered by the definition of cottage industry may
also apply for de-registration. Likewise, a retailer may also apply for de-registration on any valid reason.
• After filing the application for de-registration, the obligation to file monthly sales tax return shall remain

482 | P a g e
suspended until he is de-registered or his application is rejected.
• The Commissioner, after satisfying himself, shall direct the applicant to discharge any outstanding
liability, if any, by filing a Final Return.
• Where the Commissioner desires to conduct audit or inquiry to determine tax liability, he shall require
the applicant to provide the requisite records. On receipt of the complete requisite records, entry to this
effect shall be made in the computerized system which shall automatically de-register the applicant on
expiry of 90 days thereof.

Question Mr. Moye Moye applied for de-registration on 05th March 2024. Determine the date by which
commissioner may issue order of de-registration if outstanding:
(a) Tax liability is paid on 15th May 2024;
(b) Tax liability is paid on 12th July 2024.
Solution
(a) The Commissioner may issue order of de-registration within later of following:
• 90 days from the date of application i.e. 3rd June 2024; or
• The date all outstanding liabilities are deposited by him i.e. 15th May 2024.
As 3rd June 2024 is later, the Commissioner may issue order of de-registration by 3rd June 2024.

(b) The Commissioner may issue order of de-registration within later of following:
• 90 days from the date of application i.e. 3rd June 2024; or
• The date all outstanding liabilities are deposited by him i.e. 12th July 2024.
As 12th July 2024 is later, the Commissioner may issue order of de-registration by 12th July 2024.

J) Blacklisting and suspension of registration –


Where the Commissioner / FBR has reasons to believe that a registered person is involved in tax fraud etc., he
may suspend the registration of such person by an order in writing and initiate such enquiry as deem fit.
Procedure for blacklisting and suspension of registration:

SUSPENSION
1. Where a commissioner is satisfied that a registered person has issued fake invoices, evaded tax or
committed tax fraud, registration of such person may be suspended through the system, without prior
notice, pending further inquiry.
The basis for suspension may inter alia include:
a) Non-availability of the registered person at the given address;
b) Refusal to allow access to business premises or refusal to furnish records to an authorized Inland
Revenue Officer;
c) Abnormal tax profile, such as taking excessive input tax adjustments, continuous carry- forwards, or
sudden increase in turnover;
d) Making substantial purchases from or making supplies to other blacklisted/suspended persons;
e) Non-filing of sales tax returns;
f) On recommendation of a commissioner of any other jurisdiction;
g) Any other reason to be specified by the Commissioner.

2. The written suspension order shall indicate the reason for suspension and shall be endorsed to the
registered person and all other concerned departments.

3. A registered person who does not file sales tax return for six consecutive months shall be suspended by

483 | P a g e
the system without any notice.

4. No input tax adjustment/refund shall be admissible to the suspended person during the period of
suspension. Similarly, no input tax adjustment shall be allowed to the buyers on the basis of invoices
issued by such suspended person (whether issued prior to or after such suspension), during the period
of suspension.

5. The Commissioner shall, within 7 days of issuance of suspension order, issue a show cause notice to
afford an opportunity of hearing within 15 days of the issuance of such notice clearly indicating that he
will be blacklisted in case:

a) there is no response to the notice


b) has not provided the required record
c) has not allowed access to his business record/premises
d) any other reason specified by the Commissioner.

6. In case show cause notice is not issued within 7 days of the suspension order, the said order shall become
void ab initio.

7. In case of non-availability of the suspended person at the given address, the notice may be affixed on
the main notice board of the LTU/RTO.

8. On receipt of the reply to the notice, if the Commissioner is satisfied, he may order for revoking of
suspension of the registered person.
BLACKLISTING
9. In case the offence is confirmed, the Commissioner shall issue an appealable order for blacklisting
and shall proceed to take legal and penal action.

10. The order of blacklisting shall contain:


• reasons for blacklisting;
• time period for which any refund or input tax shall be inadmissible claimed by blacklisted person or
by his buyers on the basis of invoices issued by him;
• any recovery to be paid or penalties to be imposed.

11. The order of blacklisting shall be issued within 90 days of the issuance of the notice of hearing. In
case, the order of blacklisting is not issued within 90 days the suspension of registered person shall become
void ab initio.

12. Copies of the order shall be endorsed to the registered person and all the concerned departments.
Copies of the order shall also be circulated, along with a computer system- generated list of invoices issued by
the blacklisted persons, to all Officers of Inland Revenue to enable them to disallow the input tax claim on the
basis of invoices issued by the said blacklisted persons.

Summary of Process

484 | P a g e
485 | P a g e
CHAPTER 24
SALES TAX RECORDS AND MISC.
1. Record keeping and Return filing requirements

a) Records – Section 22:


A registered person making taxable supplies shall maintain and keep certain records / documents at his
registered office including the following:
1. Records of purchases/imports and supplies made indicating description, quantity, value of goods, name,
address and NTN of the buyer/ supplier and the amount of sales tax charged or paid;
2. Records of zero-rated and exempt supplies;
3. Invoices, credit / debit notes, bank statements, banking instruments, inventory records, utility bills,
salary and labor bills, cash book, agreement in respect of rent, sale and purchase;
4. gate passes, inward or outward, and transport receipts;
5. Double entry sales tax accounts;
6. Business bank accounts should be declared;
7. Electronic version of records;
8. Such other records as may be specified by the Board.

The above record shall be retained for 6 years – Section 24.


• Provided that the persons paying retail, tax shall keep such record as may be specified by the Board.
• The Board may also require a registered person or class of registered persons to declare and use only as
many numbers of business bank accounts as may be specified by the Board in such notification to make
or receive payments on account of purchase and sale transactions for the purpose of the Sales Tax Act,
1990 or rules made thereunder and to make payment of due tax from such accounts only.
• The Board may specify to keep such other records for the sales tax law purposes.
• The Board may specify to use such electronic fiscal cash registers as are approved by the Board.
• The registered person shall keep the aforesaid record at his business premises or registered office in
English or Urdu language
• The registered persons, whose accounts are subject to audit under the Companies Act, 2017, shall be
required to submit a copy of the annual audited accounts, along with a certificate by the auditors
certifying the payment of due tax by the registered person

b) Return filing requirements


Every registered person is required to file monthly return electronically through FBR e-portal by the prescribed
date of filing of return.
A general due date of filing of return is 15th of the next month. However, in case where due date is 15th of a
month, the tax due shall be deposited by 15th and the return shall be submitted electronically by 18th of the
same month.
Tax shall be deposited in National bank on the prescribed payment challan or through electronic payment system
devised for this purpose.

The date of payment in case of payment through cash or cheque shall be treated as the date on which the
payment is received by the bank. In case of payment through pay order or bank draft, date on which the pay
order or bank draft is tendered at the bank counter.

c) Electronic filing of sales tax:

486 | P a g e
A registered person shall enter data electronically of his supplies in Annexure-C (i.e. output tax) and data of
Debit or Credit Notes in Annexure-I by the 10th day of the month next following the tax period. This data shall
be immediately available to the other parties in their “Purchase Data” and “Debit or Credit Note Data” to enable
them to claim input tax.
Data relating to purchases made from unregistered persons or from such registered persons as allowed by the
FBR in this respect shall be entered manually in Annexure-A
If a registered person’s claim reduces his sales tax liability such as:
- Input tax on his purchases; or
- Credit Note issued by him in respect of sales return or increase in purchase value on account of any
mistake or otherwise in purchase invoice earlier received
And a discrepancy was found by the automated system of the FBR then the said adjustment would be allowed
to the registered person on provisional basis and he would be given an opportunity to contact and persuade the
other party to resolve the discrepancy by declaring output tax.
If the discrepancy is resolved then the objection raised by the automated system shall stand settled and the
registered person shall be informed accordingly.
If the discrepancy is not resolved by the 10th day of the next month, then the adjustment earlier allowed on
provisional basis shall be adjusted or recovered from the registered person.

Quantitative details – rule 14


Registered manufacturers of the specified goods are required to furnish the details of quantities of goods manufactured
and supplied along with the monthly returns. The examples of specified goods include sugar, cigarettes, paper, cement,
refrigerators, ACs, TV, vehicles, ice cream etc.

d) Annual Return or Special Return


FBR may require a person to file annual statement in the prescribed form.
Every company registered for sales tax shall file annual sales tax return for a financial year by 30th September
of the next financial year.

e) Extension of time for filing of return


For the purpose of extension of time for filing of return, the registered person is required to file an application
to the Commissioner and the Commissioner may grant an extension of time up to 15 days if he is satisfied that
the applicant is unable to furnish the return by the due date because of –
(i) Absence from Pakistan;
(ii) Sickness or other misadventure; or
(iii) Any other reasonable cause.
Under exceptional circumstances a longer time extension may also be granted by the Commissioner.
Refusal of extension by the Commissioner
If a commissioner refuses to extend the time for filing of return, then the registered person may file an
application to the Chief Commissioner who may allow such extension of time for 15 days and a longer time
under exceptional circumstances.

f) Revised returns:

487 | P a g e
A registered person may file a revised return with the approval of Commissioner within 120 days of filing the
original return. However, Commissioner’s approval is not required where the return is revised within 60 days
or where tax payable in the revised return is more than the tax payable declared in the original return.
When filing a revised return, the following amounts will be paid depending upon the time it is furnished

Time of Furnishing revised return Amount required to be paid


Before receipt of notice of audit Amount of short tax paid/ evaded + default
surcharge
During audit and before receipt of show cause Amount of tax pointed by officer + 25% of
notice penalty
After issuance of show cause notice Amount of Tax evaded + Default Surcharge +
100% penalty [If this amount is paid show cause
notice shall stand abated]

g) Final return:
Before de-registration, if any tax liability is required to be paid, it would be paid through a final return.

Particulars of Return:
i. The return shall indicate; Sales tax registration number (STRN), name and address of the supplier.
ii. name, address and registration, number of the recipient and NIC or NTN of the unregistered person, as the
case may be, excluding supplies made by a retailer where the transaction value inclusive of sales tax amount
does not exceed rupees one hundred thousand, if sale is being made to an ordinary consumer
iii. Explanation—For the purpose of this clause, ordinary consumer means a person who is buying goods for his own
consumption and not for the purpose of resale or processing: Provided that the condition of NIC or NTN shall be
effective from 1st August, 2019;

iv. Date of issue of invoice;

488 | P a g e
v. description, including count, denier and construction in case of textile yarn and fabric and quantity of
goods;
vi. Tax credit carried forward from previous period.
vii. Value of supplies
viii. Output tax due on supplies as under:
a) Local taxable supplies
b) Exempted supplies
c) Zero rated supplies
ix. Value of purchases;
x. Input tax paid on purchases as under:
a) Local taxed goods
b) Imported taxed goods
c) Exempted purchases
d) Zero rated purchases
e) other purchases
xi. Arrears payable
xii. Amount payable / refundable.

• The registered person shall deposit in the banks, the amount of sales tax indicated as “Sales Tax
Payable” in the return at the time of filing of return.
• In case no amount of sales tax is payable by the registered person, he shall file “Nil” return without
depositing any amount.
• If it is subsequently proved that CNIC provided was by the purchaser was not correct, liability of tax
or penalty shall not arise against the seller, in case of sale made in good faith.

2. Assessment & Audit


a) Audit – Section 25
1. Audit may be conducted by the Commissioner. An inquiry or investigation may also be conducted if
the Commissioner has sufficient evidence that the registered person is involved in tax fraud.
2. However, an officer of Inland Revenue may also conduct an audit, if the same were earlier conducted
by the office of the Auditor General of Pakistan.
3. The Commissioner may conduct audit proceedings electronically through video link.
4. The Assistant Commissioner (Audit) shall issue his audit observations if he finds any defects during
audit. The registered person is required to submit his point of view within 15 days against such audit
observations. The Assistant Commissioner shall issue an audit report specifying the sales tax demand,
if any.
5. If the registered person makes payment of sales tax evaded or short paid voluntarily before receiving
notice for audit then no penalty shall be imposed. 25% penalty shall be imposed if the same is paid
during audit but before issuance of show cause notice [i.e., audit observations]. Full penalty shall be
imposed after issuance of show cause notice.
b) Assessment of tax and recovery of tax not levied or short-levied or erroneously refunded. - Section 11
An Officer Inland Revenue shall make an assessment of tax including default surcharge and penalty where:

489 | P a g e
- A person fails to file a return of sales tax; or
- A person pays tax less than the tax actually payable due to miscalculation; or
- A person claimed incorrect input tax or refund; or
- The tax due on supplies made by a person has not been paid or short paid; or
- Any tax has not been levied or short levied by reason of any inadvertence, error or misconstruction or
deliberately.
For the purpose of audit, a show cause notice may be given within a period of 5 years specifying the ground of
such assessment and an opportunity of being heard shall be provided.

Time limit for said assessment is within 120 days of issuance of show cause notice (may be extended to a
further 90 days for reasons to be recorded in writing).

However, any period during which the proceedings are adjourned for any reason including stay order shall be
excluded from the said time limit for completion of assessment.

Where a tax has not been levied, the tax shall be recovered as tax fraction of the value of supply.

Where a person, required to file a return, files the return after the due date and pays the amount of tax along
with default surcharge and penalty, the show cause notice and the order of assessment shall abate.

However, where a registered person has not paid tax payable with return or short paid then tax payable shall
be recovered without notice by:
- Attachment of his business bank account; and
- stopping removal of any goods from business premises However, show cause notice is required for
imposition of any penalty.

Assessment giving effect to an order: section 11B


Where an assessment order is required to be issued on the instruction of an appellate authority, the
Commissioner or an officer of Inland Revenue empowered in this behalf shall issue the order within one year
from the end of the financial year in which the appellate order was served.

Powers of tax authorities to modify orders: Section 11C


Where a question of law has been decided by the Appellate Tribunal or High Court in the case of a registered
person, the Commissioner may follow the said decision including in respect of an identical situation in the
subsequent years of the said registered person even if the tax department has filed an appeal against such
decision.

If the higher appellate authority reverses such decision of the Appellate Tribunal or High Court, the
Commissioner is empowered to modify the assessment within 1 year from the date of receipt of the decision
of the higher appellate authority and in this case normal time limit for amendment shall not apply.

c) Access to Records and Documents – Section 25


Officer Inland Revenue is authorized to have access to the records and documents maintained under the Sales
Tax Act or under any other law and use of such machine (e.g. computer) on which data related to sales tax is
kept.
d) Drawing of Samples – Section 25A
An authorized officer of Inland Revenue, if consider it necessary, may take a sample of any goods or raw
materials, for the purpose of:

490 | P a g e
(i) determining the liability to sales tax of a registered person; or
(ii) for the purpose of establishing value of goods; or
(iii) for any other reason.
➢ The sample drawn shall be a minimum quantity of goods or raw materials sufficient to enable a proper
examination or analysis to be made.
➢ At the time of taking the sample the person in possession of the goods shall be informed and given the
opportunity to sign the representative samples, so drawn, and take
➢ corresponding sample for his own record.
➢ Any sample taken as above shall be taken against a proper receipt a copy each of which shall be kept
in the record by the registered person and the large Taxpayers unit or Regional Tax Officer, as the
case may be.

491 | P a g e
Chapter 25
Sales Tax Theoretical Questions
Question # 1 Autumn 2014 Q. 5

(a) Under the Sales Tax Act, 1990 and Rules made thereunder:
(i) List the persons who are required to be registered. (05)
(ii) Change in rate of tax during a tax period (04)

(b) There are certain food items in the inventory of XY Limited (XYL) which were returned by the customers
after the expiry date. Specify the procedure which must be followed under the Sales Tax Rules, 2006 if
XYL wishes to destroy these items. (03)

Question # 2 Spring 2015 Q. 8

Saleem is registered under the Sales Tax Act, 1990 and is engaged in the business of export and distribution of
electronic appliances.
Required:
Under the provisions of the Sales Tax Act, 1990 and Rules made thereunder, advise Saleem on the following
matters:
(a) any six situations in which input tax is not allowed to be adjusted against the output tax liability. (06)
(b) exports which are outside the purview of zero rating. (03)
(c) eligibility for a refund if input tax is paid in excess of the output tax payable for the month. (02)
(d) concept of provisional and final adjustment in relation to ‘Apportionment of input tax’. (02)

Question # 3 Autumn 2015 Q. 6

(a) Under the provisions of the Sales Tax Act, 1990 explain the following:
(i) Input tax in relation to a registered person (03)
(ii) Supply (04)

(b) Baber Associates, who is registered with the Inland Revenue Department for sales tax purposes, has
supplied a heavy duty motor to Mubarak Enterprises on one month’s credit. However, due to sharp decline
in petroleum prices, the price of the motor has reduced by 10% in the local market. Upon request from
Mubarak Enterprises, Baber Associates has finally agreed to reduce the price of motor by 8%.
In view of the Sales Tax Rules, 2006 describe the procedure which may be followed by both the parties to
give effect to the above price change. (03)

Question # 4 Spring 2016 Q. 6

Under the provisions of the Sales Tax Act, 1990 and Rules made thereunder, briefly describe the following:
(a) How and under what circumstances the Inland Revenue Department may recover the amount of sales tax
from a person without issuing him a show cause notice. (04)
(b) Rule relating to change in the particulars of registration other than the change of business category. (05)
(c) What evidence(s) a person may be required to submit if he is applying for registration as a manufacturer
on shared premises. (02)

Question # 5 Autumn 2016 Q. 6

(a) Under the Sales Tax Act, 1990 and Rules made thereunder, briefly describe the concept of ‘Residual input
tax’. How it differs from ‘Residual input tax credit’? (03)

492 | P a g e
(b) Under the provisions of the Sales Tax Act, 1990 enumerate any four features distinguishing the concept
of ‘Zero rating’ from ‘Exempt supply’. (04)

(c) Identify the records which a registered person making taxable/exempt supplies is required to maintain at
his business premises or registered office under the Sales Tax Act, 1990. (Note: details of contents not
required) (04)

Question # 6 Spring 2017 Q. 8

(a) Under the provisions of Sales Tax Act, 1990 and Rules made thereunder, identify the circumstances in
which:
(i) a registered person is not allowed to reclaim or deduct input tax paid. (06)
(ii) a registered person may be liable for deregistration. (03)

(b) On 2 June 2016, Abid Limited inadvertently issued a tax invoice with an incidence of sales tax amounting
to Rs. 25,000 as against the applicable tax of Rs. 45,000. The error was detected on 15 February 2017 i.e.
after expiry of 180 days.

Advise Abid Limited in the light of Sales Tax Rules, 2006. (04)

Question # 7 Autumn 2017 Q. 7

Zubair has recently been registered under the Sales Tax Act, 1990. You are required to advise him on the
following matters:
(a) Type of exports which are outside the purview of zero rating. (03)
(b) Eligibility for a refund if input tax is paid in excess of output tax payable for the month. (03)
(c) The conditions required to be fulfilled for filing a revised return. (02)
(d) Concept of provisional and final adjustment in relation to ‘Apportionment of input tax’. (02)
(e) How to deal with change in rate of tax during a tax period. (04)

Question # 8 Spring 2018 Q. 6

Under the provisions of the Sales Tax Act, 1990:


(a) List the exceptions to the following general rule:
(i) Where the taxable supplies are made to a person who has not obtained registration number, there
shall be charged, levied and paid a further tax at the rate of 4% of the value in addition to the normal
rate of 18%. (03)
(ii) Goods exported shall be charged to tax at the rate of zero percent. (03)

(b) Explain the term ‘Temporary registration’. Briefly discuss the rights, obligations and responsibilities of a
person who has obtained temporary registration. (06)

Question # 9 Autumn 2018 Q. 6

(a) Under the Sales Tax Act, 1990 and Rules made thereunder, briefly describe:
(i) temporary sales tax registration and rights, obligations and responsibilities of a person holding
temporary registration. (05)
(ii) differences between rules applicable to exempt and zero rated supplies. (04)
(iii) the provisions related to excess/additional amount of sales tax collected by a registered person. (03)

(b) Where a Commissioner of Inland Revenue, having jurisdiction, is satisfied that a registered person has
issued fake invoices, evaded tax or committed tax fraud, he may suspend the registration of such person

493 | P a g e
without prior notice, pending further inquiry.

Under the Sales Tax Act, 1990 and Rules made thereunder, state any four basis of such satisfaction which
allow the Commissioner to suspend the registration as described above. (03)

Question # 10 Spring 2019 Q. 6

(a) Briefly discuss the situations under which the following are required to be registered under the Sales Tax
Act, 1990 and Rules made thereunder:
(i) Cottage industry (02)
(ii) Retailer (01)

(b) Under the provisions of Sales Tax Act, 1990 and Rules made thereunder, identify the circumstances in
which a registered person may be liable for deregistration. (03)

(c) Bashir (Private) Limited (BPL) was incorporated on 1 January 2019 and registered with sales tax
department on 1 February 2019. BPL is in process of submitting its first sales tax return for the month
ended 28 February 2019. The finance department has identified following transactions which took place
before registration:
(i) Goods costing Rs. 5 million were purchased from a registered supplier. 80% of the goods remained
unsold as at 1 February 2019. The supplier charged sales tax at the rate of 18%. (03)
(ii) Advance payment of Rs. 2.5 million was received on 15 January 2019 for the supply of taxable
goods to a registered person. The goods were delivered in February 2019. (02)

Required:
Advise the finance department about the sales tax implications of the above transactions on BPL’s first sales
tax return.

Question # 11 Autumn 2019 Q. 6

(a) Under the provisions of the Sales Tax Act, 1990 and Rules made thereunder, briefly describe the treatment
of the following:
(i) Recording of partial payments received in advance during a tax period in respect of both taxable and
exempt supplies. (02)
(ii) Change in rate of tax during a tax period. (04)

(b) There are certain goods returned by the customer as they are unfit for consumption and the seller has no
option but to destroy them.

Specify the procedure which must be followed by a registered person under the Sales Tax Rules, 2006 for
the destruction of such goods. (02)

(c) Who is required to file the following sales tax returns? Also mention the due date of filing of these returns.
(i) Monthly return (ii) Special return
(iii) Final return (iv) Annual return (04)

Question # 12 Spring 2020 Q. 7

(a) Raheel, an unregistered person, runs a garment shop in the posh area of Karachi. He has received a notice
from the Commissioner Inland Revenue requiring him to register with the sales tax authorities within 30
days.

Under the provisions of the Sales Tax Act, 1990 and Rules made thereunder, advise Raheel regarding the
following:

494 | P a g e
(i) Whether the Commissioner is justified in issuing the notice to him. (03)
(ii) Would it be necessary for him to respond to the notice. (04)

(b) Under the provisions of the Sales Tax Act, 1990 and Rules made thereunder, discuss the following:
(i) Difference between zero rated supplies and exempt supplies. (04)
(ii) How and under what circumstances the Inland Revenue Department may recover the amount of sales
tax from a person without issuing him a show cause notice. (04)
(iii) Concept of provisional and final adjustments in relation to ‘Apportionment of input tax’.
(02)

Question # 13 Autumn 2020 Q. 5

Sun Associates (SA) has recently been registered with the Inland Revenue Department under the Sales Tax Act,
1990.
Required:
Under the Sales Tax Act, 1990 and Rules made thereunder,
(a) identify the documents which SA may require for claiming/adjusting the input tax relating to the following
activities:
(i) supply of taxable goods (01)
(ii) import of goods into Pakistan (02)
(iii) goods purchased in an auction (02)
(b) state the requirements relating to retention of records and documents that SA should comply with. (02)

Question # 14 Autumn 2020 Q. 6

Rapid Associates (RA) has been registered under the Sales Tax Act, 1990 since 2014. During the month of June
2020, RA issued fake sales tax invoices amounting to Rs. 5 million to one of its customers. Apart from this, RA
has always been in compliance with all the regulations of the Sales Tax Act, 1990.

Required:
Under the provisions of the Sales Tax Act, 1990 and Rules made thereunder, discuss the consequences which
RA may have to face due to issuance of fake invoices. (06)

Question # 15 Spring 2021 Q. 7

(a) In the light of the provisions of Sales Tax Act, 1990 and Rules made thereunder, briefly explain as to the
chargeability/adjustment of sales tax in respect of each of the following independent matters:
(i) Free provision of taxable goods to the company’s CEO as per the terms of his employment.
(ii) Free replacement of defective parts in the case of taxable goods, sold under warranty.
(iii) Payment of machine fuel by one of the directors using his own credit card. The machine is used to
manufacture taxable goods.
(iv) Taxable goods sold on instalment to a customer at a price inclusive of mark up.
(v) Advance payment received against taxable goods to be supplied to a registered person in next month.
(vi) Local supplies of goods manufactured by a cottage industry.
(vii) Material purchased for the construction of office building.
(viii) Electronic cash register purchased for retail outlet. (08)

(b) Under the Sales Tax Act, 1990 and Rules made thereunder, briefly describe temporary sale tax registration.
Also state the rights, obligations and responsibilities of a person holding temporary registration. (05)

Question # 16 Autumn 2021 Q. 8

(a) Under the provisions of the Sales Tax Act, 1990 and Rules made thereunder, briefly explain the following:

495 | P a g e
(i) how and under what situations the Inland Revenue Department may recover the amount of sales tax
from a person without issuing him a show cause notice. (04)
(ii) extra tax and capacity tax. (05)

(b) On 4 February 2021, it was revealed to Fahad that he inadvertently reported an output sales tax of Rs
27,000 in a tax invoice, issued on 5 July 2020, to a customer instead of Rs. 72,000 in his sales tax return
for July 2020.

Required:
In the light of the Sales Tax Act, 1990 and Rules made thereunder, advise how Fahad can rectify this error
after the expiry of 180 days. (03)

Question # 17 Spring 2022 Q. 5(a)

Following are the independent transactions carried out by different enterprises during the month of February
2022:
(i) Taxable goods of Rs. 800,000 were sold to one of the dealers. The amount was net of 20% trade discount
which was in accordance with market norms. The discounted price was not shown on the tax invoice.
(ii) Taxable goods of Rs. 1,500,000 were used for internal testing and evaluation purposes. 40% of these goods
were locally procured while remaining 60% of these goods were own manufactured.
(iii) Advance of Rs. 600,000 was received for goods to be delivered in April 2022.
(iv) 1,000 units of taxable goods listed in the Third Schedule were sold at a unit price of Rs. 5,000. Retail price
of each unit was Rs. 6,000.
(v) New parts of Rs. 1,200,000 were issued free of cost to replace the defective parts under warranty.
(vi) Taxable goods of Rs. 400,000 were sold at credit terms of 2/10, n/30. Customer paid the amount within
ten days and availed the discount.
Required:
In the light of the provisions of the Sales Tax Act, 1990 and Rules made thereunder, state the value of supply
chargeable to tax for the month of February 2022. Also state the reason for your treatment. (08)

Question # 18 Spring 2022 Q. 6

(a) Under the Sales Tax Act, 1990 and Rules made thereunder, briefly discuss the chargeability of sales tax in
case of a retailer. (05)
(b) Shajee Limited (SL) purchases cosmetic products from Tajee (Private) Limited (TPL). In the month of
October 2021, SL received a consignment of 5000 units from TPL. On delivery, SL found that 50% of the
items were expired and decided to return them. Both SL and TPL are registered under the Sales Tax Act,
1990 and file sales tax return on regular basis.
Required:
Under the Sales Tax Rules, 2006 specify the document which must be issued by SL on return of goods to TPL.
Also state the particulars that should be mentioned on the document to be issued. (04)

Question # 19 Autumn 2022 Q. 6

(a) Under the provisions of the Sales Tax Act, 1990 and Rules made thereunder, explain whether the following
persons are required to be registered with the Inland Revenue Department:
(i) A manufacturer of taxable supplies located in the residential area of Korangi, Karachi.
(ii) A retailer whose annual turnover is Rs. 20 million.
(iii) A distributor of exempt supplies.
(iv) An exporter of taxable goods. (04)

(b) Goods exported shall be charged to tax at the rate of zero percent. What are the exceptions to this
general rule? (03)

496 | P a g e
Question # 20 Spring 2023 Q. 6

(a) Following are the independent transactions carried out by different enterprises:
(i) In November 2022, an agreement for the acquisition of machine on hire purchase was signed. In December
2022, the machine was acquired under this agreement against 25% down payment of Rs. 30 million. The
remaining balance is to be paid in 24 equal monthly installments of Rs. 4.5 million each.
(ii) In December 2022, an advance of Rs. 12 million was received against delivery of goods to be made in
March 2023.
(iii) A machine of Rs. 38 million was purchased in July 2022 but it was put to use in November 2022.
(iv) Goods of Rs. 7 million were sold in September 2022 but due to limited storage capacity at buyer’s premises,
the goods were delivered in January 2023.
Required:
In the light of the provisions of the Sales Tax Act, 1990 and Rules made thereunder, identify and discuss the
time (month) of supply for the chargeability of sales tax in respect of the above transactions. (05)

(b) In the light of the Sales Tax Act, 1990 and Rules made thereunder, identify the situation(s) under which the
Inland Revenue Department may recover the amount of sales tax from a person without issuing him a show
cause notice. Also state the procedure for the recovery of the said amount. (04)

Question # 21 Autumn 2023 Q. 6


(a) Sarfraz is a sole proprietor. Under the provisions of the Sales Tax Act, 1990, discuss whether each of the
following individuals/entities is an associate of Sarfraz:
(i) Jehanzeb: He has been working as an accountant for the past twenty years and has been responsible for filing
the tax returns of both Sarfraz and his business.
(ii) Falah Limited (FL): Sarfraz owns 20% shares in FL, while his mother owns 30% shares in FL.
(iii) Sarah: She is the adopted daughter of Sarfraz’s wife, Fatima. Sarah was adopted by Fatima before her
marriage to Sarfraz.
(iv) Umeed Trust (UT): Sarfraz is one of the trustees of UT. The trust was established to oversee the operations
of an orphanage. (06)

(b) Under the provisions of the Sales Tax Act, 1990 and the Rules made thereunder, list different types of
returns alongwith the due dates for filing of such returns. (05)

497 | P a g e
ANSWERS
Answer # 1 Autumn 2014 Q. 5

(a) i. The following chart explains following persons are required to be registered
Importer Registration is required.
Importer shall pay sales tax on import stage and subsequently charge sales tax on value
of taxable supply
However, commercial importers shall pay sales tax as per the rule

Wholesaler/ Registration is required and sales tax shall be charged on value of taxable supply
Distributor
Retailer Only Tier 1 retailers (i.e. specified retailers) are required to be registered.
Manufacturer
Cottage Exempt as per serial 3, Table 2, 6th Schedule
industry
Other than Registration is required and sales tax shall be charged on value of taxable supply
cottage
industry

(ii) If there is a change in the rate of tax


▪ a taxable supply made by a registered person shall be charged to tax at such rate as in force at the time
of supply.
▪ Imported goods shall be charged to tax at such rate as is in force -
- in case the goods are entered for home consumption, on the date on which a [goods declaration]
is presented.
- in case the goods are cleared from warehouse, on the date on which a [goods declaration] for
clearance of such goods is presented.
▪ Where a [goods declaration] is presented in advance of the arrival of the conveyance by which the
goods are imported, the tax shall be charged as is in force on the date on which the manifest of the
conveyance is delivered.
▪ If the tax is not paid within seven days of the presenting of the [goods declaration] the tax shall be
charged at the rate as is in force on the date on which tax is actually paid.

(b) In order to destroy the goods, the following conditions must be fulfilled:
1. Prior permission from the Collector of Sales Tax having jurisdiction.
2. Goods should be destroyed under the supervision of an inland revenue officer of Sales Tax not below
the rank of an Assistant Collector as may be deputed by the Collector for the purpose.

Answer # 2 Spring 2015 Q. 8

(a) A registered person shall not be entitled to claim or deduct input tax paid on:
(i) goods or services used or to be used for any purpose other than for taxable supplies made or to be
made by him; OR goods or services used or to be used for making the exempt goods supplies.
(ii) any other goods which the Federal Government may, by a notification in the official Gazette, specify.
(iii) the goods which are subject to extra tax in addition to normal tax payable at 18%.
(iv) fake invoices.
(v) taxable goods or services which has not been deposited into government treasury by the supplier.
(vi) purchases made by a registered person, who fails to furnish the information required by the Board
through a notification.
(vii) purchases where payment has not been made through banking channel

(b) Provision relating to zero rating shall not apply in respect of a supply of goods which:
(i) are exported, but have been or are intended to be re-imported into Pakistan; or

498 | P a g e
(ii) have been entered for export under the Customs Act, 1969, but are not exported; or
(iii) have been exported to a country specified by the Federal Government, by Notification in the official
Gazette.

(c) Refund of input Tax


A registered person, shall not be allowed to adjust input in a tax period exceeding 90% of the output tax
Excess input tax shall be carried forward to the next tax period, and shall be treated as input tax for that
period. The adjustment/refund of input tax, shall be made on yearly basis in the second month following
the end of the financial year of the registered person. If the input tax paid by a registered person on
purchases exceeds the output tax because of:
▪ zero rated local supplies or
▪ exports
the excess input be refunded within 45 days of of refund claim. The Board may direct that refund shall be
paid with duty drawback

(d) Monthly adjustment of input tax shall be treated as provisional adjustment and at the end of financial year,
the person shall make final adjustment on basis of taxable and exempt supplies made during the year.

Answer # 3 Autumn 2015 Q. 6

(a)
(i) In relation to a registered person, means;
▪ tax levied under this Act on supply of goods to the person;
▪ tax levied under this Act on the import of goods by the person;
▪ in relation to goods or services acquired by the person, tax levied under the Federal Excise Act, 2005
in sales tax mode as a duty of excise on the manufacture or production of the goods, or the rendering
or providing of the services;
▪ Provincial Sales Tax levied on services rendered or provided to the person; and
▪ levied under the Sales Tax Act, 1990 as adapted in the State of Azad Jammu and Kashmir, on the
supply of goods received by the person;
(ii) Definition The term supply is defined in section 2(33) of the Act in the following way:
Supply means “A sale or other transfer of the right to dispose of goods as owner, including such sale or
transfer under a hire purchase agreement and also includes”
▪ putting to private, business or non-business use of goods produced or manufactured in the course of
taxable activity for purposes other than those of making a taxable supply;
▪ auction or disposal of goods to satisfy a debt owed by a person;
▪ possession of taxable goods held immediately before a person ceases to be a registered person;
▪ in case of manufacture of goods belonging to another person, the transfer or delivery of such goods to
the owner or to a person nominated by him; and
▪ Production, transmission and distribution of electricity
Provided that the Board with approval of the Federal Minister Incharge, may by notification in the official
Gazette, specify such other transactions which shall or shall not constitute supply.

(b) Where, for any valid reason the value of supply mentioned in the invoice issued has decreased. Babar
Associates shall issue a Credit Note (in duplicate), with prescribed particulars. The original copy, shall be
sent to the Mubarak Enterprise and the duplicate shall be retained for record. Mubarak Enterprise shall
issue a Debit Note in response to Credit Note issued by the Babar Associates as an acknowledgment of
the receipt. Same details will be provided in the Debit note.
Answer # 4 Spring 2016 Q. 6

(a) Short paid amounts recoverable without notice:


Where a registered person pays the amount of tax less than the tax due as indicated in his return, the short
paid amount of tax along with default surcharge shall be recovered from such person by stopping removal
of any goods from his business premises and through attachment of his business bank accounts, without

499 | P a g e
giving him a show cause notice and without prejudice to any other action prescribed under section 48 of
this Sales Tax Act or the rules made thereunder:

(b) Change in the particulars of registration (other than change of business category):
(i) In case there is a change in the name, address or other particulars as stated in the registration
certificate, the registered person shall notify the change in the Form STR-1 to the computerized
system, within fourteen days of such change.
(ii) In case of approval of the change applied for, a revised registration certificate shall be issued through
computerized system, which shall be effective from the date the person applied for the change.
(iii) Where a person is unable to file application for change in particulars of registration directly in
computerized system, he may submit the prescribed application and required documents to the
concerned Commissioner Inland Revenue at RTO, who shall ensure entry of the application and
documents in computerized system within three days.
(iv) The commissioner may, based on available information or particulars and after making such inquiry
as he may deem necessary and after providing reasonable opportunity of being heard to a person, by
an order in writing, make modifications in registration of the person.

(c) In case the person applying for registration as manufacturer is sharing the premises, he shall provide
evidence of:
(i) demarcation of manufacturing premises for registration, and
(ii) installation of sub-meter by the relevant utility company, in case he does not have independent
industrial utility connection but is using electricity or gas through sub- meter.

Answer # 5 Autumn 2016 Q. 6

(a) Residual input tax Vs. Residual input tax credit:


“Residual input tax” means the amount of tax paid on raw materials, components and capital goods being
used for making taxable as well as exempt supplies but does not include the input tax paid on raw materials
used wholly for making taxable or exempt supplies; Whereas “Residual input tax credit” is that amount
of residual input tax which is apportioned to the value of taxable supplies using the following formula:

Value of taxable supplies × Residual Input Tax


Residual Input Tax Credit on taxable supplies =
(Value of taxable + exempt supplies)

(b) Features distinguishing the concept of ‘Zero rating’ from ‘Exempt supply’:
Distinction points Zero Rated Supply Exempt Supply
Definition “Zero rated supply means a taxable “Exempt Supply means a supply
supply which is charged to tax at the which is exempt from tax.
rate of zero per cent.
Products covered Goods exported as notified by FBR or Goods specified by Federal
listed in the Fifth Schedule are charged Government and FBR and goods
to sales tax at the rate of zero per cent. listed in Sixth Schedule are exempt
supplies.
Invoicing Invoice shall be raised for the goods No sales tax invoice shall be raised.
Requirements supplied but sales tax shall be charged
at the rate of zero per cent
Registration A person engaged in zero rated supplies A person engaged exclusively in the
has to be registered with the Sales tax exempt supplies is not liable to be
department. registered.
Input tax credit Input tax paid related to zero rated Input tax paid related to exempt
supplies is refundable. supplies is inadmissible, therefore,
neither adjustable nor refundable.

(c) Records:

500 | P a g e
A registered person making taxable/exempt supplies shall maintain the following records of goods
(including zero rated and exempt supplies):
(i) records of supplies;
(ii) records of goods purchased;
(iii) records of goods imported;
(iv) records of zero-rated and exempt supplies;
(v) double entry sales tax accounts;
(vi) Following further records is desired:
Tax invoices, Credit notes, debit notes, Bank statements, Banking instruments in terms of section 73,
Inventory records, Utility bills, Salary and labour bills, Rental agreements, Sale-purchase
agreements, Lease agreements, Record relating to gate passes, inward or outward, and transport
receipts.
(vii) Such other records as may be specified by the Board.
(viii) The Board may specify to use such electronic fiscal cash registers as are approved by the Board.

Answer # 6 Spring 2017 Q. 8

(a)
(i) A registered person shall not be entitled to reclaim or deduct input tax paid on:
▪ the goods or services used or to be used for any purposes other than for taxable supplies made or to
be made by him;
▪ the goods on which extra amount of tax is payable
▪ any other goods or services which the Federal Government may by a notification in the official Gazette
specify;
▪ the goods or services in respect of which sales tax has not been deposited in the Government treasury
by the respective supplier;
▪ goods which are destroyed with the permission of the collector of sales tax
▪ purchase from suppliers who are black listed by the Commissioner
▪ if the payment in case of a transaction on credit is not transferred within 180 days of issue of the tax
invoice
▪ if payment is not made for the supplies in the business bank account of the supplier
▪ fake invoices;
▪ purchases made by a registered person in case he fails to provide information relating to his imports,
purchases, sales etc. as required by the Board.
▪ purchases in respect of which a discrepancy is indicated by CREST or input tax of which is not
verifiable in the supply chain;
▪ goods and services not related to the taxable supplies made by the registered person;
▪ goods and services acquired for personal or non-business consumption;
▪ vehicles falling in Chapter 87 of the First Schedule to the Customs Act, 1969.
▪ services in respect of which input tax adjustment is barred under the respective provincial sales tax
law;
▪ import or purchase of agricultural machinery or equipment subject to sales tax under Eighth Schedule
to this Act; and
▪ from the date to be notified by the Board, such goods and services which, at the time of filing of return
by the buyer, have not been declared by the supplier in his return or he has not paid amount of tax due
as indicated in his return.
▪ the goods which are subject to extra tax in addition to normal tax payable at 18%.
▪ gifts and giveaways.

(ii) The following registered persons may apply for deregistration:


▪ Who ceases to carry on his business
▪ Whose supplies become exempt from tax

The Commissioner may de-register a person if that person fails to file tax return for six continuous months.

501 | P a g e
(b) Time limitation of 180 days shall not apply in the given case as it is applicable only in the case of decrease
in output tax and increase in input tax. The above increase of output tax may be declared without any time
limitations.

Since Abid Limited has already accounted for the output tax in the sales tax return for the supplies, it can
issue a debit note in the month of February 2017 when the error was detected, and increase the amount of
output tax in the return for February 2017 by Rs. 20,000.

Answer # 7 Autumn 2017 Q. 7

(a) Following exports are outside the purview of zero rating:


(i) Goods which are intended to be re-imported into Pakistan
(ii) Goods which have been entered for export under the Customs Act, 1969 but are not exported
(iii) Goods exported to a country specified by the Federal Government, by Notification in the official
gazette.

(b) There are two types of refunds


(i) Refund of input tax in excess of 90% of output tax ; and
(ii) Refund of input tax related to zero ratings and exports.

In case of (i) a registered person would be required to submit a statement along with annual audited
accounts, duly certified by the auditors, showing value additions less than the limit prescribed.

The refund of input tax shall be made on yearly basis in the second month following the end of the financial
year of registered person.

In the case of (ii) the refund claim shall have to file in such manner and subject to such conditions as the
Board may, by notification in the official gazette specify.

(c) A revised return may be filed to correct any omission or wrong declaration made in a return subject to
approval of the Commissioner Inland revenue having jurisdiction within 120 days of filing of return.

(d) Monthly adjustment of input tax claimed by a registered person is treated as a provisional adjustment and
at the end of each financial year, a final adjustment is made on the basis of taxable and exempt supplies
made during the course of that year.

(e) If there is a change in the rate of tax,


▪ a taxable supply made by a registered person shall be charged to tax at such rate as in force at the time
of supply.
▪ imported goods shall be charged to tax at such rate as is in force:
− in case the goods are entered for home consumption, on the date on which a goods declaration is
presented.
− in case the goods are cleared from warehouse, on the date on which a goods declaration for
clearance of such goods is presented.
▪ where a goods declaration is presented in advance of the arrival of the conveyance by which the goods
are imported, the tax shall be charged as is in force on the date on which the manifest of the conveyance
is delivered.
▪ where the tax is not paid within seven days of the presenting of the goods declaration the tax shall be
charged at the rate as is in force on the date on which tax is actually paid.
If there is a change in the rate of tax during a tax period, a separate return has to be furnished in respect of
each portion of the tax period showing the application of different rates.

Answer # 8 Spring 2018 Q. 6

502 | P a g e
(a)
(i) 4% further tax is not payable in the following cases even if the supplies are made to unregistered persons:
▪ Electricity energy supplied to domestic and agricultural consumers.
▪ Natural gas supplied to domestic consumers and CNG stations.
▪ Motor oil, diesel oil, jet fuel, kerosene oil and fuel oil.
▪ Goods sold by the retailers to end customers.
▪ Supply of goods directly to end customers including food, beverages, fertilizers and vehicles.
▪ Items listed in Third Schedule to the Sales Tax Act, 1990.
▪ Second hand worn clothing and other worn articles falling under PCT heading 6309.0000.
▪ Fertilizers.
▪ Supplies by steel melters, re-rollers and ship breakers operating under Chapter XI of Sales Tax Special
Procedure Rules, 2007.
▪ Supplies covered under the Fifth Schedule to the Sales Tax Act, 1990.

(ii) Following is the list of goods which are exported but shall not be charged at the rate of zero percent:
▪ Goods exported but have been or are intended to be re-imported into Pakistan; or
▪ Goods which have been entered for export under section 131 of the Customs Act, 1969 (IV of 1969),
but are not exported; or
▪ Have been exported to countries specified by the Federal Government, by notification in the official
Gazette in this regard.

(b) Where a person files application for sales tax registration as a manufacturer prior to installation of
machinery, for the purpose of import of machinery to be installed by him, temporary registration as
manufacturer shall be allowed to him for a period of sixty days.

After receiving temporary registration, the person shall be allowed to import plant, machinery and raw
materials, etc. as a manufacturer, subject to submission to the customs authorities of a post-dated cheque
equal to the difference in duties and taxes to be availed as a manufacturer.

In case the list of machinery is not provided within sixty days of issuance of the temporary registration,
such temporary registration shall be disabled and the post- dated cheques submitted shall be encashed.

A person holding temporary registration shall file monthly return, but shall not issue a sales tax invoice
and if such invoice is issued, no input tax credit shall be admissible against such invoice.

No sales tax refund shall be paid to the person during the period of temporary registration and the amount
of input tax may be carried forward to his returns for subsequent tax periods.

Answer # 9 Autumn 2018 Q. 6

(a)
(i) Where a person files application for sales tax registration as a manufacturer prior to installation of
machinery, for the purpose of import of machinery to be installed by him, temporary registration as
manufacturer shall be allowed to him for a period of sixty days.
After receiving temporary registration, the person shall be allowed to import plant, machinery and raw
materials, etc. as a manufacturer, subject to submission to the customs authorities of a post-dated cheque
equal to the difference in duties and taxes to be availed as a manufacturer.
In case the list of machinery is not provided within sixty days of issuance of the temporary registration,
such temporary registration shall be disabled and the post-dated cheques submitted shall be encashed.
A person holding temporary registration shall file monthly return, but shall not issue a sales tax invoice
and if such invoice is issued, no input tax credit shall be admissible against such invoice.
No sales tax refund shall be paid to the person during the period of temporary registration, however, the
amount of input tax may be carried forward to his returns for subsequent tax periods.

(ii) Features distinguishing the concept of ‘Zero rating’ from ‘Exempt supply’:

503 | P a g e
• Distinction • Zero Rated Supply • Exempt Supply
points
• Taxability • Zero rated supply is charged to • Exempt Supply means a
tax at the rate of zero per cent. supply which is exempt from tax.
• Products • Goods exported as notified by • Goods specified by Federal
covered FBR or listed in the Fifth Schedule are Government and FBR and goods
charged to sales tax at the rate of zero listed in Sixth Schedule are
per cent. exempt supplies.
• Invoicing • Invoice has to be raised for • No sales tax invoice shall be
Sales the goods supplied raised.
• Tax
Requirements
• Registration • A person engaged in zero rated • A person engaged exclusively in
supplies has to be registered with the supply of exempt goods is not liable to
Sales tax department. be registered.
• Input tax • Input tax paid related to zero • Input tax paid related to exempt
credit rated supplies is refundable. supplies is inadmissible, therefore, it is
neither adjustable nor refundable.

(iii) Collection of excess sales tax


▪ Any person who has collected or collects any tax or charge which was not payable as tax or charge or
which is in excess of the tax or charge actually payable and the incidence of which has been passed
on to the consumer, shall pay the amount of tax or charge so collected to the Federal Government.
▪ Any amount payable to the Federal Government shall be deemed to be an arrear of tax or charge
payable under this Act and shall be recoverable accordingly and no claim for refund in respect of such
amount shall be admissible.
▪ The burden of proof that the incidence of tax or charge has been or has not been passed to the consumer
shall be on the person collecting the tax or charge.

(b) The basis for such satisfaction may inter alia include the following:
(i) non-availability of the registered person at the given address;
(ii) refusal to allow access to business premises or refusal to furnish records to an authorized Inland
Revenue Officer;
(iii) abnormal tax profile, such as taking excessive input tax adjustments, continuous carry-forwards, or
sudden increase in turnover;
(iv) making substantial purchases from or making supplies to other blacklisted or suspended person;
(v) non-filing of sales tax returns;
(vi) on recommendation of a commissioner of any other jurisdiction;
(vii) any other reason to be specified by the Commissioner;

Answer # 10 Spring 2019 Q. 6

(a)
(i) Cottage industries are required to be registered when their:
▪ annual turnover from taxable supplies made in any tax period during the last twelve months
ending any tax period does exceed Rs. 8 million or
▪ Employees are more than 10

(ii) Those retailers are required to register under the Sales Tax Act who are liable to pay sales tax except
such retailers who are required to pay sales tax through their electricity bill.

(b) A registered person may be liable for deregistration due to any of the following reasons:
(i) He ceases to carry on his business;
(ii) His supplies become exempt from tax;

504 | P a g e
(iii) He transfers or sells his business;
(iv) Merger with another person; or
(v) Failure to file tax return for six consecutive months.
(c)
(i) The tax paid on goods purchased by a person who is subsequently registered under this Act or the
rules made thereunder, shall be treated as input tax, provided that such goods were purchased by him
from a registered person against a tax invoice issued under section 23 during a period of thirty days
before making the application for registration and constitute his verifiable unsold stock on the date
of application for registration.
Considering the above, BPL can claim input tax subject to availability of tax invoice and verifiability
of unsold stock.

(ii) Under the STA-1990, time of supply in relation to supply of goods means the time at which the goods
are delivered or made available to the recipient of the supply or the time when any payment is
received by the supplier in respect of that supply, whichever is earlier.

However, in respect of exempt supply, it shall be accounted for in the return for the tax period during
which the exemption is withdrawn from such supply.
Considering the above, BPL has to include advance payment of Rs. 2.5 million received from a
registered person in its sales tax return for the month ended February 2019.

Answer # 11 Autumn 2019 Q. 6

(a) (i) Where any part payment is received in a tax period in respect of a:
▪ taxable supply, it shall be accounted for in the return for that tax period; and
▪ exempt supply, it shall be accounted for in the return for the tax period during which the exemption is
withdrawn from such supply.

(ii) Change in the rate of tax


If there is a change in the rate of tax,
▪ a taxable supply made by a registered person shall be charged to tax at such rate as in force at the time
of supply.
▪ imported goods shall be charged to tax at such rate as is in force -
− in case the goods are entered for home consumption, on the date on which a goods declaration is
presented.
− in case the goods are cleared from warehouse, on the date on which a goods declaration for
clearance of such goods is presented.

▪ where a goods declaration is presented in advance of the arrival of the conveyance by which the goods
are imported, the tax shall be charged as is in force on the date on which the manifest of the conveyance
is delivered.
▪ where the tax is not paid within seven days of the presenting of the goods declaration the tax shall be
charged at the rate as is in force on the date on which tax is actually paid.

If there is a change in the rate of tax during a tax period, a separate return has to be furnished in respect of
each portion of the tax period showing the application of different rates.

(b) Destruction of goods


In order to destroy the goods, the following conditions must be fulfilled:
(i) Prior permission from the Collector of Sales Tax having jurisdiction.
(ii) Goods should be destroyed under the supervision of an Inland Revenue officer of Sales Tax not below
the rank of an Assistant Collector as may be deputed by the Collector for the purpose.
(c)
Nature of return Filer Due date

505 | P a g e
(i) Monthly return Registered person 15th of next month following any tax period
(Electronic filing -18th of next month, where sales
tax payable with the return paid till 15th day as
specified above.)
(ii) Special return Registered or On the date specified by the Commissioner in its
Unregistered persons notice calling for such return.
(iii) Final return Person applied for
deregistration On the date specified by the Commissioner.
(iv) Annual return Every private or public
limited company 30th of September following the year end.

Answer # 12 Spring 2020 Q. 7

(a)
(i) Yes, the Commissioner is justified in issuing the notice to Raheel. According to STR 2006 if the
Commissioner Inland Revenue or any other officer, as may be authorized by the Board, after such
inquiry as deemed appropriate, is satisfied that a person is required to be registered, but does not
apply for registration. He may issue a notice to such person.
(ii) Under the STR 2006, Raheel may submit his response within the specified time, contesting his
liability to be registered. Based on his response, the Commissioner shall grant him opportunity of
personal hearing, if so desired by him, and shall there after pass an order whether or not such person
is liable to be registered compulsorily. He shall cause the said person to be registered through
computerized system.

However, if Raheel fails to respond within the time specified in the notice, the Commissioner shall
cause to compulsorily register him through computerized system.
(b)
(i) Features distinguishing the concept of ‘Zero rating’ from ‘Exempt supply’:
Distinction points Zero Rated Supply Exempt Supply
Definition “Zero rated supply means a taxable “Exempt Supply means a supply
supply which is charged to tax at the which is exempt from tax.
rate of zero per cent.
Products covered Goods exported or listed in the Fifth Goods specified by Federal
Schedule are charged to sales tax at Government in the Sixth Schedule
the rate of zero per cent. are exempt supplies.
Invoicing Invoice shall be raised for the goods No sales tax invoice be issued.
Requirements supplied but sales tax shall be charged
at the rate of zero per cent
Registration A person engaged in zero rated A person engaged exclusively in
supplies has to be registered with the the exempt supplies is not liable to
Sales tax department. be registered.
Input tax credit Input tax paid related to zero rated Input tax paid related to exempt
supplies is refundable. supplies is inadmissible, therefore,
neither adjustable nor refundable.

(ii) Short paid amounts recoverable without notice:


Where a registered person pays the amount of tax less than the tax due as indicated in his return, the
short paid amount of tax along with default surcharge shall be recovered from such person by
stopping removal of any goods from his business premises and through attachment of his business
bank accounts, without giving him a show cause notice and without prejudice to any other action
prescribed under section 48 of this Sales Tax Act or the rules made thereunder:

(iii) Provisional and final adjustment:


Monthly adjustment of input tax claimed by a registered person is treated as a provisional adjustment,

506 | P a g e
whereas at the end of each financial year, an adjustment is made on the basis of taxable and exempt
supplies made during the course of that year. This is termed as final adjustment.

Answer # 13 Autumn 2020 Q. 5

(a) Documents required for the deduction of input tax:


(i) A taxable supply of goods
Tax invoice bearing his name and registration number.

(ii) Goods imported into Pakistan


Bill of entry or a goods declaration showing his name and registration number, duly cleared by the
Custom authorities.

(iii) Goods purchased in an auction


Treasury receipt showing his name, registration number and the payment of sales tax.

(b) Retention of record and documents for six years


A person shall retain the record and documents for a period of six years after the end of the tax period to
which such record or documents relate or till the final decision in any proceedings.

Answer # 14 Autumn 2020 Q. 6

(i) The Commissioner Inland Revenue, having jurisdiction, may suspend the registration through the system,
without prior notice, pending further inquiry.
(ii) The suspension of registration shall take place through a written order of the Commissioner concerned,
giving reasons for suspension.
(iii) No input tax adjustment/refund shall be admissible to RA during the currency of suspension. Similarly, no
input tax adjustment/refund shall be allowed to any other registered persons on the strength of invoices
issued by RA (whether issued prior to or after such suspension), during the currency of suspension;
(iv) The Commissioner shall, within seven days of issuance of order of suspension, issue a show cause notice
(through registered post or courier service) to RA to afford an opportunity of hearing within fifteen days
of the issuance of such notice clearly indicating that he will be blacklisted, in case–
▪ there is no response to the notice;
▪ he has not provided the required record;
▪ he has not allowed access to his business record or premises; and
▪ any other reason specified by the Commissioner;

Answer # 15 Spring 2021 Q. 7

(a)
(i) Short paid amounts recoverable without notice:
Where a registered person pays the amount of tax less than the tax due as indicated in his return, the short
paid amount of tax along with default surcharge shall be recovered from such person by stopping removal
of any goods from his business premises and through attachment of his business bank accounts, without
giving him a show cause notice and without prejudice to any other action prescribed under section 48 of
this Sales Tax Act or the rules made thereunder:

(ii) Extra tax


The Federal Government is empowered to levy and collect tax at such extra rate or amount not exceeding
18% in addition to the amount of sales tax or retail tax, levied under Sales Tax Act, 1990. This tax shall
be levied on the value of such goods or class of goods, on such persons or class of persons, in such mode,
manner and at time and subject to such conditions and limitations as may be prescribed.

Capacity tax

507 | P a g e
On the goods specified in the Tenth Schedule, in lieu of levying and collecting tax on taxable supplies, the
tax shall be levied and collected, in the mode and manner specified therein on-:
▪ production capacity of plants, machinery, undertaking, establishments or installations producing or
manufacturing such goods; or
▪ fixed basis, as it may deem fit, from any person who is in a position to collect such tax due to the
nature of the business.

(b) Since Fahad has already accounted for the output tax in the sales tax return for the supplies, it can issue a
debit note in the month of February 2021 when the error was detected, and increase the amount of output
tax in the return for February 2021 by Rs. 45,000.

Time limitation of 180 days shall not apply in the given case as it is applicable only in the case of decrease
in output tax and increase in input tax. The above increase of output tax may be declared without any time
limitations.

Answer # 16 Autumn 2021 Q. 6

(a)(i) As this is covered under the definition of supply, this is subject to sales tax by applying tax % on open
market price of these goods.
(ii) The free replacement of defective parts is considered as original supply and not a separate supply.
Therefore, such replacement is not chargeable to tax.
(iii) Payment must be verifiable from the business bank accounts of both the buyer and the seller. In given
matter, company will not be able to obtain input tax on its payment due to non-verifiability of the said
payment from the business bank account of the company.
(iv) Value of supply in this case should be open market price so value of supply does not include the mark up.
(v) Time of supply is the earlier of delivery of goods or the time when payment is received so the advance
received is subject to sales tax in this month.
(vi) These are exempt from sales tax.
(vii) Input tax paid on purchase of construction material for office building is not admissible.
(viii) Input tax paid on purchase of electronic cash register is admissible and registered person shall be entitled
to deduct this input tax from output tax.

(b) Temporary registration


▪ Where a person files application for sales tax registration as a manufacturer without having installed
machinery, for the purpose of import of machinery to be installed by him, temporary registration as
manufacturer shall be allowed to him for a period of sixty days subject to furnishing of the complete
list of machinery to be imported along with Bill of Lading or Goods Declaration.
▪ The temporary registration shall be issued by the computerized system within seventy-two hours of
filing of the complete application.
▪ After receiving temporary registration, the person shall be allowed to import plant, machinery and raw
materials, etc. as a manufacturer, subject to submission to the customs authorities of a post-dated
cheque equal to the difference in duties and taxes to be availed as a manufacturer.
▪ In case the list of machinery is not provided within sixty days of issuance of the temporary registration,
such temporary registration shall be disabled and the post-dated cheques submitted shall be encashed.
▪ A person holding temporary registration shall file monthly return, but shall not issue a sales tax invoice
and if such invoice is issued, no input tax credit shall be admissible against such invoice.
▪ No sales tax refund shall be paid to the person during the period of temporary registration and the
amount of input tax may be carried forward to his returns for subsequent tax periods.

Answer # 17 Spring 2022 Q. 5(a)

508 | P a g e
Value of Supply Reason
1 1,000,000 Discount can be claimed if it is as per market norms and has
(800,000 ÷ 80%) been shown on tax invoice. Since the amount of discount has
not been shown on tax invoice, it shall be chargeable to tax at
gross amount.
2 900,000 Use of own manufactured items for in-house consumption will
(1,500,000×60%) be subject to sales tax. However, goods locally procured is not
deemed to be supply.
3 Nil Time of supply is the time at which goods are delivered or make
available to the recipient. Since goods were not delivered in
February, this was not chargeable to tax in the month of
February.
4 6,000,000 For taxable supplies specified in third schedule, sales tax is
(1000×6,000) charged on the retail price of goods.
5 Nil Free replacement of defective parts is considered as original
supply and not a separate supply so this was not chargeable to
tax in February return.
6 400,000 Cash discount shall not be deducted while computing value of
supply, so gross amount shall be chargeable to tax.

Answer # 18 Spring 2022 Q. 6

(a) Tier-1 retailers:


Tier-1 retailers are required to be registered under Sale Tax Act, 1990. They shall pay tax at the rate as
applicable to the goods sold under relevant provisions of the Sales Tax Act or a notification issue
thereunder.

Tier-1 retailers shall integrate their retail outlets with Board’s computerized system for real-time reporting
of sales.

In case a Tier-1 retailer does not integrate his retail outlet in the manner as prescribed during a tax period
or part thereof, the adjustable input tax for whole of that tax period shall be reduced by 60%.

Other than Tier-1 retailers:


Retailers other than those falling in Tier-1 are not required to be registered under Sales Tax Act, 1990. Tax
shall be charged from them, through their monthly electricity bills, at the rate of five percent where the
monthly bill amount does not exceed rupees twenty thousand and at the rate of seven and half per cent
where the monthly bill amount exceeds the rupees twenty thousand and the electricity supplier shall
deposit the amount so collected directly without adjusting against his input tax. The above tax is other
than normal tax of 17%, further tax of 3% and extra tax.

(b) Return of supply


SL shall issue a debit note (in duplicate) in respect goods returned, indicating the quantity being returned,
its value determined on the basis of the value of supply as shown in the tax invoice issued by the supplier
and the amount of related sales tax paid thereon, as well as the following, namely:
▪ name and registration number of SL;
▪ name and registration number of TPL;
▪ number and date of the original sales tax invoice;
▪ the reason of issuance of the debit note; and
▪ signature and seal of the authorized person issuing the note.

Answer # 19 Autumn 2022 Q. 6

509 | P a g e
(a)
1) All manufacturers of taxable supplies are required to be registered under the Sales Tax Act, 1990,
unless the manufacturer falls in the category of cottage industry. A manufacturer located in residential
area of Korangi, Karachi shall be considered to fall under cottage industry if it also fulfills the
following conditions:
▪ it does not have an industrial gas or electricity connection;
▪ it does not have a total labour force of more than ten workers; and
▪ its annual turnover from all supplies does not exceed ten/eight million rupees.
2) Tier 1 retailer is required to register while other than Tier 1 is not required to register.
3) Distributors of exempt supplies is not required to register.
4) An exporter of taxable goods who intends to obtain sales tax refund against his zero rated supplies is
required to register.

(b) Following is the list of goods which are exported but shall not be charged at the rate of zero percent:
▪ Goods exported but have been or are intended to be re-imported into Pakistan; or
▪ Goods which have been entered for export under section 131 of the Customs Act, 1969 (IV of 1969),
but are not exported; or
▪ Have been exported to countries specified by the Federal Government, by notification in the official
Gazette in this regard.

Answer # 20 Spring 2023 Q. 6

(b) Short paid amounts recoverable without notice:


Where a registered person pays the amount of tax less than the tax due as indicated in his return, the short paid
amount of tax along with default surcharge shall be recovered from a such person without giving him a show
cause notice and without prejudice to any other action prescribed under the Sales Tax Act or the rules made
thereunder, as follows:

• By stopping the removal of any goods from his business premises, and
• Through the attachment of his business bank accounts, Provided that no penalty under the Sales Tax
Act shall be imposed unless a show cause notice is given to the such person.

Answer # 21 Autumn 2023 Q. 6


(a) (i) Two persons cannot be associates solely by reason of the fact that one is employee
of another. Therefore, Jahanzeb is not an associate of Sarfraz.

510 | P a g e
(ii) A shareholder in a company and the company can be associates where the
shareholder either alone or together with an associate controls fifty percent or
more of the voting power in the company.
In the given scenario, Sarfraz together with his mother holds 50% shareholding
in FL so FL is the associates of Sarfraz.

(iii) Adopted child falls under the definition of relative. Therefore. Sarah is the
associates of Sarfraz.

(iv) Trust and any person who benefits or may benefit under the trust shall be
associates. In the given scenario, although the beneficiaries are orphans, UT may
be an associate of Sarfaraz, if he is deriving any benefit from the trust.

(b) Monthly return 15th of next month following any tax period (Electronic
filing – 18th of next month, where sales tax payable with the
return paid till 15th day as specified above.)
Annual return 30th of September following the year end
Special return On the date specified by the commissioner in its notice
calling for such return
Final return On the date specified by the commissioner

511 | P a g e
Chapter 26 SALES TAX NUMERICALS
PRACTICAL QUESTIONS ICAP CAF
Important Points Before Solving Numerical
1. General rate of sales tax is 18%. Further tax @ 4% shall also be charged when the goods are
supplied to any unregistered person or inactive taxpayer.
2. It means that the tax rate is 18% + 4% on supplies of goods to unregistered person or inactive
person. However, further tax shall not be charged in certain cases such as supply of goods directly
to end consumers including supplies by a retailer and items falling under 3rd. Schedule. Further
tax shall not become part of output tax and therefore it is payable to FBR as a bottomline figure.
3. Certain Imports are subject to 25% Rate and their subsequent supplies also charged at 25%.
4. Advance against sales was subject to sales tax and similarly input tax was claimable on
advance against purchases.
The Finance Act, 2021 has changed the definition of time of supply and now advance against
sales and purchases is not subject to sales tax.

5. Local supply against international tender is now subject to sales tax.

512 | P a g e
Sales Tax (Numerical Format)

Mr name
Tax period

Calculation of sales tax payable and refundable

Sales Tax Credits [Input Tax] Value Rate Amount

purchase from Registered Persons of Taxable Goods xxx 18% xxx


Purchases from Registered Person of Exempt Goods xxx Exempt
Packaging Material xxx 18% xxx
Import of raw material (XXX x 120%) xxx 18% xxx
Sales tax on utility bill xxx Given xxx
Purchase from Suspended seller xxx disallowed
purchase from blacklisted seller xxx disallowed
Purchase of zero-rated goods xxx 0% 0
Purchase from unregistered person xxx disallowed
Stationary purchased xxx disallowed
payment after 180 days xxx 18% ( xxx)
Tax not deposited by supplier xxx 18% ( xxx)
discrepancy indicated xxx 18% ( xxx)
Goods destroyed xxx disallowed
tax paid on crookery utensils and furniture xxx disallowed

Less : Purchase return (xxx) 18% (xxx)

Common Input tax XXX

Add: Input tax fixed assets xxx


total Input tax xxx

Sales tax debits (Output tax)

value Rate Amount


Sales to registerd person xxx 18% xxx
Sales to un registerd person xxx 18% xxx
Wrong discount given xxx 18% xxx
Sales of 3rd schedule xxx 18% xxx
Specified goods (Imports) xxx 25% xxx
sales to Reg in EPZ xxx 18% xxx

513 | P a g e
Free Replacement xxx N/A N/A
Advance Received xxx N/A N/A

total taxable supplies XXX XXX

Exempt supplies xxx N/A N/A


zero rated supplies xxx 0% 0

Total Supplies XXX XXX

Sales tax Payable/Refundable


Total output tax xxx
Less: Sales return (xx)
Net output tax xxx
Less : Input Tax Credit (Lower Of)
b/f input tax xx
Add: Input tax for the period xx

90% of output tax xx (xx)

Less : Input tax on fixed asset (xx)

Add : further tax sales to unreg(4%) xx

sales tax payable xxx

Input tax C/F xx


input tax C/F on fixed assets xx

Sales tax refundable xx


Less: Penalty , surcharge (xx)
net tax Refundable xxx

514 | P a g e
Question # 1 Autumn 2009 Q. 8(b)

Mr. Asif is registered under the Sales Tax Act, 1990. Following information for the month of August 2023 has
been extracted from his business records:
(i) Supplies made during the month were as follows:
Rupees
to registered persons 5,000,000
to unregistered persons 3,000,000
export supplies 11,000,000
exempt supplies 2,000,000

(ii) Goods costing Rs. 8,000,000 were purchased from registered persons
(iii) Goods purchased from unregistered persons amounted to Rs. 2,000,000 and were used exclusively for
making taxable supplies.

Required:
Compute the sales tax payable and/or to be carried forward by Mr. Asif in the return for the month of August
2023. (09)

Question # 2 Spring 2010 Q. 8

Mr.Kaleem is registered under the Sales Tax Act, 1990 as a manufacturer as well as a commercial importer. He
has provided you the following information for the month of February 2023:

Rs. in Million
Export sales – manufactured goods 30
Local sales of exempt manufactured goods 20
Taxable supplies – manufactured goods 120
Purchases
Local purchases of raw material from:
Registered persons 160
Unregistered persons 50

All the above amounts are exclusive of sales tax.


Required:
Compute the sales tax liability of Mr.Kaleem along with input tax to be carried forward (if any) in his sales
tax return for the month of February 2023 (Ignore the effect of minimum value addition in case of commercial
imports) (11)
Question # 3 Autumn 2010 Q. 6
Mr. Abdul Ghaffar is registered as a manufacturer, under the Sales Tax Act, 1990. He carried out the following
activities during the month of August 2023:
Rs. in “000”
Supplies
Manufactured goods
Local – taxable goods 22,000
Local – exempt goods 3,000
Exports 5,000
Purchases
Local purchases of raw material 8,000
Import of raw material 17,000

Other relevant information is as follows:

515 | P a g e
I. All the above amounts are exclusive of sales tax.
II. In July 2023, an amount of Rs. 365,000 was carried forward as sales tax credit
III. Sales tax is payable @ 18%
Required:
Compute the following for the month of August 2023:
(a) Sales tax payable / refundable.
(b) Input tax credit to be carried forward, if any (11)

Question # 4 Spring 2011 Q. 5


Maroof Engineering Limited (MEL) is registered under the Sales Tax Act, 1990. The company is engaged in
the manufacture and supply of spare parts. Following information has been extracted from the records of MEL
for the month of February 2023.
Rupees
Purchases
Local material:
From registered suppliers 15,000,000
From un-registered suppliers 8,000,000
Supplies
Manufactured goods
Local taxable supplies to registered persons 10,000,000
Local taxable supplies to un-registered persons 3,000,000
Export to Taiwan 10,000,000
Exempt goods 2,000,000

Following additional information is also available:


i. Purchases from registered suppliers include an amount of Rs. 1.0 million which was invoiced on May 15,
2022. The input tax on this invoice could not be claimed in the relevant period
ii. Material purchased from un-registered suppliers was exclusively used for making taxable supplies
iii. Goods worth Rs.500,000 were returned by different customers. Proper debit/credit notes were raised within
the specified period.
iv. A new machinery of Rs. 2.4 million was purchased and put to use during the same month
v. MEL’s purchases from registered suppliers include material worth Rs. 2 million against which an advance
was paid in the month of January 2023. However, due to a dispute, sales tax invoice was delayed and was
received by the company after filing of return.
vi. Parts worth Rs. 15,000 were delivered to the CEO for his personal use, free of cost.
vii. Sales tax credit of Rs. 50,000 was brought forward from previous month.
viii. Rs. 20,000 was paid to a courier company for delivering gifts to MEL’s high value customers.
Sales tax is payable at the rate of 18%. All the above amounts are exclusive of sales tax.
Required:
(a) Compute the sales tax payable/refundable.
(b) Input tax credit to be carried forward, if any. (15)

Question # 5 Autumn 2011 Q. 6


Mr. Agha is registered under the Sales Tax Act, 1990. He is engaged in the supply of household appliances and
has provided you the following information for the month of August 2023:
1) Supplies made during the month were as follows:

Rupees
Local taxable supplies to registered persons 35,500,000
Local taxable supplies to unregistered persons 1,700,250
Exports to USA and Canada 25,500,000
Supplies of exempt goods 5,235,000

Goods worth Rs. 1,500,000 were returned by a registered person. Proper debit/credit notes have been issued in

516 | P a g e
this regard

2) Following purchases were made during the month:

Rupees
Purchases from registered persons 54,550,000
Purchases from unregistered persons 10,600,000

Goods purchased from unregistered persons were exclusively used for making taxable supplies. An amount of
Rs. 750,000 is payable to a registered person since 1 February 2023

3) Sales tax credit of Rs. 610,000 has been brought forward from previous month.
4) Sales tax is payable at the rate of 18%. All the above amounts are exclusive of sales tax.
5) Agha is also required to pay a penalty of Rs. 10,000 under the Sales Tax Act, 1990 on account of certain
defects in the maintenance of records.
Required:
Compute the sales tax payable/(refundable) by/to Mr. Agha along with input tax to be carried forward, if any,
in the sales tax return for the month of August 2023. (12)

Question # 6 Spring 2012 Q. 7


Mr. Folad is registered under the Sales Tax Act, 1990 and is engaged in the business of manufacture and supply
of home appliances. Following information has been extracted from the records of Mr. Folad for the month of
February 2023.

Rupees
Purchases – Local
▪ From registered suppliers 70,250,000
▪ From un-registered suppliers 15,750,000

Supplies:
▪ Local taxable supplies to registered persons 72,870,000
▪ Local taxable supplies to un-registered persons 9,850,000
▪ Exports to Canada and USA 12,700,000

Following additional information is also available:


1. A new machine purchased for Rs. 12 million was commissioned into operations during February 2023
2. Sub-standard supplies amounting to Rs. 4,500,000 were returned to vendors. Proper debit/credit notes were
raised in this regard
3. Goods worth Rs. 7,200,000 were returned by different customers. Proper debit/credit notes were raised
within the specified period.
4. An amount of Rs. 820,000 on account of purchases made from a registered supplier is outstanding since
July 2022. The related input tax was accounted for in the relevant tax period
5. Sales tax credit brought forward from previous month amounted to Rs. 910,500.

Sales tax is payable at the rate of 18%. All the above figures are exclusive of sales tax.
Required:
Compute sales tax payable/refundable and input tax credit to be carried forward, if any. (12)

Question # 7 Autumn 2012 Q. 9

Zainab is registered under the Sales Tax Act, 1990 and is engaged in the manufacture and supply of Products A
and B. Following information has been extracted from her records for the month of February 2023:

517 | P a g e
Product A Product B
Rupees
Supplies
Local supplies 5,350,000 1,010,000
Exports to Thailand 2,550,000 3,950,000
Purchases
Local materials from registered persons 6,000,000
Local materials from unregistered persons 850,000

Additional information:
i. Product A is exempt from the charge of sales tax.
ii. Sales tax credit brought forward from previous month amounted to Rs. 262,500.
iii. Substandard supplies worth Rs. 150,000 were returned to the registered vendors and proper debit and credit
notes were issued.
iv. An invoice dated September 3, 2022 amounting to Rs. 100,000 had not been claimed inadvertently. This
oversight was detected in the month of February 2023.
v. Sales tax is payable at the rate of 18%. All the above amounts are exclusive of sales tax.
Required:
In the light of Sales Tax Act, 1990 and rules made thereunder, calculate the following for the month of February
2023:
a) Sales tax payable / refundable
b) Input tax to be carried forward, if any (11)
c)
Question # 8 Spring 2013 Q. 7(b)

Mr. Clever, a manufacturer of household appliances, is registered under the Sales Tax Act, 1990. Following
information has been extracted from his records for the month of February 2023:

Rupees
Supplies
▪ Local supplies of manufactured goods to registered persons 26,860,000
▪ Local supplies of manufactured goods to unregistered persons 3,550,000
▪ Local supplies of zero-rated goods 1,250,000
▪ Exports to Malaysia 15,000,000
Local purchases
▪ Registered persons 40,550,000
▪ Unregistered persons 5,000,000

Following additional information is also available:


i. Supplies worth Rs. 1,300,000 were returned by different registered persons. Proper debit/credit notes were
raised within the specified time
ii. Local purchases from registered persons include an invoice of Rs. 60,000 which was issued in the name of
Mr. Clever’s uncle.
iii. A new machine amounting to Rs. 3,000,000 was imported from China and put into operation during the
same month
iv. Sales tax credit of Rs. 410,000 was brought forward from previous month.
Sales tax is payable at the rate of 18%. All the above amounts are exclusive of sales tax.
Required:
Compute the sales tax payable by/refundable to Mr. Clever along with input tax to be carried forward, if any, in
the sales tax return for the month of February 2023. (12)

Question # 9 Autumn 2013 Q. 7

518 | P a g e
Faizan is registered under the Sales Tax Act, 1990 and is engaged in the business of manufacture and supply of
engineering goods. Following information has been extracted from his records for the month of August 2023.
1) Supplies made during the month were as follows:

Gross Discounts Net amount


amount allowed
Rupees
Local supplies to registered persons
• Noori Limited 16,000,000 800,000 15,200,000
• Soori Limited 4,000,000 400,000 3,600,000
Local supplies to unregistered persons 4,200,000 210,000 3,990,000
Exports to Jordan 6,000,000 -- 6,000,000

2) Faizan normally allows 5% discount to all its customers. However, as a special case, a discount of 10% was
allowed to Soori Limited. All the discounts were shown on the invoice
3) Supplies worth Rs. 617,500 (net of discount) were returned by Noori Limited. Proper debit and credit notes
were issued in this regard.
4) Following purchases were made during the month:
Rupees
Local materials from registered persons 27,000,000
Local materials from unregistered persons 3,000,000

5) Records indicate that a pump and a motor were given to Faizan’s friend, free of cost. The list price of the
pump and motor was Rs. 33,000
6) Faizan is required to pay a penalty of Rs. 10,000 under the Sales Tax Act, 1990 on account of certain defects
in the maintenance of records.
7) Sales tax credit brought forward from previous month amounted to Rs. 850,280.
8) Sales tax is payable at the rate of 18%. All the above figures are exclusive of sales tax.

Required:
Compute the sales tax payable by or refundable to Faizan along with input tax to be carried forward, if any, in
the sales tax return for the month of August 2023. (13)

Question # 10 Spring 2014 Q. 9

Zaheer is registered under the Sales Tax Act, 1990. He is engaged in the manufacture and supply of spare parts.
Following information has been extracted from the records for the month of February 2023.

Rupees
Local purchases of raw-material:
from registered suppliers 23,000,000
from un-registered suppliers 9,000,000
Supplies of manufactured goods:
local taxable supplies to registered persons 12,000,000
local taxable supplies to un-registered persons 4,000,000
local exempted supplies to registered persons 3,000,000
exports to. Malaysia 11,000,000

Following additional information is also available:


(i) Raw materials of Rs. 1.5 million were purchased from a registered supplier. Invoice was received on 15
November 2022. However, the input tax on this invoice could not be claimed in the relevant period.
(ii) Taxable supplies amounting to Rs. 1.2 million were returned by different customers. Proper debit/credit
notes were raised in respect of such supplies.

519 | P a g e
(iii) Raw materials purchased from registered suppliers include an amount of Rs. 2.5 million against which
100% advance was paid in the month of January 2023. However, due to a dispute, sales tax invoice was
delayed and was received by the company after filing of return.
(iv) Sales tax credit of Rs. 1.2 million was brought forward from previous month. Sales tax is payable at the
rate of 18%. All the above figures are exclusive of sales tax.

Required:
Compute the sales tax payable by or refundable to Zaheer along with input tax to be carried forward, if any, in
the sales tax return for the month of February 2023. (13)

Question # 11 Autumn 2014 Q. 6

Ali Trading Company (ATC) is registered under the Sales Tax Act, 1990 and is engaged in the business of
manufacture and supply of consumer goods. Following information has been extracted from the records of
ATC for the month of August 2023.
Rupees
Supplies
Local supplies to wholesalers 14,500,000
Local supplies to distributors 10,254,980
| Exports 18,650,000
Local supplies to registered retailers 980,000
Supply of exempted goods 5,500,000

Purchases
Local purchases from registered persons 50,982,000
Local purchases from un-registered persons 9,200,000

Following additional information is also available:


(i) Supplies amounting to Rs. 540,000 were returned by registered retailers.
(ii) An early settlement discount of Rs. 250,000 was given to local distributors.
(iii) An amount of Rs. 500,000 was received from Imran Associates, representing 25% advance payment in
respect of supply of a special order. ATC will supply this order in November 2023.
(iv) Goods pledged with a bank, were disposed of by the bank for Rs. 4 million in satisfaction of debt owed
by ATC.
(v) Sales tax credit brought forward from previous month amounted to Rs. 854,700.
(vi) Proper debit and credit notes have been issued wherever necessary.
Sales tax is payable at the rate of 18%. All the above figures are exclusive of sales tax
Required:
Under the provisions of the Sales Tax Act, 1990 compute sales tax payable/refundable and input tax credit to
be carried forward, if any, for August 2023. (14)

Question # 12 Spring 2015 Q. 7

Bashir is registered under the Sales Tax Act, 1990 and is engaged in the business of export and supply of
consumer goods. Following information has been extracted from his records for the month of February 2023.

Rupees
Supplies
To registered persons 25,980,000
To unregistered persons 2,500,000
Exempt supplies 1,874,000
Export to USA 2,000,000

Purchases

520 | P a g e
Purchases from registered person 21,710,000
Import of a machine 2,500,000

Following additional information is also available:


(i) supplies to registered persons include goods amounting to Rs. 300,000 which were supplied to an
associated company at a special discount of 25%.
(ii) input tax amounting to Rs. 55,900 was paid in January, 2023 but inadvertently it could not be claimed in
the return for January 2023.
(iii) a registered supplier had supplied goods worth Rs. 500,000 to Bashir in February 2023. However, Bashir
did not receive the sales tax invoice from the supplier.
(iv) the imported machine was put into operation during February, 2023.
(v) sales tax credit of Rs. 410,000 is to be brought forward from January 2023.
Sales tax is payable at the rate of 18%. All the above amounts are exclusive of sales tax, wherever applicable.

Required:
Under the provisions of the Sales Tax Act, 1990 and Rules made thereunder, compute sales tax payable/
refundable and input tax credit to be carried forward, if any, for tax period February 2023. (13)

Question # 13 Autumn 2015 Q. 5

Rahbar is registered under the Sales Tax Act, 1990 and is engaged in the business of manufacture and supply
of specialized equipment. Following information has been extracted from his records for the month of August
2023.

Rupees
Supplies:
to corporate customers - registered 20,000,000
to Government hospitals - un-registered 3,780,000

Purchases - Raw material:


from cottage industry 550,000
from local registered persons 25,800,000

Following information is also available:

(i) Purchases from local registered persons include the following:


▪ Material worth Rs. 1,600,000 against which a discrepancy has been indicated by the CREST.
▪ Raw-material of Rs. 2,000,000 purchased from AB Enterprises on 2 August 2023. The payment was
made on the same day by pay order. On 15 August 2023, AB Enterprises informed Rahbar that with
effect from 1 August 2023 their registration has been suspended by the Commissioner Inland Revenue.
▪ Wires and cables of Rs. 500,000 and electrical and sanitary fittings of Rs. 900,000. These items were
used in the renovation of a factory building.
(ii) An electronic cash register was purchased from High Tech Limited at Rs. 250,000.
(iii) On 18 August 2023 Rahbar acquired a machine on operating lease from Aroma Limited. The total lease
rentals payable over the lease term of two years are Rs. 3,500,000. The fair value at the inception of the
lease amounted to Rs. 3,100,000.
(iv) On 28 August 2023, Rahbar paid sales tax of Rs. 170,000 on electricity bill.
(v) Own manufactured equipment worth Rs. 375,000 was used for internal testing purposes in R&D
department.
(vi) Rahbar made free replacement of faulty parts on request from three of his customers. These parts were
covered under warranty and had a market value of Rs. 175,000.

All the above figures are exclusive of sales tax, wherever applicable. Sales tax is payable at the rate of 18%.

521 | P a g e
Required:
Under the provisions of the Sales Tax Act, 1990 and Rules made thereunder, compute the amount of sales tax
payable by or refundable to Rahbar and the amount of sales tax to be carried forward, if any, for the tax period
August 2023. (18)
Note: show all relevant exemptions, exclusions and disallowances.

Question # 14 Spring 2016 Q. 5

Mulaqat Associates (MA), an association of persons, is registered under the Sales Tax Act, 1990 and is engaged
in the business of manufacture and distribution of various products. Following information has been extracted
from MA’s records for February 2023:
Rupees
Supplies:
Jet fuel to Pak Airways proceeding to Oslo 800,000
Taxable goods to registered customers 500,000
Taxable goods to un-registered customers 375,000

Purchases:
Taxable goods from registered suppliers 650,000
Taxable goods from un-registered suppliers 150,000
Exempt goods from registered suppliers 100,000
Imports - raw material 280,000

Following information is also available:


(i) Taxable goods purchased from registered suppliers include furniture of Rs. 45,000 which was acquired
for use in the office of marketing manager.
(ii) Imports include raw materials worth Rs. 125,000 for the manufacture of shaving cream, covered under
Third Schedule. However, en route from port to MA’s warehouse in Uthal a serious damage was caused
to the consignment. MA received insurance claim of Rs. 90,000 after surrendering the right of disposal of
consignment in favour of the insurance company.
(iii) MA purchased 150 bags of cement, covered under Third Schedule, for the construction of a bungalow for
managing partner. Cement was purchased at the wholesale price of Rs. 400 per bag. However, the retail
price was Rs. 500 per bag.
(iv) Advance of Rs. 268,000 was made to Nomi Corporation for the purchase of packing materials.
(v) Taxable goods to un-registered customers include goods worth Rs. 200,000 sold to cottage industry in
Bela. The rest of the goods were sold to educational institutions in Zhob.
(vi) On 15 February 2023 MA signed an agreement with Bali Traders (BT), a registered customer, for the sale
of goods worth Rs. 290,000. On 20 February 2023 the goods were made available to BT. However, BT
took the delivery of goods on 5 March 2023.
(vii) MA sold goods worth Rs. 52,000 to one of its customers on two months credit. The amount was inclusive
of 4% mark-up.
(viii) MA distributed free samples of one of its new detergents Zeta among corporate clients. The value of these
samples amounted to Rs. 65,000.
(ix) MA issued a debit note of Rs. 35,000 to Hali Brothers to rectify a mistake in MA’s sales invoice. The
invoice was originally raised in November 2022.
(x) On 1 February 2023 MA sold 4,000 packs of a new caramel ice cream, covered under Third Schedule, at
a discounted price of Rs. 100 per litre pack. The retail price of the ice cream was Rs. 160 per litre pack.
(xi) Sales tax credit brought forward from January 2023 amounted to Rs. 245,000. This amount was inclusive
of input tax of Rs. 120,000 paid on a chemical which could not be used before the expiry date and was
consequently destroyed in February 2023.

All the above figures are exclusive of sales tax, wherever applicable. Sales tax is payable at the rate of 18%.

Required:

522 | P a g e
Under the provisions of the Sales Tax Act, 1990 and Rules made thereunder, compute the amount of sales tax
payable by/refundable to MA and the amount of sales tax to be carried forward, if any, for the tax period
February 2023. (18)

Question # 15 Autumn 2016 Q. 5

Samaaj Associates (SA) is registered under the Sales Tax Act, 1990 and is engaged in the business of
manufacturing, trading and export of electronic, chemical and other consumer goods. Following information
has been extracted from SA’s records for the month of August 2023:
Rupees
Supplies:
To registered persons 2,500,000
To un-registered persons 875,000
To persons registered as exporter 625,000
Purchases:
Raw material from registered persons 930,000
Finished goods from un-registered persons 725,000
Packing material from registered persons 510,000
Local machinery from un-registered persons 360,000
Imports - finished goods 472,000

Packing material from registered persons include material worth Rs. 150,000 which was used for packing
electric motors. On 31 August 2023 these motors were still part of SA’s unsold stock.

Following transactions pertaining to August 2023 are not included in the above table:
(i) Sales tax of Rs. 70,000, Rs. 45,000 and Rs. 68,000 was paid in cash on electricity, gas and telephone bills
respectively.
(ii) SA purchased high quality cables and wires worth Rs. 250,000 from a registered supplier for the
installation of local machinery purchased from un-registered suppliers.
(iii) Three cartons of imported shampoo, falling under third schedule, were supplied to un-registered
distributors at a price of Rs. 110,000 per carton. The distributors normally supply such shampoo to retailers
at a price of Rs. 135,000 per carton.
(iv) Five electric kettles worth Rs. 75,000 were purchased for use in the offices of factory manager and first
line-supervisors of production workers.
(v) On 5 August 2023 SA received advance of Rs. 600,000 against supply of electric shavers to Bari
Electronics. SA agreed to deliver the goods in September 2023.
(vi) On 25 August 2023 SA issued discount coupons worth Rs. 450,000 to its customers for participating in
grand annual sales exhibition to be held in December 2023.

Other related information is as under:


(i) On 10 February 2023 SA purchased liquid nitrogen worth Rs. 300,000 from Mughal Chemicals (MC), a
registered supplier, on credit. On 15 August 2023 SA paid the outstanding amount to MC by way of a
crossed cheque drawn on SA’s bank account.
(ii) In April 2023 SA inadvertently charged sales tax of Rs. 58,000 instead of 85,000 on supply of chemicals
to one of its registered customers. So far, SA has not obtained permission from the Commissioner Inland
Revenue for revision of return.
(iii) In July 2023 SA claimed input tax of Rs. 80,000 on purchase of hydrochloric acid from JB Traders. The
supplier has not yet deposited the amount of sales tax collected from SA in Government treasury.

In July 2023 the excess of input tax over output tax amounted to Rs. 20,000. Whereas, unadjusted input tax in
excess of 90% of output tax amounted to Rs. 10,000.

All the above figures are exclusive of sales tax, wherever applicable. Sales tax is payable at the rate of 18%.

523 | P a g e
Required:
Under the provisions of the Sales Tax Act, 1990 and Rules made thereunder, compute the amount of sales tax
payable by or refundable to SA and the amount of sales tax to be carried forward, if any, for the tax period
August 2023. (18)
Note: Show all relevant exemptions, exclusions and disallowances.
Ignore value addition tax payable at import stage.

Question # 16 Spring 2017 Q. 7

Jahangir Ali (JA) is registered under the Sales Tax Act 1990. JA runs multiple businesses. Following
information has been extracted for the month of February 2023:
Rupees
Supplies
Taxable goods exported to Qatar 100,000
Taxable goods to registered customers 750,000
Taxable goods to unregistered customers 550,000

Purchases
Taxable goods from registered suppliers 3,000,000
Exempt goods from registered suppliers 70,000
Taxable goods from unregistered suppliers 95,000

The following further information is available:


(i) Taxable goods supplied to registered customers include goods amounting to Rs. 300,000 supplied to an
associated company at a special discount of 25%.
(ii) Taxable goods supplied to unregistered customers include goods worth Rs. 100,000 supplied to Saleem
Brothers (SB). JA did not charge any sales tax from SB as it has submitted an undertaking that it is a
cottage industry and exempt from sales tax under the Sixth Schedule of the Sales Tax Act, 1990.
(iii) Taxable goods purchased from registered suppliers include:
▪ goods worth Rs. 320,000 purchased from Akram Limited who was blacklisted on 25 February 2023
due to issuance of flying invoices.
▪ goods purchased from ZA Traders amounting to Rs. 30,000. ZA Traders did not declare this amount
in its tax return for the month of February 2023.
▪ a new machine purchased for Rs. 500,000 which was commissioned into operation during February
2023.
▪ office equipment of Rs. 200,000, purchased for the warehouse.
(iv) Goods pledged with a bank were sold by the bank in an auction for Rs. 1,000,000. The normal selling
price of these goods was Rs. 1,200,000.
(v) Excess of input tax over output tax brought forward from January 2023 was Rs. 110,000.

Rate of sales tax is 18%. All figures are exclusive of sales tax.

Required:
Under the provisions of the Sales Tax Act, 1990 and Rules made thereunder, compute the amount of sales tax
payable by or refundable to JA and the amount of sales tax to be carried forward, if any, for the tax period
February 2023. (17)

Question # 17 Autumn 2017 Q. 6

Cyma Associates (CA) is registered under the Sales Tax Act, 1990, as manufacturer-cum- distributor-cum-
retailer. Following information has been extracted from its records for the month of August 2023:

Rupees
Supplies

524 | P a g e
Taxable goods to registered persons 15,000,000
Taxable goods to unregistered persons 2,800,000
Exports 1,500,000
Exempt supplies 1,700,000

Purchases
Taxable goods from registered suppliers 20,000,000
Taxable goods from unregistered suppliers 1,800,000
Exempt goods from registered suppliers 400,000
Fixed assets (machinery) from a registered supplier 1,000,000

The following additional information is available for August 2023:

(i) Supply of taxable goods to registered persons include the following:


▪ Goods invoiced at Rs. 325,000 (net of special discount of Rs. 125,000) sold to a government official.
▪ On 1 August 2023, CA launched 'Halloween Tooth Brush' which is covered under 3rd schedule. The
retail price of the tooth brushes is Rs. 100 each. However, being the first month of launching, it was
sold at a discounted price of Rs. 75 each. 4,000 tooth brushes were sold in August 2023.

(ii) Exports include supply of taxable goods of Rs. 500,000 to a retailer in Export Processing Zone.
(iii) Exempt supplies include distribution of free samples of exempt goods among the vendors. Value of such
goods amounted to Rs. 80,000.
(iv) Purchases from registered suppliers include:
▪ material worth Rs. 350,000 the payment of which was made by depositing cash directly in the business
bank account of the supplier.
▪ material worth Rs. 800,000 against which a discrepancy has been indicated by the CREST.
▪ an amount of Rs. 2,000,000 paid for purchase of raw material. However, only 30% of the goods were
supplied during August for which sales tax invoice has been issued by the supplier.
(v) On 1 August 2023, CA executed an agreement with Majeed Sons (MS) for sale of locally purchased goods
worth Rs. 225,000. The agreement empowers MS to obtain delivery of these goods anytime it likes.
(vi) Supplies returned by different registered persons amounted to Rs. 756,000. Proper debit and credit notes
were raised within the specified time.
(vii) The auditors have proposed a provision against obsolete and expired stock of Rs. 285,000 which is lying
in CA's warehouse since January 2022.
(viii) Machinery purchased during the month was commissioned into operations on 31 August 2023.
(ix) Excess of input tax over output tax in July 2023 amounted to Rs. 75,000.

Except where otherwise specified, all figures are exclusive of sales tax. Rate of sales tax is 18%.

Required:
Compute the sales tax liability of CA for the month of August 2023. (17)

Question # 18 Spring 2018 Q. 7

Faiz Associates (FA) is a partnership concern and registered under the Sales Tax Act, 1990 as manufacturer-
cum-distributor. Following information has been provided by FA for the month of January 2023:

Rupees
Supplies
Taxable goods to registered customers 3,450,000
Taxable goods to un-registered customers 1,000,000
Consumable goods supplied on PIA’s international flight 500,000
Export 700,000

525 | P a g e
Purchases
Taxable goods from registered suppliers 2,000,000
Taxable goods from un-registered suppliers 450,000
Exempt goods from registered suppliers 600,000
Input tax brought forward from December 2022 265,000

Additional information:
(i) Supply of taxable goods to registered customers include the following:
▪ Goods amounting to Rs. 80,000 sold to Hafiz Brothers (HB) on 31 January 2023. HB started business
in January 2023 and had filed an application for registration under the Sales Tax Rules 2006 on 30
January 2023. However, no sales tax registration number was issued till 31 January 2023.
▪ Goods having market value of Rs. 600,000 which were supplied to Parveen Limited, an associated
company, for Rs. 500,000.
▪ An invoice erroneously issued for Rs. 450,000 whereas the correct amount of the invoice was Rs.
540,000.
▪ Sale to Ghalib Corporation of goods worth Rs. 225,000. The contract for sale has been signed but
neither invoice was issued nor any delivery and payment was made in January 2023.

(ii) Purchases from registered suppliers include:


▪ purchase of two air-conditioners amounting to Rs. 150,000 for FA's new office.
▪ an invoice of Rs. 500,000 dated 22 January 2023 issued by Taqi Corporation (TC). However, TC was
blacklisted by the Commissioner on 6 February 2023.
(iii) FA destroyed certain goods worth Rs. 45,000 after following the due process under the Sales Tax Rules,
2006. Input tax on these goods was claimed in December 2022.
(iv) Free replacement of defective parts costing Rs. 400,000 relating to goods which were sold under 1-year
warranty. The market value of such parts was Rs. 550,000.
(v) A debit note for Rs. 100,000 issued by a customer in respect of goods returned was duly settled and the
relevant credit note has been issued within the stipulated time.
(vi) During the month, FA paid Sindh Sales Tax worth Rs. 8,500 on franchise services. Under the Sindh Sales
Tax Laws, such tax is not an admissible credit.

All the above figures are exclusive of sales tax, wherever applicable. Sales tax is payable at the rate of 18%.

Required:
Compute sales tax payable by or refundable to Faiz Associates along with input tax to be carried forward, if
any, in the sales tax return for the month of January 2023. (18)
Note: Show all relevant exemptions, exclusions and disallowances.

Question # 19 Autumn 2018 Q. 8

Abid Khan is registered for sales tax purposes and is engaged in the manufacturing of electrical appliances in
Multan. His sales and purchases for the month of August 2023 are summarized below:

Rupees
Supplies
Taxable goods to registered customers 15,118,000
Taxable goods to un-registered customers 10,150,000
Exports 5,000,000
Exempt supplies 4,500,000

Purchases
Taxable goods from registered suppliers - for taxable supplies 25,000,000
- for exempt supplies 1,500,000
Packing materials from un-registered suppliers 9,500,000

526 | P a g e
Additional information:
(i) Supplies of taxable goods to registered customers include:
▪ an amount of Rs. 4,225,000 against sale of electric toasters at a trade discount of 35%. As per normal
business practice, he allows a discount of 10% only.
▪ goods supplied against which advance payment of Rs. 2,500,000 had been received in June 2023.

(ii) Taxable supplies returned by different registered customers amounted to Rs. 900,000. Proper debit and
credit notes were raised within the specified time.
(iii) A plant costing Rs. 2,700,000 was commissioned into operation on 15 August 2023. The plant is being
used for taxable supplies only.
(iv) An electricity bill of Rs. 2,600,000 was paid in cash which includes sales tax amounting to Rs. 350,000.
(v) Input tax brought forward from July 2023 is Rs. 595,000.

All the above figures are exclusive of sales tax, wherever applicable. Sales tax is payable at the rate of 18%.

Required:
Compute sales tax payable by or refundable to Abid Khan along with input tax to be carried forward/refundable,
if any, in the sales tax return for the month of August 2023.
Note: Show all relevant exemptions, exclusions and disallowances. (14)

Question # 20 Spring 2019 Q. 7

Following information has been extracted from the records of four registered persons for the month of February
2023:
Registered Persons
A B C D
--------------------Rupees--------------------
Purchases
Taxable supplies from registered persons 1,500,000 1,500,000 - -
Taxable supplies from un-registered persons - - 1,500,000 1,500,000
Fixed assets (machinery) from a registered supplier - - - 2,500,000

Supplies
Taxable supplies to registered persons 1,200,000 - 1,000,000 1,000,000
Exempt supplies to registered persons 300,000 - - 800,000
Taxable supplies to un-registered persons - - 800,000 -
Zero rated supplies 300,000 1,800,000 - -

All the above figures are exclusive of sales tax, wherever applicable. Sales tax is payable at the rate of 18%.

Required:
In the light of the provisions of the Sales Tax Act, 1990 and Rules made thereunder, compute the amount of
sales tax payable by or refundable to each of the above registered persons and input tax to be carry forward, if
any, for the tax period February 2023. (15)

Question # 21 Autumn 2019 Q. 7

MH Associates (MHA) is registered under the Sales Tax Act, 1990 as a manufacturer, distributor and retailer.
Following information has been provided by MHA for the month of August 2023:

Rupees
Supplies
Taxable goods to registered persons 7,850,000

527 | P a g e
Taxable goods to unregistered persons 815,000
Exempt goods to unregistered persons 800,000

Purchases
Taxable goods from registered persons 5,400,000
Exempt goods from registered persons 1,500,000
Taxable goods from unregistered persons 1,100,000

Additional information:
(i) Supplies of taxable goods to registered persons include an invoice erroneously issued to Rasheed for Rs.
270,000 whereas the correct amount of invoice was Rs. 720,000.
(ii) Supplies of taxable goods to unregistered persons include sale of Rs. 365,000 to end consumers.

(iii) Purchases from registered suppliers of taxable goods include:


▪ an amount of Rs. 1,800,000 paid for purchase of raw material. However, only 40% of the goods were
supplied during August 2023.
▪ goods worth Rs. 1,200,000 against which a discrepancy has been indicated by the CREST.

(iv) Two machines A and B costing Rs. 900,000 and Rs. 1,200,000 respectively were acquired and
commissioned into operation on 15 August 2023. Machine A has been used for taxable supplies only
whereas Machine B has been used for exempt supplies only.
(v) Input tax amounting to Rs. 120,000 was paid on 15 March 2023 but inadvertently it could not be claimed
in the return for March 2023 and thereafter.
(vi) An electricity bill of Rs. 670,000 was paid in cash which included sales tax amounting to Rs. 95,000.
(vii) Taxable supplies of Rs. 90,000 were returned by the registered customers during the period. Proper
debit/credit notes were issued within the specified time.
(viii) Sales tax credit brought forward from previous month amounted to Rs. 255,000.
Except where otherwise specified, all figures are exclusive of sales tax. Rate of sales tax is 18%.
Required:
In the light of the provisions of the Sales Tax Act, 1990 and Rules made thereunder, compute the amount of
sales tax payable by or refundable to MHA and input tax to be carry forward, if any, for tax period August
2023. (16)

Question # 22 Spring 2020 Q. 6

Following information has been extracted from the records of two different persons registered under the Sales
Tax Act, 1990 for the month of February 2023:
Registered persons
Taha Shan
---------Rupees---------
Purchases
Taxable supplies from registered persons - 11,000,000
Taxable supplies from unregistered persons 3,500,000 -
Exempt goods - 3,000,000
Fixed assets (machinery) from a registered supplier (Note A) 5,000,000 6,000,000

Supplies
Taxable supplies to registered persons - 10,000,000
Taxable supplies to unregistered persons 2,000,000 -
Exempt supplies to registered persons 3,800,000 5,500,000
Zero rated supplies 2,500,000 -
Note A:
▪ In case of Taha, the machinery has been used for exempt as well as zero rated supplies.
▪ In case of Shan, the machinery has been used for taxable supplies only.

528 | P a g e
All the above figures are exclusive of sales tax. Sales tax is payable at the rate of 18%.
Required:
In the light of the provisions of the Sales Tax Act, 1990 and Rules made thereunder, compute the amount of
sales tax payable by or refundable to each of the above registered persons and input tax to be carried forward,
if any, for the tax period February 2023. (13)

Question # 23 Autumn 2020 Q. 7

JF Associates (JFA) is registered under the Sales Tax Act, 1990 as a manufacturer. Following information has
been provided by JFA for the month of August 2023:
Rupees
Supplies
Taxable goods to registered persons 7,500,000
Taxable goods to unregistered persons 1,300,000
Exempt goods to unregistered persons 1,000,000
Exports to Saudi Arabia 500,000

Purchases
Taxable goods from registered persons 7,400,000
Taxable goods from unregistered persons 1,100,000
Fixed assets (machines) from a registered supplier 2,500,000

Additional information:
(i) Supplies of taxable goods to registered persons include:
▪ goods worth Rs. 560,000 sold to a new customer at discount of 20%. JFA normally allows discount
of 10% to its customers.
▪ an invoice issued to Qasim erroneously for Rs. 590,000 whereas the correct amount of invoice was
Rs. 950,000.
(ii) Supplies of taxable goods to unregistered persons include sales of Rs. 28,500 to end consumers.
(iii) Exempt supplies of Rs. 50,000 were returned by the unregistered customers during the period.
(iv) Two machines A and B costing Rs. 1,500,000 and Rs. 1,000,000 respectively were acquired and
commissioned into operation in August 2023. Machine A has been used for the manufacture of taxable
(local) as well as exempt supplies whereas Machine B has been used only for manufacture of export
supplies.
(v) Input tax on an invoice of Rs. 1,200,000 was paid on 15 March 2023 but inadvertently it could not be
claimed in the return for March 2023 and thereafter.
(vi) Electricity bill of Rs. 859,950 was paid in cash. The bill was inclusive of sales tax of Rs. 124,950.
(vii) Sales tax credit brought forward from previous month amounted to Rs. 425,000.

Except where otherwise specified, all figures are exclusive of sales tax. Rate of sales tax is 18%. Proper
debit/credit notes were issued within the specified time wherever required.

Required:
In the light of the provisions of the Sales Tax Act, 1990 and Rules made thereunder, compute the amount of
sales tax payable by or refundable to JFA and input tax to be carried forward, if any, for tax period August
2023. (18)

Question # 24 Spring 2021 Q. 6

Hadi Associates (HA), a sole proprietor business, is registered under the Sales Tax Act, 1990 as manufacturer
cum importer and is engaged in the manufacturing and supply of consumer products. Following information
has been extracted from HA’s records for the month of February 2023:
Rupees

529 | P a g e
Supplies
Taxable goods to registered customers 2,750,000
Taxable goods to un-registered customers 1,050,000
Exports of taxable goods to Saudi Arabia 1,500,000

Purchases
Taxable goods from registered suppliers 1,890,000
Taxable goods from un-registered suppliers 1,000,000
Packing material from un-registered suppliers 445,000

Additional information:
(i) Supplies of taxable goods to registered customers include:
▪ goods worth Rs. 225,000 (net of special discount of Rs. 75,000). These goods were sold to an
associated undertaking. The special discount was not reflected on the invoice.
▪ goods worth Rs. 120,000 supplied to a customer in Multan. HA had received full payment against the
goods in November 2023.

(ii) Supplies of taxable goods to unregistered customers include sales of Rs. 130,000 to end consumers.
(iii) Purchases from registered suppliers include:
▪ goods worth Rs. 100,000 purchased from Haq Enterprises on 5 February 2023. On 20 February 2023,
Haq Enterprises informed HA that with effect from 1 February 2023, its registration has been
suspended by the Commissioner Inland Revenue.
▪ goods worth Rs. 85,000 purchased in cash.
▪ goods worth Rs. 50,000 purchased from AB Traders. The supplier did not declare the sale of these
goods in its tax return for the month of February 2023.

(iv) Taxable goods worth Rs. 150,000 were used in the business meeting held for the promotion of HA’s
business.
(v) A machine costing Rs.2,500,000 was acquired and commissioned into operation in February 2023. The
machine was used for both taxable and zero rated supplies.
(vi) Electricity bill of Rs. 90,000 for the month of September 2022 was paid in October 2022. However, related
input tax of Rs. 13,000 has still inadvertently remained unclaimed.
(vii) The auditors have proposed to make a provision of 50% against obsolete and expired stock of Rs. 350,000.
The goods are lying in warehouse since July 2019. Input tax relating to this stock was claimed in July
2021.
(viii) Sales tax credit of Rs. 415,000 has been brought forward from previous tax period.

All the above figures are exclusive of sales tax, except where it is specified otherwise. Sales tax is payable at
the rate of 18%.

Required:
In the light of the provisions of the Sales Tax Act, 1990 and Rules made thereunder, compute the amount of
sales tax payable by or refundable to HA and input tax to be carried forward, if any, for the tax period February
2023. (16)

Question # 25 Autumn 2021 Q. 7

Mehrban Associates (MA) is registered under the Sales Tax Act, 1990. MA is engaged in the business of
manufacturing and supplying of various consumer goods. Following information is available from MA’s records
for the month of August 2023:
Rupees
Purchases
Taxable goods from registered persons 4,960,000
Taxable goods from unregistered persons 1,400,000

530 | P a g e
Exempt goods from unregistered persons 520,000

Supplies
Taxable goods to registered persons 8,650,000
Taxable goods to unregistered persons 1,560,000
Exempt goods to local unregistered persons 1,740,000
Export of taxable goods to UAE 1,300,000
Export of exempt goods to UAE 1,900,000

Additional information:
(i) Taxable goods from registered persons include:
▪ materials worth Rs. 296,000, which were exclusively used for manufacturing exempt supplies.
▪ materials worth Rs. 675,000, which were exclusively used for manufacturing export related goods.
▪ goods worth Rs. 150,000 which were purchased in cash from a supplier.
▪ 500 kg of tea purchased at a cost of Rs. 360,000 in one kg packing, covered under Third Schedule.
Retail price of tea per kg is Rs. 900. By end of August 2023, 300 kg were supplied to an unregistered
wholesaler at a price of Rs. 790 per kg.

(ii) Taxable goods supplied to unregistered persons include goods worth Rs. 320,000 which were sold to a
customer who did not provide his CNIC or NTN details. These goods were purchased from a registered
supplier for Rs. 275,000 during August 2023.

(iii) Following fixed assets were purchased during the month of August 2023:
Fixed assets Purchase cost (Rs.) Usage
Machine A 2,000,000 To ensure quality standards of packing for exports
To manufacture taxable (local) as well as exempt
Machine B 3,000,000
(local) goods
Furniture and fittings 1,000,000 To use in office premises

(iv) Electricity bill of Rs. 959,450 was paid in cash. The bill was inclusive of sales tax of Rs. 154, 250.
(v) Sales tax credit brought forward from last month amounted to Rs. 1,137,580.
(vi) Input tax of Rs. 186,000 pertaining to purchase made on 1 February 2023 was inadvertently remain
unclaimed.

All the above figures are exclusive of sales tax, except where it is specified otherwise. Sales tax is payable at
the rate of 18%.

Required:
In the light of the provisions of the Sales Tax Act, 1990 and Rules made thereunder, compute the amount of
sales tax payable by or refundable to MA and input tax to be carried forward, if any, for the tax period August
2023. (Show all relevant exemptions, exclusions and disallowances) (17)

Question # 26 Spring 2022 Q. 5(b)

Kazmi Traders (KT) is registered under the Sales Tax Act, 1990. KT is engaged in the business of manufacturing
and supply of paper products. Following information is available from KT’s records for the month of February
2023:
Rupees in
million
Purchases
▪ Taxable goods from registered persons (20% of these goods were returned to suppliers) 320
▪ Taxable goods from unregistered persons 32
▪ Exempt goods from registered persons 56
▪ Electrical and sanitary fittings (60% used in factory building and 40% used in office 17

531 | P a g e
building)
▪ Plant and machinery 84
Supplies
Taxable goods to registered persons (10% of these goods were returned by the customers) 200
Exports 98

Electricity bill paid in February 2023includes sales tax of Rs. 1.36 million. 60% of the amount was related to
the factory building while remaining amount was related to the office building.

All the above figures are exclusive of sales tax, except where it is specified otherwise. Sales tax is payable at
the rate of 18%

Required:
In the light of the provisions of the Sales Tax Act, 1990 and Rules made thereunder, compute the amount of
sales tax payable by or refundable to KT and input tax to be carried forward, if any, for the tax period February
2023. (Show all relevant exemptions, exclusions and disallowances) (08)

Question # 27 Autumn 2022 Q. 7

Zahid Enterprise (ZE) is registered under the Sales Tax Act, 1990. ZE is engaged in the business of
manufacturing garments. Usman who recently joined ZE as an officer, has prepared the following
computation of ZE’s sales tax liability for the month of June 2022:

Description Taxable Sales


Amount Tax
Input Tax: ---- Rs. in million ---
-
Purchases from registered persons 140 23.80
Purchases dated 21 December 2021 inadvertently not claimed in December
2021 return 10 1.70
Gas bill paid in cash 2 0.34
Electricity bill of rented premises showing particulars of landlord 3 0.51
Debit note dated 20 June 2022 issued to a supplier for taxable supply against
invoice dated 31 March 2022 6 1.02
Payment on 3 June 2022 against sales tax invoice dated 5 April 2022 70 11.90
Payment of provincial sales tax @ 13% against transporter’s invoice
dated 17 June 2022 for supplying goods to export processing zone 8 1.04
Advance paid to a supplier at the time of placing an order for
customized goods. The goods will be delivered in October 2022 12 2.04
Purchase of plant and machinery 20 3.40
45.75
Output Tax:
Sales of taxable goods to registered persons Note 1 180 30.60
Sales of taxable goods to unregistered persons Note 2 44 7.48
Credit note dated 15 June 2022 issued to a customer for taxable Note 3
supply against invoice dated 1 December 2021 (7) (1.19)
Garments withdrawn by owner during June 2022 4 -
36.89
Sales tax refundable 8.86

Note 1: It includes goods of Rs. 50 million supplied to the export processing zone for further
manufacturing.
Note 2: 20% of these customers are end users.

532 | P a g e
Note 3: Assume that the collector has not extended the time period for the issuance of debit or credit
note(s).

All the above figures appeared in taxable amount column are exclusive of sales tax. Sales tax is payable at
the rate of 17% except where it is specified otherwise.

Required:
In the light of the provisions of the Sales Tax Act, 1990 and Rules made thereunder:
(a) prepare ZE’s corrected computation of sales tax liability for the month of June 2022. (15)
(b) state the reason(s) for ignoring the transaction, if any, while computing ZE’s corrected sales tax
liability in part (a) above. (04)

Question # 28 Spring 2023 Q. 7

Confidence Engineering (CE), a sole proprietorship, is registered under the Sales Tax Act, 1990.
CE is engaged in the business of manufacturing and supplying auto parts. Following information
is available from CE’s records for the month of February 2023:

Rs. in million
Purchases
Taxable goods from registered persons 338
Taxable goods from unregistered persons 60

Supplies
Taxable goods to registered persons 220
Taxable goods to unregistered persons 40

Additional information:
(i) Taxable goods from registered persons include wires and cables of Rs. 6 million,
which were used in the repair of machinery.
(ii) Taxable goods from unregistered persons include goods worth Rs. 14 million
purchased from a cottage industry.
(iii) Taxable goods to registered persons include:
auto parts of Rs. 5 million, which were supplied free of cost in exchange of
defective parts covered under warranty.
auto parts of Rs. 4 million, which were withdrawn by the owner for his personal use.

raw materials of Rs. 2 million, which were given to factory engineer for his
personal use.
goods of Rs. 32 million, which were sold to Pray Traders (PT) at a credit term of
2/10, n/30. PT paid the amount within 10 days and availed the discount.
(iv) Taxable goods to unregistered persons include goods of Rs. 16 million provided to a
cottage industry.
(v) Raw materials of Rs. 25 million were destroyed due to a fire incident at the factory store.
All such materials were purchased from registered suppliers.
(vi) Raw materials of Rs. 26 million were returned to suppliers. The materials were
purchased in November 2022.
(vii) A sales tax invoice of Rs. 8 million dated 15 July 2022 was erroneously not declared in
the sales tax returns for the month of July 2022 and onwards.
(viii) Taxable goods of Rs. 19 million were returned by the customers. These goods were sold
in July 2022.

533 | P a g e
(ix) Electricity bill of Rs. 1.2 million was paid in cash.
(x) Sales tax credit brought forward from last month amounted to Rs. 27 million.

All the above figures are exclusive of Sales Tax which is payable at the rate of 18%. Assume that
proper debit/credit notes were issued for the purchase and sales returns.

Required:
(a) In the light of the provisions of the Sales Tax Act, 1990 and Rules made thereunder,
compute the amount of sales tax payable by CE and input tax to be carried forward, if any,
for the tax period February 2023. (Show all relevant exemptions, exclusions and
disallowances) (15)
(b) State the reason(s) for your treatment of all exemptions, exclusions and disallowances while
computing CE’s sales tax liability in part (a). (04)

Question # 29 Autumn 2023 Q. 7

Zeenat Enterprises (ZE), a sole proprietorship, is registered under the Sales Tax Act, 1990. ZE
is engaged in the business of manufacturing and supplying cosmetic products. The following
information has been extracted from the records of ZE for the month of August 2023:

Rs. in million
Purchases
Taxable goods from registered persons 44
Taxable goods from unregistered persons 9

Supplies
Taxable goods to registered persons 30
Taxable goods to unregistered persons 8
Exports 12

Additional information:
(i) Purchases of taxable goods from registered persons include:
raw material of Rs. 9.2 million (net of special discount of Rs. 1.2 million) from an
(xxxviii)
associated undertaking.
(viii) glass bottles worth Rs. 4 million. Among these, 20% of the total cost corresponds to
bottles that sustained damage in transit due to an accident. As a goodwill gesture, the
supplier provided replacement bottles without any additional consideration,
accompanied by a sales tax invoice
▪ a machine of Rs. 15 million procured from an importer. 60% of the payment was
made during the month, while the remaining amount is scheduled to be paid in equal
instalments over the next three months
▪ pigments in retail packing, listed in the Third Schedule, for Rs. 7.5 million. The
rretail price of these pigments is Rs. 7.9 million
(ii) Purchases of taxable goods from unregistered persons comprise of beeswax obtained from
a bee farmer.
(iii) Supplies of taxable goods to registered persons include various cosmetics products, listed
in the Third Schedule, sold to a distributor for Rs. 23 million. The retail price of these
products were Rs. 25 million. During the month, the distributor returned 10% of the goods.
Corresponding debit/credit notes were issued on 5 September 2023.
(iv) Supplies of taxable goods to unregistered persons comprise of various cosmetic
products, listed in the Third Schedule, to unregistered Tier-1 retailers.

534 | P a g e
(v) Exports comprise of various cosmetic products shipped to the UAE.
(vi) Packaging material worth Rs. 6 million were returned to the supplier on 10
August 2023, and corresponding debit/credit notes were issued on the same date. These
returned materials were purchased on 2 February 2023.
(vii) A purchase of raw material from a registered person for Rs. 2.9 million on 10
February 2023, was inadvertently omitted in the sales tax returns for the month of February
2023 and onwards.
(viii) Perfumes having the retail price of Rs. 1 million were provided to the marketing staff as
part of a promotional campaign, allowing walk-in-customers at malls and kiosks to try the
fragrances for free.

All the above figures are exclusive of sales tax, except where it is specified otherwise. Sales
tax is payable at the rate of 18%.

Required:
In light of the provisions of the Sales Tax Act, 1990 and the Rules made thereunder, compute
the amount of sales tax payable by or refundable to ZE and input tax to be carried forward, if any,
for the tax period August 2023. (Show all relevant exemptions, exclusions and disallowances)
(17)

535 | P a g e
CHAPTER 26
SALES TAX NUMERICALS
PRACTICAL SOLUTION
Answer # 1 Autumn 2009
Mr. Asif
Computation of Sales Tax Liability
for the Tax Period August 2024

RS. RS.
Sales tax payable/refundable
Output tax (W-2) 1,440,000
Less input tax (W-3) (548,571)
W-3
Input tax for the month 548,571
OR
90% of output tax(1,440,000*90%) 1,296,000
891,571
Add: further tax (3,000,000*4%) 120,000
Sales tax payable 1,011,000
Sales tax refundable (754,286)
Apportionment of input tax
Value Input tax
Taxable supplies 8,000,000 548,571
Exempt supplies 2,000,000 137,143
Zero rated supplies 11,000,000 754,286
Total 21,000,000 1,440,000

Sales tax credit: (W-1)


Value Rate Amount
Good purchased from registered person 8,000,000 18% 1,440,000
Good purchased from un registered person 2,000,000 - -
Input tax for the period 1,440,000
Sales tax debit: (W-2)
Value Rate Amount
Sales to registered perosn 5,000,000 18% 900,000
Sales to registered perosn 3,000,000 18% 540,000
Taxable supplies 8,000,000 1,440,000
Export 11,000,000 0% 0
Exempt supplies 2,000,000 N/A N/A

Answer # 2 Spring 2010

Computation of Sales Tax Liability


TAX PERIOD: FEBRUARY, 2024
RS. RS in millions.
Sales tax payable/refundable
Output tax (W-2) 21.6

536 | P a g e
Less input tax (W-3) (19.44)
W-3
Input tax for the month 20.32
OR
90% of output tax(1,440,000*90%) 19.44
Sales tax payable 2.16
Sales tax carryforward 0.88
Sales tax refundable 5.08
Apportionment of input tax
Value Input tax
Taxable supplies 120 20.32
Zero rated supplies 30 5.08
Exempt supplies supplies 20 3.4
Total 170 28.8
Sales tax credit: (W-1) Rs in “millions”
Value Rate Amount
Good purchased from registered person 160 18% 28.8
Good purchased from un registered person 50 - -
Input tax for the period 28.8
Sales tax debit: (W-2)
Value Rate Amount
Taxable supplies 120 18% 21.6
Exempt supplies 20 N/A N/A
Zero rated supplies 30 0% 0

Answer # 3 Autumn 2010 Q. 6

MR. ABDUL GHAFFAR


Computation of sales tax liability
For the period February 2023
Rs. in ‘000
Value Rate Amount
SALES TAX CREDIT (INPUT TAX)
Imports for domestic consumptions 17,000 18% 3,060
Local purchases 8,000 18% 1,440

Input Tax For the Period 4,500

SALES TAX DEBIT (OUTPUT TAX)


Domestic supplies of manufactured goods 22,000 18% 3,960
Exempt goods 3,000 N/A N/A
Export 5,000 0% 0
Total Output Tax 3,960

Sales Tax Payable/Refundable Rs (000) Rs (000)

Output Tax 3,960

Less: Input Tax Credit


a) Input tax for the Period (W-1) 3,300

537 | P a g e
or
b) 90% of Output Tax (3,960 x 90%) 3,564 (3,300)
Sales Tax Payable 660
Sales Tax Refundable (750)

W-1: APPORTIONMENT OF INPUT TAX


Value Input Tax
Taxable Supplier 22,000 3,300
Exempt Supplies 3,000 450
Zero Rated Supplies 5,000 750

Total 30,000 4,500

Answer # 4 Spring 2011 Q. 5

Computation of Sales Tax Payable/ Refundable


For the tax period February 2023
Rs. in ‘000
Gross Value Taxable Value
SALES TAX CREDIT (INPUT TAX)
Local purchases:
- From registered persons (Rs. 15.0 m – Rs.1.0 m) 14,000 2,520
- From unregistered persons 8,000 -

Fixed assets (Machinery) 2400 432


Material against which sales tax invoice has not been received 2,000 -
Input tax attributable to both taxable and exempted supplies 2,952
Less: Inadmissible/ un-adjustable input tax (W-1) (1,444)
Input tax for the month 1,508
(+) Previous month credit brought forward 50
Accumulated credit 1,558

Sales Tax Debit (Output Tax)


Domestic supplies of manufactured goods:
to registered persons 10,000 1800
to unregistered persons 3000 540
Export to Taiwan 10000 0
Exempt goods 2000 -
Parts provided to the CEO 15 2.7
Output tax for the month 2,344.7
Less: Sales return 500 (90)
Debit for the month 2,254.7
Admissible credit (90% of Rs. 2,254.7 or 1,558 whichever is lower) 1,558
Sales tax payable 696.7
Add Further tax on supplies to un-registered persons 120
Total sales tax payable with the return 816.7
Input tax to be carried forward -
Refund claim (input consumed in export) (W-1) 1204
W-1 : Apportionment of input tax Rs. In ‘000
Total residual input tax 2,952
Allocation of residual input tax to exempt and zero rated supplies
Exempt supplies (2,000/24,515 x 2,952) 240

538 | P a g e
Zero rated supplies (10,000/24,515 x2,952) 1,204
Inadmissible/ non-adjustable input tax 1,444

Answer # 5 Autumn 2011 Q. 6

Computation of Sales Tax Payable/ Refundable


For the tax period August 2023
Residual Input Tax
Local purchase from registered person (Rs. 54,550,000 x 18%) 9,819,000
Less: Input tax on purchases outstanding for more than 180 days (Rs. 750,000 x 18%) 135,000
9,954,000

Apportionment of residual Input tax:


Turnover Input Tax
Local taxable supplies to registered persons 35,500,000
Local taxable supplies to unregistered persons 1,700,250
Sales return (1,500,000)
35,700,250 5,347,897
Local Exempt Supplies 5,235,000 784203
Exports 25,500,000 3,821,900
66,435,250 9,954,000

Sales Tax Liability


Output tax 18% of Rs 37,200,250 6,696,045
Less: sales returns Rs. 1,500,000 x 18% (270,000)
6,426,045
Less: Input tax credit of previous tax period 610,000
Input tax against local taxable supplies 5,347,897
5,957,897
Actual input tax or 90% of output tax whichever Is lower 5,783,440
642,605
Add: Further tax @ 4% of Rs 1,700,250 68,010
Sales Tax Payable 710,615

Refund against exports 3,821,900


Less: Penalty under the sales tax laws (10,000)
Sales Tax refundable 3,811,900
Input tax carried forward (Rs. 5,783,440-5,957,897) 174,457

Answer # 6 Spring 2012 Q. 7

Computation of Sales Tax Payable/ Refundable


For the tax period February 2023
Input Tax
Local purchases from registered suppliers 70,250,000 x 18% 11,942,500
Input tax on machinery
12 m x 17% = 2.04m separate consideration --
Less: Input tax on purchase returns 4.5m x 18% 765,000
11,177,500

Apportionment of residual Input tax:


Input Tax
Turnover Input Tax
fixed Asset

539 | P a g e
Local supplies to registered person 72,870,000
Local supplies to unregistered person 9,850,000
Sales return (7,200,000)
75,520,000 9,568,406 1,746,325
Exports 12,700,000 1,609,094 293,675
88,220,000 11,177,500 2,040,000

Sales Tax Liability


Output tax 17% of Rs 82,720,000 14,062,400
Less: sales returns Rs. 7,200,000 x 18% (1,224,000)
12,838,400
Less: Input tax credit of previous tax period 910,500
Input tax against local taxable supplies 9,568,406
(10,478,906)
Input tax on fixed asset 2,359,494

Add: Further tax @ 4% of Rs 9,850,000 295,500


Sales Tax Payable 908,669

Refund against exports (1,609,094 + 293,675 ) 1,902,769

Answer # 7 Autumn 2012 Q. 9

Computation of Sales Tax Payable/ Refundable


For the tax period February 2023

Gross Value Taxable Value Sales Tax


Amount in Rupees
Sales tax credit (input tax)
Local purchases:
- from registered persons 6,000,000 6,000,000 1,020,000
- from unregistered persons 850,000 - -
1,020,000

(-) Purchase returned (150,000) (150,000) (25,500)


994,500
(+) Input tax, not claimed in the return for Sept 22 100,000 100,000 17,000

Input tax attributable to both taxable and exempt 1,011,500


supplies
Inadmissible/nonadjustable input tax (W-1) (932,058)
Input tax for the month 79,442
(+) Previous month credit brought forward 262,500
341,942

Sales tax debit (Output tax)


Taxable supplies to registered persons 1,010,000 1,010,000 171,700
Exports to Thailand 6,500,000 6,500,000 0
Exempt Supplies (7,900,000 - 2,550,000) 5,350,000 --
Output tax for the month 171,700
Admissible credit (Lower of 90% of Rs. 171,700 or (154,530)
input tax)
Sales tax payable 17,170

540 | P a g e
Input tax to be carried forward (Rs. 341,941 - Rs. 187,411
154,530)
Refund claim (W-1) 511,256
W-1: Apportionment of Residual Input Tax
Total input tax on purchases 1,011,500
Total supplies 12,860,000
Allocation of input tax to exempt and zero rated supplies
Input tax on exempt supplies 1,011,500 X (5,350,000)/12,860,000 420,803
Zero rated supplies 1,011,500X(3,950,000+ 2,550,000)/12,860,000 511,256
Inadmissible input tax 932,058

Answer # 8 Spring 2013 Q. 7(b)

Computation of Sales Tax Payable/ Refundable


For the tax period February 2023
Input Tax
Local purchases from registered suppliers (40,550,000-60,000) x 17% 6,883,300
Input tax on machinery
3 m x 17% = Rs. 510,000 separate considerations --
6,883,300

Apportionment of residual Input tax:


Input Tax
Turnover Input Tax
fixed Asset
Local supplies to registered person 26,860,000
Local supplies to unregistered person 3,550,000
Sales return (1,300,000)
29,110,000 4,417,391 327,295
Exports + Local zero rated supply 16,250,000 2,465,909 182,705
45,360,000 6,883,300 510,000

Sales Tax Liability


Output tax 17% of Rs 30,410,000 5,169,700
Less: sales returns Rs. 1,300,000 x 17% (221,000)
4,948,700
Less: Input tax credit of previous tax period 410,000
Input tax against local taxable supplies 4,417,391
4,827,391
Restricted to 90% of output tax (4,453,830)
494,870

Input tax on fixed asset 327,295


167,575
Add: Further tax @ 3% of Rs 3,550,000 106,500
Sales Tax Payable 274,075

Refund against exports (2,465,909 + 182,705) 2,468,614

Input tax c/f Rs. 4,827,391 – 4,453,830 373,561

Answer # 9 Autumn 2013 Q. 7

Computation of Sales Tax Payable/ Refundable

541 | P a g e
For the tax period August 2023

Input tax
Local purchases from registered suppliers 27m x 17% 4,590,000

Apportionment of residual Input tax:


Turnover Input Tax
Local supplies to
▪ Noori Ltd 15,200,000
▪ Sale return from Noori Ltd (617,500)
▪ Soori Ltd 4,000,000 – 5% discount 3,800,000
▪ Motor to a friend 33,000 – 5% discount 31,350
▪ Local taxable supplies to unregistered persons 3,990,000
22,403,850 3,620,413
Exports 6,000,000 969,587
28,403,850 4,590,000

Sales Tax Liability


Output tax 17% of Rs 23,021,350 3,913,630
Less: sales returns Rs. 617,500 x 17% (104,975)
3,808,655
Less: Input tax credit of previous tax period 850,280
Input tax against local taxable supplies 3,620,630
4,470,910
Restricted to 90% of output tax (3,427,789)
380,866

Add: Further tax @ 3% of Rs 3,990,000 119,700


Sales Tax Payable 500,566

Refund against exports 969,587


Less: Penalty 10,000
Refundable 959,587

Input tax c/f Rs. 4,491,323-3,427,789 1,063,534

Answer # 10 Spring 2014 Q. 9

Computation of Sales Tax Payable/ Refundable


For the tax period February 2023
Input Tax
Local purchases from registered suppliers 23,000,000 x 17% 3,910,000
Input tax on unclaimed invoice of November 1,500,000 x 17% 255,000
3 m x 17% = Rs. 510,000 separate considerations 4,165,000

Less: Input tax on purchase for which tax invoice not received (2,500,000 x 17%) 425,000
3,740,000

Apportionment of residual Input tax:


Turnover Input Tax
Local supplies to 16,000,000
Sales Returns 1,200,000
14,800,000 1,921,945

542 | P a g e
Local Exempt supplies 3,000,000 389,583
Exports 11,000,000 1,428,472
28,800,000 3,740,000

Sales Tax Liability


Output tax 17% of Rs 16,000,000 2,720,000
Less: sales returns Rs. 1,200,000 x 17% (204,000)
2,516,000
Less: Input tax credit of previous tax period 1,200,000
Input tax against local taxable supplies 1,924,945
3,121,945
Restricted to 90% of output tax (2,264,400)
251,600

Add: Further tax @ 3% of Rs 4,000,000 120,000


Sales Tax Payable 371,600

Refund against zero rated supplies 1,428,472

Input tax c/f Rs. 3,121,945 – 2,264,400 857,545

Answer # 11 Autumn 2014 Q. 6

Computation of Sales Tax Payable/ Refundable


For the tax period August 2023

Input tax
Local purchases from registered suppliers 50,982,000 x 18% 9,176,760

Apportionment of residual Input tax:


Turnover Input Tax
Local supplies to wholesaler 14,500,000
Local supplies to distributors 10,254,980
Local supplies to retailers 980,000
Pledged good sold by bank 4,000,000
Sale return (540,000)
Taxable supplies 29,194,980 5,022,120
Local Exempt supplies 5,500,000 946,110
Exports 18,650,000 3,208,530
53,344,980 9,176,760

Sales Tax Liability


Output tax 18% of Rs 29,734,980 5,352,296
Less: sales returns Rs. 540,000 x 18% (97,200)
5,255,096
Less: Input tax credit of previous tax period 854,700
Input tax against local taxable supplies 5,022,120
5,876,820
Restricted to 90% of output tax (4,729,586)

Sales Tax Payable 525,509

Refund against exports 3,208,530

543 | P a g e
Input tax c/f Rs. 5,876,820 – 4,729,586 1,147,234

Answer # 12 Spring 2015 Q. 7

Computation of Sales Tax Payable/ Refundable


For the tax period February 2023
Input Tax
Local purchases from registered suppliers (21,710,000 x 18% ) 3,907,800
Less: Invoice of Rs 500,000 not yet received (500,000 x 18%) (90,000)
Input tax not claimed in that period 3,817,800
Input tax on machinery (2.5m x 18% = Rs 450,000) separate consideration --

Apportionment of residual Input tax:


Input Tax
Turnover Input Tax
fixed Asset
Local supplies to registered person (Rs. 245,980,000 + 26,080,000
special discount Rs 100,000)
Local supplies to unregistered person 2,500,000
28,580,000 3,362,065 396,119
Exempt supplies 1,874,000 220,452 25,974
Exports zero rated supply 2,000,000 235,283 27,907
32,454,000 3,817,800 450,000

Sales Tax Liability


Output tax 18% of Rs 28,580,000 4,858,600
Less: Input tax credit of previous tax period 410,000
Input tax against local taxable supplies 3,362,065
Actual Input Tax 3,772,065
Or Lower of 90% of Output tax (3,772,065)
1,086,535

Input tax on fixed asset (396,119)


690,416
Add: Further tax @ 4% of Rs 2,500,000 100,000
Sales Tax Payable 790,416

Refund against exports (235,283+ 27,907) 263,190

Answer # 13 Autumn 2015 Q. 5

Computation of Sales Tax Payable/ Refundable


For the tax period August 2023
Input Tax
Rs. Rs.
Purchases from cottage industry (input tax is not allowed) --
Purchases from registered persons 25,800,000
Discrepancy indicated by the CREST: provisionally allowed --
Purchase from AB enterprises : disallowed being suspended (2,000,000)
Wires & Cables: input tax on building material not allowed (500,000)
Electric & Sanitary fitting: input tax on building not allowed (900,000)

544 | P a g e
Electronic cash register (250,000 x 17%) separate apportion. --
Leased machine : leased asset is not subject to sales tax --
22,400,000
18% sales tax on above 4,032,000
Input tax on electricity 170,000
Residual input tax other than fixed asset 4,202,000

Apportionment of residual Input tax:


Value of Input Tax Input Tax
supply fixed asset other
Supplies to corporate customers 20,000,000 37,847 3,534,062
Local supplies to unregistered person 2,500,000 7,153 667,938
23,780,000 45,000 4,202,000

Sales Tax Liability


3,600,000
Less: Input tax
Actual 3,534,062
90% of Output tax 3,240,000
Whichever is lower 3,240,000

360,000
Input tax on fixed asset (37,847)
Sales tax payable 322,153

Sales tax c’f (3534062-3240000) 294,062

Answer # 14 Spring 2016 Q. 5

Mulaqat Associates (MA)


Computation of Net Sales Tax Liability
For the tax period February 2023
Sales Tax Amount of Sales
SALES TAX CREDIT (INPUT TAX) Taxable Value
Rate Tax
Taxable goods from registered suppliers [650,000 – 605,000 18% 108,900
45,000]
Furniture for use in marketing manager’s office 45,000 inadmissible -
Taxable goods from un-registered suppliers 150,000 inadmissible -
Exempt goods from registered suppliers 100,000 - -
Import of raw material 280,000 18% 50,400
Purchase of cement [being personal in nature] 75,000 inadmissible -
[150x500]
Advance against purchase of packing material -
159,300

545 | P a g e
Add: Credit brought forward from previous month 245,000
Less: input tax on chemicals destroyed (120,000)
125,000
Input Tax for the month (Accumulated credit) 284,300
Less Input tax inadmissible W-1 44,794
239,506

SALES TAX DEBIT (OUTPUT TAX)


Jet fuel to Pak Airways 800,000 0% 0
Taxable goods to registered customers 500,000 18% 90,000
Taxable goods to Cottage Ind. In Bela 200,000 18% 36,000
Taxable goods to un-registered -end consumers 175,000 18% 31,500
Raw material to insurance company [treated as 90,000 18% 16,200
supply]
Taxable goods to Bali Traders 290,000 18% 52,200
Taxable goods on two months credit 50,000 18% 9000
Free samples of detergent Zeta 65,000 18% 11,700
Debit note issued to Hali Brothers 35,000 18% 6300
Caramel ice cream [4,000 x 160] 640,000 18% 115,200
Output tax for the month 368,100
Further tax on supplies to cottage Ind. Exempt
Further tax on supplies to un-registered end consumers-Exempt
Admissible credit (lower of 239,506 or 90% of 368,100 =239,506 (239,506)
Sales tax payable 128,594
Sales tax refundable on zero rated supplies 44,794
Total supplies 2,845,000
Inadmissible input tax [(800,000/2,845,000)×159,300] 42,305

Answer # 15 Autumn 2016 Q. 5

Samaaj Associates (SA)


Computation of Net Sales Tax Liability
For the tax period August 2023

SALES TAX CREDIT (INPUT TAX) Taxable Value Sales Tax Rate Amount of Sales
Tax
Raw material from registered persons 930,000 18% 167,400
Finished goods from un-registered persons 725,000 inadmissible -
Packing material from registered persons 510,000 18% 91,800
Local machinery from un-registered persons 360,000 inadmissible -
Imports – finished goods 472,000 18% 84,960
Sales tax paid on utility bills - - 183,000
[70,000+45,000+68,000]
Purchase of cables and wires 250,000 18% 45,000
Purchase of electric kettles 75,000 inadmissible -
Purchase of liquid nitrogen [more than 180 days] 300,000 inadmissible (51,000)
Input tax claimed on HCL (80,000)
441,160
Add: Credit brought forward from previous month [20,000+10,000] 30,000
Input Tax for the month (Accumulated credit) 471,160

546 | P a g e
SALES TAX DEBIT (OUTPUT TAX)
Taxable goods to registered persons 2,500,000 18% 450,000
Taxable goods to un-registered persons 875,000 18% 157,500
Taxable goods to registered exporters 625,000 18% 112,500
Self-consumption of packing material 150,000 - -
Shampoo to un-registered distributors [3 × 405,000 18% 72,900
135,000]
Advance against supply of electric shavers 600,000 18% -
Discount coupons to customers (actionable claims) 450,000 inadmissible -
Short payment of sales tax in April 2016 27,000 - -
Output tax for the month 792,900
Further tax on supplies to un-registered persons [875,000 × 4%] 35000
Further tax on supply of shampoo to un-registered distributors [3rd schedule item]
[405,000 × 0%] -
Admissible credit (lower of 471,160 or 90% of 792,900) (471,160)
Sales tax payable 356,740
Input tax to be carried forward Nil

Answer # 16 Spring 2017 Q. 7

Jahangir (JA)
Computation of Net Sales Tax Liability
For the tax period February 2023
SALES TAX CREDIT (INPLTT TAX) Taxable Sales Tax
value @ 18%
Taxable goods from registered suppliers 351,000
(3,000,000-320,000-30,000-500,000-200,000) 1,950,000
Exempted goods from registered suppliers 70,000 -
Taxable goods from unregistered suppliers 95,000 -
Fixed assets (Machinery) 500,000 90,000
441,000
Less: Inadmissible/un-adjustable input tax (W-1) (16,333)
Admissible input tax 424,667
Add: Excess of input tax over output tax 110,000
Input Tax for the month (Accumulated credit) 534,667

SALES TAX DEBIT (OUTPUT TAX)


Taxable goods supplied to Qatar (zero rated) 100,000 0
Taxable goods to registered persons (750-300) 450,000 81,000
Taxable goods to an associated undertaking at a special discount (300/0.75) 400,000 72,000
Taxable goods supplied to unregistered customers 550,000 99,000
Goods disposed of by the bank 1,200,000 216,000
Total supplies / Output tax for the month 2,700,000 468,000
Input tax on fixed assets 86,667
381,333
Admissible credit (90% of output tax i.e Rs. 343,200 (381,333 x 0.9 )or
input tax excluding Fixed Assets (461,000) whichever is lower (343,200)
Excess of output tax over input tax 38,133
Add: Further tax on supplies made to unregistered person [550,000 x 4%] 22,000
Sales tax payable 60,133

547 | P a g e
Input tax to be carried forward (461,000 - 343,200) 117,800

Sales tax refund on zero rate supplies (W-1) 16,333

W-1: Apportionment of input tax


Total supplies 2,700,000
Input tax 441,000

Inadmissible input tax relating to zero rate supplies


(100,000/2,700,000x441,000)) 16,333

Answer # 17 Autumn 2017 Q. 6

Cyma Associates
Computation of Sales Tax Payable/Refundable
For the tax period August 2023
Taxable value Sales tax Sales Tax
rate
------------------- Rupees -------------------
SALES TAX CREDITS (INPUT TAX)
Purchases from registered suppliers 18,850,000 18% 3,393,000
(20,000,000−350,000−800,000)
Purchases from unregistered suppliers 1,800,000 inadmissible
Purchases of exempt goods from registered suppliers 400,000 inadmissible
Purchases against which cash deposited in bank account 350,000 inadmissible
Purchases against which discrepancy indicated by CREST 800,000 inadmissible
Fixed assets (Machinery) 1,000,000 18% 180,000
Total input tax 23,200,000 3,573,000
Less: Inadmissible/unadjusted input tax (W-1) (466,179)
Input tax for the month 3,106,821
Add: Excess of input tax over output tax of July 2017 75,000
Accumulated credit 3,181,821

SALES TAX DEBIT (OUTPUT TAX)


Local taxable supplies - to registered suppliers 2,677,500
(15,000,000+500,000−325,000−300,000) 14,875,000 18%
Local taxable supplies - to unregistered persons 2,800,000 18% 504,000
Local taxable supplies - according to agreement with 40,500
Majeed 225,000 18%
Sons
Local taxable supplies - to government official 18% 81,000
(325,000+125,000) 450,000
Local taxable supplies - sale of tooth brushes 400,000 18% 72,000
18,750,000 -
Exempt supplies - no effect of free samples given 1,700,000 -
Export (zero rated) (1,500,000−500,000) 1,000,000 0
Output tax for the month 21,450,000 3,375,000
Less: Sales return (756,000) 18% (136,080)
Total supplies/Output tax for the month 20,694,000 3,238,920

Admissible credit (90% of output tax i.e. Rs. 2,915,028 (3,238,920×0.9) or 2,876,137 (2,915,028

548 | P a g e
input tax excluding Fixed Assets (3,181,821−180,000=3,001,821) whichever is )
lower.
323,892
Input tax on fixed assets (180,000 x 18,750,000 / 20,694,000) (163,090)
160,802
Further tax on supplies to unregistered persons = 2,800,000×4% 112,000
Sales tax payable 272,802

Sales tax to be carried forward (3,001,821−2,915,028) 104,270


Sales tax refundable (3,573,000×1,000,000/20,694,000) 172,658

W-1: Apportionment of input tax


Residual input tax 3,374,500
Exempt supplies and export sales (1,700,000+1,000,000) 2,700,000
Total supplies 21,450,000
Inadmissible input tax [(2,700,000/20,694,000)×3,573,000] 466,179

Answer # 18 Spring 2018 Q. 7

Faiz & Faraz Associates


Computation of sales tax payable/refundable
For the tax period January 2023

Taxable Sales Tax Sales Tax


Value Rate
------------------- Rupees -------------------
Sales tax credits – Input Tax
Purchases from registered suppliers 2,000,000
Purchases of air conditioners for office use 150,000
1,850,000 18% 330,000
Purchases from unregistered suppliers 450,000 inadmissible -
Purchases exempt goods from registered suppliers 600,000 exempt -
Invoice issued by Taqi Corporation which was declared
blacklisted in next period -
Sindh sales tax paid on franchise service - inadmissible -
Total input tax 330,000
Less: Inadmissible/unadjusted input tax (65,820)
Input tax for the month 264,180
Opening balance 265,000
Less: Input tax on goods destroyed (45,000×0.18) 8,100
256,900
Accumulated credit 521,080

Sales tax debits - Output Tax


Local taxable supplies 3,450,000
Local taxable supplies - *to Hafiz Brothers (80,000)
Local taxable supplies - to Ghalib Corporation (225,000)
Local taxable supplies - to Parveen Limited – associated
undertaking 100,000
(600,000–500,000)
Local taxable supplies - correction of invoice (540,000– 90,000
450,000)

549 | P a g e
3,335,000 600,300
- unregistered persons 1,000,000 U.R 180,000
Local taxable supplies - to Hafiz Brothers* 80,000 U.R 14,400
4,415,000
Export (zero rated) 700,000 Z.R -
Consumer goods supplied to PIA international flight 500,000 Z.R -
1,200,000
Output tax for the month 5,615,000 794,700
Less: Sales return (100,000) (18,000)
Total supplies/Output tax for the month 5,515,000 776,700
Free replacement of defective parts - -
Admissible credit [90% of output tax i.e. Rs. (776,700×0.9 =
699,030) or input tax Rs. 521,080 whichever is lower] (521,080)
255,620
Further tax on supplies to unregistered persons
(1,000,000+80,000)=1,080,000×4% 43,200
Sales tax payable 298,820

Sales tax to be carried forward -


Sales tax refundable 65,820

Answer # 19 Autumn 2018 Q. 8

Mr. Khan
Computation of sales Tax payable / Refundable
For the tax period August 2023

Sales Tax
Taxable Value Sales Tax
Rate
----------------- Rupees -----------------
Sales tax credits - input tax
Purchases from registered suppliers 25,000,000 4,500,000
Purchases from unregistered suppliers - packing material 9,500,000 inadmissible -
Purchases goods used exclusively for making exempt
supplies 1,500,000 inadmissible -
Cash payment of electricity bill inclusive of sales tax - 350,000
Fixed assets (Machinery) 2,700,000 486,000

Total input tax 38,700,000 5,336,000


Less: Inadmissible/unadjusted input tax (Note 1) (936,370)
Input tax for the month 4,399,630
Add: Input tax brought forward from previous month 595,000
Accumulated credit 4,994,630

Sales tax debits - output tax


Local taxable supplies - registered suppliers 1,510,740
(15,118,000–4,225,000–2,500,000) 8,393,000
Local taxable supplies - registered suppliers 1,053,000
(4,225,000×0.9÷0.65) 5,850,000
- unregistered persons 10,150,000 1,827,000

550 | P a g e
24,393,000 -
Export (zero rated) 5,000,000 -
Exempt supplies (Rs. 4,500,000) - -
Output tax for the month 29,393,000 4,390,740
Less: Sales return - taxable supplies (900,000) (162,000)
Total supplies / Output tax for the month 28,493,000 4,228,740
Admissible credit (90% of output tax i.e. Rs. 3,805,866 (4,228,740×0.9) or input tax
excluding Fixed Assets (4,994,630– 486,000= 4,508,630) whichever is lower. (3,805,866)
422,874
Input tax on fixed assets (486/28,493 x 24,393) (416,000)
6,874
Further tax on supplies to unregistered persons = 10,150,000×4% 406,000
Sales tax payable 412,874

Sales tax to be carried forward (4,508,630–3,805,866) 702,764


Sales tax refundable (5,059,000×5,000,000/28,493,000) 936,370

Note 1: Apportionment of input tax


Residual input tax 5,336,000
Total supplies 28,493,000
Inadmissible input tax i.e. Export [(5,000,000/28,493,000)× 5,336,000] 936,370

Answer # 20 Spring 2019 Q. 7

Taxable Sales tax


amount @ 18%
-------- Rupees --------
Registered person (A)
Input tax
Supplies from registered persons 1,500,000 270,000
Less: Inadmissible input tax/adjustable input tax
(270,000×600,000/1,800,000) (90,000)
Input tax for the month 180,000

Output tax
Taxable supplies to registered persons 1,200,000 216,000
Exempt supplies to registered persons 300,000 -
Zero rate supplies 300,000 -
Output tax for the month 216,000

Admissible credit [Lower of 180,000 or 194,400 (90% of 216,000)] 180,000


Sales tax payable (216,000 – 180,000) 36,000
Sales tax refundable (300,000/1,800,000×270,000) 45,000

Registered person (B)


Since all supplies are zero rated and all purchases are made from registered suppliers, the registered
person can claim the refund of all input tax i.e. 270,000 (1,500,000×18%).

Registered person (C)


Input tax
Supplies from un-registered persons 1,500,000 -

551 | P a g e
Output tax
Taxable supplies to registered persons 1,000,000 180,000
Taxable supplies to un-registered persons 800,000 144,000
Output tax for the month 324,000

Further tax on supplies to unregistered persons (800,000×4%) 32,000


Sales tax payable [324,000+32,000] 356,000

Registered person (D)


Input tax
Supplies from un-registered persons 1,500,000 -
Fixed assets (machinery) 2,500,000 450,000
Inadmissible input tax (800,000÷1,800,000×450,000) (200,000)
250,000
Output tax
Taxable supplies to registered persons 1,000,000 180,000
Taxable exempt supplies 800,000 -
Output tax for the month 180,000
Less: Input tax on fixed assets (250,000)
Sales Tax refundable/carry forward (70,000)

Answer # 21 Autumn 2019 Q. 7

MH Associates
Computation of sales tax payable/refundable
For the tax period August 2023

Taxable Sales Tax


Sales Tax
Value Rate
------------------- Rupees -------------------
Sales tax credits - Input Tax
Purchases taxable goods from registered suppliers
[5,400,000–1,200,000] 4,200,000 18% 756,000
No adjustment will be made for Rs. 1.8 million -
Purchases taxable goods from unregistered suppliers 1,100,000 -
Purchases exempt goods from registered suppliers 1,500,000 -
Un-claimed invoice for 15 Mar 2023 (6 month not
expired) 120,000
Cash payment of electricity bill 95,000
971,000
Fixed assets (Machinery) – used solely for taxable
supplies 900,000 18% 162,000
Fixed assets (Machinery) – used solely for exempt 216,000
supplies 1,200,000 18%
Total input tax 8,900,000 1,349,000
Less: inadmissible / unadjusted input tax (W-1) (295,064)
Input tax for the month 1,053,936
Add: Input tax brought forward from previous month 255,000
Accumulated credit 1,308,936

Sales tax debits – Output tax


Local taxable supplies – registered suppliers

552 | P a g e
(7,850,000–270,000+720,000) 8,300,000 18% 1,494,000
Local taxable supplies – unregistered persons
(450,000+365,000) 815,000 18% 146,700
9,115,000
Exempt supplies to unregistered persons 800,000 -
Output tax for the month 9,915,000 1,640,700
Less: Sales return – taxable supplies (90,000) 18% (16,200)
Total supplies / Output tax for the month 9,825,000 1,624,500
Admissible credit (90% of output tax i.e. Rs. 1,462,050
(1,624,500×0.9) or input tax excluding fixed assets (1,308,936– (1,146,936)
162,000=1,146,936) whichever is lower
477,564
Input tax on fixed assets – for taxable supplies only 162,000 (162,000)
315,564
Further tax on supplies to unregistered persons 450,000 4% 18,000
Sales tax payable 333,564

W-1: Appointment of input tax


Residual input tax 971,000
Exempt sales 800,000
Total supplies 9,825,000
Inadmissible input tax – [(800,000/9,825,000)×971,000] 79,064
Add: fixed assets (machinery) used for exempt supplies 216,000
295,064

Answer # 22 Spring 2020 Q. 6

Taxable amount Sales tax @ 18%


-------- Rupees --------
Registered person (Taha)
Input tax
Supplies from unregistered persons 3,500,000 -
Purchase of machine 5,000,000 900,000
Less: Inadmissible input tax / unadjustable input tax (900,000)
Input tax for the month -

Output tax
Taxable supplies to unregistered persons 2,000,000 360,000
Exempt supplies to registered persons 3,800,000 -
Zero rate supplies 2,500,000 -
Output tax for the month 360,000

Further tax on supplies to unregistered persons (2,000,000×4%) 80,000


Sales tax payable 440,000

553 | P a g e
Sales tax refundable [Input tax paid on machines relating to zero
rated supplies (900,000×2,500÷6,300)] 357,143

Registered person (Shan)


Input tax
Supplies from registered persons 11,000,000 1,980,000
Exempt goods 3,000,000 -
Machine for taxable supplies 6,000,000 1,080,000
3,060,000

Output tax
Taxable supplies to registered persons 10,000,000 1,800,000
Exempt supplies to registered persons 5,500,000 -
Output tax for the month 1,800,000

90% of output tax (1,800,000×0.9=1,620,000)(excluding tax on


fixed assets)(3,060,000–1,080,000 = 1,980,000)OR actual input
tax whichever is lower. 1,620,000
180,000
Input tax on machine 1,080,000
Excess of input tax over output tax (900,000)

Sales tax to be carried forward


[360,000 (1,980,000–1,620,000)+900,000] 1,260,000

Answer # 23 Autumn 2020 Q. 7

Taxable Sales tax


amount @ 18%
-------- Rupees --------
Sales tax credits - Input Tax
Purchases taxable goods from registered suppliers 7,400,000 1,258,000
Purchases taxable goods from un-registered suppliers 1,100,000 -
Un-claimed invoice for 15 Mar 2023 (6 months not expired) 1,200,000 204,000
Cash payment of electricity bill 735,000 124,950
1,586,950
Fixed assets (Machine-A) - used for taxable as well as exempt 1,500,000 255,000
supplies
Fixed assets (Machine-B) - used solely for taxable supplies only 1,000,000 170,000
(export)
Total input tax 12,935,000 2,011,950
Less: Inadmissible/unadjusted input tax (W-1) (409,253)
Input tax for the month 1,602,697
Add: Input tax brought forward from previous month 425,000
Accumulated credit 2,027,697

Sales tax debits - Output Tax


Local taxable supplies - registered persons
(7,500,000–560,000 –590,000) 6,350,000 1,079,500
Local taxable supplies – r .persons at discount (560,000/0.8×0.9)
630,000 107,100

554 | P a g e
Local taxable supplies - r. persons at discount – wrongly
recorded 950,000 161,500
Local taxable supplies - unreg. persons 1,300,000 221,000
Export to Saudi Arabia 500,000 -
9,730,000
Exempt Supplies to un-registered persons 1,000,000 -
Output tax for the month 10,730,000 1,569,100
Sales return - Exempt supplies (50,000) -
Total supplies / Output tax for the month 10,680,000 1,569,100
Admissible credit (90% of output tax i.e. Rs. 1,412,190 (1,569,100×0.9) or input
tax excluding Fixed Assets (2,027,697–231,203=1,796,494) whichever is lower. (1,412,190)
156,910
Less: Input tax on fixed assets - Machine "A" only (255,000–23,797) (W-1) (231,203)
Sales tax to be carried forward (fixed assets portion only) (74,293)
Further tax payable on supplies to unregistered persons [(1,300,000–28,500)×4%] 50,860
Sales tax to be carried forward (2,027,697–1,412,190–231,203) (384,304)
Sales tax refundable (export) (W-1) (74,295+170,000) (244,295)
W-1: Apportionment of input tax Supplies Fixed assets Total
A B
Residual input tax 1,586,950 255,000 170,000

Taxable supplies (Local) 9,230,000 9,230,000


Zero rate supplies 500,000 500,000
Exempt supplies (1,000,000–50,000) 950,000 950,000
Total supplies 10,680,000 10,180,000 500,000

Inadmissible 141,161 23,797 - 164,958


Unadjusted and refundable 74,295 - 170,000 244,295
Total 215,457 23,797 170,000 409,253

Answer # 24 Spring 2021 Q. 6


Hadi Associate
Computation of Sales Tax Payable / Refundable
For the tax period February 2023
Taxable Sales tax
amount @ 18%
SALES TAX CREDITS (INPUT TAX) -------- Rupees --------
Taxable goods from registered customers 1,890,000
Less: Goods purchased from Haq Enterprises - Suspended (100,000)
Goods purchased in cash (85,000)
Goods purchased from AB traders, not declared in its return (50,000)
1,655,000 297,900
Taxable goods from un-registered customers 1,000,000 -
Packing material from un-registered person 445,000 -
Sales tax paid on electricity bill - September 2022 90,000 13,000
310,900

555 | P a g e
Fixed assets purchased 2,500,000 450,000
Total input tax 760,900
Add: Credit brought forward from previous month 415,000
Less: Inadmissible / un-adjustable input tax (W-1) (199,634)
934,716
SALES TAX DEBITS (OUTPUT TAX)
Taxable goods to registered customers 2,750,000
Add: Discount given to associated undertaking 75,000
Less: Goods against which payment was received in Nov 22 (120,000)
2,705,000 486,900
Taxable goods to un-registered customers 1,050,000 178,500
Export - taxable goods (Zero rated) 1,500,000
Taxable supplies goods used for business promotion 150,000 189,000
Total supplies / output tax for the month 5,405,000 854,400
Admissible credit (90% of output tax i.e. Rs. 768,960 or input tax excluding fixed
assets (725,900) whichever is lower. (725,900)
Sales tax payable 128,500
Less: Input tax on fixed assets – machine [Taxable supplies portion only(W-1)] (325,115)
Sales tax to be carried forward – fixed assets 196,615
Sales tax to be carried forward – taxable supplies

Further tax payable on sale to un-registered person (1,050,000–130,000=920,000×4%) 36,800

Sales tax refundable [117,946+124,885(W-1)] 242,831

No adjustment would be made in the sales tax return on account of slow moving stocks

Answer # 25 Autumn 2021 Q. 7


Mehrban Associates
Computation of Sales Tax Payable/Refundable
For the tax period August 2023
Taxable amount Sales tax @ 18%
-------- Rupees --------
Sales tax credits - Input Tax
Taxable goods from registered persons 4,960,000 892,800
Materials exclusively used for exempt supplies (296,000) (53280)
Materials exclusively used for zero rated (675,000) (121,500)
Goods purchased on cash (150,000) (27,000)
500 kg of tea covered under Third Schedule to be taxed at retail price 16,200
[450,000(500×900) 360,000] 90,000
Goods sold to unregistered who have not provided their CNIC or NTN (275,000) (49500)
3,654,000 657,720
Taxable goods from unregistered persons 1,400,000 -
Exempt goods from unregistered persons 520,000 -
Sales tax paid on electricity bill 959,450 154,250
Input tax related to purchase made in February 2021 (It may be
claimed in August return being input tax can be claimed in six
succeeding tax periods) 186,000
997,970
Fixed asset purchase – Machine A 2,000,000 360,000

556 | P a g e
– Machine B 3,000,000 540,000
Furniture and fittings (inadmissible) 1,000,000 -
Total input tax 1,897,970
Less: Unadjusted/inadmissible input tax (W-1) (726,870)
Input tax for the month 1,084,560
Add: Input tax b/f from previous month 1,137,580
Accumulated credit 2,222,140

Sales tax debits - Output Tax


Local taxable goods to registered persons 8,650,000 1,557,000
Taxable goods to unregistered persons 1,560,000 280,800
300 kg of tea covered under third schedule to be taxed at retail price 5940
[300×(900‒790)] 33,000
Taxable supplies local 10,243,000 1,843,740
Export of taxable goods to UAE (zero rated supplies) 1,300,000 0
Export of exempt goods to UAE (zero rated supplies) 1,900,000 0
Total taxable supplies / output tax 13,443,000 1,843,740
Exempt goods to local unregistered persons 1,740,000 -
Total supplies / output tax for the month 15,183,000 1,843,740

Admissible credit (90% of output tax i.e. Rs. 1,659,366 (1,843,740×90%) or input tax
excluding fixed assets i.e. Rs. 1,786,195 [2,222,140‒435,945 (W-1)], whichever is lower. (1,567,179)
Sales tax payable 174,131
Less: Input tax on fixed assets – Machine B (Taxable supplies portion only) (W-1) (435,945)
Sales tax to be carried forward (fixed asset portion only) 261,814
Sales tax to be carried forward (2,222,140‒1,567,179‒435,945) 219,016
Total sales tax to be carried forward 480,830
Further tax payable on supplied to unregistered persons
(1,560,000‒ 237,000(300×790))×3% 39,690

Sales tax refundable on zero rated supplies:


Input tax on material exclusively used for export items (675,000×18%) 114,750
Input tax computed in working 1 542,633
657,383

Answer # 26 Spring 2022 Q. 5(b)

Computation of tax payable / refundable


For the tax period February 2023
Taxable Amount Sales Tax
@ 18%
--- Rs. in million ---
Input Tax
Taxable goods from registered persons 256 46.08
(320×80%)
Taxable goods from unregistered persons 32 -
Exempt goods from registered persons 56 -
Electrical and sanitary fitting 17 -
Electricity bills 1.36 x 60% 0.816
46.896
Less: Refundable input tax (for zero rated) (W-1) (16.53)

557 | P a g e
30.366
Output Tax:
Taxable goods to registered persons 180 32.4
(200×90%)
Exports 98 0
32.4

Admissible credit (90% of output tax i.e. 29.16 (32.4×90%) or input tax i.e. Rs. 30.366, 29.16
whichever is lower
Sales tax payable 3.24
Less: input tax on fixed assets (taxable supplies) (W-1) (9.79)
Sale tax to be carried forward (fixed asset portion only) 6.55
Sale tax to be carried forward (30.366–29.16) 1.20
Total sale tax to be carried forward 7.756
Sales tax refundable on zero rated supplies [16.53(W-1)+5.33(W-1)] 21.86

W-1: Apportionment of input tax


Value of supply Input tax on plant and machinery Residual input tax
180 9.79 30.37
98 5.33 16.53
278 15.12 46.90
(84×18%)

Answer # 27 Autumn 2022 Q. 7

(a) Zahid Enterprises


Computation of tax payable / refundable
For the tax period June 2022
Taxable Sales Tax
Amount @ 18%
--- Rs. in million ---
Input Tax
Purchases from registered persons during June 2022 140 25.2
Purchases - invoice dated 21 December 2022 10 1.80
Gas bill paid in cash 2 0.36
Electricity bill of rented premises showing particulars of landlord - -
Debit note dated 20 June 2022 issued to supplier against invoice
dated 31 march 2022 (6) (1.08)
Payment on 3 June 2022 against invoice dated 5 April 2022 - -
Payment of provincial sales tax @ 13% against transporter’s
invoice dated 17 June 2022 for supplying goods to export
processing zone 8 1.44
Advance paid to supplier at the time of placing order for
customized goods which will be delivered in October 2022 - -
27.72
Less: Refundable input tax (for zero rated) (6.08 (W-1)+1.04) (7.12)
20.6
Output Tax:
Sales of taxable goods to registered person 130 23.4
Supplies for further manufacturing to export processing zone
(Zero rated) 50 -
Sales of taxable goods to unregistered person 44 7.92
Credit note dated 15 June 2022 issued to customer for taxable
supply against invoice dated 1 Dec 2021 - -
Garments withdrawn by owner 4 0.72

558 | P a g e
32.04
Admissible credit (90% of output tax i.e. 28.83(32.04×90%) or input tax i.e. Rs.
20.6, whichever is lower 20.6
11.44
Less: Input tax on plant and machinery (other than zero rated) (W-1) (2.81)
Sales tax payable 8.63
Further tax payable for sales to unregistered persons (44×80%×4%) 1.408
Sales tax to be carried forward -
Sales tax refundable (6.08 + 1.04 + 0.79) 7.91

W-1: Apportionment of input tax


Value of supply Plant and machinery Residual input tax
Description
------------------------ Rs. in million ------------------------
Taxable supplies
178 2.81 21.64
(Non zero rated) (130+44+4)
Taxable supplies
50 0.79 6.08
(Zero rated)
228 3.60 27.72

(b) Reason for ignoring the transaction:


(i) For claiming input tax against electricity bill, it should have registration number and
the address where the connection is installed for which a return is furnished. Since
bill does not contain the said particulars of Zahid Enterprise, input tax thereon shall
not be allowed.

(ii) Input tax is claimed once in purchase period so it shall not be claimed again at the
time of payment.
(iii) Time of supply is the time at which goods are delivered or made available to the
recipient. Since goods will be delivered in October 2022, this is not
chargeable to tax in June 2022 and consequently input tax thereon shall be
adjustable in October 2022 return.
(iv) Since credit note to a customer is issued after one hundred and eighty days
from related taxable supply, reduction in output tax shall not be made.

Answer # 28 Spring 2023 Q. 7


(a) Confidence Engineering
Computation of tax payable
For tax period February 2023
Taxable Sales tax
Amount @ 18%
Input Tax: ----- Rs. in million -----
Wires and cables 6 1.08
Taxable goods from remaining registered persons 332 59.76
(338–6)
Purchases from cottage industry 14 -
Purchases from remaining unregistered person 46 -
(60–14)
Raw material given to the factory engineer (2) (0.36)
Raw material destroyed by fire (25) (4.5)
Purchase return within 180 days (26) (4.68)
Electricity bill 1.2 0.216
51.56

559 | P a g e
Input tax brought forward from last year 27.00
Total input tax 78.516

Output Tax:
Supplies under warranty period 5 -
Goods withdrawn by owner for personal use 4 0.72
Raw material given to factory engineers 2 -
Sales to Pray Traders 32 5.76
Remaining sales to registered person 177 31.86
(220–5–4–2–32)
Sales to a cottage industry 16 2.88
Remaining sales to unregistered person 24 4.32
(40–16)
Invoice dated 15 July 2022 8 1.44
Sales return (lapse of 180 days) (19) -
46.98
Admissible credit (90% of output tax i.e. 42.282 (46.98×90%) or
input tax i.e. 78.516, whichever is lower (42.282)
Sales tax payable 4.698
Further tax payable for sales to unregistered person (24×4%) 0.96
Sales tax to be carried forward (78.516 – 42.282) 36.234

(b) (i) Input tax on purchases from unregistered person including cottage industry shall not
be allowed.
(ii) The ‘sale price’ of the product sold includes the cost of parts, if any, to be supplied
during the warranty period, therefore it is not considered as a ‘separate supply’ and hence
no sales tax is chargeable at the time of disposal of ‘parts’ to meet the warranty claim.

(iii) Putting to private use of raw material is not a supply and therefore it is not
chargeable to tax.
(iv) The adjustment on account of sale return credit can only be claimed if credit note is
issued within 180 days of the relevant supply. Therefore, no tax credit will be available
to CE.

Answer # 29 Autumn 2023 Q. 7


Zeenat Enterprises
Computation of tax liability For the
month of August 2023
Taxable Sales Tax
Amount @ 18%
--- Rs. in million ---
Input Tax
Raw material from associated company 10.40 1.87
(9.2+1.2)
Glass bottles 4.00 0.72
Glass bottles – free of cost 0.80 0.14
(4×20%)
Pigments in retail packaging 7.90 1.42
Remaining purchases 8.30 1.50
(44–35.7)
Purchases from unregistered persons 9.00 -
Purchase return - -

560 | P a g e
Purchase dated 10 February 2023 2.90 0.52
6.17
Less: Refundable input tax (1.40)
4.77
Output Tax:
Cosmetics products 25.00 4.5
Sales return - -
Remaining sales to registered person 7.00 1.26
(30–23)
Sales to unregistered Tier 1 retailer 8.00 1.44
Exports 12.00 -
Perfume for marketing staff 1.00 0.18
7.38
Less: Admissible credit [90% of output tax i.e. 6.64(7.38×90%) or input tax i.e. Rs.
4.77 million, whichever is lower] (4.77)
2.61
Less: Input tax on machine (other than zero rated) (2.09)
Sales tax payable 0.52

Further tax payable -


Sales tax to be carried forward -
Sales tax refundable (0.61+1.40) 2.01

W-1: Apportionment of input tax


Taxable supplies Machine Residual
Description
------------------------ Rs. in million ------------------------
Exports 12.00 0.61 1.40
Local sales 41.00 2.09 4.77
(25+1+7+8)
53.00 2.70 6.17
(15×18%)

561 | P a g e
SECTION- C BASICS
Syllabus Weightage

Syllabus Teaching
Grid Weightage
Ref. Hours
A Objective, System and Historical Background, Constitutional 10-15 5-10
Provisions and Ethics

Key Examinable Technical Competencies

Syllabus Proficiency Testing


Learning Outcomes
Ref. levels levels
Objective, System and Historical Background,
A.
Constitutional Provisions and Ethics
a Basic Concepts of taxation and Constitutional Provisions
1 Discuss the implication of direct and indirect taxation. P1 T1
b Federal and Provincial Financial Procedures
1 Describe Federal Consolidated Fund and Public Account. P1 T1
2 Describe Provincial Consolidated Fund and Public Account. P1 T1
Explain the provisions related to the distribution of
3 P1 T1
revenues between the Federation and Provinces.
Discuss taxes that can be raised under the authority of
4 P1 T1
Parliament.
Describe the powers of provincial assemblies in respect of
5 P1 T1
provincial taxes.
c Ethics
Discuss the objectives and rights of the state to tax its
1 P1 T1
citizens.
Discuss morality behind compliance with tax laws by
2 P2 T1
taxpayers and tax practitioners.
Describe the powers vs ethical responsibilities of tax
3 P2 T1
implementation authorities
Discuss pillars of tax administration, namely; fairness,
4 P2 T1
transparency, equity and accountability.
Explain the basic difference between evasion and
5 avoidance of tax P1 T1

562 | P a g e
CHAPTER 27
BASIC CONCEPTS OF TAXATION
1. DEFINITION OF TAXATION AND OBJECTIVES OF TAXATION LAWS
Definition of tax
Tax is defined as follows:
1. Taxation is collection of share of individual and organization's income by Government under authority of
law.
2. Taxation is used by Government for increasing revenue under authority of law to promote welfare and
protection for its citizens.
Non-revenue objectives of taxation
The main purpose of taxation is to collect revenue for the Government. The Government levies tax to achieve
following objectives.
1. To collect revenue to run and administer Government.
2. Tax is a tool for implanting its policies.
3. Tax is used for fair distribution of wealth.
In addition to finance Government expenses, tax is used as a tool to carry out national objective of social and
economic development as follows:
1. Government can encourage the production of certain goods by introducing exemptions.
2. By charging high tax rates on imports the Government can encourage local purchase.
3. Taxes can be used to reduce inequalities in distribution of wealth.
4. Tax prevents wealth being concentrated in a few hands of the rich.
5. Through tax Government can encounter the effect of inflation and depression
6. To promote science and invention, education systems, health care systems, energy system and military
defense.
7. It can be used to discourage investment abroad.
8. Tax can be used as a bargaining tool in trade negotiation with other countries.
9. Tax laws can be used for documentation of economy (Any amount transferred otherwise than banking
channel will be deemed as income)
10. Government can discourage use of harmful goods by levying heavy rates of tax on certain sectors.
11. Tax can be used to discourage certain undesirable sectors and activities.
12. Government can encourage research & developments by introducing tax credits.
Question
Explain the objectives of following tax laws?
Solution
Tax Law Objective
Tax on salary income Revenue Collection
Money transferred not through banking channel is Documentation of economy
Documentation of economy considered as income
Tax on moveable assets of taxpayers Fair distribution of wealth
Higher taxes on import of luxury goods Reduction in imports of unnecessary goods
Allowing research as an expense Promotion of research
Making exports exempt from tax Promotion of Exports
Tax credit on Donations to approved institutions To promote culture of payment of donation to only
organized and regulated institutions
Tax credit on investments Promote investments in listed companies
Tax exemptions to software exports Promote software Industry
2. BASIC CONCEPTS

563 | P a g e
Basics of tax laws
Adam Smith's in his famous book "Wealth of Nations" has elaborated following canons of taxation:
Equality
Tax payments should be proportional to income.
Certainty
Tax payable should be clear and certain to taxpayer.
Convenience of payment
Tax should be collected from taxpayer at a convenient time.
Economy of collection
Taxes should not be expensive to collect.
2.1 Tax as means for development
Tax is a main source of development. Tax helps in the development of country as follows:
1. Exemption of tax to agriculture sector to promote agriculture.
2. Charities can be encouraged by providing tax credits on them.
3. Imposing high custom duties on import of luxury items will encourage local manufacturing.
4. Taxing rich at higher rate and low income group at lower rate.
5. Government can mark areas as tax free zones, industrial zone and economic zone to provide tax incentives
to such areas. It will encourage business concerns to establish business units that will bring employment
opportunities.
6. Investment in new shares can be encouraged through introducing tax credits.
7. Investment in new plant and machinery can be encouraged through introduction of tax credits.
2.2 Kinds of taxes/Structure of taxes
Proportional tax/ Flat tax
It is a tax where the rate of tax is fixed. A fixed rate is applied on person's income whether the income is high
or low. Under this system people who earn more are not charged at a higher percentage as compared to
progressive tax. E.g Sales tax, Corporate Tax

Progressive tax
It is a tax in which the tax rate increases as the income base increases. A progressive tax takes a larger amount
of tax from the high-income group as compared to low-income group. This tax proportionately equal to a
person's status in the society. E.g Income Tax
Regressive tax
It is a tax where tax rate decreases as the amount of income increases. The higher income group pays less in
taxes than the lower income group. Regressive taxes impose greater tax burden on the poor.

2.3 Principles for levy of tax


Following are the principles for levy of tax.
The Benefit Principle
This principle says that taxes should be based on the benefits received. It means that those who receive the
greatest benefits from Government projects should pay the most taxes. The benefit principle is commonly
used for highways, libraries, etc.
The Ability-to-Pay principle
The tax should be based on ability to pay. It means that a person who is earning more income should pay more
tax. Progressive tax rates are an example of it.
The Equal Distribution Principle
It says that incomes and transactions should be taxed at a fixed rate. Therefore, people who are earning more
income shall pay more tax but not at higher rate.
2.4 Characteristics of tax laws

564 | P a g e
Following are some of the characteristics of a taxation system:
1. Administration and compliance costs should be low.
2. It should be understandable to the taxpayer.
3. The burden of taxes should be distributed in proportion to the ability of the tax-payer, i.e., it should be
progressive in character.
4. The tax should be payable in cash. It means that payment through cheque should not be accepted.
5. It should be mandatory in nature and payment should not be voluntary in nature.
6. Taxes should be collected at a convenient time for tax payers.
7. Tax should be imposed by state which has the jurisdiction over the person to recover i.e. it should be
imposed by Parliament in Pakistan.
8. It is levied for public purpose i.e. taxes are imposed to support Government to implement projects.
9. Taxes should be charged on incomes, transactions or property.
2.5 The principles of a sound tax system
1. Fiscal adequacy
The sources of revenue should be sufficient to meet expenses of government. Revenues of Government should
be capable of expanding or contracting annually in response to variations in public expenditures.
2. Equality or Theoretical Justice
Taxes charged must be based upon the ability of the citizen to pay.
3. Administrative Feasibility
Tax laws should be clear and plain to taxpayers. Capable and well-trained public officers should enforce it.
Time of payment should be convenient.
4. Consistency or Compatibility with Economic Goals
Tax laws should be consistent with economic goals of the government. The goal of Government is to provide
basic services for general public.

3. TYPES OF TAXES IN PAKISTAN


Federal taxes in Pakistan are classified into 2 broad categories:
1. Direct taxes
2. Indirect taxes

Direct Taxes
Income Tax
Income tax is imposed for each tax year, at specified rates on every person who has taxable income for the
year. Taxable income for charge of tax is divided under the following heads:
a) Salary;
b) Income from Property;
c) Income from Business;
d) Capital Gains; and
e) Income from Other Sources.

Capital Value Tax


Capital value tax on different transaction such as transfer of immoveable property, transfer of rights etc.

Indirect Taxes
Sales Tax
Sales tax is levied at various stages of economic activity:
a) All taxable supplies made by a registered person in the course of any taxable activity carried on by him; and
b) All taxable goods imported into Pakistan.
c) All taxable services provided in Pakistan

565 | P a g e
For determining tax liability input paid at the time of purchase or import is deducted from output tax charged
on sales. The sales tax is chargeable at the rate of 18%. In Pakistan, the Government has the flexibility to levy
sales taxes at varying rates, which may be higher or lower than 18%.

Customs Duty
Customs Act in Pakistan specifies import and export duties on certain goods. A major portion of
Government's revenue is collected through this tax. The rate of custom duty is determined by socio-
economic factors. Under this Act high rate is applied on luxury items as well as less essential goods. The
import tariff on industrial plant and machinery is lower than that of consumer goods.

Federal Excise Duty


Under Federal Excise Act tax is charged at specified rate on:
• Goods produced or manufactured in Pakistan
• Goods imported in Pakistan
• Services provided in Pakistan
The base on which tax is charged may be value or retail price or weight. Classification of goods is done as per
Harmonized Commodity Description and Coding system. All exports are charged to tax at the rate of zero
percent.
4. HISTORY OF TAX LAWS IN PAKISTAN
In Pakistan Federal Government has the right to collect tax on the income of a person.

The history of modern tax came from year 1860. Before the partition, income tax was introduced for the first
time in 1860. The British Empire first introduced the Income Tax Act 1860 to fulfill the deficit faced due to
war of independence of 1857. The tax was repealed in 1865.
Thereafter Income Tax Act of 1886 was introduced and it was imposed on traders by some of provinces. Its
basic scheme survives till today. It introduced the definition of agricultural income and it was made exempt
from tax and the exemption is applicable till now.
After the World War, the Government required more funds. So in 1916 for the first time progressive rates of
tax were introduced. Government thought that flat rates of taxes are not justified and the amount of the tax
levied should depend upon the income of the earner. For collecting additional resources, a Super Tax was
levied in 1917,
The most comprehensive tax introduced was Income Tax 1922 in which income tax and super tax were
consolidated. Its salient features were:
• Rates to be decided every year through Finance Act.
• In certain cases tax withholding was made mandatory.
• Re-opening of assessment was allowed.
After independence, the Government of Pakistan adopted this Act.
Till 1979, lot of amendments took place in Income Tax Act, 1922. Resultantly, the Act became a complicated
law.
Keeping in view the difficulties, the Government promulgated a new income tax law namely "The Income
Tax Ordinance, 1979". This law was amended through number of finance acts and after 23 years of its
existence, a new law was promulgated termed as "Income Tax Ordinance, 2001. The fundamental change
introduced was the introduction of a regime of Universal Self-Assessment Scheme.
This law has also been amended through number of finance acts and changes are still going on.

566 | P a g e
ICAP PAST PAPER QUESTIONS
Question # 1 Autumn 2014 Q. 7
The primary objective of a taxation system is to collect revenue. You are required to list the other objectives
(non-revenue) which a taxation system can achieve. (10)
Question # 2 Spring 2015 Q. 9
State any five ways through which taxes can be used for development of the country. (5)
Question # 3 Spring 2015 Q. 10
Briefly explain any three indirect taxes applicable in Pakistan. (5)
Question # 4 Spring 2016 Q. 7(b)
Besides financing government operational expenditures, taxation is also utilized as a tool to carry out the
national objective of social and economic development.’ List any five non-revenue objectives of taxation. (5)
Question # 5 Spring 2018 Q. 5(a)
Taxes are primary revenue yielding tools of the Government of modern ages. You are required to state any three
non-revenue objectives which the Government achieves by imposing taxation. (3)
Question # 6 Autumn 2018 Q. 7
(a) Briefly explain indirect taxes applicable in Pakistan. (4)
(b) State one objective of tax laws in each of the following cases:
(i) High tax rate on high income
(ii) Higher taxes on import of luxury goods
(iii) Tax credit on donations to approved institutions
(iv) Tax credit on investments
(v) Creation of tax free zones
(vi) Tax on income of individuals and companies
(vii) Tax on transactions not made through normal banking channel
(viii) Zero rating under the Sales Tax Act, 1990 (06)

Question # 7 Autumn 2019 Q. 8


(a) Briefly discuss three broad principles for levy of taxes. (04)
(b) Briefly explain any three indirect taxes applicable in Pakistan. (05)
Question # 8 Spring 2021 Q. 8(a)
Taxes are primary revenue yielding tools of the Government of modern ages. State any five ways by which
taxes can be used for the development of the country. (05)
Question # 9 Autumn 2021 Q. 8(a)
State one objective of tax laws in each of the following independent cases:
i) High tax rate on import of goods
ii) Zero rating under the Sales Tax Act, 1990
iii) Decrease in sales tax rate
iv) Tax on cash deposit/withdrawal by non-filer
v) Introduction of tax holiday period for construction related industries
vi) High tax rate on interest income
vii) Decrease in tax rate for online sales
viii) Tax credit to persons employing fresh graduates
ix) High tax rates on luxury items
x) Allow expenditure on research and development (05)

Question # 10 Spring 2022 Q. 7

(a) State any four non-revenue objectives which the government achieves by imposing taxation. (03)
(b) Discuss any three principles of a sound tax system. (03)

ANSWERS

567 | P a g e
Answer # 1 Autumn 2014 Q. 7

Taxation is also utilized as a tool to carry out the national objective of social and economic development.
▪ To strengthen anaemic enterprises by granting them tax exemptions or other conditions or incentives for
growth;
▪ To protect local industries against foreign competition by increasing local import taxes;
▪ As a bargaining tool in trade negotiations with other countries;
▪ To counter the effects of inflation or depression;
▪ To reduce inequalities in the distribution of wealth;
▪ To promote science and invention, finance educational activities or maintain and improve the efficiency of
local forces;
▪ To implement laws which eliminate discrimination among various elements in the markets/businesses.
▪ To discourage certain undesirable sectors and activities.

Examiner Comments:
This question carrying 10 marks required the students to demonstrate their understanding
of the non-revenue objectives of a taxation system. This topic had been introduced in the
syllabus for the first time. Most of the students had prepared it well and were able to secure
high marks. However, some students used pure guesswork and presented replies which were
entirely out of context. Some such statements are presented below:
▪ Non-revenue system applies to agriculture system.
▪ Non-revenue system applies to non-profit organizations.
▪ Non-revenue objectives do not apply to Federal Government.
▪ Non-revenue objectives is to provide exemptions to specific industries in particular area
for the development of these industries.
▪ Non-revenue objective is to provide tax credit on donations and charitable institutions to
develop the interest of public in public welfare.
▪ Tax incentives could encourage foreign investment in the country.

Answer # 2 Spring 2015 Q. 9

Taxes are one of the main sources for development. This is not because revenue collected by the state is used
on developmental projects. Rather, taxes can be used in many different ways for development of the country.
Some examples are as under:
▪ The Government can declare some areas as free zone, industrial zone, and economic zone and provide tax
▪ incentives to such areas. Such incentives could attract businessman/industrialist who may opt to establish
business concerns/industrial units that would bring employment, opportunities and overall prosperity in
these under developed areas.
▪ Taxing the rich at higher rates while taxing the low income groups at lower tax rates.
▪ Imposition of high custom duty rates on luxury items. This promotes local manufacturing industry.
▪ Tax credits on charitable donations to promote welfare activities.
▪ Tax exemptions to charity organisation/educational institutions to promote these activities.
▪ Exemption of tax to Agriculture sector to promote agriculture.

Examiner Comments:
This question required the candidates to explain five ways through which taxes could be
used for development of the country. The overall performance was very poor as vast
majority of candidates were not prepared for this topic which has been introduced in the
new syllabus which became applicable from the last attempt. Most of the candidates started
explaining either the 'Canons of Taxation' or used pure guess work and presented replies
which were entirely out of context such as:
▪ High import duty should be imposed on imports to protect local industry.
▪ Govt. could promote savings by introducing tax credits.

568 | P a g e
▪ Govt. could promote industry by making free zones for industry.
▪ By levying reduced rates of taxes for exports Govt. could promote exports.
▪ By introducing initial allowance and tax credit on new plants and machinery Govt. could
promote production sector.
▪ Taxes are part of fiscal policy and taxation could be a tool for improving economy.

Answer # 3 Spring 2015 Q. 10

Following are the indirect taxes under the Pakistani Taxation System.
(i) Custom Duty
Goods imported and exported from Pakistan are liable to custom duties as prescribed in Pakistan Custom
Tariff.
(ii) Federal Excise Duty
Generally, federal excise duty is charged on the basis of excise value or retail price. However, some items
are chargeable to duty on the basis of weight or quantity. All exports are liable to Zero per cent Federal
Excise Duty.
(iii) Sales Tax
Sales tax is levied on
▪ Import of goods into Pakistan, payable by the importers;
▪ Taxable supplies made in Pakistan by a registered person in the course of furtherance of any
business carried on by him;
A registered person can make adjustment of tax paid at earlier stages against the tax payable by him on his
supplies. For most of the goods sales tax is payable @ 17%.

Examiner Comments:
This question required the candidates to explain the concept of indirect taxes in Pakistan.
The response was mixed. Generally, the candidates correctly identified the three indirect
taxes i.e. Custom Duty, Federal Excise Duty and Sales Tax but failed to explain them
correctly. Some of the common mistakes were as under:
▪ Many candidates mentioned that withholding tax, VAT and provincial sales taxes are
indirect taxes applicable in Pakistan.
▪ Many candidates mentioned tax on dividend and capital gains as indirect taxes.
▪ Many candidates mentioned that the government can either impose sales tax on a product
or excise duty.
▪ Many candidates mentioned that excise duty is to be paid by industries.

Answer # 4 Spring 2016 Q. 7(b)

Non-revenue objectives:
Following are the five non-revenue objectives of taxation:
(i) To strengthen anaemic enterprises by granting them tax exemptions or other conditions or incentives for
growth;
(ii) To protect local industries against foreign competition by increasing local import taxes;
(iii) As a bargaining tool in trade negotiations with other countries; (iv) To counter the effects of inflation or
depression;
(v) To reduce inequalities in the distribution of wealth;
(vi) To promote science and invention, finance educational activities or maintain and improve the efficiency
of local forces;
(vii) To implement laws which eliminate discrimination among various elements in the markets/businesses.
(viii) To discourage certain undesirable sectors and activities.

Examiner Comments:
This question carrying 5 marks required the students to state any five non-revenue
objectives of a taxation system. This topic has recently been introduced in the syllabus.

569 | P a g e
Most of the students had prepared it well and were able to secure high marks. However,
some students used pure guesswork and presented replies which were entirely out of
context as they tried to link these to agricultural income, export processing zones, rebates
on donations, charity and welfare work, infrastructure development, non-profit
organizations, foreign investment, etc.

Marking Scheme:
01 mark for identification of each non-revenue objectives of taxation 5.0

Answer # 5 Spring 2018 Q. 5(a)

The taxation system may also be utilized to achieve the following non revenue objectives as follows:
▪ To strengthen anemic enterprises by granting them tax exemptions or other conditions or incentives for
growth;
▪ To protect local industries against foreign competition by increasing local import taxes;
▪ As a bargaining tool in trade negotiations with other countries;
▪ To counter the effects of inflation or depression;
▪ To reduce inequalities in the distribution of wealth;
▪ To promote science and invention, finance educational activities or maintain and improve the efficiency of
local forces;
▪ To implement laws which eliminate discrimination among various elements in the markets/businesses.
▪ To discourage certain undesirable sectors and activities.
▪ To promote documentation in the economy.
▪ To promote export of the country.
▪ To promote investment in listed companies.
▪ To promote information technology specially software houses.
▪ To promote culture of payment of donation to only organized and regulated institutions

Examiner Comments:
This question required the students to state any three non-revenue objectives of a taxation
system. Most of the students had prepared it well and were able to secure high marks.
However, some students used pure guesswork and presented replies which were entirely
out of context as they gave a long list of welfare work which the government performs.

Marking Scheme:
01 mark for identification of each non-revenue objective which the Government achieves by
imposing taxation 3.0

Answer # 6 Autumn 2018 Q. 7

(a) Following are the indirect taxes under the Pakistani Taxation System.
(i) Custom Duty
Goods imported and exported from Pakistan are liable to custom duties as prescribed in Pakistan
Custom Tariff.
(ii) Federal Excise Duty
Generally, federal excise duty is charged on the basis of excise value or retail price. However, some
items are chargeable to duty on the basis of weight or quantity. All exports are liable to Zero per cent
Federal Excise Duty.
(iii) Sales Tax
Sales tax is levied on
▪ Import of goods into Pakistan, payable by the importers;

570 | P a g e
▪ Taxable supplies made in Pakistan by a registered person in the course of furtherance of any
business carried on by him;
A registered person can make adjustment of tax paid at earlier stages against the tax payable by him
on his supplies. For most of the goods sales tax is payable @ 17%.

(b) Objectives of tax laws in each case are specified below:


(i) Fair distribution of wealth
(ii) Reduction in imports of unnecessary goods and balance of trade.
(iii) To promote culture of payment of donation to only organised and regulated institutions
(iv) Promote investment in listed companies
(v) To give incentives to underdeveloped areas.
(vi) Revenue collection
(vii) Documentation of economy
(viii) Promotion of Exports

Examiner Comments:
This part required brief explanation of indirect taxes applicable in Pakistan. The
performance was good as most of the students identified the indirect taxes correctly.
However, many students gave incorrect answers also which included items such as
withholding taxes, capacity tax, further tax, etc. Moreover, the explanations of federal
excise duty were mostly incorrect as the students seemed unaware of the basis of
chargeability.

Marking Scheme:
(a) Up to 1.5 marks for explaining each indirect tax applicable in Pakistan 4.0

Answer # 7 Autumn 2019 Q. 8

(a) Following are some broad principles for levy of taxes:


(i) The Benefit Principle
This principle holds that the individuals should be taxed in proportion to the benefits they receive
from the governments and that taxes should be paid by those people who receive the direct benefit of
government programs and projects out of the taxes paid.

(ii) The Ability-to-Pay Principle


This principle holds that taxes should relate with the person’s income or the ability to pay, that is,
those with greater income or wealth who can afford to pay should be taxed. Similarly, even rate of
tax could increase with higher income.

(iii) The Equal-Distribution Principle


Income, wealth and transaction may be taxed at a fixed percentage; that is, people who earn more and
spend more should pay more taxes, but not pay a higher rate of tax.

(b) Following are the indirect taxes under the Pakistani Taxation System
(a) Custom Duty
Goods imported and exported from Pakistan are liable to rates of customs duties as prescribed in
Pakistan Custom Tariff.

(b) Federal Excise Duty


Generally, federal excise duty is charged on the basis of excise value or retail price. However, some
items are chargeable to duty on the basis of weight or quantity. All exports are liable to Zero per cent
Federal Excise Duty.

571 | P a g e
(c) Sales Tax
Sales tax is levied on:
▪ Import of goods into Pakistan, payable by the importers;
▪ Supplies made in Pakistan by a registered person in the course of furtherance of any business carried
on by him;

A registered person can make adjustment of tax paid at earlier stages against the tax payable by him on his
supplies. For most of the goods sales tax is payable @ 17%.

Examiner Comments:
Some of the examinees failed to comprehend the requirement of the question and
deliberated on the canons of taxation. Whereas, few examinees could not distinguish
between the concepts of ‘Ability to pay principle’ and ‘Equal distribution principle’.
Majority of the examinees performed well in this part of the question. However, responses
in respect of Federal Excise Duty were mostly incorrect as examinees were unaware of the
basis of its chargeability. Some examinees erred and mixed-up direct taxes with that of
indirect taxes.

Marking Scheme:
(a) Up to 01 mark for identification of each situation in which fair market value of the asset
shall be treated to be the cost of the asset 3.0
(b) (i) Computation of:
− consideration received on disposal of vehicle 1.5
− written down value 1.0
Mentioning correct head of income 0.5
(ii) Computation of:
− consideration received on disposal of vehicle 1.5
− cost of machine 1.0
Mentioning correct head of income 0.5

Answer # 8 Spring 2021 Q. 8(a)


Following are the different ways by which taxes can be used for the development of country:
▪ The Government can declare some areas as free zone, industrial zone, and economic zone and provide tax
incentives to such areas. Such incentives could attract businessman/industrialist who may opt to establish
business concerns/industrial units that would bring employment, opportunities and overall prosperity in
these under developed areas.
▪ Taxing the rich at higher rates while taxing the low income groups at lower tax rates.
▪ Imposition of high custom duty rates on luxury items or items which are also manufactured in Pakistan.
This promotes local manufacturers and industry.
▪ Tax credits on charity/donations to promote welfare activities.
▪ Tax exemptions to charity organization /educational institutions to promote these activities.

Examiner Comments:
Good performance was observed in this part.

Marking Scheme:
01 mark for identification of each way by which taxes can be used for the development of
the country 5.0

Answer # 9 Autumn 2021 Q. 8(a)

572 | P a g e
▪ To protect local industries against foreign competition.
▪ To promote exports of the country.
▪ To counter the effects of inflation.
▪ To document the economy.
▪ To bring the investments in construction related sectors.
▪ To discourage savings in bank account.
▪ To promote online businesses
▪ To increase employment rate in the country
▪ To remove inequality in distribution of wealth
▪ To promote science and innovation

Examiner Comments:
Good performance was observed in this part.

Marking Scheme:
0.5 mark for stating objective of each tax law 5.0

Answer # 10 Spring 2022 Q. 7

(a) To strengthen anemic enterprises by granting them tax exemptions or other conditions or incentives for
growth;
▪ To protect local industries against foreign competition by increasing local import taxes;
▪ As a bargaining tool in trade negotiations with other countries;
▪ To counter the effects of inflation or depression;
(b)
(i) Fiscal adequacy
The sources of revenue taken as a whole should be sufficient to meet the expenditures of the
government, regardless of business, export taxes, trade balances and problems of economic
adjustments. Revenues should be capable of expanding or contracting annually in response to
variations in public expenditures.
(ii) Equality or theoretical justice
Taxes levied must be based upon the ability of the citizen to pay.
(iii) Administrative feasibility
In a successful tax system, tax should be clear and plain to taxpayers, capable of enforcement by the
adequate and well-trained public officials, convenient as to the time and manner of payment and not
unduly burdensome to discourage business activity.

Examiner Comments:
(a) Performance in this part of the question was good.
(b) Performance in this part of the question was good.

Marking Scheme:
(a) 0.75 mark for each non-revenue objective 3.0
(b) 01 mark for each principle of a sound tax system 3.0

573 | P a g e
CHAPTER 27
CONSTITUTIONAL PROVISIONS
FEDERAL FINANCIAL PROCEDURES
Federal Consolidated Fund and Public Account [78]
• All revenues received/loans raised by the Federal Government and
• Amount received in repayment of loan,
Shall be credited to Federal Consolidated Fund.
All other moneys-
(a) Received by the Federal Government; or
(b) Deposited with the Supreme Court/any other court;
Shall be credited to the Public Account of the Federation.
Custody, etc., of Federal Consolidated Fund and Public Account [79]
The custody of the Federal Consolidated Fund and Public Account of the Federation, the payment and
withdrawal from the Funds, all other connected matters shall be regulated by Act of Majlis-e-Shoora
(Parliament) or rules made by President.
Annual Budget Statement [80]
1. In each financial year, the Federal Government shall lay before the National Assembly a statement of
the estimated receipts and expenditures of the Government. It will be called as Annual Budget
Statement.
2. The Annual Budget Statement shall show separately-
(a) Amounts required to meet expenses charged by Constitution on Federal Consolidated Fund;
(b) Amounts required to meet other expenses proposed to be made from Federal Consolidated Fund;
and shall distinguish between revenue expenses and other expenses.
Expenditure charged upon Federal Consolidated Fund [81]
The following expenditures shall be charged upon the Federal Consolidated Fund:-
(a) The remuneration of President and other expenses of his office, and the remuneration of-
i. The Judges of the Supreme Court and the Islamabad High Court;
ii. The Chief Election Commissioner;
iii. The Chairman and the Deputy Chairman;
iv. The Speaker and the Deputy Speaker of the National Assembly;
v. The Auditor-General;
(b) The admin expenses, including the remuneration of officers and servants of Supreme Court, the Islamabad
High Court, the department of the Auditor-General, the Office of the Chief Election Commissioner and the
Secretariats of the Senate and the National Assembly.
(c) Debt charges of the Federal Government, including interest, the repayment of capital, and other expenses
for raising loans, and redemption of debt on the security of Federal Consolidated Fund;
(d) Amount paid for any judgment/order against Pakistan by any court or tribunal; and
(e) Any other amount declared by the Constitution or by Act of Majlis-e- Shoora (Parliament).

Procedure relating to Annual Budget Statement [82]


(1) Expenses charged on Federal Consolidated Fund in Annual Budget Statement shall be discussed in the
National Assembly. A voting will not be required.

574 | P a g e
(2) Other expenses in Annual Budget Statement shall be submitted to National Assembly for raising grant. The
Assembly may or may not agree to demand. It may also reduce the demand.
(3) The demand can only be made with recommendation of Federal Government.
Authentication of schedule of authorized expenditure [83]
(1) The Prime Minister shall sign a schedule specifying-
(a) The grants made by the National Assembly and
(b) Amounts required for expenses charged on the Federal Consolidated Fund. It shall not exceed the amount
mentioned in statement previously laid before the National Assembly.
(2) The authenticated schedule shall be laid before National Assembly. It shall not be open for discussion or
vote.
(3) An expense from Federal Consolidated Fund shall be considered as authorized when above procedure is
followed
Supplementary and excess grants [84]
If in a financial year it is found-
(a) That authorized expense for a service is insufficient, or expense for a new service is required which is not
included in the Annual Budget Statement; or
(b) That amount spent on any service exceeds the amount granted;
The Federal Government can authorize expenditure from the Federal Consolidated Fund. For this, Federal
Government shall lay before National Assembly a Supplementary or Excess Budget Statement.
Votes on account [85]
The National Assembly can make grant in advance of estimated expenditure for a part of financial year, not
more than 4 months, if the authentication of schedule is pending
Power to authorize expenditure when Assembly stands dissolved [86]
If National Assembly is dissolved, the Federal Government may authorize expense from the Federal
Consolidated Fund for estimated expenditure for a period of not more than 4 months in financial year, if the
authentication of the schedule of authorized expenditure is pending
Secretariats of Majlis-e-Shoora (Parliament) [87]
(1) Each House shall have a separate Secretariat: However posts common to both Houses can be awarded.
(2) Majlis-e-Shoora (Parliament) may regulate recruitment of persons appointed to the Secretarial staff of both
Houses
(3) The Speaker or the Chairman may, with the approval of the President, make rules regulating the recruitment
of persons appointed to the secretarial staff of both Houses.
Finance Committees [88]
(1) The expenditure of the National Assembly and Senate shall be controlled by the National Assembly or
Senate acting on the advice of its Finance Committee.
(2) The Finance Committee may make rules for regulating its procedure.
(3) The Finance Committee shall consist of the Speaker or the Chairman, the Minister of Finance and other
members elected by the National Assembly or the Senate.

PROVINCIAL FINANCIAL PROCEDURES


Comparison of Financial and Provincial Procedures
Terms Federal Financial Procedures Provincial Financial Procedure
Fund FCF PCF
Public Account No Change No Change
Courts Supreme Court/ IHC Provincial Court
Authority Parliament Provincial Assembly
Assembly National Assembly Provincial Assembly
Head of State/Province President Governor

575 | P a g e
Chief Executive Prime Minister Chief Minister
Annual Budget Statement No Change No Change
Supplementary/ Excess Budget No Change No Change
Approval of Expenditure in Advance 4 Month 3 Month
Approval when Assembly is 4 Month 4 Month
dissolved

Provincial Consolidated Fund and Public Account [118]


• All revenues received/loans raised by the Provincial Government and
• amount received by it in repayment of loan, Shall form part of Provincial Consolidated Fund.
(2) All other moneys-
(a) Received by the Provincial Government; or
(b) Received with the High Court/any other court;
Shall be credited to the Public Account of the Province.
Custody, etc., of Provincial Consolidated Fund and Public Account [119]
• The custody of the Provincial Consolidated Fund, the payment and withdrawal from the Fund,
• The custody of Public Account of the Province, the payment and withdrawal from the Fund and
• All other connected matters shall be regulated by Act of the Provincial Assembly or provisions made
by
• Governor in this behalf.
Annual Budget Statement [120]
(1) In each financial year, the Provincial Government shall lay before the Provincial Assembly a statement of
the estimated receipts and expenditures of the Government. It will be called as Annual Budget Statement.
(2) The Annual Budget Statement shall show separately-
(a) Amounts required to meet expenses charged by Constitution on Provincial Consolidated Fund;
(b) Amounts required to meet other expenses proposed to be made from Provincial Consolidated Fund; and
shall distinguish expenditure on revenue account from other expenditure.
Expenditure charged upon Provincial Consolidated Fund [121]
The following expenditures shall be charged upon the Provincial Consolidated Fund:-
(a) The remuneration of Governor and other expenses of his office, and the remuneration of-
(i) The Judges of the High Court;
(ii) The Speaker and the Deputy Speaker of the Provincial Assembly;
(b) The administrative expenses, including the remuneration of officers and servants of the High Court, and the
Secretariat of the Provincial Assembly.
(c) Debt charges of the Provincial Government, including interest, the repayment or amortization of capital, and
other expenses for raising of loans, and the service and redemption of debt on the security of Provincial
Consolidated Fund;
(d) Amount paid for any judgment/order against Province by any Court or tribunal; and
(e) Any other amount declared by the Constitution or by Act of Provincial Assembly.
Procedure relating to Annual Budget Statement [122]
(1) Expenses charged on Provincial Consolidated Fund in Annual Budget Statement shall be discussed in the
Provincial Assembly. However a voting will not be required.
(2) Other expenses appearing in Annual Budget Statement shall be submitted to the Provincial Assembly for
raising grant. The Assembly may or may not agree to the demand of grant. It may also reduce the demand.
(3) The demand for a grant can only be made with the recommendation of the Provincial Government.
Authentication of schedule of authorized expenditure [123]
(1) The Chief Minister shall sign a schedule specifying-
(a) The grants made by the Provincial Assembly and
(b) Amounts required to meet the expenses charged on the Provincial Consolidated Fund. It shall not
exceed the amount mentioned in the statement previously laid before the Provincial Assembly.

576 | P a g e
(2) The authenticated schedule (as above) shall be laid before the Provincial Assembly. It shall not be open for
discussion or vote.
(3) An expense from the Provincial Consolidated Fund shall be considered as authorized when it is specified in
the authenticated schedule and schedule is laid before the Provincial Assembly
Supplementary and excess grant [124]
If in a financial year it is found-
(a) That authorized expense for a particular service is insufficient, or expense for a new service is required which
is not included in the Annual Budget Statement; or
(b) That amount spent on any service exceeds the amount granted;
The Provincial Government has power to authorize expenditure from the Provincial Consolidated Fund. For
this, Provincial Government shall lay before the Provincial Assembly a Supplementary Budget Statement or an
Excess Budget Statement specifying the amount of expenditure.
Votes on account [125]
The Provincial Assembly can make any grant in advance of estimated expenditure for a part of financial year,
not more than 3 months, if the authentication of the schedule of authorized expenditure is pending
Power to authorize expenditure when Assembly stands dissolved [126]
If Provincial Assembly is dissolved, the Provincial Government may authorize expenditure from the Provincial
Consolidated Fund for estimated expenditure for a period of not more than 4 months in the financial year, if the
authentication of the schedule of authorized expenditure is pending
Provisions relating to National Assembly, etc., to apply to Provincial Assembly, etc [127]
The provisions of Article 87 and 88 shall apply to Provincial Assembly or the Provincial Government in a way
that:
(a) any reference to Majlis-e-Shoora (Parliament), a House or the National Assembly shall be read as a reference
to the Provincial Assembly;
(b) any reference to the President shall be read as a reference to the Governor of the Province;
(c) any reference to the Federal Government shall be, read as a reference to the Provincial Government;
(d) any reference to the Prime Minister shall be read as a reference to the Chief Minister;
(e) any reference to the National Assembly shall be read as a reference to the Provincial Assembly;

DISTRIBUTION OF REVENUES BETWEEN FEDERATION AND PROVINCES [National


Finance Commission [160]]
(1) Constitution of NFC/who may be member of NFC
The President shall, at intervals not exceeding 5 years, constitute a National Finance Commission consisting of
• The Minister of Finance of the Federal Government, [The Ministers of Finance of the Provincial
Governments, and
• Such persons as appointed by President after consultation with Governors of the Provinces.
(2) Duties of NFC
It shall be the duty of the National Finance Commission to make recommendations to President regarding-
(a) The distribution of tax receipts between the Federation and the Provinces
(b) The making of grants-in-aid by the Federal Government to the Provincial Governments;
(c) The exercise by the Federal Government and the Provincial Governments of the borrowing powers;
(d) Any other finance matter referred by the President.
(3) Taxes raised by Parliament
The taxes raised under authority of Majlis-e-Shoora (Parliament) are as follows:-
(1) Taxes on income, including corporation tax, but not including taxes on income consisting of
remuneration paid out of the Federal Consolidated Fund;
(2) Taxes on sales and purchases of goods imported, exported, manufactured or consumed;
(3) Export duties on cotton, and such other export duties as specified by the President;
(4) Such duties as specified by the President; and

577 | P a g e
(5) Such other taxes as specified by the President.
(3A) The share of the Provinces in each Award (recommendation) of National Finance Commission shall not
be less than the share in the previous Award.
(3B) The Federal Finance Minister and Provincial Finance Ministers shall monitor the implementation of the
Award biannually and lay their reports before both Houses of Majlis-e-Shoora (Parliament) and Provincial
Assemblies.
(4) Order by President
On the basis of recommendation of the National Finance Commission, the President shall, specify:
• The share of the tax receipts which is to be allocated to each Province and
• That share shall be paid to the Government of the Province concerned, and,
• Share shall not form part of the Federal Consolidated Fund.
(5) The recommendations of the National Finance Commission shall be laid before both Houses and the
Provincial Assemblies.
(6) Before any order regarding share of tax receipts the President may, make such amendments in the law
relating to the distribution of revenues between the Federal Government and the Provincial Governments as he
considers necessary.
(7) The President may, by order, make grants-in-aid to the Provinces in need of assistance and such grants shall
be charged upon the Federal Consolidated Fund.
Natural gas and hydro-electric power [161]
1)(a) The net proceeds (receipts) of
• Federal excise duty on natural gas charged at well-head and
• Royalty collected by the Federal Government
Shall not form part of the Federal Consolidated Fund and shall be paid to the Province where well- head is
located;
(b) The net proceeds of Federal excise duty on oil charged at well-head shall not form part of the Federal
Consolidated Fund and shall be paid to the Province where well-head is located.
2) The net profits earned from the bulk generation of power at a hydro-electric station shall be paid to the
Province.
Explanation - "net profit" means revenue from supply of power less operating expenses (including taxes, duties,
interest, and depreciations and element of obsolescence).
Prior sanction of President to Bills affecting taxation in which Provinces are interested [162]
Following bills will not be moved in the National Assembly without sanction of the President. A bill which:
• Imposes/varies a tax, the proceeds of which is allocated to any Province, or
• Varies the meaning of the "agricultural income" for charging income-tax, or
• Affects the principles on which moneys are distributed to Provinces,
Provincial taxes in respect of professions, etc. [163]
A Provincial Assembly may impose taxes, not exceeding limits specified by Majlis-e-Shoora (Parliament), on
persons engaged in professions, trades or employments.
Grants out of Consolidated Fund [164]
The Federation or a Province may make grants for any purpose.
Exemption of certain public property from taxation [165]
(1) The Federal Government is not liable to pay tax on its property or income under any Act of Provincial
Assembly
A Provincial Government is not liable to pay tax on its property or income under:
• Act of Majlis-e-Shoora (Parliament) or
• Act of the Provincial Assembly of other Province.
(2) If any trade/business is carried on by a Provincial Government outside Province, that income may be taxed
under Act of Majlis-e-Shoora (Parliament) or Act of the Provincial Assembly where business is carried on.
(3) However fee for services can be imposed.

578 | P a g e
Power of Majlis-e-Shoora (Parliament) to impose tax on the income of certain corporations, etc
[165(A)]
(1) Majlis-e-Shoora (Parliament) can make law for charging of tax on the income of a company established
by or owned or controlled by a Federal or a Provincial Government.
(2) Order of court, including the Supreme/High Court, which conflicts with above provision shall be void.
FEDERAL LEGISLATIVE LIST
Powers of the Federation to legislate on taxes (Areas whereby Federal Government can legislate to levy taxes)
Following entries in the Federal legislative list contained in constitution of Pakistan relates to taxes or the taxes
which can be imposed by the Federation.
(1) Duties of customs, including export duties.
(2) Duties of excise, including salt, but not including alcoholic liquors, opium or other narcotics;
(3) Taxes on income other than agricultural income;
(4) Taxes on corporations.
(5) Taxes on the sales and purchases of goods imported, exported, produced, manufactured or consumed,
except sales tax on services.
(6) Taxes on the capital value of the assets, not including taxes on immovable property.
(7) Taxes on mineral oil, natural gas and minerals for use in generation of nuclear energy.
(8) Taxes and duties on the production capacity of any plant, machinery, undertaking
(9) Terminal taxes on goods or passengers carried by railway, sea or air; taxes on their fares and freights.
Using above provisions, following laws are enacted by the Federal Government:
Legislative powers of Federation Laws enacted thereunder
Taxes on income other than agricultural income Income Tax Ordinance,2001
Taxes on corporations
Taxes on mineral oil, natural gas and minerals for
use in generation of nuclear energy.
Taxes on the sales and purchases of goods imported, Sales Tax Act, 1990, Federal Excise Act, 2005,
exported, produced, manufactured or consumed, Customs Act, 1969
except sales tax on services
Taxes and duties on the production capacity of any
plant, machinery or installation of any one or more
of them.
Taxes on the capital value of the assets, not Capital Value Tax levied through Finance Act, 1989
including taxes on immovable property.

Powers of the Provinces to legislate on taxes (Areas where Provincial Government can legislate to levy
taxes)
All taxes other than the mentioned in above list of Federal legislative list as contained in the Constitution of
Pakistan are covered in the scope of legislation of Provinces. Accordingly, various types of taxes are introduced
by the Provinces:
• Sales tax on services
• Taxes on transfer of immoveable property
• Professional tax
• Tax on luxury houses
• Tax on registration of luxury vehicles etc.
• Property tax
PAST PAPER QUESTIONS

Question # 1 Autumn 2014 Q. 9


(a) List the taxes which can be imposed by the Federal Government. (06)

579 | P a g e
(b) Briefly describe the duties of National Finance Commission. (04)
Question # 2 Autumn 2015 Q. 8
What do you understand by ‘Federal consolidated fund’? Enumerate the expenditures which are charged upon
the Federal consolidated fund. (08)
Question # 3 Spring 2016 Q. 7(a)
Under the provisions of Article 160 of the Constitution of Pakistan, briefly describe the formation of National
Finance Commission. Who may be the member(s) of such Commission? (03)
Question # 4 Autumn 2016 Q. 7(b)
List any five types of taxes which can be imposed by the Federation as provided in the Federal legislative list
under the Constitution of Pakistan. (05)
Question # 5 Spring 2018 Q. 5(b)
List any five taxes which can be imposed by the Federal Government. (05)
Question # 6 Spring 2019 Q. 8

(a) National Finance Commission has the duty to make recommendations to the President with regard to finance
related matters. You are required to list such recommendations. (04)

(b) State the taxes and duties which may be raised under the authority of Parliament.
Also state four types of taxes which are covered:
(i) under the scope of legislation of the Federation
(ii) under the scope of legislation of the Provinces. (06)

Question # 7 Autumn 2022 Q. 8(b)

Following is the list of various types of taxes/duties:


(i) Agriculture income tax
(ii) Custom duty
(iii) Tax on transfer of immovable property
(iv) Capital value tax
(v) Property tax
(vi) Sales tax on services

Required:
In respect of each of the above mentioned taxes/duties, identify whether they are covered under the scope of
legislation of the Federation or the Provinces. (03)

ANSWERS

Answer # 1 Autumn 2014 Q. 9

(a) Taxes which can be imposed by the Federation:


Following are the types of taxes which can be imposed by the Federation as provided in the Federal legislative
list under the Constitution of Pakistan:
(i) Duties of customs, including export duties.
(ii) Duties of excise, including salt, but not including alcoholic liquors, opium or other narcotics;
(iii) Taxes on income other than agricultural income;
(iv) Taxes on corporations.
(v) Taxes on the sales and purchases of goods imported, exported, produced, manufactured or consumed,
except sales tax on services.

580 | P a g e
(vi) Taxes on the capital value of the assets, not including taxes on immovable property.
(vii) Taxes on mineral oil, natural gas and minerals for use in generation of nuclear energy.
(viii) Taxes and duties on the production capacity of any plant, machinery, undertaking, establishment or
installation in lieu of any one or more of them.
(ix) Terminal taxes on goods or passengers carried by railway, sea or air; taxes on their fares and freights.

(b) It shall be the duty of the National Finance Commission to make recommendations to the President as to:
a) the distribution between the Federation and the Provinces of the net proceeds of the taxes;
b) the making of grants-in-aid by the Federal Government to the Provincial Governments;
c) the exercise by the Federal Government and the Provincial Governments of the borrowing powers
conferred by the Constitution; and
d) any other matter relating to finance referred to the Commission by the President.

Examiner Comments:
(a) This part was based on the constitutional provisions relating to taxation and required the
candidates to identify the types of taxes which may be imposed as per the federal legislative list
of taxes. Majority of the candidates knew the answer and secured full marks. However, some of
the students ended up explaining the types of taxation including Direct/Indirect taxes or
Progressive/Regressive taxation.
(b) This part required the students to briefly describe the duties of National Finance Commission
[NFC]. It was well answered by majority of the candidates. However, some candidates
described the provisions relating to constitution and functionality of NFC which were not
required. Some candidates considered National Finance Commission as being responsible to
table the Annual Budget before the Parliament or to monitor the economic progress of
Federation and Provinces.

Answer # 2 Autumn 2015 Q. 8

• All revenues received by the Federal Government, all loans raised by that Government and all monies
received by it in repayment of any loan, shall form part of a consolidated fund, to be known as the Federal
Consolidated Fund.
• All other monies:
• received by or on behalf of the Federal Government; or
• received by or deposited with the Supreme Court or any other court established under the authority of
the Federation;
shall be credited to the Public Account of the Federation.

The following expenditure shall be expenditure charged upon the Federal Consolidated Fund:-
• the remuneration payable to the President and other expenditure relating to his office, and the remuneration
payable to-
• the Judges of the Supreme Court and the Islamabad High Court
• the Chief Election Commissioner;
• the Chairman and the Deputy Chairman;
• the Speaker and the Deputy Speaker of the National Assembly;
• the Auditor-General;

Examiner Comments:
This question required the students to demonstrate their understanding of the term ‘Federal
Consolidated Fund’. In addition, the students were also required to identify the expenditures
to which this Fund may be applied. The response to this question was excellent as most of the
students had clear idea of ‘Federal Consolidated Fund’ and the expenditures which are
charged upon such Fund. However, some students could not answer it well as they tried to
answer this question using guesswork.

581 | P a g e
Marking Scheme:
• Definition of Federal consolidated fund 2.0
• 01 to 02 marks for stating each expenditure which is charged upon the Federal consolidated
fund 6.0

Answer # 3 Spring 2016 Q. 7(a)

National Finance Commission:


Within six months of the commencing day and thereafter at intervals not exceeding five years, the President
shall constitute a National Finance Commission consisting of the Minister of Finance of the Federal
Government, the Ministers of Finance of the Provincial Governments, and such other persons as may be
appointed by the President after consultation with the Governors of the Provinces.

Examiner Comments:
In this question carrying 3 marks the students were required to briefly describe the provisions
of Article 160 of the Constitution of Pakistan relating to formation of National Finance
Commission and its members. The overall performance of the candidates was satisfactory.
However, some of the common mistakes are listed below:
• Some candidates described the provisions relating to functionality of NFC which was not
required.
• A large number of students were not aware of the members of NFC and included
President, Prime Minister, Chief Ministers, secretaries of Federal and Provincial
Government, Chartered Accountants, etc. as the members.

Marking Scheme:
• Describing the formation of the National Finance Commission 1.5
• Identification of the members of the National Finance Commission 1.5

Answer # 4 Autumn 2016 Q. 7(b)

Taxes which can be imposed by the Federation:


Following are the types of taxes which can be imposed by the Federation as provided in the Federal legislative
list under the Constitution of Pakistan:
(i) Duties of customs, including export duties.
(ii) Duties of excise, including salt, but not including alcoholic liquors, opium or other narcotics;
(iii) Taxes on income other than agricultural income;
(iv) Taxes on corporations.
(v) Taxes on the sales and purchases of goods imported, exported, produced, manufactured or consumed, except
sales tax on services.
(vi) Taxes on the capital value of the assets, not including taxes on immovable property.
(vii) Taxes on mineral oil, natural gas and minerals for use in generation of nuclear energy.
(viii) Taxes and duties on the production capacity of any plant, machinery, undertaking, establishment or installation
in lieu of any one or more of them.
(ix) Terminal taxes on goods or passengers carried by railway, sea or air; taxes on their fares and freights.

Examiner Comments:
This part was based on the constitutional provisions relating to taxation and required the
candidates to identify the five types of taxes which may be imposed as per the federal legislative
list of taxes. Majority of the candidates knew the answer and secured goods marks. However,
some of the students ended up explaining the types of taxation including Direct/Indirect taxes

582 | P a g e
or Progressive/Regressive taxation. Some students mentioned the names of Statutes / Acts
legislated by the Federal Government.

Marking Scheme:
01 mark for identification of each tax that can be imposed by the Federation. 5.0

Answer # 5 Spring 2018 Q. 5(b)

Following taxes can be imposed by the Federal Government


(i) Taxes on income other than agricultural income. (ii) Taxes on corporations.
(iii) Taxes on the sales and purchases of goods imported, exported, produced, manufactured or consumed, except
sales tax on services.
(iv) Taxes on the capital value of the assets, not including taxes on immovable property.
(v) Taxes on mineral oil, natural gas and minerals for use in generation of nuclear energy.
(vi) Taxes on the production capacity of any plant, machinery, undertaking, establishment or installation in lieu of
any one or more of them.
(vii) Terminal taxes on goods or passengers carried by railway, sea or air; taxes on their fares and freights.

Examiner Comments:
This part was based on the constitutional provisions relating to taxation and required the
candidates to identify five taxes, which may be imposed by the Federal Government. Generally
the performance was good but still many students included incorrect items/terms in the answer
such as import tax, property tax, sales tax on services. Some students even included items such
as taxes proposed by the President and National Finance Commission. Some of the students
ended up explaining the types of taxation including direct / indirect taxes or progressive /
regressive taxation.

Marking Scheme:
01 mark for listing each tax which may be imposed by the Federal Government 5.0

Answer # 6 Spring 2019 Q. 8

(a) The duties of the National Finance Commission to make recommendations to the President with regard
to finance related matters include:
(i) the distribution between the Federation and the Provinces of the net proceeds of the taxes.
(ii) the making of grants-in-aid by the Federal government to the Provincial Governments.
(iii) the exercise by the Federal Government and the Provincial Governments of the borrowing powers
conferred by the Constitution;
(iv) any other matter relating to finance, referred to the Commission by the President.

(b) Following taxes may be raised under the authority of the Parliament:
(i) taxes on income, including corporation tax, but not including taxes on income consisting of remuneration
paid out of the Federal Consolidated Fund;
(ii) taxes on the sales and purchases of goods imported, exported, produced, manufactured or consumed;
(iii) export duties on cotton, and such other export duties as may be specified by the President;
(iv) such excise duties as may be specified by the President; and
(v) such other taxes as may be specified by the President.

(i) Taxes which can be imposed by the Federation:

583 | P a g e
Following are the types of taxes which can be imposed by the Federation as provided in the Federal
legislative list under the Constitution of Pakistan:
• Duties of customs, including export duties.
• Duties of excise, including salt, but not including alcoholic liquors, opium or other narcotics;
• Taxes on income other than agricultural income;
• Taxes on corporations.

(ii) Taxes which can be imposed by the Provinces:


Following taxes are covered in the scope of legislation of Provinces:
• Agriculture income tax
• Sales tax on services
• Taxes on transfer of immoveable property
• Professional tax

Examiner Comments:
(a)
(i) Candidates described the provisions relating to constitution and functionality of NFC, which
were not required.
(ii) National Finance Commission was made responsible to table the Annual Budget before the
Parliament or to monitor the economic progress of Federation and Provinces.

(b) Students either did not attempt this question due to selective study or secured full
marks.

Marking Scheme:
(a) 01 mark for identification of each recommendation 4.0
(b) 0.5 mark for listing each tax which may be imposed by the Parliament 2.0
0.5 mark for listing each tax which may be imposed by the Federation 2.0
0.5 mark for listing each tax which may be imposed by the Provinces 2.0

Answer # 7 Autumn 2022 Q. 8(b)

(i) Agriculture income tax Provinces


(ii) Custom duty Federation
(iii) Tax on transfer of immovable property Provinces
(iv) Capital value tax Federation
(v) Property tax Provinces
(vi) Sales tax on services Provinces

584 | P a g e
CHAPTER 29
ETHICS
Ethics - meaning and application
Ethics means characters and customs. Aristotle was the first one to study ethics. He believed that ethical
behavior is knowledge that actions are done for betterment of common good. To determine what is ethically
good for the individual and society, Aristotle said it is necessary to possess 3 virtues of practical wisdom:
1. Temperance,
2. Courage, and
3. Justice.

1. ETHICS FOR TAX LEGISLATORS


"Legislative office is a public trust, and every effort for personal gain through official conduct is a violation of
that trust"
The constitution requires the legislature:
• To make a code of ethics prohibiting conflicts between the public duty and private interests of members of the
legislature.
• To enact a code of ethics for all officials and employees of the state and its political subdivisions.

Canons of taxation:
Canons of taxation are the basic principles to run a good tax system. Adam Smith was the first economist to
develop a list of canons of taxation. Later on, some new canons were added. These canons help tax legislators
in constituting a good tax system. The canons of taxation are discussed below:
Original and Main Canons of Taxation discussed by Adam Smith in famous books "The Wealth of Nations"
1. Canon of Equity
This principal says that a person who earns more should pay more to the Government as compared to a person
who earns less. As a result the person earning high income should pay tax at higher rate as compared to a person
who is earning less income.
It is one of the fundamental concepts to bring social equality and equal distribution of wealth in a country. 2.
Canon of Certainty
The tax that a person is going to pay should be certain. Everything should be made clear and simple for the
taxpayer. It ensures that the taxpayer should have full knowledge about amount of tax payment, mode of
payment and the due-date. If this canon does not exist, it leads to tax evasion.
Another aspect of certainty is that the Government is also certain regarding the amount it will collect from tax.
3. Canon of Convenience
The procedure for tax payment should be easy, convenient and taxpayer-friendly. The time and manner of
payment must be convenient for the tax payer so that he is able to pay his taxes in due time. If the time and
manner of the payment is not convenient, then it may lead to tax evasion and corruption.
4. Canon of Economy
The canon of economy states that the cost of collecting taxes should be as minimum as possible. The cost of tax
collection should be lower than amount of tax received by Government.
The purpose of collecting taxes is to generate revenue for the Government. If the canon of economy isn't applied,
the collected amount will not be sufficient for Government.
Additional Canons of Taxation
Functions of the government increased significantly since Adam Smith's time. Accordingly, modern economists
gave following additional canons of taxation.
5. Canon of Productivity

585 | P a g e
It says that there should be fewer taxes with large revenues, rather than more taxes with lesser amounts of
revenue. It is better to impose only those taxes that are able to produce larger returns. More taxes tend to create
panic and confusion among the taxpayers.
6. Canon of Elasticity
Taxes imposed should be elastic i.e., they can be increased or decreased, according to the demands of the
Government. If the tax is elastic in nature, Government can easily increase its revenue by increasing the rate of
tax. An example of such tax can be the income tax, which is considered very much ideal in accordance with the
canon of elasticity.
7. Canon of Flexibility
Flexibility is different from elasticity. A flexible tax quickly adjusts to the new conditions. If the tax system is
not flexible, new areas will remain un-taxed.
8. Canon of Simplicity
The system of taxation should be simple. The entire process should be non-technical and straightforward. If it
is difficult and complicated to understand, then it will lead to corruption.
9. Canon of Diversity
It says that the Government should collect tax from different sources rather than relying on a single source. If
Government relies on a single tax source it can be harmful for the economy. Therefore Government introduces
both direct and indirect taxes.

2. ETHICS FOR TAX PRACTITIONERS [I.P.P.C.O]


Following are the 5 fundamental principles of ethics for tax practitioners:
1. Integrity
The tax practitioner should be straightforward and honest in all professional and business relations. He must act
honestly and with integrity. All tax laws should be followed properly. He should not misuse the money given
by his client for payment of tax.
2. Objectivity
Practitioner should not allow bias, conflicts of interest or undue influence of others to override their professional
or business judgments.
3. Confidentiality
The tax practitioner should keep all information confidential acquired in professional and business relationships.
The information received from client should not be disclosed to anyone unless it is required by law. Further the
information obtained from client should also not be used for personal benefit or for benefit of a third party.
4. Professional Competence and due care
Tax practitioner has a duty to maintain professional knowledge and skill at such level that client receives a
competent service. The practitioner should have complete knowledge and skill of the service provided by him.
All tax laws should be applied properly in accordance with the circumstances. Staff members must also be well
trained.

5. Professional Behavior
The practitioner should follow the laws and regulations and should avoid actions which discredits the profession.
He should follow the code of ethics developed for tax practitioner. He should behave with courtesy with people.
3. ETHICS FOR TAX ADMINISTRATORS/TAX IMPLEMENTING
• Powers of tax administrators
The tax administration is not responsible for tax policy and tax legislation but deals with current tax system.
Federal Board is the main administrative authority under the law to monitor various types of taxes. During
administration, FBR itself or through its staff exercises the following powers:
• Power to collect revenue
• Power to change the method of accounting

586 | P a g e
• Power to make assessment of tax and
• Power to attach bank accounts
• Power to seize property
The authorities can misuse the above-mentioned powers which can result in following against taxpayer:
• Loss of property and income
• Imprisonment
So above powers if wrongly used can result in loss of fundamental human rights. So there is need for ethics so
that tax authorities can use the powers fairly and with transparency and morality.
Illustration-1
Mr. Zahid is running a textile unit and tax amounting to Rs. 5M is assessed against him. His bank accounts balance is Rs.
10M. However, he has to fulfil his exports orders. In case he fails to fulfil his orders, he would lose his customers and that
orders. Considering his present critical financial position, Zahid believes that tax recovery proceedings by recovery from
bank account (Attachment of bank account) would cause an irreparable loss. So he files a request to FBR for allowing him
to pay the tax dues in instalments.
FBR staff has the power to allow him relief or recover this tax directly from his bank account. Justice and equity demands
that his request should be entertained so that his continuation and prosperity of business would eventually result in payment
of better taxes in future.
Illustration-2
Income Tax Ordinance, sales tax law, Federal excise law empower tax authorities to select cases for audit. This power can
be misused by selecting some cases while leaving many unaudited.
• Pillars of tax administration
For the benefit of taxpayer and keep check on the misuse of powers by tax authorities, following are the pillars
of tax administration:
➢ Fairness
Tax authorities should try to be fair, neutral and impartial while administering the law. There
should be no discrimination on the basis of race, sect etc.
➢ Transparency
All Proceedings must be transparent and must be seen as transparent.
➢ Accountability
There must be a strong system of accountability for wrong doers which should stop corruption,
nepotism and maladministration.
➢ Equity
Similarly situated taxpayers should be taxed in same way by tax authorities. Due care should
be exercised to ensure that salaried class is not taxed more than business class. Administrators
should not achieve their objectives in an irrational way.
Under 4 pillars, following are the ethical issues which are faced by tax administrating authorities:
1. Corruption
2. Confidentiality/Secrecy
3. Conflict of interest
4. Inconsistency in application of law
5. Lack of autonomy
6. Political influence
7. Acceptance of gifts
8. Discretion

To avoid pitfalls of abusive use of discretion, 7 principles for structuring discretion are as follows:
1. Open plans
2. Open policy statements
3. Open rules
4. Open findings
5. Open reasons

587 | P a g e
6. Open precedents
7. Fair informal procedure

Responsibilities of tax implementing authorities


Tax Administrators shall:
1. Try to be impartial, fair, neutral and consistent in administration of law ignoring the race or economic
circumstances.
2. Provide efficient and quality service to all stakeholders in an effort to exceed their expectation.
3. Follow the law completely and no exemptions or credit should be provided to the person who is not eligible
for it.
4. Take all measures to collect tax on timely basis in the interest of the Government and at lowest cost.
5. Make the payment of refunds on timely basis.
6. Show the honesty and integrity to gain confidence of the government and taxpayers;
7. Avoid participation in political activity.
8. Educate taxpayers about their rights and obligations so that tax payers voluntarily comply with the
requirements of law.
9. Keep the confidentiality of the information provided by the taxpayer with highest level of security.
10. Refrain from receiving gifts from actions and non-actions.

4. ETHICS FOR TAXPAYERS


Morality behind Tax Compliance
There are three approaches to ethics for tax compliance which are as under:
▪ Utilitarianism tells us to aim for greatest happiness across the population. 'Happiness' is the satisfaction
of our desires.
▪ Deontology bases ethics on the idea of duty.
▪ Virtue ethics focus on the virtues we should have. A broad conception must be used here. Virtue is not
only honesty but using one's talents and leading a fulfilled life is also a virtue.

Ethics and morality for tax payers regarding taxation compliance


• For Taxpayers following utilitarian approach, the important economic goal is to ensure that goods and
services are available to everyone for decent life. Utilitarian compliance level will be better because
there is a great need of resources for general public and country.
• Deontologist approach lay down absolute duties. There is a duty of individuals to pay for social
resources provided by Government. The one who uses a public hospital or a public road, should pay for
it. So this approach says that it is obligation of taxpayer to pay tax for use of public facilities.
• Virtue ethics says one should use one's talents to the full. Financial incentives encourage people to use
their talents but very high tax rates reduce those incentives. Another virtue is charity either in form of
cash or time. The more take-home cash people have, the more charities they can afford; and also find
time to perform charity work. A third virtue is independence. It is good to live on own money. Lower
tax rate make independence easy.
• Tax can be used for all purposes, and it is clear that what ethicists would say about these purposes.
Purpose Utilitarian Deontology Virtue Ethics
Providing law and Will approve tax because Will approve tax because
order, health care and more goods and services this will enable people to
education service to will be available resulting lead prosperous lives and
public in increase in happiness utilize their talents to full.
among people.

588 | P a g e
Helping the needy and Will approve tax because it Will approve Will approve tax because
poor with tax money will lead to equal because it is our taking care of less
distribution of wealth and duty to take care fortunate is itself a virtue
happiness among society. of the poor.

• Morality for citizens to pay taxes is justified so that:


❖ State is able to provide proper infrastructure,
❖ State is able to provide level playing field to all.

In recent years, tax avoidance is a considerable public concern. It is compliance with the law, though aggressive
or abusive avoidance, as opposed to simple tax planning which seek to comply with the letter of the law. Tax
evasion occurs when someone acts against the law.
• Tax avoidance is the legal use of the tax law to one's own advantage, to reduce the amount of tax
payable. For example, it is up to tax payer that whether the business is started as a partnership or
company. In case of partnership tax rates is lower than that of company. Another example is starting
business in a tax free zone.
• Tax evasion is an illegal practice where individuals, AOPs and companies unlawfully avoids payment
of tax. In tax evasion taxpayer intentionally conceals the true state of affairs to tax authorities. It is
punishable under the law. For example, concealment of income, misclassifying income under another
head, claiming excess deductions, and claiming wrong tax credits.

Difference between ‘Tax Avoidance’ & ‘Tax Evasion’


Tax Avoidance Tax Evasion
Tax payment is avoided by complying with the Tax is avoided through illegal means or fraud.
provisions of law but defeating the intension of the
law.
It is done by taking advantage of loop holes in law It is done by employing unfair means
Tax Avoidance is not done with malafide intention Tax Evasion is an unlawful way of paying lower
but to comply with law taxes
Tax Avoidance looks like a tax planning and is done Tax evasion is a fraud and is done after the tax
before the tax liability arises. liability has arisen.
Example is: Examples include:
Donation to approved charity recognized by the Falsifying income tax return by non-disclosure of
FBR to claim a tax credit. off-shore accounts e.g. Panama Paper Leaks (Nawaz
Sharif)

Situation Evasion/Avoidance
Mr. A is in the business of selling shoes. His sales It is tax evasion and a Criminal Act. He cannot buy
are Rs. 20 M. He kept all cash in his bank locker to any asset form this money unless he declares this
hide it from tax authorities. income and pays tax on it
Mr. B earned income of Rs 50 M. However, he This is also a tax evasion. Understatement is also an
declared only that income which is verifiable from offence.
the banks i.e. 36M. Remaining amount has been
hidden in a separate bank account.
Mr. C earned Rs 50 M. However, he recorded 37 M Tax avoidance, which is legally permissible.
expenses using legal tactics to reduce his income.

• A utilitarian, concerned with aggregate welfare, might be willing for tax avoidance. If tax is avoided,
wealth is kept in the private sector (means it will not go to Government). The utilitarian says that

589 | P a g e
charging tax will put more burden on middle class who cannot afford hiring tax lawyers. The
dissatisfaction of middle class will be more as compared to the satisfaction achieved by rich class.
• A virtue ethicist would dislike tax avoidance. It is not ethical to exploit rules knowing that one is
exploiting it for oneself.
• A deontologist will not favor tax avoidance, but might not condemn it either. Deontologists can easily
argue for a duty to obey the law

ICAP PAST PAPER QUESTIONS


Question # 1 Autumn 2014 Q. 8
Briefly explain the ethical responsibilities of the tax implementing authorities. (10)
Question # 2 Autumn 2015 Q. 7
Briefly describe any three main canons of taxation which can be helpful in formulating a good tax system. (03)
Question # 3 Spring 2016 Q. 7(c)
List any six ethical issues which may be faced by tax administration authorities while discharging their duties
under the four pillars of tax administration. (03)
Question # 4 Autumn 2016 Q. 7(a)
List any seven responsibilities of tax administrators emanating from best ethical practices. (07)
Question # 5 Autumn 2017 Q. 8
(a) Briefly describe the pillars/principles of tax administration which are meant to safeguard the interest of
taxpayers and avoid abuse of powers by the tax administrators. (04)
(b) Differentiate between the terms ‘Tax evasion’ and ‘Tax avoidance’. Give one example in each case. (03)
(c) List any six ethical issues that which administrators may face while discharging their duties. (03)
Question # 6 Spring 2020 Q. 8
(a) List any seven responsibilities of tax administrators arising from best ethical practices. (07)
(b) State any six ethical issues which the administrators may face while discharging their duties. (03)
Question # 7 Autumn 2020 Q. 8
(a) Identify any four powers of the Federal Board of Revenue (FBR) with respect to collection of tax. What
consequences may be faced by taxpayer if such powers are misused by any officer(s) of FBR? (03)
(b) Briefly explain any two pillars of tax administration which may be helpful in safeguarding the interest of
taxpayers and avoiding the abuse of powers by the tax administration. (02)
(c) List any six ethical issues which may be faced by tax administration authorities while discharging their duties
under the four pillars of tax administration. (03)
Question # 8 Spring 2021 Q. 8(b)
List six types of taxes which are covered within the legislative powers of Provinces. (03)
Question # 9 Autumn 2021 Q. 8(b)
List the fundamental principles of ethics for tax practitioners. Also describe any one of the principles. (03)
Question # 10 Spring 2023 Q. 8
Give two examples, each of tax avoidance and tax evasion. Also suggest the appropriate tax measures to prevent
that tax evasion. (06)
Question # 11 Autumn 2023 Q. 8(b)
List the fundamental principles of ethics for tax practitioners. Also, describe any two of the principles. (04)

ANSWERS
Answer # 1 Autumn 2014 Q. 8

A concise code which can enlist responsibilities of Tax Administrators can be as under:
1) Obey all laws relating to taxation and grant no exemptions, credit or advantage to any taxpayer that is not
provided by the law;

590 | P a g e
2) Be dedicated to the highest ideals of honesty and integrity in all matters in order to maintain the respect and
confidence of the government and taxpayers;
3) Strive to be impartial, fair, neutral and consistent in administering the law without regard to race, social status
or economic circumstances;
4) Provide prompt, efficient and quality service to all stakeholders in an effort to exceed their expectations;
5) Refrain from actively participating in partisan political activities;
6) Accurately record proceedings and maintain taxpayer information in the strictest confidence and highest level
of security;
7) Refrain from soliciting gifts for actions and non-actions;
8) Make reasonable effort to collect the proper amount of tax revenue due at the lowest possible cost to the state,
and in a manner that warrants the highest degree of confidence in our integrity, efficiency, effectiveness and
fairness;
9) Respond to valid taxpayer refund claims with the same diligence as employed in collection of taxes;
10) Educate taxpayers on their rights and responsibilities to ensure the highest possible levels of voluntary
compliance to the laws.

Examiner Comments:
This question tested the topic of ethics in taxation. The students were required to briefly explain
the ethical responsibilities of the tax implementing authorities. Here also, many students were
able to answer well, however, the performance of some candidates was rather poor, most
probably due to selective studies.

Some candidates did not comprehend the question correctly and wasted their time in specifying
the cannons of taxation which was entirely irrelevant. Many candidates confused it with the
ethical responsibilities of tax legislating authorities. Some students also explained the powers
of tax implementing authorities which was not asked for and they simply wasted their valuable
time.
Surprisingly, considerable number of candidates explained the code of ethics for professional
accountants as ethical responsibilities of tax implementing authorities.

Answer # 2 Autumn 2015 Q. 7

Canon of Equity
The principle aims at providing economic and social justice to the people. According to this principle, every
person should pay to the government depending upon his ability to pay. Rich people should pay higher taxes to
the government, because without the protection of the government authorities (Police, Defence, etc.) they would
not have earned and enjoyed their income. Adam Smith argued that the taxes should be proportional to income,
i.e., citizens should pay taxes in proportion to the revenue which they respectively enjoy under protection of the
State.

Canon of Certainty
According to Adam Smith, the tax which an individual has to pay should be certain, not arbitrary. The taxpayer
should know in advance how much tax he has to pay, at what time he has to pay the tax, and in what form the
tax is to be paid to the government. In other words, every tax should satisfy the canon of certainty. At the same
time a good tax system also ensures that the government is also certain about the amount that would be collected
by way of tax.

Canon of Convenience
The mode and timing of tax payment should be as far as possible, convenient to the taxpayers. For example,
land revenue is collected at time of harvest and income tax is deducted at source. Convenient tax system
encourages people to pay tax and increases tax revenue.

591 | P a g e
Examiner Comments:
In this simple question carrying 3 marks the candidates were required to briefly describe any
three canons of taxation. The response to this question was excellent and majority of the
students were able to identify and briefly describe the canons of taxation and secured full
marks. There were two common errors as canon of equity was termed as canon of equality
whereas canon of convenience was termed as canon of conveyance.

Marking Scheme:
• 0.5 mark for identification of each canon of taxation 1.5
• 0.5 mark for brief description of each canon of taxation 1.5

Answer # 3 Spring 2016 Q. 7(c)

Following are the six ethical issues faced by tax administration authorities under the four pillars of tax
administration:
(i) Acceptance of gifts
(ii) Conflict of Interest
(iii) Selective application of the law/ or inconsistency in applying the law
(iv) Political influence
(v) Confidentiality/secrecy
(vi) Discretion
(vii) Corruption
(viii) Lack of Autonomy

Examiner Comments:
This part of the question carrying 3 marks required the candidates to list six ethical issues
faced by tax administration authorities in discharging their duties. Majority of the candidates
were aware of the issues and were able to list them correctly. However, similar to part (b)
many students discussed irrelevant material also such as lack of professional skills, knowledge
of the standards, and code of ethics for professional accountants.

Marking Scheme:
0.5 mark for listing each ethical issue faced by the tax administration authorities 3.0

Answer # 4 Autumn 2016 Q. 7(a)

Responsibilities of the tax implementing authorities:


A concise code which can enlist responsibilities of Tax Administrators can be as under:
(i) Obey all laws relating to taxation and grant no exemptions, credit or advantage to any taxpayer that is not
provided by the law;
(ii) Be dedicated to the highest ideals of honesty and integrity in all matters in order to maintain the respect and
confidence of the government and taxpayers;
(iii) Strive to be impartial, fair, neutral and consistent in administering the law without regard to race, social status
or economic circumstances;
(iv) Provide prompt, efficient and quality service to all stakeholders in an effort to exceed their expectations;
(v) Refrain from actively participating in partisan political activities;
(vi) Accurately record proceedings and maintain taxpayer information in the strictest confidence and highest level
of security;
(vii) Refrain from soliciting gifts for actions and non-actions;

592 | P a g e
(viii) Make reasonable effort to collect the proper amount of tax revenue due at the lowest possible cost to the state,
and in a manner that warrants the highest degree of confidence in our integrity, efficiency, effectiveness and
fairness;
(ix) Respond to valid taxpayer refund claims with the same diligence as employed in collection of taxes;
(x) Educate taxpayers on their rights and responsibilities to ensure the highest possible levels of voluntary
compliance to the laws.

Examiner Comments:
This part required the students to list responsibilities of tax administrators emanating from
best ethical practices. The overall performance of the candidates was very encouraging and
many candidates secured full marks. However, many candidates discussed cannons of taxation
which was totally irrelevant. Some students used pure guesswork and made the following types
of statements:
• Tax administrator must keep eye on person’s foreign accounts and suspect any possible
ways of tax evasion by the individuals paying tax.
• Tax administrator should collect tax from rich rather than concentrating on collecting
more tax from poor.
• Tax administrator should collect tax at the time and manner convenient to the people.

Marking Scheme:
01 mark for description of each responsibility of the tax implementing authorities 7.0

Answer # 5 Autumn 2017 Q. 8

(a) Pillars of tax administration


Pillars of tax administration which safeguard the interest of taxpayers and avoid abuse of powers by the tax
administration are as follows:
(i) Fairness
Strive to be impartial, fair, neutral and consistent in administering the law without regard to race, social or
economic circumstances.
(ii) Transparency
All proceedings must be transparent and must be seen as transparent.
(iii) Equity
Treating all tax payers fairly. Refraining from unduly pressurizing a tax payers or a body of tax payers
merely to achieve tax targets.
(iv) Accountability
There must be a strong system of accountability for wrong doers which should curb corruption, nepotism
and maladministration.
(b)
Tax Evasion Tax Avoidance
Attempts to minimize a taxpayer’s liability Attempts to minimize a tax payer’s liability through legal
through illegal means is called Tax Evasion. means and without violating tax laws is called Tax
Avoidance.

Example of tax evasion include deliberately Examples of tax avoidance involve using tax deductions,
concealing the true state affairs to the tax changing one’s business structure through incorporation.
authorities to reduce tax liability and dishonest Establishing companies in tax heavens through legal
tax reporting. means.

(c) Ethical issues facing tax administration in the discharge of their duties are:
(i) Acceptance of gifts

593 | P a g e
(ii) Conflict of interest
(iii) Selective application of the law/ or inconsistency in applying the law
(iv) Political influence
(v) Confidentiality/secrecy
(vi) Discretion
(vii) Corruption
(viii) Lack of autonomy

Examiner Comments:
This part required the candidates to briefly describe pillars/principles which are meant to safe
guard the interest of taxpayer and avoid abuse of power by tax administrator. The response to
this part was excellent and almost all students performed well.

Marking Scheme:
(a) 01 mark for brief explanation of each pillar of tax administration 4.0
(b)
• Explanation of the term ‘Tax Evasion’ with example 1.5
• Explanation of the term ‘Tax Avoidance’ with example 1.5
(c) 0.5 mark for identification of each issue that administrators may face while discharging their
duties 3.0

Answer # 6 Spring 2020 Q. 8

(a) Responsibilities of the tax implementing authorities:


A concise code which can enlist responsibilities of Tax Administrators can be as under:
(i) Obey all laws relating to taxation and grant no exemptions, credit or advantage to any taxpayer that is not
provided by the law;
(ii) Be dedicated to the highest ideals of honesty and integrity in all matters in order to maintain the respect
and confidence of the government and taxpayers;
(iii) Strive to be impartial, fair, neutral and consistent in administering the law without regard to race, social
status or economic circumstances;
(iv) Provide prompt, efficient and quality service to all stakeholders in an effort to exceed their expectations;
(v) Refrain from actively participating in partisan political activities;
(vi) Accurately record proceedings and maintain taxpayer information in the strictest confidence and highest
level of security;
(vii) Refrain from soliciting gifts for actions and non-actions;

(b) Ethical issues facing tax administration in the discharge of their duties are:
(i) Acceptance of gifts
(ii) Conflict of interest
(iii) Selective application of the law
(iv) Political influence
(v) Confidentiality
(vi) Discretion

Examiner Comments:
(a) Examinees performed well in this part of the question. However, some of the examinees
discussed cannons of taxation which was totally irrelevant
(b) Good performance was observed in this part of the question.

Marking Scheme:
(a) 01 mark for listing each responsibility of tax administrators arising from best ethical practices
7.0

594 | P a g e
(b) 0.5 mark for identifying each ethical issue which the tax administrators may face while
discharging their duties 3.0

Answer # 7 Autumn 2020 Q. 8

(a) Federal Board of Revenue has the following powers under the law to monitor, assess, levy and collect
tax:
• Assess taxes (including best judgment);
• Collect Revenue;
• Seize Property;
• Attach bank accounts;

A misuse of power by FBR may result in the following against the tax payer:
• Loss of property and income;
• Imprisonment

(b) Pillars of tax administration


Pillars of tax administration which safeguard the interest of taxpayers and avoid abuse of powers by the
tax administration are as follows:
(i) Fairness
Strive to be impartial, fair, neutral and consistent in administering the law without regard to race,
social or economic circumstances.

(ii)Transparency
All proceedings must be transparent and must be seen as transparent. (c) Ethical issues facing tax
administration in the discharge of their duties are:
(i) Acceptance of gifts
(ii) Conflict of interest
(iii) Selective application of the law/ or inconsistency in applying the law
(iv) Political influence
(v) Confidentiality/secrecy
(vi) Discretion
(c) Ethical issues facing tax administration in the discharge of their duties are:
(i) Acceptance of gifts
(ii) Conflict of interest
(iii) Selective application of the law
(iv) Political influence
(v) Confidentiality
(vi) Discretion

Examiner Comments:
Good performance was observed in this question.

Marking Scheme:
(a)
• 0.5 mark for identifying each power of FBR with respect to collection of tax 2.0
• 0.5 mark for identifying each consequence which may be faced by the taxpayer if such
powers are misused by the officers of FBR 1.0
(b) 01 mark for identifying each pillar of tax administration which may be helpful in safeguarding
the interest of taxpayers and avoiding the abuse of powers by the tax administration 2.0
(c) 0.5 mark for identifying each ethical issue which may be faced by the tax administration
authorities while discharging their duties under the four pillars of tax administration 3.0

595 | P a g e
Answer # 8 Spring 2021 Q. 8(b)

Powers of the Provinces to legislate on taxes


Following taxes are covered in the scope of legislation of Provinces
• Agriculture income tax
• Sales tax on services
• Taxes on transfer of immovable property
• Professional tax
• Tax on luxury houses
• Tax on registration of luxury vehicles

Examiner Comments:
Good performance was noted in this question.

Marking Scheme:
0.5 mark for identification of each tax which is covered within the legislative powers of
Provinces 3.0

Answer # 9 Autumn 2021 Q. 8(b)

(i) Integrity
(ii) Objectivity
(iii) Professional competence and due care
(iv) Confidentiality
(v) Professional behaviour

Integrity:
Tax practitioners should be straightforward and honest in all professional and business relationships. Integrity
implies not just honesty but also fair dealing and truthfulness.

Examiner Comments:
Good performance was observed in this part.

Marking Scheme:
• 0.5 mark for each principle up to maximum of 02 marks 2.0
• Describing any one principle 1.0

Answer # 10 Spring 2023 Q. 8

Tax Avoidance:
▪ Make donations to an approved charity organization to claim a tax credit.
▪ A partnership firm may be converted into a small company to avail benefit in tax rate of such company.

Tax Evasion
1)Mr. B earned an income of Rs. 10 million. However, he only declared Rs. 6 million which is verifiable from
the banks. He has hidden the remaining amount in a separate bank account.

596 | P a g e
2) Mr. C earned a turnover of Rs. 10 million. However, he kept it as cash in his locker and hid it from tax
authorities. He paid all related expenses from this cash.
Measures to prevent tax evasion
1. Cash inflow/outflow shall be made through a declared bank account.
2. Only those expenses shall be admissible which are paid through banking channel / Cash expenses
shall not be admissible.

Examiner Comments:
Upon reviewing the examples provided by examinees, it became evident that they lacked a
comprehensive understanding of the clear differentiation between tax avoidance and tax
evasion. Some examinees provided definitions of tax avoidance and tax evasion that were not the
requirement of the question

Marking Scheme:
• 01 mark for each example of tax avoidance and tax evasion
• 01 mark for each appropriate tax measure

Answer # 11 Autumn 2023 Q. 8(b)

(i) Integrity
(ii) Objectivity
(iii) Professional competence and due care
(iv) Confidentiality
(v) Professional behaviour
Integrity:
Tax practitioners should be straightforward and honest in all professional and business relationships. Integrity
implies not just honesty but also fair dealing and truthfulness.
Objectivity:
Tax practitioners should not allow bias, conflicts of interest or undue influence of others to override their
professional or business judgments.
Professional competence and due care:
Tax practitioners have a duty to maintain their professional knowledge and skill at such a level that a client or
employer receives competent service, based on current developments in practice, legislation and techniques.
Tax practitioners should act diligently and in accordance with applicable technical and professional standards.
Confidentiality:
Tax practitioners should respect the confidentiality of information acquired as a result of professional and
business relationships and should not disclose such information to third parties without authority or unless there
is a legal or professional right or duty to disclose. Confidential information acquired as a result of professional
and business relationships should not be used for the personal advantage of tax practitioners or third parties.
Professional behavior:
Tax practitioners should comply with relevant laws and regulations and should avoid any action which discredits
the profession. They should behave with courtesy and consideration

Marking Scheme:
• 2 marks for listing the fundamental principle of ethics
• 01 mark for describing each principle

597 | P a g e
CHAPTER 30
CODE OF ETHICS
CODE OF ETHICS FOR CHARTERED ACCOUNTANTS AS APPLICABLE TO TAX SERVICES
Tax Services
Tax services include preparing tax return, making tax calculations for accounting entries, suggesting tax
planning etc.
Providing tax services to an audit client might create a self-review or advocacy threat.
Code of Ethics includes requirements that prohibit firms (and network firms) from providing certain tax
services to audit clients in some circumstances because the threats cannot be addressed by applying
safeguards.
Tax Services to Audit clients
Factors for evaluating level of threats created by providing tax service to an audit client include:
The characteristics of the engagement.
The tax expertise of client's employees.
The system by which tax authorities assess and administer the tax and the role of the firm (or network firm) in
that process.
The complexity of tax regime (and degree of judgment necessary in applying it).
1) Tax return preparation
All Audit Clients
Providing tax return preparation services (to audit client) does not usually create a threat. Tax return
preparation services involve:
0 Assisting clients with tax reporting obligations by compiling information, including payment of tax.
1 Advising on tax treatment of past transactions and responding on behalf of audit client to tax
authorities' requests for additional information and analysis (for example, providing explanations for approach
being taken).
Tax return preparation services are usually based on historical information and principally involve
presentation of such information under existing tax law. Further returns are also reviewed by tax authorities.
2) Tax Calculations for the Purpose of Preparing Accounting Entries
a) All Audit Clients
Preparing calculations of current and deferred tax liabilities (or assets) for an audit client for accounting
entries that will be later on audited by the firm creates a self-review threat.
In addition to the factors discussed above (4 bullets), a factor relevant for evaluating threat is that calculation
might have a material effect on the financial statements on which the firm will express an opinion.
b) Audit Clients that are Not Public Interest Entities
Safeguards to address self-review threat when the audit client is not a public interest entity include:
• Using professionals who are not audit team members to perform the tax service.
• Having an appropriate reviewer who was not involved in:

the service review


the audit work or
service performed.
c) Audit Clients that are Public Interest Entities
A firm (or a network firm) shall not prepare tax calculations of current and deferred tax liabilities (or assets)
for an audit client (public interest entity) for preparing accounting entries that are material to the financial
statements on which the firm will express an opinion.
If impact of accounting entries on financial statements is immaterial then above safeguards mentioned in point
(b) will be applied.
Example-1
A Pakistani audit firm is faced with the following situations:
Situation 1

598 | P a g e
ABC limited an audit client has requested your firm to prepare current and deferred tax working for the
purpose of preparing accounting entries that will be reviewed by your firm at the time of Audit. ABC Ltd is
not a public interest entity.
Situation 2
ABC limited an audit client has requested your firm to prepare current and deferred tax working for the
purpose of preparing accounting entries that will be reviewed by your firm at the time of Audit. ABC Ltd is a
public interest entity.
Required:
With reference to the ICAP Code of Ethics, what are the threats presented by the events described above. Also
comment upon the safeguards to be taken to reduce the said threat (if any)
Answer
Situation 1
Preparing calculations of current and deferred tax liabilities (or assets) for an audit client for preparing
accounting entries that will be subsequently audited by the firm creates a self-review threat. The significance
of the threat will depend on:
0 The complexity of tax regime (and degree of judgment necessary in applying it)
1 The tax expertise of client's employees
2 The materiality of the amounts to the financial statements.
Safeguards
Safeguards to be applied to eliminate/reduce threat:
0 Using professionals who are not audit team members to perform the tax service.
1 Obtaining advice from an external tax professional
2 Having an appropriate reviewer who was not involved in:
0 the service reviews
1 the audit work or
2 service performed.
Situation 2
Except in emergency situations, a firm (or a network firm) shall not prepare tax calculations of current and
deferred tax liabilities (or assets) for an audit client (public interest entity) for preparing accounting entries
that are material to the financial statements on which the firm will express an opinion.
In emergency or other unusual situations when it is impractical for the audit client to make other
arrangements, we may provide tax service. This may be the case when:
0 only the firm has the resources and knowledge of the client's business to assist client in. timely
preparation of its calculations of current and deferred tax liabilities (or assets), and
1 Stopping firm from providing service will result in significant difficulties for the client (for example,
failure to meet regulatory reporting requirements).
Safeguards
0 Those who provide the services are not members of the audit team;
1 The services are provided for only a short period of time and are not expected to recur; and
2 The situation is discussed with those charged with governance.
Example-2
Bilal Azhar, ACA is Manager Taxation at a large tax consultancy firm and reports to Bader Ali, FCA who is
one of the partners of the firm.
Bilal Azhar is presently engaged in the preparation of the income tax return of Digital Systems Limited
(DSL), an IT company. During the review of the tax workings, he discovers that DSL has charged certain
expenses against which no supporting documents are available. He brings this matter to the attention of Bader
Ali who has responded him that since this is not an audit engagement, it is not our responsibility to highlight
such matters.
Required:
Briefly discuss how Bader Ali may be in breach of the fundamental principles of ICAP's Code of Ethics. Also,
state the potential threats that Bilal Azhar may face in the above circumstances and how he should respond.
[ICAP study Text]
Answer
Fundamental principles breached:

599 | P a g e
In the given situation, Bader may be in breach of the following fundamental principles of Code of Ethics for
Chartered Accountants:
Professional behavior
The practitioner should follow the laws and regulations and should avoid actions which discredits the
profession.
Bader has breached the fundamental principle of professional behavior as his proposed suggestion in respect
of ignoring the appropriate adjustments to the income tax return would affect the good reputation of the
profession.
Integrity
The tax practitioner should be straightforward and honest in all professional and business relations. Bader has
breached the fundamental principle of integrity as he has knowingly ignored the required adjustments to be
made in the income tax return.
Potential threat:
Intimidation threat
Bilal may face intimidation threat from Bader as refusal to obey instruction may risk his job.
Safeguards:
Identified threats are significant as Bilal is being instructed from the highest level of management. In order to
reduce the threat to an acceptable level, one or more of the following safeguards should be applied:
0 Discuss the matter with Bader and persuade him to follow code of ethics/contact the tax client to
make necessary adjustments.
1 Consider informing appropriate authorities like a senior partner in the firm.
2 Refuse to implement the given proposals.
3 Seek legal advice.
4 In case threat could not be reduced consider resigning from the job

ICAP PAST PAPER QUESTIONS


Question # 1 Autumn 2022 Q. 8(a)

Discuss whether the following tax services can be provided to an audit client which is not a public interest
entity:
(i) Preparation of tax returns
(ii) Tax calculation for the purpose of preparing accounting entries (05)

ICAP PAST PAPER SOLUTIONS


Answer # 1 Autumn 2022 Q. 8(a)

(i) Providing tax return preparation services does not usually create a threat so this service can be provided to
audit client.
(ii) Preparing calculations of current and deferred tax liabilities (or assets) for an audit client for the purpose of
preparing accounting entries that will be subsequently audited by the firm creates a self-review threat. The
significance of the threat will depend on:
• The complexity of the relevant tax law and regulation and the degree of judgment necessary in
applying them;
• The level of tax expertise of the client's personnel; and
• The materiality of the amounts to the financial statements.
Safeguards
Safeguards shall be applied, when necessary, to eliminate the threat or reduce it to an acceptable level.
Examples of such safeguards include:
• Using professionals who are not members of the audit team to perform the service;

600 | P a g e
• If the service is performed by a member of the audit team, using a partner or senior staff member with
appropriate expertise who is not a member of the audit team to review the tax calculations; or
• Obtaining advice on the service from an external tax professional.

601 | P a g e
Tax Rates Used for Tax Year 2024

Tax rates for salaried individual (Salary Exceeds 75% of Taxable Income)
Sr.no Taxable Income Rate of Tax
1 Where the taxable income does not exceed Rs. 600,000 0%
2 Where taxable income exceeds Rs. 600,000 but does not 2.5% of the amount exceeding Rs.
exceed Rs. 1,200,000 600,000
3 Where taxable income exceeds Rs. 1,200,000 but does not Rs. 15,000 plus 12.5% of the amount
exceed Rs. 2,400,000 exceeding Rs. 1,200,000
4 Where taxable income exceeds Rs. 2,400,000 but does not Rs. 165,000 plus 22.5% of the amount
exceed Rs. 3,600,000 exceeding Rs. 2,400,000
5 Where taxable income exceeds Rs. 3,600,000 but does not Rs. 435,000 plus 27.5% of the amount
exceed Rs. 6,000,000 exceeding Rs. 3,600,000
6 Where taxable income exceeds Rs. 6,000,000 Rs. 1,095,000 plus 35% of the amount
exceeding Rs. 6,000,000

Tax rates for non-salaried individuals and Association of Persons


Sr.no` Taxable Income Rate
1 Where the taxable income does not exceed Rs.600,000 0%
2 Where the taxable income exceeds Rs.600,000 but does 7.5% of the amount exceeding
not exceed Rs. 800,000 Rs.600,000
3 Where taxable income exceeds Rs. 800,000 but does not Rs. 15,000 plus 15% of the amount
exceed Rs. 1,200,000 exceeding Rs. 800,000
4 Where taxable income exceeds Rs. 1,200,000 but does not Rs. 75,000 plus 20% of the amount
exceed Rs. 2,400,000 exceeding Rs, 1,200,000
5 Where taxable income exceeds Rs. 2,400,000 but does not Rs. 315,000 plus 25% of the amount
exceed Rs. 3,000,000 exceeding Rs. 2,400,000
6 Where taxable income exceeds Rs. 3,000,000 but does not Rs. 465,000 plus 30% of the amount
exceed Rs. 4,000,000 exceeding Rs. 3,000,000
7 Where taxable income exceeds Rs. 4,000,000 Rs. 765,000 plus 35% of the amount
exceeding Rs. 4,000,000

Capital gain on immovable property

Constructed
Sr.no Holding Period Open Plot Flats
Property
1. Where holding period does not exceed one year 15% 15% 15%
2. Exceeds one year but does not exceed 2 years 12.5% 10% 7.5%
3. Exceeds 2 years but does not exceed 3 years 10% 7.5% 0%
4. Exceeds 3 years but does not exceed 4 years 7.5% 5% -
5. Exceeds 4 years but does not exceed 5 years 5% 0% -
6. Exceeds 5 years but does not exceed 6 years 2.5% - -
7. Exceeds 6 years 0% -

602 | P a g e
Tax year 2024 - Where securities acquired on or after 01 July 2022

Holding Period Rate of Tax


Less than one year 15%
More than one year but less than two years 12.5%
More than two years but less than three years 10%
More than three years but less than four years 7.5%
More than four years but less than five years 5%
More than five year but less than six years 2.5%
More than 6 years 0%

Tax year 2024 - Where securities acquired between 01 July 2013 to 30 June 2022

Holding Period Rate of Tax


Irrespective of the holding period 12.5%
Tax year 2024 - Where securities acquired before 01 July 2013

Holding Period Rate of Tax


Irrespective of the holding period 0%

Depreciation Table
Sr.no Type of Assets Rate
I. Building (all types). 10%
Furniture (including fittings) and machinery and plant (not otherwise
II. specified), Motor vehicles (all types), ships, technical or professional 15%
books.
Computer hardware including printer, monitor and allied items,
III. Machinery & Equipment used in manufacture of IT Products, aircrafts 30%
and aero engines.
In case of mineral oil concerns the income of which is liable to be
IV. computed in accordance with the rules in Part-I of the Fifth Schedule. 20%
q Offshore platform and production Installations.
A ramp built to provide access to persons with disabilities not
V. exceeding 100%
Rs. 250,000 each.

603 | P a g e

You might also like